Download as pdf or txt
Download as pdf or txt
You are on page 1of 675

Visit https://telegram.me/ntse_tg for more books.

Corporate
DISHA PUBLICATION
Office
45, 2nd Floor, Maharishi Dayanand Marg,
Corner Market, Malviya Nagar, New Delhi - 110017
Tel : 49842349 / 49842350

© Copyright
No part of this publication may be reproduced in
any form without prior permission of the publisher.
The author and the publisher do not take any legal

Disha
responsibility for any errors or misrepresentations
that might have crept in. We have tried and made
our best efforts to provide accurate up-to-date
information in this book.
All Right Reserved

Typeset by Disha DTP Team

www.dishapublication.com www.mylearninggraph.com
Books & Etests
ebooks for
School & for
Competitive Competitive
Exams Exams

Write to us at feedback_disha@aiets.co.in
Contents
NTSE Stage 2 Past Papers (2020-2015)
• 2020 Stage 2 mat Solved Paper (with solutions)
(Held on 14 feb. 2021) 2020-1-18

• 2020 Stage 2 SAt Solved Paper (with solutions)


(Held on 14 feb. 2021) 2020-19-40

• 2019 Stage 2 mat Solved Paper (with solutions) 2019-1-18

• 2019 Stage 2 SAt Solved Paper (with solutions) 2019-19-42

• 2018 Stage 2 mat Solved Paper (with solutions) 2018-1-16

• 2018 Stage 2 SAt Solved Paper (with solutions) 2018-17-32

• 2017 Stage 2 mat Solved Paper (with solutions) 2017-1-8

• 2017 Stage 2 SAt Solved Paper (with solutions) 2017-9-30

• 2016 Stage 2 mat Solved Paper (with solutions) 2016-1-10

• 2016 Stage 2 SAt Solved Paper (with solutions) 2016-11-29

• 2015 Stage 2 mat Solved Paper (with solutions) 2015-1-10

• 2015 Stage 2 SAt Solved Paper (with solutions) 2015-11-30


NTSE Stage 1 (2020-15) Solved Questions

• NTSE Solved Questions MAT (with solutions) M-1-52

• NTSE Solved Questions SAT (with solutions) S-1-104

Mock Tests

• mat Mock Test-1 (Stage 1) MT-1-12 • Sat Mock Test-1 (Stage 1) MT-13-24

• mat Mock Test-2 (Stage 1) MT-25-36 • Sat Mock Test-2 (Stage 1) MT-37-48

• mat Mock Test-3 (Stage 2) MT-49-62 • Sat Mock Test-3 (Stage 2) MT-63-76

• mat Mock Test-4 (Stage 2) MT-77-90 • Sat Mock Test-4 (Stage 2) MT-91-104

• mat Mock Test-5 (Stage 2) MT-105-118 • Sat Mock Test-5 (Stage 2) MT-119-134

mock test [Hints & Solutions] 1-54


2019-20 MAT SOLVED PAPER 2020-1

Solved Paper
2020
Held On
14 feb. 2021

Stage 2

Time : 120 Minutes Max. Marks : 100

Instructions for Candidates


Read the following instructions carefully before you open the questions booklet :

1. Answers are to be given on a separate answer-sheet.

2. Write your eight-digit Roll Number very clearly on the test-booklet and answer-sheet as given in your letter/
admission card.

3. Write down the Booklet Number in the appropriate box on the answer sheet.

4. There are 100 questions in this test. All are compulsory. The questions number 1 to 40 belong to Social Science, 41
to 60 belong to Mathematics and 61 to 100 are on Science.

5. Please follow the instructions for marking the answers given on the answer sheet.

6. For questions 1 –100, put a cross mark ( ) on the number of the correct alternative on the answer-sheet against the
corresponding question number.

7. If you do not know the answer to any question, do not spend much time on it and pass on to the next one. Time
permitting, you can come back to the questions, which you have left in the first instance and try them again.

8. Since the time allotted for this question paper is very limited you should make the best use of it by not spending too
much time on any one question.

9. Rough work can be done anywhere in the booklet but not on the answer sheet/loose paper.

10. Every correct answer will be awarded one mark.

11. Please return the Test-booklet and answer-sheet to the invigilator after the test.

Visit https://telegram.me/ntse_tg for more books.


2020-2 NTSE Solved Paper Stage-2

1. If MENTAL = 390, ABILITY = 546, REASONING = 8. Which expression will replace the question mark?
918, then COMPETENCY = ?
(1) 782 (2) 842 (3) 1190 (4) 1340 B8 ?
2. Which number replaces the question mark in the given H J
figure?

2 1 8 3 6 ? F I
4 21 5 98 3 94 A6 C9
(1) 4 (2) 5 (3) 7 (4) 9
(1) E 7 (2) D 10 (3) D 7 (4) E 10
DIRECTIONS (Qs. 3-4) : The following questions are based 9. The digits/numbers from 1 to 12 of the clock dial are
on the in.formation given below: replaced by the letters of the English alphabet. The
• A cuboid shaped wooden block has 4 inches length, 3 replacement starts with letter ‘C’ but vowels and immediate
inches breadth and 6 inches height. next consonants of vowel are not included in the
• Two faces measuring 6 inches x 4 inches are coloured in replacement. The classes in the school start at N: T and last
black. till a time when the minute hand is at K and the hour hand
• Two faces measuring 4 inches x 3 inches are coloured in between S and T, very slightly ahead of S. Five periods of
red. equal duration are held during this interval. The break of 7
• Two faces measuring 6 inches x 3 inches are coloured in minutes is given to students after 1st period and duration
yellow. of break increases by 2 minutes after each period. The exact
• Now the block is divided into small cubes of side 1 inch duration of a period in minutes is:
each. (1) 32 (2) 33 (3) 34 (4) 35
3. How many small cubes will have no faces coloured? 10. If TEACHER is coded as KBADFBM, MATURE is coded
(1) 8 (2) 12 (3) 36 (4) 18 as ALONEG, then the code of BOARD will be ___.
4. How many small cubes will have at least two faces (1) AMDJC (2) MADKC
coloured, one with red and another with yellow? (3) MACKD (4) AMCJD
(1) 4 (2) 8 (3) 12 (4) 24 11. In a performing Arts competition, there are six participants
5. In a certain code 34 means ‘project work’. What is the M, P, B, K, L and V. It is given that M and P are good at
meaning of 4 and 3 in that code? dancing and acting. B and K are good at singing and playing
Based on the following statements, select the correct tabla. L and M are good at tabla but not good at singing. P
option: and V are good at playing Guitar and also good at singing.
Statement I: ‘173’ means ‘completed project on time’ Who among them is/are good at dancing, singing, playing
Statement II: ‘640’ means ‘received award for the hard work’ Guitar and acting?
(1) The data in statement I alone are sufficient to answer (1) M and P (2) Only P
the question (3) Only V (4) Only B
(2) The data in statement II alone are sufficient to answer 12. What is the total number of squares in the given figure?
the question
(3) The data in both the statements together are sufficient
to answer the question
(4) The data even in both the statements together are
not sufficient to answer the question
6. Fact 1: All tortoises like to jump.
Fact 2: Some tortoises like to fly.
Fact 3: Some tortoises look like their followers.
Is the first three statements are facts, which of the
following statements must also be a fact?
I. All tortoises who like to fly look like their followers. (1) 62 (2) 72 (3) 82 (4) 92
II. Tortoises who like to fly also like to jump. 13. In a certain coding system if
III. Tortoises who like to jump do no look like their 25 + 10 x 4 = 19
followers. 10 ÷ 3 – 3 = 10
(1) I only (2) II only Then the value of
(3) I and III only (4) II and III only 16 x 5 + 40 – 10 ÷ 2 =
7. At what time between 3PM and 4PM the angle between (1) 10 (2) 11 (3) 12 (4) 13
the minute and hour hands be nine degrees, the minute 14. In the given sequence, which symbol comes five symbols
hand being ahead of the hour hand? before the symbol which comes nine symbols after the
(1) 3 h 15 m 45 s (2) 3 h 16 m second appearance of the symbol which occur four times
(3) 3 h 16 m 30 s (4) 3 h 18 m in the sequence?
(1) A (2) S (3) @ (4) 8
2019-20 MAT SOLVED PAPER 2020-3

15. Find the values of W, X, Y and Z in the given figure. 19. How many actors are magicians?
(1) g (2) g + h
2 Z 3 (3) f + g + h (4) f + g ¡V h
4 4 X 20. How many actors are also philosophers but not historians
8 5 2 or musicians?
(1) I + k (2) g + h + k
7 9 8
(3) m + f + I + k (4) m + f + s
3 9 5 W 1 7 6 4 8 21. Bhupen’s birthday is on the 13th of June and Mainu’s
4 3 9 birthday is on 23rd of April. If in a particular year Mainu’s
7 8 Y birthday was on Monday. What will be the day on Bhupen's
birthday in the same year?
6 2 3 (1) Tuesday (2) Wednesday
5 6 5 (3) Thursday (4) Friday
(1) 2, 9, 6, 7 (2) 3, 5, 1, 5 DIRECTIONS (Qs. 22-24) : Study the following information
(3) 4, 1, 8, 7 (4) 8, 1, 2, 5 and answer the questions below:
DIRECTIONS (Qs. 16-17) : In the following questions, the Dr. Ashutosh is available at a clinic from 12 Noon to 4 Pm on
symbols @, $, #, © and % are used with the following meanings Tuesday, Thursday and Sunday.
as illustrated below: Dr. Dhanwantri is available at the same clinic from 10 AM to 2
P$Q means ‘P’ is not smaller than ‘Q’. PM on Monday, Thursday, Friday and Sunday.
P©Q means ‘P’ is neither greater nor equal to Q. Dr. Shehnaz is available at the same clinic between 9 AM to
P#Q means ‘P’ is neither smaller nor equal to Q. 12:30 PM on Monday, Wednesday and
P%Q means ‘P’ is not greater than Q. Thursday, and 2 PM to 4 PM on Friday, Saturday and Sunday.
P@Q means ‘P’ is neither greater nor smaller than Q. 22. At what time duration of a week are all the doctors available
Assuming the given statements to be true, find which of the at the clinic?
conclusions given below is definitely true? 1
16. Statement: T%R, R$M, M@D, D©H (1) Sunday for 3 hrs (2) Thursday for hr
2
Conclusions: (3) Thursday for 1 hr (4) Sunday for 1 hr
I. D %R II. H#R 23. On how many days and hours, Dr. Ashutosh and Dr.
III. T©M IV.T%D Shehnaz are available at the same time?
(1) Only I (2) I and III
(3) I and IV (4) III and IV 1
(1) 1,2 (2) 1, 2
17. Statement: M@B, B#N, N$R, R©K 2
Conclusions:
1
I. K„¦R II. R©B (3) 2,2 (4) 2, 2
III. M$R IV. N©M 2
(1) I and III (2) I, II and III 24. For how many hours in a week, Dr. Dhanwantri and Dr.
(3) II, III and IV (4) I, III and IV Shehnaz are together available at the same time?

DIRECTIONS (Qs. 18-20) : Study the diagram and answer the 1


(1) 5 (2) 6
questions: 2
Magicians 1
a
(3) 5 (4) 2
Philosophers Musicians 2
z b 25. In a certain school, 62% of candidates qualified all the three
subjects, namely English, Mathematics and Social Science.
c d e The following diagram gives the number of candidates who
y
f g are not qualified in different subjects. What is the percentage
o of candidates who are not qualified in at least two subjects?
m l h j i p
k
rs n
Writers
Actors u vt q English Mathematics
w 30 10 75
5
x 12 8
Historians Social
50
18. Identify the region which represents philosophers who are Science
musicians and writers.
(1) h + i + j + k (2) h + i
(1) 2.63 (2) 6.00 (3) 7.00 (4) 18.42
(3) k + j (4) j
2020-4 NTSE Solved Paper Stage-2

26. Samungou’s mother is the only daughter of Juhi’s father. Note


How is Juhi’s husband related to Samungou? books Books
(1) Brother (2) Father
(3) Son (4) Uncle

Accommodation
Mess
27. A matrix of certain character is given in the following. These (3) charges

Travel &
School
characters follow a certain trend, row-wise or column wise. fees
Find out this pattern and choose the missing character.

17 21 12 8
33 29 38 42 Books

Accommodation
41 37 ? 50

Travel &
(4) School
(1) 12 (2) 42 (3) 46 (4) 50 fees
28. Fine the value of # in the figure given below:

13 17 # Mess
Note books
charges
4 11 13 30. In an examination students are graded with four grades
namely A, B, C and D and 72 students of the school secured
49 48 44
A grade. The ratio of students who secured A grade to
5 4 7 6 6 8 students with D grade is 2:5. Out of the total students 30%
students secured B grade and 40% secured C grade. What
(1) 10 (2) 15 is the total number of students in the school?
(3) 19 (4) 21 (1) 840 (2) 600 (3) 420 (4) 360
29. The yearly percentage expenditure of a school student on 31. Given below are three statements followed by four
various items is shown in table given below: alternatives. Select the alternative which follows logically,
from the given statements.
Items Expenditure in percentage I. Only boys can register for a competition.
II. Many of the competitors are toppers.
Books 30 III. All the name of toppers are market with green colour.
Note Books 10 (1) All toppers are competitors
School fee 20 (2) Some of the competitors are boys
Mess Charges 5 (3) Some of the competitors are marked with green colour
(4) Only the names of boys are market with green colour
Travel & 32. Find the missing number in the given sequence.
35
Accom mod ation 1, 2, 2, 4, ___, 4, 2
(1) 1 (2) 2 (3) 3 (4) 4
Which of the following figures represent the above data? 33. Which figure among the given options will appropriately
Note books replace the question mark?

Mess charges
School
fees
?
(1) Travel &
Books Accommodation

(1) (2)
Mess charges
Note books
Accommodation

School
fees (3) (4)
Travel &

(2)
Books 34. Given below are 9 pictures A, B, C, D, E, F, G, H and I. On
the basis of similar relationship, classify the pictures into
three groups. Which among the given options represents
the suitable group combination?
2019-20 MAT SOLVED PAPER 2020-5

39. Find the value of X and Y.

9 16 49
64 2 15 100 2 24 X Y 70
A B C D E
25 36 100

(1) 289, 3 (2) 289, 5 (3) 121, 7 (4) 121, 4


40. Which is the missing term of the following sequence?
002 B, 009 I, 028 J, ___, 126 I
F G H I (1) 048 L (2) 065 K (3) 172 G (4) 186 N
(1) (A, D, F) : (B, C, H) : (E, G, I) 41. How many triangles are there in the given figure.
(2) (A, D, H) : (B, C, F) : (E, G, I)
(3) (A, D, F) : (B, C, H) : (E, G, I)
(4) (B, C, H) : (D, E, F) : (A, G, I)
35. If – means ÷, + means x, ÷ means –, × means +, then which
of the following is correct?
A. 36 – 12 × 6 ÷ 3 + 4 = 60
B. 52 ÷ 4 + 5 × 15 – 3 = 37
C. 36 × 4 – 12 + 5 ÷ 3 = 420
D. 43 × 7 + 5 + 4 – 8 = 25 (1) 21 (2) 22 (3) 23 (4) 24
(1) A (2) B (3) C (4) D 42. In a class of 60 students, where girls and boys are in the
36. The following figure represents numbers of students in each ratio 2:3, the boy ‘Kartik’ is ranked 17th from the top. If
of the club in a school. there are 9 girls ahead of Kartik, what is the ratio of number
of girls and boys after the rank position of Kartik?
Mathematics Science (1) 9:27 (2) 15:28 (3) 16:27 (4) 16:28
club 72 14 48 club
43. Select the pair that has the same analogy as given pair 9876
11 8 : 12234567
36 12 24 (1) 34562 : 89776 (2) 1234 : 122345
10 15
(3) 654321 : 922346 (4) 9993 : 8886
27 36 44. A square piece of paper is folded as shown, punched and
Eco 16 Music
club club unfolded. Which of the alternative figures resembles the
unfolded paper?
Find the number of students representing at least two clubs
of the school.
(1) 90 (2) 134 (3) 146 (4) 183
37. The ratio of boys and girls in a school for the last five
academic years are given in the following graph. If the
number of girls in 2016 – 17, what is the ratio of boys in
2017 -18 to boys in 2016 – 17?

3 (1) (2)
2.90
Ratio of boys to girls

2.5
2.00
2
1.90
1.40
1.5
(3) (4)
1
1.00
0.5
45. If
2013-14 2014-15 2015-16 2016-17 2017-18
(1) 1:1 (2) 3:2 (3) 14:5 (4) 7:3 = 5
38. A manufacturer of tennis balls is claimed that their balls
are the best as it (a ball) would rise constantly 10% of the = 7
height from which it was dropped. If the ball is dropped
from 27 feet, how much approximately in feet the ball = 15
travels (in feet) before coming to rest?
(1) 32.4 (2) 33 (3) 34.6 (4) 36 = 24
2020-6 NTSE Solved Paper Stage-2

Then, 23 is given by

(1)
a b c d e f
(2)

(3) m n p
(1) a, d, m; b, e, n; c, f, p (2) a, b, m; c, d, n; e, f, p
(4) (3) a, d, f; b, e, p; c, m, n (4) a, c, n; b, e, m; d, f, p
46. Complete the following figure series by choosing the 50. An identify is given below through some symbols. In the
correct answer from the given alternatives. options these symbols are decoded.
Identify the correct option of which the signs satisfy the
given identify.
? 12 O 13 15 5 * 180 ^ 21
(1) ^ is –, * is =, is ÷, is +, O is x
(2) O is –, ^ is =, * is +, is ÷, O is x
(3) is –, O is =, ^ is x, * is +, is ÷
(1) (2) (4) is ÷, is +, O is –, ^ is +, * is x
51. Complete the following figural series by choosing the
correct answer from the given alternatives.

?
(3) (4)

47. A problem figure is given below. When it is rotated, it fits


(1) (2)
into one of the option figures. Identify the option figure.

(3) (4)

52. A pentagonal figure is given below. Identify which two of


the pieces among A, B, C, D and E given below will NOT
(1) (2) be required to complete the pentagonal figure.

(3) (4)

48. The problem figure given below is a figure matrix.


Complete the matrix with suitable option figure.

A B C D E
(1) Both A and C (2) Both B and D
(3) Both A and D (4) Both B and E
? 53. A problem figure is given below. When it is folded into a
cube , which one of the cubes will be formed from the
given alternatives?
(1) (2)

(3) (4)
49. Nine figures are given below. Identify the correct group of
classification.
2019-20 MAT SOLVED PAPER 2020-7

6 17 19 11 4
(1) (2) 7 a 14 24 10 5 9
1 6 8 18 34 b 9 7 3
5 2 13 21 15 2 5
(3) (4) 5 16 20 12 c

54. A problem figure is given of which the mirror image is X (1) 4, 16, 5 (2) 3, 16, 7
and the water image of X is Y. Identify the appearance of Y (3) 3, 15, 5 (4) 2, 18, 7
from the given alternative figures: 59. Directions: Take the given statements as true though in
reality they may not be so and decide which of the
conclusions logically follow the statements?
X Statements:
(1) All the cucumbers are carrots.
(2) Some carrots are brinjals.
(3) All the brinjals are tomatos.
Y Conclusions:
I. Some tomatos are carrots.
II. All the carrots are tomatos.
(1) (2) III. Some brinjals are cucumbers.
IV. All the brinjals are cucumbers.
(1) Both the conclusion II and IV follows.
(2) Both the conclusion II and III follows.
(3) Both the conclusion III and IV follows.
(3) (4)
(4) Both the conclusion I and III follows.
60. Aman, Ayaz and Ashwinder are members of a joint family.
55. A hexagonal figure is given below. Identify which among Among them Aman is the eldest of all. Aman is six years
the pieces A, B, C, D, E and F given below will NOT be elder than Ayaz. Ayaz is eight years elder then Ashwinder.
required to complete the hexagonal figure. The sum of the present ages of Aman and Ayaz is five times
the age of Ashwinder four years ago.
The present age of Aman is
(1) 20 years (2) 24 years
(3) 28 years (4) 32 years
61. Direction: Complete the given analogy by choosing the
correct group from the given alternatives
B F I D L: N R U PX : :AJ E K C : ?
(1) G M F I K (2) I G D K F
(3) M V Q W O (4) K I F M G
A B C D E F DIRECTIONS (Qs. 62) : In an extension of a city, the total
(1) B (2) D (3) E (4) F population is 3000. The distribution of the population of 3000
56. Which of the given rules the number 70 follows? people is as follows:
(1) n3 + 4n (2) 2n3 + 2n 1. Equal number of men and women are there in the age group
(3) n + 3n/2
3
(4) n2 + 5n of above 60 years.
57. What are the values of x and y in the given matrix? 2. Number of male and female below 60 years are 1250 and
1150 respectively.
C1 C2 C3 3. Number of boys and girls below 18 years are 240 and 220
R1 9 x 102 respectively.
R2 12 53 140 62. What is the difference between the female population of
above 60 years and the adult female population below 60
R3 16 63 y
years in the extension?
R4 13 50 118 (1) 850 (2) 710 (3) 630 (4) 420
63. A set of three numbers is given 484, 529, 961. Choose the
(1) 25, 136 (2) 30, 148 set which is similar to the given set from the given
(3) 35, 128 (4) 40, 156 alternatives.
58. The numbers are arranged in a certain pattern. The values (1) 841, 625, 196 (2) 729, 576, 324
of a, b and c respectively in the pattern are (3) 784, 676, 289 (4) 441, 361, 225
2020-8 NTSE Solved Paper Stage-2

64. What are the missing numbers in the third figure? One house will perform between Violet and Indigo house. Based
3 1 ? on the above information select the correct option.
67. Which house the first to perform?
383 7 143 3 503 9 (1) Violet (2) Indigo
(3) Green (4) Red
68. Which house performed on Thursday?
77 19 29 7 ? 25 (1) Red (2) Green
(1) 2, 53 (2) 4, 101 (3) Indigo (4) Yellow
(3) 5, 166 (4) 6, 223 69. What are the value of # and @ in the figure given below?
DIRECTIONS (Qs. 65) : The unfolded figures of the same cube
are shown with three different figures. The given three figures 32 75 1311
are folded in the shape of cube.

F D E D E B BC B EG C K# O
A D A A
E C B F 2 3 @

B F C C
(1) M, 15 (2) M, 60
(3) N, 30 (4) N, 50
65. In the alternatives, which one does NOT show the correct 70. Which one of the following does NOT turn out to be
position of faces? meaningful by substitution of the set of signs ( , –, =, )
A E sequentially in the given alternatives?
(1) 24 * 3 * 10 * 120 * 2
(1) D F (2) B C (2) 12 * 4 * 8 * 160 * 4
(3) 10 * 8 * 16 * 192 * 3
(4) 16 * 4 * 14 * 200 * 4
B D 71. Statements
(3) (4) E A A # B means ‘A is the daughter of B
F C
A $ B means ‘B is the brother of A’
A = B means ‘B is the sister of A’
66. Direction: The different faces of a cube are shown through A % B means ‘A is the son of B’
three folded cube shape. Identify which one of the figures A * B means ‘A is the father of B’
given in the alternatives represents the unfolded cube. A @ B means ‘A is the mother of B
Assuming that spouse pair is unique, which of the following
indicates ¡¦R¡¦ is the granddaughter of M?
(1) M * T # 4 @ Z @ R = P
(2) M * P # Y @ T @ Z = R
(3) M = T @ Y @ Z # Z $ R
(4) M @ T # Y = Z @ R * P
72. Which of the following pair is different?
(1) (2) (16, 18); (56, 63); (96, 108); (86, 99)
(1) 16, 18
(2) 96, 108
(3) 56, 63
(4) 86, 99
(3) (4) 73. If you have to make the right side ball arrangement look
like left side how many minimum number of circles you
would require to move?
DIRECTIONS (Qs. 67-68): Read the information and select
the answer carefully.
All the students in a school are divided into five houses: Violet,
Indigo, Green, Yellow and Red. All houses perform from Monday
to Friday in a week, such that only one house performs on one
day. Yellow or Red should not be either the first or last to perform.
Red house should be immediately followed by Green house. Left side Right side
Indigo house should perform immediately after Yellow house.
(1) 3 (2) 4 (3) 5 (4) 6
2019-20 MAT SOLVED PAPER 2020-9

74. Which figure comes in place of ‘?’ in the figure given


below? A 1 2 1 2 2 2 2
2 1 1 2 3 3 1 2
1 3 2 1 1 2 3 2
1 2 1 1 1 2 3 2
3 2 1 3 1 3 2 3 1
1 2 2 1 2 2 2 1
? 1 1 2 3 3 1 3 1
2 3 2 1 1 2 2 2
1 1 3 1 3 3 1 B
(1) (2)
(1) 15 (2) 19 (3) 27 (4) 37
80. On a staircase, Yaimma is further up than Aloka but is lower
(3) (4) than Srinivas. Ranjan is in the middle. Jee is between Yaima
and Ranja, Alok is between Ranjan and Danial. There is
DIRECTIONS (Qs. 75-76) none below Barisha. Whos is in the fifth position
(1) Aloka (2) Danial (3) Jeet (4) Yatima
I. , , , and are cousins
DIRECTIONS (Q. 81)
II. is twice as old as
III. is half the age of Yatin and Anandi are a married couple with two children, Krishi
and Kanimi. Kaniki is married to Samson who is the son of Nur
IV. is half the age of and Nirmaan. Mishi is the daughter of Samson, Aliza, whose is
V. is twice the age of Samson’s sister is married to Hatim and has two sons, Kuku and
75. Based on the above statements, find who is the second eldest Kiki. Nui is the grandmother of Kiki. Krishi is the maternal aunt
amongst the five cousins. of Nishi
81. What is the difference between number of females and
(1) (2) (3) (4)
males in the generation to which Samson belongs?
76. Who all are younger to . (1) 1 (2) 2 (3) 3 (4) 0
(1) , (2) , (3) , (4) , 82. Which figure completes the statement?
77. Which numbers will replace the question marks if the
numbers in the circle have linkages with:

3 4 4 5 ? If is to then is to ?
15 0 45 30 ?

7 7 8 ? 9
20 55 30 ? 40

(1) 5, 15; 8, 5 (2) 5, 0; 8, 20


(3) 6, 15; 9, 5 (4) 6, 20; 9, 35 (1) (2)
78. Find out the figure that does not match with the other three
figures.

(3) (4)

A B C D 83. Identify the box that can be formed from the sheet of paper
(1) A (2) B (3) C (4) D given below:
79. What are the numbers of blocks to be crossed for covering
the shortest path from ‘A’ to ‘B’? One cannot move
diagonally and on a block with the number ‘2’.
2020-10 NTSE Solved Paper Stage-2

92. Read the information and then select the correct option:
Fact 1 : Islands are surrounded by water.
Fact 2 : Mani is an island.
(I) (II) (III) (IV) Fact 3 : Mani was formed by a volcano.
(1) Only I (2) Only II and III If the above three statements are facts, which of the
(3) Only III (4) Only IV following statements must also be a fact?
DIRECTIONS (Qs. 84-86): Study the following information and I. Mani is surrounded by water.
answer the given questions. II. All islands are formed by volcanoes.
III. All volcanoes are on islands.
I. B and E are good at Fine Arts and Social Science. (1) I only
II. A and B are good at Social Sciences and Chemistry. (2) III only
III. A, D and C are good at Chemistry and Biology. (3) I and III only
IV. C and A are good at Chemistry and Physics. (4) I and II only
V. D and E are good at Biology and Fine Arts 93. The vowels of English are coded by their letter numbers
84. Who is good in Chemistry, Biology and Physics but not in appearing as two-digited and then reversed. Thereafter these
Social Sciences? numbers are represented through a Pie diagram. What will
(1) A (2) B (3) C (4) D be the central angle (nearest to degree) of the sector
85. Who is good in Social Sciences, Biology and Fine Arts? representing ‘U’?
(1) A (2) B (3) D (4) E (1) 75 (2) 78 (3) 37 (4) 20
86. Who is good in Chemistry, Fine Arts and Social Sciences? 94. For finding a number code of 3 digits, the following
(1) A (2) B (3) D (4) E information is given:
87. Observing the pattern given in the following sequence,
identify the missing number in the sequence. (A) 6 8 2 Only one number is correct and
3, 18, 219,________, 100005
(1) 1743 (2) 1746 properly placed.
(3) 4096 (4) 4100 (B) 6 1 4 One number is correct but wrongly
88. A solid cube of 4 cm side, painted on all its faces, is cut up
into small cubes of 1 cm side. What is the ratio of the cubes placed.
without paint to the cubes with paint on exactly two faces?
(C) 2 0 6 Two numbers are correct but wrongly
(1) 1: 2 (2) 1 : 3 (3) 2 : 3 (4) 3 :2
89. A man walks 20 m towards South, then after turning to his placed.
left, he walks 22 m. Then he turns 90° in an anti –
clockwise direction and walks 26 m. Then again he turns (D) 7 1 8 Nothing is correct.
to the left and walks upto 30 m. How far (in metres) is the
man from his initial position (E) 7 8 0 One number is correct but wrongly
(1) 16 (2) 14 (3) 10 (4) 9
placed.
90. What comes in the place of the blanks?
XXIV, XXII, XXV, XXIII, XXVI, ___________. , XXV (1) 0 2 4 (2) 0 4 2
(1) XXIV, XXVII
(2) XXVII, XXXVIII (3) 6 4 2 (4) 6 0 4
(3) XXV, XXVI
(4) XXIV, XXV 95. What are the number of triangles that can be formed by
91. In which of the ways given in the options would you places connecting the vertices of a regular octagon with the
the number 1 – 7 side by side, so that: condition that exactly one side of the triangle will be one
The sum of the numbers 2 and 7 and all the numbers of the sides of the octagon.
between them total 28. (1) 18 (2) 32
The sum of the numbers 3 and 6 and all the numbers (3) 48 (4) 56
between them total 14. 96. Three students Shaurya, Ruhani and Seerat are standing in
The sum of the number 1 and 7 and all the numbers between a queue. There are six students between Shaurya and Ruhani
them total 23. and nine students between Ruhani and Seerat. If there be
The sum of the numbers 2 and 4 and all the numbers
exactly three students ahead of Seerat and 21 students
between them total 10.
behind Shaurya, what could be the minimum number of
(1) 2 4 3 5 6 7 1
(2) 1 2 3 4 5 6 7 students in the given queue?
(3) 2 3 1 4 6 5 7 (1) 22 (2) 28
(4) 3 2 1 5 6 7 4 (3) 29 (4) 30
2019-20 MAT SOLVED PAPER 2020-11

97. Observe the matrix carefully. DIRECTIONS (Qs. 98-100):


A is the incharge of setting the speakers for a debate at school. In
37 addition to the moderator, there will be speakers in favour and
against the theme. Besides, there will be a person to maintain
time and a reporter to record the points. The members involved
in this programme are B, C, D, E and F.
35 • The moderator must sit in the middle in seat number 3.
40 • The time-keeper cannot sit next to the reporter.
• B and F sit on either side of the moderator.
The values , , of in the given matrix are • B who is not the moderator sits between E and C
• The moderator does not sit next to D or E.
respectively.
• B, the time keeper sits on seat number 2.
(1) 16, 12, 19
98. Who is the moderator?
(2) 12. 15, 10
(1) B (2) D (3) C (4) F
(3) 9, 11, 17
99. What is the seat number of E?
(4) 15, 10, 12
(1) 1 (2) 2 (3) 3 (4) 4
100. Who are the speakers for the debate?
(1) E & F (2) B & D
(3) C & E (4) F & B
2020-12 NTSE Solved Paper Stage-2

Hints & Explanations


13 5 14 20 1 12
1. (3) As, M E N T A L Project Work 3 4
13 + 5 + 14 + 20 + 1 + 12 = 65 Completed Project On time 17 3
65 × 6 = 390
1 2 9 12 9 20 25
Received award for the hard Work 6 4 0
And, A B I L I T Y
1 + 2 + 9 + 12 + 9 + 20 + 25 = 78
78 × 7 = 546 6. (2) Jump
Similarly, Fly Tourist
3 15 13 16 5 20 5 14 3 25
COM P E T ENCY
3 + 15 + 13 + 16 + 5 + 20 + 5 + 14 + 3 + 25 = 119 Look Like their followers
119 × 10 = 1190
(Sum of position number of letters present in that word) Statement I :
× (number of letters present in that word) Statement II :
2. (3) The pattern is as follows : Statement III :
12 + 22 + 42 = 21 From the given statements it is clear that tortoise who like
32 + 82 + 52 = 98 to fly will also like to jump as all tortoise like to jump.
2 7. (4) 3 PM to 4 PM
Similarly, 7 62 32 94
11 11
Sol. (3-4) M 30H 9 M 30 3 9
2 2

11 2
Red Black M 9 90 M 99 18
2 11
3h. 18 min .
So, at 3 hrs. 18 min. the angle between minute hand
6 and hour hands be nine degree.
8. (4) Position number of inner letter is written on outer side
Yellow
Yellow and position number of outer letter is written in opposite
side (in square). Position number of J is 10 and 5 is the
3 position no. of E. So, “E10” will replace the question
Black
mark.
4
Red
12 T
9. (2) 11 1 S C
10 2 R D
Total number of cubes = 3 × 4 × 6 = 72
3. (1) There are 16 small cubes attached to the outer walls of 9 3 Q G
the cuboid. Therefor remaining inner small cubes will 8 4 N H
be the cubes having two sides green coloured. 7 5 M K
6 L
So the required number = 24 – 16 = 8
4. (3) 4 edges each common to red and yellow faces having
3 cubes each = 4 × 3 = 12. Start time of class N:T
8:00
5. (*) Since given statement is ‘project work’ which means
34, now from statement I and given statement it is clear End time of class S:K
11:25
that 3 is ‘project’ so 4 is ‘work’ and from statement II
Total time 3 hrs 25 min 205 min.
and given statement it is clear that 4 is ‘work’ so 3 is
‘project’ so we can determine answer from both the Number of period = 5
statements individually. Break time = 7 min, 9 min, 11 min, 13 min
2019-20 MAT SOLVED PAPER 2020-13

= 40 min (total) 15. (1) Sum of numbers vertically, horizontally and diagonally
is equal and by observation we see it is 45.
205 40
Duration of each period 33 min. W + 43 = 45 W = 2
5
X + 36 = 45 X = 9
20 5 1 3 8 5 18
39 + Y = 45 Y = 6
13 1 20 21 18 5
10. (1) T E A C H E R ; M A T U R E Z + 38 = 45 Z = 7
Sol. (16-17)
$
K B A D F B M A L O N E G
11 2 1 4 6 2 13 1 12 15 14 5 7 P # Q
Similarly, %
2 15 1 18 4 @
B O A R D
16. (1) T%R, R$M, M@D, D©H
T R M D H
I. D%R
A M D J C
1 13 4 10 3 D R, True
Either sum or difference of position number of II. H # R
corresponding letters = total number of letters present H>R False
in that word. III. T © M
T<M False
11. (2) M P B K L V IV. T % D
Dancing T D False
Acting 17. (3) M@B, B#N, N$R, R©K
Singing M=B>N R<K
Playing tabla I. K©R
Playing guitar K<R False
II. R © B
Only P. R<B True
12. (2) Squares formed from 2 figures = 24 III. M $ R
Squares formed from 4 figures = 16 True
M R
Squares formed from 8 figures = 13
IV. N©M
Squares formed from 16 figures = 9
N<M True
Squares formed from 18 figures = 4
18. (1) Area common to bigger rectangle, bigger triangle and
Squares formed from 32 figures = 1
smaller rectangle.
Squares formed from 36 figures = 4
h+i+j+k
Sqaure formed from 64 figures = 1
Total squares = 72. 19. (3) Area common to vertical rectangle and circle.
13. (4) Since, 25 – 10 + 4 = 19 f+g+h
and 10 × 3 ÷ 3 = 10 20. (4) Area common to circle and bigger rectangle but not
– both triangles.
m+f+s

21. (2) Mainu Bhupen


23rd April 13th June
16 × 5 + 40 – 10 ÷ 2 Monday ?
On exchanging the signs Number of days between 23rd April and 13th June
= 16 + 5 – 40 ÷ 10 × 2 = 7 (April) + 31 (May) + 13 (June) = 51 days
= 16 + 5 – 4 × 2 = 21 – 8 = 13. 51 days = 2 odd days
14. (3) Since ‘>’ occur 4 times. There is no symbol on the 13th June same year will be Monday + 2 = Wednesday
ninth number after ‘>’, so if we consider cyclic order
then correct answer is @.
2020-14 NTSE Solved Paper Stage-2

Sol. (22-24)
30
Timing M T W T F S S 29. (2) Central angle of book 360 108
100
9 - 10 S S S
10 - 12 DS S DS D D 10
12 - 12:30 DS A S ADS D AD Central angle of Note book 360 36
100
12:30 - 2 D A AD D AD
2-4 A A S S AS 20
Central angle of School fee 360 72
22. (2) All doctors are available on Thursday for ½ hrs. 100
23. (3) Dr Ashutosh and Dr. Shehnaz are available on Thursday
5
for ½ hrs and on Sunday for 2 hrs. Central angle of Mess charges 360 18
100
1
hrs.
So, for 2 days and 2 Central angle of Travel & Accommodation
2
24. (3) Dr. Dhanwantri and Dr. Shehnaz are available on 35
360 126
100
1 1
Monday for 2 hrs and on Thursday for 2 hrs. 30. (1) A = 72.
2 2
Ratio A to D is 2 : 5.
So, for 5 hrs.
25. (3) Total number of students who are not qualified in atleast A 72
1 subject are A 2 72
D 5 D 5 180
(30 + 10 + 75 + 5 + 12 + 8 + 50) = 190 2
38% of total students = 190
A + D = 252
( 62% of candidates qualified all three subjects)
30% secured ‘B’ grade.
190 40% secured ‘C’ grade
100 500
38 A + D secured = 100 – 30 – 40 = 30%.
Total students = 500 If x is the number of total student
Candidates not qualified in atleast 2 subjects 30% of x = 252
= 10 + 5 + 12 + 8 = 35 252 100
x 840
35 30
Required % of such candidates 100 7%.
500 31. (3) Since some of the competitors are toppers and all topper
are marked with green. So, some competitors (those
+ were toppers) are definitely marked with green.
26. (2) Father
32. (2) The pattern of the series is as :
– +
Juhi Husband 2, 3, 5, 7, 11, 13, 17, 19

+1 +2 +2 +4 +2 +4 +2
Samungou 33. (2) Let,
(where “–” denotes Female and “+” denotes Male) =P
From the family tree we can see that Juhi’s Husband is =Q
the father of Samungou. =R
27. (3) The pattern of Matrix in the row is as sum of number 1st 2P = Q, 2Q = R, 2P + Q = R
and 3rd is equal to the sum of number 2nd and 4th. So, 4P = R
34. (*) Option a and c are same.
+ 35. (2) By the options -
+
52 ÷ 4 + 5 × 15 – 3 = 37
17 21 12 8 On exchanging the signs -
33 29 38 42
41 37 46 50

28. (3) The pattern is as :


(13 – 4) × 5 + 4 = 49
(17 – 11) × 7 + 6 = 48 52 – 4 × 5 + 15 ÷ 3 = 37

( 19 – 13) × 6 + 8 = 44 52 20 5 37 37 37.
2019-20 MAT SOLVED PAPER 2020-15

36. (3) Atleast 2 clubs


= (14 + 36 + 16 + 24 + 11 + 10 + 15 + 8 + 12) = 146.
J
37. (3) Let B1 and G1 represents the number of boys and girls
represented in year 2016-2017.
And, B2 and G2 represent the number of boys and girls A K B
represented in year 2017-2018.
E F
B1 B G 1 I
1, 2 1.40, 1 41. (2)
G1 G2 G2 2
G H
B2 (1.4G2 ) (1.4G2 ) 2.8 28 14 D C
M
B1 G1 (G2 / 2) 1 10 5
Required ratio = 14 : 5.
L
27 27 IEF, IHF, IGH, GEI, EFM, EGH, EGF, GFH,
38. (2) 27 2 2 ....
10 100 ABI, BIC, DIC, DAI, ABC, ADC, ADB, DBC,
AJK, JKB, AJB, DML, LMC, DCL
27 27
2 27 2 2 .... 27 42. (2) 60 Total students
10 100
Girls = 24, boys = 36
1 1
2 27 1 .... 27
10 100 9 Girls & 7 Boys

Kartik
1
54 27 60 27 33.
1 43
1
10

39. (1) As, 9 3, 25 5 And, 16 4, 36 6 Kartik’s rank is 17 in which 9 are girls rest are boys
that is 7.
So, 3 × 5 = 15 4 × 6 = 24
Ratio of girls and boys after Kartik’s is
3 + 5 = 8, 82 = 64 4 + 6 = 10, (10)2 = 100
(5 – 3) = 2 6 – 4 = 2. 24 9 15
Similarly, 36 8 28
49 7, 100 10 Required ratio is 15 : 28.

7 × 10 = 70 43. (4) 9876 : 12234567


,
9993 : 8886
7 + 10 = 17, (17)2 = 289 Sum Sum = 30 Sum Sum = 30
30 30
10 – 7 = 3.
44. (4)
40. (2) The pattern of the sequence is as :
45. (4) From 1 and 2 L=2
13 + 1 = 2, 23 + 1 = 9, 33 + 1 = 28, 43 + 1 = 65 , From 2 and 3 =8
5 + 1 = 126
3
From above equation and 4 = 16
They are obtained by adding the digit So, clearly 23 = 16 + 5 + 2
2–B = + ( /) + L = L /
9–I 46. (1) The pattern of the series is as :
2+8–J is moving at 1, 3, 5 and 7 places, clockwise
6+5– K is moving at 1, 4, 9 and 16 places, anti-clockwise
1+2+6=I is moving at 1, 2, 3 and 4 places, anti-clockwise
47. (4) 48. (4) 49. (4)
50. (1) From the options :
12 × 13 + 15 ÷ 5 = 180 – 21
156 + 3 = 159 159 = 159.
2020-16 NTSE Solved Paper Stage-2

51. (4) x + y = 5z – 20,


x + y – 5z = –20
52. (4) C z + 14 + z + 8 – 5z = –20 [ x z 8 6, x z 14]
D 22 3z 20 3z 20 22
A
3z 42, z 14
So, Ashwinder = 14, Ayaz = 14 + 8 = 22 and
53. (4) In a cube opposite signs will not appear together.
Aman = 22 + 6 = 28.
(Opposite) 61. (3) As,

(Opposite) B F I D L
+12 +12 +12 +12 +12
(Opposite)
N R U P X
54. (2)
Similarly,
55. (3) A J E K C
F
D C +12 +12 +12 +12 +12

A M V Q W O
B
Boys - 240
3 3n
56. (3) From the option (c), n 62. (3) Population below 18 years
2 Girls - 220

3 3 4 12 Male - 1250
If we put n = 4, then 4 64 70. Population below 60 years
2 2
Female - 1150
57. (4) C1 = 9 + 16 = 12 + 13
So, adults population 18 to 60 years
C2 = x + 63 = 53 + 50 x 53 50 63 40.
Male - (1250 – 240) = 1010
C3 = 102 + y = 140 + 118 y = 140 + 118 – 102 = 156.
6 17 11 4 Female - (1150 – 220) = 930
58. (2) First row 19 (Middle No.)
2 Above 60 years population = 3000 – (1250 + 1150) =
600
7 3 14 10 5 9
Second row 24 (Middle No.) Male - 300
2
Above 60 years Ratio (1 : 1)
1 6 8 18 16 9 7 3 Female - 300
Third row 34 (Middle No.) So, the difference -
2
930 300 630
5 2 13 15 2 5 (adult female population) (female population
Fourth row 21 (Middle No.) above 60 years)
2
63. (3) The pattern of numbers is as :
5 16 12 7
Fifth row 20 (Middle No.) 484 4 8 4 16
2
529 5 2 9 16
59. (*) 961 9 6 1 16
Cucumbers Brinjals Tomatos Similarly,
Carrots 784 7 8 4 19
676 6 7 6 19
I. II. III. IV.
289 2 8 9 19
Only conclusion I follows.
60. (3) Let the present age of Aman = x, Ayaz = y and 64. (2) The pattern of the figures is as :-
Ashwinder = z
3 2 1 7
Given that, x = y + 6
7 3 2 19
y=z+8 First figure
x + y = (z – 4) × 5, 19 4 1 77
77 5 2 383
2019-20 MAT SOLVED PAPER 2020-17

4 2 1 9 76. (1) and is younger than .


9 3 2 25 77. (3) These are a clock’s time -
Similarly, in figure 3 25 4 1 101 +45 minutes 3:15, 4:00, 4:45, 5:30, (6:15)
+35 minutes 7:20, 7:55, 8:30, (9:05), 9:40
101 5 2 503
65. (4) 66. (4) 78. (3) All angles form in figure A, B and D are same except
Sol. (67-68) figure C.
Monday Tuesday Wednesday Th ursday Friday
Violet Yellow Indigo Red Green
79. (4) A 1 2 1 2 2 2 2
67. (1) 68. (1)
2 1 1 2 3 3 1 2
69. (1) The pattern of the figures is as :
1 3 2 1 1 2 3 2
Reverse
BC 2,3 32 (Upper side) 1 2 1 1 1 2 3 2
3 2 1 3 1 3 2 3 1
B 2 (Down side)
1 2 2 1 2 2 2 1
Reverse
EG 5, 7 75(Upper side) 1 1 2 3 3 1 3 1
2 3 2 1 1 2 2 2
C 3 (Down side)
1 1 3 1 3 3 1 B
KM 11,13 1311(Upper side)
So, there are 37 number of blocks.
O 15 (Down side)
80. (3) 7. Srinivas 6. Yaima
70. (1) Using options : 5. Jeet 4. Ranjan
Option (a) – 24 * 3 * 10 * 120 * 2 3. Aloka 2. Danial
After putting signs, 1. Barisha
24 × 3 – 10 = 120 ÷ 2
72 10 60 62 60 81. (1) Yatin Anandi Nui Nirman
71. (2) Given Codes :
Krishi Kaniki Samson Aliza Hatim
A is B is
# % * @ $ = Kuku
Nishi Mishi Kiki
Daughter Son Father Mother Brother Sister
of B of A Female Male

There are three females and 2 males in the Samson’s


+ –
M Y generation.
So, required difference = 3 – 2 = 1.
– –
where, (+) Male, 82. (2) Bottom box upside down on top with two remaining
P T
(–) Female boxes combined together.

Z R
83. (1)
From the family tree it is clear that ‘R’ is the grand-
daughter of M.
72. (4) In all other pairs the ratio of the two numbers is 8 : 9,
except (86, 99).
Sol. (84-86)
73. (1)
Fine Social
Chemistry Physics Biology
Arts Science
A
B
74. (4) In the column, Ist row – IInd row = IIIrd row.
C
Sol. (75-76) > > > > D
E
75. (2) Second eldest among the five cousins is
2020-18 NTSE Solved Paper Stage-2

84. (3) 85. (4) 86. (2)


So, 2 is at right place in A. 2
87. (4) The pattern of the sequence is as :
21 + 1 = 3
42 + 2 = 18 From options only (b) 0 4 2 is correct.
63 + 3 = 219
F E
84 + 4 = 4100 95. (2)
105 + 5 = 100005 G D
88. (2) Using formula -
H C
Cubes with no paint (n 2)3 (where, n = 4)
A B
(4 – 2)3 = 8
Cubes with pain on two faces = (n – 2) × 12 = 24 On every side 4 triangles can be formed.
Required ratio = 8 : 24 = 1 : 3. For example for side AB
AGB, AFB, AEB, ADB
89. (3) 30 m For 8 sides 8 4 32
8m
So, there are 32 numbers of triangles.
Final 6m
Point 96. (2)
Starting 26 m 3
Point Seerat
20 m
2
22 m
Shaurya
6
Distance 2 2
8 6 100 10 m Ruhani
14

90. (1) XXIV, XXII, XXV, XXIII, XXVI, XXIV, XXVII, XXV
Total students = 28.
Dual series 97. (4) + + = 37
Series 1 14, 15, 16, 17
Series 2 12, 13, 14, 15
2 + = 40 ...(i)
91. (3) 2 3 1 4 6 5 7 satisfies all the given conditions.
92. (1) Only I satisfies the fact as Mani is an island. So it must
be surrounded by water. ( +2 = 35) × 2
We can’t say anything about all island formation or all
volcanoes. One on islands. 2 +4 = 70 ...(ii)

93. (4) a e i o u From (i) and (ii)


01 05 09 15 21 3 = 30 = 10
10 50 90 51 12 reverse (sum = 213) Only in option (d)
= 10
360
Central angle of u 12 20.281 20 Sol. (98-100) 1. E — Speaker
213
2. B — Timekeeper
94. (2) From A and B 6X
3. C — Moderator
From D 7, 1, 8X 4. F
5. D
98. (3) 99. (1) 100. (1)
class 10 TARGET Solved Paper

NTSE
2020
Held On

SAT 14 feb. 2021

Stage 2
Scholastic Aptitude
Test National Talent Search Examination

Time : 120 Minutes Max. Marks : 100

Instructions for Candidates

Read the following instructions carefully before you answer the questions :

1. Answers are to be given on a separate answer-sheet.

2. Write your eight-digit Roll Number very clearly on the test-booklet and answer-sheet as given in your letter/ admission
card.

3. Write down the Booklet Number in the appropriate box on the answer sheet.

4. There are 100 questions in this test. All are compulsory. The questions number 1 to 40 belong to Social Science, 41 to
60 belong to Mathematics and 61 to 100 are on Science.

5. Please follow the instructions for marking the answers given on the answer sheet.

6. For questions 1 – 100, put a cross mark (X) on the number of the correct alternative on the answer-sheet against the
corresponding question number.

7. If you do not know the answer to any question, do not spend much time on it and pass on to the next one. Time
permitting, you can come back to the questions, which you have left in the first instance and try them again.

8. Since the time allotted for this question paper is very limited you should make the best use of it by not spending too
much time on any one question.

9. Rough work can be done anywhere in the booklet but not on the answer sheet/loose paper.

10. Every correct answer will be awarded one mark.

11. Please return the Test-booklet and answer-sheet to the invigilator after the test.
2020-20 NTSE Solved Paper Stage-2

1 A taxonomist during his voyage found a solitary marine


animal with spines on skin made of calcium carbonate. Phytoplankton
However, its Coelom was made of pouches pinched off
from endoderm. Assign the specimen to the most Zootoplankton
appropriate Phylum. (2)
(1) Chordata (2) Nematoda Herring
(3) Coelenterata (4) Echinodermata
2. An individual with genotype AaBbCcddEe is crossed with Sea
another individual with genotype AabbCcDdEe. Assuming Lion
Mendelian pattern of inheritance, predict the proportion
of aabbccddee among the progeny of this cross? Sea Lion
1 1
(1) (2) Herring
32 64

1 1 (3) Zootoplankton
(3) (4)
128 256
3. Which one of the four methods of propagation is likely to Phytoplankton
lead to maximum variation in DNA sequence through
generations?
Sea Lion
(1) Budding in Hydra
(2) Binary fission in Amoeba
Herring
(3) Reproduction in human beings
(4) Vegetative propagation of sugarcane (4) Zootoplankton
4. A case of bio-magnification was being studied.
A laboratory received equal quantities of three samples M, Phytoplankton
N and O. The levels of pesticides found in these samples
are as follows: M–1 mg, N– 0.2 mg, O – 3mg. 6. Which of the following traits would an evolutionary
The samples M, N and O respectively could be: biologist consider to understand the divergent evolution
(1) Grass, grasshoppers and adipose tissue of birds process?
(1) Hind limb of sheep, flipper of whale and wing of a bat
(2) Grasshoppers, grass and adipose tissue of birds
(2) Flipper of shark, slipper of penguin and flipper of
(3) Grass, adipose tissue of birds and grasshoppers
dolphin
(4) Adipose tissue of birds, grasshoppers and grass
(3) Bat wing, bird wing and wing of a butterfly
5. Illustration of a pyramid of number of an aquatic ecosystem
(4) Human arm, seal forelimb and wing of a bird
is given.
7. In adjacent agricultural lands of nearly equal dimensions,
Sea two farmers A and B had cultivated crops of their choice
Lion and observed standard practices. A pathogen attacked the
crops and destroyed it in the land belonging to farmer A,
Herring
as a result of which he suffered complete loss. Although
the pathogen attacked the adjacent land belonging to
Zootoplankton farmer B, he was able to earn some money by selling the
yield. The possible explanation for the above is:
Phytoplankton (1) Farmer A must have cultivated only one crop whereas
The pyramid of energy for the same ecosystem would be: farmer B must have cultivated two crops.
(2) Farmers A and B must have cultivated the same crop
Sea with a fence between the two agricultural lands.
Lion
(3) Farmer A over irrigated the crop due to which it
Phytoplankton attracted the pathogen.
(4) Farmer B removed weeds from the cultivated land.
(1) Herring 8. A biology teacher placed a hen’s egg in three different
solutions:
Zootoplankton Solution A: Pure water,
Solution B: saturated salt solution,
Solution C: Hydrochloric acid
2019-20 SAT SOLVED PAPER 2020-21

The sequence of treatments and the ensuring probable Function of the


effect on the egg are listed below: Organelle Marker
marker
i. A B C Remains unchanged Involved in ATP
ii. B C A Swells A Cytochrome oxidase
synthesis
iii. C A B Shrinks B Ribosomal RNA Part of ribosome
iv. B A C Loses salts Degrades different
Based one the above sequence to treatment which one of C Acid hydrolyase
molecules
the options will be correct?
Based on the information given in the table, identify the
(1) I and II (2) I and IV
organelles A, B and C.
(3) II and III (4) III and IV
(1) A – Plastids; B – Rough Endoplasmic Reticulum (RER);
9. Observe the flow chart below. C – Lysosomes
(2) A – Mitochondria; B – Rough Endoplasmic Reticulum
Tadpoles (RER); C – Lysosomes
(3) A – Mitochondria; B – Smooth Endoplasmic Reticulum
A B (SER); C – Golgi apparatus
(4) A – Plastids; B – Smooth Endoplasmic Reticulum
Tadpoles Tadpoles (SER); C – Golgi apparatus
12. Positions of endocrine glands are labeled A – E in the given
Water Water diagram. Match the symbols of glands in column 1 with
without with the type of hormone it secretes given in column 2.
iodine iodine
Column 1 Column 2
Tadpoles Frogs A I. Progesterone
B II. Insulin
Which of the following best explains the observed results? C III. Parathyroid hormone
(1) Iodine helps to produce thyroxine D IV. Melatonin
(2) Iodine inhibits thyroid gland activity E V. Follicle stimulating hormone
(3) Absence of iodine leads to starvation VI. Thyroxine
(4) Iodine promotes cell growth and division VII. Aldosterone
10. An experiment conducted in the laboratory is tabulated
below:
E
Test tube – A Test tube – B Test tube – C
Saliva Starch Starch
+ + +
Iodine Saliva Saliva D
+
incubation Incubation Enzyme
Inhibitor

Incubation
+
Iodine
What would be the colour observed in test tube A, B and C C
at the end of the experiment? B
(1) A – Yellow, B – No color, C – Blue black
(2) A – No color, B – Blue black, C – Yellow
(3) A –Blue black, B – Yellow, C – No color A
(4) A –No color, B – Yellow, C – Blue black
11. The presence of specific molecule (called markers) in an Choose the correct combination from the following:
organelle can be used to identify the presence of that (1) A – I, B – II, C –VII, D –III, E – V
organelle. A researcher has three test tubes with organelles (2) A – I, B – IV, C –II, D –III, E – VI
A, B and C, each of which shows the presence of one (3) A – V, B – II, C –IV, D –III, E – VII
marker as shown below: (4) A – V, B – IV, C –VII, D –III, E – II
2020-22 NTSE Solved Paper Stage-2

13. Virulent forms of the bacterium staphylococcus aureus is a Z1 Z2


human pathogen, some strains of which cause “flesh-eating (1) 95% 5%
disease”. Earlier the antibiotic Penicillin was used to control (2) 94% 6%
this pathogen. After some years Penicillin was ineffective. (3) 93% 7%
Hence, a powerful antibiotic-Methicillin was used in
(4) 92% 8%
treatments. Subsequently, Methicillin also became
20. Detergents are also called surface active agents
ineffective and the strains showed resistance to multiple
(surfactants). These have two distinct parts: one
antibiotics also called “multi-drug resistance”. Which one
hydrophilic spherical part and another hydrophobic long
of the following statements regarding development of multi-
tail made of carbons chain. Two experiment ‘A’ and ‘B’
drug resistance is correct?
were carried out. In experiment ‘A’, surfactant was added
(1) Antibiotics led to mutation in the DNA of bacterium
in a beaker containing water. In experiment ‘B’, surfactant
thus creating drug resistant strains.
was added in a beaker containing hexane.
(2) Antibiotics helped in the selection for bacterium with
Following are possible results in these experiments.
mutations in the DNA conferring drug resistance which
were already present in the population. I. In experiment ‘A’(see figure) ‘a’ micelle is formed,
(3) Even without the use of antibiotics the drug resistant where hydrophobic part is inside the micelle and
strains would have evolved at the rate as observed in hydrophilic part is outside the micelle.
the above situation. II. In experiment ‘B’(see figure ‘b’) micelle of reverse type
(4) Presence of antibiotics induces changes in the is formed where hydrophilic part is inside the micelle
metabolism of the bacterium leading to drug resistance. and hydrophobic part is outside the micelle.
14. 1.80 g of glucose(C6H12O6) was dissolved in 36 g of water. III. Micelle of reverse type is formed in experiment ‘A’.
The number of oxygen atoms in solution are IV. Micelles are large enough to scatter light in ‘A’.
(1) 6.68 1023 (2) 12.40 1022
(3) 6.68 10 22 (4) 12.40 1023
15. Consider the following statements:
I. F, Cl, N and O are electronegative elements
II. Electrons in the atoms give in statement I are in K and
L shell only. Figure a Figure b
III. Tendency of forming cations decreases in second Correct observations are
period of periodic table upto F Correct statement(s) (1) I, II & III only (b) I, II & IV only
is/are
(3) I, III & IV only (d) II, III & IV only
(1) I only (2) I and II only
21. Reaction of organic compound ‘A’ with ‘B’ in acidic
(3) I and III only (4) I, II and III condition gives compound ‘C’. Compound ‘B’ reacts with
16. Let T = Temperature; H = Humidity and v = Wind speed alkaline KMnO4 solution and gives compound ‘A’.
Which of the following are the best suited conditions for Compound ‘C’ gives compound ‘B’ as one of the product
drying up of clothes? when reacted with sodium hydroxide solution. Which of
(1) T = 40° C, H = 10%, v = 45 m/s the following statements is/are correct?
(2) T = 28° C, H = 20%, v = 35 m/s I. ‘A’ is CH3COOH
(3) T = 20° C, H = 30%, v = 25 m/s II. ‘B’ is CH3CH2OH
(4) T = 15° C, H = 40%, v = 15 m/s III. ‘C’ is CH3COOCH2CH3
17. 100 mL of solution containing 0.1 mole of NaOH per litre IV. ‘A’ is sweet smelling substance
was mixed with 100 mL solution containing 0.02 mole of (1) I and II only (2) I, II and III only
H2SO4 per litre. The amount of NaOH in the mixture in
(3) I, III and IV only (4) III and IV only
grams will be
22. Equal volumes of solutions containing 1 mole of an acid
(1) 0.12 (2) 0.24
and 1 mole of a base respectively are mixed. Which of these
(3) 2.4 (4) 0.36 mixture will give pH more than 7?
18 . On oxidation with alkaline KMnO4 followed by acidification (1) Sodium hydroxide + Acetic acid
of the reaction mixture, which one of the following alcohols
(2) Potassium hydroxide + Sulphuric acid
would produce an acid having three structural isomers?
(3) Ammonium hydroxide + Sulphuric acid
(1) Propanol (2) Butanol
(4) Sodium hydroxide + Hydrochloric acid
(3) Pentanol (4) Hexanol
23. A part of the modern periodic table is shown below in which
19. Atomic number of an element Z is 16. Element Z has two
elements have been represented by English letters of the
isotopes Z1 and Z2 with 16 and 18 neutrons, respectively.
alphabets
The average atomic mass of a sample of the element Z is
32.1 . Which one of the following percentage of Z1 and
Z2 in the sample is correct?
2019-20 SAT SOLVED PAPER 2020-23

Group (3) CaCO3, CaHCO3, CaSO4


Period 1 2 13 14 15 16 17 (4) CaHCO3, CaCO3, CaSO4
27. Figures given below show velocity – time curves for a
1 A moving object. Identify the one which may be realized in
2 B H J K practice.
3 C E
4 D F M v v
5 N
On the basis of the above periodic table, which one of the (1) (2)
following statements is incorrect? O
O t
(1) M will have -1 valency t
(2) C will form an ionic compound with K
(3) H will form a covalent compound with A
(4) B is small in size as compared to D and K v v
24. Consider the electrochemical cells (I and II) shown in the
following figures and select the correct statement about (3) (4)
these cells
O O
t t

28. Two balls A and B are released towards point W from point
Impure Impure
X and point Z respectively, on a perfectly smooth track as
Cu Cu shown in the figure. The balls move along the track without
Pure Pure losing contact. What will be the ratio of their speeds VA/
Cu Cu VB at point W?
CuSO4 CuSO4
Solution Solution x
Cell I Cell II

(1) Cell I produces purer copper than cell II


z
(2) In both cells, insoluble impurities settle down w
(3) Copper from cathode will deposit on anode in cell I Y
(4) Copper from anode will deposit on cathode in cell II 45m 15m 35m 27m
25. Read the following table:
Metal Reaction with 1 2 3
(1) 1 (2) (3) (4)
ZnSO4 solution FeSO4 solution CuSO4 solution 2 3 2
X No reaction No reaction No reaction 29. A marble P of mass ‘m’ lies at rest on the edge of a perfectly
Y No reaction No reaction Displacement horizontal surface of a table of height ‘h’, as shown in the
reaction figure. A second identical marble Q having same mass
Z Displacement Displacement Displacement moving at a speed ‘u’ strikes it perfectly elastically. The
speed acquired by marble P after the collision is
reaction reaction reaction
Based on the above table consider the following statements [In an elastic collision, momentum as well as kinetic energy
I. Reaction of Y with CuSO4 solution produces Cu metal. are conserved]
II. Z is the most reactive element and X is the least reactive
III. Y is more reactive than X and less reactive than Z
m
Q P
IV. Metal Y produces Zn on reaction with ZnSO4 solution.
Which of the following options gives the correct statements?
(1) I, II & III only (2) I, III & IV only
(3) II & III only (4) III & IV only h
26. If excess of CO2 is passed through the suspension of a
compound ‘X’ in water, a compound ‘Y’ is formed.
Substances ‘X’ and ‘Y’ dissolve in H2SO4 giving white
compound ‘Z’ which is insoluble in water. Identify the 1
compounds ‘X’, ‘Y’ and ‘Z’. (1) 0 (2) u
2
(1) CaSO4, CaCO3, gypsum
(3) u (4) 2u
(2) CaSO4, CaHCO3, lime
2020-24 NTSE Solved Paper Stage-2

30. A block floats with its fraction E inside water when


immersed in a beaker containing water and kept on the
earth. The beaker along with the block is shifted to the 1
surface of the moon. If M is the fraction of the block now v
immersed in water, which of the following relations is
correct? (1)
1 1
(1) M E (2) M 6 E u
6
1
(3) M E (4) M E
6
31. The weight of an object on a planet is 0.25 times of its
weight on earth. A pendulum clock that ticks once every
1
second on earth is taken to the planet. In that planet the
clock would tick once in every. v

(1) 1.0 s (2) 2.0 s (2)


(3) 3.0 s (4) 4.0 s 1
32. A ball is thrown vertically upwards at a speed u and returns u
back to the thrower. There are two instants at which the
ball has equal kinetic and potential energies. The difference 1
between these two instants is u
1 u u
(1) (2) 1
2 g g
(3) v

u u
(3) 2 (4) 2
g g
33. The potential energy stored in a spring when compressed 1
1 2 u
by a length ‘x’ is kx and the force required to compress
2
it is ‘kx’; ‘k’ is a constant of the spring known as spring 1
constant. The spring is placed on a floor upright and a (4) v
stone of mass 10 kg falls and hits the spring with a speed
10 m/s. The spring is compressed by 5 cm. Assuming
that there is no loss of energy, what is the value of ‘k’? 36. An object is placed at point A in front of a convex lens of
[Given: acceleration due to gravity is 10 m/s2 ] focal length f. Its real, inverted and magnified image is
formed behind the lens. When the objact is brought closer
(1) 2.0 × 10–2 N/m (2) 8.0 × 104 N/m
to the lens and placed at a point B, a virtual and erect
(3) 4.0 × 105 N/m (4) 2.0 × 106 N/m images, but with exactly the same magnification
34. A girl drops a ball from a height h = 20 m. It strikes the (in magnitude) as before is formed in front of the convex
ground elastically and returns to her hand. An echo of the lens. Let F be the focus of the lens in front of it. Which of
thud of the ball striking the ground is produced from a the following relations is correct?
nearby cliff. The echo is head at exactly the same moment (1) AF = FB (2) AB = f
when the ball returns to the girl’s hand. (Take g = 10 m/s2 (3) AF-BF = f (4) AB = 2f
and Vsound = 350 m/s). The distance of the cliff from the 37. Nethra, who is a back-bencher, discovers one day in the
girls is close to class that she is unable to discern the details on the
(1) 350 m (2) 350 2 m blackboard very well. When she visits an optician, he
prescribes glasses for her.
(3) 700 m (4) 3500 m
Which of the following statement(s) is/are false?
1 1 I. She suffers from myopia where the far point is nearer
35. Four graphs between and given for spherical mirrors.
u v than the blackboard.
Which one of them suitable represents a convex mirror, as II. A concave lens with a suitable power an help correct
per the new Cartesian sign convention?
her vision.
2019-20 SAT SOLVED PAPER 2020-25

III. Her eye is defective and is forming images in front of 43. If (ax + b) (x5 + 1) –(5x + 1) is divisible by x2 + 1, then the
the retina. value of 2a + 3b is
IV. A concave lens or a convex lens may be used to (1) 5 (2) 10 (3) 12 (4)13
correct her vision. 44. Suppose the graphs of 15x + 20y = – 2 and x – y = – 2
intersect at a point P. If the graph of 2x + 3y = k2 passes
(1) Only I (2) I, II and III
through P, then k is
(3) I, II and IV (4) Only IV (1) an integer (2) a positive integer
38. Consider three resistors of resistances R1, R2 and R3 such (3) a negative integer (4) not an integer but rational
that R1 < R2 < R3. Two of them are connected in parallel, 45. The sum of the squares of the third and the thirteenth terms
and then connected in series with the third. Which one of of an A.P. is 5 and the product of the fourth and twelfth
the following configurations yields the highest current terms is R. Then the product of the third and thirteenth
when connected to the same battery? terms of the AP is
(1) R1 and R2 in parallel, with R3 in series. 80 50 R 80 50 R
(2) R1 and R3 in parallel, with R2 in series. (1) (2)
41 82
(3) R2 and R3 in parallel, with R1 in series. 100 R 45 100 R 45
(3) (4)
(4) It will depend on the precise values of R1, R2 and R3. 82 41
39. Figures shows three electrical appliances connected to a 46. If and are the roots of the quadratic equation
220 V ac mains. What is the amperage (current rating) of 2x2 – 5x – 6 = 0 and Pn+1 = n – n, then the value of
the fuse that should be used in the circuit? P9 3P7
4P8 is

3 5 7 9
(1) (2) (3) (4)
8 8 8 8
47. A number is picked up at random from the numbers from
– 70 W 1 to 1000. The probability that it is of the form mn (where
1220 W
220 Y 220 Y 120 Y m > 1, n > 1) is
110 W
220 Y FAN
1 1 1 1
(1) (2) (3) (4)
20 25 30 39
(1) 1.0 A (2) 2.0 A 48. Let A(–5, 5), B (4, – 5) and C (4, 5) be the vertices of the
(3) 5.0 A (4) 10.0 A triangle ABC. If a circle passes through the vertices of
40. A positively charged plate and negatively charged plate ABC then the area (in sq. units) lying inside the circle but
are kept parallel to each other at a distance of 10 cm. outside the ABC is
An electron is release near the negative plate. Looking
181 181
from the negative plate towards the positive plate, the (1) 45 (2) 40
magnetic field produced by the moving electron will be 2 2
(1) clockwise 181 181
(3) 45 (4) 40
(2) anti-clockwise 4 4
(3) positive to negative plate 49. The coordinates of point A, B and C are (7, 4), (3, 1) and
(4) negative to positive plate (0, k) respectively. Then, the value of k, such that
AC + BC is minimum is
5 2 x 10 2
41. If x , then the value of is 5 19 5 9
2 3 5 2 x 10 2 5 (1) (2) (3) (4)
4 10 4 10
15 10 15 10 50. Two tangents PA and PB are drawn to a circle with centre O
(1) (2) from an external point P. The chord AB intersects the line
41 41 segment PO at Q. Then the square of the radius of the circle is
A
15 10 15 10
(3) (4)
43 47
42. On dividing a natural number x by 11, the remainder is 3, P O
Q
and on dividing x by 17, the remainder is 9. If the number x
lies between 300 and 400, then the remainder on dividing x
by 21 is
B
(1) 9 but not 11 (2) 11 but not 9 (1) OQ × QP (2) OQ × OP
(3) both 9 and 11 (4) neither 9 nor 1 (3) PQ × AB (4) PA × PB
2020-26 NTSE Solved Paper Stage-2

51. In the given figure, ABCD is a rectangle. Then the area of


the shaded region is 56. If u cos sin sin 2 sin 2 then | u | is less than
A or equal to
B
2 cm (1) 1 sin 2 (2) 1 cos2
R 3 cm
(3) 2 sin 2 (4) 2 cos2
8 cm 57. Two straight roads OA and OB intersect at O. A tower is
situated in the interior of the angle formed by them and
subtends an angle of 45° and 30° at the points A and B
respectively, where the roads are nearest to it. If OA = a
and OB = b, then the height of the tower is
a2 b2 b2 a2
D 12 cm C (1) (2)
2 2
(1) 1.2 sq. units (2) 1.4 sq. units
(3) 1.6 sq. units (4) 1.8 sq. units 3(b 2 a) 2 3(a 2 b) 2
(3) (4)
52. In the given figure, ABC is an isosceles triangle with 2 2
AB = AC. If AE = AF and BAE 40 , then the measure
of the angle FEC is 58. ABCD is a square of side 8 cm. P is the mid point of AD and
A is joined with vertex B, A perpendicular is drawn from the
vertex C on BP, which intersects BP at point E. The area of
the triangle BEC is

64 2 64
(1) cm (2) cm 2
5 5

F 32 2 32
(3) cm (4) cm 2
5 5
B E C
(1) 15° (2) 20° 59. The mean of three numbers is 10 more than the least number
(3) 40° (4) 60° and 15 less than the greatest number. If the median of three
53. In an equilateral ABC, side BC is produced to D and numbers is 5, then the sum of squares of these number is
DF AB such that DF is intersecting AC at E. If BC = 2 CD
(1) 625 (2) 650
and AF = 6 cm, then the length (in cm) of BF is
(3) 675 (4) 725
(1) 9 (2) 12
60. A and B are two metallic solid spheres such that the surface
(3) 15 (4) 18
area of B is 800% more than that of A. If the volume of A is
54. Water is flowing at the rate of 10 cm/minute through a pipe
X% less than that of B, then the value of ix closest to
of diameter 10 cm into an empty bucket, which is in the
form of frustum of a cone of height 30 cm with radii of its (1) 64.2 (2) 72.4
lower and upper ends as 10 cm and 20 cm respectively. (3) 95.5 (4) 96.3
Then, the time in which the level of water in the bucket will 61. Which of the following statements regarding the position
rise 15 cm. is and role of women during the French Revolution are correct?
I. Olympe de Gouges was a supporter of ‘ The
17 19 Declaration of Rights of Man and Citizen’.
(1) minutes (2) minutes
10 5 II. Women were disappointed that the Constitution of
1791 reduced them to passive citizens.
17 19 III. The Revolutionary Government made education
(3) minutes (4) minutes
5 10 compulsory for girls, marriage was made into a contract
55. The largest possible of ABC with AB = 5 cm and the sum and divorce was made legal.
of other two sides as 7 cm is
III. The Revolutionary Government finally recognized
5 women’s struggle for equal Political Rights and gave
(1) 5 6 cm 2 (2) 6 cm 2
2 them the right to vote.
(1) I and II (3) I and IV
5 (3) II and III (4) III and IV
(3) 3 cm 2 (4) 5 3 cm2
2
2019-20 SAT SOLVED PAPER 2020-27

62. Which of the following statements about socialism are 66. In the light of political developments that took place in
correct? the first quarter of twentieth century India, match the
following:
I. Robert Owen was the founder of New Harmony.
Place Event Year
II. Louis Blanc wanted Government supported co-
operatives. I. Amritsar A. Mill workers Strike i. 1916
II. Kheda B. Rowlatt Act ii. 1917
III. Marx argued that all property should be socially
III. Ahmedabad C. Peasant Strike iii. 1918
controlled
IV. Champaran D. Plantation Workers iv. 1919
IV. Robert Owen also believed that workers should Strike
construct a radically socialist society. E. Khilafat movement v. 1920
(1) I, II and III
(1) I-B-iv, II-C-ii, III-A-iii, IV-D-i
(2) I, II and IV (2) I-E-ii, II-B-iv, III-A-iii, IV-D-v
(3) I, III and IV (3) I-D-iv, II-C-ii, III-A-iii, IV-B-i
(4) d. II, III, IV (4) I-C-ii, II-B-iv, III-A-iii, IV-E-i
63. Which of the following statements about Maasais is/are 67. A history excursion of your school involved a visit to
correct? four countries. It first went to a city that had a treaty signed
I. Maasais are found in Tanzania and Kenya. in the early nineteenth century approving of ‘new
II. Samburu National Park is situated in Tanzania. conservatism’. It then travelled to the former kingdom of
Sardinia- Piedmont, followed by a visit to the country
III. The title Maasai is derived from the word ‘maa’,
one ruled by the ‘Hohenzollern dynasty’ and finally
which means ‘my land’.
reaching a city where many feel sowed the seeds of Nazism
IV. Maasai land was taken away by not only British and the Second World War were sown.
Kenya, but also German Tanganyika.
The correct sequence of the countries visited would be :
(1) I and II (2) I and IV
(1) Austria – Italy – Germany and France
(3) I, II and III (4) II, III and IV
(2) Germany – Italy – Austria and France
64. With regard to Polo, identify correct statements from the
(3) France – Germany – Italy and Austria
following.
(4) Austria – Italy – France and Germany
I. Polo was a game of European origin.
68. Nationalism in India which emerged as a force in the late
II. Sultan Qutubuddin Aibak died while playing polo.
nineteenth century meant strong devotion for
III. Polo was suitable for military and athletic young men.
(1) All countries of the world
a. Only II is true
(2) One’s own country, its history and culture
(1) Only II is true
(3) One’s own country and hatred towards others
(2) Both I and II are true, but III is false
(4) One’s own country without an appreciation of other
(3) Both I and III are true, but II is false
nations
(4) Both II and III are true, but I is false
69. Which of the following provides the most appropriate
65. With regard to women clothing after World War in Europe; sequence of events in the context of the French revolution?
identify the correct statements from the ones given below. (1) Increase in population – scarcity of grains-rising food
I. Wars eroded distinctions among women in Europe. prices – inability of the poor to buy bread – food riots
II. World War I shortened women’s clothes for practi (2) Scarcity of grains – increase in population – rising
cal necessity. food prices – inability of the poor to buy bread-food
III. New schools encouraged luxurious dressing and or riots
namentation. (3) Food riots – scarcity of grains – bad harvest – rising
(1) Only I is true food prices – inability of the poor to buy bread
(2) I and II are true but III is false (4) Increases in population – rising food prices – scarcity
(3) II and III are true but I is false of grains – food riots – inability of the poor to buy
(4) I and III are true but II is false bread
2020-28 NTSE Solved Paper Stage-2

70. Imagine yourself as a Kulak during Stalin’s Collectivisation (1) Statement-I is true, Statement-II is false
programme. Which of the following would you have (2) Statement-I is False, Statement-II is true
excluded from your objection(s) to Collectivisation? (3) Both Statement are True and Statement-II provides
I. Support to socialism. explanation to Statement-I
II. Independent cultivation (4) Both Statements are True and Statement-II does not
III. Work in collective farms provide explanation to Statement-I
75. Statement-I : Some form of social grouping has to be
IV. Transfer of land to collective farms
expressed in politics through gender division.
(1) I and II only (2) I and IV only
Statement-II : The Panchayati Raj Act was enacted to
(3) II and III only (4) III and IV only have a fair proportion of women in the local bodies.
71. Statement-I: During the Civil Disobedience Movement, (1) Statement-I is true, Statement-II is false
‘no rent’ campaign were carried out in most places. (2) Statement-I is False, Statement-II is true
Statement-II: The relationship between the poor peasants (3) Both Statement are True and Statement-II provides
and the Congress remained uncertain. explanation to Statement-I
(1) Statement-I is true, Statement-II is false (4) Both Statements are True and Statement-II does not
(2) Statement-I is False, Statement-II is true provide explanation to Statement-I
(3) Both Statement are True and Statement-II provides 76. In India, there are landlocked states as well as states with
long coastlines. Madhavan is planning to travel from
explanation to Statement-I
Srinagar to Kanyakumari. What is the minimum number
(4) Both Statements are True and Statement-II does not of landlocked and coastal states that he would have to
provide explanation to Statement-I traverse excluding the origin and destination UTs/States?
72. Statement-I: Mahatma Gandhi successfully organized the (1) 3, 2 (2) 3, 3
Satyagraha movement of 1916 and 1917 in favor of peasant. (3) 2, 2 (4) 2, 3
Statement-II: In Champaran, Gandhi Ji inspired the middle 77. Geological structure, physiography and precipitation regimes
class to struggle against the oppressive plantation system influence the evolution of drainage patterns. India with its
diversity in the above-mentioned attributes show cases a
and in Kheda district of Gujarat he supported their demand
variety of drainage patterns across regions. Match the
for relaxation in revenue collection affected by crop following drainage patterns found in the region given below
failure.
Drainage Pattern Region
(1) Statement-I is true, Statement-II is false
A Centripetal I. Narmada Basin
(2) Statement-I is False, Statement-II is true
B Radial II. Godavari Basin
(3) Both Statement are True and Statement-II provides C Trellis III. Loktak
explanation to Statement-I D Dendritic IV. Amarkantak
(4) Both Statements are True and Statement-II does not V. Aravalli
provide explanation to Statement-I (1) A-III, B-IV, C-V, D-II (2) b. A-IV, B-II, C-V, D-III
73. Statement-I: Khadar soils are poor in organic matter yet (3) A-III, B-IV, C-I, D-II (4) A-V, B-III, C-I, D-IV
these soils are very fertile. 78. Colonialism has been so far defined in terms of political,
Statement-II: Khadar soils are fertile because they fall in economic and social changes brought in the colonies. The
the flood plain zone of the river. aspect related to changes bringing in the biodiversity of
(1) Statement-I is true, Statement-II is false the colonies has received little attention. One such
(2) Statement-I is False, Statement-II is true practice was the introduction of new species of trees by
(3) Both Statement are True and Statement-II provides the colonizers in the colonies.
explanation to Statement-I Identify two trees that were introduced by colonizers in
(4) Both Statements are True and Statement-II does not India.
provide explanation to Statement-I I. Birch II. Teak
74. Statement-I: Indian citizens have the right to freedom. III. Chir Pine IV. Rhododendron
Statement-II: Indian citizens have the freedom to criticize (1) I and II (2) II and III
the core values of the Constitution. (3) II and IV (4) III and IV
2019-20 SAT SOLVED PAPER 2020-29

79. Indian farmers adopt diverse farming practices in different Himalayan and Trans-Himalayan ranges from their cross-
environmental conditions in order to maximize yield. section given below.
Identify the type of farming where the second crop is seeded A B C D
even before the harvesting of the previous standing crop. North
South
(1) Inter cropping (2) Mixed cropping
(3) Relay cropping (4) Multiple cropping (1) A = Karakoram; B=Zaskar: C=Ladakh; D=Himadri.
80. During the South-West monsoon season, India receives (2) A=Zaskar; B=Karakoram; C=Himadri; D=Ladakh.
the maximum amount of rainfall. However, it varies from (3) A=Karakoram; B=Ladakh; C=Zaskar; D=Himadri.
place to place. Choose the correct sequence of regions (4) A=Zaskar; B=Himadri; C=Ladakh; D=Karakoram
arranged in descending order of rainfall received from 85. Rivers are an important element of the physical landscape
South West Monsoon. of India. Variations in the environmental factors have
(1) Khasi Hills, Western Ghats, Bengal Delta resulted in the evolution of diverse drainage systems.
Which of the following statements is/are incorrect about
(2) Western Ghats, Khasi Hills, Bengal Delta
the drainage system of India?
(3) Bengal Delta, Khasi Hills, Western Ghats I. The Beas flows into Pakistan and joins Sutlej.
(4) Bengal Delta, Western Ghats, Khasi Hills II. Sutlej and Indus are examples of antecedent drainage.
81. Different types of soils are found in India having special III. River Luni drains into Sambar Lake which is an
characteristic features. One of these extends approximately example of inland drainage.
between 13 degree N to 25 degree N latitudes and 72 IV. The rivers flowing from the western slopes of
degree E to 82 degree E longitudes. Identify the soil type Western Ghats are swift and have a short course.
from the given options. (1) I and III (2) I, II and III
(1) Red soil (2) Black soil (3) II and III (4) II, III and IV
86. Understanding the spatio-temporal aspects of the population
(3) Laterite soil (4) Red and Yellow soil
is one of the main concerns of demographers. They have
82. Samanwita is taking her friends from Gandhinagar to her tried to measure the aspects of the same by selecting certain
Grand Parents’ home located in Kolkata. They board the key indicators. Match the indicators listed in column I with
flight from Gandhinagar and fly over Bhopal and Ranchi the explanations given in column II
to reach Kolkata. Which of the following statements are
true regarding their travel? Column I Column II
I. Travelled over saline soils, badlands, calcareous soil A. Density of population I Increase or decrease in
B. Population Growth II population.
Number of people in a
and alluvial soils.
given area.
II. Flew across Vindhyas, Bundelkhand, Chotanagpur C. Natural Growth III Man-Land ratio.
plateau and Rahr regions. D. Distribution of IV Birth Rate minus Death
III. Almost traversed along the Tropic of Cancer. population Rate
IV. Crossed rivers Chambal, Son and Damodar on the way. - - V In migration minus Out
Migration
(1) I and II (2) I and IV
(3) II and III (4) III and IV (1) A-II, B-V, C-I, D-III
83. The Western Ghats and Eastern Ghats are marked by many (2) A-III, B-IV, C-I, D-II
differences in terms of geographical aspects. Which of the (3) A-III, B-I, C-IV, D-II
following statements are true about the Eastern and (4) A-II, B-IV, C-III, D-V
Western Ghats?
87. Federalism is the most popular form of democratic
I. Western Ghats are more continuous than Eastern Ghats.
governance today. With reference to a federal political
II. Cardamom Hills, Javadi Hills, Shevaroy Hills and
Nallamalai Hills are part of Eastern Ghats. system, which of the following does NOT hold true?
III. Western Ghats have higher elevation than Eastern Ghats. (1) Spain, Pakistan and South Africa examples of a
IV. Doda Betta and Mahendragiri are the highest peaks federal system.
of Western and Eastern Ghats respectively. (2) Holding together federations always give equal
(1) I and II only (2) I, II and III only power to its constituent units.
(3) II and IV only (4) III and IV only
(3) The jurisdiction and authority of each tier of
84. The Himalayas are the young mountains that originated
from the sedimentary deposits of the Tethys Sea due to government is constitutionally mandated.
collision of continental plates. The process has remained (4) For a dispute relating to division of powers it is the
active over millions of years resulting into a series of High Courts and Supreme Court of India which
almost parallel ranges of different heights. Identify the interpret the Constitution.
2020-30 NTSE Solved Paper Stage-2

88. Regular elections are the backbone of democracy. Arrange Which of the above statement/s given above is/are correct?
the following election-related activities in the correct (1) I only (2) II only
sequence.
(3) Both I and II (4) Neither I nor II
A. Announcement of the election schedule. 92. The Constitution of India provides for a division of power
B. Election Campaign. between the Union and the States enumerated in three lists.
C. Making of vorter’s list. Based on the division of subjects in the lists, identify the
D. Polling of votes ones which are correctly matched.
I. Citizenship and extradition - Union List
E. Counting of votes.
II. Public health and sanitation - State list
F. Announcement of election results and issue of press
III. Forest and trade - Concurrent list
note.
IV. Computer software and digital privacy -State list
Which of the following indicates the correct sequence of Choose the correct option :
activities?
(1) I and II (2) I, II and III
(1) C, D, F, E, B, A (2) F, C, A, B, D, E (3) I, III and IV (4) I, II, III and IV
(3) A, B, C, E, D, F (4) C, A, B, D, E, F 93. In a social science class, the teacher asked the students to
89. the Constitution of India was drafted by a group of elected give their opinion about ensuring food security in India.
representatives called the Constituent Assembly. With Opinions given by Pahi, Saju, Zara and Veda are given
reference to the above, the members of the Constituent below. Whose opinion is NOT suitable for achieving food
Assembly from the Provinces were_____ _. security?
(1) Directly elected by the people of those Provinces. (1) Saju: Provide subsidy for export of food grains.
(2) Elected by the Provincial Legislative Assemblies. (2) Pahi: Increase food grain production in our country.
(3) Nominated by the India National Congress and the (3) Veda: Penalise the persons who waste food grains
Muslim League. in our country.
(4) Zara: Provide free food grains to all people below
(4) Nominated by the Government for their expertise on
the poverty line in our country.
constitutional matters.
94. Inexpensive Chinese locks, are flooding the Indian markets,
90. Consider the following statements about the Rule of Law : thus destroying the traditional lock industry of India.
I. Everybody shall be ruled by law as decided by the Which of the following methods can the Government of
judiciary. India take up to protect the Indian lock industry?
II. No man shall be punished except for a clear breach I. Revalue Indian Currency.
of law. II. Give subsidy on the import of Chinese lock.
III. Impose import tax on the import of Chinese lock.
III. Everybody except persons holding constitutional
IV. Place limit on the number of goods that can be
positions like the President and the Election
imported.
Commissioner shall be subjected to law.
(1) I and III (2) I and IV
IV. The term ‘Rule of Law’ was coined by F.A. Hayek.
(3) II and IV (4) III and IV
Which of the above statement/s is/are correct? 95. Of the 200 households in the village of Chandanwadi, 100
(1) I, II and III (2) I, II and IV households are debtors. They have borrowed money from
(3) II, III and IV (4) II only the following sources.
91. Consider the following statements about the process of Sources of Credit No. of Households
Amendment in the Constitution of India : Landlords 22
I. An amendment to the Constitution of India can be Bank of India 5
initiated by the introduction of a bill in the Lok Sabha Farmer's cooperative 15
only. bank
II. If such an amendment seeks to make changes in the Money lenders 18
federal character of the Constitution, the amendment Bank of Allahabad 10
Friends and relatives 15
needs to be ratified by the legislature of all the States
Maharashtra State 15
of India.
Cooperative Bank
2019-20 SAT SOLVED PAPER 2020-31

Based on the table given above which of the following Name of Location Daily No. of Size of No. of
statements are correct? Head of of wage work the working
I. Formal sources of credit are lower than informal sources. House hold residence days per house members
person hold
II. Informal sources of credit are lower than formal
Jeewan Mumbai 100 15 7 2
sources. Yashwant Palampur 80 25 3 3
III. One-fifth of debtors borrowed from friends and relatives. Village
IV. Money lenders and landlords continue to be major Sheelam Bangalore 100 25 4 3
sources of credit in the village. Sumer Dindori 100 15 6 2
Village
(1) I and III
(1) Yashwant, Sumer, Sheelam, Jeewan
(2) I and IV
(2) Sheelam, Yashwant, Jeewan, Sumer
(3) II and III
(3) Jeewan, Sumer, Sheelam, Yashwant
(4) III and IV
(4) Sumer, Sheelam, Yashwant, Jeewan
96. Dhanno gets up in the morning and milks her cow. She
sells milk to three houses. She then cooks food for her 99. The following data are given according to the Economic
family and prepares her children for school. At 10 a.m., Survey 2012-13.
she goes to the market with vegetables from her garden Life Expectancy at Infant Death
and sells them. By 11.30 a.m. she goes to Simranjeet’s birth (2006-10) (in Mortality rate Rate
house and cooks food for Simranjeet’s family. At 2.00 years) (2011) (Per
p.m. she goes to Harpreet’s house and washes clothes. By (Per 1000 Live
5.00 pm she goes home and washes her family’s clothes. births)
Odisha 63 57 8.5
Identify the economic activities performed by Dhanno.
Rajasthan 66.5 52 6.7
I. Getting her children ready for school
West Bengal 69 32 6.2
II. Cooking food for her family
III. Cooking for Simranjeet’s family Maharashtra 69..9 25 6.3
IV. Washing her family’s clothes
Which alternative shows the states with descending order
V. Washing Harpreet’s clothes
of health indicators ?
VI. Selling vegetables
(1) Maharashtra, West Bengal, Rajasthan, Odisha
VII. Selling milk
(2) Maharashtra, West Bengal, Odisha, Rajasthan
(1) I, III, IV and VI
(3) West Bengal, Maharashtra, Rajasthan, Odisha
(2) II, V, VI and VII
(4) Odisha, Maharashtra, West Bengal, Rajasthan
(3) I, II, III and V
100. Economic tools and their relevant objectives are as follows :
(4) III, V, VI and VII
97. Himmatveer has inherited land and Rs. 2,50,000 from his Tools : A. Issue Price
father. He decided to build a factory on the land. He spent B. Min imum Support Price
Rs. 2,00,000 for the building. To purchase the machines Objectives: I. To create more buffer stock
he took a loan of Rs. 75,000 from the bank and purchased II. To redu ce malnutrition in In dia
machines. After six months, he could start production. He III. To encourage farmers to produce
used the rest of the money that he has inherited to purchase more food grains
the raw materials required. His fixed capital and the IV. To distribute food grains in deficit
working respectively are : areas and among poor families
(1) Rs. 2,00,000 and Rs. 50,000 Which alternatives gives a correct combination of tools
(2) Rs. 2,75,000 and Rs. 50,000 and their objectives :
(3) Rs. 50,000 and Rs. 2,00,000 (1) A – I and II, B – III and IV
(4) Rs, 50,000 and Rs. 2,75,000 (2) A – II and IV, B – I and III
98. Based on the given table, arrange the following households (3) A – I and III, B – II and IV
in the order of the most poor to the least poor. (4) A – I and II, B – II and IV
2020-32 NTSE Solved Paper Stage-2

Hints & Explanations


1. (4) The specimen can be assigned to the phylum- 6. (4) Homologous organs are the organs having fundamental
Echinodermata. similarity in structural and origin but different function.
Chordata : The characteristic feature of the animal They represent a case of divergent evolution. Thus in
belonging to the phylum chordata is the presence of these animals, the same structure developed along
notochord, a dorsal hollow nerve cord and paired different directions due to adaptation because of
pharyngeal gills. different needs. Human arm, seal forelimb and wings of
Nematoda - They are very small and slender worms. a bird are homologous organs and they show divergent
Coelenterata - They are aquatic, free-swimming, radial evolution.
symmetrical animals. The name cnidaria a derived from 7. (1) Farmer B performed the practice of mixed cropping.
the cnidoblast which contain the stinging capsules or Mixed cropping reduces risk and gives some insurance
nomatocysts present on the tentacles and the body. against failure of one of the crops.
Echinodermata - These animals have an endoskeleton 8. (3) Egg shell is made almost entirely of CaCO3 crystals. It
of calcareous ossicles. Their coelom develop as is semipermeable membrane, which means that air and
pouches that are pinched off from the endoderm tissue. moisture can pass through its pores. The shell also has
These pouches undergo fusion to form mesoderm, thin outermost coating called the bloom or cuticle that
which then forms the coelom. help to reep out bacteria and dust.
2. (4) AaBbCcddEe × AabbCcDdEe - Genotypes When Egg shell kept in HCl it reacts with CaCO3 and
(1) Aa × Aa get dissolves.
AA Aa Aa aa 9. (1) Iodine helps to produce thyroxine hormone which plays
a major role in the metamorphosis of frog.
1 Metamorphosis is a process which converts larval stage
Aa to a tadpole and finally into a frog.
4
(2) Bb × bb
Bb Bb bb bb 10. (1) Test tube – A Test tube – B Test tube – C
Saliva Starch Starch
2 + + +
bb Iodine Saliva Saliva
4 +
(3) Cc × Cc Enzyme
incubation incubation
CC Cc Cc cc inhibitor

1 Incubation
cc +
4 Iodine
(4) dd × Dd
Yellow colour No colour Blue black colour
Dd dd Dd dd
* iodine is camel brown * When iodine reacts
2 in colour with starch gives blue
dd black colour
4
(5) Ee × Ee 11. (2) Based on the information given in the table in the
EE Ee Ee ee organelle present are as follows -
Test tube A - Mitochondria
1 Test tube B - Rough Endoplasmic Reticulum (RER)
ee Test tube C - Lysosome
4
The proportion of aabbccddee among the progeny of 12. (1) The correct match of endocrine gland w.r.t. hormones
are given below -
1 1 1 1 1 2 1
this cross is .
4 2 4 2 4 1024 256 Column 1 Column 2
3. (3) Sexual reproduction brings about maximum variation in A. Ovary I. Progesterone
DNA due to recombination and crossing over events.
B. Pancreas II. Insulin
4. (2) M = 1 mg. Grass hopper
N = 0.2 mg, Grass C. Adrenal gland VII. Aldosterone
O = 3 mg Adipose tissues of birds D. Parathyroid III. Parathyroid hormone
MNO
Grasshopper, Grass, adipose tissue of bird. E. Pituitary V. Follicle stimulating hormone
5. (4) Pyramid of energy is the only type of ecological pyramid, gland
13. (2) Multiple drug resistance (MDR) is anti microbial
which is always upright because the energy flow in a resistance shown by a species of micro organism to
food chain is always unidirectional.

Visit https://telegram.me/ntse_tg for more books.


2019-20 SAT SOLVED PAPER 2020-33

atleast one antimicrobial drug in three or more x 95%


antimicrobial categories. So, z1 = 95%, z2 = 100 – x = 5%.
Antibiotics helped in the selection for bacterium with 20. (2) Detergent ( )
mutations in the DNA conferring drug resistance which
were already present in the population. hydrophobic hydrophilic
14. (4) Number of moles of glucose Since in detergent, hydrophobic part binds with
Mass of glucose hydrophobic solvent (hexane) and hydrophilic part
1.80 2
10 mole binds with H2O. So in experiment ‘A’ micelles are formed
Molar Mass of glucose 180 and in ‘B’ reverse micelles are formed. Micelles can
No. of molecules of glucose also scatter light.
21. (2)
No. of moles × 6.022 × 1023 molecules H+
1 mol. O
= 10–2 × 6.022 × 103 = 6.022 × 1021.
Total number of oxygen atom present in glucose CH3COOH + C2H5OH CH3 – C – O – C2H5
= 6 × 6.022 × 1021 = 36.132 × 1021 (A) (B) (C)
36 KMnO 4
Number of moles of water 2 mole C2H5OH + [O] CH3COOH
18 (A)
(B)
Number of H2O molecule = 2 × 6.023 × 1023
No. of oxygen atom in water = 12.044 × 1023 O
Total number of oxygen atom present in solution NaOH
= 12.044 × 1023 + 36.132 × 1021 CH3 – C – O – C2H 5 + H2O
= (12.044 + 0.36132) × 1023 (C)
= 12.40 × 1023 CH3CH2OH + CH3COOH
15. (3) Statement I : F, Cl, N & O are electronegative element. (B)
Statement II : K L M Compound ‘C’ is sweet smelling substance as it is an
F9 2 7 ester.
Cl17 2 8 7
22. (1) The solution of NaOH (Strong base) and CH3COOH
O8 2 6
Statement III : In second period of periodic table the (weak acid) will exhibit pH more than 7.
tendency of forming cation decreases upto fluorine 23. (4) Atomic size generally decreases as we move from left
because across the period atomic size decreases. As to right with in a period in the periodic table, so size of
the size decrease, effective nuclear charge increases B is not smaller than K.
which results into increase in ionisation energy. 24. (4) When current is applied to electrochemical cell copper
16. (1) Rate of evaporation is proportional to temperature and on the anode are oxidised to Cu (II) ion and goes into
wind speed and inversely proportional to humidity. the solution and reduced to Cu atom at the cathode.
The cathode gains mass and anode loses mass.
17. (2) NaOH H 2SO 4 NaSO 4 H 2O 25. (1) As per given data, the order of reactivity of metal is
0.1 M 0.04 M Z > Zn > Fe > Y > Cu > X
0.01 mole 0.004 mol Z is the most reactive element as it is able to displace all
Mole of NaOH left = 0.01 – 0.004 = 0.006 other given elements from their salt solution. Metal Y is
Amount of NaOH = 0.006 × 40 = 0.24 g. not able to produce Zn on reaction with ZnSO4 solution
as it is less reactive than Zn.
18. (1) CH 3CH 2 CH 2 OH [O] CH 3 CH 2 COOH
Structural isomers of resulted acid is 26. (3) CaCO3 H 2 O CO2 Ca(HCO3 )2
(i) CH3 – CH2 – COOH(ii) CH3 – COO – CH3 (X) (Y)
(iii) HCOOC2H5
19. (1) Atomic number of element z = 16 CaCO3 H 2SO 4 CaSO4 H 2 O CO2
(X) (Z)
No. of neutrons in z1 = 16
No. of neutrons in z2 = 18
Ca(HCO3 )2 H 2SO 4 CaSO 4 2H 2O 2CO 2
So atomic mass of z1 = 16 + 16 = 32 (Y) (Z)
Atomic mass of z2 = 16 + 18 = 34
Let the percentage of z1 and z2 in sample be x and 27. (4) Two different values at one time of an object is not
100 – x respectively. possible. Hence option (4) v-t graph is correct and may
Average atomic mass be realized in practice.

Percentage of z1 Percentage of z2 v
Atomic mass of z1 Atomic mass of z2
100
x 32 (100 x ) 34 O
32.1 t
100
2020-34 NTSE Solved Paper Stage-2

28. (4) Drop in potential energy = Gain in kinetic energy where k = spring constant
1 2
For ball A : mg (h1 h2 ) mVA2 1 5 1 5
2 10 (10)2 10 10 k
2 100 2 100
1 2
g (45 27) VA VA 2 g 18 6 g m/s k
2 500 5 k 505 800
800
1 1 1
Similarly for ball B : mg (h1 h2 ) mVB2 k 4 105 N/m.
2 34. (1) Sound will be produced when ball hit ground.
1 2 Time taken by ball to return girl's hand.
g (35 27) VB VB 2g 8 4 g
2 From v u gt 0 u gt
VA 6 g 3 u 20
. t 2 s.
VB 4 g 2 g 10
29. (3) When two bodies of equal masses undergo elastic Distance of cliff from the girl,
collision in one dimension, their velocities gets v t 350 2
interchanged. Hence speed acquired by marble d 350 m.
2 2
P = u.
35. (2) By sign convension, for convex mirror
30. (3) Buoyant force = Weight of body (floating)
v = +ve, u = –ve, f = +ve
V l g Mb g From mirror formula
V l Mb 1 1 1 1 1 1
So, independent of acceleration due to gravity hence f v u v u f
fraction remains same i.e., M E y = mx + c
31. (2) As per question, wt. of an object on a planet 1 1
= 0.25 × mg (wt. of object on earth) Hence versus graph as shown in figure.
i.e., mg' = 0.25 × mg v u

g
g' 0.25 g
4
1
l v
From T 2
g

l
T' 2 2T 1
g/4
u
Hence clock would tick once in every 2.0 s.
36. (1) When object is at point A i.e., u = – OA (fig.) its real,
32. (3) K.E. = P.E. at two instants at the half mark of upward
inverted and magnified image is formed.
and downward motion of the ball.
And when object is at point B i.e., u = – BO its virtual
KE max and erect image is formed.
K.E. = P.E. =
2

1 2 1 mu 2 u2 u
mv1 v1 2F A F B O
2 2 2 2 2
For time t1 (when ball reaches from A to maximum
height).
f
Magnification, m
u t1 f u
v u gt1 0 gt1 t1 v1 For u = – AO m = –ve
2 A and for U = –BO m = +ve
A1
u f f
t1 m
2g and m
f AO f BO
Difference between two instants [ Magnification remains same]
uu f BO AO f
2t1 2 2
2g g 2f AO BO
33. (3) Here, P.E. + K.E. = Energy stored in spring
2f ( AF OF ) (OF BF )
1 2 1 2
mv mgx kx
2 2
2019-20 SAT SOLVED PAPER 2020-35

2f 2f AF BF 43. (4) Let f ( x) (ax b)( x5 1) (5 x 1)


AF BF
37. (4) Nethra is a back-bencher, and she is unable to discern ax 6 bx5 (a 5) x b 1
the details on the blackboard very well i.e., she is unable
( x 2 1)(ax 4 bx3 ax 2 bx a )
to see far-off objects but can see nearby objects hence
she is suffering from myopia. In this defect image formed (a b 5) x (b a 1)
before retina. To correct myopie eye, a concave lens [on dividing f(x) by x2 + 1]
with a suitable power is used. Since, f(x) is divisible by x2 + 1
38. (3) From the ohm's law, V = IR Taking remainder = 0
For current to be maximum, Req should be minimum. We get a + b = 5 and –a + b = 1
To get minimum resistance R2 and R3 should be in On solving we get a = 2, b = 3
parallel and R1 should be connected in series. So, 2a + 3b = 13.
39. (4) Current flowing through bulb,
44. (1) 15 x 20 y 2 ...(i)
PBulb 110
IBulb = A x y 2 ...(ii)
VBulb 220
6 4
Current flowing through fan, On solving, we get x ,y
5 5
70
Ifan A 6 4
220 Intersection point is P , .
Current flowing through heater, 5 5
1200 6 4
IHeater A Now, 2 3 k2
220 5 5
Ifuse = IBulb + Ifan + IHeater = 6.27 A
Fuse wire should be of 10 A. k2 0 k 0
+ – So, k is an integer.
40. (2) –
e 45. (3) Let the common difference of an A.P. be ‘d’.

The direction of flow of current is opposite to the (a13 a3 ) 2 2


a13 a32 2a13 a3
direction of motion of electron. So, when observed from (10d )2 5 2a13 a3 2
[ a13 a32 5]
–ve plate current is towards observer. Thus by right
hand thumb rule magnetic field is anti-clockwise. 5 2a13 a3
41. (3) We have, d2 ...(i)
100
5 2 5 2 Now, a4 a12 (a3 d ) (a13 d)
x
2 3 5 2 1 a3 ) d 2 [ a4 a12 R]
2 ( 5)2 1 2 5 2 R a3 a13 d (a13
2
R a3 a13 9d 2 [ a13 a3 10d 2 ]
5 2
x 5 2a13 a3
5 1 R a3 a13 9
2 2 100
2
100 R 45
a13 a3
5 2 ...(i) 82
x x 10 5 2
10 46. (2) Given that, are roots of 2 x 2 5 x 6 0
x 10 2 5 15 10 2 2 5
So, [From (i)] 2 5 6 0 3 ...(i)
x 10 2 5 3 5 2 43 2
42. (1) Since on dividing x by 17, the remainder is 9. 2
Similarly, 2 ...(ii)
x = 17q + 9 = 11q + 6q + 9 3
and on dividing x by 11, the remainder is 3. 2
x – 3 must be divisible by 11. P9 3P7 ( 8 8
) 3( 6 6
)
11q 6q 6 is divisible by 11 Now, 7 7
4 P8 4( )
6q 6 is divisible by 11
6 2 6 2
q 10, 21, 32,....... ( 3) ( 3)
7 7 [From (i) and (ii)]
For, q = 21 [ x lies between 300 and 400] 4( )
So, x = 17 × 21 + 9 = 366
Remainder where 366 is divided by 21 is 9. 5 7 7
[ ]
2 5
7 7 8
4[ ]
2020-36 NTSE Solved Paper Stage-2

47. (2) Total numbers = 1000 OQA OAP


Now numbers of form mn are either perfect square or
perfect cube or of form m5 or m7 OQ OA
OA2 OQ OP
Now, for n = 2, m can take 30 values i.e. 22, 32, ....312 OA OP
for n = 3, m can take 7 values (excluding 64 and 729 as
these are counted in perfect square) r 2 OQ OP
51. (4) In, PXQ and CXB
i.e. 23 , 33 , 53 , 63 , 73 , 83 , 103
PXQ CXB [V.O.A.]
for n = 5, m can take 2 values i.e. 25, 35
for n = 7, m can take 1 value i.e. 27 QPX BCX [Alt. angles]
Total favourable numbers = 40.
A P B
40 1 2 cm
Probability M
1000 25 Y N
Q X
48. (3) Given that A (–5, 5), B (4, –5), C (4, 5)
By distance formula, we get 8 cm

AB 181, BC 10, AC 9
Since 10 2 9 2 181 AB 2 BC 2 AC 2
So, ABC is right angled triangle and C 90 ,
D C
Hypotenuse AB 181 = diameter of circle.
PXQ ~ CXB
181
Radius of circum circle XM PQ 1
2 XN 4 XM ...(1)
XN BC 4
So, required area = Area circle – Ar ABC
XM XN 12 3 9
2
181 1 5 XM 9 [From (i)]
10 9
2 2 9
XM cm
5
181
45 1 1 9
4 area ( PXQ) PQ XM 2 1.8 cm 2
49. (2) In figure, B' (–3, 1) is image of B (3, 1) in y-axis 2 2 5
BD B ' D, CD CD (common) and 52. (2) Let ABC ACB x and CEF y
CDB CDB ' 90 [By exterior angle property]
y then, AFE AEF x y
A (7,4) [By exterior angle property]
C (0,k)
AEC ABE BAE
x 2 y 40 2 y y 20

A
B (–3,1) D B (3,1)
x' x
40°
y'
CB ' D CBD
CB CB '(CPCT ) x+y
For CB + CA is minimum F
x x+y
CB ' CA is also minimum which is possible when B y x
B', C and A are collinear. E C
Area of triangle ACB' is zero. 53. (4) Let CD = x cm then BC = 2x = AB = AC
7(k 1) 0(1 4) 3(4 k ) 0 Since, AF 6 AE 6cosec30 12
19 CE 2 x 12 and CDE CED CE CD
7 k 19 0 k
10 2 x 12 x x 12
50. (2) In OQA and OAP BF 2 x 6 18
PAO PQA 90
POA QOA (common)
2019-20 SAT SOLVED PAPER 2020-37

A 4u 2 4(u 2 )(u 2 sin 2 ) 0


6
F 12
30° 4u 2 (1 u 2 sin 2 ) 0
E
u2 1 sin 2
2x – 6 150°
30° 1 sin 2 u 1 sin 2
(2x – 12)
| u | 1 sin 2
60° 60° 120° 30°
57. (1) Let PQ be the tower of height x.
B 2x C x D
54. (*) Since height of bucket is 30 cm, when water level rise 15 P
cm then radius of top surface of water level
20 10
15 cm
2 A
Water flow in 1 minute = 10 cm = Length of pipe 45°
a x
Volume of bucket
Required time
Volume of water in 1 minute O
1 Q
[152 102 15 10] 15 30°
3 = 9.5 minutes b
5 5 10
55. (2) Let AC = x cm then BC = 7 – x cm
B
A

PQ x
tan 45
AQ AQ
x 5 AQ x
PQ 1 x
tan 30
BQ 3 BQ

C B BQ 3x
7–x
Now, OA2 AQ 2 BQ 2 OB 2 OQ 2
AB BC AC 5 7
S 6 a2 x2 3x 2 b2
2 2
6 (1)(6 x)( x 1) a2 b2 a2 b2
2 x2 a2 b2 x2 x
2 2
6(6 x x x 2 6)
a2 b2
6x 2
42 x 36 PQ
2
2 2
6x 42 x 36 58. (1)
2 D 75
Maximum value of D C
4x 2
75 3 5
Maximum value of 5 6
2 2 2
4
56. (1) u 2 2
cos (sin sin sin )

u sec sin sin 2 sin 2


P
(u sec sin ) 2 sin 2 sin 2
90°–
(squaring both side)
4
u 2 sec 2 2u tan sin 2 E 90°–
u 2 tan 2 2u tan u2 sin 2 0 A B
8
For real value of tan
2020-38 NTSE Solved Paper Stage-2

Olympe de Gouges was a French political activist who


PB 42 82 80 4 5 cm advocated for the right of women. French constitution
1 1791 reduced women status to Passive citizens
ABC ar( ABCD ) ...(i) 62 (1) New Harmony was a town located in Indiana, Robert
2
Owen, a social reformer reorganized the New Harmony
and ar ABC ar BPC ...(ii) in 1825 and conducted social experiments to establish
[at same base and between two parallel lines] an utopian socialist society that emphasized the rights
From (i) and (ii), of workers.
1 Louis Blanc was French socialist, historian and
ar( PCB ) ar( ABCD ) politician. He wanted government-supported co-
2
operatives that could guarantee the urban poor
1 employment.
64 32 sq. cm.
2 Karl Marx was a socialist who wanted properties to be
owned by the public rather than individuals.
1
PB CE 32 63. (2) Maasais are nomadic pastoralists of East Africa.
2 ‘Masai’ is an eastern Sudanic language. Maasai land
1 was taken Britishers who were ruling Kenya back then.
4 5 CE 32 German tangnaiyaka also came under their
2
occupation.
16 Samburu National Park is situated in Kenya.
CE
5 64. (4) Sultan Qutubuddin Aibak died while he was playing
Polo in 1210.
Also, PAB ~ BEC Polo originated in Persia, today’s Iran, between the
PA AB PB 6th century BC and the first century AD.
BE EC BC 65. (2) World war were that of men rather than women as they
were thought to biologically incapable to fight which
4 8 5 5 was essential at that time that is why Women’s status
BE 16 2 decreased during the world war.
To and fro movement from here and there during World
8 war and continuous changing positions made women
BE
5 carry clothes available for necessity.
66. (a)
1
ar( BEC ) CE BE
2 Place Event Year
1 16 8 64 I. Amritsar B. Rowlatt Act iv. 1919
cm2
2 5 5 5 II. Kheda C. Peasant Strike ii. 1917
59. (2) Let three numbers are a, b and c. III. Ahmedabad A. Mill workers Strike iii. 1918
a is smallest and c is largest then median = b = 5
IV. Champaran D. Plantation Workers i. 1916
a b c
A.T.Q., a 10 c 15 Strike
3
On solving, we get 67. (1) The correct sequence of the countries visited should
a 0 and c = 25 look like this.
So, a2 + b2 + c2 = 650. Austria-Italy-Germany-France.
60. (4) Let radius of sphere A is a The Treaty of Vienna was signed in Austria in 1815.
and radius of sphere B is b. The kingdom of Sardinia-Piedmont was a part of
800 modern-day Italy. Germany was ruled by the
4 b2 4 a2 1 ‘Hohenzollern dynasty’. The Treaty of Versailles was
100 signed in France in 1919.
68. (2) Nationalism in India emerged as a result of the British
b2 a2 9 b 3a
imperialist policies that exploited the country’s
4 3 4 3 x resources and people. British barbarity to the Indians
a b 1
3 3 100 induced sense of nationalism in Indians and they started
to recognize their history and culture to portray their
x a3 a3 1 identity as native rather than that of British origin.
1 69. (1) There was an increase in the population of France.
100 3 3 27
b 27a
Improper distribution of resources and that too being
x 96.3% (Approx.) concentrated to clergy and upper class monarchs
61. (3) The Declaration of Right of Man and Citizen’ was a angered the lower class who were called ‘bourgeoisie’
civil document that granted rights like liberty, property, and layed foundation of French revolution.
security and resistance to oppression to only men in 70. (*) Only I is correct
France as a result of the French Revolution of 1789.
2019-20 SAT SOLVED PAPER 2020-39

Kulaks opposed programme started by Stalin and his net return per unit area which allows them to make
associates which state that people should collectively efficient utilization of their resources.
work in farms and harvest produce and distribute it 80. (1) Mawsynram in the Khasi Hills of Meghalaya receives
through government equally which was against the rainfall over 1,080 cm in a year. Western Ghat too
ethos of kulaks. receives rainfall from the South-West monsoon ranging
71. (4) The ‘no rent’ campaign was launched by Bal from 250 cm to 400 cm. Though the Bengal delta
Gangadhar Tilak as a result of the 1896 famine in receives rainfall during the South-West monsoon
India seeking peasants not to pay rent to their season, the amount of rainfall is less compared to the
landlords. Congress did not support Tilak and the other two regions.
message which went to the poor farmers was that they 81. (2) Deccan plateau region has Black soil or Regur soil.
were not being supported by the Congress. Regur soil is favorable to cotton production.
72. (1) The two satyagrahs started by Mahatma Gandhi was 82. (4) Flight from Gandhinagar to Kolkata is most likely to
in support of peasants regarding the excess taxation fly over the Tropic of Cancer. The passengers will
which was introduced by the British on to them. cross the rivers Chambal, Son and Damodar on the
Mahatma Gandhi organized two satyagrahas, one in way, while crossing the states of Madhya Pradesh,
Champaran, Bihar and the other in Kheda, Gujarat Jharkhand and West Bengal.
between 1916 and 1917 in favour of peasants. 83. (2) Statements I, II and III are correct.
73. (3) Khadar is the newer alluvium, deposited by floods 84. (3) In the given cross-sectional view of the Himalayas,
annually. These are fertile soil which comprise of A,B,C and D represent Karakoram, Ladakh, Zaskar
mud, silt, clay and sand and suitable for the and Himadri respectively. The Karakoram, Ladakh,
cultivation of sugarcane, oilseeds, wheat, maize and Zaskar ranges are located north to the Great Himalayas.
rice. These are new soil found in the river bed after Towards their south is the Great or Inner Himalayas,
floods. also known as the ‘Himadri.
74. (4) Article 19 of the Indian constitution, allows the A,B,C and D represent Karakoram, Ladakh, Zaskar
citizens of India the right to freedom of speech and and Himadri respectively. South of these mountains
expression. Criticism to an extent has been allowed are Great or Inner Himalayas, also known as the
under article 19 however exceptions have been made ‘Himadri. The Karakoram, Ladakh, Zaskar ranges are
for those which act against the stat and are not located north to the Great Himalayas.
included in the Right to free speech and comes under 85. (1) Other than the statement I and III, all the statements
sedition. are true. Luni river drains in rann of kutch flowing
75. (3) Panchayati raj provided reservation to women (around throughout rajasthan and The Beas rises at Rohtang
one third seats were allotted to women including the Pass in Himachal Pradesh and joins the river Sutlej at
caste reservations) Harike in Punjab..
76. (2) While travelling from Srinagar to Kanyakumari, One 86. (3)
will have to cross three landlocked and three coastal
states. Column I Column II
3 coastal states- Tamil nadu, Karnataka. Maharashtra. A Density of I Man-Land ratio.
3 landlocked states- Madhya pradesh, Rajasthan, population
Punjab. (Considering Jammu & Kashmir as union B Population Growth II Increase or decrease in
territory.)
population.
77. (3) Loktak region has Centripetal drainage pattern.
Centripetal drainage pattern is formed when rivers C Natural Growth III Birth Rate minus Death
discharge their waters from all directions into a lake or Rate
a depression. D Distribution of IV Number of people in a
The river originating from Amarkantak would be a population given area.
perfect example of Radial drainage pattern. Radial 87. (2) All the statements other than (b) are true. In a federal
drainage patterns form when rivers originate from a form of governme nt, both stat e and union
hill and flow in all directions. governments operate. But because India follows
Narmada river has trellis drainage pattern. Trellis unitary method with federalism it is slightly
drainage pattern is formed when the primary tributaries different than exact federalism as exercised in the
of main rivers flow parallel to each other and secondary west..
tributaries join them at right angles. 88. (4) Elections provide us the opportunity to our right to
A drainage pattern which looks like tree branches with vote the candidate that we favor. EC also announces
lots of twigs is known as dendritic drainage pattern. the election schedule and then political parties start
78. (2) Colonization changed the face of forests and forestry their election campaign. It is followed by the polling
in India. New species of plants and animals were also of vote & counting and finally the announcement of
introduced. Teak and Chir Pine were introduced in election results.
India by the British to fulfil the demands of wood in 89. (2) The members of the Constituent Assembly from the
the European mainland. Provinces were elected by the Provincial Legislative
79. (3) Relay cropping is a variant of double cropping. The Assemblies.
second crop is planted before the harvest of the first 292 members of the total 389 were elected through
crop. Thus, both crops share some part of the season. the Provincial Legislative Assemblies.
Small farmers can increase their crop productivity and
2020-40 NTSE Solved Paper Stage-2

90. (4) The rule of law was expounded by Dicey in 1885, 275000, where Rs. 200000 is the amount spent on the
although it had been used earlier by Sir Edward building and Rs. 75000 is the amount taken from the
Coke formally. Rule of law as defined by dicey is bank as a loan to purchase machines. His working
something which is ot governed by anyone but has capital amounts to Rs. 50000 from his father which he
its own method of ruling like anyone is not above used to purchase raw materials.
the law whether it is the most supreme leader of the 98. (3) Average income is used to determine poverty here.
country. The average family income can be calculated as
91. (4) Amendment to the Constitution of India can be (Daily Wage*No. of working days*No. of working
initiated by either of the houses i.e Lok Sabha and members)/Size of the household
Rajya Sabha. Federal character of the constitution Jeewan= (100*15*2)/7= 428.75
cannot be amended given that any act which satisfies Sumer= (100*15*2)/6= 500
unitary method with federalism it may be amended Sheelam= (100*25*3)/4= 1875
with ratification by the half of the states. Yashwant= (80*25*3)/3= 2000
92. (2) Schedule VII of the Indian constitution provides three 99. (1) Maharashtra has the best health prospects than Odisha,
lists of subjects i.e. Union List, State list and Rajasthan and West Bengal from above mentioned
Concurrent list. Residuary subjects are not included data. Higher life expectancy, Low infant mortality
In Schedule VII. Computer software and digital and lower death rate show are good health indicators.
privacy belong to residuary subjects. 100. (2)
93. (1) Food security is a principle which emphasizes on the Minimum support price (MSP) is a “minimum price”
availability, affordability and accessibility of food to for any crop that the government considers as
all the people in a country. remunerative for farmers and hence deserving of
94. (4) In order to protect the industries of its own nation, the “support”. It is also the price that government agencies
government needs to impose import taxes on goods pay whenever they procure the particular crop.
imported from abroad. Revaluation of currency will The Centre currently fixes MSPs for 23 farm
not the current economy rather it will affect inflation commodities — 7 cereals (paddy, wheat, maize, bajra,
for larger part. jowar, ragi and barley), 5 pulses (chana, arhar/tur, urad,
Subsidizing the imports from china will become moong and masur), 7 oilseeds (rapeseed-mustard,
burden for the country. groundnut, soyabean, sunflower, sesamum, safflower
95. (2) There are 100 debtors in total. Out of them, 55 and nigerseed) and 4 commercial crops (cotton,
(22+18+15) debtors rely on informal sources of credit. sugarcane, copra and raw jute).
Informal sources of credit include landlords, money
lenders and friends and relatives. Whereas, 45 Tools Objectives
(5+15+10+15) debtors went to banks and other Issue Price To reduce malnutrition in India
financial institutions to avail credit. Thus, money To distribute food grains in deficit
lenders and landlords continue to be major sources of
areas and among poor families
credit in the village.
96. (4) Minimum To create more buffer stock
97. (2) Fixed capital is the long term assets of a business. Support To encourage farmers to produce
Working capital is the current assets of a business. In Price more food grains
the case of Himmatveer fixed capital amounts to Rs.
Visit https://telegram.me/ntse_tg for more books.
Stage-1 (2020-15) Solved Questions
Mental Ability Test – MAT

Directions: Qs. (1- 4): In each of the following questions find Directions: Qs. (11 - 12) : Which of the following question is
out the alternative which will replace the question mark. based on the following alphabet series:
(Andhra Pradesh-2020-21) (Bihar-2020-21)
1. Darwin : Evolution : : Archimedes : ? ABCDEFGHIJKLMNOPQRSTUVWXYZ
(1) Friction (2) Lubrication 11. Which letter is exactly midway between G and Q in the
(3) Buoyancy (4) Liquids given alphabet?
2. Snake : Fang : : Bee : ? (1) K (2) L (3) M (4) N \
(1) Honey (2) Humming 12. Which letter is sixteenth to the right of the letter which is
(3) Wax (4) Sting fourth to the left of I?
3. Tea : Cup : : Tobacco : ? (1) S (2) T (3) U (4) V
(1) Leaves (2) Hookah Directions: Qs. (13 - 14) : Two matrices containing letters are
(3) Toxin (4) Cheroot given below. The rows and columns are numbered 0 to 4 in matrix
1 and 5 to 9 in matrix II. Each letter from these matrices are
4. Aeroplane : Cockpit : : Train : ?
represented by its row number and the next by its column number.
(1) Wagon (2) Coach
(Karnataka-2020-21)
(3) Compartment (4) Engine
Directions: Qs. (5 - 6) : The letters in a word are replaced by 0 1 2 3 4 5 6 7 8 9
certain other letters according to a specific rule to form its 0 P W R MA 5 S T L K D
code. Answer the following questions accordingly. 1 M A P WR 6 K D S T L
(Andhra Pradesh-2020-21) 2 A P WR M 7 D S T L K
5. In a certain code "NEW YORK" is written as 111, how "NEW 3 W R MA P 8 T L K D S
JERSEY" will be written in that code ? 4 R M A P W 9 L K D S T
(1) 104 (2) 124 Matrix - I Matrix - II
(3) 134 (4) 114 13. The set of numbers which represents the word PALM is
6. In a certain code "MADRAS" is coded as NBESBT, how (1) 12, 73, 21, 43 (2) 21, 33, 58, 03
"BOMBAY" will be coded in that code ? (3) 34, 42, 86, 24 (4) 12, 20, 87, 32
(1) CPNCBX (2) CPNCBZ 14. The set of numbers which represents the word WARD is
(3) CPOCBZ (4) CQOCBZ (1) 01, 42, 43, 59 (2) 30, 33, 41, 88
Directions: Qs. (7 - 10) : In each of the following questions , (3) 44, 20, 31, 98 (4) 13, 04, 40, 75
various terms an alphabet series/number series/ alpha-numeral
Directions: Qs. (15 - 16) : Read the following information and
series are given with one term missing as shown by (?). Choose
answer the questions :
the missing term out of the given.
(Karnataka-2020-21)
(Bihar-2020-21)
The order of the letters that needs to come in the boxes to
3 4 9 13 complete a certain pattern is :
7. , , (?), ,
7 11 31 17 15. BY DW GT
(1) JP (2) JQ (3) KQ (4) KP
6 6 6 5
(1) (2) (3) (4) 16. L MN K N KM
15 19 17 15
(1) MLML (2) KLML
8. 625, 5, 125, 25, 25, (?), 5
(3) NMLL (4) KMML
(1) 5 (2) 25 (3) 125 (4) 625
17. In a code 2357 stands for the word WORK and the code
9. 4, 8, 28, 80, 244, (?)
14649 stands for the word STATE, then the code 14359
(1) 278 (2) 428
stands for: (Karnataka-2020-21)
(3) 628 (4) 728
(1) S C O R E (2) S T O R E
10. AJS, GPY, (?), SBK, YHQ
(3) S T E O R (4) S T O E R
(1) DMV (2) MVE
(3) OUA (4) QZI
M-2 Target NTSE
18. If a word BALL is coded as 288, then the word JACK is Directions: (Qs.29 - 31) : Each question has four terms. Three
coded as: (Karnataka-2020-21) terms are alike in some way. One term is different from three
(1) 330 (2) 240 others. Find out the correct term which is different from three
(3) 220 (4) 140 others and write its alternative number on your answer sheet
Directions(Qs. 19 - 20): In each of the following questions write against the proper question number.
which correct term in sequence replaces the question mark? (Uttar Pradesh 2020-21)
(Maharashtra-2020-21) 29. (1) Guru Ramdas (2) Guru Govind Singh
19. CD, HI, MN,? (3) Guru Granth Sahib (4) Guru Nanak Dev
(1) QS (2) OP 30. (1) 289 (2) 216
(3) RS (4) PQ (3) 512 (4) 729
20. BM26 , EN70 , HO120 , KP176? 31. (1) Sky–stars (2) Stadium–players
(1) NQ250 (2) NP224 (3) Hospital–patient (4) Moon–bird
(3) MQ221 (4) NQ238 Directions: (32 - 34): Question are based on the alphabet series
Directions (Qs. 21 - 22): In the following specific group of letters which is given below. Read the alphabet series carefully and
are given. From the given alternatives, find out the right letters find out the correct answer for each question and write its
which matches the given group. alternative number on your answer sheet against the proper
question number.
(Maharashtra-2020-21)
(Uttar Pradesh 2020-21)
21. GECA ZXVT SQOM
noijonp tak t lnopujet bnap ub
(1) YWUT (2) VTRQ
32. Which letter is been repeated the most in this series?
(3) MKIH (4) LJHF
(1) o (2) n
22. BEIN EHLQ ILPU (3) p (4) t
(1) NQUZ (2) HKOS 33. In this series, how many times the vowel occurs just before
(3) LOSY (4) JMQT and just after consonant?
Directions: Qs. ( 23 - 24) : In the following questions specific (1) one times (2) two times
group of numbers are given. From the given alternatives, find (3) three times (4) four times
out the right number which matches the given group. 34. In this series many time the vowel occurs just before the
consonant but hot just after consonant?
(Maharashtra-2020-21)
(1) One times (2) two times
23. 416 749 525
(3) three times (4) four times
(1) 982 (2) 864 (3) 637 (4) 319
35. Question is based on definite series. In given question
24. 294 648 448
some symbols are missing shown by (-). The missing
(1) 84 (2) 94 (3) 100 (4) 194
symbols are given in proper sequence as one of the four
Directions: (Qs. 25 - 27) : In each of the questions, a series is alternatives given under each question. Find out the correct
given with one term missing shown by question mark (?). This alternative and write number on the answer sheet against
term is one of the four alternatives given under it. Find the the question number. (Uttar Pradesh 2020-21)
correct alternative. c _ ba _ cb _ cc _ ac _ ba
(Rajasthan-2020-21) (1) cabac (2) ccabc
25. R, V, A, G, N, ? (3) acabc (4) bcaac
(1) U (2) V Directions: (Qs. 36 - 37) : In each of the questions, there are
(3) W (4) X four items, three of which are alike by some means or other
26. COVIDNINETEEN, VIDNINETEENCO, DNINETEENCOVI, ? while one is out of the class. Find out the odd items and indicate
(1) IDNINETEENCOV (2) NINETEENCOVID your answer by filling the circle of the corresponding letter on
(3) INETEENCOVIDI (4) INETEENCOVIDN the OMR Answer sheet.
27. 4, 5, 12, 39, 160, ? (West Bengal-2020-21)
(1) 225 (2) 695 36. (1) Bar (2) Pie
(3) 805 (4) 790
(3) Rectangle (4) Pictogram
28. Arrange the following in a meaningful sequence :
37. (1) SARS (2) COVID-19
(Rajasthan-2020-21)
1. Crop 2. Root (3) EBOLA (4) Typhoid
3. Stem 4. Seed Directions : (Qs.38 - 39): First pair is connected by some
5. Flower relationship. The same relationship is applicable for the next
(1) 4, 1, 2, 3, 5 (2) 2, 4, 3, 5, 1 pair. Identify the missing term in the second pair.
(3) 2, 3, 4, 5, 1 (4) 4, 2, 3, 5, 1 (Tamil Nadu-2020-21)
Stage - I Solved Questions M-3

38. throw : collect : : push : __?___ 49. Amit ranked 16th from the top and 29th from the bottom
(1) pull (2) door among those who passed the examination. 6 students does
(3) window (4) knock not participate and 5 failed in it. How many students were
39. apparel : shirt : : __?__ : necklace there in the class? (Delhi-2020-21)
(1) gold (2) jewellery (1) 50 (2) 55 (3) 44 (4) 52
(3) silver (4) ring Directions: Qs. (50 - 51) : You are provided with substitutes for
40. Arrange the given words in dictionary alphabetical order various mathematical symbols or numerals, followed by a question
and choose the word which comes in the middle. involving calculation of an expression or choosing the correct
(Tamil Nadu-2020-21) equation. You are required to put in the real signs or numerals in
(a) credential (b) creed the given equation and then solve the questions as required
(c) colour (d) credible (Follow BODMAS)
(e) create
(Andhra Pradesh-2020-21)
(1) (a) (2) (e)
50. If × means ÷, – means ×, ÷ means + and + means –, then the
(3) (b) (4) (d)
value of (3 – 15 ÷ 19) × 8 + 6 is
41. The sum of the first 12 terms of an AP. Whose nth term is
(1) –1 (2) 2
given by an = 3n + 4 is: (Delhi-2020-21)
(3) 4 (4) 8
(1) 262 (2) 272
51. If + means –, × means ÷, ÷ means + and – means ×, then the
(3) 282 (4) 292
value of 252 × 9 – 5 + 32 ÷ 92
1 1 (1) 95 (2) 168
42. If x 2 + 2 = 98 ( x > 0), then the value of x3 + 3 is
x x (3) 192 (4) 200
(Delhi-2020-21) 52. In a class of 52 students, 24 students drink milk, 28 students
(1) 970 (2) 1030 drink tea and 8 students do not drink anything. Find how
(3) –970 (4) –1030 many students drink both milk and tea.
43. If – stands for, + stands for ×, ÷ stands for – and × stands (1) 8 (2) 6
for +, which of the following equations is correct? (3) 2 (4) 10
(Delhi-2020-21) 53. If P means ‘÷’, Q means ‘+’, R means ‘–’ and S means ‘×’,
(1) 40 – 10 + 5 ÷ 4 × 5 = 21 (2) 40 + 10 – 4 × 5 ÷ 3 = 80 then what will be the value of 10R192P48S48P96Q1?
(3) 40 ÷ 10 – 4 × 5 + 3 = 32 (4) 8 – 4 × 40 ÷ 2 + 15 = 30 (Bihar-2020-21)
(1) 10 (2) 9
(3) 8 (4) 7
44. 10 + 25 + 108 + 154 + 225 find the value : 54. If ‘+’ means ‘×’, ‘–’ means ‘÷’, ‘×’ means ‘+’ and ‘÷’ means
(Delhi-2020-21) ‘–’ then compute the value of following expression :
(1) 10 (2) 8 (3) 6 (4) 4 45 – 9 + 4 × 5 (Bihar-2020-21)
(1) 21 (2) 25 (3) 26 (4) 23
45. If 7sin 24 : cos : 0 < , then the value of 55. Pointing to K, M says that I am the daughter of his
2 grandfather's only son. How is M related to K?
14 tan 75cos 7sec is equal to (Delhi-2020-21) (Bihar-2020-21)
(1) 3 (2) 4 (3) 1 (4) 2 (1) Mother (2) Maternal Aunt
46. A tower is observed from a point on the horizontal through (3) Daughter (4) Sister
the foot of the tower. The distance of this point from the 56. After walking 6 km., I turned right and covered a distance
foot of the tower is equal to the height of the tower. The 2 km., then turned left and covered a distance of 10 km. In
angle of elevation on the top of the tower is: the end I was moving towards the North. From which
(Delhi-2020-21) direction did I start journey? (Bihar-2020-21)
(1) 60° (2) 45° (3) 40° (4) 20° (1) North (2) South
(3) East (4) West
47. The next term of the AP 18, 50, 98, .... is
57. In a row of men, Manoj is 30th from the right and Kiran is
(Delhi-2020-21) 20th from the left. When they interchange their position,
(1) 146 (2) 128 Manoj becomes 35th from the right. What is the total
number of men in a row? (Bihar-2020-21)
(3) 162 (4) 200 (1) 34 (2) 54 (3) 45 (4) 44
48. If (p + q)th term of an AP is m and (p – q)th term is n, then pth 58. A clock runs 5 minutes faster in 1 hour. It has been set right
term is (Delhi-2020-21) at 12:00 in the afternoon. Now the clock is showing 6:30 in
(1) mn (2) mn the evening. So, that will be the actual time?
(Bihar-2020-21)
m n m +n (1) 5:00 pm. (2) 5:15 pm
(3) (4)
2 2 (3) 5:30 pm (4) 6:00 pm
M-4 Target NTSE

Directions: Qs. (59 - 61): Study the following Pie-chart and the 68. Raju walked 20 m towards south, then turning to his right
table. Answer the questions based on them. Total books in a and he walks 25 m, then turning to his left and he walks 30
library. m. Again he turns to his left and walks 25 m. How far is he
from his initial position (Assuming that all turns are at right
(Karnataka-2020-21)
angles): (Karnataka-2020-21)
(1) 20 m (2) 30 m
(3) 100 m (4) 50 m
Book s in bad condition 69. If according to mathematical code language 8+ 2 = 70 , 9+3
Tamil Kannada
27% 23% Type of = 87 , 10+ 4 =106 then 7 + 5 = ?
Percentage
book (1) 65 (2) 58
Kannada 2 (3) 51 (4) 63
70. If a and b are natural numbers. If A +B = 2020 then what will
Englis h 1
Telugu
English be the value of (-1)a +(-1)b. be (Rajasthan-2020-21)
20% Hind i 6 (1) 1 or –1
16%
Hindi Telugu 4 (2) 2020 or –2020
14% (3) 1010 or –1010
Tamil 8
(4) 2 or –2
59. If the total number of books in the library is 10,000, then the Directions: (Qs. 71 - 72) : The following table shows the record
total number of Hindi books that are in bad condition is: of surgery done in a hospital. This record shows the data from
(1) 82 (2) 84 (3) 90 (4) 80 January to July. Study the table carefully and answer the
60. If the number of Kannada books in bad condition is 92, following questions.
then total number of English books in the library will be : (Rajasthan-2020-21)
(1) 1200 (2) 1600 (3) 4800 (4) 4000
61. If the total number of books in bad condition is 860, then Total Total
total number of books in the library will be : Month successful unsuccessful
(1) 20,000 (2) 8600 (3) 4300 (4) 43,000 surgery surgery
62. 'a' and 'b' together can complete a piece of work in 9 days. January 5 3
'a' alone can complete the work in 36 days. The number of February 4 4
days 'b' alone take to complete the work :
March 5 2
(Karnataka-2020-21)
(1) 18 (2) 30 (3) 12 (4) 14 April 6 3
63. If the sum of one-third of a number and half of the same May 4 2
number exceeds two-third of the number by 5, then two-
June 3 3
fifth of the number is equal to : (Karnataka-2020-21)
(1) 12 (2) 15 (3) 18 (4) 30 July 2 4
64. A fruit seller bought 72 oranges at ` 600. He sold 50 of them 71. As per the above table, what is the percentage of successful
at ` 11 each and the remaining at ` 236. His profit in this surgery?
business is : (Karnataka-2020-21) (1) 32% (2) 29%
(1) ` 786 (2) ` 600 (3) ` 442 (4) ` 186 (3) 58% (4) 21%
65. A beaker half filled with water weighs 600 g. When it is 72. In which month the percentage of successful surgery was
2 highest in the hospital?
empty weighs 150 g. The weight of the beaker when of it (1) January (2) April
5
(3) July (4) March
is filled with water is : (Karnataka-2020-21)
73. How many integers are in between 1 and 100 which satisfying
(1) 240 g (2) 450 g (3) 360 g (4) 510 g
all the following conditions? (Rajasthan-2020-21)
66. If the length of a rectangle is increased by 30% and breadth
(i) After divided by 3, remainder is 1.
is decreased by 20%, then the variation in the area of new
(ii) After divided by 5, remainder is 1.
rectangle is : (Karnataka-2020-21)
(iii) After divided by 7, remainder is 0.
(1) Decreases by 4% (2) Increases by 4%
(1) 0 (2) 1
(3) Decreases by 25% (4) Increases by 25%
(3) 2 (4) 3
67. Ratio of Ravi's age to Raju's age is equal to 4 : 3. Ravi will be
74. What is the difference between the biggest 3-digit number
26 years old after 6 years. The present age of Raju is:
having 8 as tens place and divisible by 4 and smallest 3-
(Karnataka-2020-21)
digit number having 8 as tens place and divisible by 4?
(1) 15 years (2) 14 years
(Uttar Pradesh 2020-21)
(3) 13 years (4) 12 years
(1) 888 (2) 808
(3) 708 (4) 788
Stage - I Solved Questions M-5

75. If 50% books in a library are in English language and 7th / 84. If 1st January, 2008 is Tuesday then what day of the week
10 of the rest in Hindi language which of the following lies on 1st January, 2009? (West Bengal-2020-21)
shaded parts approximately represent books in Hindi (1) Wednesday (2) Thursday
language. (3) Sunday (4) Monday
Directions: Qs. (85 - 88) Choose the correct one and give the
answer by filling the circle of the letter denating your selected
(1) (2) answer on the OMR Answer Sheet.
(West Bengal-2020-21)
85. Ibrahim ranks 8th in a class of 35 students. What is his rank
from the last?
(1) 26th (2) 27th
(3) (4) (3) 29th (4) 28th
86. If there is a common root of the equation x 2 ax b 0
76. The length of each of two parallel chords AB and CD is 12 and x 2
bx a 0 then the value of a + b is
cm. If the length of the radius of the circle is 10 cm, then the (1) 1 (2) –1
distance between two chords is (West Bengal-2020-21)
(1) 12 cm (2) 14 cm (3) 16 cm (4) 18 cm 1
(3) 0 (4)
77. The ratio of the volumes of two cubes is 1 : 27, the ratio of 2
total surface areas of two cubes is (West Bengal-2020-21) 87. A principal becomes twice of its amount in 10 years at a
(1) 1 : 6 (2) 1 : 8 (3) 1 : 9 (4) 1 : 18 certain rate of simple interest. At the same rate of simple
78. If sin cos 0, 0 90 an d sec cosec x interest, that principal becomes thrice of its amount in
(1) 15 years (2) 20 years
then x will be (West Bengal-2020-21)
(3) 25 years (4) 30 years
(1) 1 (2) 2 (3) 2 2 (4) 2 88. Keeping the radius of a right circular cone same, if the
79. If L denotes ×, M denotes ÷, P denotes + and Q denotes – height of its increased thrice, the volume of it will be
, then 7P24M8Q6M2L3 = (West Bengal-2020-21) increased by
(1) 1 (2) 2 (3) 3 (4) 4 (1) 100% (2) 200%
Directions: (Qs. 80 - 82) : The diagram shows the survey on a (3) 300% (4) 400%
sample of 500 persons with respect to their knowledge of 89. If a + b = 51; x + y = 5, then p - q is: (Tamil Nadu-2020-21)
Bengali, Hindi and English. (1) 1 (2) – 1
(3) 3 (4) 20
(West Bengal-2020-21)
90. After walking 10 kms, a person turned right and covered a
distance of 5 kms. Then, turned left and covered a distance
of 20 kms. In the end, the person was moving towards south.
70 120
65 In which direction did the person start his journey?
Hindi Bengali
(Tamil Nadu-2020-21)
53 (1) West (2) East
47 62
(3) North (4) South
83 91. A group of 150000 persons consists of captains and soldiers.
English There is one captain for every 15 soldiers. The number of
captains in the group is: (Tamil Nadu-2020-21)
(1) 10000 (2) 9375
80. How many persons know the Bengali and English language (3) 9275 (4) 9475
but not know the Hindi language? 92. The number of numbers from 1 to 200 which are divisible
(1) 53 (2) 62 (3) 65 (4) 37 by 10 but not divisible by 30 is: (Tamil Nadu-2020-21)
81. How many persons know all the three languages? (1) 13 (2) 15 (3) 14 (4) 20
(1) 65 (2) 62 (3) 53 (4) 47 93. What value will replace ‘?’ in the figures given below :
82. How many persons who do not know Hindi language? (Delhi-2020-21)
(1) 265 (2) 200 (3) 255 (4) 201
83. Rahim walks 15 km towards North. From there he walks 9 42 36 38
km towards South. Then he walks 8 km towards East. How
far and in which direction is he now from his starting point? 3 2 15 3 1 9 10 ? 20
(West Bengal-2020-21)
(1) 7 km North-East (2) 10 km North-East 6 9 19
(3) 10 km South-West (4) 7 km South-East (1) 0 (2) 1 (3) 2 (4) 3
M-6 Target NTSE

Directions: Qs. (94 - 96): In each of the following letter series. (1) Both (A) and (R) are true and (R) is the correct
Some of the letters are missing which are given in that order as explanation of (A)
one of the alternatives below it. Choose the correct alternatives. (2) Both (A) and (R) are true but (R) is not the correct
explanation of (A)
(Andhra Pradesh-2020-21)
(3) (A) is true but (R) is false
94. A__BBC__AAB__CCA__BBCC
(4) (A) is false but (R) is true
(1) B, A, C, B (2) A, C, B, A
102. Assertion (A) : Vaccines prevents disease.
(3) A, B, B, A (4) C, A, B, A
Reason (R) : Vaccine must be given to children.
95. B__AC__CC__CB__AB__AC
(1) Both (A) and (R) are true and (R) is the correct
(1) C, B, A, B, A (2) B, B, A, A, C
explanation of (A)
(3) A, B, B, B, C (4) A, A, B, B, A
(2) Both (A) and (R) are true but (R) is not the correct
96. __ACCA__CCCA__ACCC__AAA
explanation of (A)
(1) A, C, C, A (2) C, A, A, A
(3) (A) is true but (R) is false
(3) C, C, A, A (4) C, A, A, C
(4) (A) is false but (R) is true
Directions: Qs. (97-100): In these questions; numbers are
Directions: Qs. (103 - 104) : Study the diagram shown below
arranged on the basis of some rules. One place is vacant, which
and answer the questions based on it. This Venn diagram shows
is indicated as ?. Find out the correct alternative to replace the
children’s choice of colour.
question mark "?"
(Bihar-2020-21)
(Andhra Pradesh-2020-21)
• Rectangle shows the choice of white colour..

25 18 38 17 89 16 • Circle shows the choice of Black colour..


97. • While capsule shaped shows the choice of Brown
6 8 ?
colour.
(1) 13 (2) 15 (3) 17 (4) 19
98. 20
?
235 4 5 10
20 50 30 100
117 7

59 15 30
29
103. How many children like both brown and white but do not
(1) 327 (2) 386 (3) 438 (4) 469 like black?
(1) 10 (2) 20
99. 93 27 79 38 67 16 (3) 50 (4) None of these
104. What is the ratio of children who do not like black to children
63 37 42 who only like black and white?
(1) 3 : 1 (2) 3 : 2
3 4 ? (3) 12 : 5 (4) 5 : 12
Directions: Qs. (105 - 106) : In each of the following questions,
(1) 5 (2) 6 (3) 8 (4) 9 a set of figure carrying certain characters is given. Assuming
100. If ‘UNCLE’ is written as 'QQYOA' then what will be written that the character in each set follow a similar pattern. Find the
to ' COLUMN'? missing character in each case. Mark the correct selected options
(1) YROXIQ (2) YRHQIQ as the answer on the answer-sheet as instructed.
(3) YHRXIQ (4) YRHXIQ
(Bihar-2020-21)
Directions: Qs (101 - 102) : In each of the questions given below,
105. 10 11 9 10 3 (?)
there are two statements labelled as Assertions (A) and Reason
(R). Mark your answer as per the options provided below the
question.
(Bihar-2020-21)
101. Assertion (A) : We prefer to wear white clothes in winter. 12100 8100 441
Reason (R) : White clothes are good reflectors of heat.
(1) 8 (2) 9 (3) 7 (4) 21
Stage - I Solved Questions M-7

110. The number that represents those who play cricket but not
106. 5 6 7 hockey is
3 4 5 (1) 18 (2) 20
(3) 26 (4) 28
9 10 11
111. The number that represents those who play neither tennis
345 460 (?) nor football is
(1) 9 (2) 16
(1) 535 (2) 577
(3) 18 (4) 26
(3) 755 (4) 775
112. In the questions given below the numbers in the figures
107. How many combinations of two digits numbers having 8 can
are related. Identify their relationships and find the missing
be made from the following numbers? (Bihar-2020-21)
numbers in the given figures. (Karnataka-2020-21)
8, 5, 2, 1, 7, 6
(1) 9 (2) 10
(3) 11 (4) 12
4 12 4 14 12 14
108. The following question is based on figures given below.
Each circle represents one item. Match the figure with items 674 1762 ?
in the question on the basis of their relationship.
(Karnataka-2020-21)
6 4 6 18 16 10

(1) 2502 (2) 2702


(3) 2603 (4) 2707
113. If the product of (1 P 6) and (2 Q) is 4 2 1 2, then the values
a b c of P and Q are respectively (Karnataka-2020-21)
(1) 4 and 2 (2) 5 and 7
(3) 4 and 7 (4) 5 and 2
114. In the following question there is a specific relationship
between the first and second term. The same relationship
exists between the third and fourth term. Considering the
relationship select correct alternative to replace question
d e f mark: (Maharashtra-2020-21)
Writers, Teachers, Researchers: 4 : 80 :: 21: ?
(1) f (2) d (1) 9702 (2) 8702
(3) c (4) a (3) 8820 (4) 421

Directions: (Qs. 109- 111) : Read the information given below Directions: Qs. (115 - 116): Which symbols will come in the
and answer the questions : order. Choose the correct alternative.
(Karnatka-2020-21) (Maharashtra-2020-21)
In the following figure circle represents those who play cricket, 115. , , ,?
triangle represent those who play cricket, triangle represents those
(1) (2)
who play football, square represents those who play hockey and
rectangle represents those who play tennis. Based on the figure (3) (4)
answer the following questions. 116. WO, WO , WO ,?
(1)O n (2) O
7 Hockey
(3) O (4) O W
Football 117. In the following questions there is a specific relation
2 8 2 between first and second term. The same relationship exists
3 4 5 9 Cricket between third and fourth term, which will replace the
6
question mark. Select the correct alternative from the given
alternative.
Tennis EJOT : VQLG::BGLQ: ? (Maharashtra-2020-21)
109. The number that represents those who play cricket, football (1) DINS
and hockey is: (2) RMHC
(1) 2 (2) 4 (3) SNID
(3) 5 (4) 8 (4) EJOT
M-8 Target NTSE

Direction (Q. 118): Observe the pyramid of number and choose 124. Statement: Unemployment is one of the main reasons for
the correct alternative which will replace question mark. the poverty of the country. (Rajsthan-2020-21)
Conclusions:
(Maharashtra-2020-21)
(I) To end poverty, it is required to create employment
118.
opportunities.
(II) All the people in the country are unemployed.
1
(1) Only conclusion I follows.
2 3 4 (2) Only conclusion II follows.
5 6 7 8 9 (3) Both conclusions I and II follow.
10 11 12 13 14 15 16 (4) Neither conclusion I nor II follows.
17 18 19 20 21 22 23 24 25 125. Which one of the following Venn-diagrams correctly
26 27 28 29 30 31 32 33 34 35 36 represents the relation between currency, Rupee and Dollar?
37 38 39 40 41 42 43 44 45 46 47 48 49 (Rajasthan-2020-21)

18284041 : 24344645 : : 20304243 : ?


(1) 22324443 (2) 21314344
(3) 22324445 (4) 24344647
(1) (2)
In the following table the digits are assigned with certain
symbols. Observe the carefully and choose the correct
alternative to answer the question.
Digits 1 2 3 4 5 6 7 8 9

Symbols
(3) (4)
119. Which number will be expressed by ?
(Maharashtra-2020-21) 126. In a coded language, ONLINE is written as LNOENI and
(1) 951478 (2) 958174 SILENT is written as LISTNE; then in the same coded
(3) 951847 (4) 951874 language, LISTEN will be written as. (Rajasthan-2020-21)
Directions (Qs. 120 - 121) : In the following figure numbers are (1) ILSNET (2) SILENT
written with a specific rule. Find the rule and decide which (3) SILNET (4) SILETN
alternative will be in place of question mark. 127. In a coded language, AVOID is written as 73564 and CHINA
is written as 28617; then in the same coded language, COVID
15 6 6 7 50 10
36 21 ? will be written as (Rajasthan-2020-21)
120. (1) 53246 (2) 25364 (3) 25346 (4) 25634
4 5 10 128. The two positions of a single cube are shown below. The
(1) 140 (2) 220 (3) 320 (4) 500 six faces of this cube are coloured with Green, Yellow, Red,
36 9 25 Black, Pink and White. If the Green coloured face is at top
then what will be the colour at bottom's face?
121. 49 26 64 81 21 25 64 ? 144 (Rajasthan-2020-21)
25 16 36
(1) 19 (2) 23 (3) 31 (4) 25
Directions (Qs. 122 - 123) : The numbers in the figure show Yellow Pink
number of tourists from different states. Observe the figure and
n

Black
Gree

choose the answer from given alternatives for following


questions. Red Green
(Maharashtra-2020-21)

(1) White (2) Green (3) Yellow (4) Red


56 52 67
21 Direction (Q. 129) : The following questions are based on the
41 46
diagram given below. Study the diagram carefully and answer
72 the questions based upon it.
122. How many tourists visited all three states? (Rajasthan-2020-21)
(1) 119 (2) 108 (3) 21 (4) 195 In the diagram
123. Find number of tourists visting only two states? (i) Circle represent unemployed
(1) 93 (2) 98 (3) 87 (4) 139 (ii) Square represent hard working
Stage - I Solved Questions M-9

(iii) Triangle represent rurals 136. In any code language PD is written as 8. What will be code
(iv) Rectangle represents intelligents. of HB in the same code language? (Uttar Pradesh 2020-21)
(1) 16 (2) 10
(Rurals) (3) 4 (4) 18
5
15 (Unemployed) 137. Numbers are placed in figure on the basis of some rules.
One place is vacant which is indicated as (?). Find out the
17 3 correct alternative for the vacant place and write its number
19 11 2 6 7 against the proper question number on your answer sheet:-
14 1 (Uttar Pradesh 2020-21)

16
23 26 29
9
12 8
(Hard 18
working) 10 169 169 ?
(Intelligents)
16 12 15 13 14 17
(1) 169 (2) 267
129. The Rurals, which are neither hard-working nor Intelligent (3) 209 (4) 389
but unemployed are represented by
(1) 1 (2) 2 (3) 3 (4) 6 Directions (Qs. 138-139) : Question are based on number/figure
series. In each series missing term is mentioned by question
Directions (Qs. 130-131) : In each of the following questions, mark (?). Find out the missing term in given alternatives and
there is a certain relationship between two given numbers on write its alternative number against the correct question number
left side of sign (::) and one number is given on the right side of on your answer sheet.
sign (::) while another number is to be found from the given (Uttar Pradesh 2020-21)
alternatives having the same relationship with the number as 138. 22, 3, 19, 9, 16, 27, 13, 81, 10, ?, ?
the numbers of the given pair bear. Choose the correct (1) 7, 243 (2) 162, 13
alternatives. (3) 243, 7 (4) 342, 4
(Rajasthan-2020-21) 139. 392, 365, 342, 323, 308, ?, 290, 287
130. 20 : 11 : : 102 :? (1) 297 (2) 293
(1) 49 (2) 52 (3) 61 (4) 96 (3) 289 (4) 301
131. 6 : 35 : : 7 : ?
(1) 48 (2) 42 (3) 34 (4) 13 Directions (Qs. 140-141) : In each question, there are two words
separated by ‘:’ and other two separated from the first two by
Directions (Qs. 132-133) : There are four terms/figures in each the symbol :: ‘Find the relation between two sets of words and
question. The terms right to the symboll :: have same relationship select one word from the right side of ‘:’ which have the same
as the two terms of the left symbol :: Out of the four terms/figure relation as left set of word of ‘::’. Fill the circle of the letter
one is missing, which is shown by (?). Four alternative are denoting your selected answer on the OMR Answer sheet.
given for each question. Find out the correct alternative and (West Bengal-2020-21)
write its number against the corresponding question on your 141. Lion : Roar :: Ass : ?
answer sheet. (1) Trumpet (2) Bray
132. 326 : 3649 :: 534 : ? (3) Bark (4) Howl
(1) 5932 (2) 25916 (3) 16925 (4) 5874 142. 42 : 56 :: 110 : ?
(1) 148 (2) 184
133. : :: : ? (3) 132 (4) 124
Directions (142-143) : In these questions, numbers are placed
in figures on the basis of some rules. One place in the figure is
indicated by the interrogation sign (?). Find out the correct
alternative to replace the question mark and indicate your
(1) (2) (3) (4) answer by filling the circle of the corresponding letter of
134. If is any code language MARCH is written as KCPEF. What alternatives in the OMR answer sheet.
will be the code of ORDER in the same code language? (West Bengal-2020-21)
(Uttar Pradesh 2020-21)
(1) MTBGP (2) MPBCP 5 6 6 7 4 8
(3) LOABO (4) QPFCT 142. 12 21 ?
135. If in any code language PEN is written as 66. What will be
4 5 10
the code of LEG in the same code language?
(1) 22 (2) 32 (3) 30 (4) 23
(Uttar Pradesh 2020-21)
(1) 67 (2) 65 (3) 53 (4) 24
M-10 Target NTSE

11 5 15 9 21 16
143. 3C 27D 9E
71 84K 12M
153.
5A ? 13G
96 144 ?
(1) 185 (2) 165 (3) 175 (4) 195 (1) 65D (2) 65G (3) 35H (4) 65B
154. Select from the given diagrams (1), (2), (3) or (4), the one
Directions (Qs. 144-147) : In each of the following questions, a that illustrates the relationship among the three classes.
number series is given with one term missing. Choose the correct Engineers, Agricultural Officers, Professionals.
alternative that will continue the same pattern and answer on (Tamil Nadu-2020-21)
the OMR Answer sheet by filling the circle.
(West Bengal-2020-21)
144. 95, 94, 92, 89, 85, 80, ?
(1) (2)
(1) 78 (2) 76 (3) 74 (4) 72
145. 720, 360, 120, 30, 6, ?
(1) 0 (2) 1 (3) 2 (4) 3
146. 5, 12, 26, 47, 75, __?___ (Tamil Nadu-2020-21)
(1) 99 (2) 110 (3) 105 (4) 93
(3) (4)
147. 2, 2, 5, 4, 10, 8, 17, 14, __?___, 22 (Tamil Nadu-2020-21)
(1) 24 (2) 25 (3) 26 (4) 30
148. 'Angle' is related to 'Radian' in the same way as 'Force' is 155. In a code language : (Tamil Nadu-2020-21)
related to (Tamil Nadu-2020-21) (i) 'im be pee' means 'petals are blue'
(1) Newton (2) Pascal (3) Joule (4) Watt (ii) 'sik hee' means 'red flowers'
149. If, in a certain code language 'FRUIT' is coded as 'ZLOCN' (iii) 'pee mit hee' means 'flowers are fragrant'.
then 'FLOWER' is coded as : (Tamil Nadu-2020-21) Then, 'fragrant are red flowers' is:
(1) ZFIQYM (2) AFIQYM (1) pee im mit hee (2) im be sik mit
(3) ZFIQYL (4) AFIQYL (3) be sik pee mit (4) sik hee pee mit
150. Select the combination of numbers so that the letters 156. Choose the best reaction from the alternatives for the given
arranged accordingly will form a meaningful word. situation.
(Tamil Nadu-2020-21) You are cycling on the narrow crowded street and suddenly
IPECLO you see a one rupee coin on the pavement. What action
123456 will you take? (Tamil Nadu-2020-21)
(1) 265143 (2) 651432 (1) Take the coin and give it to a beggar.
(3) 265134 (4) 261543 (2) Take the coin and pocket it yourself.
151. Two statements (I) and (II) are followed by conclusions (1) (3) Leave it where it is.
and (2), Read the statements and conclusions and give the (4) Take the coin and deposit in the nearest police station.
correct answer. (Tamil Nadu-2020-21) Directions Qs. (157 - 161): Read the following and answer the
Statements : questions given below.
(I) Some fans are sweets. (II) All sweets are tube lights.
(Andhra Pradesh-2020-21)
Conclusions :
(1) There are six persons A, B, C, D, E and F.
(1) Some fans are tube lights.
(2) C is the sister of F.
(2) Some tube lights are fans.
(3) B is the brother of E's husband.
(1) Either conclusions follow
(4) D is the father of A and grandfather of F.
(2) Only conclusion (1) follows
(5) There are two fathers, three brothers and a mother in the
(3) Only conclusion (2) follows
group.
(4) Neither conclusions (1) nor (2) follow
157. Who is the mother ?
Directions: (152-153) : Find the missing character. (1) A (2) B (3) D (4) E
(Tamil Nadu-2020-21) 158. Who is E's husband ?
(1) B (2) C (3) A (4) F
28 159. How many male members are there in the group ?
? 29 (1) 1 (2) 2 (3) 3 (4) 4
5 160. How is F related to E ?
152. (1) Uncle (2) Husband
4 –2
(3) Son (4) Daughter
3
161. Which of the following is a group of brothers ?
(1) 23 (2) 22 (3) 27 (4) 3 (1) ABF (2) ABD (3) BFC (4) BDF
Stage - I Solved Questions M-11

Directions (Qs. 162 - 166) : Study the following information Directions (Qs. 173 - 176): Read the information given below
and answer the questions given below: carefully and answer the questions that follow:
(Bihar-2020-21) (Rajasthan-2020-21)
P is the father of A whose son is S. A, B, C, D, E, F and G are seven students of a school, who are
L is K's mother who is also the sister of S. studying in seven different classes V, VI, VII, VIII, IX, X and XI.
162. How is A related to L ? Everyone of them likes a particular subject viz. English, Hindi,
(1) Brother (2) Husband Physics, Chemistry, Mathematics, History and Geography.
(3) Wife (4) Sister Students, classes and subjects can always be in different order
163. How is A related to K ? as given.
(1) Father (2) Mother B likes History and does not study in Class VII. The one who
(3) Father in Law (4) Son likes English studies in Class IX. The one who studies in Class V,
164. How is P related to L ? likes Chemistry. F likes Physics. C is studying in Class VI and
(1) Father (2) Son doesn’t like Geography. E is studying in Class X and likes
(3) Father in law (4) Daughter in law Mathematics. D likes Chemistry. G doesn’t like English and G is
165. If 'ROME' is written as 'MORE' then 'DARE' is written as: not in Class VII or VIII. F is not in Class VIII.
(1) RDAE (2) RDEA 173. F is in which Class?
(3) RAED (4) RADE (1) VII (2) IX
166. p, q, r, s and t are seated around a circular table facing the (3) XI (4) Data inadequate
centre: (Karnataka-2020-21) 174. G is in which Class?
i. 't' sits second to the right of 's'. (1) VI (2) VII
ii. 'r' sits second to the right of 't'. (3) XI (4) IX
iii. 'p' is not an immediate neighbour of 's'. 175. Which subject does A like?
Who sits between 's' and 't': (1) Hindi (2) Geography
(1) q (2) r (3) p (4) t (3) English (4) Chemistry
176. Who studies in Class - VIII?
Directions Qs. (167 - 169) : Read the information given below
(1) B (2) C
and answer the questions :
(3) A (4) E
(Maharashtra-2020-21)
Sushil, Vipin, Prashant, Amar are four class friends. Sushil does Directions (Qs. 177 - 178): Read the information given below
not like dance. Vipin likes only music and dance. Only three of and answer the questions based on it.
them like dance and craft. Prashant likes all subjects except music. (Uttar Pradesh 2020-21)
Sushil is master in drawing and music. Kriti and Amit are two children of Smt and Shri Mittal. Amit,
167. Amar likes which subjects: married to Ananya, who is daughter of Smt. Goyal. Shri Goyal
(1) music and craft (2) dance and drawing had married to Rita. Sonu and Rakesh are children of Sumer and
(3) dance and craft (4) music and drawing Rita. Urvi and Poorvi are daughters of Ananya and Amit.
168. Which subject Vipin, Prashant and Amar likes? 177. What is the relation of Sumer with Poorvi?
(1) drawing (2) music (1) Brother (2) Uncle
(3) dance (4) craft (3) Maternal Grandfather (4) Father
169. Who likes drawing? 178. What is the relation of Urvi with Kriti?
(1) Sushil and Vipin (2) Vipin and Prashant (1) Sister (2) Niece
(3) Sushil and Prashant (4) Prashant and Amar (3) Aunty (4) Daughter
Directions (Qs. 170 - 172): A, B, C, D, E, F, G, H eight friends are Directions (Qs.179 - 182) : Question are based on the
sitting around their circular table. C is sitting in front of A and information given below. Read the information carefully and
H is sitting at the first position to the left of A. F is in front of H. find out the correct answer from the four alternatives and write
Whereas D is at the first position to right of E and C is at the first its alternative number to your answer sheet against the proper
position to left of E. A is between G and H. Choose the correct question number.
alternatives for the question given below: (Uttar Pradesh 2020-21)
(Maharashtra-2020-21) There is a family consists of six member A B C D E and F. C is the
170. Who is sitting between B and C? sister of F. B is brother of E's husband , F is grandson of D.
(1) D (2) G (3) F (4) H 179. How many male members are there in the family?
171. Who is sitting between A and B? (1) 3 (2) 2 (3) 4 (4) 1
(1) D (2) G (3) E (4) F 180. How is B related to F?
172. If E and B would have interchanged the places then who (1) Father (2) Brother
will be sitting at the first position to the left of B? (3) Uncle (4) Aunty
(1) G (2) C (3) F (4) E 181. Who is the husband of E?
(1) D (2) B (3) F (4) A
M-12 Target NTSE
182. Pointing to a person, a man said to a woman, “His mother is 191. Choose the figure which is different from the rest.
the only daughter of your father.” How was the woman
related to the person? (West Bengal-2020-21)
(1) Wife (2) Daughter
(3) Aunt (4) Mother
Directions (Qs. 183 - 185) : Read the following information
and answer the questions given below. (1) (2) (3) (4)
(West Bengal-2020-21)
Directions: (Qs.192 - 194): Each of the following questions
There are six children playing football namely A, B, C, D, E and F.
consists of five figures marked A, B, C and D constitute the
A and E are brother; F is the sister of E; C is the only son of A’s
problem set while figures 1, 2, 3 and 4 constitute the answer set.
uncle; B and D are the daughters of the brother of C’s father.
There is a definite relationship between figures A and B. Establish
183. How many female players are there?
a similar relationship between figures C and D by selecting a
(1) Two (2) Three (3) Four (4) Five
suitable figure from the Answer set that would replace the
184. How C is related to F?
question mark (?) in figure (D).
(1) Brother (2) Uncle
(3) Son (4) Cousin (Andhra Pradesh-2020-21)
185. How D is related to A? 192. Problem Figures :
(1) Cousin (2) Sister
(3) Niece (4) Uncle
Directions (Qs.186 - 188) : Read the following and answer the questions: ?
(Tamil Nadu-2020-21)
(i) There are four persons P, Q, R, S
(A) (B) (C) (D)
(ii) They wear different colour caps - Red, Green, Blue and White.
(iii) P is neither wearing White nor Green. Answer Figures :
(iv) Q is not wearing White.
(v) S wears Red.
Based on the above information answer the questions.
186. R wears : (1) (2)
(1) White cap (2) Green cap
(3) Red cap (4) Blue cap
187. P wears :
(1) White cap (2) Green cap
(3) Red cap (4) Blue cap
(3) (4)
188. Q wears :
(1) White cap (2) Green cap
(3) Red cap (4) Blue cap 193. Problem Figures :
Directions (Qs.189 - 191) : Out of the four figures (1), (2), (3),
(4) given in each question, three are similar in certain way.
Choose the figure which is different from the other figures.
?
(Andhra Pradesh-2020-21) (A) (B) (C) (D)
189. Choose the figure which is different from the rest. Answer Figures :

(1) (2)

(1) (2) (3) (4)


190. Choose the figure which is different from the rest. (3) (4)

194. Problem Figures :

?
(1) (2) (3) (4)
(A) (B) (C) (D)
Stage - I Solved Questions M-13

Answer Figures : Direction (Q. 198): In the given question a complex figure is
given. Find out which of the simple figures given in the
alternatives is hidden in the complex figure.
(1) (2)
(Maharashtra-2020-21)
198. Question figure :

(3) (4)

Direction (Q. 195) : Find the missing part of the given figure
from the alternatives. Answer figure
(Karnatka-2020-21)

(1) (2)

195.

(3) (4)

(1) (2) Direction (Q. 199) : Find the correct mirror image of the given
figure, when mirror is placed on right side of the figure.
(Rajsthan-2020-21)
199. Question Image :
(3) (4)

196. Complete the given figure analogy by choosing the correct


answers from the given alternative. (Karnatka-2020-21)

: :: : ?

Answer Image :

(1) (2) (1) (2)

(3) (4) (3) (4)

197. Find number of triangles in the given figure. 200. Go through the diagram and choose the answer.
(Maharashtra-2020-21) The number of triangles in the given diagram is :
(Tamil Nadu-2020-21)

(1) 16 (2) 20 (3) 24 (4) 32 (1) 17 (2) 11 (3) 8 (4) 10


M-14 Target NTSE

a b c a b c
201. If , then value of will be
3 5 7 b Above 40
years Maharashtra
(Delhi 2019) 20% 5%
15%
(1) 7 (2) 3 (3) 10 (4) 5 Below 20 years 20 - 40 Others Karnataka
60% years
West 60%
202. If 17 x 11 20%
6 , then value of x will be
2
11 Bengal
20%
(Delhi 2019)
(1) 11 (2) 23 (3) 6 (4) 24
203. If P denotes +, Q denotes –, R denotes × and S denotes , 209. Number of visitors from Karnataka in the age group of
which of the following statements is correct? (Delhi 2019) 20-40 years
(1) 36 R 4 S 8 Q 7 P 4 = 10 (1) 20000 (2) 18000
(3) 12000 (4) None of these
(2) 16 R 12 P 49 S 7 Q 7 = 200
210. Number of visitors from Maharashtra below the age of 20
(3) 32 S 8 R 9 = 160 Q 12 R 12 years
(4) 8 R 8 P 8 S 8 Q 8 = 57 (1) 3000 (2) 5000
204. If a and b are the roots of x2 – 2x – 1 = 0, then value of (3) 60000 (4) 8000
a2b + ab2 is (Delhi 2019) 211. How many visitors were below 20 years of age but were
neither from Karnataka, nor Maharashtra or West Bengal?
(1) –2 (2) 2 (3) 1/2 (4) 4 (1) 7000 (2) 15000
205. A vessel contains 60 litres of milk, 12 litres of milk is taken (3) 9000 (4) 6000
out of it and is replaced by water. Then again from 212. The ratio of visitors from West Bengal below 20 years to
the mixture 12 litres are taken out and replaced by water. visitors from Maharashtra above 40 years in
Find the amount of milk left after the operation. (1) 1 : 3 (2) 12 : 1
(Delhi 2019) (3) 3 : 4 (4) 3 : 1
(1) 28.4 ltrs. (2) 21.6 ltrs. 213. Complete the pattern. (Rajasthan 2019)
(3) 36 ltrs. (4) 38.4 ltrs. 1
100, 50, 33 , 25, 20, ?
206. In a row of boys, A is 20th from left and B is 16th from right, 3
interchange their position, then A becomes 30th from left.
How many boys are there in the row? (Delhi 2019) 1 2 2
(1) 15 (2) 16 (3) 17 (4) 16
(1) 46 (2) 44 3 3 3
(3) 45 (4) 48 214. Which of the following Venn diagrams correctly represents
207. At what time between 8 and 9 will be hands of a clock be house, kitchen and bathroom ? (Rajasthan 2019)
together? (Delhi 2019)
7 (1) (2)
(1) 40 minutes past 8 (2) 43 minutes past 8
11

8 10
(3) 43 minutes past 8 (4) 44 minutes past 8
11 11
(3) (4)
208. Two pipes A and B can fill a tank in 12 and 15 minutes
respectivley. A third pipe C can empty it in 10 minutes. How
long will it take to fill in the tank if all pipes are opened Directions (Qs. 215-217) : Read the information given below
simultaneously? (Delhi 2019) carefully:
(1) 20 minutes (2) 30 minutes
A is the son of B. B’s sister C has a son D and a daughter E. F is
(3) 40 minutes (4) 25 minutes maternal uncle of D. (Rajasthan 2019)
Directions (Qs. 209-212): The pie chart describes the 215. How is A related to D ?
characteristics of Indian visiting UK from various states during (1) Cousin (2) Nephew
given years.
(3) Brother (4) Uncle
Answer the following questions given below: 216. How is E related to F ?
Assume that the age wise distribution data applies to all states
(1) Sister (2) Daughter
and that in the given year, 1,00,000 Indian visited UK
(Delhi 2019) (3) Niece (4) Wife
Stage - I Solved Questions M-15

217. How many nephews does F have ? 226. Choose the analogous pair.............
(1) 0 (2) 1 Fan: Sweat :: ? : ? (Chhattisgarh 2019)
(3) 2 (4) 3
(1) Rain : Drought (2) Fire : Smoke
218. A, B, C and D are playing carrom C, A and D, B are partners.
D is to the right of C, C is facing west. Then, B is facing (3) Wind : Evaporation (4) Walk : Dance
which direction? (Rajasthan 2019)
(1) North (2) South T X
227. Choose the best alternate in place of ‘?’ : 2 :: : ?
J H
(3) East (4) West
219. If it was Saturday on 17th December, 2002, then what was (Chhattisgarh 2019)
the day on 22nd December, 2004? (Rajasthan 2019)
(1) 2 (2) 4
(1) Monday (2) Sunday
(3) Friday (4) Tuesday 23
(3) 3 (4)
220. If A + B > C + D and D + A < B + C, then (Rajasthan 2019) 7
(1) D > B (2) C > D 228. Which term comes next in the series? (Chandigarh 2019)
(3) A > D (4) B > D YEB, WFD, UHG, SKI ....
221. Find the missing term. (Andhra Pradesh 2019) (1) QGL (2) TOL
16 49 (3) QOL (4) QNL
? 9 229. I am facing south, I turn right and walk 20 metres. Then I
3 8 turn right again and walk 10 metres. Then I turn left and
7 4 walk 10 metres and then turning right walk 20 metres. Then
I turn right again and walk 60 metres. In which direction am
(1) 49 (2) 64 I from the starting point? (Chandigarh 2019)
(3) 96 (4) 56 (1) North-west (2) North-east
222. A letter series is given below, in which some letters are (3) North (4) West
missing. The missing letters are given below in the proper
sequence as one of the alternatives. Find the correct 230. How many squares are there in the given figure?
alternative. (Andhra Pradesh 2019) (Gujarat 2019)
BAA__AAB__A__A__BAA__
(1) BABAB (2) AABBA
(3) ABABA (4) ABAAB
(1) 11 (2) 21
223. Choose the number group which is different from others.
(3) 24 (4) 26
(Chhattisgarh 2019)
231. If CHARACTER is written 241612376 and CHILDREN is
(1) 1, 5, 2 (2) 7, 113, 8
written 24859670 in sign language, then what will be writ-
(3) 2, 20, 4 (4) 3, 27, 4 ten for HIRALAL? (Gujarat 2019)
224. Pointing towards a lady in a photograph Ram said “She is (1) 4861551 (2) 4861515
the daughter of the only son of my grandfather. “How lady (3) 48651551 (4) 4681515
is related to Ram? (Chhattisgarh 2019) 232. If third Friday is on 16th of a month then what will be the
(1) Sister (2) Mami (Aunt) date on fourth Tuesday of same month? (Gujarat 2019)
(3) Niece (4) Cousin (1) 20 (2) 22
225. Find the missing term..... (Chhattisgarh 2019) (3) 27 (4) 29
233. A, B, C, D, E and F are sitting around a round table facing
its centre. (Karnataka 2019)
A E K (a) C is just left of E
(b) A is the only one between D and E
G K Q
(c) B is sitting opposite to E
(d) D is the only one between A and B
M Q ? Then where is F sitting?
(1) Between B and D (2) Between A and D
(1) G (2) W
(3) Between B and C (4) Between A and C
(3) X (4) V
M-16 Target NTSE
234. Six elephants P, Q, R, S, T and U are weighed. The elephant
U is heavier than R but lighter than T. Elephants P and S are
of same weight. Elephant Q is lighter than P and R is heavier (1) (2)
than S. Which one of the elephants is the heaviest?
(Karnataka 2019)
(1) Q (2) T
(3) (4)
(3) R (4) U
235. The minute hand of a clock in the horizontal plane is on the
number 6 of the clock and is pointing towards west. If the 242. In a certain code 2 × 3 = 56, 5 × 6 = 1130, 8 × 9 = 1772, then
hour hand is pointing towards southeast, what time will 4×1=? (Odisha 2019 )
the clock show exactly after three hours from now ? (1) 54 (2) 45 (3) 55 (4) 44
(Karnataka 2019)
243. There are four prime numbers written in ascending order.
(1) 1.30 hour (2) 4.30 hour The product of the first three is 1001 and that of the last
(3) 7.30 hour (4) 10.30 hour three is 2431. The last number is: (Tamil Nadu 2019)
236. The interchange of which of the given numbers/signs in
the alternatives will make the given equation meaningful ? (1) 17 (2) 19 (3) 23 (4) 13
(20 +3) × 6 + 8 – 32 = 76 (Karnataka 2019) 244. Traffic lights at three different road crossings change after
(1) 8 and 6 (2) + and – every 48 sec, 72 sec and 108 sec respectively. If they all
(3) 3 and 8 (4) × and + change simultaneously at 7:00:00 hours, then at what time
237. Find the set of figures which have similar characteristics, will they again change simultaneously? (Tamil Nadu 2019)
choosing from the given alternatives. (Karnataka 2019) (1) 7 : 14 : 00 hours
(2) 7 : 14 : 12 hours
(3) 7 : 07 : 12 hours
A B C D E F
(1) A and C (2) B and E (4) 7 : 09 : 12 hours
(3) D and A (4) F and D
1 1 1 1
238. If A × B means A is to the south of B; A+B means A is to the 245. 5 6 + 6 7 + 7 8 +... + 24 25 = ? (Tamil Nadu 2019)
north of B; A % B means A is to the east of B; A – B means
A is to the west of B; then in P% Q + R–S, S is in which (1) 0.36 (2) 0.16
direction with respect to Q? (Odisha 2019)
(3) 0.016 (4) 1.6
(1) South–west (2) South–east
(3) North–east (4) North–west 246. A lead pencil is in the shape of right circular cylinder. The
239. A clock is started at noon. By 10 minutes past 5 PM, on the pencil is 28 cm long and its radius is 3 mm. If the lead is of
same day, the hour hand has turned through: radius 1 mm, the volume of the wood used is:
(Odisha 2019) (Tamil Nadu 2019)
(1) 145º (2) 150º (1) 0.352 cm 3 (2) 7.04 cm3
(3) 155º (4) 160º (3) 3.52 cm3 (4) 70.4 cm3
240. Based on the given positions of the dice find the number of
247. Complete the given series: [Delhi 2018]
dots on the face opposite to the face with one dot.
25, 255, 2545, 25455, …
(Odisha 2019)
(1) 254545 (2) 25555
(3) 254555 (4) 255454
248. If ‘SKY WAS BLUE’ is 123 [Delhi 2018]
‘SEA IS BLUE’ is 245
(i) (ii) (iii) ‘PEOPLE SWIMMING IN SEA’ is 4678
‘PEOPLE LIKE SKY’ is 801and
(1) 2 (2) 3
‘BIRDS IN SKY’ IS 169. Then ‘PEOPLE LIKE BIRDS’ will
(3) 4 (4) 6 have the number.
241. Find out from the four alternatives as to how the pattern (1) 809
would appear when the transparent sheet is folded at the
(2) 104
dotted line. (Odisha 2019)
(3) 036
(4) 806
Stage - I Solved Questions M-17

Direction (Q. 249): In the number series given below, one number Direction (Q. 257): Select the one word-pair/number-pair/
is missing. Each series is followed by four alternatives (1), (2), letter cluster which is different from the other three alternatives.
(3), and (4). One of them is the right answer. Identify and indicate
it as per the “instructions”. [Haryana 2018]
257. (1) Light : Heavy (2) Kind : Cruel
249. 13, 74, 290, 650, ..... [Andhra Pradesh 2018]
(3) Soft : Hard (4) Vacant : Empty
(1) 1248 (2) 1370 (3) 1346 (4) 1452 258. From the given alternative words, select the word which
Directions (Qs. 250 to 251): In each of the following questions, cannot be formed using the letter of the given word:
a letter series is given, in which some letters are missing. The COMMISSIONER [Haryana 2018]
missing letters are given in the proper sequence as one of the (1) COMMON (2) MISSION
alternative. Find the correct alternative (3) MISSILE (4) SIREN
259. Which one number is wrong in the number series?
250. A_BBC_AAB_CCA_BBCC [Andhra Pradesh 2018]
13, 17, 19, 23, 27, 29 [Haryana 2018]
(1) BACB (2) ABBA (3) CABA (4) ACBA
(1) 29 (2) 27 (3) 23 (4) 19
251. BA_B_AAB_A_B
(1) AABB (2) BABB (3) BAAB (4) ABBA Directions (Qs. 260-261): In each of the questions 3 to 4 a
number series is given with one term missing shown by question
Directions (Qs 252 & 253): Some letters are given in column I mark (?). This term is one of the four alternatives given under it.
and some digits are given in column II. Each digit of column II find the right alternative.
represents only letter of column II represent any letter of column I.
Study the columns and write the alternative letter after choosing 260. 1, 3, 7, 13, 21, ?, 43, 57. [Haryana 2018]
the correct alternative against the corresponding question. (1) 31 (2) 29 (3) 30 (4) 32
261. 5, 3, 10, 8, 17, 15, ?, 24. [Haryana 2018]
[Andhra Pradesh 2018]
(1) 25 (2) 23 (3) 26 (4) 27
Column-I Column-II
ABLMS 24538 Direction (Q. 262): In this questions alternatives are alike in a
QRLBA 93526 certain way but the rest one is different. Find out the odd one
PTQAB 52601 and write correct answer.
LRNPQ 93716 [Haryana 2018]
ATRNP 29071
MSPTQ 84106 262. (1) Afghanistan (2) Kabul
QPNAR 16729 (3) Spain (4) Iraq.
RABLS 29583 263. ‘A+B’ means Ais the son of B. ‘A – B’ means A is the wife of
TSLBA 80325 B. Then what does P + R – Q mean? [Haryana 2018]
PLQST 31860 (1) Q is the father of P (2) Q is the son of P
252. The code for M is.... (3) P is the father of Q (4) R is the son of Q
(1) 0 (2) 8 (3) 1 (4) 4 Directions (Qs. 264 - 265): In each of the following questions, a
253. The code for N is .... series of number/alphabets is given which follow certain rules.
(1) 9 (2) 6 (3) 1 (4) 7 One of the number/alphabet is missing. Choose the missing
Directions (Qs. 254 - 255): There are four terms in each question. number/alphabets from the alternatives given below and mark
The term right to symbol : : have some relationship as the term it on your answer-sheet as directed.
of the left to symbol : : and out of the four, one term is missing, [Jharkhand 2018]
which is among one of the given four alternatives. Find the 264. 18, 24, 21, 27, ?, 30, 27
correct alternatives. (1) 33 (2) 30 (3) 24 (4) 21
[Andhra Pradesh 2018] 265. ELFA, GLHA, ILJA, – , MLNA
254. KMF: LLH : : RMS: ... ? ... (1) OLPA (2) KLMA
(1) TVT (2) SUS (3) LLMA (4) KLLA
(3) SLR (4) SLU Directions (Qs. 266 - 267): In these questions pairs of words/
255. Smoke: Pollution:: War: ...... letters numbers to the left/right of :: have certain relationship
(1) Victory (2) Death with each other. You are required to select the correct alternative
(3) Army (4) Enemy so that similar relationship is established to the right/left of ::
256. Arrange the following words as per order in the dictionary.
1. Dissident, 2. Dissolve, 3. Dissent, 4. Dissolute, [Jharkhand 2018]
5. Dissolution [Haryana 2018] 266. Institute : Academy :: Decree : ?
(1) 3,1,4,2,5 (2) 3,1,4,5,2 (1) Blame (2) Court
(3) 3,2,4,5,1 (4) 3,2,1,4,5 (3) Judge (4) Mandate
M-18 Target NTSE
267. 63 : 80 : : 120 : ? (1) 8 pm (2) 9 pm
(1) 125 (2) 137 (3) 10 pm (4) 11 pm
(3) 170 (4) 180 276. Five persons are standing in a line facing North. One of the
Direction (Q. 268): In each of the following questions write which two persons standing at the extreme ends is a teacher and
term in sequence replaces the questions mark? the other is a businessman. A doctor is standing to the
right of a student. A clerk is to left of the businessman. The
[Maharashtra 2018] student is standing between the teacher and the doctor.
268. BJ, DL, HP, PX, ? Counting from the left the doctor is at which place?
(1) FN (2) FX [Delhi 2018]
(3) TB (4) VD (1) I (2) III
Direction (Q. 269): Find the odd term (3) II (4) IV
277. If > stands for + [Delhi 2018]
[Maharashtra 2018]
< stands for –
269. (1) 141 (2) 101
stands for ×
(3) 107 (4) 131
270. In the following question a specific group of numbers is stands for ÷
given. From the given alternatives, find out the right number Then what is the value of 52 < 4 5> 8 2
which matches the given group. (1) 38 (2) 36 (3) 124 (4) 312
341, 572, 781 [Maharashtra 2018]
Directions (Qs. 278 & 279): Questions have become wrong due
(1) 634 (2) 891
to wrong order of signs.
(3) 909 (4) 990
271. In a certain code language ZEAL = 11, Written then in that Choose the correct order of signs from the four alternatives given
language BEAT = ? [Maharashtra 2018] under each question, so that the equations becomes right. Write
(1) 7 (2) 13 it in your answer sheet against the corresponding question
(3) 14 (4) 19 number.
278. 6 + 3 = 4 × 22 [Andhra Pradesh 2018]
Direction (Q. 272): In chart one the rows and columns are (1) × + = (2) + – × (3) = × – (4) + – =
labelled with 0 to 4 number. In chart two rows and columns are 279. 7 ÷3 = 8 -13
labelled with the numbers 5 to 9. The letter in the chart is identified (1) ÷ + = (2) × – = (3) ÷ = + (4) – + =
firstly by its row number and then by its column number. 280. If Friday falls on 15th of September 2000, what will be the
For example, 5 is denoted by 22,41 number. day on 15th of September, 2001? [Haryana 2018]
[Maharashtra 2018] (1) Thursday (2) Friday
CHART I CHART II (3) Saturday (4) Sunday
281. If ‘water’ is called ‘air’, ‘air’ is called ‘tree’, ‘tree’ is called
0 1 2 3 4 5 6 7 8 9 ‘sky’, ‘sky’ is called ‘sea’ and ‘sea’ is called ‘fire’, where do
0 F O M S R 5 A T D I P aeroplanes fly? [Haryana 2018]
(1) water (2) sky
1 S R F O M 6 I P A T D
(3) fire (4) sea
2 O M S R F 7 T D I P A 282. If ‘S’ denote ‘+’, ‘D’ denotes ‘–’, ‘Q’ denotes ‘÷’ and ‘P’
3 R F O M S 8 P A T D I denotes ‘×’, then value of following equation is:
4 M S R F O 9 D I P A T 35 Q 7 P 5 S 5 D 6= ? [Haryana 2018]
(1) 22 (2) 24
272. Which group of words represent the word MOST? (3) 26 (4) 28
[Maharashtra 2018]
(1) 40, 44, 22, 89 (2) 33, 20, 11, 79 Direction (Q. 283): Some equations are solved on the basic of a
(3) 21, 00, 03, 88 (4) 02, 13, 34, 56 certain system. On the same basis, find out the correct answer,
273. If x means –, + means÷, – means × and ÷ means + then 15–2 from amongst the four alternatives, for the unsolved equation.
÷ 900 + 90 × 100 = ? [Delhi 2018] [Haryana 2018]
(1) 190 (2) 180 (3) 90 (4) – 60 283. 85 × 14 = 44, 68 × 28 = 64, 79 × 45 = ?
274. Eight members of a group shake hand with one another (1) 72 (2) 83
once. How many hand shakes were done altogether (3) 96 (4) 124
[Delhi 2018] 284. If North direction is called East and South direction is called
(1) 64 (2) 16 (3) 28 (4) 18 West, then what will be called North-East direction?
275. A clock is set right at 5 am. The clock loses 16 minutes in 24 [Haryana 2018]
hours. What will be the right time when the clock indicates (1) Nort East (2) East-South
10 pm on the 4th day? [Delhi 2018] (3) West South (4) North-West
Stage - I Solved Questions M-19

285. In a cricket match five batsman A, B, C, D and E scored an 273. In the given arrangement of numbers after removing all
average of 36 runs. D scored 5 more than E; E scored 8 fewer even numbers which is the middle most number?
than A; B scored as many as D and E combined: and B and [Delhi 2018]
C scored 107 between them. How many runs did E score? 1859471258365927645279264123514283
[Jharkhand 2018] (1) 5 (2) 7
(1) 62 (2) 45 (3) 28 (4) 20 (3) 6 (4) 9
286. Five bells begin to toll together and toll respectively at 294. A clock is set right at 5 am. The clock loses 16 minutes in 24
intervals of 6, 5, 7, 10 and 12 seconds. How many times will hours. What will be the right time when the clock indicates
they toll together in one hour excluding the one at the start? 10 pm on the 4th day? [Delhi 2018]
[Jharkhand 2018]
(1) 8 pm (2) 9 pm
(1) 7 times (2) 8 times
(3) 10 pm (4) 11 pm
(3) 9 times (4) 11 times
295. Choose the correct alternative that represents the relation-
287. A is 3 years younger than C but one year older than D. D is
ship among illiterates, poor people and unemployed.
one year older than B but 4 years younger than C. C is 15
[Delhi 2018]
years old. What is the age of B in years?
[Jharkhand 2018]
(1) (2)
(1) 10 (2) 11 (3) 12 (4) 13
288. Kunal walks 10 kilometers towards North. From there, he
walks 6 kilometers towards south. Then h e walks (3) (4)
3 kilometers towards East. How far and in which direction is
he with reference to his starting points? Directions (Qs 296 & 297): In these questions, numbers are
[Jharkhand 2018] placed in the figures on the basis of some rules. One place is
(1) 5 km. West (2) 5 km. North-East vacant which is indicated as “?”. Find out the correct
(3) 7 km. East (4) 7 km. West alternatives to replace the question mark “?”.
289. Six teachers of a workshop have sat down for a photo session [Andhra Pradesh 2018]
as shown below. A is sitting in between K and S. M is at a
corner. There is no one sitting in between N and S. Then 296.
32 72 112
where is The person ‘T’ sitting? [Maharashtra 2018]
(1) At the central position between K and M 28 11 31 70 ? 73 115 21 108
(2) At the central position between N and A 30 74 106
(3) At the central position between S and K
(4) At the central position between M and A (1) 14 (2) 15 (3) 16 (4) 17
290. On a playground J, K, L, M, N, O, P, Q, R are sitting in one 5 13 7
row to watch a cricket match. L is at the right side of M and
297. 3 4 ?
is occupying third place from N at the right side. K is sitting
either at first or last position. Q is in between O and P, O 13 12 9 4 5 16
sitting at the third position at the left side of K. O is sitting (1) 5 (2) 4 (3) 10 (4) 6
next to ‘J’ at the right side. Who is sitting at the centre
among them? [Maharashtra 2018] Direction (Q. 298): Select the missing number from the given
(1) L (2) O responses. [Haryana 2018]
(3) J (4) Q 298.
291. Seema went 9 km to west. She turned to right and went 7 km.
She turned to left and went 8 km. From there she turned 22 30 40 67 200 416
back and went 11 km. Then she turned to right are went 7
km.
How much distance is she from origin?
2 3 ?
(Seema turns every time in 90° angle) [Maharashtra 2018]
(1) 6 (2) 4
(1) 9 km (2) 6 km
(3) 5 (4) 7
(3) 3 km (4) 7 km
299. There are two dots placed in the question figure. Find out
292. Find the missing letter : [Delhi 2018]
the answer. Figure which has the possibility of placing the
3 L 4 dots satisfying the same conditions as in the question figure?
1 Q 17 [Haryana 2018]
5 ? 4

(1) V (2) P
(3) Q (4) T
M-20 Target NTSE

303. How many students got Distinction in both Physics and


(1) (2) Maths subjects, when the students who got Distinction in
Chemistry subject is not included?
(1) 26 (2) 15
(3) 28 (4) 24
304. The four different positions of the dice are given below.
(3) (4) Which number is on the face opposite to 6 ?
[Rajasthan 2018]
300. In the given question there are two statements and they
have two conclusions I and II. You have to take the given 6 6 5 1
statements to be true even if they seem to vary to commonly
3 2 2 4 6 4 4 2
known facts. Read the conclusions and decide which of the
given conclusions logically follows from the two given (1) (2) (3) (4)
statements even disregarding commonly known facts.
Statements: (I) : All women are intelligent. (1) 1 (2) 2
(II) : Some women are educated. (3) 3 (4) 4
Conclusions: (I) : All educated women are intelligent. 305. Find the missing number (?) from the given. alternatives,
(II) : All intelligent are women. when same rule is applied in all three situations.
[Rajasthan 2018] [Rajasthan 2018]
(1) Only conclusion I is true
5 21 51
(2) Only conclusion II is true
(3) Both conclusions I and II are true 16 109 2 22 53 19 17 ? 48
(4) Neither conclusion I nor conclusion II is true. 6 15 13
301. Which of the following venn diagrams correctly represents
Bus, Car and vehicle? [Rajasthan 2018] (1) 7 (2) 25
(3) 49 (4) 129
306. Study the pattern of numbers in the following questions
(1) (2) and select the missing numbers in the place of question
mark (?) Mark the correct alternative on your answer-sheet
as directed. [Jharkhand 2018]

(3) (4) 7
286 16
302. Which of the following venn diagrams correctly represents
white colour, clothes and natural flowers?
[Rajasthan 2018] 142 34
?
(1) (2)
(1) 72 (2) 70 (3) 68 (4) 66
307. How many 7’s immediately preceded by 6 but n ot
immediately followed by 4 are there in the following series?
(3) (4) 742764 367535 784376 72406 743
[Jharkhand 2018]
Direction (Q. 303): Out of 500 students, the following venn (1) 1 (one) (2) 2 (two)
diagram represents the number of students who got distinction (3) 4 (four) (4) 6 (six)
in Physics, Chemistry and Maths subjects. Answer Q. 8 based on Direction (Q. 308): In the following square numbers are written
venn diagram. [Rajasthan 2018] with a specific rule. Find the rule and decide which alternative
will be in place of question mark. [Maharashtra 2018]

17 308. 13 61 2
Chemistry 47 42 Physics
11 ? 5 29
18 15
10 122 26
50
Maths (1) 49 (2) 58 (3) 71 (4) 102
Stage - I Solved Questions M-21

Directions (Qs. 309-311): Read the information given below 317. Who plays Hockey, Cricket and Volleyball?
and answer the following questions. (1) Madhav (2) Govind
(3) Hemant (4) Anant
Ten friends A, B, C , D, E, F, G , I, J are sitting on the opposite 318. Who does not play Baseball? Choose the correct alternative?
sides of a rectangular table, five on each side of a pair of opposite [Maharashtra 2018]
sides of the table. J and F are sitting nect to each other. B is sitting (1) Govind (2) Hemant
at middle position on one of the sides and C is sitting as far from B (3) Madhav (4) Ramesh
as B is sitting from A. A, B and C are sitting on the same side of the
table. G and I are sitting opposite to each other, D is on one of the Directions (Qs. 319-320): Select the correct alternatives which
ends. E has an equal number of persons sitting on his either side. will fit in the place of the sign of interrogation for a correct
I is sitting to the immediate right of D. [Delhi 2018] pattern. [Delhi 2018]
309. Who is sitting opposite to G?
(1) H (2) I (3) J (4) A
310. In between in which two persons I is sitting?
319. ?
(1) D – E (2) J– E
(3) B – C (4) D – B
311. In which of the following pairs, given persons cannot be 1. 2.
sitting opposite to each other?
(1) D – C (2) F– C
(3) E – B (4) G – H 3. 4.
Directions (Qs. 312 & 313): As given above questions.
A group of friends is sitting in a square facing the centre. They 320. ?
are sitting one each at the comers and one each at the midpoints
of the sides of the square. Madhvi is sitting diagonally opposite
to Uma who is to Girija’s right. Rajesh is next to Girija and opposite
1. 2.
to Gyan who is on Vinod’s left. Satish is not on Madhvi’s right
but opposite to Priya. [Haryana 2018]
312. Who is opposite to Vinod? 3. 4.
(1) Girija (2) Madhvi
(3) Priya (4) Satish
313. Who is between Gyan and Madhvi? Directions (Qs. 321 to 323): Out of the four figure (1), (2), (3),
(1) Rajesh (2) Satish (4), given in each problem, three are similar in a certain way.
(3) Vinod (4) Priya Choose the figure which is different from the other figures.

Directions (Qs. 314 - 316) : Study the following information and [Andhra Pradesh 2018]
answer the questions given below it.
AlI the six members of a family A , B, C, D, E and F are travelling
together. B is the son of C but C is not the mother of B. A and C 321.
are a married couple. E is the brother of C. D is the daughter of A.
F is the brother of B. [Jharkhand 2018] (1) (2) (3) (4)
314. How many male members are there in the family?
(1) 1 (2) 2
(3) 3 (4) 4
315. Who is the mother of ‘B’?
(1) D (2) F 322.
(3) E (4) A
316. How many children does ‘A’ have? (1) (2) (3) (4)
(1) One (2) Two
(3) Three (4) Four
Directions (Qs. 317 & 318) : Madhav and Govind play Hockey
and Volleyball, Hemant and Madhav play Hockey and Baseball. 323.
Ramesh and Govind play Cricket and Volleyball. Hemant, Ramesh
and Anant play Football and Baseball. Then, answer the
following questions. [Maharashtra 2018]
M-22 Target NTSE
324. How many triangles are there in the following figure?
[Haryana 2018] (1) (2)

(3) (4)

Direction (Q. 328) In each of the following question is there is a


(1) 20 (2) 25 specific relationship between the first and second figure. The
(3) 32 (4) 34 same relationship exists between the third and the fourth figure
325. select the correct water image of the given figure. which will replace the question mark. Select the correct term
[Rajasthan 2018] from the given alternative.
Question-figure
328. ?
A

Answer-figures [Maharashtra 2018]

(1) A (2) (1) (2)

(3) A (4)
(3) (4)
Direction (Q. 326): A Square transparent sheet with a pattern is
folded along the dotted line. Which of the following answer 329. If the figure given along side is folded to construct a cube,
figures is formed after folding the transparent sheet? find out the correct cubical figure from the given alternative
figures. [Maharashtra 2018]
[Rajasthan 2018]
326. Transparent sheet

Answer-figures
(1) (2) (1) (2)

(3) (4) (3) (4)

330. The following figure is rotated in anticlockwise direction


and its mirror image is obtained. Select the correct mirror
327. Find out the correct image in place of question marks (?)
image from four alternatives given. [Maharashtra 2018]
from the given alternatives. [Jharkhand 2018]

(1) (2)

(3) (4)
Stage - I Solved Questions M-23

Directions (Qs. 331 & 332) : Find the odd figure 337. If in a coded language the word SCHOOL is written as
UAJMQJ then PRINCIPLE will be written as
[Maharashtra 2018]
[Rajasthan 2017]
× ×= (1) RTKPEKRCN (2) NPGLAGNJB
331. (1) =* (2) * (3) RPKLEGRJG (4) RPKLEKRJG
° °
Directions (Qs. 338-339): Following alphabets are written in a
×= ×* special coded language like:
(3) * (4) =
° ° B LA C K WHITE
01 2 34 56 7 89
332. (1) (2) 338. Then code 62830 will be written as [Rajasthan 2017]
(1) HATCB (2) HATEC
(3) HATBC (4) HATCE
(3) (4)
339. 'BHICK' will be coded as [Rajasthan 2017]
333. Observe the adjoining figure and answer the following (1) 06734 (2) 67340 (3) 67430 (4) 06743
question. Choosing the correct alternative.
Directions (Qs. 340-341) : From each question has four terms.
How many isosceles trapezium are in the figure?
Three terms are alike in some way one term is different from three
[Maharashtra 2018]
others. Find out the correct term which is different from other
three. [Uttar Pradesh 2017]
340. (1) GI LP (2) D F I M
(3) BD JK (4) E G J N
341.
(1) 16 (2) 10 (3) 8 (4) 14
334. The following question figure given at left side is
incomplete. Select the correct alternative which can complete (1) (2) (3) (4)
the figure. [Maharashtra 2018]
Directions (Qs. 342-343) : Question in column I are coded in the
form of numbers. Which are written in column II, but the order of
numbers is different. Read carefully code of letters. Find out
correct answer in given alternative and write its alternative
number against the corresponding question number on your
(1) (2) answer sheet.
[Uttar Pradesh 2017]
(3) (4)
Column-I Column-II
335. Statements: CJ L 359
All crows are parrots. All parrots are doves. Some doves are EJP 092
cats. PCK 304
Conclusions: KN D 478
I Some cats are crows. NE V 721
II Some doves are crows. 342. What will be code of K N P?
Give answer. [Haryana 2017] (1) 870 (2) 327 (3) 951 (4) 470
(1) If only conclusion I follows 343. What will be code of C J E?
(2) If only conclusion II follows
(1) 123 (2) 392 (3) 724 (4) 803
(3) If both I and II follow
(4) If neither I nor II follows 344. What will be code of L J K?
(1) 270 (2) 903 (3) 594 (4) 741
Direction (Q.336) : A series of small letters is given which follow 345. What will be code of P V D?
a certain pattern. However some letters are missing from the (1) 018 (2) 372 (3) 209 (4) 743
series. You have to find out the right set of letters from alternatives
346. Find out the alternative which will replace the question (?)
that can be inserted into the blanks of the series.
mark. [Odisha 2017]
336. h_eg_fegh_eghfe_ [Haryana 2017] REASON : SFBTPO : : THINK : ?
(1) gffh (2) hhgg (1) SGHMJ (2) UIJOL
(3) ffgh (4) fhfg (3) UHNKI (4) UJKPM
M-24 Target NTSE

Direction (Q. 347): Here are some words translated from an 357. Looking into a mirror, the clock shows 9 : 30 as the time. The
artificial language. Answer the related question. [Odisha 2017] actual time is [Delhi 2017]
(1) 2 : 30 (2) 3 : 30
347. 'peslligen' means 'basketball court' (3) 4 : 30 (4) 6 : 30
'ligenstrisi' means 'courtroom' 358. If '–' stand for division, '+' for multiplication, ' ' for subtraction
'oltaganti' means 'placementtest' and '×' for addition, which one of the following equation is
Which word could mean "guest room"? correct?
(1) peslstrisi (2) vosefstrisi (1) 6 20 × 12 + 7 – 1 = 70 [Haryana 2017]
(3) gantipesl (4) oltastrisi (2) 6 + 12 – 12 7 × 1 = 62
348. Alphabet series: [Maharashtra 2017] (3) 6 – 20 12 × 7 + 1 = 57
x c w m vc x w m w m x c x w m x m (4) 6 + 20 – 12 7 + 1 = 38
In the given alphabet series how many times m is succeeded 359. If day before yesterday was Friday, what will be the third
by w and preceded by w? day after the day after tomorrow? [Haryana 2017]
(1) 1 (2) 0 (3) 2 (4) 3 (1) Thursday (2) Friday
349. If sum of squares of zeros of a quadratic polynomial g(y) = (3) Saturday (4) Sunday
y² – 6y + p is 10. What will be the value of p. [Delhi 2017]
(1) 13 (2) 12 (3) 11 (4) 10 Direction (Qs. 360): In question three alternatives are alike in a
350. How many numbers between 10 and 90 are divisible by 8 certain way but the rest one is different. Find out the odd one
completely [Delhi 2017] and write correct answer. [Rajasthan 2017]
(1) 12 (2) 10 (3) 11 (4) 8 360. (1) (10)3 (2) (100)2 10
(3) 23 × 53 (4) 2000 – 2
x 1 x 1 (2x 2 2)
351. Simplify [Delhi 2017] 361. Sailesh introduces Mahipal as the son of the only brother
x 1 x +1 2
x +1 of his father's wife. How is Mahipal related to Sailesh?
[Rajasthan 2017]
4x 2 8x 2 (1) Cousin (2) Son
(1) (2)
x4 1 x4 1 (3) Maternal uncle (4) Son-in-law

4x 2 1 Direction (Q. 362) : The questions have become wrong because


(3) 1 (4) of the wrong order of signs. Choose the correct order of signs
x4 1
from the four options given below so as to make the equations
352. Parth can row 16 km downstream and 8km upstream in 6 right. Write the alternative number of the correct option on the
hours. He can row 6 km upstream and 24 km downstream in answer sheet against the corresponding question number.
6 hours. Find the speed of Parth in still water. [Delhi 2017]
[Uttar Pradesh 2017]
(1) 5 km/hr (2) 3 km/hr
(3) 6 km/hr (4) 8 km/hr 362. 15 = 5 + 2 × 1 7
(1) × + = (2) + × =
75 5 32
353. Value of log 2 log log is [Delhi 2017] (3) + = × (4) × +=
16 9 243
363. Two person are sitting back to back. If the first person face
(1) log 3 (2) 2 log 2 is towards north. In which direction will be right hand of the
(3) log 5 (4) log 2 second person. [Uttar Pradesh 2017]
354. Which number in the following will completely divide 315 + (1) South (2) East
316 + 317 [Delhi 2017] (3) West (4) North
(1) 11 (2) 14 (3) 13 (4) 17
364. A number consists of two digits. If the digits interchange
355. Which is the smallest number? [Delhi 2017] places and the new number is added to the original number,
(1) –7 7 × 7 + 7 (2) (7 + 7 × 7) 7 – 7 then the resulting number will definitely be divisible by:
(3) 7 – 7 × 7 7 + 7 (4) 7 – (7 7 × 7 + 7)
[Odisha 2017]
356. In a certain code [Delhi 2017]
(1) 3 (2) 5
P stands for +
(3) 9 (4) 11
Q stands for –
R stands for × 365. Kiran was born on 12th September this year. Soham is 12
days younger to Kiran. In the same month Hindi day was
S stands for
celebrated on Friday. What is the birthday of Soham?
Then number corresponding to
6R8S1R3Q5P7Q4P2 is [Maharashtra 2017]
(1) 144 (2) 148 (1) Wednesday (2) Sunday
(3) 146 (4) 116 (3) Saturday (4) Monday
Stage - I Solved Questions M-25

366. A farmer travels 20 m north from his house. He then turns Direction (Q. 375) : In the following question, numbers are
east and walks 6 m, from there he again turns south and written according to same patterns and one number is missing
walks 12 m. How far is he from his original position? shown by question mark. Find the missing number that replaces
[Maharashtra 2017] the question mark. [Haryana 2017]
(1) 6 meter (2) 8 meter
(3) 10 meter (4) 14 meter 375.
367. If the first half of the English alphabet is reversed and so is 9
the 2nd half, then which letter is 7th to the right of the 12th 102 12
letter from the left side? [Delhi 2017]
(1) S (2) U (3) R (4) T 18
54
368. In a group of five persons Kamal is the tallest while Leela is ?
the shortest. Rashi is shorter than Kamal but taller Vinita
and Priti. Priti is second shortest person in the group. Who
is the third tallest? [Delhi 2017] (1) 40 (2) 48
(1) Vinita (2) Rashi (3) 30 (4) 24
376. Amit said, this girl is the wife of the grandson of my mother.
(3) Priti (4) Leela
How is Amit related to the girl? [Haryana 2017]
369. 15 6 6 7 50 10
[Delhi 2017] (1) Father (2) Grand Father
(3) Husband (4) Father-in-law
36 21 ?
Directions (Qs. 377-378): In each of the questions 31-32 a
4 5 10 number series is given with one term missing shown by question
(1) 140 (2) 220 mark (?). This term is one of the alternatives among the four
(3) 500 (4) 320 numbers given under it. Find the right alternative.
370. If in a certain language ‘how can you go’ is written as ‘je de 377. 121, 144, 169, ?, 225, 256. [Rajasthan 2017]
ke pe’, ‘you come here’ is written as ‘ne ke se’ and ‘come (1) 196 (2) 296
and go’ as ‘re pe se’, then how will ‘here’ be written in the (3) 220 (4) 222
language? [Delhi 2017] 378. 5, 10, 20, ?, 80 [Rajasthan 2017]
(1) je (2) pe (1) 35 (2) 40
(3) ne (4) ke (3) 45 (4) 50
379. Which of the following Venn diagrams correctly represents
Direction (Qs.371-372) : Find the missing number/numbers in female, mother and doctor? [Rajasthan 2017]
the series.
371. 2, 65, 7, 59, 12, 53, ?, ? [Haryana 2017]
(1) 15, 42 (2) 17, 45 (1) (2)
(3) 17, 47 (4) 18, 48
372. If [Haryana 2017]
54 + 43 = 2
60 + 51 = 10 (3) (4)
Then 62 + 72 = ?
380. How many 5s are there in the following sequence which are
(1) 30 (2) 18 (3) 20 (4) 9
immediately followed by 3 but not immediately preceded
373. At 12 : 30 the hour hand of a clock faces north and the
by 7? [Rajasthan 2016]
minute hand faces south. At 2 : 45 the minutes hand will be
43657536457357353
in which direction? [Haryana 2017]
(1) 0 (2) 1
(1) North-West (2) West (3) 2 (4) 3
(3) South-East (4) East
374. Find the missing number: [Haryana 2017] Directions (Qs. 381-382) : Questions are based on number/figure
series. In each series missing term is mentioned by question mark
20160 4 (?). Find out the missing term in given alternatives.
? 4 [Uttar Pradesh 2017]
381. 16, 19, 28, 43, 64, ?
480 8 (1) 91 (2) 86
96 24
(3) 97 (4) 76
382. 63, 58, 51, 40, 27, ?
(1) 860 (2) 1140 (1) 8 (2) 12 (3) 10 (4) 14
(3) 2880 (4) 3240
M-26 Target NTSE
383. 3, 4, 8, 17, 33, 58, ? Direction (Q. 391) : Using alphabet A to Z in sections of figure I
(1) 76 (2) 94 (3) 84 (4) 98 and II a code has been created. First letter in every section is
Direction (Q. 384) : In question, number are placed in figure on coded according to its shape and the second letter is coded
the basis of some rules. One place is vacant which is indicated as using a dot.
(?). Find out the correct alternative for the vacant place and
write its number against the proper question number on your eg: A is coded as ; M is coded as ;
answer sheet. [Uttar Pradesh 2017] K is coded as ; P is coded as .
Figure I Figure II
384.
64 36 ?
AM NF LO JC
6 5 7
BU TV DG KP HS
48 30 35
EW IZ XY RQ
(1) 36 (2) 25 (3) 49 (4) 18 391. What will be the code of EUKAR? [Maharashtra 2017]
Direction (Q. 385): Find out the wrong number in the given series.
(1)
385. 582, 605, 588, 611, 634, 617, 600 [Odisha 2017]
(1) 634 (2) 611 (3) 605 (4) 600
(2)
Direction (Q. 386): Complete the series.
386. C, Z, F, X, I, V, L, T, O, ? ? [Odisha 2017] (3)
(1) O,P (2) P, Q (3) R, R (4) S, R
387. Find the number of squares/rectangles in the given figure.
(4)

Direction (Q. 392) : Direction of the following question there is


[Odisha 2017] a specific relationship between the first and second term. The
same relationship exists between the third and the fourth term.
Finding the relationship the correct term from the alternative
given.
(1) 20 (2) 18 (3) 16 (4) 15
[Maharashtra 2017]
Direction (Q. 388): In the following question a specific group is 392. 4 : 48 : : ? : 100
given. From the given alternatives, find out the right term which (1) 7 (2) 5 (3) 6 (4) 10
matches the given group.
Direction (Q. 393): Read the following information carefully
388. 37, 46, 28 [Maharashtra 2017]
and answer the question given below:
(1) 56 (2) 55 (3) 54 (4) 83
M, P, J, B, R, T and F are sitting around a circle the centre. B is the
Direction (Q. 389) : Write which number in sequence replaces third to the left of J who is second to the left of M. P is third to the
the question mark. left of B and second to the right of R. T is not an immediate
neighbour of M.
1 1 2 2
389. 11 ,12 ,14 ,16 ,? [Maharashtra 2017] 393. Who is fourth to the right of M? [Delhi 2017]
9 2 7 3 (1) B (2) T
1 1 1 (3) R (4) M
(1) 8 (2) 19 (3) 20 (4) 22
3 2 3 Directions (Qs. 394-396): Read the given information carefully
to answer the question.
Direction (Q. 390): Find the correct alternative which will
replace the question mark. A, B, C, D, E, F and G are sitting in a row facing North. F is to the
immediate right of E. E is 4th to the right of G. C is the neighbour of
390. B and D. Person who is third to the left of D is at one of the ends.
21 5 28 13 16 2 394. Who are to the left of C? [Odisha 2017]
24 30 ? (1) Only B (2) G, B and D
17 7 25 7 10 8 (3) G and B (4) D, E,F and A
395. Who are the neighbours of B? [Odisha 2017]
[Maharashtra 2017] (1) C and D (2) C and G
(1) 24 (2) 30 (3) 36 (4) 40 (3) G and F (4) C and E
Stage - I Solved Questions M-27

396. Which of the following statements is not true? 402. Questions Figure [Rajasthan 2017]

720
[Odisha 2017]
(1) E is to the immediate left of D
(2) A is at one of the ends
Answer Figures
(3) G is to the immediate left of B
(4) F is second to the right of D
(1) (2) (3) (4)
Directions (Qs. 397) : Answer the following questions based on
the information given below: Directions (Q. 403) : In the following question, figures showing
(1) Bhanudas, Gopal, Amar, Akhil and Chaitanya each practice a sequence of folding and cutting a paper are given. Which
one of the professions. Farmer, Lawyer, Doctor, Teacher, could resemble the figure in the Answer figure?
Photographer. (2) Akhil who is richer than Amar and Chaitanya, 403. Problem Figures [Rajasthan 2017]
is a lawyer. (3) Farmer is the richest of all. (4) Occupation of the
poorest is photography. (5) Amar is a doctor and Gopal is a
photographer. (6) Akhil is more richer than Amar but less richer
than Bhanudas. Then: [Maharashtra 2017]
397. What is the occupation of Chaitanya? Answer Figures
(1) Lawyer (2) Doctor
(3) Farmer (4) Teacher
398. Who among the following is farmer?
(1) Akhil (2) Bhanudas
(3) Gopal (4) Chaitanya
(1) (2) (3) (4)
399. A rhythmic arrangement of alphabets is given. The missing
letter alphabets appear in the same order as in one of 404. Look at the sequence of symbols to find the pattern. Select
—bcdbc—dcabd—bcdbc—dc—b. correct pattern at the (?) mark.
(1) aaaaa (2) bbbbb
(3) ccccc (4) ddddd
?
400. Which one of the answer figure would occupy the next
position in the problem figure, if they continue in the same
order? [Haryana 2017]
Problem Figures
(1) (2) (3) (4)
X
X X ? [Odisha 2017]
(1) 1 (2) 2 (3) 3 (4) 4
Answer Figures 405. Which letter represents the question mark? [Odisha 2017]

D G
X X W T
X X NM Q
J
(1) (2) (3) (4)
C L
X ?
401. The two positions on a dice are shown below. If 1 is at the I B
bottom then what will be on the top? [Rajasthan 2017] R Y

(1) M (2) O (3) Q (4) R


406. Which number replaces the question mark? [Odisha 2017]

4 13 3 21
(1) 2 (2 ) 3 12 7 5 2
9 9 6 8 5 5 12 ?
(3) 4 (4) 5
3 12 7 16
Direction (Q. 402) : In question find the correct water image of
the given figure. (1) 4 (2) 6 (3) 8 (4) 12
M-28 Target NTSE

Direction (Q. 407): Select a figure form amongst the Answer 410. The following figure is folded to form a cube. Observe the
figures which will continue the same series as established by the cube. Of the following cube figure find the most appropriate
five problem figures. figure. [Maharashtra 2017]

407. Problem Figures [Odisha 2017]

(A) (B) (C) (D) (E)

Direction (Q. 408) : In the following question there is a specific


relationship between the first and second figure. The same (1) (2)
relationship exists between the third and fourth figure which
will replace the question mark. Select the correct term from the
alternatives given. [Maharashtra 2017] (3) (4)

408. Direction (Q. 411) : In the following question figures change in


? a particular order. Find out the correct figure from the alternative
which will replace the question mark?

(1) (2) 411. ?

[Maharashtra 2017]

(3) (4)
(1) (2)
409. A square piece of paper is folded and cut at specific spot as
shown in the figure. The paper when unfolded will look like
as shown in one of the alternatives. Select the correct (3) (4)
alternative. [Maharashtra 2017]
Directions (Qs. 412 & 413) : In each of the questions 1 and 2,
there are four items, three of which are alike by some means or
other while one is out of the class. Find out the odd item and
indicate your answer by filling the circle of the corresponding
letter on the O.M.R. Answer-Sheet.
412. (1) Flourine (2) Bromine
(3) Chromium (4) Chlorine
[West Bengal 2016]
(1) (2) 413. (1) Titan (2) Eris
(3) Uranus (4) Haumea
[West Bengal 2016]
Directions (Qs. 414 – 416) : In each question below there are
two words separated by ‘:’ in the upper row. Below that there are
(3) (4) some words on each side of the symbol ‘:’. Find the relation
between two upper words and select one word from the right
side of ‘:’ below which have the same relation as above. Fill the
circle of the letter denoting your selected answer on the O.M.R.
Answer-Sheet.
Stage - I Solved Questions M-29

414. Octopus : Mollusca 422. Which number denotes the poets who are essay writer,
Spider : ? [West Bengal 2016] dramatician and writer also ? [Jharkhand 2016]
(1) Arthropoda (2) Clean (1) 7 (2) 5 (3) 6 (4) 8
(3) Porifera (4) Genus 423. Which numbers only denotes writers who are not poets
neither dramatician nor essay writer? [Jharkhand 2016]
415. Pituitary : Brain
(1) 2 & 3 (2) 1 & 3 (3) 4 & 5 (4) 8 & 6
Thymus : ? [West Bengal 2016] 424. Which numbers only denotes, poets who are not writers
(1) Throat (2) Chest neither essay writer nor dramatician? [Jharkhand 2016]
(3) Spinal Cord (4) Larynx (1) 2 & 4 (2) 8 & 3 (3) 7 & 9 (4) 5 & 1
416. Eye : Myopia Direction (Q. 425) : In the series given below what will come in
Teeth : ? [West Bengal 2016] place of question mark (?)
(1) Cataract (2) Trachoma
425. AEB, FJG, LPM, ? [Jharkhand 2016]
(3) Pyorrhoea (4) Eczema (1) NOP (2) SWT
417. M, N, P, R and T, each has got different marks in an exam. R (3) STW (4) MNO
has got more marks than M and T. N has got less marks
than P. Who has got the highest marks in third rank ? Direction (Q. 426) : Out of nine cells of a square one cell is left
blank and in the rest of the cells numbers are written which
(1) N (2) R [Jharkhand 2016]
follow same rules. Get the rule and find out the proper option for
(3) M (4) Data in-sufficient the blank cell (?)
418. Age of A is three times the age of B. Four years earlier age
of C was two times the age of A. After four year A will 426. 9 7 8 [Jharkhand 2016]
become 31 years old. What is the present age of B and C? 3 6 7
[Jharkhand 2016] 3 7 ?
(1) 10, 50 (2) 10, 45 (3) 9, 50 (4) 9, 45 30 55 31
419. The sum of present age of a father and son is 70 years.
After 10 years the age of son will be half the age of his (1) 25 (2) 3 (3) 4 (4) 9
father. What is their present age ? [Jharkhand 2016]
Directions (Qs. 427-428) : In each of questions 6 to 7 below,
(1) 45 years, 25 years (2) 50 years, 20 years there is a number series with one term missing shown by ‘?’. The
(3) 47 years, 23 years (4) 50 years, 35 years term is given as one of the alternatives among four numbers
420. If ' ' means '–', '+' means ' ', '–' means ' ' and ' ' mean '+', given below it. Find the term and indicate your answer by filling
then. [Jharkhand 2016]
the circle of the corresponding letter of alternatives in the O.M.R.
15 – 12 900 + 90 × 100 = ?
Answer-Sheet.
3
(1) 180 (2) 190 (3) (4) 90 427. 4, 13, 41, 126, ?, 1151 [West Bengal 2016]
1600
(1) 382 (2) 582 (3) 882 (4) 1082
421. Choose the correct arrangement of mathematical signs at
the place of * for equating given equation. 428. 3, 15, 35, 63, ?, 143 [West Bengal 2016]
[Jharkhand 2016] (1) 69 (2) 79 (3) 89 (4) 99
7*2*3 *5*6
Directions (Qs. 429 – 430) : In questions, numbers are placed in
(1) –, +, =, × (2) +, ×, =, +
figures on the basis of some rules. One place in the figure is
(3) =, ×, +, – (4) ×, =, +, +
indicated by the interrogation sign (?). Find out the correct
Directions (Qs. 422-424) : In the diagram given below big alternative to replace the question mark and indicate your
triangle represents writer, rectangle represents poet, small answer by filling the circle of the corresponding letter of
triangle represents dramatician and circle represents essay alternatives in the O.M.R. Answer-Sheet.
writer. Study the diagram and choose the correct answers of the 429.
given questions.
9 13 8 14 7 16
1
8
2 4
121 118 ?
67 5
3 9
10 [West Bengal 2016]
(1) 123 (2) 119 (3) 118 (4) 121
M-30 Target NTSE
430.
435. C C C
12 8 16 7 25 21
C C
A B C D E

80 207 ?
C C C C
[West Bengal 2016] (1) (2) (3) (4)
(1) 184 (2) 210
(3) 241 (4) 425 [Jharkhand 2016]
431.
Directions (Qs. 436 - 437) : In each of the following questions
there is a specific relationship between the first and. second
5 8 9 term. The same relationship exists between the third and fourth
3 7 11 4 8 10 term which will replace the question mark (?). Select the correct
306 372 ? term from the alternatives given :
9 4 6 5 12 2 436. 531 : 99 :: ? : ? [Maharashtra 2015]
6 3 5 (1) 451 : 55 (2) 321 : 44
(3) 642 : 66 (4) 212 : 11
[West Bengal 2016] 437. ACG : ZXT : : HJN : ? [Maharashtra 2015]
(1) 442 (2) 616 (1) SQM (2) TRN
(3) 576 (4) 324 (3) SQN (4) SOM
Directions (Qs. 432-433) : In every question a dice has been Direction (Q. 438) : Find the odd term out. [Maharashtra 2015]
shown in different faces on which numbers/symbol/colour have 438. (1) 336 (2) 508
been written randomly. Carefully study the faces and answer the
(3) 504 (4) 720
question based on it.
432. Which symbol is just opposite to symbol ' '? Directions (Qs. 439 - 440) : Observe the following pyramid of
alphabets and answer the following questions.
– a
0 + × 0 + + 0 b c
f e d
g h i J
(1) (2) + [Jharkhand 2016] o n m l k
(3) 0 (4) p q r s t u
433. Which number will come at the bottom of last cube ? b a z y x vw
c d e f g h i j
2 3 4
439. onza, nmyz, mlxy, ? [Maharashtra 2015]
4 5 4 2 1 5 (1) nmst (2) hilm
(3) abcd (4) lkwx
440. aqze : wtxh : : ghno : ? [Maharashtra 2015]
(1) 3 (2) 4 [Jharkhand 2016] (1) abce (2) qrfe
(3) 6 (4) 1 (3) jilk (4) ijlh
Direction (Q. 434) : Choose the correct water image of the given Directions (Qs. 441- 442): In the given questions there are four
word numbers from amongst the alternatives. groups of numbers / pairs of numbers / group of letters of which
434. PR9YA [Jharkhand 2016] three are alike and one is different. Find the one which is different.
(1) RPY9A (2) [Karnataka 2015]
(3) (4) 441. (1) GAINFUL (2) S E C T A R Y
Direction (Q. 435) : In each of the following questions, find the (3) FO UN DE R (4) Q U A D R I C
figure from the answer-set (i.e., 1, 2, 3 and 4) which will continue 442. (1) 686,21
the series given in the problem set (i.e. A, B, C, D and E). (2) 1024,24
(3) 2000,30
(4) 2662,30
Stage - I Solved Questions M-31

Directions (Qs. 443 - 444): Complete the given analogy by (3) 18 > 54 27 9 45 9
selecting the correct answers from the alternatives.
(4) 18 >54 27 9 45 9
443. 147 : 49 : : ? : ? [Karnataka 2015]
(1) 186 : 66 (2) 189 : 33 452. Which one of the following equations is NOT meaningful
(3) 198 : 66 (4) 201 : 33 by substituting the set of given mathematical signs
444. 195 : 15 : : ? : 25 [Karnataka 2015] (÷, + =, ×) sequentially in the given equations?
(1) 575 (2) 600 [Karnataka 2015]
(3) 625 (4) 650 (1) 80 ÷ 5 + 4 = 5 × 4
445. TPLNR : BXTVZ : : NJFHL : ? [Karnataka 2015] (2) 30 ÷ 2 + 5 = 2 × 10
(1) VNRTP (2) VRNPT (3) 40 ÷ 10 + 4 = 10× 6
(3) URMPT (4) RVPTN
(4) 60 ÷ 3 + 10 = 6 × 5
446. ‘Environment Day’ falls on Thursday, then on which day
‘Children’s Day’ will fall in the same year ? Direction (Q. 453) : There are five members in a family. The
[Maharashtra 2015] comparison of their ages is as follows:
(1) Friday (2) Saturday I. Anju’s age is twice the age of Rama, but is half the age of
(3) Sunday (4) Monday Harsha.
447. Yash starts moving towards South and walks for 12 m. Then II. Balu’s age is half the age of Rama, but is twice the age of
he takes the left turn and walks for 8 m. Again he takes a Mala.
right turn and walks for 24 m. Now he takes a turn to North-
east direction and walks 25 m. From that he takes a turn 453. Which one of the following pair represents the oldest and
towards North and walks 20 m. Find the distance between the youngest members in the family?
the start point and the finishing point of his movement. [Karnataka 2015]
[Maharashtra 2015] (1) Anju and Mala
(1) 12 m (2) 15 m (2) Harsha and Balu
(3) 17 m (4) 25 m (3) Balu and Rama
Directions (Qs. 448 – 449) : Six friends A, B, C, D, E, F are sitting (4) Harsha and Mala
around a round table. F is sitting in between A and D. C is Direction (Q. 454) :
sitting in between E and B. E is not sitting between D and C. D is
at second position to the left of C. Answer the following questions. 454. In a code A = 26 ....... Z = 1 if G = 25 and MILK = 83 then, find
the code for WATER. [Karnataka 2015]
448. Who is sitting in between A and C ? [Maharashtra 2015]
(1) 67 (2) 68
(l) B (2) E
(3) 92 (4) 93
(3) D (4) Cannot be said
449. What is the position of D ? [Maharashtra 2015] Direction (Q. 455) : Question below is based on three statements
(1) Second position to the right of A I, II and III. Decide whether the data in the statements is sifficient
(2) The next to the right of B to find the answer to the given question. The comparison of
(3) Third position to the left of B heights of 4 persons A, B, C and D are as follows:
(4) Fourth position to the right of A Statements:
450. In a certain code language if 3+3 = 9, 6 – 3 = 2, 4 × 3 = 1, I. A is taller than B
3 ÷ 2 = 5, then find 24 × 6 ÷ 8 – 2 + 3 = ?
II. A is shorter than C
[Maharashtra 2015]
(1) 33 (2) 30 III. C is taller than D
(3) 28 (4) 06 455. To find whether B is taller than D, [Karnataka 2015]
Direction (Q. 451): (1) Data in statement I is sufficient
(2) Data in statement II is sufficient
451. If > stands for +
< stands for – (3) Data in statements I, II and III are sufficient
stands for × (4) Data in all the statements I, II and III are not sufficient
> stands for = and stand for Direction (Q. 456) : Write which number or alphabet in sequence
Then which one of the following equations given below is replaces the question mark (?).
correct. [Karnataka 2015]
456. 3, 14, 54, 159, 314, ? [Maharashtra 2015]
(1) 18 < 54 27 > 9 45 > 9 (1) 309 (2) 372
(2) 18 > 54 27 9 45 < 9 (3) 391 (4) 398
M-32 Target NTSE

Directions (Qs. 457-458) : In the following Venn Diagram the Directions (Qs. 462-463): Take the given statements as true and
number of students who likes Science, Mathematics and Language decide which of the conclusions logically follow from the
is given. Observe the figure and answer the following questions: statements.
462. Statements: [Karnataka 2015]
Mathematics 1. All horses are dogs.
Science
9 8 2. All dogs are cats
5
Conclusions:
I. All horses are cats
11
II. All cats are horses.
4 3 (1) Only conclusion I follows
(2) Only conclusion II follows
(3) Both conclusions I and II follow
2 (4) Neither conclusion I nor II follows
Language 463. Statements: [Karnataka 2015]
1. Some apples are mangoes.
457. Find the number of students who likes Mathematics but 2. All mangoes are bananas
not Language. [Maharashtra 2015] Conclusions:
(1) 15 (2) 14 I. Some bananas are apples
(3) 11 (4) 10 II. All mangoes are apples
458. How many students, who like Science and Mathematics do III. Some bananas are mangoes
not like Language? [Maharashtra 2015] IV. All apples are bananas
(1) 4 (2) 8 (1) Only conclusion II follows
(3) 11 (4) 5 (2) Conclusion II and IV follow
(3) Conclusions I and III follow
Directions (Qs. 459-460) : Find the correct alternative which
(4) Conclusion I, II and III follow
will replace the question’ mark (?).
459. Direction (Q. 464) : When the given Problem figure is folded as
a cube identify which one of the cubes with faces shown below is
19 62 38 possible.
464. [Karnataka 2015]
31 11 17 27 13 37 41 ? 72

54 43 74
[Maharashtra 2015]
(1) 12 (2) 14 (3) 15 (4) 16
460. In a certain code language if BUSH is written as 58, MARK (1) (2)
is written as 65, then in the same code how will you write
the word PERL? [Maharashtra 2015]
(1) 48 (2) 51
(3) (4)
(3) 54 (4) 57
Direction (Q. 461) : Find the missing number in the given
Directions (Qs. 465-466) : The information about the students
matrices.
wearing T-shirts having numbers 7, 8, 9 and 10 is given below.
461. [Karnataka 2015] Read the given information and answer the following
questions.
7 9 4 (i) The players having number 7 and 9 play Football as well as
4 3 ? Cricket.
(ii) The fat player having number 8 participates in all the games
6 8 9 except Kho-Kho and Football.
84 108 126 (iii) The player carrying number 9 is very short. He is expert in
Kabaddi. The player having number 10 is very slim. He
(1) 12 (2) 9 participates in all the games except Football.
(3) 8 (4) 7 (iv) The player having number 7 is very tall.
Stage - I Solved Questions M-33

465. Which game can be played by one and only one player ? 469. Find the number of equilateral triangles in the given figure.
[Maharashtra 2015] [Maharashtra 2015]
(1) Kabaddi (2) Kho-Kho (1) 32 (2) 18
(3) Cricket (4) Football (3) 5 (4) 0
466. Name the game, which is played by the slim player but 470. Find the number of parallelograms which are not rectangles
cannot be played by the fat player ?[Maharashtra 2015] from the given figure. [Maharashtra 2015]
(1) Cricket (2) Football (1) 4 (2) 6
(3) Kho-Kho (4) Kabaddi (3) 8 (4) 10
467. Directions- Choose the water image of the ‘Question Figure’ 471. A square piece of paper is folded and cut at specific spots
from the given alternatives. [Maharashtra 2015] as shown in the figure. The paper when unfolded will look
Question Figure: as shown in one of the alternatives. Select the correct
alternative. [Maharashtra 2015]
Question figure:

Answer Figures:

(1) (2) Answer Figures:

(3) (4)
(1) (2) (3) (4)

468. Select a suitable figure from the answer figure that would Direction (Q. 472) : Complete the following number/letter/
replace the question mark (?) [Maharashtra 2015] figural series by selecting from the given choices.
Question Figure: 472. [Karnataka 2015]

+
× + ?
×
Answer Figures:

×
×+
(1)
+
(2) (3) (4)
?
Directions (Qs. 469-470) :- Observe the following figure and
answer the following questions by choosing the correct
alternative given below.
(1) (2)

(3) (4)
M-34 Target NTSE

Direction (Q. 473) : Identify the number of specified geometric Direction (Q. 475) : Choose the figure from the alternatives which
shapes in the given diagram and mark the correct answer. is suitable to put the dot ( ) as found in the problem figure.
473. How many triangles are in the given figure? 475. Problem figure [Karnataka 2015]
[Karnataka 2015]

(1) (2)
(1) 21 (2) 22
(3) 23 (4) 24
(3) (4)
Direction (Qs. 474) : Find the correct mirror images for the
following problem figures choosing from the alternatives.
474. Problem figures [Karnataka 2015] Direction (Q. 476): Choose which one of the figures shown in the
alternatives will be formed by joining the given parts of the figure.
476. [Karnataka 2015]

(1) (2)

(1) (2)

(3) (4)

(3) (4)
Hints & Solutions M-35

HINTS & SOLUTIONS


14. (4) W A R D
1. (3) Evolution is proposed by Darwin, Buoyancy is
proposed by Archimedes. 13 04 40 75
2. (4) Snake bite with Fang, Bee bite with sting. 15. (4) KP
3. (2) Tea is taken in cup, Tobacco enhanced through B + 2 = D + 3 = G + 4 = K.
Hookah. Y – 2 = W – 3 = T – 4 = P.
4. (4) Aeroplane is controlled by cockpit, train is controlled 16. (2) KLML
by engine. LKMN, LKMN
5. (2) Assign numbers to alphabets and add them. 17. (2) WORK STATE
(NEW JERSEY = 14 + 5 + 23 10 + 5 + 18 + 19 + 5 + 25 2 35 7 14649
= 124) STORE = 14359
6. (2) Replace each alphabet in BOMBAY with the next 18. (1) BALL JACK
alphabet. 2 × 1 × 12 × 12 10 × 1 × 3 × 11
M A D R A S Option (1) - 330
+1 +1 +1 +1 +1 +1 –5
–5
N B E S B T
19. (3) C D H I M N R S
B O M B A Y +5 +5
+1 +1 +1 +1 +1 +1 +5 +5
20. (4) B (2) × M (13) = 26
C P N C B Z
E (5) × N (14) = 70
+1 +2 +3 +4
H (8) × O (15) = 120
7. (2) 3 4 6 9 13 K (11) × P (16) = 176
7 11 19 31 47 N (14) × Q (17) = 238
+4 +8 +12 +16 21. (4) GECA, ZXUT, SQOM
–2, –2, –2, –2 in each term
×5 ×5
So, option 4 satisfies the condition.
22. (1) BEIN, EHLQ, ILPU
8. (3) 625, 5, 125, 25, 25, 125, 5
+3, +4, +5, +6 in each term.
÷5 ÷5 ÷5 So, option (1) satisfy given condition.
9. (4) 4, 8, 28, 80, 244, ____ T=4
31 + 1 = 4 +1 +1
32 – 1 = 8
33 + 1 = 28 23. (2) 416, 525 749 864
2 2 2 2
34 – 1 = 80 (4) (5) (7) (8)
35 + 1 = 244 24. (1)
36 – 1 = 728 25. (2) R V A G N V
10. (2) A J S, G P Y, M V E, S B K, Y H Q +4 +5 +6 +7 +8
26. (4) The first two letters of a term are shifted to the last to
1 7 13 19 25 form the next term of the series.
+6 +6 +6 +6 So after, DNINETEENCOVI, the next term will be
10 16 22 2 8 INETEENCOVIDN.
+6 +6
=V 27. (3) The pattern is,
19 25 31 4×1+1=5
+6 +6
= E = 31 – 26 = 5 5 × 2 + 2 = 12
11. (2) H I J K L M N O P 12 × 3 + 3 = 39
12. (3) Fourth to left of I is (E) and 16th to the right of E is (U). 39 × 4 + 4 = 160
160 × 5 + 5 = 805
13. (3) P A L M
28. (4) The sequence is based on the growth of a crop from a seed.
34 42 86 24 The sequence will be : Seed, Root, Stem, Flower, Crop
i.e. 4, 2, 3, 5, 1.
M-36 Target NTSE
29. (3) All except Guru Granth Sahib are coming among the
first ten Sikh Gurus. 10 25 121
30. (1) All are perfect cubes except 289.
31. (4) All second word is the part of the first word except 10 25 11
Moon-Bird.
32. (2) ‘n’ repeated the most. 10 36 10 6 16 4.
33. (3) Three times. 24 7
ijo opu apu 45. (4) tan , cos
7 25
34. (3) onp akt etb
14 tan 75cos 7 sec
Three times.
35. (2) ccba | ccba | ccba | ccba 24 7 25
14 75 7
36. (2) Pie is circular representation of data, while rest are 7 25 7
rectangular / tabular format. = 48 – 21 – 25 = 2.
37. (4) All other are virus infections, except Typhoid. 46. (2) Horizontal and vertical distance are same.
38. (1) The words in each pair are antonyms.
39. (2) ‘Shirt’ belong to ‘apparel’, ‘necklase’ belongs to perpendicular
tan 1 45
‘jewellery’. base
40. (1) (c) Colour (e) Create
47. (3) 18, 50, 98, .......
(a) Credential (d) Credible
(b) Creed 3 2, 5 2, 7 2
41. (3) an 3n 4
9 2 162
Put n = 1, 1, 3 [series will be 7, 10, 13, ....]
48. (4) ap q m, a p q n
a 7, d 3, n 12
a ( p q 1)d m, a ( p q 1)d n
n
s12 [2a (n 1) d ] Add both equations
2
2a 2( p 1)d m n
12
[2(7) (12 1)3] m n
2 pth term a ( p 1)d
= 6 (14 + 33) = 282. 2
49. (2) Students who passed the examination
2 1 = (16 + 29) – 1 = 44.
42. (1) x 2 98 2
x2 So, total number of students in the class
= 44 + 6 + 5 = 55.
1
x 10 50. (2) (3 15 19) 8 6 2
x
3
51. (4) (252 9) 5 32 92 200.
3 1 1 1 52. (1) Student who don’t drink milk = 52 – 24 = 28
x x 3 x
x3 x x Student who don’t drink Tea = 52 – 28 = 24
Student who don’t drink anything = 8
103 3 10 1000 30 970.
43. (1) 40 – 10 + 5 ÷ 4 × 5 = 21 Don't drink anything
After changing signs
40 ÷ 10 × 5 – 4 + 5 = 21
20 4 5 21
20 8 16
21 21.

44. (4) 10 25 108 154 225 Not drink Milk Not drink Tea
No. of students who drink both Milk and Tea
10 25 108 154 15 = 52 – (20 + 8 + 16)
= 52 – 44 = 8.
53. (2) 10R192P48S48P96Q1
10 25 108 169
10 192 48 48 96 1
10 4 0.5 1 10 2.0 1 11 2 9.
10 25 108 13
Hints & Solutions M-37

54. (2) 45 ÷ 9 × 4 + 5 = 5 × 4 + 5 = 20 + 5 = 25.


46 x 20 x 84 x 64 x 216 x
55. (4) Sister 4 4 4 4
860
10 10 10 10 104
I 430 x
860
104
F 860
x 10 4 2 10 4 x 20000.
430

MK M 1 1 1
62. (3)
a b 9
N
56. (2) 1 1
10 a 36
2
6 1 1 1
S b 9 36

20th 30th 1 3 1
57. (2) Left k m
Right
36 36 12
m k b can complete in 12 days.
19 35th
34 63. (1) Let the number be x.
So total man = 19 + 1 + 34 = 54. x x 2x
5
58. (4) Clearly from option if it is gain 6 hrs then it covers 30 3 2 3
min. So at 6:00 o’clock incorrect watch shows 6:30 pm.
59. (2) Hindi books = 14% of 10000 = 1400 5x 2 x 15
2 3 5x 4 x 30 x 30.
1400
6
6% are Bad = 6% of 1400 6 84.
100 2 2
So, of x 306 12.
60. (4) Total no. of books = x 5 5
23 x 64. (4) CP of 72 oranges = `. 600
23% of x (no. of Kannada books) SP of 72 oranges = 11 × 50 = `. 550
100
SP of remaining (61) oranges = `. 236
Bad condition 2% of 23x = 92
Total SP of oranges = 236 + 550 = `. 786
23x Profit = SP – CP = 786 – 600 = `.186.
2 92
104 65. (4) As 50% is 450 gm so 40% will be 360 gm + 150 = 510.
66. (2) Increase by 4% as
92 104 40000 Let L = 10, B = 10 Area = 100
x 20000.
23 2 2 4%
20 New L = 13, B = 8 Area = 104
No. of English books 20000 4000.
100 67. (1) Ravi’s age = x
61. (1) Let the total no. of books be x. Raju’s age = y
Then x 4
2% of (23% of x ) 1% of (20% of x) y 3
Kannada English
x 6 26 x 20
6% of (14% of x ) 4% of (16% of x )
20 4 20 3
Hindi Telugu y 15 years
y 3 4
8% of (27% of x)
= 860
Tamil
68. (4) 20 m
2 23 1 20 6 14
x x x
100 100 100 100 100 100 25 m
30 m
4 16 8 27
x x 860
100 100 100 100 25 m
M-38 Target NTSE
69. (3) 8 + (8 – 2) = 70
2

76. (3) D 6 E 6
92 + (9 – 3) = 87 C
x
102 + (10 – 4) = 106 10
72 + (7 – 5) = 51 O
70. (4) When a + b = 2020 10 x
2020 is an even number. So, if a is even, b is also even A 6 F 6 B
(to make sum even). Similarly, if a is odd, b is also odd
to make the sum even.
When a and b are even, (–1)a + (–1)b = 2 x = OE 10 2 62 = 8 cm = OF
When a and b are odd, (–1)a + (–1)b = 2 EF = EO + OF = 8 + 8 = 16 cm.
71. (3) Total successful surgeries = 5 + 4 + 5 + 6 + 4 + 3 + 2 = 29. V1 a 3 1
Total unsuccessful surgeries = 3 + 4 + 2 + 3 + 2 + 3 + 4 = 21. 77. (3) = 3 =
V2 A 27
Total number of surgeries = 21 + 29 = 50.
a 1
=
29 A 3
Percentage of successful surgeries 100 = 58%.
50 S1 6a 2 1 1
2
= 2
= ‚ = .
S2 6A 3 9
72. (4) Total Total
Total
Percentage of
Month Successful Unsuccessful
Surgeries
Successful 78. (3) sin cos tan = 1
every
Surgeries Surgeries Surgeries
month = 45
Jan 5 3 8 5 / 8 × 100 = 62.5%
Feb 4 4 8 4 / 8 × 100 = 50%
then, sin 45 + cosec45 = 2 + 2 = 2 2
March 5 2 7 5 / 7 × 100 = 71.4% 79. (1) 7 P 24 M 8 Q 6 M 2 L 3
April 6 3 9 6 / 9 × 100 = 66.6% After putting signs we get
May 4 2 6 4 / 6 × 100 = 66.6% 7 + 24 ÷ 8 – 6 ÷ 2 × 3
June 3 3 6 3 / 6 × 100 = 50% = 7 + 3 – 3 × 3 = 7 + 3 – 9 = 10 – 9 = 1.
July 2 4 6 2 / 6 × 100 = 33.3% 80. (2) Persons known the Bengali and English language but
not known the Hindi = 62.
The percentage of successful surgery is highest in 81. (3) 53 persons knows all the three languages.
the month of March. 82. (1) Persons who do not know Hindi language
73. (2) The 7 multiples that leave a remainder of 1 when = 120 + 83 + 62 = 265.
divided by 3 and 5 are to be counted. B
In other words, the 7 multiples that leave a remainder 83. (2)
9 km
of 1 when divided by 15 are to be counted. 8 km
15 km O C
Between 1 and 100 :
6 km
7 multiples are 7, 14, 21, 28, 35, 42, 49, 56, 63, 70, 77, 84,
91 and 98. A
Starting Point
15 multiples between 1 and 100 are 15, 30, 45, 60, 75, 90.
(Multiples of 15) + 1 are 16, 31, 46, 61, 76, 91 AC = AO 2 + OC 2 = 62 +82 = 36 + 64 = 100 = 10.
91 is the only multiple of 7 which is 1 more than a 10 km North-East.
multiple of 3 and 5. 84. (2) If 1st Jan 2008 is Tuesday so 1st Jan 2009 is Thursday.
74. (2) The greatest 3-digit number having 8 in tens place and 85. (4) Total students = 35
divisible by 4 is 988. Ibrahim Rank = 8th
The smallest 3-digit number having 8 in tens place and So students after him in class = 27
divisible by 4 is 180. So his rank from last = 28th.
So, the difference is 808. 86. (b) Let ‘x’ be the common root.
75. (2) Let us say that the total number of books are N. x 2 + ax + b = 0 ...(1)
Number of English books = 0.5N x 2 + bx + a = 0 ...(2)
Number of Hindi books
(a b) x (b a) = 0
7
( N 0.5 N ) 0.7 0.5 N = 0.35 N a b
10 x= =1 x =1
a b
The shaded portion, that represents the number of books
Then, {1 + a + b = 0}
in Hindi, should be more than quarter and less than half.
From equation (1), a + b = 1
Hints & Solutions M-39

87. (1) Let, P = x, A = 2x 99. (4) (93 – 27) = 66 – 63 = 3


x R 10 (79 – 38) = 41 – 37 = 4
2x R = 20% (67 – 16) = 51 – 42 = 9
100
Then, ATQ 100. (4) UNCLE QQYOA
x 20 T ' U N C L E
3x = –4 +3 –4 +3 –4
100
Q Q Y O A
T'
3 T ' = 15 years C O L U M N
5
–4 +3 –4 +3 –4 +3
1 2 Y R H X I Q
88. (3) V1 r h ...(1)
3 101. (4) By observation (A) is false but (R) is true.
1 2 102. (2) By observation Both (A) and (R) are true but (R) is not
Also, V2 r h(3h) ...(2) correct explanation of (A).
3
103. (4) By observation.
Volume is increased by 300%.
104. (1) Children who do not like black = 100 + 30 + 20 = 50.
89. (1) a + b = 51 Children who only like black and white = 50
150 3
Ratio
26 + 25 = 51 50 1
(Alphabets from Right to left) Required ratio = 3 : 1.
105. (3) Top numbers square multiply given in bottom.
x + y =5
(102 112 ) 100 121 12100.
(92 × 102) = 81 × 100 = 8100
3 + 2 =5 (32 × 72) = 9 × 49 = 441
106. (2) In column : 3 (9 × 5) = 345
p q 1 11 10 =1.
Similarly: 5 (7 ×11) = 577.
S 107. (3) Combinations of two digits number having 8.
90. (4) 85, 58, 82, 28, 81, 18, 87, 78, 85, 68, 88
20 So, 11 combinations can be made.

5
108. (4) W T

10
R
S
150000 109. (4)
91. (2) = 9375
16 110. (1) (4 + 5 + 9) = 18.
111. (3)
92. (3) 10, 20, 30 4050 60 7080 90 100 112. (2) The pattern is as :
(4 + 12 + 6 + 4)2 –2 = 674
110, 120 , 130, 140, 150 , 160, 170, 180 , 190, 200 (4 + 14 + 6 + 18)2 –2 = 1762
14 (12 + 10 + 14 + 16)2 –2 = 2702

93. (1) (42 6) (15 3) 2 113. (2) 1 5 6


2 7
(36 9) (9 3) 1
4 2 1 2
(38 19) (20 10) 0 Maharashtra
94. (2) Sequence AABBCC is repeated.
114. (1) As, 43 42 80
95. (4) BAAC / ACCB / CBBA / BAAC
96. (4) CA / CCAA / CCCAAA / CCCCAAAA Similarly, 21 212 9702.
3

97. (2) 25 + 17 = 42 = 6 × 7 115. (4) moving 1 place in right side.


38 + 18 = 56 = 8 × 7 116. (1) Let most element is eliminating and new elements add
89 + 16 = 105 = 15 × 7 at right most place remaining elements shift place
towards left side.
98. (4) 4 × 2 –1 = 7, 7 × 2 + 1 = 15, 15 × 2 – 1 = 29, 29 × 2 + 1 = 59
59 × 2 – 1 = 117, 117 × 2 + 1 = 235, 235 × 2 – 1 = 469.
M-40 Target NTSE

E J O T : V Q L G :: B G L Q : S N I D
117. (3) 5 10 15 20 22 17 12 7 2 7 12 17 19 14 9 4 Red
+5+5+5 –5–5–5 +5+5+5 –5–5–5
Yellow Black
Black Green Yellow

Pink
Red
+2 +2 +2
118. (1) By observations. Pink
White White
119. (4) - 951874
White
120. (4) Multiply the corner elements with each other i.e. 15 ×
6 × 4 = 360 then divided it by 10. 129. (4) The number in the intersection of triangle (Rural) and
360 ÷ 10 = 36 is the middle number. circle (Unemployed), but not covered by square
Q 50 × 10 × 10 = 5000 ÷ 10 = 500 (Hardworking), and rectangle (Intelligents), but not in
121. (3) 36 + 64 + 25 + 49 = 26 the circle (Unemployed), i.e. 6.
20 + 2
9 + 25 + 16 + 81 = 21 130. (2) = 11
2
25 + 144 + 36 + 64 = 31
102 + 2
122. (3) “21” represent the common part of all three destination = 52
that’s why 21 is answer. 2
123. (4) 52 + 41 + 46 = 139 131. (1) 62 – 1 = 35
72 – 1 = 48
124. (1) From the given statement on unemployment, it can be
concluded that, to end poverty more employment 132. (3) 326 : 3649
opportunities need to be created. But the statement 32 = 9, 22 = 4, 62 = 36
doesn’t convey that all people in the country are Which is written in reverse order as 3649.
unemployed. Similarly,
So, the conclusion (I) follows but conclusion (II) 534 : 16925
doesn’t follow. 52 = 25, 32 = 9, 42 = 16
125. (3) Rupee and Dollar are currencies for different countries. Which is written in reverse order as 16925.
They do not have anything in common. 133. (3) The closed figure is separated into two equal parts,
then it is rotated 180 degree and inside figure is rotated
90 degree.
Currency
134. (1) As, M A R C H
–2 +2 –2 +2 –2
Rupee Dollar K C P E F
Similarly,
O R D E R
–2 +2 –2 +2 –2
126. (3) As, O N L I N E
M T B G P
L N O E N I 135. (3) P = 16, E = 5, N = 14
So, (16 × 5 – 14) = 66.
And, S I L E N T
Similarly, L = 12, E = 5, G = 7
L I S T N E So, (12 × 5 – 7 ) = 53.
Similarly, L I S T E N 136. (3) P D 8
16 4
S I L N E T
127. (2) Decoding AVOID = 73564 letter by letter we get, 16 4
A = 7, V = 3, O = 5, I = 6, D = 4 4 × 2 = 8
H B
Similarly, in CHINA = 28617;
8 2
C = 2, H = 8, I = 6, N = 1, A = 7.
So, the code for COVID : 8 2
C = 2, O = 5, V = 3, I = 6, D = 4 2 2 2
i.e., COV ID = 25364.
128. (1) The colours adjacent to Green and Yellow, Red, Pink 2 2 2 = 4
and Black. The only colour remaining is White which
will be on the opposite face to Green.
Hints & Solutions M-41

23 26 29 148. (1) S.I. unit of ‘angle’ is ‘radian’.


137. (3) Similarly S.I. unit of ‘Force’ is ‘Newton’.
149. (3) As,
169 169 ? F R U I T
16 12 15 13 14 17 –6 –6 –6 –6 –6
16 × 12 – 23 = 169; 15 × 13 – 26 = 169; 14 × 17 – 29 = 209 Z L O C N
138. (3) Two alternative series.
Similarly,
22 19 16 13 10 7
F L O W E R
–3 –3 –3 –3 –3 –6 –6 –6 –6 –6 –6

3 9 27 81 243 Z F I Q Y L
×3 ×3 ×3 ×3 150. (1) P O L I C E
243, 7 is the answer. 265143
139. (1) 392, 365, 342, 323, 308, 297, 290, 287
151. (1) T
–27 –23 –19 –15 –11 –7 –3
140. (2) As lion is related with Roar, similarly, Ass is related F S
with Bray.

141. (3) 72 7 : 82 8 : : 112 11:122 12 152. (1) The pattern is as :


42 56 110 132
52 + 3 = 28
142. (2) Multiply all outer number divided by 10.
52 + 4 = 29
120
5 × 6 × 4 = 120, = 12 52 – 2 = 23
10
210
6 × 7 × 5 = 210, = 21 153. (1) 3C 27D 9E C(+1) D (+1) E
10
320 27D
4 × 8 × 10 = 320,= 32
10
143. (1) Top numbers square’s difference. 7I 84K 12M I(+2) K (+2) M
x
112 52 121 25 = 96 ?
5A 13G A(+3) D (+3) G
15 2
9 2
225 81 = 144 65D

212 16 2 441 256 = 185


144. (3) 95, 94, 92, 89, 85, 80, 74
154. (2)
–1 –2 –3 –4 –5 –6
145. (2) 720, 360, 120, 30, 6, 1 Agricultural Engineers
Officer
÷2 ÷3 ÷4 ÷5 ÷6
146. (2) The pattern of the series is as : Professionals
5 + 7 = 12
12 + 14 = 26
26 + 21 = 47 155. (4) (i) ‘im be pee’ - ptals are blue
47 + 28 = 75 (ii) ‘sik hee’ means ‘red flowers’
75 + 35 = 110 (iii) ‘pee mit hee’ means ‘flowers are fragrant’
147. (3) The pattern of the series is as : From (i) and (iii), Pee - are
I. 2, 5, 10, 17, ? From (ii) and (iii), hee - flower, Slk - red and mit - fragrant
+2 +5 +7 +9 Then, ‘fragrant are red flowers’ - mit Pee Sik hee
17 + 9 = 26 156. (4) As per the law and moral values, you have to deposit
in the nearest police station whether its small or big
II. 2, 4, 8, 14, 22
amount.
+2 +4 +6 +8
M-42 Target NTSE

D(+) 170. (3)


Sol. (157-161) 171. (2)
172. (2)
E A B Sol. (173-176):
(–) (+) (+)

C(–) F (+) Subject Class


A English IX
157. (4) E is the mother.
158. (3) A is E’s husband. B History VIII
159. (4) There are 4 male members in the group.
160. (1) F is son of E. C Hindi VI
161. (1) ABF are brothers.
D Chemistry V
Sol. (162-165) P
E Maths X
F Physics VII
A L
G Geograph y XI
S K
173. (1) From the table, F is in VII class.
( - Male, - Female) 174. (3) From the table, G is in XI class.
162. (2) 175. (3) From the table, A likes English.
163. (1) 176. (1) From the table, B is in VIII class.
164. (3) Sol. (177-178)
165. (4) As,
Smt. Mittal Shri Mittal Smt. Rita Goyal Shri Sumer Goyal
R O M E

Kriti Amit Ananya Sonu Rakesh


M O R E
Similarly, Urvi Poorvi
D A R E 177. (3) From the family tree, Sumer is the maternal grandfather
of Poorvi.
178. (2) From the family tree, Urvi is the niece of Kriti.
R A D E
t D
166. (1) Sol. (179-181)
p q
E– A+ B+

r s + –
F C
167. (3) Sushil Vipin Prashant Amar 179. (1) Three, A, B and F.
Drawing Music Drawing Dance 180. (3)
Move Dance Dance Craft
181. (4)
Craft Craft
168. (3) 182. (4) Mother
169. (3)
F
C
Sol. (170-172) E F
Woman P
Same
D B

H G Person
A
Hints & Solutions M-43

Sol. (183-185)
Two figures are possible.
( Male, Female) 1 2

As uncle 3 4

A E F C B D 5 8
6 7
As uncle

a b c
A E F B D C B D 201. (2) k say
3 5 7
183. (2) Three Female F, B, D
a = 3k, b = 5k, c = 7k
184. (4) Cousin
185. (1/2)Cousin or Sister. a b c 3k 5k 7 k 15k
3
Sol. (186-188) b 5k 5k
P is neither wearing White nor Green Cap
i.e. P – White, Green ...(1) 202. (4) 17 x 11 11 6
Q is not wearing White Cap Squaring both sides,
i.e. Q – White ...(2)
S wears Red Cap 17 x 11 11 6 2 66
i.e. S – Red ...(3) 17 x 11 17 2 66
From (1) P is either Red or Blue
S is already Red so P is Blue and x 11 2 66
Q is Green from (2).
2 66
P Q R S x 2 6
11
Blue Green White Red 2
186. (1) R wears White Cap. x2 2 6 24
187. (4) So P wears Blue Cap. 203. (4) Checking option
188. (2) So Q wears Green Cap. (1) 36 × 4 ÷ 8 – 7 + 4 = 10
189. (4) (1), (2), (3) figures contains number of sides 3, 4, 5, 6
15 10
either in clockwise (or) anticlockwise.
But (4) do not obey the same rule. (2) 16 × 12 + 49 ÷ 7 – 7 = 200
190. (3) 192 200
191. (4) Figure (4) contains both dots in same side of diagonal. (3) 32 ÷ 8 × 9 = 160 – 12 × 12
192. (2) First take mirror image and then water image. 36 16
193. (2) A : B : : C : D (4) 8 × 8 + 8 ÷ 8 – 8 = 57
Interchange positions of two figures. 57 = 57
194. (2) Take mirror image of figure and reverse the tail. 204. (1) a + b = sum of roots = 2 and
195. (1)
ab = product of roots = (–1)
196. (2) By observation.
a2b + ab2 = ab (a + b)
197. (3)
198. (2) By observation. = (–1) (2) = (–2)
205. (4) 12 litres is taken out & replaced by water
199. (4) new ratio is 4 : 1
4 1
Now milk is part and water is part
5 5
4 48
Thus in 12 litres quantity of milk = 12 Ltrs.
5 5
200. (1) The triangles are 48
1, 2, 3, 4, 5, 6, 7, 8, (1, 2), (1, 3), (2, 4), (3, 4), (6, 7), (5, 6, So remaining milk 48 38.4 Ltrs.
5
7), (3, 4, 8), (5, 3, 4), (6, 7, 8)
So, total triangles are ‘17’.
M-44 Target NTSE

20 16 30 16 South
206. (3) B
Left AB Right Left BA Right
From the diagram total no. of boys = 30 + 16 – 1 = 45
207. (2) The relative speed of minute hand and hour hand is 218. (1) East A
C
5.5°
Angle between hour & minute hand at 8 O’ clock
= 240° D
For the hands to be together, they must coincide, the North
angle of 240° must be covered by them by the relative
219. (2) No. of odd days from :
240 7
speed in time per minute 43 minutes past 8. 17th Dec 2002 – 17th Dec 2003 = 1
5.5 11 17th Dec 2003 – 17th Dec 2004 = 2
208. (1) Let ‘x’ minutes be the time taken by pipe C 17th Dec 2004 – 22nd Dec 2004 = 5
For pipes A, B and C to fill the tank together
We have : 5 2 1 8
Total no. of odd days 1 odd day
7 7
10 10 10
1 x = 20 minutes 22nd Dec 2004 will be Saturday + 1 = Sunday
12 15 x 220. (4) A + B > C + D ...... (1)
Sol. (209-212): B+ C > D +A ...... (2)
209. (3) No. of visitors from Karnataka in the age group of Adding (1) and (2)
20 – 40 years A + 2B + C > A + 2D + C
= 60% of 20% of total population 2B > 2D
60 20 221. (2) Square of opposite number
100000 = 12000 3 9
100 100
210. (1) No. of visitors from Maharashtra below 20 years 4 16
7 49
5 60 8 64
100000 = 3000
100 100 222. (1) BAAB AABAABAABAAB
211. (3) No. of visitors below 20 years of age who were neither Letters BAA are repeated
from Karnataka nor Maharashtra or West Bengal BABAB
15 60
100000 = 9000 223. (4) By (1), 12 2 2 5 (1, 5, 2)
100 100 By (2), 72 + 82 = 113 (7, 113, 8)
212. (2) Required ratio is : By (3), 22 + 42 = 20 (2, 20, 4)
60% of 20% of 100000 : 20% of 5% of 100000 By (4), 32 + 42 = 25 (3, 25, 4)
60 20 20 5 So option (4) should be (3, 25, 4) but not (3, 27, 4)
= 100000 : 100000 224. (1) Ram’s grand father’s son is Ram’s father. Ram’s father’s
100 100 100 100
daughter is Ram’s sister.
= 12000 : 1000 = 12 : 1
225. (2) By observing row wise, we get
331 16 2 A = 1, E = 1 + 4 = 5, k = 5 + 6 = 11
213. (4) 100 50 3 25 20 3 G = 7, k = 7 + 4 = 11, Q = 11 + 6 = 17
100 100 100 100 100 100 M = 13, Q = 13 + 4 = 17, 17 6 23 w
1 2 3 4 5 6
226. (1)
214. (3) Kitchen and bathroom are both parts of house.
Sol. (215-217) : T X
227. (3) : 2 :: : ?
J H
(Sister)
(Brother/Sister) B C F (Brother) Position of T in alphabets = 20
Position of J = 10

(Son) A (Son) D
T 20
E (Daughter) 2
J 10
215. (1) A is the cousin of D.
216. (3) E is the niece of F. X 24
Similarly, 3
217. (3) A and D are both nephews of F. H 8
Hints & Solutions M-45

228. (3) Given series consists of 3 patterns. 237. (3) By observing the pattern figure D and A have similar
characteristics.
2 2 2 2
Y W U S Q
E
1 2 3 4 238. (2) P
E F H K O S
2 3 2 3 N S
B D G I L
Q R
North W
S is to the south east of Q.
North-East 239. (3) Hour hand rotates by 30° in 60 minutes (1 hour) in
60 m
30
20 m 10 minutes hour hand rotates by 10 5
60
229. (2) in 5 hours and 10 minutes hour hand rotates by 5
10 m
× 30° + 5° = 155°.
10 m
East 240. (4) Adjacent faces of 1 are 2, 3, 4, 5.
20 m So, opposite face of 1 is 6
So, I am in North - East direction from starting point. 241. (3)
242. (1) The logic is
230. (3)
2 × 3 = 56

(2 + 3) (2 × 3)
4 × 1 = 54
No. of squares = 24 243. (1) Let four prime numbers in ascending order are x, y, z
231. (2) CHARACTER 241612376 and t respectively,
CHILDREN 24859670 xyz 1001
Comparing the values from above examples yzt 2431
HIRALAL 4861515
232. (3) Fourth Friday will be on 23rd of the month. 7
Fourth Tuesday will be on 27th of the month. x 1001 7
C t 2431 17
17
233. (3) E F
x 7
t 17
A B last number = 17
D 244. (3) 48 sec, 72 sec, 108 sec
Between B and C. 48 = 2 × 2 × 2 × 2 × 3
234. (2) Correct sequence as per weight is : 72 = 2 × 2 × 2 × 3 × 3
T> U> R> S= P > Q 108 = 2 × 2 × 3 × 3 × 3
235. (2) E LCM of 48, 72, 108 = 2 × 2 × 2 × 2 × 3 × 3 × 3
12
= 432
11 1
10 2 432
432 secs mins 7 mins12 sec
N 9 3 S 60
8 4 Hence, they all will again change simultaneously at
7 6 5 7 : 07 : 12 hrs.
W 1 1 1 1
245. (2) ......
Minute hand point towards west and Hour hand 5 6 6 7 7 8 24 25
towards south-east
Time = 1 : 30 hr 1 1 1 1 1 1 1 1
.....
Time after three hours = 1 : 30 + 3 = 4 : 30 hours. 5 6 6 7 7 8 24 25
236. (4) After interchanging the signs × and + we have
1 1 5 1 4
(20 × 3) + 6 × 8 – 32 = 76 0.16
5 25 25 25
76 = 76.
M-46 Target NTSE
246. (2) Volume of wood = volume of pencil-volume of lead 260. (1) The pattern is:
2 1, 3, 7, 13, 21, 31, 43, 57
3 1
28 28
10 10 +2 +4 +6 +8 + 10 + 12 + 14
So, missing number is 31.
9 1 261. (3) The pattern is
28
100 100 5 , 3 , 10 , 8 , 17 ,15 ,26 24
,
8 22 8 –2 +7 –2 + 9 –2 + 11 –2
28 28 = 7.04 cm3
100 7 100 +2 +2
247. (1) 25 255 2545 25455 254545 So, missing number is 26.
262. (2) All are countries and kabul is city
×10+5 255×10–5 2545×10+5 25455×10–5 263. (1) According to question, P + R – Q
248. (1) From given statements :– – +

Blue 2, Sky 1, Was 3, People 8 R Q


Like 0, In 0, Bird 9
‘People like birds’ 809 +
249. (2) The pattern is: P
13, 74, 290, 650 So, Q is father of P.
264. (3) The pattern is:
22 + 32 = 13
52 + 72 = 74 +3 +3
112 + 132 = 290
18 24 21 27 24 30 27
172 + 192 = 650
232 + 292 = 1370 +3 +3 +3
So, missing number is 1370. So, missing number is 24
250. (4) A – BBC – AAB – CCA – BBCC 265. (4) The pattern is :
A A BB/CCAA/BBCC/AABB/CC +2 +2 +2
251. (4) BAAB / BAAB / BAAB
ABBA is answer ELFA, GLHA, ILJA, KLLA, MLNA
Sol. (252-253): +2 +2 +2
M 4 (as 48 is common number in code for MSPTQ & ABLMS) So, missing term is KLLA.
N 7 (as 1 is common number in code for PTQAB & ATRNP) 266. (4)
252. (4) 267. (2) According to question,
253. (4) 63 : 80 : : 120 : 137
254. (4) AS, K M F Similarly, R M S +17 +17
+1 –1 +2 +1 –1 +2 268. (1) The Pattern is :
L L H S L U
+2 +4 +8 +16
255. (2) As, Smoke Pollution BJ DL HP PX FN
Similarly, War Death
256. (2) According to dictionary; +2 +4 +8 +16
3. Dissent 1. Dissident The next term = FN.
4. Dissolute 5. Dissolution 269. (1) Only 141 is not a prime number but other numbers
2. Dissolve are prime number.
So, option (2) is correct. 270. (4) The pattern is :
257. (4) Except option (4) all are antonyms. But Option (4) is +2 +2 +2
Synonyms. 3 4 1, 5 7 2, 7 8 1, 990
258. (3) There is L in MISSILE which is not present in COM-
3+1=4 7+1=8
MISSIONER. So, MISSILE cannot be formed using the
letter of the given word. 5+2=7 9+0=9
259. (2) The pattern is : So, option (4) is correct answer.
25
44
271. As, ZEAL = 26 + 5 + 1 + 12 11
4
13 17 19 23 27 29 28
Similary, B E A T = 2 + 5 + 1 + 20 7
4 2 4 2 4 4
So, 27 is wrong number. Logic Sum of all place then divide by 4.
Hints & Solutions M-47

272. (4) 02, 13, 34, 56 285. (4) According to question,


Check row and column. Let A scored = x, then A + E + D + B + C = (36 × 5) = 180
273. (4) 15 – 2 ÷ 900 + 90 × 100 = ? x + (x – 8) + (x – 3) + (2x – 11) + (118 – 2x) = 180
15 × 2 + 900 ÷ 90 – 100 = 15 × 2 +10 – 100 3x = 84 x = 28
30 + 10 – 100 = 40 – 100 = – 60 E = x – 8 = 28 – 8 = 20
274. (3) 7 + 6 + 5 + 4 + 3 + 2 + 1 = 28 286. (2) According to questions,
275. (4) Total hours = 89 hours LCM of 6, 5, 7, 10 and 12 is 420 so, that bells will toll
together after every 420 seconds ie. 7 minutes
44 356
Faulty time 23 hours & 44 min = 23 i.e, hr Now 7 × 8 = 56 & 7 × 9 = 63
60 15
So, in 1 hour (60 min) the bells will toll together 8 times,
356 excluding the one at start
hr of incorrect watch = 24 hr of correct watch
15 287. (1) According to questions,
C 15 yrs
24 15
89 hours of incorrect watch = 89 90 hrs A 12 yrs (C – 3)
356
D 11 yrs (A – 1)
Actually watch will be 1 hr faster than faulty watch.
B 10 years (D – 1) Age of B = 10 years.
i.e., 11 pm
288. (2) According to question,
276. (2)
B
Standing Positions:
Facing North 6 km
Teacher Student Doctor Clerk Businessman
277. (2) 52 < 4 5 > 8 2 10 km C 3 km D
52 – 4 × 5 + 8 ÷ 2
4 km
52 – 4 × 5 + 4
56 – 20 = 36
278. (1) If the order is A
×+ =
Then 6 × 3 + 4 = 22 is true AD = 42 + 32 = 25 = 5km(North east)
So, option (1) is correct answer. 289. (1) According to question,
279. (2) If the order is × – =
Then 7 × 3 – 8 = 13
So, option (2) is correct answer. MT KA S N
280. (3) According to question, T is sitting between M and K.
15th September 2000 Friday 290. (3) Logic Pattern N R M L J O Q P K
15th September 2001 Saturday [because 2001 is a J is sitting at the centre.
non-leap year i.e. +1 odd day] 291. (2)
281. (4) Aeroplanes fly in the sky, sky is called sea. so, sea 292. (4) 3 × 4 = 12 (Alphabet Position of L)
is answer.
1 × 17 = 17 (Alphabet position of Q)
282. (2) Here, S = +, D = –, Q = ÷ and p = x
5 × 4 = 20 (Alphabet position of T)
35 ÷ 7 × 5 + 5 – 6 = 24
293. (4) After removing even numbers :
283. (2) According to question,
(8 × 5) + (1 × 4) = 44 15 9715 3597 5 9 13513
(6 × 8) + (2 × 8) = 64 Middel Number
Similarly, 294. (4) Total hours = 89 hours
(7 × 9) + (4 × 8) = 83 44 356
Faulty time 23 hours & 44 min = 23 i.e, hr
284. (2) 60 15
356
hr of incorrect watch = 24 hr of correct watch
15
24 15
89 hours of incorrect watch = 89 90 hrs
356
Actually watch will be 1 hr faster than faulty watch.
i.e., 11 pm
M-48 Target NTSE
306. (2) The pattern is :
295. (2) Unemployment Illiterates 7 × 2 + 2 = 16
16 × 2 + 2 = 34
34 × 2 + 2 = 70
70 × 2 + 2 = 142
142 × 2 + 2 = 286.
So, missing number is 70.
307. (2)
Poor
308. (2)
296. (4) The pattern is: Sol. (309-311):
32 + 31 + 30 + 28 = 121 = (11)2 Sitting Arrangement.
70 + 72 + 73 + 74 = 289 = (17)2
112 + 108 + 106 + 115 = 441 = (21)2 A/C G B H C/A
missing number = 17
297. (4) The pattern is:
5 + 12 + 13 = 30 = |3 – 0| = 3
13 + 9 + 4 = 26 = |2 – 6| = 4
7+ 5 + 16 = 28 = |2 – 8| = 6
missing number is 6. D I E J/F F/J
298. (1) According to question, 309. (2) 310. (1) 311. (4)
Sol. (312-313):
3 30 – 20 = 2
Rajesh Priya Madavi
3 67 – 40 = 3
3 416 – 200 = 6
So, missing number is 6.
Girija Vinod
299. (4) By observation. Option (4) is correct answer.
300. (1)

E
W Uma Satish Gyan
312. (1)
I 313. (3)
Sol. (314-316) :
I
+
C
+
E
II A

So, only conclusion (I) is true


301. (3)
302. (1)
303. (2) 15 Students got distinction in both physics and maths
subject only.

304. (1) As the numbers 2, 3, 4 and 5 are adjacent to 6. D B
+
F
+

Hence, the number on the face opposite to 6 is 1. 314. (4)


2 2
315. (4) ‘A’ is mother of ‘B’
305. (2) 5 (5 – 2) = 9 21 (21 – 19) = 4 316. (3)
16 109 2 22 53 19 Sol. (317-318):
2 6 2 15 PLAYERS Foot Ball BaseBall Cricket VolleyBall Hockey
(16 – 6) = 100 (22 – 15) = 49
Hemant
100 + 9 = 109 49 + 4 = 53
Madhav
2
51 (51– 48) = 9 Govind
Ramesh
17 ? 48 Anant
2 13
(17 – 13) = 16 317. (2) 318. (1) 319. (3) 320. (4) 321. (4)
16 + 9 = 25 322. (4) 323. (4)
Hints & Solutions M-49

324. (3) C 341. (2) Except option (2), in all other 'T' and ' 'are opposite to
each other.
Sol. (342-345): After matching two cloumn I and II, we can
decode that
B M D
Letter C D E J K L N P V
A E Code 3 8 2 9 4 5 7 0 1
P O N
342. (4) 343. (2) 344. (3) 345. (1)
R E A S O N
G F 346. (2) As, +1 +1 +1 +1 +1 +1
325. (2) A S F B T P O
T H I N K
Similarly, +1 +1 +1 +1 +1

U I J O L
326. (3) 327. (4)
328. (1) 4 side divided into 4
347. (2) By observation, the 'guest room' Means 'vosefstrisi'
10 side divide into 10
348. (1) The given alphabet series is
329. (2) 330. (2) 331. (3) 332. (4) 333. (3)
334. (3) x c w m v c x w m w m x c wm x m
335. (2) The venn diagram is as following:
So, only 1 time m is succeeded by w and preceded
by w.
349. (1) Given : + = 6, = p and 2 2 10
Now, ( + ) – 2
2
= 2+ 2
Crows Doves
rro
ts (6)2 – 2 × p = 10
Pa Cats
36 – 2p = 10

p = 13
Conclusions:
I. II. 90 10 90 10 80
350. (2) Here, 10
So, only II follows. 8 8 8 8
336. (4) The pattern is as follows: ( x 1) ( x 1) (2 x 2 2)
hfeg/hfeg/hfeg/ hfeg 351. (2) Given :
( x 1) ( x +1) ( x 2 +1)
S C H O O L Now,
337. (3) As, +2 –2 +2 –2 +2 –2
( x +1)2 ( x 2 +1) +( x 1) 2 (x 2 1) (x 2 1)(2 x 2 2)
U A J M Q J
( x 2 1)( x 2 +1)
P R I N C I P L E
Similarly, +2 –2 +2 –2 +2 –2 +2 –2 +2
( x 2 2 x 1)( x 2 1) (x 2 2 x 1)
(x 2 1) (x 2 1)(2 x 2 2)
R P K L E G R J G
( x 4 1)
338. (1) The code for 62830 will be written as HATCB
339. (1) 'BHICK' will be coded as 06734. x4 x 2 2 x3 2 x x 2 1 x 4 x2
G I L P 2 x3 2 x x 2 1 2 x 4 2 x2 2 x2 2
340. (3) As, , (x 4 1)
+2 +3 +4
D F I M 2(2 x 2 )(2) 8 x 2
( x 4 1) x4 1
+2 +3 +4 352. (1) Let the speed of Parth in still water be x km/hr and
E G J N speed of stream be y km/hr.
and Then,
+2 +3 +4 According to the Question,
B D J K 16 8
But, + =6 ...(i)
+2 +6 +1 x y x y
So, BDJK is different.
M-50 Target NTSE

24 6 362. (4) Given : 15 = 5 + 2 × 1 ÷ 7


and + =6 ...(ii) By putting signs in option (4)
x y x y
15 ÷ 5 × 2 + 1 = 7
From (i) and (ii), we get
3×2+1=7
x + y = 8 and x – y = 2
6 + 1= 7
On solving, we get
7= 7
x = 5 km/hr and y = 3 km/hr.
363. (3) According to the question, if the first person face is
Hence, the speed of Parth in still water is 5 km/hr.
towards north then the right hand of second person
75 5 32 will be in west direction.
353. (4) Given : log 2log +log ‚
16 9 243 N
2
75 5 32 Ist Person facing North
log + log + log ‚
16 9 243 W E
75 81 32 2nd Person facing South
log log 2 Right
16 25 243
S
354. (3) Given : 315 + 316 + 317
315 (1 + 3 + 32) 364. (4) Let the ten's digit be x and unit's digit be y.
315 (1 + 3 + 9) Then, number = 10x + y
315 (13) Number obtained by interchanging the digits = 10y + x
Hence, it will be completely divisible by 13. (10x + y) + (10y + x) = 11(x + y)
355. (4) From the options :
which is divisible by 11.
(1) –7 7 × 7 + 7 = 0
(2) (7 + 7 × 7) 7 – 7 = 1 365. (3) Birth date of Kiran = 12th September.
(3) 7 – 7 × 7 7 + 7 = 7 Soham is 12 days younger thus, his birth date is = 1st
(4) 7 – (7 7 × 7 + 7 ) = – 7 September.
356. (1) Given : 6 R 8 S 1 R 3 Q 5 P 7 Q 4 P 2 Hindi day = 14 September : (considered as Friday).
Now putting code : Therefore, 1st September of same month is Saturday.
= 6×8 1×3– 5+7–4+2 366. (3) The direction diagram of a farmer is as follows:
= 6 × 8 × 3 – 5 + 7 – 4 + 2 = 144 – 5 + 7 – 4 + 2 = 144
357. (1) By observation, A 6m B N
= 12 : 00 – 9 : 30 = 2 : 30
12m
358. (1) By option (1), 20 m 6m W E
6 20 × 12 + 7 – 1 = 70 D
8 C
By putting signs, O 10
S
6 – 20 + 12 × 7 1 = 70 Farmer's
6 – 20 + 12 × 7 = 70 House
6 – 20 + 84 = 70 Here, In ODC,
70 = 70
D = 90°
359. (2)
OC = 82 + 62 = 100 = 10 .
Fri Sat Sun Mon Tue Wed Thu Fri
So, he is 10 m from his Original position.
Today Third
Day 367. (2) The sequence is :
So, if day before yesterday was friday, then friday will ML K J I H G F E D C BA ZY XWV UT S R Q PO N
be the third day after the day after tomorrow.
12th letter from left 7th letter right of B.
360. (4) As, (10)3 = 1000
So, U is 7th to the right of the 12th letter from the left side.
(100)2 ÷ 10 = 10000 ÷ 10 = 1000
368. (1) The arrangement of five persons is as following:
and 23 × 53 = (10)3 = 1000
Kamal > Rashi > Vinita > Priti > Leela
But, 2000 – 2 = 1998 3rd in order of height is Vinita.
So, except (4), all result in 1000.
361. (1) The relation is as follows : 15 6 4
369. (3) As, In 1st figure, =36
Father wife Mother 10
Mother's only Uncle 6 7 5
and In 2nd figure, = 21
brother 10
Uncle's son Cousin
50 10 10
So, Mahipal is cousin to sailesh Similarly, In 3rd figure , =500
10
Hints & Solutions M-51

370. (3) how can you go Je de ke pe ...(i) 378. (2) The pattern of the series is as follows :
you come here ne ke se ...(ii) 5 10 20 40 80

come and go re pe se ...(iii) 5 10 20 40


From (ii) here ne 5 10 20
371. (3) Alternate series: 5 10
+5 +5 +5 5
2 65 7 59 12 53 17 47 379. (3) The correct venn diagram represents female, mother
and doctor is as shown below.
–6 –6 –6
372. (4) As, (5 – 4) + (4 – 3) 1+1 2 Female
and (6 – 0) + (5 – 1) 6 + 4 10
Mother
Similarly, (6 – 2) + (7 – 2) 4+5 Doctor
9

373. (2) N N
12 12 380. (2) The sequence is as follows :
W 9 3 E W 9 3 E 4 36 5 7 5 3 6 4 5 7 3 57 3 5 3
6 6 So, it is clear that only one 5s which is immediately
51followed by 3 but not immediately preceded by 7.
S S 381. (1) The pattern of the series is as follows:
Given : 12 : 30 2 : 45
It is clearly shown that the minutes hand will be in 16 19 28 43 64 91
west direction. +3 +9 +15 +21 +27
374. (3) The pattern is as follows:
4×1=4 +6 +6 +6 +6
4×2=8 382. (3) The pattern of the series is as follows :
8 × 3 = 24 63 58 51 40 27 10
24 × 4 = 96
–5 –7 –11 –13 –17
96 × 5 = 480 383. (2) The pattern of the series is as follows :
480 6 = 2880
3 4 8 17 33 58 94
2880 × 7 = 20160
375. (3) The pattern is as follows: +1 +4 +9 +16 +25 +36
9 + 3 = 12 2 2 2 2 2 2
1 2 3 4 5 6
12 + 6 = 18 +1 +1 +1 +1 +1
384. (2) As, I Figure : 48 ÷ 6 = (8)2 = 64
18 + 12 = 30
II figure : 30 ÷ 5 = (6)2 = 36
30 + 24 = 54 Similarly,
54 + 48 = 102
III figure : 35 ÷ 7 = (5)2 = 25
376. (4) The blood relation diagram is as shown below:
385. (1) Alternate series :
Mother 582 588 634 600
Father-in-law Amit +6 +6 +6
Grandson
605 611 617
Girl Wife Son
+6 +6
So, Amit is father-in-law to the girl. There should be 594 in place of 634.
377. (1) The pattern of the series is as follows : 386. (3) Alternate series :
121 144 169 196 225 256 C F I L O R

+3 +3 +3 +3 +3
(11)2 +1
(12) 2 +1
(13) 2 +1
(14) 2 +1
(15)2 +1
(16) 2
Z X V T R

–2 –2 –2 –2
M-52 Target NTSE
387. (1) The figure may be labelled as shown. 394. (3) G and B are to the left of C.
C 395. (2) C and G are the neighbours of B.
B L 396. (1) E is to the immediate left of D is not true.
A N D Sol. (397– 398):
M
Q Arrangement from Richest to poorest
K
I Bhanudas > Akhil > Amar > Chaitanya > Gopal
P O (Farmer) (Lawyer) (Doctor) (Teacher)
H E
J (Photographer)
F 397. (4) Chaitanya's occupation is Teacher.
G 398. (2) Bhanudas is a farmer.
The simplest squares/rectangles are - ABCI, LCDK, 399. (1) The pattern is as follows:
HIJG, JKEF, MLQI, LNK, QKOJ and QJPIQ i.e 8.
abcd/bcad/ ca bd / abcd/bcad/ cab
The squares/rectangles composed of two components
400. (1) ‘X’ shifts to downwards and ‘–’ shifts to left side.
each are ABKJ, ILEF, HLKG, IJCD, MLQJP, LNOJ,
MNKI and IKOP i.e 8 401. (2)
The squares/rectangles composed of th ree/four 4
components each are - ABEF, HGDC, ILKJ and 6 3 5
MNOPL i.e 4.
Total no. of squares/rectangles in the figure = 8 + 8 + 4 = 20. 2
388. (2) As, 37 = 3 + 7 = 10 46 = 4 + 6 = 10
and 28 = 2 + 8 = 10 1
Similarly, In option (2), 55 = 5 + 5 = 10 So, if 1 is at the bottom then 3 will be on the top.
389. (3) The pattern of the series is as follows: 402. (2) The water image is as shown below:
100 , 25 , 100 , 50 , 100 i.e. 20 720
water
9 2 7 3 5
–2 –2 403. (4) By observation, option (4) is correct answer.
390. (2) As, In 1st figure : 404. (3) In each pair, both figures interchange their position.
(5 7) (21 17) 12 4 48 Inner figures becomes small and stands on the top of
= = = 24 the first figure.
2 2 2
and In 2nd figure : opposite opposite
405. (2) As, B alphabet
Y, D alphabet
W
(13 7) (28 25) 20 3 60
= = = 30 opposite
2 2 2 Similarly, L alphabet
O
Similarly,
406. (2) As, In Ist figure : 12 + 4 + 9 + 3 = 28 ÷ 4 = 7
In 3rd figure :
In 2nd figure : 5 + 13 + 6 + 12 = 36 ÷ 4 = 9
(2 8) (16 10) 10 6 60 and In 3rd figure : 8 + 3 + 2 + 7 = 20 ÷ 4 = 5
= = = 30
2 2 2
Similarly, In 4th figure : 21 + 5 + 16 + 6 = 48 ÷ 4 = 12
391. (4) The code of EUKAR will be 407. (3) By observation, option (3) is correct.
392. (2) As, 42 × (4 – 1) = 16 × 3 = 48 408. (1) Option (1) is correct.
Similarly, 52 × (5 – 1) = 25 × 4 = 100 409. (1) Option (1) is correct answer.
393. (3) The arrangement is as following : 410. (3) Option (3) is correct answer.
M 411. (4) Option (4) is correct.
F 412. (3) 413. (3) 414. (1) 415. (2) 416. (3)
P
417. (4) 418. (3) 419. (2) 420. (4) 421. (4)
422. (3) 423. (2) 424. (1) 425. (2) 426. (2)
B J 427. (1) 428. (4) 429. (4) 430. (1) 431. (2)
432. (3) 433. (3) 434. (2) 435. (1) 436. (2)
437. (1) 438. (2) 439. (4) 440. (3) 441. (2)
T R 442. (2) 443. (3) 444. (1) 445. (2) 446. (1)
It is clearly shown that, R is fourth to the right of M. 447. (3) 448. (2) 449. (1) 450. (2) 451. (2)
So, correct option. is (3). 452. (3) 453. (4) 454. (4) 455. (4) 456. (1)
Sol. (394-396): 457. (2) 458. (4) 459. (3) 460. (4) 461. (4)
The sitting arrangement is as following : 462. (1) 463. (3) 464. (4) 465. (2) 466. (3)
G B C D E F A Facing 467. (2) 468. (4) 469. (4) 470. (1) 471. (3)
North 472. (2) 473. (1) 474. (2) 475. (4) 476. (4)
Stage-1 (2020–15) Solved Questions
Scholastic Aptitude Test – SAT
SECTION 1. PHYSICS
1. Assertion (A) : Cooking food is difficult on hills. (1) 8 m/sec (2) 12 m/sec
Reason (R) : The boiling point decreases with increase of (3) 16 m/sec (4) 28 m/sec
pressure. (AP_FIITJEE 2020-21) 10. The weight of a body is 9.8 Newton, when g = 9.8 m/sec2 the
(1) Both A and R are true and R is the correct explanation mass of the body is (Bihar 2020-21)
of A (1) Zero (2) 9.8 kg (3) 10 kg (4) 1 kg
(2) Both A and R are true but R is not the correct explanation 11. Flying birds has (Bihar 2020-21)
of A (1) only kinetic energy
(3) A is true and R is false (2) only potential energy
(4) Both A and R are false (3) both kinetic energy and potential energy
2. A plane mirror is approaching you at a speed of 5 cm/sec. (4) only pressure energy
The image will approach you with a speed 12. Two bodies of mass 2 gram and 4 gram having same kinetic
(AP_FIITJEE 2020-21) energy having their ratio of linear momentum as
(1) 0 cm/sec (2) 5 cm/sec (Bihar 2020-21)
(3) 10 cm/sec (4) 20 cm/sec
(1) 2 : 1 (2) 2 :1 (3) 1 : 2 (4) 1 : 16
3. A body mass 50 kg has momentum of 3000 kg-m/sec. Then
its kinetic energy is (AP_FIITJEE 2020-21) 13. The sound of same pitch and loudness are "distinguished
(1) 900 J (2) 90,000 J from one another by their (Delhi 2020-21)
(3) 90 J (4) 9,00,000 J (1) Wavelengths (2) Velocity
4. The distance between the near point and the eye of a person (3) Quality (4) Tones
suffering with hypermetropia is 50 cm. Then what is the 14. A water pumps lifts water from a level 10 m below the ground.
power of the lens to be used by the person? The water is pumped at the rate of 30 kg/min with negligible
(AP_FIITJEE 2020-21) velocity. Calculate the minimum power the pump should
(1) +1D (2) +2D (3) +3D (4) +4D have to do this work. (Delhi 2020-21)
5. The power of a lens in the spectacles of a person is –2D. (1) 49 J/s (2) 490 J/s
The person suffers from (AP_FIITJEE 2020-21) (3) 500 J/s (4) 48 J/s
(1) Hypermetropia (2) Myopia 15. Six identical resistors connected between points A, B and C
(3) Colour blindness (4) Presbyopia as shown in diagram. The equivalent resistance would be
6. Two block of masses 8 kg and 12 kg connected at two ends maximum between (Delhi 2020-21)
of a inextensible string. The string passes over a frictionless C
pulley. Then the acceleration of the system is
(AP_FIITJEE 2020-21) R R
R
g g g g R R
(1) (2) (3) (4)
4 5 8 6 R
7. In Circular motion, particle moves with (Bihar 2020-21) A B
(1) Constant Velocity (2) Zero Acceleration (1) A and B
(3) Constant Speed (4) Speed changes (2) B and C
8. A car travels half distance with speed 20 m/sec and next half (3) A and C
distance travel with 30 m/sec, the average speed of the car (4) Option (1), (2) & (3) are correct
is (Bihar 2020-21) 16. A particle of mass 0.3 kg is subjected to a force F = Kx with
(1) Zero (2) 25 m/sec (3) 24 m/sec (4) 5 m/sec K = 15 N/m, what will be its acceleration if it is released from
9. A body is travelling with speed 20 m/sec having acceleration a point x = 20 cm. (Delhi 2020-21)
4 m/sec2 the speed of the body after 2 sec is (1) 1 m/s 2 (2) 10 m/s 2

(Bihar 2020-21) (3) 100 m/s 2 (4) 0.1 m/s2


S-2 Target NTSE
17. An object is moving in a straight line. The velocity time 23. The resistance of rheostat shown in the figure is 0 - 30 W,
graph is as shown in figure. Then (Delhi 2020-21) neglecting the resistance of ammeter and connecting wire
the minimum and maximum currents through the ammeter
B will be (Delhi 2020-21)

A 2.2v
velocity

A
O time 10W

(1) In part OA acceleration is increasing constant 30W


(2) In part AB acceleration is increasing 20W
(3) In part OA acceleration is decreasing
(1) (0.08 A, 0.33 A) (2) (0.06 A, 0.08 A)
(4) In part AB acceleration is decreasing
(3) (0.06 A, 0.33 A) (4) (0.33 A, 0.09 A)
18. A force of 100 N acts on a body so that the body acquire a 24. Three particles A, B and C are thrown from top of a building
velocity of 10 m/s after some time. Now the force of 100 N is with same speed. A is thrown upwards, B is thrown
replaced by another force F which decelerates the body and downwards and C is thrown horizontally, they hit the ground
body come to the rest then. (Delhi 2020-21)
with speed VA, VB and VC respectively then
(1) F > 100 N (2) F < 100 N (Delhi 2020-21)
(3) F = 100 N (4) All options are possible
19. 2 points A and B are at electric potentials 10 V and 100 V (1) V A = VB = VC (2) VB > VC > VA
respectively. A charge q is taken from A to B and 18 Joule of (3) VA = VB > VC (4) VA > VB = VC
work is done. The value of q is (Delhi 2020-21)
25. An object of height 2.0 cm is placed on the principal axis of
(1) 2 Coulomb (2) 0.2 Coulomb
a concave mirror at a distance of 12 cm from the pole. If the
(3) 20 Coulomb (4) 0.02 Coulomb image is inverted, real and 5 cm in height then location of
20. Which of the following is NOT correct for magnetic field the image and focal length of the mirror respectively are
lines? (Delhi 2020-21) (Delhi 2020-21)
(1) The direction of magnetic field lines outside the magnet (1) (–30 cm, +8.6 cm) (2) (–30 cm, –8.6 cm)
is from north pole to south pole.
(3) (+30 cm, +8.6 cm) (4) (+30 cm, –8.6 cm)
(2) The direction of magnetic field lines inside the magnet
26. Luggage porters place a round piece of cloth on their head
is from south pole to north pole. when they have to carry heavy loads. By doing this, the
(3) The degree of closeness of magnetic field lines tells area of contact between the load and head :
the relatives strength of magnetic field.
(Karnataka 2020-21)
(4) Magnetic field lines never from closed loop.
(1) Decreases and reduces the pressure
21. A car moving along straight line covers 1/5th of total distance
(2) Increases and reduces the pressure
with speed v1 and remaining part of distance with speed v2.
The average speed of car over entire distance is (3) Decreases and increases the pressure
(4) Increases as well as increases the pressure
(Delhi 2020-21)
27. A boy covers a distance of 260 m in 20 s and another distance
5v1v2 4v1v2 of 300 m in 20 s. The average speed of the boy is :
(1) v2 + 4v1 (2) 5v1 + v2 (Karnataka 2020-21)
–1 –1
(1) 14 ms (2) 28 ms
5v1v2 4v1v2 (3) 42 ms
–1
(4) 56 ms
–1
(3) 4v2 + v1 (4) 4v1 + v2 28. In the following, the phenomena which were successfully
22. Light travels through a glass slab of thickness t and having explained by the law of gravitation are :
refractive index n. If c is the velocity of light in vacuum then (Karnataka 2020-21)
the time taken by light to travel this thickness of glass is a. Motion of moon around the earth
(Delhi 2020-21) b. Formation of tides
c. Motion of an aeroplane
t nt
(1) (2) d. Force that binds us to the earth
nc c e. Rotation of blades in a ceiling fan
n2t t (1) b, c, d (2) d, e, a
(3) (4) (3) c, d, e (4) a, b, d
c n2 c
Stage - I Solved Questions S-3

29. The image formation in a convex lens is shown in the following 6V


figure. (Karnataka 2020-21) Bulb
Switch
A
C F2 B'
B F1 O 2F2

20 cm 20 cm A'
AB is the object and A'B' is the image. The focal length of
the lens is Electrode
(1) 40 cm (2) 10 cm (3) 20 cm (4) 05 cm
Alcohol solution
30. The image formation in human eye is shown in the given
diagram. (Karnataka 2020-21) (1) The circuit is open
O - Object (2) The battery is not connected properly
I - Image (3) The electrodes are too short
(4) Alcohol does not conduct electricity
O I 35. A ray of light follows the path as shown in figure as it travels
through different media. Choose the correct relation
regarding refractive indices from the given alternatives.
(Maharashtra 2020-21)
The lens used to correct this type of eye defect is
(1) Plano concave lens (2) Concave lens 40°
(3) Cylindrical lens (4) Convex lens Medium (1) m1
31. The lenses of power +3.5D, +2.5D and +1D are placed in 50°
Medium (2) m2
contact with each other in an optical device. The effective
power of combination of these lenses is : 30° m3
Medium (3)
(Karnataka 2020-21)
(1) 5 D (2) 6 D (3) 7 D (4) 8 D Medium (4) m4
32. Read the following and select the correct option. 45° m5
(Karnataka 2020-21) Medium (5)
Statement (a) : When we pour little petrol on our palm, it (1) m1 > m 2 < m3 = m 4 > m 5
causes the palm to feel cool.
(2) m1 = m2 < m3 = m 4 > m5
Reason (b) : Particles of petrol evaporate using the energy
from our palm which causes the palm to feel cool. (3) m1 > m 2 < m3 > m 4 < m5
(1) 'a' is correct and 'b' is not the correct reason. (4) m1 < m 2 < m3 = m 4 > m 5
(2) 'a' is correct and 'b' is the correct reason. 36. The main objective of ______ satellite launched by COEP
(3) Both 'a' and 'b' are correct but 'b' is not the reason for 'a' (College of Engineering, Pune) orbiting at a height of
(4) 'a' is not correct so 'b' is not the correct reason _________ is to provide point-to-point messaging services.
33. Schematic atomic structure of four elements are given below. (Maharashtra 2020-21)
Observe and choose the right statement. (1) Samarpan, 550 km (2) Sampoornam, 540 km
(Karnataka 2020-21) (3) Swayam, 515 km (4) Sayam, 500 km
37. Observe the columns I and II, match them and select the
correct alternative form the given options.
(Maharashtra 2020-21)
Column I Column II
a b c d
A. Motion of earth around (i) Nuclear force
sun
(1) Atomic number of 'c' is 8. So it is inert gas. B. Motion of stone tied to (ii) Electromagnetic force
(2) 'a' is the schematic atomic structure of oxygen. a string and whirled in
(3) 'b' has 8 electrons and it represents octet configuration. a circle
C. Motion of electron (iii) Gravitational force
(4) 'd' is the schematic atomic structure of sodium.
around nucleus
34. The bulb is not glowing in this experimental setup. Because D. Motion of blades of fan (iv) Tension
(Karnataka 2020-21)
S-4 Target NTSE
(1) (A)-(i), (B)-(ii), (C)-(iv), D-(iii) 45. If the speed of wave is 250 m/s and its wavelength is 50 cm
(2) (A)-(ii), (B)-(iii), (C)-(i), D-(iv) then the frequency will be (Rajasthan 2020-21)
(3) (A)-(iv), (B)-(i), (C)-(iii), D-(ii) (1) 5 Hz (2) 500 Hz
(4) (A)-(iii), (B)-(iv), (C)-(i), D-(ii) (3) 50 Hz (4) 12500 Hz
46. An object of mass 10 gm is moving with an acceleration of
38. If mass of a planet is 25 times mass of earth and radius of the 10 m/s2. Force acting on the object will be
planet is 125 times radius of earth then escape velocity of an
(Rajasthan 2020-21)
object from the plant (VP) is _______ times the escape
velocity from earth (VE). (Maharashtra 2020-21) (1) 1 N (2) 0.1 N
(3) 1000 N (4) 100 N
1 1
(1) (2) 5 (3) (4) 5 47. Lens formula is (Rajasthan 2020-21)
5 5
39. If approximate refractive index of sapphire is 1.8, then 1 1 1 1 1 1
(1) + = (2) + =
approximate decrease in velocity of light, when light enters v u f v 2u f
sapphire is ________ (Maharashtra 2020-21)
1 1 1 1 1 1
(1) 40% (2) 45% (3) 50% (4) 55% (3) - = (4) - =
v u f v 2u f
40. Group of vector quantities are : (Rajasthan 2020-21)
(1) displacement, velocity, time 48. Focal length of a lens is 50 cm. In dioptre, power of lens will
be (Rajasthan 2020-21)
(2) area, density, mass
(1) 0.02 D (2) 2 D
(3) speed, length, impulse
(3) 0.2 D (4) 50 D
(4) velocity, acceleration, force
49. Correct relation between radius of curvature (R) and focal
41. The velocity-time graph of an object moving with uniform length (F) of spherical mirror is (Rajasthan 2020-21)
velocity is (Rajasthan 2020-21)
F
y y (1) R= (2) R = F
2
(3) R = 2F (4) R
50. Refraction from denser to rarer medium for a light ray, the
Velocity

Velocity

value of angle of refraction at the condition of critical angle


(1) (2)
is (Rajasthan 2020-21)
x x (1) 0° (2) 180° (3) 45° (4) 90°
Time Time
51. The resistance of a bulb marked 220V, 10W is
y y (Rajasthan 2020-21)
(1) 242W (2) 4840W
(3) 121W (4) Zero
Velocity

Velocity

52. A person of mass 100 kg reaches a height of 5 meters in 10


(3) (4) seconds .............. power used by the person (g = 10 m/s2)
x x (Rajasthan 2020-21)
Time Time (1) 500 watt (2) 250 watt
42. If force, momentum and displacement are represented by A, (3) 5000 watt (4) 50 watt
æ AC ö Rg - Ra
B and C respectively, then the term ç ÷ will represent: 53. a= , where Rg , Ra – Resistance, T, T 0 –
è B ø Ra (T - T0 )
(Rajasthan 2020-21) Temperature, then the unit of a is : (Tamil Nadu 2020-21)
(1) momentum (2) acceleration (1) W / °C (2) W°C (3) / °C (4) °C / W
(3) velocity (4) displacement 54. A girl of mass 40 kg stands on her feet of surface area 80 cm2
43. The mass of a person on earth surface is 60 kg then his mass (g = 10 m/s2). The pressure exerted by her feet is ?
on moon will be (Rajasthan 2020-21) (Tamil Nadu 2020-21)
(1) 60 kg (2) 360 kg (1) 5 × 103 Pa (2) 5 × 104 Pa
(3) 20 kg (4) 10 kg (3) 0.5 Pa (4) 2 Pa
44. On halved th e distance between two masses, the 55. If the EMF of a cell is 1.5 V, then the energy provided by the
gravitational force between them will be cell to drive 0.5 C of charge around the circuit :
(Rajasthan 2020-21) (Tamil Nadu 2020-21)
(1) half (2) one-fourth (1) 1 J (2) 0.75 J
(3) four times (4) double (3) 0.5 J (4) 0.25 J
Stage - I Solved Questions S-5

66. A stone is allowed to fall freely downwards initially at rest


56. North
from the top of a tower. The time taken by the stone to reach
5N the bottom of the tower is 4 seconds. What is the height of
3N the tower/ Take, acceleration due to gravity = 32 ft/s2
East (West Bengal 2020-21)
West 7N
(1) 64 ft. (2) 32 ft.
4N
(3) 48 ft. (4) 256 ft.
4N
67. The coefficient of linear expansion of a solid is x and the
South coefficient of volume expansion of the solid is y, then
In which direction does the net force act? (Tamil Nadu 2020-21) (West Bengal 2020-21)
(1) North (2) South (3) East (4) West
y x y x
57. Pick out the correct statements regarding the properties of (1) x= (2) y = (3) x = (4) y =
Magnetic Lines of force. (Tamil Nadu 2020-21) 3 3 2 2
(a) Magnetic Lines of force never intersect. 68. In case of refraction of light from a medium to air the critical
angle is found to be 45°. What is the refractive index of the
(b) They will be maximum at the equator than at the poles. medium with respect to air ? (West Bengal 2020-21)
(c) Magnetic Lines of force start from South pole and end
at the North pole. (1) 2 (2) 3 (3) 2 (4) 3
(d) Magnetic Lines of force are closed continuous curves 69. Which of the following pair have same unit?
extending to the body of the magnet. (West Bengal 2020-21)
(1) (a), (b) and (c) only (2) (a), (b) and (d) only (1) Heat and Specific heat
(3) (a) and (d) only (4) (b) and (c) only (2) Thermal capacity and water equivalent
58. Pick out the right answer to correct the given statement. (3) Specific heat and Thermal capacity
The distance travelled by sound in one second is called (4) Heat and Work
time period of the sound. (Tamil Nadu 2020-21)
70. Which of the following is an electromagnetic wave?
(1) Loudness (2) Pitch
(West Bengal 2020-21)
(3) Velocity (4) Frequency
59. What is the far point for normal human eyes? (1) a-ray (2) b-ray
(Uttar Pradesh 2020-21) (3) g-ray (4) cathode ray
(1) 25 cm (2) 50 cm 71. In case of a convex lens, what is the minimum distance
between an object and its real image ?
(3) 100 cm (4) Infinity
60. Refractive index of water is : (Uttar Pradesh 2020-21) (West Bengal 2020-21)
(1) 1.00 (2) 1.33 (3) 1.52 (4) 2.42 (1) 2.5 times of focal length (2) 2 times of focal length
61. A man used a convex lens of focal length of 20 cm in his (3) 4 times of focal length (4) equal to focal length
spects, the power of this lens is: (Uttar Pradesh 2020-21)
72. What will be the power consumed by a 50W wire if it is kept
(1) +2D (2) –2D (3) +5D (4) –5D
across a potential difference of 200 V?
62. In an electric circuit, the voltmeter is used :
(West Bengal 2020-21)
(Uttar Pradesh 2020-21)
(1) in series (2) in parallel (1) 0.8 kW (2) 80 kW
(3) in both manner (4) None of these (3) 400 W (4) 8 kW
63. One horse power (H.P.) is equal to: 73. A boy starts from rest is accelerated uniformly for 30s. If x1,
x2, x3 are the distance travelled in first 10s, next 10s and last
(Uttar Pradesh 2020-21)
10s respecrively, then : x1 : x2 : x3 is (Delhi 2019)
(1) 467 watt (2) 500 watt
(1) 1 : 2 : 3 (2) 1 : 1 : 1 (3) 1 : 3 : 5 (4) 1 : 3 : 9
(3) 746 watt (4) 1000 watt
74. A bomb of mass 3m kg explodes into two pieces of mass
64. A magnet attracts : (Uttar Pradesh 2020-21) m kg and 2 m kg. If the velocity of m kg mass is 16 ms–1, the
(1) only iron (2) only cobalt total kinetic energy in the explosion is (Delhi 2019)
(3) only nickel (4) All the above (1) 192 mJ (2) 96 mJ
65. The mass, linear momentum and kinetic energy of a body (3) 384 mJ (4) 768 mJ
are m, p and E respectively, then (West Bengal 2020-21)
75. Figure shows a ray of light as travels from medium 1 to
(1) p = 2mE (2) E = 2mp medium 2. If reflective index of medium 1 with respect to

(3) p = 2E (4) E = 2p 2
medium 2 is then the value of angle x is (Delhi 2019)
3
S-6 Target NTSE
83. “Metal dishes” (Dish Antennas) are used for receiving TV
signals from distant communication satellites. These ‘Metal
Medium 2
45° Dishes’ are (Chandigarh 2019)
(1) Convex Reflectors
45° (2) both convex and concave reflectors
(3) Concave reflector
(4) Convex refractors
x=? 84. If a symmetrical convex lens of focal length ‘f’ is cut into
two parts along the principal axis as shown in the figure, the
Medium 1 focal length of each part will be (Chandigarh 2019)

(1) 30° (2) 60° (3) 15° (4) 45°


76. Which of the following statements is true? (Delhi 2019)
f’
(1) A convex lens with power + 4D has a focal length
– 0.25 m
(2) A convex lens with power –4D has a focal length +
0.25 m (1) f/2 (2) f/4
(3) A concave lens with power + 4D has a focal (3) f (4) ¥
length –0.25 m 85. A wire of resistance 12 ohm is bent in the form of a circular
(4) A concave lens with power – 4D has a focal length ring. The effective resistance between the two points on
– 0.25 m any diameter of the circle is (Karnataka 2019)
77. If the current through a resistor is increased by 50%, the (1) 24 W (2) 12 W (3) 6 W (4) 3 W
increase in power dissipated will be (assume the temperature 86. For a nuclear reactor 48 KJ of energy is produced per minute.
remains constant) (Delhi 2019) If the energy released per fission is 3.2 × 10–11 J then the
(1) 225% (2) 200% (3) 250% (4) 125% number of fissions which would be taking place in a reactor
78. The linear distance between a consecutive compression and per second is (Karnataka 2019)
a rarefaction in longitudinal wave is (Delhi 2019) (1) 5 × 1014 (2) 2 × 1014 (3) 5.2 ×1013 (4) 2.5 ×1013
l l 3l 87. The speed of sound in air at NTP is 332 m/s. If air pressure
(1) l (2) (3) (4) becomes four times the normal then the speed of sound
2 4 4
79. The distance between two masses is to be halved. The waves will (Karnataka 2019)
gravitational force between them will be-(Rajasthan 2019) (1) Double
(1) Double (2) One-fourth (2) Quadruple
(3) Quadruple (4) Half (3) Remain the same
80. The speed of a waves is 350 m/s and wavelength is 70 cm. 1
The frequency of wave is (Rajasthan 2019) (4) become of the original value
4
(1) 500 Hz (2) 700 Hz (3) 50 Hz (4) 200 Hz
81. The equivalent resistance of the given circuit between points 88. If V1 and V2 are the volume of one gm water at 0°C and 4°C
A and B is (Rajasthan 2019) respectively, then- (Uttar Pradesh 2019)
10W (1) V1 > V2 (2) V1 = V2
A (3) V1 < V2 (4) V1 £ V2
10W 89. The displacement time graph of a body in motion is given as
10W 10W below- (Haryana 2019)

10W B
(1) 40 W (2) 4 W (3) 5 W (4) 0.2 W
Time (s)

82. Farsighted people, who have lost their spectacles, can still
read a book by looking through a small (3 - 4 mm) hole in a
sheet of a panel because (Chandigarh 2019) Displacement (m)
(1) the fine hole produces an image of the letters at a longer Velocity of body is (in m/s)-
distance 1 1
(2) in doing so, the distance of the object is increased. (1) 3 (2) (3) 3 (4)
3 3
(3) in doing so, the focal length of the eye lens is 90. If a body is in equilibrium under the effect of some collinear
effectively decreased. forces, then the minimum number of such forces acting upon
(4) in doing so, the focal length of trie eye-lens is the body are [Delhi 2018]
effectively increased. (1) 3 (2) 2 (3) 5 (4) 4
Stage - I Solved Questions S-7

91. A heater coil is cut into two equal parts and only one part is
used in the heater, the heat generated now will be 4v2 3v 2 6v 2 9v2
(1) (2) (3) (4)
[Delhi 2018] g g g g
(1) doubled (2) four times 102. A bottle full of water containing an air bubble is rotated in
(3) one fourth (4) halved horizontal circle by a string tied to the neck of the bottle.
92. A bar magnet placed in non-uniform magnetic field Then air bubble will: [Haryana 2018]
experiences [Delhi 2018] (1) be collected at bottom
(1) only torque (2) only force (2) remain unaffected
(3) both torque and force (4) neither force nor torque (3) be collected at the wall of bottle
93. How much water a pump of 2 kW power can raise in one (4) be collected at the neck
minute to a height of 10 m? (g = 10 m/s2) [Delhi 2018] 103. Three voltmeters all having different resistances are joined
(1) 1000 litre (2) 1200 litre (3) 10 litre (4) 2000 litre as shown. When some potential difference is applied across
94. The Kinetic energy of a body becomes 4 times of its initial A and B, then readings in voltmeter V1, V2 and V3 are :
value. The new linear momentum will be [Delhi 2018] [Haryana 2018]
(1) same as initial momentum V1 V2
(2) four times the initial momentum
(3) two times the initial momentum
A B
(4) eight times the initial momutum
95. In a simple pendulum mass of bob is m and effecting length
is L. Work done on the pendulum in one complete oscillation V3
in gravitational field of earth is [Delhi 2018] (1) V1 = V2 (2) V2 < V2
1 1 (3) V1 + V2 = V3 (3) V1 + V2 > V3
(1) mgL (2) mgL (3) zero (4) mgL 104. What is potential difference across AB?
4 2
96. The mass of earth is 80 times that of moon and its diameter
A B
is double that of moon. If the value of acceleration due to
3A 1W 6W
gravity on earth is 9.8 ms–2 then the value of acceleration 3V
due to gravity on moon will be [Delhi 2018] [Haryana 2018]
(1) 0.98 ms–2 (2) 0.49 ms–2 (3) 9.8 ms –2 (4) 4.9 ms –2 (1) 24 V (2) 0 V (3) 6 V (4) 18 V
97. Two lenses of focal length f1 and f2 are kept in contact coaxially. 105. Three equal resistors connected in series across a source of
The power of the combination will be [Delhi 2018] emf, dissipate 10 watt. If the same resistors are connected in
parallel across the same emf, the power dissipated will be:
f1f 2 f1 + f 2 f1f 2
[Haryana 2018]
(1)
f1 + f 2 (2) f1f 2 (3) f1 - f 2 (4) f1 + f2 10
98. In figure, a ray of light undergoes N (1) 10 watt (2) 30 watt (4) watt (4) 90 watt
3
refraction from medium A to medium 106. A long wire carries a steady current, is bent into a circle of
B. If the speed of light in medium A one turn and magnetic field at the centre of coil is B. When
is n then the speed of light in X 60o Y
30o it is bent into n-turns, magnetic field at centre of coil will be:
medium B will be [Delhi 2018] [Haryana 2018]
N1 (1) 2n2B (2) 2nB (3) n2B (4) nB
n n 107. If ‘p’ and ‘q’ are distance of object and image from principal
(1) 3n (2) (3) 2n (4) focus ‘f’ of a concave mirror then what is the relation
3 2
99. A body falls freely from a tower and travels a distance of 40 between ‘p’, ‘q’ and ‘f’? [Haryana 2018]
m in its last two seconds. The height of the tower is 1
[Delhi 2018] (1) pq = f (2) pq = f (2) pq = f 2 (4) pq =
f
(1) 54 m (2) 45 m (3) 80 m (4) 65 m
108. When the object is at distances u1 and u2 from a lens, a real
100. Two bodies of masses ma and mb are dropped from different
and virtual images are formed respectively having the same
heights ‘a’ and ‘b’. The ratio of time taken by them to reach
magnification. The focal length of lens is: [Haryana 2018]
the ground is: [Haryana 2018]
u2 u –u u1 + u 2
1 1 (1) u1 + (2) 1 2 (3) (4) u1 + u2
(1) a : b (2) a : b (3) : (4) ma : mb 2 2 2
a b
109. A pump motor is used to deliver water at a certain rate from
101. A person throws ball with a velocity ‘v’ from top of a building a given pipe. To obtain twice as much water from the same
in vertically upward direction. The ball reaches the ground pipe in same time, power of motor has to be increased:
with a speed of ‘3v’. The height of the building is: [Haryana 2018]
[Haryana 2018] (1) 16 times (2) 4 times (3) 8 times (4) 2 times
S-8 Target NTSE
110. Consider the following five graphs (note the axes carefully). 119. What is the equivalent resistance of the given circuit
Which of the following represents motion at constant speed? between points A and B? [Rajasthan 2018]
[Rajasthan 2018]
1W 2W
1W 3W 2W
A B
Distance

1W 2W

Speed
(A) (B)

Time Time 14 17
(1) 10 W (2) 4 W (3) W (4) W
3 6
120. A car covers 30 km at a uniform speed of 60 km/hr. and the
next 30 km at a uniform speed of 40 km/hr. The total time
Acceleration

Velocity taken is: [Madhya Pradesh 2018]


(C) (D) (1) 30 min (2) 45 min
(3) 75 min (4) 120 min
Time Time
121. A stone is thrown upwards with a speed “u” from the top of
a tower. It reaches the ground with a velocity “3u”. The
height of the tower is: [Madhya Pradesh 2018]
Acceleration

u2 2u 2 3u 2 4u 2
(E) (1) (2) (3) (4)
g g g g
Time 122. When the speed of a particle is doubled, the ratio of its
(1) D only (2) D and E (3) A, B and C (4) A and D kinetic energy to its momentum: [Madhya Pradesh 2018]
111. A bullet of mass 50 gm is horizontally fired with a velocity (1) remains the same (2) gets doubled
100 ms –1 from a gun of mass 10 kg. What will be the recoil (3) becomes half (4) becomes four times
velocity of the gun? [Rajasthan 2018] 123. Calculate the wavelength of radio waves of frequency 109
–1 –1 –1 Hz. The speed of radio waves is 3 × 108 m/s.
(1) 100 ms (2) 500 ms (3) 0.5 ms (4) Zero
[Madhya Pradesh 2018]
112. A ball is shot vertically upward with a given initial velocity. (1) 60 cm (2) 40 cm (3) 30 cm (4) 10 cm
It reaches a maximum height of 100 m. If on a second shot, 124. A force of 16 N is distributed uniformly on one surface of a
the initial velocity is doubled then the ball will reach a cube of edge 8 cm. The pressure on this surface is:
maximum height of [Rajasthan 2018] [Madhya Pradesh 2018]
(1) 70.7 m (2) 141.4 m (3) 200 m (4) 400 m (1) 3500 Pa (2) 2500 Pa
113. Let M denotes the mass of earth and let R denotes its radius. (3) 4500 Pa (4) 5500 Pa
The ratio g/G at earth’s surface is [Rajasthan 2018] 125. In which of the following caes, is the work done maximum?
(1) R2/m (2) M/R2 (3) M/R (4) R/M [Madhya Pradesh 2018]
114. The unit ‘hertz’ is same as [Rajasthan 2018]
(1) second (2) second –1(3) metre (4) metre–1 F F
115. A sound wave has a frequency of 10 kHz and wavelength 3 (1) (2)
mm. How much time will it take to travel 3 metre?
30° 45°
S S
[Rajasthan 2018]
(1) 0.1 sec (2) 1 sec (3) 10 sec (4) 0.01 sec
116. The size of image formed by a concave mirror is same as the F
F
size of object. The position of the object will be (3) (4)
S
[Rajasthan 2018]
S
(1) at F (2) between F and C 126. The total electrical resistance between the points A and B of
(3) at C (4) between C and infinity the circuit shown in: [Madhya Pradesh 2018]
117. A convex lens has focal length 30 cm. If an object is placed
at a distance of 15 cm from it then the magnification produced 10W10W10W
by the lens is [Rajasthan 2018]
(1) 6.66 (2) 0.5 (3) 1 (4) 2 A 20W B
118. The electrical resistivity of a conducting wire is K. If its
length and area of cross-section are doubled then the new 20W 20W
resistivity of the wire will be [Rajasthan 2018]
(1) K (2) 2K (3) K/2 (4) K/4 (1) 9.23 W (2) 15 W (3) 30 W (4) 100 W
Stage - I Solved Questions S-9

127. Two wires of same material have lengths L and 2L and cross- (1) Equal to the weight of liquid displaced
sectional areas 4A and A respectively. The ratio of their (2) Equal to the volume of liquid displaced
resistances would be: [Madhya Pradesh 2018] (3) Is greater than the weight of liquid displaced
(1) 1 : 1 (2) 1 : 8 (3) 8 : 1 (4) 1 : 2 (4) Is less than the weight of liquid displaced
139. Two resistance each of 2 ohm are connected in series and in
128. Wire of resistance R is streched to thrice of its original length,
parallel separately. Equivalent resistance is–
what is its new resistance? [Madhya Pradesh 2018]
[Uttar Pradesh 2018]
R (1) 4 W, 1 W (2) 1 W, 4 W
(1) 9 R (2) (3) 3R (4) R/3 (3) 2 W, 2 W (4) 4 W, 4 W
9
129. Galaxy in which we live is: [Madhya Pradesh 2018] 140. The distance travelled by a body falling freely from rest in
(1) Milky way (2) radio galaxy 2nd, 3rd and 5th second of its motion are in the ratio
(3) circular galaxy (4) irregular galaxy [Delhi 2017]
(1) 7 : 5 : 3 (2) 3 : 5 : 7
130. Opposition of flow of electric current is called–
(3) 5 : 3 : 7 (4) 5 : 7 : 3
[Uttar Pradesh 2018]
141. For an incident ray directed towards centre of curvature of
(1) Potential difference
a spherical mirror the reflected ray [Jharkhand 2016]
(2) Electric charge (1) retraces its path
(3) Resistance (2) passes through focus
(4) Electromagnetic induction (3) passes through the pole
131. Which of these does not require a medium– (4) becomes parallel to the principal axis
[Uttar Pradesh 2018] 142. A bomb of mass 9 kg initially at rest explodes into two pieces
(1) Conduction (2) Convection
of masses 3 kg and 6 kg. If the kinetic energy of 3 kg mass is
(3) Radiation (4) None of these
216 J then the velocity of 6 kg mass will be [Delhi 2017]
132. Capacity of a measuring flask is 1 litre. What it will be in
(1) 4 m/s (2) 3 m/s (3) 2 m/s (4) 6 m/s
cubic centimetre– [Uttar Pradesh 2018]
(1) 1 Cubic Centimetre (2) 10 Cubic Centimetre 143. A particle starts its motion from rest under the action of a
(3) 100 Cubic Centimetre (4) 1000 Cubic Centimetre constant force. If the distance covered in first 10s is S 1 and
133. Noise is produced by– [Uttar Pradesh 2018] that covered in first 20s is S2 then [Delhi 2016]
(1) Vibration with high frequency (1) S2=S1 (2) S2=2S1
(2) Regular vibration (3) S2=3S1 (4) S2=4S1
(3) Regular and periodic vibration 144. The volume of a substance is 20 cm3. The mass of the
(4) Irregular and non periodic vibration substances if its relative density is 2.5 will be [Delhi 2016]
134. A ray passing through which part of a lens emerges (1) 5g (2) 50g (3) 500g (4) 5 kg
undeviated– [Uttar Pradesh 2018] 145. Sound travels fastest in [Delhi 2016]
(1) Focus (1) Aluminium (2) Water
(2) Centre of curvature (3) Hydrogen (4) Oxygen
(3) Optical centre 146. A real and enlarged image can be formed by using a
(4) between Focus and centre of curvature (1) convex mirror [Jharkhand 2016]
135. To convert temperature in ‘F into °C we use the formula– (2) plane mirror
[Uttar Pradesh 2018] (3) concave mirror
F C – 32 5 (4) either convex or a plane mirror
(1) = (2) C = [ F – 32]
100 180 9 147. How many planets are there in our solar system
(1) 5 (2) 7 [Jharkhand 2016]
5 9
(3) F = [C – 32] (4) C = [ F – 32] (3) 9 (4) 8
9 5 148. If a wire of resistance 1W is stretched to double its length,
136. A swimming pool appears less deeper than its real depth– then resistance will be [West Bengal 2016]
[Uttar Pradesh 2018]
(1) due to reflection (2) due to refraction 1 1
(1) W (2) 2 W (3) W (4) 4 W
(3) due to dispersion (4) due to lateral displacement 2 4
137. A person is standing 4m away from plane mirror. Distance 149. Two bodies with kinetic energies in the ratio 2 : 3 are moving
between mirror and image is– [Uttar Pradesh 2018] with equal momentum. The ratio of their masses
(1) 4 metre (2) 8 metre
(3) 2 metre (4) 6 metre (1) 1 : 3 (2) 1 : 2[West Bengal 2016]
138. According to law of floating weight of a floating body is– (3) 3 : 2 (4) 2 : 3
[Uttar Pradesh 2018] 150. The resultant focal length of the lenses as shown in the
figure is : [Delhi 2015]
(1) 2f (2) f/ 2 (3) f/ 4 (4) f
S-10 Target NTSE

SECTION 2. CHEMISTRY
1. When 4 g of carbon is completely burnt in air, the volume (1) 22.4 (2) 2.24 (3) 0.224 (4) 5.6
occupied by the liberated gas at STP is ______ 11. A bottle of ammonia and a bottle of dry hydrogen chloride
(AP_FIITJEE 2020-21) connected through a long tube are opened simultaneously
(1) 11.2 L (2) 6.46 L (3) 7.46 L (4) 22.4 L at both ends. The white ammonium-chloride ring first formed
2. Copper present in 50 g of CuSO4 is (AP_FIITJEE 2020-21) will be - (Bihar 2020-21)
(1) 19.90 g (2) 39.81 g (3) 29.5 g (4) 31.71 g (1) At the centre of the tube
3. The mass of 3.011 × 1023 number of nitrogen atoms is (2) Near the hydrogen chloride bottle
(AP_FIITJEE 2020-21) (3) Near the ammonia bottle
(1) 14 g (2) 3.5 g (3) 7 g (4) 28 g (4) Throughout the length of the tube
12. When lead nitrate is heated a brown gas is evolved, the
4. The four quantum numbers of 19th electron of potassium is
evolved gas is ________ (Delhi 2020-21)
(AP_FIITJEE 2020-21)
(1) Dioxygen (2) Nitrogen dioxide
1 (3) Nitrous oxide (4) Dinitrogen
(1) n = 4, l = 3, m1 = 0, ms = +
2 13. When a solution of lead (II) nitrate and potassium iodide
1 are mixed, the yellow ppt is formed, the ppt is of ________
(2) n = 4, l = 0, m1 = 0, ms = +
2 (Delhi 2020-21)
(3) n = 4, l = 0, m1 = 1, ms = 0 (1) KNO3 (2) KCl (3) PbI2 (4) PbI4
1 Conc. H SO
(4) n = 4, l = 1, m1 = 0, ms = + 14. CH3 CH 2 OH ¾¾¾¾¾¾2 4 ® Products
(Delhi 2020-21)
2 443 K
5. A solution of sodium sulphate in water is electrolysed using The products formed in the above reaction is/are
inert electrodes. The products at the anode and cathode are
(1) Ethene and H2O (2) Ethyne and H2O
respectively - (Bihar 2020-21)
(3) Ethane and H2O (4) Methane and H2O
(1) H2, O2 (2) O2, H2 (3) O2, Na (4) O2, SO2
15. Which one of the following oxide is insoluble in water?
6. A substance on treatment with dilute H2SO4 liberates a
colourless gas which produces (Bihar 2020-21) (Delhi 2020-21)
(1) Na2O (2) CuO (3) K2O (4) CaO
(i) turbidity with baryta solution and
16. Which one of the following will have the largest number of
(ii) turns acidified dichromate solution green. atoms ? (Delhi 2020-21)
The reaction indicates the presence of - (1) 100 g of He (2) 100 g of Na
(1) CO32– (2) S2– (3) SO32– (4) NO3–
(3) 100 g of Li (4) 100 g of Al
7. A gas formed by the action of alcoholic KOH on ethyl iodide
decolourless alkaline KMnO4 solution, the gas is - 17. The set of products formed when electricity is passed
through brine is : (Karnataka 2020-21)
(Bihar 2020-21)
(1) Cl2, Na2CO3 and CO2 (2) Cl2, NaCl and H2
(1) CH4 (2) C2H6 (3) C2H4 (4) C2H2
(3) Cl2, NaOH and H2 (4) Cl2, HCl and NaOH
8. A light greenish salt is soluble in water. On passing H2S gas
18. The equation of the reaction when aluminium is heated in
into the solution, a black precipitate is obtained which
air is given below. (Karnataka 2020-21)
dissolves readily in HCl. The metal ion present is -
(Bihar 2020-21) xAl + yO 2 ® zAl 2O3
(1) Fe2+ (2) Co2+ (3) Ni2+ (4) Mn 2+ The value of 'z' when the reaction is balanced is :
9. An organic compound (X) on treatment with acidified (1) 2 (2) 1 (3) 4 (4) 3
K2Cr2O7 gives a compound (Y) which reacts with I2 and 19. Observe the following chemical equation of carbon compound.
sodium carbonate to form tri-iodomethane. The compound In presence
(X) is - (Bihar 2020-21) CH3Cl + Cl 2 ¾¾¾¾¾® x + HCl
of sunlight
O Name of the chemical reaction and the product 'x' respectively
||
(1) CH3OH (2) CH3 - C - CH3 (Karnataka 2020-21)
(1) Oxidation reaction and CH2Cl
(3) CH3CHO (4) CH3 - CH - CH3 (2) Combustion reaction and CH2Cl2
|
OH (3) Substitution reaction and CH2Cl2
10. How many litres of CO2 at STP will be formed when 100 mL (4) Addition reaction and CH2Cl
of 0.1 M H2SO4 reacts with excess of Na2CO3? 20. Observe the given diagram and read the following statements
(Bihar 2020-21) (Karnataka 2020-21)
Stage - I Solved Questions S-11

29. Conjugate acid-base pair is (Rajasthan 2020-21)


Cell wall
(1) HCO3- , CO32 - (2) NH +4 , NH 2–
Nucleus
(3) OH - , H 2 O 2 (4) NO -2 , NO3-
30. Gas used for precipitation of pure common salt (NaCl) from
saturated solution of common salt is -(Rajasthan 2020-21)
(i) These are cells of collenchyma tissue (1) H2 (2) Cl2 (3) HCl (4) CO2
(ii) At maturity these cells lose their nucleus and cytoplasm 31. Correct increasing order of reactivity of elements is -
(iii) The cells are living and thick at their corners (Rajasthan 2020-21)
(iv) This tissue gives flexibility to plant parts (1) Au, Cu, K, H (2) Au, Cu, H, K
Correct statements are : (3) Cu, Au, K, H (4) Cu, Au, H, K
(1) (i), (iii) and (iv) (2) (i), (ii) and (iv) 32. An element 'X' has 8 electrons in the M shell when it gains
one electron. Which statements given below are not correct
(3) (i) and (iii) only (4) (i) and (iv) only
about the element 'X'? (Tamil Nadu 2020-21)
21. How many covalent bonds are present in propyne molecule? (i) It belongs to 3rd period in the modern periodic table.
(Maharashtra 2020-21)
(1) 5 (2) 7 (3) 6 (4) 5 (ii) It has a valency of 7.
22. Oxidation product of methyl alcohol is ________ (iii) It belongs to 18th group in the modern periodic table.
(Maharashtra 2020-21) (iv) Its valency is 1.
(1) acetic acid (2) methyl amine (1) (i) and (ii) (2) (ii) and (iii)
(3) formic acid (4) ethyl acetate (3) (ii) and (iv) (4) (i) and (iv)
23. From the given diagram, identify the element 'X' and 'Y'? 33. The Rule of Eight was proposed by :(Tamil Nadu 2020-21)
(Maharashtra 2020-21) (1) Kossel and Lewis (2) Henry Moseley
Group 13 14 15 16 17 18 (3) Dimitri Mendeleev (4) John Newland
Period 34. (a) CH3 - CH 2 - OH (b) CH3 - O - CH3
II B
If compounds (a) and (b) have same molecular formula but
III Si different kind of arrangements, then compound (b) is an :
IV 'x' As (1) alcohol (2) ether (3) aldehyde (4) acid
V 'y' Te 35. If the difference in electronegativity between two elements
is 1.7, then the bond is _______ (Tamil Nadu 2020-21)
VI At (1) 60% covalent 40% ionic (2) 50% covalent 50% ionic
(1) Ga and Sn (2) Ge and Po (3) 40% covalent 60% ionic (4) 70% covalent 30% ionic
(3) Ge and Sb (4) Sb and Po 36. The acid which makes iron passive is _______
24. How many atoms are present in a mole of Ca(HCO3)2? (Tamil Nadu 2020-21)
(Maharashtra 2020-21) (1) Conc. HCl (2) Conc. H2SO4
(1) 5 × 6.02 × 1023 (2) 7 × 6.02 × 1023 (3) Conc. HNO3 (4) Conc. HF
(3) 9 × 6.02 × 1023 (4) 11 × 6.02 × 1023 37. The value of pOH of rain water is : (Tamil Nadu 2020-21)
25. Carbon dating method developed by Willard Libby is based f\ (1) 7 (2) 6 (3) 8 (4) 9.5
upon the radioactive decay of naturally occuring carbon 38. Main component of LPG is (Uttar Pradesh 2020-21)
___________ (Maharashtra 2020-21) (1) Methane + Ethane (2) Ethane + Propane
(1) C14 (2) C11 (3) C6 (4) C22 (3) Propane + Butane (4) None of these
26. Number of molecules present in 0.25 moles of water is 39. What is the product when chlorine gas passes through dry
(Rajasthan 2020-21) slaked lime- (Uttar Pradesh 2020-21)
(1) 3.011 × 1023 (2) 30.11 × 1023 (1) CaCl2 (2) CaO
(3) 1.5055 × 1023 (4) 15.055 × 1023 (3) CaOCl2 (4) None of these
27. Substance having equivalent number of molecules as in 9 g 40. An alloy which does not contain copper is
of water is (Rajasthan 2020-21) (Uttar Pradesh 2020-21)
(1) 12 g of Magnesium (2) 12 g of Carbon (1) Magnalium (2) Bronze
(3) 17 g of Ammonia (4) 11 g of Carbondioxide (3) Brass (4) German Silver
28. Molecular formula of chloride of a metal 'M' is MCl2. 41. Which of the following does not belong to a group-
Molecular formula of oxide of 'M' will be (Uttar Pradesh 2020-21)
(Rajasthan 2020-21) (1) Li, Na, K (2) Be, Mg, Ca
(1) MO (2) M2O (3) MO2 (4) M2O2 (3) N, O, F (4) He, Ne, Ar
S-12 Target NTSE
42. A byproduct of soap industry is- (Uttar Pradesh 2020-21) (1) Acidic (2) Basic
(1) Sodium hydroxide (2) Sodium palmitate (3) Neutral (4) Amphoteric
53. Number of neutrons in isotope of hydrogen, tritium is
(3) Glycerol (4) Gat or Oil
(Rajasthan 2019)
43. The ratio of s and p bonds in propyne is
(1) 0 (2) 1 (3) 2 (4) 3
(West Bengal 2020-21)
54. Molecule containing coordinate covalent bond among the
(1) 1 : 3 (2) 3 : 1 (3) 2 : 3 (4) 3 : 2 following is (Rajasthan 2019)
44. At a given temperature what will be the percentage increase
(1) H2O (2) HNO3 (3) BaCl2 (4) CaO
in pressure for a 5% decrease in the volume of the gas
55. Element having highest atomic radius among the following
(West Bengal 2020-21) is (Rajasthan 2019)
(1) 5% (2) 5.26% (3) 6.26% (4) 10.26% (1) Li (2) Be (3) B (4) C
45. The gas that gives a black precipitate with aqueous Pb (NO3) 56. Valency electron in Cl– ions is (Madhya Pradesh 2019)
solution and a white precipitate with aqueous ZnCl2 solution
(1) 16 (2) 8 (3) 17 (4) 18
is (West Bengal 2020-21)
57. Isoelectronic species are (Karnataka 2019)
(1) CO2 (2) NO2 (3) NH3 (4) H2S
a. Na+ b. Al3+ c. Mg2+ d. Ca2+
46. Find out the position isomers from the following pairs of
compounds (West Bengal 2020-21) (1) a, b and c (2) a, c and d
(3) a, b and d (4) a, b, c and d
(1) H 2 C = C - CH 2 OH, H 2 C = C - OCH 3 58. Identify the correct order of elements according to their
| | metallic character. (Karnataka 2019)
CH3 CH3
(1) B > Al > Mg > K (2) Al > Mg > B > K
(2) H2C = CH – CH2COOH, H2C = CH – COOCH3 (3) Mg > Al > K > B (4) K > Mg > Al > B
59. Identify the solid acid at room temperature.
(3) H 2 C - CH- O - CH 3 , H 3C - CH 2 - CH 2 - O - CH 3
| (Karnataka 2019)
CH3
COOH
(1) | (2) CH3COOH
O O COOH
|| ||
(4) H 2 C - CH 2 - C - CH3 , H3C - CH- C - O - H (3) H2CO3 (4) HCOOH
|
CH3 60. Choose the correct statements about the given chemical
reaction. (Karnataka 2019)
47. Egg albumin in water is a (West Bengal 2020-21) 3Fe( s ) + 4H 2 O( g ) ® Fe3O4( s ) + 4H 2( g )
(1) True solution (2) Colloid a. Iron is getting exidised.
(3) Suspension (4) Supersaturated Solution b. Water is getting reduced.
c. Water is acting as reducing agent.
-
Pt Rh
48. NH3 + O 2 ¾¾¾®
D
A + H2O (West Bengal 2020-21) d. Water is acting as oxidising agent.
(1) a, b and c (2) c and d (3) a, b and d (4) b and c
A + O 2 ¾¾
®B 61. Which block elements are called transition elements?
(Maharashtra 2019)
B + O 2 + H 2 O ¾¾
®C
(1) S-block (2) P-block
A, B and C respectively are (3) D-block (4) F-block
(1) N2O, NO2 and HNO3 (2) NO, NO2 and HNO3 62. In which of the following ink silver nitrate is used?
(3) NO2, NO and HNO3 (4) N2O, NO and HNO3 (Maharashtra 2019)
49. Which of these salts will give acidic solution? (Delhi 2019) (1) Voting ink (2) Writing ink
(1) Na2CO3 (2) NaCl (3) NH4CI (4) COONa (3) Printing ink (4) Marker pen ink
50. pH is defined as : (Delhi 2019) 63. 15 ml of NaOH solution gets complete neutrilised with 10 ml
- log é H 3O +ù of HCl solution. What volume of the same HCI solution will
(1) ë û (2) - log [ H 2O ]
be required to neutralise 30 ml of same NaOH solution?
- log éOH - ù + - (Uttar Pradesh 2019)
(3) ë û (4) - log éë H ùû éëOH ùû
(1) 5 ml (2) 10 ml (3) 15 ml (4) 20 ml
51. A solution turns methyl orange into yellow. The approximate 64. When water gas mixed with half its volume of hydrogen and
pH of solution is (Delhi 2019) the mixture is compressed to 300 atm. pressure and passed
(1) 1.2 – 2.8 (2) 3.1 – 4.4 over ZnO-Cr2O3 catalyst a colourless liquid is obtained
(3) 6.0 – 7.6 (4) 8.3 – 10.00 which is used as solvent for paints & varnishes. The liquid
52. Aqueous solution of SO2 is (Delhi 2019) will be- (Haryana 2019)
(1) methanol (2) ethanol (3) ether (4) acetone
Stage - I Solved Questions S-13

65. Reaction with sodium hydrogen carbonate can be used to (3) Example of emulsion in which dispersed phase is solid
distinguish between- (Haryana 2019) and dispersing medium is liquid.
(1) Ethanoic acid & Methanoic acid (4) Example of gel in which dispersed phase is liquid and
(2) Ethanol and Methanol dispersing medium is solid.
(3) Ethanol and Ethanoic acid 78. On reacting a compound of calcium (X) with water,
compound (Y) is obtained. (Y) on boiling with NH4Cl a gas
(4) Ethylacetate and Ethanol
(Z) is obtained. X, Y and Z respectively, are:
66. Which of the following sub shells present in atom?
[Haryana 2018]
[Delhi 2018]
(1) CaCO3, CaO, NH3 (2) CaCO3, CaO, Cl2
(1) s, p, d, f (2) a, b, c, d
(3) CaO, CaCl2, Cl2 (4) CaO, Ca(OH)2, NH3
(3) s, d, n, g (4) None 79. A colourless gas with choking smell is evolved when Cu
67. Which elements are used in atomic reactors to control the turning are heated with conc. H2SO4. The gas is:
speed of neutrons? [Delhi 2018]
[Haryana 2018]
(1) Boron and cadmium (2) Cadmium and aluminium
(1) SO2 (2) SO3 (3) H2S (4) S
(3) Boron and iron (4) Sodium and potassium
80. Acetic acid is reduced with LiAlH4 to give:
68. How many atoms are present in 1 kg of silver (atomic mass
of silver = 108)? [Delhi 2018] [Haryana 2018]
(1) CH3CH2OH (2) CH3CHO
(1) 2.03 ´ 1023 atoms (2) 5.57 ´ 1024 atoms
(3) CH3OH (4) CH3CH3
(3) 4.27 ´ 10–23 atoms (4) 6.23 ´ 1023 atoms
81. Which of the following metal is not placed in eighth group
69. Buckminister fullerenes is [Delhi 2018] of Mendeleev’s periodic table? [Haryana 2018]
(1) isotope of carbon (2) isobar of carbon (1) Fe (2) Na (3) Pt (4) Ni
(3) allotrope of carbon (4) none of these 82. Baking powder is a mixture of: [Haryana 2018]
70. What is the correct electronic configuration of Cr? (At. No. (1) Sodium carbonate and sodium hydrogen carbonate
24) [Delhi 2018] (2) Sodium carbonate and acetic acid
(1) [Ar] 4s1 3d 5 (2) [Ar] 4s2 3d 4 (3) Sodium hydrogen carbonate and methanoic acid
(3) [Ar] 4s0 3d 6 (4) None of these (4) Sodium hydrogen carbonate and tartaric acid
71. Nature of Al2O3 (Aluminium oxide) is [Delhi 2018] 83. Assertion: CO2 is a gas but SiO2 is a solid at room
(1) Acidic (2) Basic temperature
(3) Amphoteric (4) Neutral Reason: CO2 contain C = O bonds but SiO2 does not contain
Si = O bonds [Haryana 2018]
72. What is the pH of dil. HCl solution which have conc. 10–8
(1) Both Assertion and Reason are true and Reason is a
mol/L? [Delhi 2018] correct explanation of Assertion.
(1) 7 (2) 8 (3) 6.98 (4) 10 (2) Both Assertion and Reason are true but Reason is not
73. Which colour appears when few drops of phenolphthalein correct explanation of Assertion.
put into test tube contains lime water? [Delhi 2018] (3) Assertion is true Reason is false
(1) Yellow (2) Orange (4) Assertion is false Reason is true.
(3) Pink (4) Colourless 84. Which has more number of particles? [Haryana 2018]
74. Which is the correct answer, if n = 4 (where n is number of (1) 46 g of Na atom
shell) then number of sub shells and electron present in (2) 8 g of O2 molecules
atom. [Delhi 2018] (3) 0.1 mole of carbon atom
(1) 16, 32 (2) 32, 16 (3) 32, 32 (4) 16, 16 (4) 28 g of N2 molecules
75. Which of the following are the Green house gases? 85. If the aluminium salt of anion 'X' is Al2X3 the formula of
[Delhi 2018] magnesium salt of 'X' will be: [Haryana 2018]
(1) CO2, CH4, N2O and O3 (1) Mg2X (2) MgX2 (3) MgX (1) Mg2X3
(2) CO2, Octane, Chlorine, Nitrogen 86. KMnO4 is a strong oxidizing agent in acidic medium. To
(3) Methane, Oxygen, Helium, Neon provide acidic medium H2SO4 is used instead of HCl
(4) None of these beacuse: [Haryana 2017]
76. Which salts are responsible for yellow colour of Taj Mahal (1) H2SO4 is stronger acid than HCl
in Agra due to acid rain [Delhi 2018] (2) H2SO4 is a dibasic acid
(1) CaCl2 & CaSO4 (2) Ca (NO3)2 & CaSO4 (3) HCl is oxidized by KMnO4 to Cl2
(4) Only H2SO4 is completely ionized
(3) Ca (NO3)2 & BaSO4 (4) CaSO4 & BaCl2
87. Read the statements about carbon and choose correct
77. Choose the correct option about cheese:
option: [Haryana 2018]
[Haryana 2018]
A. It has small atomic size
(1) Cheese is an example of emulsion in which dispersed B. Its melting & boiling point is low as compared to other
phase is a liquid and dispersing medium is solid. members of group
(2) Example of gel in which dispersed phase is solie=d C. It shows electropositive character
and dispersing medium is liquid.
S-14 Target NTSE
D. It shows maximum tendency of catenation 102. What type of colloidal system is fog?
(1) A and B are correct (2) B and D are correct [Uttar Pradesh 2018]
(3) A, C and D are correct (4) A and D are correct (1) Gas in liquid (2) Liquid in gas
88. A metal 'X' is placed below Al and above Pb. The extraction (3) Liquid in liquid (4) Solid in gas
of metal is done by reacting carbon with its oxide. Metal 103. Unsaturated hydrocarbon among the following is:
oxide is used to join cracks of machine parts and rail lines by [Uttar Pradesh 2018]
reacting it with Al. The metal is: [Haryana 2018] (1) Ethane (2) Methane
(1) Zn (2) Cu (3) Fe (4) Mg (3) Ethylene (4) Propane
89. The method for separation of mixture of common salt and 104. Nature of oxides of non-metal is: [Uttar Pradesh 2018]
ammonium chloride is [Rajasthan 2018] (1) acidic (2) basic
(1) fractional distillation (2) sublimation (3) amphoteric (4) neutral
(3) chromatography (4) crystallization 105. Which among the following coal contains highest % of
90. Conjugate base of HCl in the following reaction is carbon? [Uttar Pradesh 2018]
HCl (aq) + H2O ® Cl– (aq) + H3O+ [Rajasthan 2018] (1) Peat (2) bituminous
(1) H3O+ (2) H2O (3) Cl– (4) HCl (3) anthracite (4) lignite
91. The chemical formula of Plaster of Paris is 106. Most reactive metal is: [Uttar Pradesh 2018]
(1) Na (2) M g (3) Cu (4) Au
[Rajasthan 2018]
107. Zinc and HCl react to form: [Uttar Pradesh 2018]
(1) CaSO4 (2) CaSO4. 2H2O
(1) H2 (2) N2 (3) CO2 (4) He
(3) CaSO4. 1/2H2O (4) CaSO4. H2O
108. Which among the following non metal is liquid at room temp?
92. Which type of catalyst is ethanol in the following reaction?
[Uttar Pradesh 2018]
C H OH
CHCl3 + O2 ¾¾¾¾® 2 5 2COCl2 + 2HCl (1) Chlorine (2) Bromine
[Rajasthan 2018] (3) Mercury (4) Phosphorus
(1) Positive catalyst (2) Negative catalyst 109. Which metal forms hydrogen gas when reacts with cold
(3) Bio-catalyst (4) Autocatalyst water? [Uttar Pradesh 2018]
93. Metalloid among the following is: [Rajasthan 2018] (1) Na (2) Mg (3) Fe (4) Zn
(1) lithium (2) sulphur 110. Thermosetting plastic is: [Uttar Pradesh 2018]
(3) sodium (4) silicon (1) Polythene (2) PVC
CH 3 (3) Bakalite (4) Polypropene
94. The IUPAC name of C = CH 2 is 111. Lens of Spectacles are made from: [Uttar Pradesh 2018]
CH 3 (1) Soft glass (2) Hard glass
[Rajasthan 2018]
(3) Pyrex glass (4) Flint glass
(1) 1,1–dimethyl–2–ethene (2) 2–methyl–1–propene
112. The reaction that take place in nuclear reactor is:
(3) 2,2–dimethyl ethene (4) 2–methyl prop–2–ene
[Uttar Pradesh 2018]
95. Which of the following elements has an electronic
configuration 2, 8, 6? [Rajasthan 2018] (1) Nuclear fusion (2) Nuclear fission
(1) Sulphur (2) Oxygen (3) Controlled nuclear fission (4) Double decomposition
(3) Phosphorus (4) Chlorin 113. Calcium carbide when reacts with water gives
96. Which of the following elements shows variable valency? [Uttar Pradesh 2018]
[Rajasthan 2018] (1) Methane gas (2) Ethane gas
(1) Na (2) Mg (3) Fe (4) Zn (3) Ethylene gas (4) Ethyne gas
97. Formula of aluminium carbonate is: [Rajasthan 2018] 114. Stainless steel is an alloy in which following is added along
(1) Al2(CO3)3 (2) Al2CO3 with iron- [Uttar Pradesh 2018]
(1) Zinc (2) Chromium
(3) Al2HCO3 (4) AlCO3
(3) Tin (4) Copper
98. Formula of Freon-112 is: [Rajasthan 2018]
115. Renewable source of energy is: [Uttar Pradesh-2018]
(1) C2F2Cl4 (2) CF2Cl2 (2) CFCl3 (4) CCl3F
(1) Coal (2) Petroleum
99. Number of molecules present in 14 g of N2 molecules is (3) Natural gas (4) Energy of flowing water
[Rajasthan 2018] 116. Which of following compound is alkaline in aqueous
(1) 6.022×1023 (2) 3.011×1023 medium. [Maharashtra 2018]
(3) 1.51×1023 (2) 3.011×1022 (1) Na2CO3 (2) NaCl
100. The element ‘X’ reacting with chlorine forms a water soluble (3) H2CO3 (4) CuSO4
compound having high melting point. Element ‘X’ is 117. Which of the following elements loses electron most
[Rajasthan 2018] readily? [Maharashtra 2018]
(1) magnesium (2) argon (1) Na (2) Mg (3) K (4) Ca
(3) carbon (3) neon 118. Which of the following species does not have electrons
101. The polymer formed by condensation of adipic acid and equal to 18? [Maharashtra 2018]
hexamethylene diamine is [Rajasthan 2018] (1) K+ (2) Cl– (3) Ca2+ (4) K
(1) isoprene (2) rayon 119. Which of the following is a double displacement reaction?
(3) terylene (4) nylon-6, 6 [Maharashtra 2018]
Stage - I Solved Questions S-15

(1) NH3 + HCl ¾¾® NH4Cl (1) Adsorption is always an exothermic process
(2) CuSO4(aq) + Fe(s) ¾¾® FeSO4 + Cu (2) The soap solution is not a colloidal solution below its
(3) Na2SO4 + BaCl2 ¾¾® BaSO4 + 2NaCl CMC.
(3) 'Argyrol' used in eye – lotion is a colloidal solution
(4) CaCO3 (s) ¾¾® CaO (s) + CO2 (g)
120. The colour of anhydrous copper sulphate is: (4) Gold number is the number of moles of gold formed in
[Maharashtra 2018] anode mud during copper refining.
(1) Blue (2) White (3) Pink (4) Green 133. A mixture of non- reacting gasses contains hydrogen and
121. Ajay has a stung by red ant, it causes itching and irritation. oxygen gases in the mass ratio of 1 : 4 respectively. What
The sting consists of which of the following acid? will be the molar ratio of the above two gases in the mixture?
[Maharashtra 2018] [Delhi 2017]
(1) Acetic acid (2) Butyric acid (1) 16 : 1 (2) 1 : 4 (3) 4 : 1 (4) 1 : 6
(3) Carbonic acid (4) Formic acid 134. Coal is called fossil fuels because: [Haryana 2017]
122. Which of the following compound conduct electricity in (1) In future coal will become completely exhausted.
aqueous solution which is a covalent compound ? (2) It was formed 300 million years ago.
[Maharashtra 2018] (3) Fossils are obtained along with mining of coal.
(1) Calcium Chloride (2) Hydrogen Chloride (4) It was formed from the dead remains of living organisms.
(3) Magnesium Chloride (4) Lithium Fluoride 135. An organic compound butanal has functional group:
123. ______ of the following metal does not react with dil. HCl [Haryana 2017]
[Maharashtra 2018] (1) Aldehyde (2) Ketone
(1) Copper (2) Aluminium (3) Alcohol (4) Amine
(3) Iron (4) Zinc 136. The compound containing both covalent and ionic bond
124. Select a pair of homologous from the following is: [Haryana 2017]
[Maharashtra 2018] (1) MgCl2 (2) NH4Cl
(3) CaO (4) AlBr3
(1) C3H6 and C4H10
137. Identify X in the following reaction– [Rajasthan 2017]
(2) CH3COOH and C2H5COOH
hot, conc.
(3) C4H8 and C3H4 ® ( X ) + H2O
CH3 - CH 2 - OH ¾¾¾¾¾
H 2SO4
(4) (CH3)2CO and C3H7CHO
125. According to IUPAC rule, which of the following compound (1) Ethane (2) Methane
is prop–1–ene? [Maharashtra 2018] (3) Ethene (4) Ethanol
(1) CH3–CH2–CH3 (2) CH3–CH = CH2 138. Electronic configuration of an atom is 2, 8, 1. Which of the
(3) CH3–CH = CH – CH3 (4) CH3–C = CH following elements is similar with it in chemical reactivity?
126. Stainless steel alloy is a mixture of [Maharashtra 2018] [Rajasthan 2017]
(1) Fe + C + Cr + Ni (2) Ni + C + Cr + Al (1) K (2) Cl (3) N (4) Ar
(3) Fe + Cu + Al + C (4) Fe + Zn + C + Ni 139. Which types of radiation absorbed by CO2 molecules in
127. Which of the following elements will form an acidic oxide? atmosphere? [Uttar Pradesh 2017]
[Haryana, Maharashtra 2018] (1) X-rays (2) gamma rays
(1) An element with atomic no. 7 (3) infra-red rays (4) UV rays
(2) An element with atomic no. 3 140. Which of the following salt does not contain the water of
(3) An element with atomic no. 12 crystallization? [Uttar Pradesh 2017]
(4) An element with atomic no. 19 (1) Blue vitriol (2) Baking soda
128. Which of the general formulae represents the alkyl group? (3) Washing soda (4) Gypsum
141. Select a compound which gives effervescence with
[Maharashtra 2018]
NaHCO3 solution: [Maharashtra 2017]
(1) CnH2n (2) CnH2n+1 (3) CnH2n+2 (4) CnH2n–1
(1) C2H6O (2) C2H4O2 (3) C2H4O (4) C3H8O2
129. Raw material required for photosynthesis is _____ and
142. X and Y are the two atomic species [Maharashtra 2017]
water. [Maharashtra 2018]
(1) chloroplast (2) sunlight X Y
(3) nitrogen (4) carbon-dioxide Number of Proton 8 8
130. A green house gas N2O remains for how many year in the Number of Neutron 8 10
atmosphere? [Maharashtra 2018] Select the correct statement about X and Y.
(1) 100 (2) 114 (3) 104 (4) 109 (1) X and Y are isobars
131. The German silver, an alloy, has the composition (2) X and Y have different chemical properties
[Delhi 2017] (3) X and Y have different physical properties
(1) Cu + Sn + Zn (2) Cu + Zn + Ni (4) X and Y are the atoms of different elements
(3) Cu + Ag + Zn (4) Ag + Hg + Sn 143. How many electrons are present in M-shell of an element
132. Out of the following, which is the incorrect statement? with atomic number 20? [Maharashtra 2017]
[Delhi 2017] (1) 8 (2) 6 (3) 18 (4) 2
S-16 Target NTSE
144. Which gas emits on burning of rice straw? (1) CaO < Na2O < CO2 < ZnO
[Maharashtra 2017] (2) Na2O < CaO < ZnO < CO2
(1) SO2 (2) NH3 (3) O3 (4) H2S (3) ZnO < CO2 < CaO < Na2O
145. The chemical reactions and its corresponding observable (4) CO2 < ZnO < Na2O < CaO
features are matched below. The correct option is 151. The substance which is chemically resistant, and can hold
aqua regia: [Karnataka 2016]
[Karnataka 2017]
(1) Ceramics (2) Glass
A. Change in temperature i. Magnesium reacting (3) Fibre (4) Thermosetting plastic
with dilute sulphuric 152. In saponification process, the fatty acid present in the oils
acid is neutralised by adding [Karnataka 2016]
B. Evolution of a gas ii. Potassium iodide (1) Sodium hydroxide (2) Aluminium hydroxide
reacting with lead (3) Calcium hydroxide (4) Magnesium hydroxide
nitrate 153. This is not a characteristic of members of a homologous
C. Formation of a iii. Sulphur dioxide gas series: [Karnataka 2016]
precipitate reacting with acidified (1) They possess varying chemical properties
potassium dichromate (2) Their properties vary in regular and predictable
manner.
solution (3) Their formulae fit the general molecular formula
D. Change in colour iv. Zinc granules reacting (4) Adjacent members differ by one carbon and two
with dilute sulphuric hydrogen atoms.
acid 154. The electronic configuration of copper can be represented
(1) A - iii, B - iv, C - i, D - ii in this/these way/ways: [Karnataka 2016]
(2) A - iv, B - i, C - ii, D - iii (1) 1s2 2s2 2p6 3s2 3p6 4s1 3d10
(3) A - i, B - iv, C - iii, D - ii (2) [Ar] 3d10 4s2
(4) A - iv, B - ii, C - iii, D - i (3) 1s2 2s2 2p6 3s2 3p6 4s2 3d9
146. The solution in which an immersed glass exhibits apparent (1) Only 1 & 2 are correct (2) Only 2 is correct
disappearance is a mixture of [Karnataka 2017] (3) Only 2 & 3 are correct (4) Only 1 & 3 are correct
(1) Hydrogen fluoride and acetic acid 155. Fe2O3 + 2Al ® Al2O3 + 2Fe, this reaction is an example of a:
(2) Hydrofluoric acid and sulphuric acid [Uttar Pradesh 2016]
(3) Conc. HCl and HNO3 in the ratio of 3 : 1 (1) combination reaction
(4) Acetone and chloroform in suitable proportion (2) double displacement reaction
(3) decomposition reaction
147. The following substances are arranged in the increasing
(4) displacement reaction
order of their pH values. The correct option is 156. Which one of the following types of medicines is used for
[Karnataka 2017] treating indigestion? [Uttar Pradesh 2016]
A. Tomato juice (1) Antibiotic (2) Analgesic
B. Tooth paste (3) Antacid (4) Antiseptic
C. Saliva (after meals) 157. The main factor of depletion of ozone layer is:
D. Coffee [Uttar Pradesh 2016]
E. Blood (1) chlorofluorocarbons (2) oxygen
(1) B C E A D (3) sulphur (4) nitrogen
(2) B E C D A 158. Methane on combustion gives [Jharkhand 2016]
(3) B A C E D (1) CO2 (2) H2O
(3) both CO2 and H2O (4) Neither CO2 nor H2O
(4) B D E C A
159. Rusting of iron takes place in [Jharkhand 2016]
148. Identify the correct oxidant and reductant in the following (1) ordinary water
reaction: [Delhi 2016] (2) distilled water
PbS + 4H2O2 ¾¾® PbSO4 + 4H2O (3) both ordinary and distilled water
(1) PbS – Oxidant (2) PbS – Reductant (4) none of the above
H2O2 – Reductant PbSO4 – Oxidant 160. Mg dissolves in hot water to form [Jharkhand 2016]
(3) PbS – Reductant (4) H2O2 – Oxidant (1) MgO (2) Mg(OH)2
H2O2 – Oxidant H2O – Reductant (3) MgOH (4) MgOMg(OH)2
149. The pH of pure water at 25°C is 7. If water is heated to boil, 161. Among LiCl, RbCl, BeCl2 and MgCl2 which two are the
then [Delhi 2016] highest and the lowest ionic compounds?
(1) pH will decrease and water will become acidic [West Bengal 2016]
(2) pH will increase and water will become basic (1) LiCl & RbCl. (2) RbCl & BeCl2.
(3) pH will decrease but water will remain neutral (3) RbCl & MgCl2. (4) MgCl2 & BeCl2.
(4) pH will remain 7 162. 10 g H2 and 64 g O2 were taken in a sealed vessel and
exploded. The amount of water produced in the reaction
150. The correct order of acidic strength of the following oxides
was— [West Bengal 2016]
is: [Delhi 2016]
(1) 3 mole. (2) 4 mole. (3) 1 mole. (4) 2 mole.
Stage - I Solved Questions S-17

163. Mass percentage of nitrogen in the compound N2O3 is (1) 42 mg (2) 84 g (3) 0.042 kg (4) 42 mol
[Uttar Pradesh 2015] 169. On addition of which metal the blue coloured copper
(1) 36.84 (2) 46.70 (3) 82.40 (4) 63.60 sulphate solution turns into colourless solution?
164. Which of the following are isomers? [West Bengal 2016] [Rajasthan 2015]
(1) Butane and Isobutene (1) Ag (2) Hg (3) Zn (4) Au
(2) Ethane and Ethene 170. pH of a solution is zero. The nature of this solution is
(3) Propane and Propyne [Rajasthan 2015]
(4) Butane and Isobutane (1) acidic (2) basic
165. Which of the following has shortest carbon-carbon bond (3) neutral (4) amphoteric
length? [Delhi 2015] 171. Graphite is used as a lubricant in machines because it has a
(1) C2H2 (2) C2H4 (3) C2H6 (4) C6H6 very high melting point and also it. [Jharkhand 2015]
166. Which of the following may be isomer of aldehyde having
general formula CnH2nO? [Delhi 2015] (1) is crystalline
(1) Alcohol (2) Ether (2) has layer structure
(3) Ester (4) Ketone (3) is a giant molecule
167. Which of the following orders of atomic radii is correctly (4) is a liquid at room temperature
represented? [Maharashtra 2015] 172. Which of the following oxyacid of phosphorus are
(1) B < Al < Ga (2) B < Ga < Al monobasic (monoprotic)? [Jharkhand 2015]
(3) Al < B < Ga (4) B > Ga > Al
(1) H3PO4 (2) H3PO3 (3) H3PO2 (4) H4P2O7
168. A sample of MgCO3 contains 3.01 × 1023 Mg2+ ions and
3.01 × 1023 CO32 - ions. The mass of the sample is
[Maharashtra 2015]

SECTION 3. BIOLOGY
1. Match the following : (AP_FIITJEE 2020-21) (1) Prothrombin (2) Fibrinogen
Group - A Group - B (3) Thrombokinase (4) Thrombin
(i) Ovary (A) development of male sex 6. The disease that affects our lungs is - (Bihar 2020-21)
organs (1) AIDS (2) Rabies
(ii) Pituitary gland (B) increases heart beat rate (3) Polio (4) Tuberculosis
(iii) Testis (C) menstrual cycle 7. Which of the following is not a plant hormone?
(iv) Adrenal gland (D) growth of bones (Bihar 2020-21)
(1) (i)-(C), (ii)-(D), (iii)-(A), (iv)-(B) (1) Auxin (2) Florigen
(2) (i)-(D), (ii)-(C), (iii)-(A), (iv)-(B) (3) Cytokinin (4) Oxytocin
(3) (i)-(C), (ii)-(D), (iii)-(B), (iv)-(A) 8. Which of the following composition represents energy rich
(4) (i)-(A), (ii)-(D), (iii)-(B), (iv)-(C) food? (Bihar 2020-21)
2. Identify the right statement about Abscisic acid (1) Vitamins and minerals
(AP_FIITJEE 2020-21) (2) Carbohydrates and fats
(1) It helps in ripening of fruits
(3) Water and roughage
(2) It promotes cell division
(3) It promotes seed dormancy (4) Proteins and mineral salts
(4) It causes stem elongation in plants 9. Which of the following is not a natural resources?
3. Identify the mismatched one (AP_FIITJEE 2020-21) (Bihar 2020-21)
(1) Tridax - Pyrethroids (2) Datura - Cocaine (1) Soil (2) Water
(3) Azardiachta - Nimbin (4) Papaver - Morphine
(3) Electricity (4) Air
4. Plasma membrane of a cell is made up of
(AP_FIITJEE 2020-21) 10. Pure Bred Pea Plant A is crossed with pure bred pea plant B.
(1) Proteins and Carbohydrates It is found that the plants which look like A do not appear in
(2) Proteins and Lipids F1 generation but re-emerge in F2 generation, which of the
(3) Carbohydrates and Lipids plants A and B are tall and dwarf? (Bihar 2020-21)
(4) Lipids and Nucleic acids (1) A are tall and B are also tall
5. When the blood flows out through a wound, the platelets (2) A are tall and B are dwarf
release an enzyme called _____ to stop the blood flow. (3) A are dwarf and B are tall
(AP_FIITJEE 2020-21) (4) A are dwarf and B are also dwarf
S-18 Target NTSE
11. UV rays cause cancer but in stratosphere the same UV rays (1) Oestrogen (2) Thyroxin
are helping us, how? (Delhi 2020-21) (3) Insulin (4) Testosterone
(1) They divert harmful UV rays back to sun. 20. Pair of an organism and its asexual method of reproduction
(2) They convert oxygen in stratosphere into ozone. is given below. Choose the correct pair :
(3) UV rays are not present in stratosphere. (Karnataka 2020-21)
(4) UV rays reach the earth surface then bounce back (1) Amoeba - Regeneration
carrying ozone to stratosphere. (2) Bryophyllum - Vegetative reproduction
(3) Yeast - Fragmentation
12. Match the following : (Delhi 2020-21) (4) Leishmania - Budding
Column-I Column-II 21. ______ is most clever animal among all non-chordates and
(i) Regeneration (a) Shoot which can change its colour. (Maharashtra 2020-21)
(ii) Rhizophus (b) Pollen grain (1) Octopus (2) Lizard (Chameleon)
(3) Snail (4) Balanoglossus
(iii) Plumule (c) Vegetative Propagation
22. Which is effective antibiotics against tuberculosis?
(iv) Rose (d) Planaria (Maharashtra 2020-21)
(v) Stigma (e) Spores (1) Cephalosporins (2) neomycin
(1) (i)-(a); (ii)-(e); (iii)-(d); (iv)-(b); (v)-(c) (3) streptomycin (4) rifamycin
(2) (i)-(b); (ii)-(d); (iii)-(a); (iv)-(c); (v)-(e) 23. Glucose and fructose syrup can be obtained from cornflour
(3) (i)-(b); (ii)-(a); (iii)-(d); (iv)-(c); (v)-(e) by action of enzymes obtained from ___________
(4) (i)-(d); (ii)-(e); (iii)-(a); (iv)-(c); (v)-(b) (Maharashtra 2020-21)
13. The opening and closing of the stomata depends upon - (1) Brevibacterium and Corynebacterium
(Delhi 2020-21) (2) Hansenula and lacto-bacillus brevis
(1) Oxygen (2) Temperature (3) Saccharomyces cerevisine and Candida
(3) Carbon dioxide (4) Water in guard cells (4) Bacillus and Streptomyces
14. Match the Column-I and Column II and select correct option. 24. Consumption of tobacco products lead to which disease in
(Delhi 2020-21) human beings? (Maharashtra 2020-21)
Column-I Column-II (1) tuberculosis (2) AIDS
(A) Ribosome (I) ATP formation (3) cancer of the lungs (4) leprosy
(B) Mitochondria (II) Photosynthesis 25. Amongst the following which disease spread through
(C) Centriole (III) Protein synthesis bacteria? (Maharashtra 2020-21)
(D) Chloroplast (IV) Cell division (1) Hepatitis (2) Pneumonia
(1) (A)-(I); (B)-(II); (C)-(IV); (D)-(II) (3) Influenza (4) Chicken pox
(2) (A)-(III); (B)-(I); (C)-(IV); (D)-(II) 26. In which of the plant group chitineous cell wall is found
(3) (A)-(IV); (B)-(III); (C)-(II); (D)-(I) (Rajasthan 2020-21)
(4) (A)-(II); (B)-(I); (C)-(III); (D)-(IV) (1) Algae (2) Fungi
15. Which of the following is a barrier method of contraception? (3) Thallophyta (4) Bryophyta
(Delhi 2020-21) 27. Lack of which element occurs when Algal Bloom is formed
(1) Diaphragm (2) Contraceptive pills in a waterbody. (Rajasthan 2020-21)
(3) Tubectomy (4) All of the above (1) Oxygen (2) Nitrogen
16. Sperms are produced in the (Delhi 2020-21) (3) Hydrogen (4) Calcium
(1) Seminiferous tubules (2) Interstitial cell 28. The plant group called "pollution indicator" is
(3) Vas deferens (4) Prostate gland (Rajasthan 2020-21)
17. The function of finger like projections called 'villi' in small (1) Bryophyta (2) Lichen
intestine is to : (Karnataka 2020-21) (3) Gymnosperm (4) Pteridophyta
(1) Supply the digested food to each and every cell of the 29. Which of the following is not a member of phylum
body Arthropoda (Rajasthan 2020-21)
(2) Separate the water from undigested food (1) House-fly (2) Earthworm
(3) Convert the glucose into carbohydrates (3) Shrimp (4) Cockroach
(4) Increase the surface area for absorption of digested 30. DNA synthesis occurs, in which stage of cell cycle.
food (Rajasthan 2020-21)
18. Because of lack of oxygen in muscle cells, 3-carbon molecule (1) G-I phase (2) G-II phase
pyruvate is converted into : (Karnataka 2020-21) (3) M-phase (4) S-phase
(1) Lactic acid (2) Ethanoic acid 31. Example of Fat digesting enzyme is (Rajasthan 2020-21)
(3) Hydrochloric acid (4) Acetic acid (1) Amylase (2) Pepsin
19. Identify the relationship between the first two words and suggest (3) Lipase (4) Nucleases
the suitable word for the fourth place related to the third word. 32. Hydroponics was demonstrated by a German Botanist
Adrenal gland : Adrenalin : : Pancreas : ____________ ______ in 1980. (Tamil Nadu 2020-21)
(Karnataka 2020-21) (1) Julius Von Sachs (2) Nehemiah Grew
(3) Robin Hill (4) Robert Brown
Stage - I Solved Questions S-19

33. Which is not a component of stele?(Tamil Nadu 2020-21) (1) Chicken pox (2) Malaria
(1) Pitch (2) Pericycle (3) Tuberculosis (4) Tetanus
(3) Cambium (4) Vascular tissue 45. Lysosome stores (West Bengal 2020-21)
34. Choose the incorrect statement. (Tamil Nadu 2020-21) (1) ATP (2) Hydrolytic enzymes
(1) Epithelium helps in absorption of nutrients. (3) Carbohydrate (4) Protein
(2) Epithelium allows the normal movement. 46. Cardiac muscle is (West Bengal 2020-21)
(3) Epithelium helps in elimination of waste products. (1) striated and voluntary (2) smooth and voluntary
(4) Epithelium secrets chemical substances. (3) striated and involuntary(4) smooth and involuntary
35. Find out the true and false statements. 47. The part of human alimentary canal where no enzyme
(Tamil Nadu 2020-21) secretion takes place is (West Bengal 2020-21)
(a) A recessive factor is altered by the presence of a dominant (1) Mouth (2) Oesophagus
factor. (3) Stomach (4) Ileum
(b) Dominant factor is expressed in both F1 and F2 generations. 48. The immunoglobulin which is transported to the foetus
(c) Recessive factor is expressed only in F1 generation of through placenta from mother is (West Bengal 2020-21)
a monohybrid cross. (1) lgG (2) lgA (3) lgE (4) lgM
(d) Gregor Mendel conducted his experiment on bean plant. 49. A flagellum is present at one end of a protozoan. It is
(1) (a) false, (b) false, (c) true, (d) false (Delhi 2019)
(2) (a) true, (b) false, (c) true, (d) true (1) Planaria (2) Paramecium
(3) (a) true, (b) true, (c) false, (d) false (3) Hydra (4) Leishmania
(4) (a) true, (b) false, (c) false, (d) false 50. The wings of house fly and the wings of a sparrow are an
36. Assertion (A) : RBC plays an important role in the transport example of (Delhi 2019)
of respiratory gases. (1) Analogous organs (2) Vestigial organs
Reason (R) : RBC do not have cell organelles and nucleus. (3) Respiratory organs (4) Homologous organs
(Tamil Nadu 2020-21) 51. Cell division in plants is promoted by : (Delhi 2019)
(1) Both (A) and (R) are true and (R) is correct explanation (1) Abscisic acid (2) Gibberllin
of (A) (3) Ethylene (4) Cytokinin
(2) Both (A) and (R) are true but (R) is not the correct 52. Flight and fight hormone is: (Delhi 2019)
explanation of (A) (1) Adrenaline (2) Thyroxin
(3) (A) is true but (R) is false (3) Oxytocin (4) Insulin
(4) Both (A) and (R) are false 53. ATP is formed by photosynthesizing plant cell by :
37. Which organele cell is called power house of cell? (Delhi 2019)
(Uttar Pradesh 2020-21) (1) Photophosphorylation
(1) Mitochondria (2) Chloroplast (2) Oxidative Phosphorylation
(3) Ribosome (4) Lysosome (3) Substrate level phosphorylation
38. Nematoblast or stinging cells are found in which phylum of (4) All of the above
animals (Uttar Pradesh 2020-21) 54. Number of male gametes in the growing pollen tube is
(1) Porifera (2) Annelida (Rajasthan 2019)
(3) Cnideria (4) Arthopoda (1) one (2) two (3) three (4) seven
39. In which organ, bile juice formation takes place? 55. Bacterial disease is (Rajasthan 2019)
(Uttar Pradesh 2020-21) (1) Dengue (2) Poliomyelitis
(1) Liver (2) Gall bladder (3) Tuberculosis (4) Chicken pox
(3) Pancreas (4) Stomach 56. Which of the following is not a secondary reproductive
40. Where, glycolysis occurs in cell (Uttar Pradesh 2020-21)
organ? (Rajasthan 2019)
(1) In Mitochondria (2) In Chloroplast
(3) In Cytopasm (4) In Nucleus (1) Fallopian tube (2) Uterus
41. Which plant hormone causes apical dominance? (3) Ovary (4) Vagina
(Uttar Pradesh 2020-21) 57. In a practical laboratory, a student while observing the slide
(1) Auxine (2) Gibbereline of tissue with the help of a microscrope, found a bunch of
(3) Cytokinine (4) Ethylene cylindrical shaped cells having interconnections belong to
42. Cholera diseases caused by which pathogen- the category of : (Chandigarh 2019)
(Uttar Pradesh 2020-21) (1) Adipose tissue (2) Heart muscle
(1) Virus (2) Bacteria (3) Fungus (4) Protozoa (3) Smooth muscle (4) Skeletal muscle
43. In human eye, at the blind spot (West Bengal 2020-21) 58. If a potted plant and a dish containing potassium hydroxide
(1) Only rod cells are present are covered by a sealed container, made up of glass, are kept in
(2) Only cone cells are present sunlight for a week, what will happen (Chandigarh 2019)
(3) Both rod and cone cells are present (1) Plant will grow taller
(4) Neither rod nor cone cells are present (2) Leaf will turn yellow due to no photosynthesis
44. The disease which usually spreads through cuts and wounds (3) Leaf will turn green due to excess photosynthesis
is (West Bengal 2020-21) (4) Leaf will turn yellow due to no oxygen in the jar
S-20 Target NTSE
59. The recessive character in pea plant in the following 69. What is the time of rest in the heart? [Delhi 2018]
(Karnataka 2019) (1) Never
(1) Violet flower (2) Axillary flower (2) While sleeping
(3) Round seed (4) Green seed (3) Between two beats
60. Haemophilia is more common in males because it is a (4) While doing yogasan
(Karnataka 2019) 70. Lacteal present in the villi of the small intestine :
(1) Recessive character carried by Y chromosome [Delhi 2018]
(2) Dominant character carried by Y chromosome (1) Help to absorb fatty acids and glycerol
(2) Secrete enzymes for digestion
(3) Dominant trait carried by X chromosome
(3) Secrete hormones
(4) Recessive trait carried by X chromosome
(4) Help to absorb proteins
61. By using only one of the two strands of DNA, mRNA is
71. How primitive life might have originated on earth was
produced the process called as _______
experimentally shown by [Delhi 2018]
(Maharashtra 2019)
(1) Urey and Miller (2) Watson and Crick
(1) transcription (2) translation
(3) Oparin and Haldane (4) Hershey and Chase
(3) translocation (4) replacement
72. Bicuspid valve is present in the human heart in between
62. Which is the sequence of four whorls of flower from
outside to inside? (Maharashtra 2019) which of the following [Delhi 2018]
(1) Calyx ® Corolla ® Androecium ® Gynoecium (1) Right atrium and right ventricle
(2) Left atrium and left ventricle
(2) Gynoecium ® Androecium ® Corolla ® Calyx
(3) Right and left atria
(3) Calyx ® Androecium ® Corolla ® Gynoecium (4) Left atrium and systemic aorta
(4) Gynoecium ® Corolla ® Androecium ® Calyx 73. Which of the following products of light dependent phase
63. Identify the correct sequence for process of energy are used during the light independent phase of photosyn-
production from carbohydrates. (Maharashtra 2019) thesis? [Delhi 2018]
(1) Carbohydrates ® Glycolysis ® Pyruvic acid ® Acetyl (1) RUBP and ATP (2) H2O and O2
CoA ® Krebs cycle ® CO2 + H2O + energy (3) NADPH and ATP (4) ATP and O2
(2) Carbohydrates ® Glycolysis ® Pyruvic acid ® Krebs 74. Grafting in monocot plants is not possible because they
cycle ® Acetyl CoA ® CO2 + H2O + energy have [Delhi 2018]
(1) Parallel venation
(3) Carbohydrates ® Glycolysis ® Acetyl CoA ® Pyru-
vic acid ® Krebs cycle ® CO2 + H2O + energy (2) Have only one cotyledon
(3) Have cambium
(4) Carbohydrates ® Glycolysis ® Acetyl CoA ® Krebs
(4) Have scattered vascular bundles
cycle ® Pyruvic acid ® CO2 + H2O + energy
75. Haemophilia disease is linked with [Delhi 2018]
64. Identify the correct statements about blood.
(Karnataka 2019) (1) Sex chromosome (2) Autosome
A. Platelets are produced in the bone marrow. (3) Bacteria (4) Virus
B. When haemoglobin combines with oxygen it forms 76. They primary building blocks of DNA are [Delhi 2018]
carboxyhaemoglobin. (1) Nitrogenous base, phosphorus and ribose
C. Calcium ions play an important role in clotting of blood. (2) Nitrogenous base, Sulphur and deoxyribose
D. Fibrins are formed by the conversion of fibrinogen by (3) Nitrogenous base, phosphorus deoxyribose
the enzyme thrombin. (4) Nitrogenous base, sulphur and ribose
(1) A and B only (2) B, C and D only 77. Which of the following helps in formation of insulin?
(3) B and D only (4) A, C and D only [Delhi 2018]
65. Which of the following carry hereditary characters to the off (1) Islets of Langerhans (2) Pituitary gland
spring in the organism? [Delhi 2018] (3) Thyroid gland (4) Adrenal gland
(1) Ribosome (2) Chromosome 78. Which tissue is found in fibrous covering of coconut?
(3) Plasma (4) Lysosome [Rajasthan 2018]
66. Which organelle of the cell is called the power house of the (1) Parenchyma (2) Collenchyma
cell? [Delhi 2018]
(3) Sclerenchyma (4) Meristematic tissue
(1) Cell - Wall (2) Nucleus
79. Nucleus of the cell was discovered by [Rajasthan 2018]
(3) Mitochondria (4) Complete cell
67. Plasma membrane is made up of [Delhi 2018] (1) Robert Hooke (2) Leeuwenhoek
(1) Protein (2) Lipid (3) Robert Brown (4) Virchow
(3) Carbohydrate (4) Both (1) and (2) 80. Which of the following is a plant hormone?
68. Which of the following is the site of fertilisation in [Rajasthan 2018]
humans? [Delhi 2018] (1) Insulin (2) Thyroxine
(1) Uterus (2) Oviduct (3) Cytokinin (4) Oestrogen
(3) Ovary (4) Vagina 81. Plant group more sensitive to the levels of sulphur dioxide
in air is [Rajasthan 2018]
Stage - I Solved Questions S-21

(1) Thallophyta (2) Lichen 99. The total no. of pairs of chromosomes in human beings are
(3) Pteridophyta (4) Gymnosperm ........................... [Maharashtra 2018]
82. Examples of perennial, evergreen and woody plants are (1) 22 (2) 23 (3) 46 (4) 44
[Rajasthan 2018]
(1) Funaria, Marchantia (2) Marsilea, Horse-tail Column A Column B
(3) Cycas, Pinus (4) Ulothrix, Spirogyra (i) Darwin (a) heritability of acquired characteristics
83. Turgidity of cell is maintained by [Rajasthan 2018] 100. (ii) Lamarck (b) inheritance
(1) Vacuole (2) Lysosome
(3) Plastid (4) Golgi body (iii) Mendel (c) natural selection
84. The substance not essential for photosynthesis is
[Maharashtra 2018]
[Rajasthan 2018]
(1) sunlight (2) chlorophyll (1) (i) – (c), (ii) – (a), (iii) –(b)
(3) nitrogen (4) carbon dioxide (2) (i) – (b), (ii) – (c), (iii) –(a)
85. The nature of nerve impulse is [Rajasthan 2018] (3) (i) – (b), (ii) – (a), (iii) –(c)
(1) chemical (2) magnetic (4) (i) – (a), (ii) – (c), (iii) –(b)
(3) electrochemical (4) electromagnetic 101. Which plant does not belong to group Thallophyta
86. The example of uricotelic animals is [Rajasthan 2018] [Maharashtra 2018]
(1) fishes (2) reptiles (1) Ulothrix (2) Spirogyra
(3) amphibians (4) mammals (3) Chara (4) Funaria
87. According to Mendel in monohybrid cross the genotypic
102. The excretory product in crystalline form of the plants
ratio of F2 generation is [Rajasthan 2018]
(1) 3 : 1 (2) 9 : 3 : 3 : 1 ................ causes itching [Maharashtra 2018]
(3) 1 : 1 (4) 1 : 2 : 1 (1) Phyroid (2) Raphide (3) Graphite (4) Cyanide
88. Example of connective tissue is [Rajasthan 2018] 103. From different areas of the brain which is vision area?
(1) cartilage (2) skeletal muscles [Maharashtra 2018]
(3) skin of animals (4) nerve cells A
89. The example of egg laying mammal is [Rajasthan 2018] B
(1) Bat (2) Kangaroo C
(3) Pigeon (4) Echidna D
90. Non-communicable disease is [Rajasthan 2018]
(1) Cancer (2) AIDS (1) A (2) B (3) C (4) D
(3) Amoebiasis (4) Jaundice 104. In female reproductive system ovaries secrete ............... hor-
91. Animals of which phylum are pseudocoelomate? mone. [Maharashtra 2018]
[Rajasthan 2018] (1) testosterone (2) estrogen
(1) Porifera (2) Platyhelminthes (3) auxin (4) thyroxine
(3) Aschelminthes (4) Mollusca 105. Find the odd one out [Maharashtra 2018]
92. Raw material required for photosynthesis is ___________ (1) Adiantum (2) Equisetum
and water. [Maharashtra 2018] (3) Selaginella (4) Riccia
(1) Chloroplast (2) Sunlight 106. Plants normally growing on sand are known as-
(3) Nitrogen (4) Carbon-dioxide [Jharkhand 2018]
93. Find the odd one out [Maharashtra 2018] (1) Lithophytes (2) Xerophytes
(1) Uterus (2) Ovary (3) Vagina (4) Testis (3) Chasmophytes (4) Psammophytes
94. The prescribed limit of sound in decibels in silent zone 107. Our skin becomes dark in colour when exposed to excess of
during daytime is. [Maharashtra 2018] sunlight. It is due to the presence of :
(1) 50 (2) 60 (3) 70 (4) 40 [Jharkhand 2018]
95. A green house gas N2O remains for how many years in the (1) Carotene (2) Melanin
atmosphere? [Maharashtra 2018] (3) Flavoxanthin (4) Haemotoxylene
(1) 100 (2) 114 (3) 104 (4) 109 108. Famous scientist Carolus Linnaeus is associated with one
96. In human being, blood goes through the heart .................... the following : [Jharkhand 2018]
times during each cycle. [Maharashtra 2018] (1) Plant Classification
(1) one (2) three (3) two (4) four (2) Binomial Nomenclature
97. Response to stimulus of touch is called ___________ (3) Identification of plants
[Maharashtra 2018] (4) Identification of Animals
(1) Tropic movement (2) Photo-tropic movement 109. Ozone hole or hole in the ozone layer in the atmosphere
(3) Hydro-tropic movement (4) Seismonastic movement refers to : [Jharkhand 2018]
98. Find the odd one out [Maharashtra 2018] (1) Development of a hole in the Ozone layer
(1) Fragmentation (2) Regeneration (2) Decrease in the Ozone layer in troposphere
(3) Budding in Yeast (4) Budding in Hydra (3) Decrease in the Ozone layer in stratosphere
(4) All of above
S-22 Target NTSE
110. In living cells synthesis of ribonucleic acid (RNA) (1) Lipids > Carbohydrates > ATP
takes place in : [Jharkhand 2018] (2) ATP > Lipids > Protein
(1) Cytoplasm (2) Nucleus (3) Golgi body (4) Nephron (3) Lipids > ATP > Carbohydrates
111. Deficiency of one of the under mentioned vitamins causes (4) Lipids > Proteins > ATP
cracking of lips in human beings : [Jharkhand 2018] 122. The sub units of ribosomes in cells of nephron of mouse are:
(1) Vitamin A (2) Vitamin B2 [Haryana 2018]
(3) Vitamin K (4) Vitamin C (1) 50S & 30S (2) 40S & 23S
112. Insectivorous plants grow only on such soils which are de- (3) 70S & 16S (4) 60S & 40S
ficient in : [Jharkhand 2018] 123. Involuntary muscles are not found in : [Haryana 2018]
(1) Calcium (2) Nitrogen (1) Iris (2) Bronchi of lung
(3) Magnesium (4) Phosphorus (3) Tongue (4) Heart
113. What will happen to the body of an adult human being if his 124. Different microorganisms taking part in nitrogen cycle are:
spleen is removed : [Jharkhand 2018] [Haryana 2018]
(1) RBC production will be reduced (i) Rhizobium in roots (ii) Ammonifying bacteria
(2) Antibodies production will be less (iii) Nitrifying bacteria (iv) Denitrifying bacteria
Which of them strictly work under anaerobic conditions ?
(3) WBC production will be less
(1) only iv (2) i & iv
(4) Filtration of dead RBCs would not be possible
(3) i, ii & iv (4) ii & iv
114. DNA (De-oxyribonucleic acid) is not present in one of the
125. The following pictures were drawn by a student to show
following [Jharkhand 2018]
different stages of binary fission: [Haryana 2018]
(1) Chloroplast
(2) Nucleus
(3) Mitochondria
(4) TMV (Tobacco Mosaic Virus)
115. Due to the discovery of one of the following in 1980, the
evolution was termed as RNA world: [Jharkhand 2018]
(1) RNA present in some viruses as genetic material (i) (ii) (iii) (iv)
(2) RNA has enzymatic property The correct sequence of these figures is:
(3) RNA is found in all living cells (1) iii, ii, iv, i (2) iii, iv, ii, i
(4) RNA is found to be associated with protein synthesis (3) ii, iii, iv, i (4) iv, iii, ii, i
116. In plants, the developing embryo is nourished by 126. Which of the following is not strictly considered as a part of
endospermic tissues its cell consist of : [Jharkhand 2018] neuron? [Haryana 2018]
(1) One genome (Haploid) (1) Dendrite (2) Myelin sheath
(2) Two genomes (Dipioid) (3) Axon (4) Cell body
(3) Three genomes (Triploid) 127. Which of the following statement about autotrophs is in-
(4) Four genomes (Tetraploid) correct? [Haryana 2018]
117. One of the following is not assoicated with gametogenesis : (1) They synthesize carbohydrates from carbon dioxide
[Jharkhand 2018] and water.
(1) Formation of Ova (2) They store carbohydrate in the form of starch.
(2) Formation of sperm (3) They convert water & CO2 into carbohydrate only in
(3) Change of spermatids to spermatozoa the absence of light.
(4) Release of ova (4) They constitute first trophic level in the food chain.
118. The part of biosphere dominated by human beings is known 128. Correct pathway of blood in circulatory system is:
as : [Jharkhand 2018] [Haryana 2018]
(1) Troposphere (2) Hemisphere (1) atria ® ventricles ® artery ® veins
(3) Stratosphere (4) Noosphere (2) ventricles ® atria ® veins ® arteries
119. The excretory organs in Earthworm is known as - (3) ventricles ® veins ® atria ® arteries
[Jharkhand 2018] (4) atria ® arteries ® ventricles ® veins
(1) Malphigian cells (2) Renal cells 129. Which of the following is essential for formation of
(3) Nephridia (4) Flame cells thyroxine hormone in the thyroid gland? [Haryana 2018]
120. Which one of the following is made of only one type of (1) Sodium (2) Chloride
macromolecule? [Haryana 2018] (3) Potassium (4) Iodine
(1) Virus (2) Plasmid 130. In a given food chain if frog has 100 J of energy then the
(3) Nucleosome (4) Ribosome energy available with plants and snake respectively will be:
121. Among carbohydrates, lipids, proteins and ATP, the relative [Haryana 2018]
energy yield in kcal/gm is best represented by: (1) 1000 J and 10 J (2) 10000 J and 10 J
[Haryana 2018] (3) 10 J and 1000 J (4) 1000 J and 100 J
Stage - I Solved Questions S-23

131. Characters that are transmitted form parents to offspring (1) Urea (2) Uric acid
during reproduction show: [Haryana 2018] (3) Ammonia (4) Lymph
(1) Only similarites with parents 146. Identify a fish who breathes air through its lungs:
(2) Only variations with parents [Maharashtra 2017]
(3) Both similarities and variation with parents (1) Lungfish (2) Rohu
(4) Neither similarities nor variation with parents (3) Dogfish (4) Siting Ray
132. Rajiv was absent in class due to muscle pain which he claims 147. A pea plant with yellow and round seeds (YYRR) is crossed
was due to excess of physical [Haryana 2018] with a pea plant having green and wrinkled (yyrr) seeds
(1) Formation of Pyruvic Acid then in F2 generation of this dihybrid cross 320 plants are
(2) Formation of Acetic Acid produced. Out of which plants have same phenotypic
(3) Formation of Lactic Acid characters. Identify this phenotype. [Maharashtra 2017]
(4) Formation of Hydrochloric Acid (1) Yellow and wrinkled seeds
133. Which of the following constitutes a good food chain? (2) Yellow and round seeds
[Haryana 2018] (3) Green and round seeds
(1) Grass, Wheat, Mango (2) Grass, Goat, Lion (4) Green and wrinkled seeds
(3) Goat, Cow, Elephant (4) Grass, Fish, Goat 148. 'Earthworm' a farmer’s friend belongs to .......... phylum.
134. The oxygen rich blood rich from lungs comes to the heart in (1) Arthropoda (2) Echinodermata
[Delhi 2017] (3) Mollusca (4) Annelida
(1) Left atrium (2) Right atrium 149. Read the following statements and select the correct option.
(3) Right ventricle (4) Left ventricle [Karnataka 2017]
135. Growth of pollen tube in the style towards the ovule in A. Wind pollinating flowers need to produce more
plants is an example of [Delhi 2017] quantity of pollen grains
(1) Geotropism (2) Hydrotropism B. Seeds from cross pollinated flowers produce weaker
(3) Phototropism (4) Chemotropism and less healthy plants.
136. The common passage of urine and sperm in human male is (1) 'A' is false 'B' is true (2) 'A' is true 'B' is false
[Delhi 2017] (3) Both 'A' and 'B' are true (4) Neither 'A' nor 'B' are true
(1) Seminal vesicle (2) Ureter 150. Protein catalysts of chemical reactions in biological system.
(3) Vas deferens (4) Urethra [Karnataka 2017]
137. In a cell which cell organelle other than nucleus contains (1) Hormones
DNA? [Rajasthan 2017] (2) Vitamins
(1) Lysosome (2) Golgi bodies (3) Enzymes
(3) Endoplasmic reticulum (4) Mitochondria (4) Both hormones and enzymes
138. Which plant group is called amphibious plants? 151. If pepsin is lacking in gastric juice, then the event in the
[Rajasthan 2017] stomach will be affected: [Delhi 2016]
(1) Algae (2) Fungi (1) Digestion of starch into sugars
(3) Bryophyta (4) Pteridophyta (2) Proteins break down into peptides
139. Enzyme responsible for digestion of protein is (3) Breaking of fats into glycerol and fatty acids
[Uttar Pradesh 2017] (4) Digestion of nucleic acids
(1) ptylin (2) pepsin 152. The part of brain which controls the balance and posture of
(3) amylopsin (4) steapsin the body is [Delhi 2016]
140. Calciferol is [Uttar Pradesh 2017] 1. Cerebellum 2. Cerebrum 3. Pons 4. Medulla
(1) vitamin A (2) vitamin B 153. Which organism will break down the food material into
(3) vitamin C (4) vitamin D simple substances outside the body and then absorb it?
141. Which one of the following does not contain any enzyme? [Maharashtra 2016]
[Odisha 2017] (1) Mushroom (2) Cuscuta
(1) Bile (2) Gastric Juice (3) Ticks (4) worms Tape
(3) Saliva (4) Pancreatic Juice 154. The vegetative reproduction in Bryophyllum takes place
142. Mark the tissue in which the starch is stored in the body of through which organ? [Maharashtra 2016]
plants [Odisha 2017] (1) Root (2) Stem (3) Leaf (4) Seed
(1) Spongy parenchyma (2) Aerenchyma 155. Ginger is [Jharkhand 2016]
(3) Apical meristem (4) Stomata (1) Root (2) Yeast
143. Which one of the following tissues contains stone cells? (3) Plasmodium (4) Leishmania
[Odisha 2017] 156. The liver stores food in the form of [Jharkhand 2016]
(1) Parenchyma (2) Collenchyma (1) Glucose (2) Glycogen
(3) Sclerenchyma (4) Tracheids (3) Albumen (4) ATP
144. Which of the following is the correct scientific name of 157. The centre of sense of smell in brain is [Jharkhand 2016]
man? [Odisha 2017] (1) Midbrain (2) Olfactrory lobes
(1) Homo Sapiens (2) Homo sapien (3) Cerebellum (4) Cerebrum
(3) Homosapien (4) Homo sapiens 158. Phototropic and geotropic response of a plant is under
145. Which of the following harmful products is not produced control of following hormone: [Delhi 2015]
in the biochemical reactions of the cell of living organisms? (1) Auxin (2) Gibberlin
[Maharashtra 2017] (3) Cytokinin (4) Ethylene
S-24 Target NTSE
159. The following blood vessel does not contain deoxygenated (1) Accumulation (2) Eutrophication
blood: [Delhi 2015] (3) Pollution (4) Bio-magnification
(1) Pulmonary artery (2) Vena Cava 163. Phototropic and geotropic response of a plant is under
(3) Hepatic vein (4) Pulmonary Vein control of following hormone: [Maharashtra 2015]
160. During lack of oxygen in tissues of our body, the pyruvate is (1) Auxin (2) Gibberlin (3) Cytokinin (4) Ethylene
converted into lactic acid in: [Delhi 2015] 164. Which scientist discovered "Tricarboxylic acid"?
(1) Mitochondria (2) Nucleus [Madhya Pradesh 2015]
(3) Cytoplasm (4) Ribosome (1) Krab (2) Watson and Crick
161. In Xerophytes, the rate of water loss get reduced due to (3) Sanger (4) Edison
covering of epidermis by [Maharashtra 2015] 165. A person is excreting about10 litres of urine per day. Which
(1) Cutin (2) Suberin (3) Lignin (4) Gum of the following endocrine gland is responsible for this?
162. The accumulation of non-biodegradable substances in a [Odisha 2015]
food chain in increasing amount at each higher trophic level (1) Pituitary (2) Thyroid (3) Parathyroid (4)Adrenal
is known as [Maharashtra 2015]

SECTION 4. HISTORY
1. Who was the ruler of France during the French revolution? (3) A - iv, B - iii, C - ii, D - i (4) A - iv, B - ii, C - i, D – iii
(Delhi 2020-21) 9. Bastille prison was destroyed in France on:
(1) Louis XV (2) Louis XIV (Andhra Pradesh-2020-21)
(3) Louis XX (4) Louis XVI (1) June 20th 1778 (2) May 5th 1786
2. Hitler was born in (Delhi 2020-21) (3) July 14th 1789 (4) July 4th 1798
(1) France (2) Austria 10. This national leader fasted on the First Independence Day
(3) Germany (4) Poland of the Nation. He said, “If I am to die by the bullet of a mad
3. Who published ‘The folklores of southern India’? man, I must do so smiling.” Who was he?
(Delhi 2020-21) (1) Jawaharlal Nehru (Andhra Pradesh-2020-21)
(1) Rabindra Nath Tagore (2) Natesa Sastri (2) Gandhiji
(3) Mahatma Gandhi (4) Pattabhi Srimalu (3) Subhash Chandra Bose
4. Who formed the secret society “young Italy”? (4) Sardar Vallabhbhai Patel
(Delhi 2020-21) 11. Consider the following statements and select the CORRECT
(1) Otto Van Bismarck (2) General Wan Run answer using the codes given below.
(3) Giuseppe Mazzini (4) King Victor Emmanual II (Andhra Pradesh-2020-21)
5. Which of the following was not related to the Gandhi Irwin (A) The first printing press was made by Johannes Gutenberg.
Pact 1931? (Delhi 2020-21) (B) Paper and printing with blocks were first developed
(1) Government agreed to release the Political prisoners. by the Chinese.
(2) Gandhi Ji consented to participate in the Second Round (C) The first printed book in the press was “The Bible”
Table Conference. Codes:
(3) Formation of Simon Commission. (A) (A), (B) and (C) are correct
(4) Gandhi Ji called off Civil Disobedience Movement. (2) (A) and (B) are correct
6. In Swaraj Flag (1921), Self Help was represented by (3) (B) and (C) are correct
(Delhi 2020-21) (4) (A) and (C) are correct
(1) Flower (2) Crescent Moon 12. Oceans in descending order in terms of their size:
(3) Two Ox (4) Spinning wheel (Andhra Pradesh-2020-21)
7. Which one of the following pairs is NOT correctly matched? (1) Pacific, Atlantic, Indian, Antarctic, Arctic
(Andhra Pradesh-2020-21) (2) Pacific, Atlantic, Indian, Arctic, Antarctic
(1) Michelangelo – Statue of David (3) Pacific, Indian, Atlantic, Arctic, Antarctic
(2) Leonardo da Vinci – Mona Lisa (4) Pacific, Arctic, Atlantic, Indian, Antarctic
(3) Filippo Brunelleschi – The Last Supper 13. Who among the following proposed a social contract
(4) Albrecht Durer – Praying Hands between people and their representatives?
8. Match List-I with List-II and select the correct answer using (Andhra Pradesh-2020-21)
the codes given below. (Andhra Pradesh-2020-21) (1) Rousseau (2) John Locke
List-I (Author) List-II (Book) (3) Thomas Jefferson (4) Montesquieu
A. Machiavelli (i) Praise of Folly 14. “God save our Noble King” was/is the National Anthem of
B. Montesquieu (ii) Utopia (Andhra Pradesh-2020-21)
C. Thomas More (iii) The spirit of Laws (1) Britain (2) Italy
D. Erasmus (iv) The prince (3) Germany (4) France
Codes: 15. The first language state was formed in Independent India:
(1) A - i, B - ii, C - iv, D - iii (2) A - i, B - ii, C - iii, D - iv (West Bengal-2020-21)
Stage - I Solved Questions S-25

(1) Andhra Pradesh (2) West Bengal 27. Which of the following was first invented?
(3) Tamil Nadu (4) Gujarat (Rajasthan-2020-21)
16. The writer of the book named “Chhere Asha Gram” was (1) Flying Shuttle Loom (2) Water Frame
(West Bengal-2020-21) (3) Spinning Jenny (4) Powerloom
(1) Manikuntala Sen (2) Dakshina Ranjan Basu 28. Which dynasty succeeded the Bourbon dynasty after the
(3) Sankha Ghosh (4) Selina Hossain 1830 revolution? (Bihar-2020-21)
17. Who was known as the “Tsar the Liberator”? (1) Habsburg (2) Orleans
(West Bengal-2020-21) (3) Tsardom (4) None of these
(1) Tsar Nicholas I (2) Tsar Nicholas II 29. Charles Albert was the King of which country?
(3) Tsar Alexander I (4) Tsar Alexander II (Bihar-2020-21)
18. The day 24th October 1929 was marked as ‘Black Thursday’ (1) Naples (2) Parma
in the USA because (West Bengal-2020-21) (3) Modena (4) Sardinia
(1) Terrorist Attack 30. Who was the author of ‘War and Peace”? (Bihar-2020-21)
(2) Natural Calamity (1) Tolstoy (2) Karl Marx
(3) The Great Economic Depression (3) Lenin (4) St. Simon
(4) Change of Political Background 31. Who built the Angkorwat Temple? (Bihar-2020-21)
19. France was named as “a museum of economic errors” by (1) Jayavarman (2) Suryavarman II
(West Bengal-2020-21) (3) Mahendravarman (4) Rudravarman
(1) Rousseau (2) Adam Smith 32. In 1878 which Viceroy passed the “Vernacular Press Act”?
(3) Montesquieu (4) Quesnay (Bihar-2020-21)
20. Jagannath Singh Dhol was the leader of (1) Lord Ripon (2) Lord Lytton
(A) Kol Rebellion (West Bengal-2020-21) (3) Lord Curzon (4) Lord Chelmsford
(B) Santhal Rebellion 33. After which incident Rabindranath Tagore surrendered the
(C) Munda Rebellion title “Knight”? (Bihar-2020-21)
(D) Chuar Rebellion Ans (1) Rowlatt Act
The incident of Chauri Choura took place in (2) Khilafat Movement
(1) 1919 AD (2) 1920 AD (3) Jallianwala Bagh Massacre
(3) 1922 AD (4) 1925 AD (4) Coming of Simon Commission
21. Which period is known as “Arab spring”? 34. Who published the newspaper “Som Prakash”?
(Rajasthan-2020-21) (1) Ishwar Chandra Vidyasagar (Bihar-2020-21)
(1) 2010-2013 (2) 2007-2010 (2) Bal GangadharTilak
(3) 2006-2016 (4) 2012-2015 (3) Ram Mohan Roy
22. Arrange the following events is chronological order and (4) M. G Ranade
select the correct answer from the codes given below: 35. Where is Sanchi stupa situated? (Uttar Pradesh-2020-21)
(Rajasthan-2020-21) (1) Gaya (2) Lumbini
(A) Korea War (B) Swej Crisis (3) Samath (4) Bhopal
(C) Congo Crisis (D) Somalia Crisis 36. Which relegion did Ashoka adopt?
Codes: (Uttar Pradesh-2020-21)
(1) A, C, B, D (2) A, B, C, D (1) Buddhism (2) Hinduism
(3) C, B, D, A (4) B, A, D, C (3) Jainism (4) Shaivism
23. Which one of the following was not included among the 37. Which is the oldest language of South India?
Axis nations during the First World War? (Uttar Pradesh-2020-21)
(1) Telugu (2) Kannada
(Rajasthan-2020-21)
(3) Tamil (4) Malayalam
(1) Italy (2) Germany
38. Who among the following had introduced market control
(3) Hungary (4) Turkey
policy? (Uttar Pradesh-2020-21)
24. Who was the writer of “Ananda Math”? (1) Balban (2) Alauddin Khilji
(1) Bal Gangadhar Tilak (Rajasthan-2020-21) (3) Muhammad Bin Tuglaq (4) Jalaluddin Khilji
(2) Bankim Chandra Chatterjee 39. Who among the following founded the Vijay Nagar empire?
(3) Dayanand Saraswati (Uttar Pradesh-2020-21)
(4) Raja Rammohan Roy (1) Vijay Rai (2) Harihar and Bukka
25. Dhapi Dadi is related to which village of Sikar peasant (3) Pushyamitra (4) Rana Sanga
movement? (Rajasthan-2020-21) 40. Which of the following cities was built by Akbar?
(1) Kudan (2) Palthana (Uttar Pradesh-2020-21)
(3) Katrathal (4) Ghassu (1) Daulatabad (2) Fatehpur Sikri
26. Which newspaper compared the Neemuchana massacre to (3) Agra (4) Delhi
the Jallianwala Bagh massacre? (Rajasthan-2020-21) 41. In which year Vasco da Gama came to India?
(1) Rajputana (2) Riyasat (Uttar Pradesh-2020-21)
(3) Rajasthan (4) Kisan (1) 1350 AD (2) 1450 AD
(3) 1498 AD (4) 1598 AD
S-26 Target NTSE
42. Who founded the Indian National Congress? 53. Ishwardas Nagar, Bhimsen Saxena were the histories of
(Uttar Pradesh-2020-21) _________ ‘s times. (Maharashtra-2020-21)
(1) Mahatma Gandhi (2) Queen Victoria (1) Jahangir (2) Aurangzeb
(3) Sardar Patel (4) A. O. Hume (3) Shahjahan (4) Akbar
43. Which among the following is the autobiography of 54. Identify the style of painting that influenced Maratha style
Gandhiji? (Uttar Pradesh-2020-21) of painting. (Maharashtra-2020-21)
(1) India Divided (2) Nation in Making (1) Art style (2) Varli Kala
(3) Neel Darpan (4) MyExperiments with Truth (3) Rajput Kala (4) Classical Art
44. Which one of the following is the oldest mountain system? 55. Newspaper ________ began to publish articles about the
(Uttar Pradesh-2020-21) nationwide situations, books in the native languages and
(1) Nilgiri (2) Aravali the politics in other countries. (Maharashtra-2020-21)
(3) Satpura (4) Vindhya (1) Maratha (2) Kesari
45. Who among the following was the first Peshwa of Shahu? (3) Deenbandhu (4) Indu Prakash
(Tamil Nadu-2020-21) 56. Who was the editor of the periodical name ‘Pragati’ (1929)?
(1) Balaji Vishwanath (2) Baji Rao (1) Pandit Narendra Sharma (Maharashtra-2020-21)
(3) Balaji Baji Rao (4) Madhava Rao (2) Balshastri Jambhekar
46. Before French Revolution the political body of France was (3) Traymbak Shankar Shejwalkar
called as ___________ (Tamil Nadu-2020-21)
(4) Gopal Hari Deshmukh
(1) Duma (2) Reich Stag
57. Identify the wrong pair related to the movie and its
(3) Lok Sabha (4) Estates General
directions. (Maharashtra-2020-21)
47. The war between Britain and France had centred on the
(1) Bal Shivaji – Prabhakar Pendharkar
control of colonies in North America
(Tamil Nadu-2020-21) (2) Dhanyate Santaji Dhanaji – Diankar D. Patil
(1) The American war of Independence (3) Bajirao Mastani – Bhalji Pendharkar
(2) Seven years war (4) Baji Prabhu Deshpande – Dadasaheb Torane
(3) The Crusades 58. Which game was not mentioned in ancient Indian literature
(4) Battle of Bunker Hill and the EPICS? (Maharashtra-2020-21)
48. Arrange the following events in the chronological order (1) dice (yet) (2) horse and the chariot races
(A) Vietnam Nationalist Party (Tamil Nadu-2020-21) (3) boxing (4) Chess
(B) Fascist Party 59. Identify the well-known play of Shakespeare on which
(C) The Union of South Africa Marathi play ‘Natsamrat’ was styled?
(D) French Communist Party (Maharashtra-2020-21)
(1) (A), (B), (C), (D) (2) (A), (D), (C), (B) (1) Hamlet (2) King Lear
(3) (C), (B), (D), (A) (4) (C), (B), (A), (D) (3) Julius Ceaser (4) Romeo-Juliet
49. Match the following (Tamil Nadu-2020-21) 60. Emperor Menelik was the ruler of which African Country?
Column – I Column– II (Maharashtra-2020-21)
(A) Hundred Days (i) 1832 (1) Ethiopia (2) Cape Colony
of Reform (3) Kenya (4) Uganda
(B) Russian (ii) 1898 61. The great military genius of the Mongols
Revolution (Maharashtra-2020-21)
(C) First Opium War (iii) 1911 (1) Napoleon Bonaparte (2) Jan Zizka
(D) The Chinese (iv) 1917 (3) Alexander (4) Genghis Khan
Revolution 62. Who is known as ‘Waterman of India’?
(1) (A) – (iv), (B) – (iii), (C) – (ii), (D) – (i) (1) Shree Anna Hajare (Maharashtra-2020-21)
(2) (A) – (iv), (B) – (i), (C) – (ii), (D) – (iii) (2) Sardar Vallabhbhai Patel
(3) (A) – (ii), (B) – (iv), (C) – (i), (D) – (iii) (3) Dr. Rajendra Singh Rana
(4) (A) – (ii), (B) – (iii), (C) – (i), (D) – (iv) (4) Shree Sundarlal Bahuguna
50. Liberty, Equality and Fraternity were the important slogans 63. Choose the correct statements related to the Ryotwari
during the ______ (Tamil Nadu-2020-21) system : (Karnataka-2020-21)
(1) American Revolution (2) French Revolution (A) The charge of collecting the revenue and paying it to
(3) Russian Revolution (4) Chinese Revolution the company was given to the village headman.
51. __________ is not a part of the world network biosphere (B) Alexander Reed introduced it in Baramahal Region.
reserves of UNESCO. (Tamil Nadu-2020-21) (C) Farmer had to pay 50% of the produce as revenue to
(1) Nilgiri (2) Agasthiyamalai the Government.
(3) Great Nicobar (4) Kachch (1) an only (2) B and C only
52. Who argued that the prevailing practices of arranging (3) A and B only (4) A, B and C only
historical events in chronological order are not right? 64. In List - A Mahajanapadas and in List - B their Capitals are
(Maharashtra-2020-21) given. Write the correct option by matching them :
(1) Michel Foucault (2) Karl Marx (Karnataka-2020-21)
(3) Leopold Von Ranke (4) Friedrich Hegel
Stage - I Solved Questions S-27

List - I List - II D. Indian Rural sociology, slums and Urbanisation are


A. Anga i. Saketha his/her major works.
B. Kashi ii. Kushinagara (1) Iravati Karve (2) C. Parvathamma
C. Kosala iii. Champa (3) M.N. Srinivas (4) A.R. Desai
D. Malla iv. Varanasi 70. ‘Rinderpest’ is a term used for (Delhi 2019)
v. Kausambi (1) A cattle disease
(1) A - ii B- i C - iv D - iii (2) A - iii B- iv C - i D - ii (2) Missing of cattle
(3) A-v B-i C - ii D - iv (4) A - ii B- iii C - iv D - v (3) Indentured Labourer
65. Select the correct chronological order of wars : (4) Mass production in a factory
(Karnataka-2020-21) 71. Gudem Rebellion was led by (Delhi 2019)
(1) The first Anglo-Sikh war, The second Anglo-Marata war, (1) Baba Ramchandra (2) Jawaharlal Nehru
The second Anglo-Mysore war, The third Carnatic war. (3) Alluri Sitaram Raju (4) Mahatma Gandhi
(2) The first Anglo-Sikh war, The second Anglo-Mysore 72. The principle of the ‘Garden City’ was developed by
war, The third Carnatic war, The second Anglo-Marata (Delhi 2019)
war. (1) Raymond Unwin (2) Barry Parker
(3) The second Anglo-Marata war, The third Carnatic war, (3) Ebenezer Howard (4) Herbert Baker
The first Anglo-Sikh war, The second Anglo - Mysore 73. Name the two imperialist countries against which the
war. nationalist Vietnamese fought? (Chandigarh 2019)
(4) The third Carnatic war, The second Anglo-Mysore war, (1) France and Britain (2) Britain and Japan
The second Anglo-Marata war, The first Anglo Sikh (3) Japan and France (4) France and Germany
war. 74. Which two new colonial powers joined European powers in
66. Identify the statement that is NOT related to the Bahamani the process of carving up of Africa between themselves at
Kingdom : (Karnataka-2020-21) Berlin in 1885? (Chandigarh 2019)
(1) Asar Mahal was one of the important monuments. (1) Britain and France (2) Italy and France
(2) Mir Syed Ali and Abdul Samad were the artists. (3) Belgium and Germany (4) Britain and Italy
(3) Kitab-E- Navras, a literary work was written.
75. What does Proto-industrialisation mean?
(4) Shahabuddin Ahmed shifted his capital.
(Chandigarh 2019)
67. Read the statements and find out to who do they belong:
(1) The first, and early form of industrialisation
(Karnataka-2020-21)
(2) Industrialisation after 1800 C.E.
i. He took Deeksha from Govindabhatta.
(3) Industrialisation after 1900 C.E.
ii. His Tatvapadas include Kalaga pada, Alaavi pada and
(4) Industrialisation after 2000 C.E.
Sawal Jawab Pada.
76. The location shown on the map of India was a French Colony
(1) Chaithanya (2) Kanakadasa
(Karnataka 2019)
(3) Shishunal Shariffa (4) Guru Nanak
68. The List - A contains the names of poets and List - B with
their works. Select the correct answer from the code given
below. (Karnataka-2020-21)
List-A List - B
A. Sree Purusha i. Gadya Chintamani
B. Vadiba Simha ii. Raghava
Pandaviya
C. Durvinita iii. Gajashastra
D. Hemasena iv. Gajastaka
v. Shabdavathara
Codes: (1) Chandernagore (2) Goa
A B C D (3) Mahe (4) Pondicherry
(1) i iii v iv 77. Select the correct order of events related to freedom struggle
(2) ii v iv iii starting from North to South as shown in the map.
(3) iv i ii v (Karnataka 2019)
(4) iii i v ii
69. Read the given statements and identify the correct N
sociologist related to all of them : (Karnataka-2020-21) i ­
A. He/she worked as the UGC national fellow and as Head
of the Department of Sociology of Bombay University.
A
B. As he/she was following the Marxist ideology in his/ ii
her sociological studies, he/she was identified as a iii
Marxist Sociologist.
C. His/her ‘The Sociological Background of Indian iv
Nationalism’ is the best work.
S-28 Target NTSE
A. Dandi Satyagraha took place List II
B. The Moplah uprising against Bristish (i) 20th June, 1789 (ii) 4th August, 1789
C. The Declaration of Poorna Swaraj (iii) 14th July, 1789 (iv) 5th May, 1789.
D. The Quit India Movement was launched A B C D
(1) i ii iii iv
(1) i-C, ii-A, iii-D, iv-B (2) i-B, ii-D, iii-A, iv-C
(2) iv iii ii i
(3) i-D, ii-C, iii-B, iv-A (4) i-A, ii-B, iii-C, iv-D (3) iv i ii iii
78. The chronological order of the incidents of the first war of (4) i iv iii ii
Indian Independence is (Karnataka 2019) 85. Consider the following Points : [Rajasthan 2018]
A. The Queen of Britain passed a Declaration assuring a (A) Mahatma Gandhi started salt March with his 78
stable Government for Indians. confidential volunteers.
B. A group of soldiers declared Bahadurshah Zafar as the (B) Mahatma Gandhi violated the salt law at Dandi on
emperor of India.
April 20th, 1930.
C. The Sepoys of Meerut revolted against their British Choose the correct answer from the codes given below.
Officers.
(1) only (A) (2) only (B)
D. Tantia Tope was executed by British at Shivapuri.
(3) both (A) and (B) (4) None of these
(1) D C B A (2) B A C D 86. After which war the British rule was founded in India?
(3) A D B C (4) C B A D [Rajasthan 2018]
79. Which the following statements are correct regarding
liberal Nationalism in 19th century Europe? (Haryana 2019) (1) Battle of Sabrao (2) Battle of Panipat
(I) Right of Liberty and Equality (3) Battle of Plassey (4) Second Anglo Mysore war
(II) Formation of people’s govt. 87. When was the Great Economic Depression between the
(III) Ownership of private property two World Wars held ? [Rajasthan 2018]
(IV) Complete control of govt. on all public and private (1) 1921 (2) 1929 (3) 1935 (4) 1939
properties 88. Who composed Ananda Math ? [Rajasthan 2018]
(1) I, II, III (2) IV, III, I (1) Rabindranath Tagore
(3) III, IV, II (4) I, II, IV (2) Munsi Premchand
80. Arrange the following historical developments in a (3) Mahatma Gandhi
chronological sequence. (Haryana 2019)
(4) Bankim Chandra Chattopadhyay.
(I) Poona Act
(II) Lahore congress : demand of ‘Purna Swaraj’ 89. "Imperialism: The Highest stage of Capitalism" was written
(III) Establishment of Oppressed Class Association by by [West-Bengal 2018]
B. R. Ambedkar (a) Lenin (b) Stalin
(IV) Second round table conference (c) Karl Marx (d) Rousseau
(1) I, II, III, IV (2) III, IV, II, I 90. 24th October, 1929 was marked as “Black Thursday” in U.S.A.
(3) II, III, IV, I (4) IV, II, III, I because [West-Bengal 2018]
81. Kheda Satyagrah was related to [Delhi 2018] (a) Terrorist Attack (b) Natural Calamity
(1) Against the oppressive plantation system (c) Great Depression (d) Change in Political aspect
(2) Movement of cotton mill workers 91. During the period of Russian Revolution the Russian ruler
(3) Relaxation in revenue collection was [West-Bengal 2018]
(4) None of the above (a) Czar Alexander–I (b) Czar Alexander–II
82. What do you mean by "Hind Swaraj"? [Delhi 2018] (c) Czar Nicholas–I (d) Czar Nicholas–II
(1) Political Party of Tilak 92. “Flying Shuttle” was invented by [West-Bengal 2018]
(2) Book of Mahatma Gandhi (a) James Hargraves (b) Edmund Cartwright
(3) Symbol of Indian National congress (c) James Watt (d) John Kay
(4) Political Party of Mahatma Gandhi 93. Sardar Vallabhbhai Patel was known as [West-Bengal 2018]
83. The first Historical novel written in Bengal was (a) Saviour of India (b) Modern Man of India
(1) Chemmin (2) Anguriya Binimoy (c) Iron Man of India (d) Machiavelli of India
(3) Chomna Didi (4) Anandmath 94. ‘Communal Awards’ (1932) in India was declared by
84. Match List I with List II correctly and choose the correct [West-Bengal 2018]
code from the following : [Rajasthan 2018] (a) Lord Irwin (b) Ramsay Macdonald
List-I (c) Md. Ali Zinnah (d) Lord Mountbatten
(a) Meeting of the Estates General 95. Pahartali European Club was attacked in 1932 by
(b) Bastille was destroyed on [West-Bengal 2018]
(c) Abolishment of feudal system in France (a) Kalpona Dutta (b) Bina Das
(d) Swore of Tennis Court (c) Pritilata Waddedar (d) Lila Nag
Stage - I Solved Questions S-29

96. “All India Trade Union Congress” (AITUC) was formed in (1) Madras Presidency.
[West-Bengal 2018] (2) Bengal Presidency.
(a) 1915 AD (b) 1920 AD (3) Central Presidency.
(c) 1922 AD (d) 1928 AD (4) Assam Presidency.
97. Bhagath Singh, Jathindas, Batukeshwar and others founded 106. The famous Quit India Resolution was passed on .....
a Revolutionary Association called [Karnataka 2018] [Andhra Pradesh 2018]
(1) Hindusthan Army for Independence (1) August 18, 1942 (2) April 4, 1942
(2) Hindusthan Socialist Republic Army (3) April 14, 1942 (4) August 8, 1942
(3) Hindusthan Socialist Revolutionary Army 107. The Duma were council assemblies which were created by:
(4) Hindusthan Kissan Movement Sabha [Haryana 2017]
98. The Kannada speaking region which came under the rule (1) Lenin (2) Karl Marx
of Madras province in 19th century is [Karnataka 2018] (3) Trotsky (4) Tsar of Russia
(1) Raichur (2) Bellari (3) Bijapura (4) Gulbarga 108. Arthsastra is written by whom? [Haryana 2017]
99. Match personals in List-A with appropriate positions in (1) Kautilya (2) Machiavelli
List-B [Karnataka 2018] (3) Chandragupta Maurya (4) Samudragupta
List-A List-B 109. Who is famous as Deshbandhu [Uttar Pradesh 2017]
a. Shaikh Abdulla I. State Reorganisation Com (1) Chandrashekhar (2) A. O. Hume
mission (3) Chittranjan Das (4) Veer Savarkar
b. Fazal Ali II. Chief of Razakars 110. Who was the Governor General of India in 1857?
c. Sardar Vallabhbhai III. King of Jammu and [Jharkhand 2016]
Patel Kashmir (1) Wellesley (2) Dalhousie
d. Qasim Razvi IV. First Home Minister of
(3) Canning (4) Minto
India
e. Raja Harisingh V. Founder of National 111. Who was the First Indian Woman President in the Indian
Conference National congress ? [Jharkhand 2016]
a b c d e (1) Mrs. Annie Besant (2) Sucheta Kripalani
(1) I V IV III II (3) Sarojini Naidu (4) Indira Gandhi
(2) V I IV III II 112. Who was called the ‘Frontier Gandhi’?
(3) V I II IV III [West Bengal 2016]
(4) V I IV II III (1) Mahatma Gandhi (2) Pattavi Sitaramaiya
100. The correct statement that is related to French East India (3) Vallabhbhai Patel (4) Khan Abdul Gaffar Khan
Company is that it was, [Karnataka 2018] 113. The Autobiography of Hitler is [Delhi 2016]
(1) Private Company (1) Mein Kampf (2) The Secret Book
(2) Private Company controlled by the French Government (3) Nazi Leaflet (4) Gypsy
114. Gandhiji’s “Champaran Satyagrah” was associated with
(3) Officially a State owned comapny
[Delhi 2016]
(4) Controlled by the French Merchant
(1) Rubber Farming (2) Cotton Farming
101. Compulsory Elementary Education Act was made in
(3) Tea Farming (4) Indigo Farming
England in the year.... [Andhra Pradesh 2018]
115. First Newspaper published in India was-
(1) 1829 (2) 1849 (3) 1860 (4) 1870
[Chhattisgarh 2016]
102. Who developed the concept of “The principle of the (1) Om Prakash (2) Sambaddha kaumudi
Garden Citv”? [Andhra Pradesh 2018]
(3) Rast-Goftar (4) Bengal Gazzet
(1) Andrew Means (2) Henry Mayhew
(3) Ebenezer Howard (4) Haussman 116. The Founder of Judaism was- [Chhattisgarh 2016]
103. Who wrote “Ninety five Theses” criticising many of the (1) Abraham (2) Moses
practices and rituals of the Roman Catholic Church ? (3) Zolwastra (4) Augustus
[Andhra Pradesh 2018] 117. Kheda Movement (1917) was started for
(1) Martin Luther (2) Thomas Pain [Maharashtra 2015]
(3) J.V. Schley (4) Richard M. Hoe (1) Farmers (2) Labourers
104. Kashi baba, a kanpur mill worker wrote and published (3) Mill Owners (4) Soldiers
“Chhote Aur Bade Ka Sawal” in 1938 to show the links
between .... [Andhra Pradesh 2018] 118. Bharat Mata, Painting was drawn by [Maharashtra 2015]
(1) Caste and Class exploitation. (1) Rabindranath Tagore (2) Nand Lal Basu
(2) Caste and Religion relation. (3) Amrita Shergil (4) Abanindra Nath Tagore
(3) Income and Untouchability. 119. Gutenberg was associated with [Maharashtra 2015]
(4) Industrialists and Politicians. (1) Powerloom (2) Rail Engine
105. The Ryotwari settlement was introduced by the British in (3) Computer (4) Printing Press
the ... [Andhra Pradesh 2018]
S-30 Target NTSE
120.‘Depressed Class Association’ was formed by 122.Reichstag was: [Maharashtra 2015]
[Maharashtra 2015] (1) French Parliament (2) Russian Parliament
(1) Jyoti Ba Phule (2) Mahatma Gandhi (3) German Parliament (4) Japan Parliament
(3) B. R. Ambedkar (4) Shahuji Maharaj 123.Who was Viceroy of India during Civil Disobedience
121.Who were the ‘Kulak’ in soviet society? Movement? [Maharashtra 2015]
[Maharashtra 2015] (1) Lord Curzon (2) Lord Minto
(1) Small Farmers (2) Big Farmers (3) Lord Canning (4) Lord Irwin
(3) Shop Keepers (4) Lawyers

SECTION 5. POLITICAL SCIENCE


1. Which of the following Secretary General said that “US war (1) National government gives some powers to the
on Iraq was not legal” (Delhi 2019)
provincial government
(1) Kofi Annan (2) B. B. Ghali (2) Power is distributed among the legislature, executive
(3) U Thant (4) Ban Ki Moon and judiciary
2. President can declare emergency when- (Delhi 2019)
(1) Prime minister advises him to do so (3) Elected officials exercise supreme power in the
(2) Parliament advises government
(3) The council of ministers, in writing, advises him to do so (4) Governmental power is divided between different level
(4) Home minister ask him to do so of government
3. The international organisation that works for human rights 11. In which House is the finance bill presented first ?
is- (Delhi 2019) [Rajasthan 2018]
(1) Amety International (2) Amnesty International (1) Rajya Sabha (2) Lok Sabha
(3) Asnesty International (4) Afnesty International (3) Both Lok Sabha and Rajya Sabha anywhere
4. Choose the odd pair of personalities from the following- (4) Reserve Bank of India
(Haryana 2019) 12. Who among the following is a part of the political executive ?
(1) Mrs. Indira Gandhi and Narendra Modi (1) District Collector [Rajasthan 2018]
(2) Balram Jakhar and Shivraj Patil (2) Secretary of the Ministry of Home Affairs
(3) Narendra Modi and Sumitra Mahajan (3) Home Minister
(4) Dr. Rajendra Prasad and Dr. V.V. Giri (4) Director General of Police'
5. Who Presides the Joint session of Parliament? 13. Which of the following institutions can make changes to
(Haryana 2019)
an existing law of our country ? [Rajasthan 2018]
(1) President (2) Vice President
(3) Speaker of Lok Sabha (4) Prime Minister (1) Supreme Court of India
6. The name of the autobiography written by Nelson Mandela (2) International Court of Justice
is (Chandigarh 2019) (3) Prime Minister
(1) Robben island (2) The long walk to Freedom (4) Parliament.
(3) Blacks (4) Aparthied 14. Which one of the following is considered as a fundamental
7. The ________ legislates on residuary subjects. right according to the Constitution of India ?
(Chandigarh 2019) [Rajasthan 2018]
(1) Union Government (2) State Government (1) Right to work
(3) Local Government (4) President (2) Right to adequate livelihood
8. Read the following statements and select the correct (3) Right to protect one's culture
option. (Karnataka 2019) (4) Right to get higher education
Assertion (A) : Directive principles are enshrined in the
15. Match the following in reference to constitution making
Constitution for the Government Administration.
Reason (R) : People can question in the cout for not imple- Process : [Rajasthan 2018]
menting Directive Principles. (A) B.N. Rao (i) President of the
(1) A is false but R is true Constituent Assembly
(2) R is fasle but A is true (B) B.R. Ambedkar (ii) Member of the
(3) A and R are true and R is correct explanation of A Drafting Committee
(4) A and R are true but R is not correct explanation of A (C) Rajendra Prasad (iii) Chairman of the
9. By whom the "Right to Constitutional Remedies" was Drafting Committee
considered as the soul and heart of Indian constitution? (D) T.T. Krishnamachari (iv) Legal Advisor
[Delhi 2018] (1) (A) - iv, (B) - iii, (c) - i, (D) - ii
(1) Mahatma Gandhi (2) Dr. Rajendra Prasad (2) (A) - iv, (B) - ii, (c) - i, (D) - iii
(3) B. R. Ambedkar (4) Jawahar Lal Nehru (3) (A)- i, (B)- iii, (c)- iv, (D)- ii
10. The distinguish feature of a federal government is (4) (A) - iii, (B) - iv, (c) - i, (D) - ii.
[Delhi 2018]
Stage - I Solved Questions S-31

16. Match List - I and List - II and choose the correct code from 23. Match list 01 of Heads/Ministers of Executive of Indian
the given codes : [Rajasthan 2018] Government with List 02 of their “functions” and select the
List - I List - II correct answer using the codes given below.
(A) Union list (i) Computer Software [Karnataka 2017]
(B) State list (ii) Communications List-01 List-02
(C) Concurrent list (iii) Police (Heads of Executive (Functions)
(D) Residuary powers (iv) Forests. of Indian Government)
A B C D A. President I. Chairperson of the
(1) iii ii i iv NITI Ayoga
(2) ii iii iv i B. Vice President II. Appointment of
(3) ii iv i iii Chief Minister to
(4) iv iii ii i States
17. “Political Science begins and ends with the State”, is stated C. Prime Minister III. Appointment of
by [West-Bengal 2018] Governors of State
(a) Gettel (b) Garner (c) Seeley (d) Aristotle D. Finance Minister IV. Presentation of
18. How many members in the Lok Sabha can be nominated by Union Budget
the President of India? [West-Bengal 2018] V. Act as Ex-Officio
(a) 2 (b) 3 (c) 4 (d) 5 Chairperson of
19. Joint Session of the Indian Parliament is presided over by Rajya Sabha
the Codes:
[West-Bengal 2018] (1) A-i, B-v C-iv D-ii
(a) Vice-President (b) Speaker (2) A-ii B-iii C-iv D-i
(c) Governor (d) President (3) A-iii B-v C-i D-iv
20. In Indian Parliamentary system of government the Council (4) A-iv B-ii C-iii D-i
of Ministers is responsible to [West-Bengal 2018] 24. Which of the following statement/statements about Anti
Defection Law is/are correct? [Karnataka 2017]
(a) President (b) Prime Minister
A. The Constitution Act of 1985 is popularly known as
(c) Parliament (d) Supreme Court the Anti Defection Law.
21. Section 25 of Indian Constitution provides following right B. Independently elected member of Parliament or a State
to Citizens of India [Karnataka 2018] Legislature will not be disqualified if he/she join any
(1) Guarantees educational facilities to all political party after elections.
(2) Guarantees reservation in political field to scheduled Choices:
castes and tribes (1) Only A is correct
(3) Guarantees reservation in the employment sector to (2) Only B is correct
scheduled castes and tribes (3) Both A and B are correct
(4) Guarantees free entry to all people to social and religious (4) Both A and B are incorrect
places 25. Consider the following two statements on power sharing
22. Match Column 'A' with Column 'B' and choose correct an- and select the answer using the codes given below.
swer. [Karnataka 2017] [Andhra Pradesh 2017]
Column-A Column-B (a) Power sharing is good for democracy.
a. group of people I. Movement (b) It helps to reduce the possibility of conflicts between
pelting stones social groups.
b. people assembled II. Campaign Which of these statements are true and false ?
near a theatre to (1) Both a and b are true. (2) Both a and b are false.
buy tickets (3) a is true but b is false. (4) a is false but b is true.
c. A group protesting III. Riot 26. Match the ministry with the news that the ministry may
against a project have released. [Andhra Pradesh 2018]
in a peaceful way (a) A new policy is being made to increase the jute
d. A group of people IV. Mob exports from the country.
involved in bringing (b) Telephone services will be made more accessible to
awareness rural areas.
a b c d (c) The price of rice and wheat sold under the Public
(1) III IV I II Distribution system will go down.
(2) I II III IV (d) A pulse polio campaign will be launched.
(3) III I IV II (e) The allowances of the soldiers posted on high
(4) IV II I III altitudes will be inceased.
S-32 Target NTSE
Codes : (3) World Bank
(4) None of these
(a) (b) (c) (d) (e) 34. President of India is elected by [Delhi 2016]
(1) By the people
(1) I iii ii iv V (2) By all the elected members of Parliament and State
Legislature
(2) iv v iii ii i
(3) By the members of Lok Sabha and Vidhan Sabha
(3) iii v ii i iv (4) By the members of Rajya Sabha
35. ‘End of Racial Discrimination’ this fundamental right is a
(4) ii v iii iv i part of [Delhi 2016]
(1) Right to equality
27. Match the following countries and the path democracy (2) Right to Freedom
has taken in that country. [Andhra Pradesh 2018] (3) Right to Education and Culture
Country Path to Democracy (4) Right against exploitation
36. The Headquarter of world Bank is- [Chhattisgarh 2016]
(a) Nepal (i) End of One party Rule.
(1) Geneva (2) Newyork
(b) Chile (ii) King agreed to give up his powers. (3) London (4) Washington D.C.
(c) Ghana (iii) End of Military Dictatorship. 37. When was Universal Human Rights day declared-
Poland (iv) Freedom from British Colonial Rule. (Delhi 2016)
Codes: (1) 1st October 2001 (2) 10th December 1948
(3) 1st March 1950 (4) 1st April 2000
A b c d
38. The Constitution of India was adopted on
(1) (i) (iii) (iv) (iii) [West Bengal 2016]
(2) (ii) (iii) (iv) (i) (1) 9 December, 1946 (2) 14 August, 1947
(3) (iii) (ii) (i) (iv) (3) 26 November, 1949 (4) 26 January, 1950
(4) (iv) (i) (iii) (ii) 39. The Chairman of the Drafting Committee of the Constituent
28. Find out the subject which is under concurrent list ? Assembly of India was [West Bengal 2016]
[Andhra Pradesh 2018] (1) B. R. Ambedkar (2) Jawaharlal Nehru
(1) Police (3) Sardar Vallabhbhai Patel (4) Dr. Rajendra Prasad
(2) Communication 40. In contemporary world, which quality makes democracy
(3) Marriages and Divorce. superior to any other form of Government:
(4) None of the above. [Maharashtra 2015]
29. A struggle known as “Bolivia’s water war” took place in (1) Efficiency (2) Responsiveness
… city. [Andhra Pradesh 2018] (3) Transparency (4) Legitimacy governance
(1) Cochabamba (2) Lapaz 41. Gender division refers to [Maharashtra 2015]
(3) Trinidad (4) Montero (1) Unequal child sex ratio
30. Which of the following does not figure in preamble Indian (2) Absence of voting rights
Constitution? [Haryana 2017] (3) Biological differences
(1) Democratic (2) Secular (4) Unequal roles assigned to men and women
(3) Republic (4) Religious Justice 42. When was the “Kittiko-Hachchiko” movement started?
31. In which game P.V. Sindhu won the silver medal for India in [Maharashtra 2015]
Rio Olympics? [Haryana 2017] (1) 1984 (2) 1986 (3) 1987 (4) 1983
(1) Lawn Tenis (2) Badminton 43. In case of conflict between the centre and the State
(3) Wrestling (4) Archery Government over a subject in the concurrent list:
32. Which of the following statements properly define the [Maharashtra 2015]
'Constitution'? [Rajasthan 2017] (1) The state has the upper hand
(A) Constitution protects the rights of citizens (2) The centre has the upper hand
(B) It determines the functioning of governments (3) Both are equally applicable
(C) It determines the process of legislation (4) Supreme Court decides whether the State Law should
(D) It decides the name of person who is going to be the prevail or Central Law
President. 44. Total languages in the Eighth schedule of Indian
(1) A, B, D (2) B, C (3) A, B, C (4) B, C, D Constitution [Karnataka 2015]
33. Which one of the following is the Institution, functioning (1) 18 (2) 19 (3) 21 (4) 22
for International law, Security, Social equality and World 45. Which one of the following is an anti-national and economic
peace? [Rajasthan 2017] crime? [Karnataka 2015]
(1) Smuggling (2) Communalism
(1) International Monetary Fund
(3) Corruption (4) Economic inequality
(2) United Nations Organisation
Stage - I Solved Questions S-33

46. ‘The Untouchability Crime Act’ was implemented in [Chandigarh 2015]


[Karnataka 2015] Name of the organization Location of Headquarter
(1) 1948 (2) 1950 (3) 1956 (4) 1955 (1) UNESCO Paris
47. Name the Law which empowers the people to carry on (2) ILO Geneva
democratic reforms: [Delhi 2015] (3) FAO Brussels
(1) Company Act (2) M. R. T. P. (4) International Court of the Hague
(3) Right to Information (4) Right to Vote Justice
48. Which of the following is the main feature of Pressure group? 50. Which of the following statements about the international
[Delhi 2015] Court of justice is not correct? [Chandigarh 2015]
(1) Direct control on political power (1) It is the principla judicial organ of the United Nations
(2) Try to influence the politics of Government (2) Its judges are elected
(3) Lax Organisation (3) It s decisions are not enforceable
(4) Direct participation in Political Power (4) Nations aggrieved by the non-implementation of the
49. Which of the following locations is not correct? decision have no further redress

SECTION 6. CIVICS
1. Who was the chairman of the drafting committee of the (3) All the voters of the village
Indian Constituent Assembly? (Delhi 2020-21) (4) Only the youth of the village
(1) Dr. Rajendra Prasad (2) Dr. B.R. Verma 8. A person who is not a member of Parliament is appointed
(3) Dr. B.R. Menon (4) Dr. B.R. Ambedkar on the minister, he/she has to get elected to one of the
2. The movement that seeks equality in the personal and family houses of Parliament within: (Delhi 2020-21)
life of women is known as – (Delhi 2020-21) (1) A month
(1) Narivadi Andolan (2) Three months
(2) Nari Sashaktikaran Andolan (3) Six months
(3) Mahila Shakti Andolan (4) Selected time is fixed by the President
(4) Mahila Adhikar Andolan 9. Respect for the National Flag and the National Anthem is:
3. In case of conflict between the central government and the (Andhra Pradesh-2021)
State Government over the subject in the Concurrent list: (1) A fundamental right of every citizen
(Delhi 2020-21) (2) A fundamental duty of every citizen
(1) Supreme Court decides which of the two should be (3) A directive principle of state policy
considered. (4) An ordinary duty of every citizen
(2) The state government should be obeyed. 10. Which one among the following was not an aim of the 42nd
(3) The central government should be obeyed. Constitutional amendment? (Andhra Pradesh-2021)
(4) Both governments should be obeyed. (1) Excluding the courts from election disputes
4. In India, seats are reserved for women in which of the (2) Strengthening the central government
following bodies:- (Delhi 2020-21) (3) Abolition of bonded labour
(1) In Lok Sabha (2) In State Legislatures (4) Making the judiciary subservient to parliament
(3) In Rajya Sabha (4) In Panchayati Raj 11. The Supreme Court judgement popularly known as the
5. Main recommendation of the Mandal Commission was: ‘Keshavananda Bharathi case’ says that: (Andhra Pradesh-2021)
(Delhi 2020-21) (1) Certain basic provisions in the Constitution cannot
(1) Reservation for Socially and Educationally Backward be changed under any circumstances
Classes. (2) The word ‘secular’ is to be added to the preamble of
(2) Reservation for schedule caste. the constitution.
(3) Reservation for schedule Tribes. (3) ‘Right to Property’ should be removed from fundamental
(4) Reservation for Minorities. rights.
6. What is Mid-Term Election? (Delhi 2020-21) (4) Laws should not be amended
(1) Election for the seat to be vacated due to some reason. 12. Constitution Day is celebrated in India every year on which
(2) Election in the event of death of one member. day? Why? (Andhra Pradesh-2021)
(3) Election to be held in whole country or state before (1) 26th November – Constituent Assembly met for the
the scheduled time. first time on that day.
(4) Completion of 5 years election. (2) 26th January – Constitution came into force on that day.
7. Who becomes the members of Gram Sabha? (3) 26th November – Constitution was finally adopted on
(Delhi 2020-21) that day.
(1) Only elderly people (4) 26thJanuary – Constitution was amended for the first
(2) Only elected members of Gram Panchayat time on that day.
S-34 Target NTSE
13. The sequence in which the given terms are mentioned in (1) R (2) A (3) M (4) Q
the constitutional preamble is : (Andhra Pradesh-2021) 27. Which one of the following terms is not included in the
(1) Sovereign, Socialist, Secular, Democratic, Republic preamble to the Indian Constitution?
(2) Sovereign, Socialist, Secular, Republic, Democratic (1) Republic (2) Justice (Bihar-2021)
(3) Sovereign, Socialist, Democratic, Secular, Republic (3) Monarchy (4) Equality
(4) Sovereign, Socialist, Republic, Secular, Democratic 28. Which of the following institutions can amend the
14. Minimum educational qualification required for a candidate to constitution of India? (Bihar-2021)
contest in Parliamentary elections are: (Andhra Pradesh-2021) (1) The Parliament (2) The Cabinet
(1) SSC or equivalent (3) The Prime Minister (4) The President
(2) Intermediate or equivalent 29. Which of the following are the features of the Federal
(3) Graduation Government? (Bihar-2021)
(4) No educational qualification is required I. Unwritten Constitution II. Division of Powers
15. “MONEY BILL is first introduced in the III. Single Citizenship IV. Independent Judiciary
(West Bengal-2021) (1) I and II (2) II and III
(1) Lok Sabha (2) Rajya Sabha (3) I and IV (4) II and IV
(3) Supreme Court (4) High Court 30. The first speaker of Lok Sabha was- (Uttar Pradesh-2021)
16. The number of judges of the International Court of Justice (1) Ganesh Vasudev Mavalankar
is (West Bengal-2021) (2) Pt. Govind Vallabh Pant
(1) 9 (2) 10 (3) 15 (4) 16 (3) Ananthasayanam Ayyangar
17. The age of retirement of the judges of the High Court is (4) C. Subramanian
(West Bengal-2021) 31. Right to Education Act came into effect on-
(1) 65 years (2) 60 years (Uttar Pradesh-2021)
(3) 62 years (4) 70 years (1) 2005 (2) 2010 (3) 2008 (4) 2012
18. The headquarter of the World Health Organization is 32. ‘Forward Bloc’ is a regional party of- (Uttar Pradesh-2021)
(West Bengal-2021) (1) Odisha (2) Jharkhand
(1) London (2) Manchester (3) West Bengal (4) Chhatisgarh
(3) Geneva (4) Paris 33. ‘National Democratic Alliance’ was founded in-
19. The minimum age for the citizen to exercise their right to (Uttar Pradesh-2021)
vote has been reduced to 18 years from 21 years through (1) May 1998 (2) June 1996
the (West Bengal-2021) (3) May 1999 (4) June 1997
(1) 42nd Amendment Act (2) 44nd Amendment Act 34. A communication issued by an appropriate authority stating
(3) 61nd Amendment Act (4) 74nd Amendment Act the policy of the Government is known as
20. The Panch-Sheel Agreement was signed between (Tamil Nadu-2021)
(West Bengal-2021) (1) Office Memorandum (2) Legislative Procedure
(1) India and China (2) India and Nepal (3) Government Order (4) Circular
(3) India and Pakistan (4) Pakistan and China 35. Which party always won elections in Mexico from its
21. When was the unification of Rajasthan completed? independence in 1930 until 2000? (Tamil Nadu-2021)
(Rajasthan-2021) (1) Revolutionary Party
(1) 30th March, 1949 (2) 15th May, 1948 (2) Mexican Revolutionary Party
(3) 26th January, 1950 (4) 1st November, 1956 (3) Institutional Revolutionary Party
22. From which district of Rajasthan, the modern three-tier of (4) Institutional Party
Panchayati Raj system was lunched on October 2nd, 1959? 36. The Government of India Act came into function in the
(Rajasthan-2021) year (Tamil Nadu-2021)
(1) Ajmer (2) Nagour (1) 1930 (2) 1932 (3) 1935 (4) 1938
(3) Jaipur (4) Bikaner 37. The State in which assembly election held in 2020-21
23. When was the constitution of India adopted, enacted and (Tamil Nadu-2021)
spirited? (Rajasthan-2021) (1) Odisha (2) Maharashtra
(1) 26th November, 1949 (2) 26th October, 1948 (3) Karnataka (4) Bihar
(3) 26th January, 1950 (4) 26th January, 1949 38. _______ is an informal body of Senior Ministers.
24. Where did the 18th summit of ‘SAARC’ take place in (Tamil Nadu-2021)
November, 2014? (Rajasthan-2021) (1) Council of Ministers (2) Cabinet Ministers
(1) Nepal (2) New Delhi (3) Ministers of State (4) Cabinet
(3) Bhutan (4) Maldiv 39. The Legislative Council still in existence in _______
25. What can be maximum number of members of Rajya Sabha (Tamil Nadu-2021)
as per constitution provision? (Rajasthan-2021) (1) Maharashtra (2) Tamilnadu
(1) R (2) A (3) M (4) Q (3) Odisha (4) Kerala
26. Till what age can the Judges of high court hold his post? 40. The Elected Representatives in turn elect their representative
(Rajasthan-2021) is known as _____ election. (Tamil Nadu-2021)
Stage - I Solved Questions S-35

(1) District (2) Indirect (1) Only A, B and D (2) Only A, C and D
(3) National Level (4) State Level (3) Only A, B and C (4) Only C and D
41. Which organ of the United Nation passes the annual budget 44. Read the following statements and write the correct option
of the United Nation as in function? (Maharashtra-2021) with which all those links.
(1) General Assembly A. It came into existence on 1st April 1974.
(2) Security Council B. It functions under the High court.
(3) Economic and Social Council C. It takes up crimes like murder, robbery, dacoity etc.
(4) Trusteeship Council D. It is also known as the District Magistrate court.
42. Choose the percentage which has been raised to increase the
(1) Civil Court (2) Commissioner’s Court
representation of women in politics. (Maharashtra-2021)
(3) Tahashildar Court (4) Criminal Court
(1) 33% (2) 50% (3) 60% (4) 58%
43. The correct group of statements related to fundamental 45. The Supreme Court declared as a fundamental right to
duties is: (Karnataka-2021) provide free and compulsory education to all children
A. Fundamental duties are delineated in article 51 ‘A’. between the age of 6 to 14 years under article 21A in :
B. 11 fundamental duties have been delineated in the 3rd (Karnataka-2021)
chapter of our constitution. (1) Golaknath v/s Punjab Government
C. 10 fundamental duties were included as per the 42nd (2) Keshawananda Bharathi v/s Kerala Government
amendment in the year 1976. (3) Unnikrishnan v/s Andrapradesh Government
D. Through the 86th amendment to the constitution in (4) Minerva Mills v/s Union of India
2002, the 11th fundamental duty was included

SECTION 7. GEOGRAPHY
1. Coffee cultivation was first introduced in 9. Consider the following statements.
(Delhi 2020-21) (Andhra Pradesh-2020-21)
(1) Himalayas (2) Aravali Hills (i) Laying ‘Pakka’ roads were devised by McAdam
(3) Garo Hills (4) Baba Budan Hills (ii) The steam locomotive made by Stephenson was
2. Which one of the following describes a system of agriculture named ‘Rocket’.
where a single crop is grown on a large area? Which of the above statements is/are CORRECT?
(Delhi 2020-21) (1) (i) only (2) (ii) only
(1) Shifting Agriculture (2) Plantation Agriculture (3) Both (i) and (ii) (4) Neither (i), nor (ii)
(3) Horticulture (4) Intensive Agriculture 10. Thick barks and wax-coated leaves is an important feature
3. Neyveli lignite mines are located in the state of of: (Andhra Pradesh-2020-21)
(Delhi 2020-21) (1) Tropical evergreen forests
(1) Kerala (2) Tamil Nadu (2) Temperate evergreen
(3) Karnataka (4) Andhra Pradesh (3) Temperate deciduous forests
4. Hirakund Dam is constructed on the river – (4) Mediterranean vegetation
(Delhi 2020-21) 11. Consider the following statements and select the correct
(1) Ganga (2) Manjira answer using the codes given below.
(3) Manas (4) Mahanadi (Andhra Pradesh-2020-21)
5. When and where first cement plant was set up? (i) India is the second-largest producer of paddy in the
(Delhi 2020-21) World after China.
(1) Chennai, 1905 (2) Chennai, 1904 (ii) India is the largest producer as well as the consumer
(3) Kolkata, 1905 (4) Kolkata, 1904 of pulses in the world.
6. Who wrote the book “Small is Beautiful”? (iii) India is the largest producer of sugarcane in the world.
(Delhi 2020-21) Codes:
(1) Gandhi Ji (2) Brundtland (1) (i), (ii), (iii) all are correct (2) (i), (ii) are correct
(3) Schumacher (4) Annie Besant (3) (ii), (iii) are correct (4) (i), (iii) are correct
7. According to the main role industry can be divided into 12. Baltic sea has very low salinity. Which of the following factors
(1) Agro-based and mineral-based (Delhi 2020-21) is mainly responsible for this? (Andhra Pradesh-2020-21)
(2) Key and consumer industries (1) Evaporation (2) Precipitation
(3) Public, private, and Joint sector (3) Ocean currents (4) Fresh water flow from rivers
(4) Heavy and light industries 13. Match List-I with List-II and select the correct answer using
8. Which of the following is not a renewable resource? the codes given below. (Andhra Pradesh-2020-21)
(Delhi 2020-21) List - I (Coast) List-II (State)
(1) Forest (2) Animals (A) Coromandel (i) Karnataka
(3) Water (4) Petroleum (B) Circar (ii) Kerala
S-36 Target NTSE
(C) Malabar (iii) Andhra Pradesh (1) Kolleru (2) Pulicat
(D) Canara (iv) Tamil Naidu (3) Chilka (4) Vembnad
Codes: 23. Diamond Quadrilateral project is related to
(1) A – i, B – ii, C – iv, D – iii (2) A – i, B – ii, C – iii, D – iv (West Bengal-2020-21)
(3) A – iv, B – iii, C – ii, D – i (4) A – iv, B – ii, C – i, D – iii (1) Air Transport (2) Rail Transport
14. Assertion (A): Coromandel coast remains mostly dry during (3) Road Transport (4) Water Transport
the South-West monsoon season. 24. The colour used for drawing contour lines in a topographical
Reason (R): It is in the rain shadow area of the Arabian sea map is (West Bengal-2020-21)
branch and is parallel to the Bay of Bengal branch. (1) Black (2) Brown
(Andhra Pradesh-2020-21) (3) Red (4) Blue
(1) Both ‘A’ and ‘R’ are true and ‘R’ is the correct 25. Which lake is located in Rajsamand district?
explanation of ‘A’. (Rajasthan-2020-21)
(2) Both ‘A’ and ‘R’ are true but ‘R’ is not the correct (1) Tal chhapar (2) Nauchouki
explanation of ‘A’. (3) Tordisagar (4) Navlekha
(3) ‘A’ is true but ‘R’ is false. 26. Which one of the following is a food crop?
(4) ‘A’ is false but ‘R’ is true. (Rajasthan-2020-21)
15. Consider the following statements. (1) Sugarcane (2) Tobacco
(Andhra Pradesh-2020-21) (3) Barley (4) Tea
(i) India is the largest producer of raw jute and jute goods 27. Which district in Rajasthan is called as cement city?
in the world. (Rajasthan-2020-21)
(ii) The first cement plant in India was setup in Jamshedpur (1) Chittorgarh (2) Kota
in 1904. (3) Bundi (4) Pratapgarh
Which of the above statements is/are CORRECT?
28. Which of the following is the highest Literacy-rate district
(1) (i) only (2) (ii) only
of Rajasthan? (Rajasthan-2020-21)
(3) Both (i) and (ii) (4) Neither (i) nor (ii)
(1) Ajmer (2) Jaipur
16. Consider the following statements
(3) Kota (4) Udaipur
(Andhra Pradesh-2020-21)
(i) Infant mortality rate is the number of children who die 29. Which of the following is ferrous metallic mineral?
within one year, our of 100 live children born. (Rajasthan-2020-21)
(ii) Literacy rate is the percentage of the literate population (1) Tin (2) Cobalt
in the 6 and above age group. (3) Gold (4) Bauxite
Which of the above statements is/are CORRECT? 30. Which is the major source of rainwater storage in the
(1) (i) only (2) (ii) only Shekhawati region? (Rajasthan-2020-21)
(3) Both (i) and (ii) (4) Neither (i) nor (ii) (1) Khadin (2) Tanka
17. Which one of the following groups of state has the largest (3) Nadi (4) Johad
number of cotton textile centres? 31. Who coined the slogan “Jo sir sante Roonkh rahe to bhi
(Andhra Pradesh-2020-21) sasto Jaan”? (Rajasthan-2020-21)
(1) Gujarat and Maharashtra (1) Smt. Amrita Devi (2) Smt. Jaana Bai
(2) Karnataka and Tamil Nadu (3) Smt. Sushila Devi (4) Smt. Harsha Devi
(3) Maharashtra and Madhya Pradesh 32. The Green revolution started in (Rajasthan-2020-21)
(4) Uttar Pradesh and Gujarat (1) 1966-67 (2) 1970-71
18. Which of the following regions is suitable for constructing (3) 1988-89 (4) 2001-02
railway lines? (Andhra Pradesh-2020-21) 33. Bharatpur Bird sanctuary is situated in –
(1) Himalaya Mountains Range (Bihar-2020-21)
(2) Garo, Khasi and Jaintia Hills (1) Gujarat (2) Rajasthan
(3) The northern Plains (3) Assam (4) Bihar
(4) Rajasthan desert 34. The highest Literacy rate in India is in ……..?
19. The time difference between Greenwich Mean Time and (Bihar-2020-21)
the Indian Standard Time is (West Bengal-2020-21) (1) West Bengal (2) Maharashtra
(1) 6 hours (2) 5 hours 30 minutes (3) Kerala (4) Punjab
(3) 5 hours 15 minutes (4) 5 hours
35. Where is ropeway in Bihar? (Bihar-2020-21)
20. The climate which is found in the Cape Town of South
Africa is (West Bengal-2020-21) (1) Bihar Shariff (2) Rajgir
(1) Equatorial climate (2) Tropical Monsoon (3) Gaya (4) Munger
climate 36. Select the correct statements – (Bihar-2020-21)
(3) Mediterranean climate (4) Hot Desert climate (A) Koshi river is the sorrow of Bihar
21. Which of the following, parallel of latitude, passes through (B) Parrot is the national bird of India
the middle of India? (West Bengal-2020-21) (C) Maruti Industry is situated in Delhi
(1) Equator (2) Tropic of Capricon (D) Varanasi is situated on the bank of river Ganges
(3) Prime Meridian (4) Tropic of Cancer (1) A and D (2) B and C
22. The lake located, in between the deltas of the Godavari (3) B, C and D (4) A, C and D
river and the Krishna river is (West Bengal-2020-21) 37. Which name is correct for Patna Airport?
(Bihar-2020-21)
Stage - I Solved Questions S-37

(1) Jai Prakash Narayan International Airport (3) (A) – (ii), (B) – (iv), (C) – (iii), (D) – (i)
(2) Patna Airport (4) (A) – (iv), (B) – (iii), (C) – (ii), (D) – (i)
(3) Rajendra Prasad International Airport 49. ______ is not a part of the world network biosphere
(4) Bihar Airport reserves of UNESCO. (Tamil Nadu-2020-21)
38. Which statement is not correct? (Bihar-2020-21) (1) Nilgiri (2) Agasthiyam
(A) Medha Patekar is related to Narmada Bachao Andolan. (3) Great Nicobar (4) Kachch
(B) New alluvial soil is termed as Bangar 50. ______ is the most common but powerful means of
communication. (Tamil Nadu-2020-21)
(C) Mangrove Forest found in the coastal area in India.
(1) Internet (2) Social Media
(D) Plantation agriculture is one of the types of commercial (3) Print Media (4) Television
farming. 51. The name Gaja (Cyclone) was named by _______
(1) A (2) B (Tamil Nadu-2020-21)
(3) B, C, and D (4) C and D (1) Bangladesh (2) Sri Lanka
39. The Golden Quadrilateral Super highway connected with (3) Pakistan (4) Oman
the following – (Bihar-2020-21) 52. The full form of HCR (Tamil Nadu-2020-21)
(1) Jammu, Bikaner, Jodhpur, Rajkot (1) Head Count Ratio (2) Human Center Ratio
(2) Porbandar, Bikaner, Amritsar, Srinagar (3) Head Calculating Rate (4) Head Continue Ratio
(3) Delhi, Mumbai, Chennai, Kolkata 53. Identify the first nuclear reactor of India functioning on
(4) Sikkim, Siliguri, Jorhat, Agartala atomic energy. (Maharashtra-2020-21)
40. The Ragur Soil is also known as: (1) Dhruv (2) Apsara
(Uttar Pradesh-2020-21) (3) Tarapur (4) Zarina
(1) Red Soil (2) Yellow Soil 54. _________ from the Western Rajasthan is considered as
(3) Black Soil (4) Alluvial Soil the driest part in India. (Maharashtra-2020-21)
41. Which of the following is the longest river of the world? (1) Jaisalmer (2) Ajmer
(3) Jodhpur (4) Jaipur
(Uttar Pradesh-2020-21)
55. In India about _______ present of passengers are carried
(1) Amazon river (2) Yangtze river by road. (Maharashtra-2020-21)
(3) Ganga river (4) Nile river (1) 75% (2) 65%
42. The Toda tribes are the original inhabitants of: (3) 85% (4) 80%
(Uttar Pradesh-2020-21) 56. In which direction of Brazil, there is no sea coast?
(1) Aravalli hills (2) Nilgiri hills (Maharashtra-2020-21)
(3) Satpura hills (4) Guru Shikhar (1) East (2) West
43. The city where the first Earth summit was organized. (3) South (4) North
(Uttar Pradesh-2020-21) 57. Which one of the following is not a major trading partner of
(1) Rio de Janeiro (2) Shangai Brazil? (Maharashtra-2020-21)
(3) Tokyo (4) Manila (1) Pakistan (2) Germany
44. Manrega was implemented from the year? (3) India (4) Canada
(Uttar Pradesh-2020-21) 58. What type of tourism is developing in Brazil?
(1) 2005 (2) 2006 (3) 2007 (4) 2008 (Maharashtra-2020-21)
45. Where is the headquarter of Tea Board located? (1) Historical (2) Cultural
(3) Eco-tourism (4) Professional
(Uttar Pradesh-2020-21) 59. India is located in the _________ hemispheres of the earth.
(1) Darjeeling (2) Bengaluru (Maharashtra-2020-21)
(3) Kolkata (4) Mumbai (1) Southern and Eastern (2) Northern and Eastern
46. Which wind is referred to as “roaring forties” at 400 ‘furious (3) Northern and Western (4) Southern and Western
fifties’ at 500 and ‘screaming =sixties’ at 600 latitudes? 60. ___________ is the most urbanized state in India.
(Tamil Nadu-2020-21) (Maharashtra-2020-21)
(1) Trade winds (2) Easterlies (1) Maharashtra (2) Gujarat
(3) Goa (4) Kerala
(3) Westerlies (4) Cyclones
61. Identify the wrong pair: (Maharashtra-2020-21)
47. Pulicat lake lies in the ________ district River Tributary
(Tamil Nadu-2020-21) (1) Ganga – Yamuna
(1) Kanchipuram (2) Chennai (2) Sindhu – Satluj
(3) Tiruvalluvar (4) Vellore (3) Krishna – Tungabhadra
48. Match the following: (Tamil Nadu-2020-21) (4) Tapi – Bhima
Column – I Column – II 62. The name Gaja (Cyclone) was named by _______
(A) Tehri Dam (i) Sutlej (Maharashtra-2020-21)
(B) Hirakud Dam (ii) Bhagirathi (1) Bangladesh (2) Sri Lanka
(C) Nagarjuna Sagar Dam (iii) Mahanadi (3) Pakistan (4) Oman
(D) Indira Gandhi Canal (iv) Krishna 63. Match Column-A with Column-B and choose the correct
(1) (A) – (ii), (B) – (iii), (C) – (iv), (D) – (i) answer. (Karnataka-2020-21)
(2) (A) – (iii), (B) – (iv), (C) – (i), (D) – (ii)
S-38 Target NTSE
Column – A Column – B 71. Per capita consumption of which energy source is
(Ports) (States) considered as an index of development ?
A. Nhava Sheva i. West Bengal (Andhra Pradesh 2019)
B. Tuticorin ii. Tamilnadu (1) Petroleum (2) Electricity
C. Paradip iii. Odisha
(3) Natural gas (4) Solar energy
D. Haldia iv. Andhra Pradesh
v. Maharashtra 72. Arrange these hills from west to east. (Chandigarh 2019)
(1) A - ii, B - iv, C - i, D - iii (2) A - v, B - ii, C - iii, D - i A. Khasi B. Garo
(3) A - iii, B - v, C - iv, D - ii (4) A - i, B - iii, C - ii, D - v C. Naga D. Jaintia
64. Read the following statements and select the correct answer: (1) C, A, B, D (2) D, B, A, C
(Karnataka-2020-21) (3) A, B, C, D (4) B, A, D, C
(i) It forms from the weathering of Granite and gneiss 73. Which of the following two extreme locations are connecting
rocks. the east west corridor? (Chandigarh 2019)
(ii) It has a more sandy and less clayey composition. (1) Mumbai and Nagpur
(iii) It does not retain moisture. (2) Ahmedabad and Kolkata
(iv) Total area covered in India about 5.2 lakh sq. km.
(3) Silchar and Porbandar
(1) Red soil (2) Black soil
(3) Laterite soil (4) Alluvial soil (4) Nagpur and Siliguri
65. Match List-I with List-II. Select the correct answer from the 74. The Himalayan yew species is in danger because
code given below: (Karnataka-2020-21) (Karnataka 2019)
List I List - II (1) a chemical compound called ‘Taxol’ is extracted from
A. Vulnerable species i. Nicobar Pigeon the bark to cure cancer.
B. Rare species ii. Blue sheep (2) the hide of the animal is extracted for producing
C. Endemic species iii. Asiatic cheetah percussive instruments.
D. Extinct species iv. Wild Asiatic buffalo (3) the birds feathers are colourful and they are collected
(1) A - i, B - ii, C - iii, D - iv (2) A - iii, B - i, C - iv, D - ii by killing.
(3) A - ii, B - iv, C - i, D - iii (4) A - iv, B - iii, C - ii, D - i (4) the insecticides have brought adverse effects on them.
66. Read the description and choose the correct answer:
75. Choose the correct type of soil with reference to the shaded
(Karnataka-2020-21)
area in the given map. (Karnataka 2019)
i. It is a joint venture of Punjab, Haryana and Rajasthan.
ii. It is built across River Sutlej.
iii. It is 226 metres high and is the highest straight gravity
dam in India.
iv. It has four power projects (houses).
(1) Hirakud project (2) Bhakra Nangal project
(3) Rihand Valley project (4) Damodar Valley project
67. Read the following statements: (Karnataka-2020-21)
(i) Delhi, Kolkata, Chennai and Mumbai cities are linked
by six-lane highways.
(ii) Srinagar to Kanyakumari linking North to South.
(iii) Silchar and Porabander connect East to West.
(iv) The major objective is to reduce the time and distance
between the megacities.
The above statements refer to :
(1) National Highways
(2) Border Roads (1) Mountain Soils (2) Black Soils
(3) State Highways (3) Laterite Soils (4) Red and Yellow soils
(4) Golden Quadrilateral superhighways
76. In India this primitive form of cultivation is called by different
68. Which mineral has excellent dielectric strength, insulating
properties, low power loss factor and resistance to high names. Select the correct answer using the code given below-
voltage? (Delhi 2019) (Haryana 2019)
(1) Aluminium (2) Lime stone A. Madhya Pradesh (i) Pama Dabi or Koman
(3) Copper (4) Mica B. Odisha (ii) Bewar of Dahiya
69. Which of the following lake lies on the Equator? C. In Western Ghats (iii) Jhumming
(Delhi 2019) D. North-East region (iv) Kumari
(1) Lake Victoria (2) Lake Malavi (1) A-ii, B-i, C-iv, D-iii (2) A-i, B-ii, C-iii, D-iv
(3) Lake Nasser (4) None of these (3) A-ii, B-iii, C-iv, D-i (4) A-iii, B-ii, C-i, D-iv
70. Out of the following states, which one receives the South- 77. Which crop takes almost a year to grow? [Delhi 2018]
West monsoon lately? (Andhra Pradesh 2019) (1) Cotton (2) Jute
(1) Karnataka (2) Kerala (3) Rice (4) Sugarcane
(3) Maharashtra (4) Gujarat
Stage - I Solved Questions S-39

78. The coriolis force is caused due is [Delhi 2018] [Karnataka 2018]
(1) Wind movement (2) Earth rotation (1) A is true, R is false
(3) Cyclonic depression (4) Jet stream (2) A is false, R is true
79. Match List - I and List II and choose the correct code from (3) Both A and R are true, but R is not correct explanation
the following: [Rajasthan 2018] of A
List - I List - II (4) Both A and R are true and R is the correct explanation
(A) Northern end (i) 8° 4' N of A
(B) Southern end (ii) 37° 6' N 87. Choose the group of statements, which are correct with
(C) Eastern end (iii) 68° 7' N respect to "Golden Quadrilateral and Corridor Project".
(D) Western end (iv) 97° 25' N [Karnataka 2018]
A B C D a. This project covers National Highways with the length
(1) ii iii iv i of 15,000 kms.
(2) i ii iv iii b. The project was started in the year 2001
(3) ii i iv iii c. Total cost of the proposed project was Rs. 54,000 crores
(4) iii ii i iv d. This is the largest project taken up and no country in
80. Which of the following is the major sugarcane producing the world has taken up such project
state ? [Rajasthan 2018] (1) a, b and c (2) b, c and d
(1) Uttar Pradesh (2) Rajasthan (3) a, b, c and d (4) a, c and d
(3) West Bengal (4) Madhya Pradesh 88. Match the nations and the respective names of cyclones
81. Important deposits of which mineral are found in Koraput and choose correct answer. [Karnataka 2018]
in Odisha ? [Rajasthan 2018] A B
(1) Iron ore (2) Coal a. India I. Thane
(3) Copper (4) Bauxite b. Bangladesh II. Giri
82. Augite metamorphosed to [West-Bengal 2018] c. Myanmar III. Mujan
(a) Horn blande (b) Pyroclastic d. Oman IV. Jal
(c) Brecia (d) Pegmatite a b c d
83. ‘Busket of Egg topography’ is a common feature of (1) IV II I III
[West-Bengal 2018] (2) II III I IV
(a) River deposition (b) Wind deposition (3) II III IV I
(c) Glacial erosion (d) Glacial deposition (4) IV III II I
84. Widest waterfall of world is [West-Bengal 2018] 89. Match the following and choose correct answer.
(a) Khone waterfall of Laos [Karnataka 2018]
(b) Salto Angel of Veneguela Animals/Birds Catagory of Existence
(c) Niagra of U.S.A species
(d) Stanley waterfall of Congo a. Indian Rhino I. Rare species
85. Match Column 'A' with Column 'B' and choose correct b. Hornbill II. Extinct species
answer. [Karnataka 2018] c. Asiatic cheetah III. Vulnerable species
Column-A Column-B d. Gangtic Dolphin IV. Endangered species
a. Kaziranga National I. Assam a b c d
Park (1) IV II I III
b. Sundarbans II. West Bengal (2) IV I II III
c. Gir National park III. Gujarat (3) III II IV I
d. Tandova National IV. Maharashtra (4) III II I IV
Park 90. Sikkim, West Bengal, Assam and Arunachal Pradesh have
a b c d common frontiers with [Andhra Pradesh 2018]
(1) I II III IV (1) China
(2) I III II IV (2) Bhutan
(3) I IV III II (3) Bangladesh
(4) IV II I III (4) Mayanmar
86. Assertion (A): Increase in population, urbanisation, 91. Which of these is hot a Himachal Range ?
Industrialisation lead to the increased use of fossil fuels. [Andhra Pradesh 2018]
Reason (R): Over use of conventional energy resources (1) Dhaula Dhar (2) Pirpanjal Range
has resulted in the phenomenon of green house effect. (3) Kailash Range (4) Mahabharat Range
Select the correct option from the given alternatives.
S-40 Target NTSE

92. Match list A with B and select the correct answer using (3) Oxidation (4) Solution
the codes given below the list. [Andhra Pradesh 2018] 101. Roches Montanees are the hillocks of rocks. that are formed
List - A List - B by [West Bengal 2016]
(a) Hyderabad is warmer (i) Altitude (1) Glacial abrasion
than Mumbai. (ii) Mango (2) Glacial abrasion and Plucking
(b) Snowfall in showers. (3) Glacial deposition
Himalayas. (iii) Distance from (4) None of the above
(c) North western plain sea. 102. Which of the following lakes has formed as a result of
gets rainfall in winter. (iv) Western tectonic activity? [Delhi 2016]
(d) Rainfall in summer. Depression (1) Wular Lake (2) Chilika Lake
(3) Pulkit Lake (4) Kolleru Lake
a b c d 103. What is the name of the Indian Research centre located in
(1) iii ii iv i Antarctica? [Delhi 2016]
(2) ii i ii iv (1) Dakshin Gangotri (2) Shwet Baharti
(3) Him Putra (4) Shivalik Mani
(3) iii i iv ii
104. Nanda Devi bio-reserve is located in-
(4) iv ii i iii [Chhattisgarh 2016]
93. With reference to Indian agriculture, which of the following (1) Uttaranchal (2) Delhi
statements is not correct? [Andhra Pradesh 2018] (3) Punjab (4) Orissa
(1) India is the largest producer as well as the consumer 105. Rainfall occurs during winter in North-Western part of India
of pulses in the world. due to- [Chhattisgarh 2016]
(2) India is the second largest producer of rice in the (1) Cyclonic Depression (2) Western
world after China. disturbance
(3) Tea is an important beverage crop introduced in (3) Retreating Monsoon (4) South west
India initially by the Persians. Monsoon
(4) Groundnut is a kharif crop and accounts for about 106. Which continent of today is not a part of Gondwana land:
half of the major oil seeds produced in the country. [Maharashtra 2015]
94. In which of these following industries, limestone is not (1) North America (2) South America
used? [Andhra Pradesh 2018] (3) Africa (4) Australia
(1) Cement industry. (2) Iron and Steel industry. 107. Which of the following rivers of India flow through a rift
(3) Oil Refinery industry. (4) None of the above. valley? [Maharashtra 2015]
95. In which of the following states Koyana multipurpose project (1) Tapi River (2) Ganga River
is set up? [Haryana 2017] (3) Mahanadi River (4) Yamuna River
(1) Karnataka (2) Maharashtra 108. Rainfall in the Northern and North-Western parts of India in
(3) Andhra Pradesh (4) Tamil Nadu the month of October-November is caused mainly due to
96. In which country the ‘slash and burn’ agriculture is known [Maharashtra 2015]
as ‘Roca’? [Haryana 2017] (1) Western disturbances (2) Jet streams
(1) Mexico (2) Vietnam (3) Advancing Monsoon (4) Upper Air Circulation
(3) Brazil (4) Indonesia 109. The slash and burn agriculture is known as Milpa in which
97. Which is the first expressway of India [Jharkhand 2016] of the following countries? [Maharashtra 2015]
(1) Delhi – Kolkata (2) Mumbai – Pune (1) Indonesia (2) Mexico
(3) Pune – Chennai (4) Delhi – Mumbai (2) Vietnam (3) Sudan
98. Capital of Lakshdweep is [Jharkhand 2016] 110. Which of the following is/are the method/s of minimizing
(1) Kavaratti (2) Daman soil erosion? [Karnataka 2015]
(3) Silvassa (4) Port Blair A. Shifting cultivation
99. Which mine is found in Khetri ? [Jharkhand 2016] B. Construction of check-dams
(1) Copper Mines (2) Lignite Mines C. Contour ploughing
(3) Bauxite Mines (4) Iron Ore Mines D. Afforestation
100. Rusting of rock is caused by the process of (1) A only (2) B, C and D
[West Bengal 2016] (3) B and D (4) D only
(1) Carbonation (2) Hydration 111. Match the National parks indicated on the map of India, (I,
II, III and IV) with their respective names.
[Karnataka 2015]
Stage - I Solved Questions S-41

112. The slash and bum agriculture is known as Milpa in Which


of the following countries? [Delhi 2015]
(1) Indonesia (2) Mexico
IV
(3) Vietnam (d) Sudan
III
113. Which of the following is a non-renewable as well as non-
II recyblable resources? [Delhi 2015]
(1) Iron (2) Coal
(3) Water (4) Forests
I
114. Why is there scanty rainfall in the Deccan plateau?
[Chandigarh 2015]
(A) It is far away from the sea
A. Gir
(B) It is near the sea
B. Nagarhole
(C) It is in the rain shown region
C. Jim Corbett
(D) None of these
D. Kaziranga
115. Flood occur frequently in the northern plains of India be-
(1) I – C II – D III – A IV – B
cause of: [Chandigarh 2015]
(2) I – A II – B III – C IV – D
(A) Long spells of rainy weather
(3) I – B II – A III – D IV – C
(B) the presence of many large river courses
(4) I – D II – B III – C IV – A
(C) Fluctuation of the level of the understand water table
(D) Uncertain and uneven occurrence of rains in the plains

SECTION 8. ECONOMICS
1. Golden Revolution in India is related to: (Delhi 2020-21) (4) From July 1st to 7th
(1) Jewelry Export 7. The World Trade Organization was founded in __________.
(2) Gold mines (West Bengal-2020-21)
(3) Honey and Horticulture (1) 1990 (2) 1995
(4) Electronic Goods (3) 2000 (4) 2005
2. When an able person is willingly unemployed while there 8. In which economy is the policy of Laissez-faire adopted?
is an opportunity to work is known as (Delhi 2020-21) (West Bengal-2020-21)
(1) Disguised unemployment (1) Capitalist Economy (2) Socialist Economy
(2) Voluntary unemployment (3) Mixed Economy (4) Any Economy
(3) Seasonal unemployment 9. Exclusion principle is not applicable in the case of.
(4) Educated unemployment (West Bengal-2020-21)
3. Which of the following prepares the Human Development (1) Capital goods (2) Consumer goods
Report? (Delhi 2020-21) (3) Public goods (4) Private goods
(1) Planning Commission of India 10. Railway in India highlighted by which of the following
(2) International Monetary Fund market form? (West Bengal-2020-21)
(3) World Health Organisation (1) Perfect competition (2) Monopolistic competition
(4) United Nation Development Programme (3) Monopoly (4) Oligopoly
4. In which city of Haryana is the automobile industry 11. Which of the following taxes follows the ability to pay
situated? (Delhi 2020-21) principle? (West Bengal-2020-21)
(1) Faridabad (2) Gurgaon (1) Wealth Tax (2) Entertainment Tax
(3) Panipat (4) Sonipat (3) Goods and Services Tax (4) Excise Duty
5. Consider the following taxes: (Andhra Pradesh-2020- 12. In underdeveloped countries most of the labour force is
21) generally engaged (West Bengal-2020-21)
(A) Corporate tax (B) Property tax (1) Industrial sector (2) Service sector
(C) Income tax (D) Service tax (3) Agricultural sector (4) Banking sector
Which of the above is/are NOT included in direct tax? 13. The nation having a mixed economy is
(1) A only (2) C only (Rajasthan-2020-21)
(3) A, B and C only (4) D only (1) China (2) India
6. In our country as per Indian National Co-operative Union (3) Japan (4) France
guidelines all the states conduct cooperative week 14. The Central bank of India is (Rajasthan-2020-21)
celebrations: (Andhra Pradesh-2020-21) (1) State Bank
(1) From November 14th to 20th (2) Indian Industrial Development Bank
(2) From January 10th to 16th (3) Export-Import Bank
(3) From April 10th to 16th (4) Reserve Bank of India
S-42 Target NTSE
15. The process which increases the role of private sector in (3) A large section of farmers are unorganised labourers.
economy, is called (Rajasthan-2020-21) (4) Migration is the major challenge faced by organised
(1) Liberalization (2) Privatization sector labourers.
(3) Globalization (4) Swadeshi 24. Choose the correct group of Direct tax:
16. At present which form of money is increasingly used apart (Karnataka-2020-21)
from paper money? (Bihar-2020-21) (1) Corporate tax, Wealth tax, Central Excise tax, Service Tax.
(1) Commodity money (2) Metallic money (2) Income Tax, Stamp Duty, Service Tax, Export Tax.
(3) Plastic money (4) All of the above (3) Value Added Tax, Service Tax, Corporate Tax, Export Tax.
17. Mixed economy means an economy where there is (4) Income Tax, Corporate Tax, Wealth Tax, Stamp Duty.
(Bihar-2020-21) 25. Identify the correct statements related to Antyodaya Anna
(1) Existence of Capitalism Yojana. (Karnataka-2020-21)
(2) Privatization, Liberalization and globalization i. It was launched in July 2001.
(3) Existence of both public and private sectors ii. Under this scheme one crore of the poorest among the
(4) Growing crop along with rearing animals BPL families covered under the targeted public
18. __________ scheme was launched in 1970-1971 in distribution system were identified.
Maharashtra. (Maharashtra-2020-21) iii. Poor families were identified by the respective state
(1) Rural Water Supply rural development departments through a BPL survey.
(2) Nutritious diet iv. Fifty kilograms of food grains were made available to
(3) To supply more electricity for pumps each eligible family.
(4) Pulse Polio Vaccination (1) Only i and ii are correct (2) Only ii and iii are correct
19. Paper Money Spread to Europe through the travels of (3) Only iii and iv are correct (4) Only i and iv are correct
________ (Maharashtra-2020-21) 26. Which of the following organisations looks after the credit
(1) Marcopolo (2) Columbus needs of agriculture and rural development in India?
(3) Magellan (4) Stanley (Delhi 2019)
20. _______ is an innovator of new ideas and business (1) FCI (2) IDBI (3) NABARD (4) SBI
processes (Maharashtra-2020-21)
27. How many phases are there in circular flow of income?
(1) Entrepreneur (2) Professionals
(3) Businesspeople (4) Organizer (Delhi 2019)
21. The List -A contains the laws and the List-B with their year (1) 2 (2) 3 (3) 6 (4) 5
of implementation. Choose the correct option that matches 28. Which is the tenth largest stock exchange in the world and
exactly. (Karnataka-2020-21) oldest stock exchange in South Asia? (Tamil Nadu 2019)
List - I List - II (1) National Stock Exchange
A. Minimum wages act i. 1956
(2) Madras Stock Exchange
B. Prevention of unethical ii. 1993
activities act (3) Bombay Stock Exchange
C. Prohibition of bonded iii. 2005 (4) Calcutta Stock Exchange
labour act. 29. Indicators involved in Human Development Index (HDI)
D. Human Rights iv. 1976 (Karnataka 2019)
protection act (1) National Income, Employment Rate and Sex Ratio
v. 1948 (2) Per Capita Income, Life Expectancy and Literacy Rate
(1) A - i, B - ii, C - v, D - iii (2) A - ii, B - iii, C - i, D - iv of women
(3) A - v, B - i, C - iv, D - ii (4) A - iv, B - v, C - ii, D - i (3) Life Expectancy, Literacy, Attainment and Purchasing
22. Read the given statements and select the correct answer : Power of People
(Karnataka-2020-21) (4) National Income, Purchasing power of People and Sex
Assertion (a): Journal is called the book of original entry. Ratio
Reason (r): The full particulars of the transactions are 30. Aspects of Money supply concepts are given below.
recorded first in the Journal.
(Karnataka 2019)
(1) Both ‘a’ and ‘r is true and ‘R is the correct explanation
of ‘a’ A. Currency Notes
(2) Both ‘a’ and ‘Eis true and ‘R is not the correct B. Coins
explanation of ‘a’ C. Savings deposits in Post Office
(3) ‘a’ is true and ‘R is false D. Time/term deposits of Commerical Banks
(4) ‘a’ is false and ‘R is true The Group which classifies the above aspects as ‘Narrow’
23. One of the statements related to Labourers is wrong. and ‘Broad’ Money respectively are
Identify it: (Karnataka-2020-21) (1) A, B and C, D (2) A, C and B, D
(1) Organised labourers come under the gratuity Act of
(3) A, D and B, C (4) B, D and A, C
1971.
(2) Unorganised sector labourers are responsible for child
labour.
Stage - I Solved Questions S-43

31. Removing barriers or restriction set by the government is (a) Money (b) Land
called (Chandigarh 2019) (c) Labour (d) Capital
(1) Liberalization (2) Investment 43. The principle based on which the insurer is liable only for
(3) Favourable trade (4) Free trade those losses which have been insured against is
32. “We have not inherited the world from our forefathers - we [Karnataka 2018]
have borrowed it from our children”. (1) Principle of indemnity
(Andhra Pradesh 2019) (2) Principle of contributions
- This quote expects us : (3) Principle of loss minimization
(1) to use non-renewable resource extensively. (4) Principle of proximate cause
(2) to extract more ground water. 44. Match the items in List-A Entrepreneurs with List-B
Entrepreneurship [Karnataka 2018]
(3) to prefer sustainability of development.
a. Naresh Goyal I. Infosys
(4) to prefer rapid industrialisation. b. Kiran Mazumdar II. Reliance Shah
33. Which of the following is considered as a component of c. Narayan Murthy III. Balaji Telefilms
social infrastructure? [Delhi 2018] d. Dhirubai Ambani IV. Biocan
(1) Transport (2) Education V. Jet Airways
(3) Communication (4) Energy a b c d
34. Which bank first introduced credit card in India (1) V IV I II
(2) IV V I II
(1) Central Bank ofIndia (2) State Bank of India
(3) III V I II
(3) ICICI Bank (4) HDFC Bank
(4) V III IV II
35. In which five year plan, Mahalanobis Model was adopted 45. Identify the group of Financial Institutions with gives loan
in India? [Delhi 2018] to small scale industries. [Karnataka 2018]
(1) Fifth (2) First (1) IDBI, IFCI, SFC, SIDBI
(3) Second (4) Third (2) RBI, NABARD, IFCI, IDBI
36. In India the currency note is issued by [Rajasthan 2018] (3) NABARD, IDBI, RBI, SFC
(1) Reserve Bank of India (4) SIDBI, RBI, NABARD, IDBI
46. Whom would you consider as unemployed related to the
(2) State Bank of India
following situations? [Karnataka 2018]
(3) NABARD A. A farmer in farm produces 100 kg paddy in one acre of
(4) Bank of India land during every season. In addition one year his
37. The source of institutional credit is [Rajasthan 2018] adult son Raju joined him in farming. But output
(1) Money lender (2) Landlord remained the same.
(3) Bank (4) Relatives. B. Raghu has income by his huge property to lead
comfort life. So he does not work.
38. The example of tertiary sector is [Rajasthan 2018]
(1) Raju is unemployed
(1) Agriculture (2) Fisheries (2) Raghu is unmeployed
(3) making sugar from sugarcane (3) Both Raju and Raghu are unemployed
(4) Banking services (4) Both Raju and Raghu are not unemployed
39. If national income increases at a higher rate than population 47. Indicators of development are given below.
the per capita income [West-Bengal 2018] [Karnataka 2018]
(a) increases (b) decreases a. National Income
(c) remains same (d) may increase or decrease b. Per capita Income
40. To control the situation of deflation it is necessary to c. Life expectancy
[West-Bengal 2018] d. Educational Achievement
(a) increase the demand for bank loan e. Rate of women’s job participation
(b) decrease the demand for bank loan f. Standard of living
(c) decrease the purchasing power of the people Which of the above indicators are measured in Human
(d) increase national saving Development Index?
41. Economic rent is the price paid for the use of (1) a, b, c (2) c, d, f
[West-Bengal 2018] (3) a, b, e (4) d, e, f
(a) land only (b) scarce resources 48. Consider the following statements.
(c) machinery only (d) building only [Andhra Pradesh 2018]
42. Which of the following is not a factor of production?
[West-Bengal 2018]
S-44 Target NTSE

(i) Equitable allocation of resources. (1) A - II, B - III, C - I, D - IV


(ii) Generation of employment. (2) A - I, B - II, C - III, D - IV
(iii) Tax concession to big corporates. (3) A - III, B - I, C - II, D - IV
(iv) Universalisation of public distribution. (4) A - IV, B - II, C - I, D - III
Which of the factors given above can bring inclusive 54. If Life Expectancy Index for a country is 0.53, Educational
growth in our country ? Attainment Index is 0.67 and Per Capital Real GDP Index is
0.42, then HDI for the country will be [Odisha 2017]
(1) (i), (ii), (iii) (2) (i), (ii), (iv)
(1) 0.93 (2) 0.70
(3) (i), (iii), (iv) (4) (ii), (Hi), (iv) (3) 0.54 (4) 0.68
49. Which of the following is wrong related to Antyodaya 55. If cash reserve ratio of banks is 20% and currency reserve in
Anna Yojana ? [Andhra Pradesh 2018] the banking system amount to 50 million rupees, the
(1) Antyodaya Anna Yojana was launched in December maximum amount of demand deposits which can be created
by the banks is [Odisha 2017]
2000.
(1) 200 million rupees (2) 250 million rupees
(2) 2 crore families have been covered under the (3) 500 million rupees (4) 1000 million rupees
Antyodaya Anna Yojana. 56. The Government of India supplies food grains and other
(3) Wheat is supplied at the rate of Rs.6 and rice at the essential commodities of BPL households through fair price
rate of Rs.7 under this scheme. shops. Name of the programme is [Odisha 2017]
(4) None of the above. (1) ICDS (2) MDM
(3) PDS (4) Antodaya
50. Find out the correct one related to under employment.
57. Identify incorrect sentence related to Asian continent:
[Andhra Pradesh 2018] [Maharashtra 2017]
(1) They do not want to work. (1) This continent is the biggest of all from the
(2) They work in a lazy manner. perspectives of area and population
(3) They work less than what they are capable, of doing. (2) The continent got the name from the word 'Aasu'
(3) The renaissance era was started from this continent
(4) They are not paid for their work. (4) The emergence of old religion and culture this continent
51. Find out the wrong one about Secondary sector. 58. ‘NABARD’ is a __________ [Delhi 2016]
[Andhra Pradesh 2018] (1) Poverty alleviation Programme
(1) Secondary sector is also called as industrial sector. (2) Bank
(2) Manufacturing of bricks and sugar come under (3) Social Security Scheme
secondary sector. (4) Agriculture Marketing agency
(3) The share of secondary sector is more in current 59. What was the aim of ‘Antyodaya Programme’?
GDP of India. [Delhi 2016]
(4) None of the above. (1) Helping the poorest of poor
52. Find out the wrong one related to Annapurna Scheme (2) Upliftment of SC/ST
(APS). [Andhra Pradesh 2018] (3) Women empowerment
(1) Introduced in the year 2000. (4) Children welfare
(2) A scheme meant for indigent senior citizens. 60. ‘MNREGA’ is reffered to- [Chhattisgarh 2016]
(3) 10 kg of food grains are supplied freely under the scheme. (1) Right to information (2) Right to Education
(4) None of the above. (3) Right to work (4) Right to protection of life
53. Match List-I with List-II and select answer using the 61. Which will be a feature of the developing country?
appropriate code from among the following alternatives. [Chhattisgarh 2016]
(1) Agriculture as the main occupation
[Odisha 2017]
(2) Mass poverty
List-I List-II
(3) Mass illiltracy
A. Disparities in income in (I) Trickle Down theory (4) High technological development
a developing economy 62. The most appropriate measure of a country's economic
B. Economic development (II) Change in growth is [Jharkhand 2016]
benefits the poor occupational structure (1) Gross Domestic Product
(2) Net Domestic Product
C. Shifting of labour from (III) Less redistribution of
(3) Net National Product
agricultural to non income in favour of (4) Per Capita Product
agricultural sector poor 63. Finance is distributed between the centre and the states on
D. Increase in the (IV) Human development the recommendations of which of the following ?
capabilities of people (1) Planning Commission [Jharkhand 2016]
Stage - I Solved Questions S-45

(2) Public Accounts Committee 67. Inflation indicates a situation of [West Bengal 2016]
(3) Finance commission (1) very high prices (2) continuous fall in prices
(4) National Development Council (3) continuous rise in prices (4) very low prices
64. The first Agricultural University of the country is 68. Who is known as the father of Green Revolution?
(1) J.N.K.V. Jabalpur [Jharkhand 2016] [Maharashtra 2015]
(2) G..B.P.A.U. Pant Nagar (1) Verghese Kurien (2) Babu Jagjivan Ram
(3) P.A.U. Ludhiana
(3) M. S. Swaminathan (4) M. S. Subhramanyam
(4) R.A.U. Bikaner
69. An example of Indirect Taxes is: [Maharashtra 2015]
65. Creation of utility for exchange is called
(1) Income Tax (2) Corporate Tax
[West Bengal 2016]
(3) Estate Tax (4) Entertainment Tax
(1) Consumption (2) investment
70. Which academy supports food security programme?
(3) Production (4) Savings
[Maharashtra 2015]
66. In which of the following situation total demand for a
(1) Academy of Development Science
necessary commodity will rise? [West Bengal 2016]
(2) Academy of Crop Science
(1) Rise in price level
(3) Academy of Food Security
(2) Rise in population size
(4) Academy of Grain Science
(3) Rise in production
71. Which summit of BRICS held in 2014?[Maharashtra 2015]
(4) Rise in the rate of interest (1) 5th (2) 6th (3) 4th (4) 7th

SECTION 9. MATHEMATICS

1. P, R and S are three persons with ages 26 years, 27 years and


x y x
28 years respectively. In what ratio must they invest money 5. If = = = a, then the value
at 10% per annum compounded yearly so that each gets 2x + y + z x + 2 y + z x + y + 2 z
same sum at the age of their retirement? of a = _______ (Given x + y + z ¹ 0) (AP_FIITJEE 2020-21)
(AP_FIITJEE 2020-21) 1 1 1 1
(1) (2) (3) (4)
(1) 100 : 121 : 132 (2) 100 : 110 : 121 3 4 8 2
(3) 80 : 100 : 99 (4) 89 : 95 : 100 6. "Cards marked with numbers 13, 14, 15, ......, 60 are placed in
2. If the mean of 28, 34, 41, 23, 45, 18, 21 is 'x', then the value of a box and mixed thoroughly. One card is drawn at random
2 – logx2 – logx3 – logx5 is ______ (AP_FIITJEE 2020-21) from the box." Read the information carefully and match the
1 following : (AP_FIITJEE 2020-21)
(1) 1 (2) –1 (3) 0 (4)
2 1
(i) The probability of the number that is (p)
4
3 x 2 (5 - x ) x 2 + x + 3 on the card drawn is divisible by 5.
3. If x + = 5, then the value of 2 + =?
x x + x + 3 x 2 (5 - x )
36
(ii) The probability of the number that is (q)
(AP_FIITJEE 2020-21) 48
one the card drawn is a prime
5 9
(1) 2 (2) 6 (3) (4)
2 2 5
(iii) The probability of the number that is (r)
4. The average age of students of a class is 15.8 years. The 24
average age of boys in the class is 16.4 years and that of the on the card drawn is a multiple of 19.
girls is 15.4 years. The ratio of the number of boys to the 1
number of girls in the class is _____ (AP_FIITJEE 2020-21) (iv) The probability of the number that is (s)
16
(1) 1 : 2 (2) 2 : 3
on the card drawn is a composite number.
(3) 3 : 4 (4) 3 : 5
S-46 Target NTSE

(1) (i)-(s), (ii)-(r), (iii)-(q), (iv)-(p) 15. A 'p' m long wire is cut into two pieces one of which is bent
into a circle and the other into a square enclosing the circle.
(2) (i)-(s), (ii)-(q), (iii)-(p), (iv)-(r)
What is the radius (in meter) of the circle? (Delhi 2020-21)
(3) (i)-(p), (ii)-(q), (iii)-(r), (iv)-(s)
p 2p p 2 pp
(4) (i)-(r), (ii)-(p), (iii)-(s), (iv)-(q) (1) (2) (3) (4)
p+ 4 p+ 4 2p + 8 p+ 4
7. Three numbers prime to each other are such that the product
of the first two is 437 and the product of the last two is 551. 16. If (x + k) is a common factor of (x2 + px + q) and
The sum of the numbers is _______ (Bihar 2020-21) (x2 + lx + m), then the value of k is : (Delhi 2020-21)
(1) 59 (2) 63 (3) 69 (4) 71
l-p m-q
8. If the sum of the remainders obtained by dividing each of (1) l+p (2) m-q (3) (4)
m-q l- p
x3 + 8 x 2 - 3kx + 7 and 2 x 3 + kx 2 - 5 x + 6 by x – 1 is 9 then
17. As a result of 40% hike in the price of rice per kg., a person is
k = _______ (Bihar 2020-21)
able to purchase 10 kg less rice for Rs. 1400. What was the
(1) 0 (2) 2 (3) 3 (4) 5 original price of rice per kg? (Delhi 2020-21)
9. If th e equations x 2 + bx + a = 0 and x 2 + ax + b = 0, (1) Rs. 50 (2) Rs. 60
(a ¹ b) have equal roots, then a + b = ______ (3) Rs. 40 (4) Rs. 30
18. One box contains four cards numbered 1, 3, 5 and 7 and
(Bihar 2020-21) another box contains four cards numbered as 2, 4, 6 and 8.
(1) –1 (2) 0 One card is drawn from each box at random. The probability
(3) 1 (4) None of these that the product of the numbers so drawn is more than 14 is:
10. In a right-angled triangle, the difference of the hypotenuse (Delhi 2020-21)
and the base is 2 cm. The hypotenuse exceeds twice the 1 7 3 5
height by 1 cm. The base of the triangle is ______ cm. (1) (2) (3) (4)
2 10 8 8
(Bihar 2020-21)
19. If sin A + sin 2 A = 1 (A is acute angle) then the value of
(1) 8 (2) 15
(3) 17 (4) 21 cos 2 A + cos 4 A is: (Karnataka 2020-21)
(1) 0 (2) 1 (3) 2 (4) 3
1 1 20. The cumulative frequency curve for a distribution is
11. If in an A.P., the pth term = and the qth term = , then
q p represented in the graph. The median for the data is :
the pqth term (Bihar 2020-21) (Karnataka 2020-21)
(1) –1 (2) 0
(3) 1 (4) None of these
60
Cumulative Frequency

2 2
12. If x = 2sin q , y = 2cos q
for all real values of q, then 50
40 Less than ogive
(Bihar 2020-21)
(1) x + y = 1 (2) x + y = 2 30
More than ogive
20
(3) x+ y £ 2 2 (4) x+ y ³ 2 2
10
13. If DABC is an equilateral triangle such that A(2, 2) and
centroid of the triangle is (–2, 2) then find the length of its 0 10 20 30 40 50 60 70
side. (Delhi 2020-21) Weight (in kg)
(1) 4 units (2) 6 units
(3)4 3 units (4) 9 units (1) 10 (2) 30
14. In a DABC, AX bisects BC and AX is also the angle bisector (3) 40 (4) 60
of angle A. If AB = 12 cm, BX = 3 cm, then what is the area of 21. Four circular plates of same size having diameter 'a' units
DABC. (Delhi 2020-21) each are placed on the square mat such that each plate
touches externally the other two as shown in the figure. The
(1) 9 cm2 (2) 2 cm2
area of the shaded region is : (Karnataka 2020-21)
(3) 9 5 cm2 (4) 9 15 cm2
Stage - I Solved Questions S-47

out is blue is double of the probability that the ball drawn is


red. Find the total number of balls in the bag.
(Maharashtra 2020-21)
(1) 15 (2) 10 (3) 5 (4) 20
29. If a + b = 2 3, and ab = 3, then a 4 + b 4 = ?
(Maharashtra 2020-21)
(1) 14 (2) 16 (3) 18 (4) 20

a2 30. PQRS is a square, PQ = 7 3 cm with centre R and radius


(1) (4 + p) square units RS, sector R QAS is drawn. Find the area of the shaded portion.
4
(Maharashtra 2020-21)
a2 P S
(2) (4 - p) square units
4

(3) 4a 2 (4 + p) square units A

(4) 4a 2 (4 - p) square units


22. The value of 'p' for which the pair of linear equations Q R
px + 3 y - ( p - 3) = 0 and 12 x + py - p = 0 has infinitely (1) 29.5 cm2 (2) 17.5 cm2
many solutions is : (Karnataka 2020-21) (3) 23.7 cm2 (4) 31.5 cm2
1
(1) 0 (2) –6 (3) (4) 6 31. Numerator of a fraction is 3 less than its denominator. If 4 is
6 added to the numerator and 2 is subtracted from the
23. 10th term of an AP is 16 and 16th term of it is 10. The common denominator then the numerator becomes 2 times of the
difference and the 26th term of the AP are respectively. denominator. What will be the fraction?
(Karnataka 2020-21) (Rajasthan 2020-21)
(1) 1 and 0 (2) –1 and 0
(3) 0 and 1 (4) 0 and –1 5 4 2 5
(1) (2) (3) (4)
24. The angles of depression of two ships as observed from the 2 5 5 4
top of a 80 m high light house from the sea-level are 30° and 32. The measures of three angles of a triangle are in ratio
45°. If two ships are on the same side of the light house and 3 : 4 : 5, then the measure of its greatest angle is -
all the three are along a straight line, then distance between (Rajasthan 2020-21)
the two ships is : (Karnataka 2020-21) (1) 55° (2) 65° (3) 80° (4) 75°
2
33. Each face of a cube is 144 cm . If cube is cut by a plane,
(1) 80 m (2) 80( 3 - 1) m
parallel to it's base in two equal parts, then the total surface
(3) 80( 3 + 1) m (4) 80 3 m area of both the parts will be - (Rajasthan 2020-21)
(1) 1152 cm 2 (2) 1052 cm2
25. A storm broke a tree and the tree top rested 13 6 m from 2
(3) 288 cm (4) 576 cm2
the base of the making an angle 45° with the horizontal. Find
the height of the tree (Maharashtra 2020-21) cos A
34. If cosec A = 2, then the value of tan A + will be
(1) (2) 13 3(2 + 2) m 1 + sin A
26 6 m
(Rajasthan 2020-21)
(3) 26 3 m (4) 26 2 + 13 3 m
1 2 1
3 (1) (2) (3) (4) 2 3
26. If sin q + cos q = then sin q × cos q = ? 2 3 3 3
2
35. Following are the points obtained by a Kabaddi team in
(Maharashtra 2020-21) various matches. (Rajasthan 2020-21)
2 5 17, 2, 7, 27, 15, 5, 14, 19, 10, 24, 9, 8, 6, 18, 28
(1) 1 (2) (3) (4) 0
3 8 the median of the points obtained by the team will be -
27. The sum of the 3rd and 8th term of an A.P. is 7 and the sum of (Rajasthan 2020-21)
the 7th term and 14th term is –3. Find the 10th term of the A.P. (1) 14 (2) 16 (3) 15 (4) 17
(Maharashtra 2020-21)
(1) –1 (2) +1 (3) 0 (4) 2 36. Discriminant of the quadratic equation 3 3 x 2 + 7 x + 3 = 0
28. A bag contains 5 red and some blue balls. One ball is taken will be (Rajasthan 2020-21)
out of the bag a random. The probability that the balls taken (1) 49 (2) 13 (3) 36 (4) 62
S-48 Target NTSE
37. In a party of 100 people, 60 had Vanilla ice-cream and 40 had 2
chocolate ice-cream. 10 people did not have any ice-cream. (2) (4) None of these
13
How many had only chocolate ice-cream?
46. The minimum value of sinqcosq is (Uttar Pradesh 2020-21)
(Tamil Nadu 2020-21)
(1) 10 (2) 0 (3) 20 (4) 30 1 1
(1) 0 (2) –1 (4)
(3) -
2 2
38. A B 47. If radius of a right circular cylinder is increased by 10%, then
28°
by what percent its height should be decreased so that its
E volume remains unchanged
y (Uttar Pradesh 2020-21)
40° (1) 17.26% (2) 17.36%
C D (3) 17.46% (4) None of these
In the given figure, AB is parallel to CD. The angle y is : 48. If x + 1 - x - 1 = 1 then value of x is
(Tamil Nadu 2020-21)
(Uttar Pradesh 2020-21)
(1) 68° (2) 34° (3) 112° (4) 56°
2
39. If the total surface area of a cube is 96 cm , then its volume 5 2 4 3
is : (Tamil Nadu 2020-21) (1) (2) (3) (4)
4 3 5 5
(1) 64 cm 3 (2) 512 cm3 49. The solution set for the equation
(3) 8 cm 3 (4) 27 cm3 2 2
40. The probability of choosing a vowel in the word 'EDUCATION' 32 x - 2.3x + x + 2 + 32( x + 2) = 0 is (West Bengal 2020-21)
is : (Tamil Nadu 2020-21) (a) {–1, 2} (b) {0, 2}
3 4 5 7 (c) {0, –1} (d) {2, 4}
(1) (2) (3) (4) x+y x–y
50. For 3 = 81, 81 = 3, we get (West Bengal 2020-21)
9 9 9 9
41. A girl of height 100 cm is walking away from the base of a 21 21
lamp post at a speed of 1.9 m/sec. If the lamp is 5 m above the (a) no solution (b) x+ ,y =
2 2
ground, the length of the shadow after 4 sec. is :
(Tamil Nadu 2020-21) 2 17 15
(c) x = 2, y = (d) x = , y =
(1) 160 cm (2) 180 cm 3 8 8
(3) 200 cm (4) 190 cm 51. 125 identical cubes are cut from a big cube and all the smaller
cubes are arranged in a row to form a long cuboid. What is
42. If the base circumference of a right circular cone and its
the percentage of increase in total surface area of the cuboid
height are 220 cm and 21 cm respectively, then the volume of
over the total surface area of the cube?
the cone is : (Tamil Nadu 2020-21)
(1) 24,650 cm3 (2) 21,750 cm3 (West Bengal 2020-21)
(3) 26,950 cm3 (4) 25,400 cm3 2 1
x y z (a) 234 % (b) 235 %
43. If a = b, b = c & c = a, then the value of xyz is 3 3
(Uttar Pradesh 2020-21)
2
1 (c) 134 % (d) None of these
(1) 1 (2) 0 (3) (4) abc 3
abc
3 -1 3 5 19
= a + b 3 , then the value of ‘a’ and ‘b’ is 52. The value of + + .... + is
44. If 2
1 .2 2 2
2 .3 2
9 .10 2
2
3 +1
(Uttar Pradesh 2020-21) (West Bengal 2020-21)
(1) a = 2, b = –1 (2) a = 2, b = 1 99 1 101
(a) (b) (c) (d) 1
(3) a = –2, b = 1 (4) a = –2, b = –1 100 100 100
45. What is the probability that a leap year contains 53 Sundays 53. Given : 0° < q < 90°, then, if sinq + cosq = x, which one of the
following is correct? (West Bengal 2020-21)
(Uttar Pradesh 2020-21) (a) x < 1 (b) x > 1.5

2 7 (c) 1£ x £ 2 (d) 1 < x £ 2


(1) (2)
7 13 54. The ratio of in which 9x – 3y – 14 = 0 divides the join of
(2, –4) and (3, 7) is (West Bengal 2020-21)
(a) 2 : 1 (b) 1 : 2
(c) 2 : 3 (d) 3 : 2
Stage - I Solved Questions S-49

55. A square is inscribed in a circle of radius ‘a’. Another circle 65. In a hostel 60% of students read Hindi newspapers, 40%
is inscribed in that square and again a square is inscribed in read English and 20% read both. A student is selected at
this circle. The side of this square is : [Delhi 2019] random. Find the probability that she reads neither Hindi
nor English newspaper. [Himachal Pradesh 2019]
a a
(1) 2a (2) (3) (4) a 1 1 1
2 2 (1) 0 (2) (3) (4)
5 3 2
56. If a cos q - b sin q = c, then a sin q + b cos q = ?
66. The centroid of a triangle is (2, 4) and circumcentre is (1, 7)
[Delhi 2019] then find the orthocentre. [Himachal Pradesh 2019]
(1) ± a 2 + b2 + c2 (2) ± a 2 + b2 - c2 (1) 3, 1 (2) 4, –2
(3) –2, 4 (4) –8, 4
(3) ± a 2 - b2 + c2 (d) ± a 2 - b2 - c 2 67. If 39 + 312 + 315 + 3n is a perfect cube (of an integer) where
57. If x2 – 3x + 2 is a factor of x4 – px2 + q, then the value of p and n Î N, then find the value of n. [Telangana 2019]
q are respectively are [Delhi 2019] (1) 18 (2) 14 (3) 16 (4) 17
(1) –5, 4 (2) –5, –5
(3) 5, 4 (4) 5, –4 1 2x
68. If x + = 5, then 2 is equal to .......
58. If x 1 , x 2 , x 3 , ..., x n are in A.P., then the value of x 3x - 5 x + 3
1 1 1 1 [Telangana 2019]
+ + + ... + is: [Delhi 2019]
x1 x2 x2 x3 x3 x4 xn -1 xn 1 1
(1) 5 (2) (3) 3 (4)
5 3
n -1 n -1 n n +1 69. If log4 7 = x, then the value of log7 16 will be ......
(1) (2) (3) (4)
x1 xn x2 xn -1 x1 xn x1 xn [Telangana 2019]
59. If x, y and z are positive real numbers and a, b and c are
rational numbers, then value of [Delhi 2019] 2
(1) x2 (2) 2x (3) x (4)
x
1 1 1 70. In DABC, D, E and F are respectively mid points of the sides
b-a c -a
+ a -b c -b
+ b -c a -c is BC, CA and AB and P is a point on BC such that AP^ BC.
1+ x +x 1+ x +x 1+ x +x If ÐDEF = 50º, then ÐFPD = [Andhra Pradesh 2019]
(1) –1 (2) 1 (3) 0 (4) 2
A
60. The value of [Bihar 2019]

π 3π 5π 7π 9π 11π 13π
sin sin sin sin sin sin sin is E
14 14 14 14 14 14 14 F
50º
1 1
(1) (2)
16 64
B C
1 P D
(3) (4) None of these
128 (1) 120° (2) 110° (3) 135º (4) 130º
61. The mean of 25 observations is 36. If the mean of the first 13
observations is 32 and that of the last 13 observations is 39 71. If x < 1, y < –1, then (x–1, y – 3) lies in
then the 13th observation is [Bihar 2019]
[Andhra Pradesh 2019]
(1) 32 (2) 30 (3) 28 (4) 23
62. The capacity of two pots are 240 litres and 112 litres (1) Q3 (2) Q4 (3) Q2 (4) Q1
respectively. Find the capacity of a container which can 72. If ax–1 = bc, by–1 = ca, cz–1 = ab, then xy + yz + zx
exactly measure the contents of the two pots
[Andhra Pradesh 2019]
[Jharkhand 2019]
(1) xyz (2) 0
(1) 9000 cm3 (2) 12000 cm3
1 1 1
(3) 16000 cm3 (4) 8000 cm3 (3) 1 (4) + +
x y z
63. A sum becomes 6 times at 5% per annum. At what rate, the
sum becomes 12 times? [Jharkhand 2019] 73. If ax = by+z then [Andhra Pradesh 2019]
(1) 10% (2) 12% log a x log b y + z
(3) 9% (4) 11% (1) = (2) =
log b y + z log a x
64. If p, q, r are the roots of the equation x3 + 5x2 – 16x + 48 = 0
then value of the expression p(qr + q + r) + qr is log a y + z
[Himachal Pradesh 2019] (3) = (4) x log a = yz log b
log b x
(1) 43 (2) –32 (3) –64 (4) 32
S-50 Target NTSE
74. In the given figure. AB || ED and BC || EF, then the value of (1) A and C (2) B, C and D
ÐABC + ÐDEF is [Rajasthan 2019]
(3) A, B and C (4) A, C and D
E F
A 81. Read the following statements carefully and choose the
correct alternative. [Maharashtra 2019]
D (A) The slope of the line parallel to X-axis can be derived
B x2 - x1
C
by the formula
y2 - y1
(1) 90º (2) 180º (3) 120º (4) 360º
75. In the given figure, AB = AC, ÐBAC = 40º, BE and CD are (B) The slope of the line parallel to Y- axis is 1
angle bisectors of ÐB and ÐC respectively. If ÐDOE = x, the (C) The cotangent ratio of an angle made by the line with
value of x is [Rajasthan 2019] the positive direction of X- axis is called the slope of
A that line.
(D) The slope of the line which makes acute angle with
40º
D E X- axis is less than zero and the slope of the line making
x
obtuse angle with X-axis is greater than zero.
O Alternative:
B C (1) Statement A and B correct
(1) 140º (2) 70º (3) 110º (4) 40º (2) Statement C and D correct
76. The shadow of a tower, when the angle of elevation of the (3) Only statement C is wrong
sun is 30º is found to be 10 metre longer than when it was (4) All statements are wrong
60º. The height of the tower will be [Rajasthan 2019] 82. In the figure, semi-circles are drawn whose centre are X, Y, Z
respectively. Points (X, Y, Z) are collinear points. AX = 2.5,
(1) 5 3m (2) 5 ( )
3 -1 m
BY = 6.5, CZ = 8.5 and AP + QC = 16; QC + CR = 27 and
CR + AP = 19 then find the value of AP + PB + BQ + QC
(3) 5 ( 3 +1 m ) (4) 3 5m = CR + RD = ? [Maharashtra 2019]
77. If the sum of ‘n’ terms of an arithmetic progression is
Sn = 3n + 2n 2 then its common difference is P Q R
[Karnataka 2019]
(1) 9 (2) 6 (3) 4 (4) 3
78. If the graphs of x–y = 2 and kx + y = 3 (k is constant) intersect A B C D
at a point in first quadrant then the value of k is X Y Z

[Karnataka 2019] (1) 37 (2) 41 (3) 53 (4) 47


(1) equal to 1 (2) greater than –1 83. In figure, PQ is a chord of a circle with centre O and PT is its
(3) less than 4 (4) between –1 and 4
3 3 tangent at P. If ÐQPT = 60º, then ÐPRQ is–
79. Three squares of a chess board are selected at random. The [Haryana 2019]
probability of getting two squares of one colour and other
of a different colour is [Karnataka 2019] Q
O
16 8 3 3
(1) (2) (3) (4)
21 21 32 8 R

5 7 P T
3 2 (1) 105º (2) 115º (3) 120º (4) 130º
80. 3 3 Choosse correct alternative for the value of 84. ABC is a right angled triangle, right angled at ÐB. If side AB
4 2 is divided into three equal parts by points D and E such that
determinant. [ Maharashtra 2019]
3 AC2 - EC 2
1 -1 æ -1 ö -1 D is nearest to A, than is equal to–
(A) (B) (C) ç ÷ (D) 3 DC2 - BC 2
8 8 è 2 ø 512
[Haryana 2019]
Stage - I Solved Questions S-51

1 1 (1) 12 cm (2) 9.6 cm


(1) 3 (2) 2 (3) 2 (4) 2
2 4 (3) 19.2 cm (4) 7.2 cm
95. If p + q + r = 0 then the value of
3+ 2 3
85. If = a + 3 b, then the value of a + b where a
3- 3 2p 2 ( q + r ) + 2q 2 ( p + r ) + 2r 2 ( p + q )
will be [Punjab 2018]
and b are rational number is [Rajsthan 2018] pqr
(1) 5 (2) 8 (3) 2 (4) 16 1
86. For which positive values of k and p, equations (1) 3pqr (2) (3) 6 (4) – 6
pqr
2x2 + px + 8 = 0 and p (x2 + x) + k = 0 have equal roots?
[Rajsthan 2018] 96. A circle is inscribed in a square of side 2.5 cm. Another circle
is circumscribing this square. The ratio of areas of outer
(1) k = 1, p = 4 (2) k = 2, p = 8
circle and inner circle is [Punjab 2018]
(3) k = 4, p = 8 (4) k = 2, p = 4
87. If a, b are zeros of polynomial x2 – p(x + 1) – k such that (1) 1 : 2 (2) 2 :1 (3) 2 : 1 (4) 3 :1
(a + 1) (b + 1) = 6, then value of k is [Rajsthan 2018] 1
97. If x = then find the value of 4x3 + 2x2 – 8x –3.
(1) 5 (2) – 1 (3) – 3 (4) – 5 3 –1
88. Which is unit digit of 618 – 510? [Rajsthan 2018]
[Punjab 2018]
(1) 5 (2) 8 (3) 1 (4) 9
89. In the following figure QT ^ PR and QS = PS. If ÐTQR = 40° (1) 0 (2) 2 (3) – 2 2 (4)
and ÐRPS = 20° then value of x is [Rajsthan 2018] 98. How many revolutions will a circular wheel of radius r units
P will make to cover a distantce of 100 times its diameter?
[Punjab 2018]
T 100 p 50
(1) (2) 100 p (3) (4)
p 100 p
x
99. If pth term of an AP is q and qth term is p then mth term of
Q R
S this AP will be:
(1) 80° (2) 25° (1) p + q + m (2) p + q – m
(3) 15° (4) 35° (3) p – q – m (4) p – q + m
1 1 100. Two dice are thrown. Find the probability that sum of
90. Which term of A.P. 20, 19 , 18 , ... is first negative term? numbers of both up sides of both dice is a perfect square.
4 2
[Rajsthan 2018] [Punjab 2018]
(1) 18th (2) 15th (3) 28th (4) 27th 7 5
1
91. The angles of elevation of the top of a 12 m high tower from (1) (2) (3) (4) 0
6 36 36
two points in opposite directions with it are complementary. 101. From the top of a tower of h m high, the angles of depres-
If distance of one point from its base is 16 m, then distance sion of two objects, which are in line with the foot of the
of second point from tower’s base is tower are a and b (a > b ). Find the distance between two
[Rajsthan 2018] objects. [Punjab 2018]
(1) 24 m (2) 9 m (3) 12 m (4) 18 m (1) h (tan a – tan b) (2) h (cot a – tan b)
cos A cos A (3) h (cot a – cot b) (4) h (cot a + cot b)
92. If m = and n = , then (m2 + n2) cos2 B is equal to
cos B sin B
102. If the distance between the points (4, q) and (1, 0) is 5 units
[Rajsthan 2018] then the value of q is: [Punjab 2018]
(1) m2 (2) n2 (3) m2 + n2 (4) m + n (1) 4 (1) – 4 (3) ± 4 (4) 0
93. If ratio of heights of two similar triangles is 4 : 9, then ratio
103. The area of an equilateral triangle is 49 3 cm2.
between their areas is [Rajsthan 2018]
Taking each vertex as centre, circles are described with
(1) 2 : 3 (2) 3 : 2
radius equal to half the length of the side of the triangle.
(3) 81 : 16 (4) 16 : 81
Find the area of the part of the triangle which is not
94. In a circle of 10 cm radius, two chords AB = AC = 12 cm, then
included in these circles.
the length of the chord BC is [Rajsthan 2018]
S-52 Target NTSE

22 112. If x2 + xy + x = 12 and y2 + xy + y = 18, then the value of


( 3 = 1.73, p = ) [Punjab 2018] x + y is ... [Andhra Pradesh 2018]
7
(1) 84 cm2 (2) 77.7 cm2 (3) 7.77 cm2 (4) 70.7 cm2 (1) 5 or – 6 (2) 3 or 4 (3) 5 or 3 (4) 6 or –3
113. If 217x + 131y = 913 and 131x + 217y = 827, then the value of
tan A tan B + tanAcotB sin 2 B x + y is [Andhra Pradesh 2018]
104. If A + B = 90° then - is
sin A sec B cos 2 A
(1) 8 (2) 5 (3) 7 (4) 6
equal to [Delhi 2018]
1
(1) cot A
2
(2) cot B2 114. If x = , (x ¹ 2), , then the value of x is
1
(3) – tan2A (4) – cot 2A 2-
1
105. The simplified form of the expression given below is 2-
[Delhi 2018] 2-x
y 4 - x4 y3
- [Andhra Pradesh 2018]
x( x + y) x
y 2 - xy + x 2 (1) 1 (2) 3 (3) 2 (4) 5
(1) 1 (2) 0 (3) –1 (4) 0 115. x1, x2, x3, ..., are in A.P. If x1 + x7 + x10 = – 6 and

x 2 - bx m - 1 x3 + x8 + x12 = – 11, then x3 + x8 + x22 = ?


106. If = , has roots which are numerically equal
ax - c m + 1 [Andhra Pradesh 2018]
but of opposite signs, the value of m must be (1) – 21 (2) – 15 (3) – 18 (4) – 31
[Delhi 2018]
(1) (a – b)/(a + b) (2) (a + b)/(a – b) 2 + 5 + 8 + ...n.terms 23
116. If = , then n value is
1 7 + 11 + 15 + ...n.terms 35
(3) c (4) [Andhra Pradesh 2018]
c
107. If we divide a two digit number by the sum of its digits we (1) 17 (2) 15 (3) 18 (4) 23
get 4 as quotient and 3 as remainder. Now if we divide that 117. If the co-ordinates of the midpoints of the sides of a triangle
two digit number by the product of its digits, we get 3 as are (1, 1), (2, –3) and (3, 4), then the centroid of the triangle
quotient and 5 as remainder. The two digit number is is ... [Andhra Pradesh 2018]
[Delhi 2018]
æ 1ö æ 2ö æ 2ö
(1) even (2) odd prime (1) çè 3, ÷ø (2) ç1, ÷ (3) (3, 1) (4) çè 2, ÷ø
3 è 3ø 3
(3) odd composite (4) odd
118. The area of two concentric circles are 1386 cm2 and
108. Four positive integers sum to 125. If the first of these
962.5 cm 2. The width of the ring is [Karnataka 2018]
numbers is increased by 4, the second is decreased by 4,
the third is multiplied by 4 and the fourth is divided by 4 we (1) 4.2 cm (2) 3.8 cm (3) 3.5 cm (4) 2.8 cm
find four equal numbers then four original integers are 119. On increasing the radius of the base and height of a cone
each by 20%, then the percentage increase in the volume
[Delhi 2018]
will be [Karnataka 2018]
(1) 16, 24, 5, 80 (2) 8, 22, 38, 57
(1) 20% (2) 40.8% (3) 60% (4) 72.8%
(3) 7, 19, 46, 53 (4) 12, 28, 40, 45
120. If – 4 is a root of the quadratic equation x 2 + px – 4 = 0 and
109. When 31513 and 34369 are divided by a certain three digit equation x2 + px + m = 0 has equal roots, then the value of
number, the remainders are equal, then the remainder is “m” is [Karnataka 2018]
[Andhra Pradesh 2018]
25 – 25
(1) 86 (2) 97 (3) 374 (4) 113 (1) – 4 (2) (3) + 4 (4)
4 4
110. The greatest number of four digits which when divided by 121. Statement-I: If 5 + 24 = x + y then x + y = 5 and
3, 5, 7, 9 leaves the remainder 1, 3, 5, 7 respectively, is
xy = 24
[Andhra Pradesh 2018]
(1) 9763 (3) 9673 (3) 9367 (4) 9969
Statement-II: The square root of 5 – 24 is ( ) ( 3– 2 )
111. e f g h is a four digit number. One hundredth of e f g h is the [Karnataka 2018]
mean of e f and g h, then the four digit number is (1) Both statement-I and II are wrong
[Andhra Pradesh 2018] (2) Statement-I is wrong, but statement-II is right
(1) 3648 (2) 4950 (3) 4590 (4) 3468 (3) Both statement-I and II are right
(4) Statement-I is right, but statement-II is wrong
Stage - I Solved Questions S-53

122. The value of cos2 50 + cos2 100 + cos2 150 + ... + cos2 850 + (1) 4 (2) 6 (3) 0 (4) 8
cos2 900 is [Karnataka 2018] x2 y2 z2
1 1 133. If = = = 2, the value of
(1) 9 (2) 9 (3) 8 (4) 8 by + cz cz + ax ax + by
2 2
123. If (x + a) is the factor of the polynomials (x2 + px + q) and c b a
+ + is [West Bengal 2018]
(x2 + mx + n) then the vlaue of ‘a’ is [Karnataka 2018] 2c + z 2b + y 2a + x
n–q m–p q–n m–p
(1) (2) (3) (4) 1 1
m–p n–q m–p q–n (1) 2 (2) (3) 4 (4)
2 4
124. The lowest common multiple of two numbers is 14 times 134. If log4[log4{log4.(log4.x)}] = 0, ‘x’ is equal to
their greatest common divisor. The sum of LCM and GCD is [West Bengal 2018]
600. If one number is 80 then other number is
(1) 256 (2) 418 (3) 2512 (4) 2564
[Karnataka 2018]
1 1
(1) 600 (2) 520 (3) 280 (4) 40 135. If x2 + y2 = z2 the value of + is
log z - y x log x + y x
125. The shaded protion in the given Venn diagram represents
[West Bengal 2018]
P Q U
(1) x (2) y (3) x + y (4) 2
136. If (x + 2) and (2x – 1) are factors of (2x3 + ax2 + bx + 10), the
value of (a2 + b2) is [West Bengal 2018]
(1) 338 (2) 218 (3) 74 (4) 198
a b
[Karnataka 2018] 137. If a + b = 2c, the value of + is
a-c b-c
(1) P¢ ÈQ¢ (2) P¢ ÇQ¢
[West Bengal 2018]
(3) (P¢ Ç Q¢)¢ (4) (P¢ È Q¢)¢
126. In D ABC, the altitudes AL, BM and CN are intersect at ‘O’. (1) 0 (2) 1 (3) 2 (4) – 1
The value of AN × BL × CM is same as 138. The compound interest for two years of the amount
[Karnataka 2018] Rs. 7500 at the rate of 8% per annum would be
(1) BN × LC × AM (2) AL × CN × BM [West Bengal 2018]
(3) OL × OM × ON (4) OC × OB × OA (1) Rs. 1,248 (2) Rs. 1,260
127. If the letters of the word “FATE” are arranged as in a (3) Rs. 1,300 (4) Rs. 1,352
dictionary without repetition, then the rank of the 139. A businessman fixed the selling price of an article after in-
arrangement of “FATE” is [Karnataka 2018] creasing the cost price by 40%. Then he allowed his cus-
(1) 12 (2) 13 tomer a discount of 20% and gained Rs. 48. The cost price
(3) 14 (4) 15 of the article is [West Bengal 2018]
128. If ax2 + bx + c = a(x – p)2, the relation among a, b, and c is (1) Rs. 200 (2) Rs. 248
[West Bengal 2018] (3) Rs. 400 (4) Rs. 448
(1) abc = 1 (2) 2b = a + c 140. The curved surface area of a right circular cylinder and that
(3) b2 = ac (4) b2 = 4ac of a sphere are equal. If their radii are equal, the ratio of their
volume is [West Bengal 2018]
129. The identity (x + 4)2 = x + 4 is possible, when (1) 3 : 2 (2) 2 : 3 (3) 3 : 4 (4) 4 : 3
[West Bengal 2018] 141. The sum of the length, breadth and height of a rectangular
(1) x £ – 4 (2) x ³ – 4 parallelopiped is 25 cm and its whole surface area is
(3) x £ – 16 (4) Not possible 264 sq. cm. The area of the square whose sides are equal to
130. The number of real roots of the quadratic equation 3x2 + 4 = the length of the diagonal of that parallelopiped is
0 is [West Bengal 2018] [West Bengal 2018]
(1) 0 (2) 2 (3) 1 (4) 4 (1) 256 sq. cm. (2) 361 sq. cm.
131. The solution of the equation 9x + 6x = 2.4x (3) 225 sq. cm. (4) 324 sq. cm
[West Bengal 2018] 142. The radii of two circles with center at A and B are 11 cm and
(1) 0 (2) 1 6 cm respectively. If PQ is the common tangent of the circles
(3) ± 2 (4) – 1 and AB = 13 cm, length of PQ is
3
132. If f(x) = 2x – 3x + 4, the value of f(x) + f(–x) is [West Bengal 2018]
[West Bengal 2018] (1) 13 cm (2) 12 cm (3) 17 cm (4) 8.5 cm
S-54 Target NTSE

143. The chords PQ and RS of a circle are extended to meet at the


1 2 1 2 1 1
point Q. If PQ = 6 cm, OQ = 8 cm, OS = 7 cm, then RS = (1) cm (2) cm (3) cm 2 (4) cm 2
4 8 16 32
[West Bengal 2018]
(1) 12 cm (2) 9 cm (3) 10 cm (4) 16 cm 151. If x% of y is equal to 1% of z, y % of z is equal to 1% of x and
144. ABC is a right angled triangle and AD is perpendicular to z% of x is equal to 1% of y, then the value of xy + yz + zx is
the hypotenuse BC. If AC = 2 AB, then BC = [Delhi 2017]
(1) 1 (2) 2 (3) 3 (4) 4
B 152. The volume and whole surface area of a cylindrical solid of
radius 'r' units are v and s respectively. If the height of
v
cylinder is 1 unit then is equal to [Delhi 2017]
x s
D
1æ 1 ö 1æ 1 ö
(1) ç1 - ÷ (2) ç1 + ÷
2 è r +1 ø 2 è r +1 ø
A 2x C 1 æ 1ö 1 æ 1ö
(3) ç1 - ÷ (4) ç1 + ÷
2è r ø 2è r ø
[West Bengal 2018]
(1) 2 BD (2) BD (3) 5 BD (4) 4 BD 153. The sum of the reciprocals of the roots of the equation
145. (x + 2), x and (x – 1) are the frequencies of the numbers 12, 15
101 1
and 20 respectively. If the mean of the distribution is 14.5, x + + 1 = 0 is [Haryana 2017]
the value of x is [West Bengal 2018] 123 x

(1) 2 (2) 3 (3) 4 (4) 5 –101 123


(1) (2) (3) –1 (4) 1
146. If two angles of a triangle are 87° 24¢ 54² and 32° 31¢ 6², the 123 101
third angle is [West Bengal 2018] 154. If x3 + y = 2249, where x and y are natural numbers and
p p p p HCF of x and y is not 1, then the value of (x + y) is
(1) (2) (3) (4) [Haryana 2017]
6 2 3 4
(1) 26 (2) 39 (3) 52 (4) 65
147. If xsin3 a + ycos3 a = sin a cos a and x sin a – ycos a = 0,
the value of x2 + y2 is [West Bengal 2018] 155. If a = 6 - 11 and b = 6 + 11 then the value of
1 1 (a + b) is : [Haryana 2017]
(1) 0 (2) 1 (3) (4) (1) (2) 2 11 (3) (4)
2 3 22 6 12
2 2
156. If 2a –2ab + b + 6a + 9 = 0, then (a + b) + ab equals:
148. Four positive integers sum to 125. If the first of these numbers
is increased by 4, the second is decreased by 4, the third is [Haryana 2017]
multiplied by 4 and the fourth is divided by 4 we find four (1) 1 (2) 2 (3) 3 (4) 4
numbers, equal then four original integers are 157. The sum of all the terms common to the arithemetic
[Delhi 2017] progressions 1, 3, 5,..........., 1991 and 1, 6, 11, ........., 1991, is :
(1) 16, 24, 5, 80 (2) 8, 22, 38, 57 [Haryana 2017]
(3) 7, 19, 46, 53 (4) 12, 28, 40, 45 (1) 199100 (2) 199200 (3) 199300 (4) 200196
149. The total number of squares on a chessboard is [Delhi 2017] 158. The cube root of
(1) 206 (2) 205 (3) 204 (4) 202 x + y + 3x1/3 y1/3 (x1/3 + y1/3) is [Rajasthan 2017]
150. In the figure, the area of square ABCD is 4 cm2 and E is mid (1) x + y (2) x1/3 + y1/3 (3) (x + y)1/3 (4) (x + y)3
point of AB, F, G, H and K are the mid points of DE, CF, DG 159. Expressing 0.23 + 0.23 as a single decimal, we get
and CH respectively. The area of DKDC is [Rajasthan 2017]
[Delhi 2017]
(1) 0.465 (2) 0.465 (3) 0.465 (4) 0.4654
A E B
160. If ( x + 2 ) is a factor of kx 2 - 2x + 1, then the value of k
is [Rajasthan 2017]
F 3 2 3 2
(1) - (2) - (3) (4)
G 2 3 2 3
H
K 161. In the equations 3x + 2y = 13xy and 4x – 5y = 2xy, the values
D C of x and y that satisfy the equations are [Rajasthan 2017]
Stage - I Solved Questions S-55

æ1 1ö æ1 1ö 171. m 4 n 4 ´ 6 m 2 n 2 ´ 3 m 2 n 2 = (mn) k , then find the value of k.


(1) (2, 3) (2) (3, 2) (3) ç , ÷ (4) ç , ÷
è 2 3ø è3 2ø [Maharashtra 2017]
162. The angles of elevation of the top of a tower from two points (1) 6 (2) 3 (3) 2 (4) 1
at a distance of 9 m and 16 m from the base of the tower and 172. In a group of students, 10% students scored marks less
in the same straight line in the same direction with it are than 20, 20% students scored marks between 20 to 40, 35%
complementary. Then height of the tower is students scored marks between 40 to 60 and 20% students
[Rajasthan 2017] scored marks between 60 to 80. Remaining 30 students
(1) 12 m (2) 15 m (3) 20 m (4) 25 m scored marks between 80 to 100. Find the mode of marks.
163. If log5 [log2 (log3x)] = 0 then the value of x is [Maharashtra 2017]
[Uttar Pradesh 2017] (1) 30 (2) 50 (3) 60 (4) 70
(1) 3 (2) 6 (3) 9 (4) 0
164. Angle between the lines 6 + x = 0 and 3 – y = 0 will be (
173. One of the root of a quadratic equation is 3 – 2 , then )
[Uttar Pradesh 2017] which of the following is that equation.
(1) 0° (2) 90° (3) 180° (4) 60° [Maharashtra 2017]
165. If number 6, 8, 2x – 5, 2x – 1, 15, 17, 20 and 22 are in ascending (1) (x2 – 6x – 7) = 0 (2) (x2 + 6x – 7) = 0
order and its median is 14 then the value of x will be (3) (x2 + 6x + 7) = 0 (4) (x2 – 6x + 7) = 0
[Uttar Pradesh 2017]
174. In DABC, m ÐB = 90°, AB = 4 5. BD ^ AC, AD = 4, then
(1) 14 (2) 7 (3) 15 (4) 20
166. If u = {1, 2, 3, 4,5,6,7,8} A(DABC) = ? [Maharashtra 2017]
A = {3, 4, 5, 6} and B = {1, 3, 5, 7} then the value of (A¢ – B¢) (1) 96 sq. units (2) 80 sq. units
is [Uttar Pradesh 2017] (3) 120 sq. units (4) 160 sq. units
(1) {2, 8} (2) {3, 5} (3) {1, 7} (4) {1, 2, 4, 6} 175. Side of a cube is increased by 50%, then what percent
1 2 increase will be in the area of the vertical faces of the cube?
167. Factors of c - 2c - 9 are [Uttar Pradesh 2017] [Maharashtra 2017]
3
(1) 125% (2) 150% (3) 100% (4) 50%
æ1 ö æ1 ö 176. A 100 meter long train moving with constant speed of 90
(1) ç c + 3 ÷ ( c + 3) (2) ç c - 3 ÷ ( c - 3 ) km/h crosses a tunnel of 300 meter long. The time taken by
è3 ø è3 ø
the train to cross the tunnel completely is
æ1 ö æ 1ö [Karnataka 2017]
(3) ç c - 3 ÷ ( c + 3) (4) ç c - ÷ ( 3c + 1)
è3 ø è 3ø (1) 16 seconds (2) 8 seconds
(3) 4 seconds (4) 2 seconds
168. In the given figure, DABC has points D and F in AC and
177. The variance of 20 observation is 5. if each observation is
point E in BC such that DE || AB and EF || BD. If CF = 4 cm multiplied by 2, then new variance of the resulting
observations is [Karnataka 2017]
and AC = 9 cm, what is the length of DC ? [Odisha 2017] (1) 7 (2) 10 (2) 20 (4) 40
178. The polynomial (x2 –1) leave remainder as R1 and R2. The
A
true statement among the following is
D [Karnataka 2017]
F
(1) R1 + R2 = 0 (2) R1 + R2 = 0
(2) 2R1 + R2 = 0 (4) R1 + 2R2 = 0
179. In D ABC, XY || BC and xy divides the traingle into two parts
B E C
AX
(1) 7 cm (2) 6 cm (3) 5 cm (4) 4 cm of equal area. the value of is [Karnataka 2017]
BX
169. If a : b = 3 : 5 and a : c = 5 : 7, what (b – c) : (b + c) equal to?
(1) 4 : 46 (2) 46 : 4 [Odisha 2017] 2 2
(1) 2 –1 (2) (3) (4) 2 +1
(3) 5 : 48 (4) 48 : 5 2 2
1 1
170. If the mth term of an A.P. is and then nth term of its is , 1+ a4
n m 180. If a = 5 + 2 6 then [Karnataka 2017]
a2
then what is the mnth term equal to? [Odisha 2017]
(1) 4 6 (2) 10 (3) 5 (4) 2 6
m n
(1) 1 (2) 2 (3) (4)
n m
S-56 Target NTSE

185. Given that x, a1, a2, y are in A.P. and x, b1, b2, b3, y are also in
181. B
2 a 2 - a1
A.P, then the value of if (x ¹ y) [Telangana 2016]
D b2 - b3

1 3 4 -4
(1) (2) (3) (4)
2 4 3 3
C 4 A x
186. Sum of the roots of the equation 4 – 3(2 x + 3 ) + 128 = 0,
is [Maharashtra 2015]
In the given figure, if AD ^ BC, AC = 4, BD = 2, AB = a and
(1) 0 (2) 7 (3) 5 (4) 8
CD = b then a2 + b2 = [Karnataka 2017]
187. A shopkeeper blends two varieties of tea costing ` 18 and
(1) 6 (2) 8 (3) 12 (4) 20
` 13 per 100 gms in the ratio 7 : 3. He sells the blended
182. If a, b be the zeros of the polynomial 2x2 + 5x + k such that variety at the rate of ` 18.15 per 100 gm. His percentage gain
21 in the transaction is: [Maharashtra 2015]
a2 + b2 + ab = , then K = ? [Jharkhand 2016] (1) 8% (2) 12% (3) 15% (4) 10%
4
(1) 3 (2) –3 (3) –2 (4) 2 188. The arithmetic mean of two given positive numbers is 2. If
the larger is increased by 1, the geometrical mean of the
183. The number of solution of the equation x 2 = x - 2 is numbers becomes equal to the arithmetic mean of the given
(1) 0 (2) 1 [West Bengal 2016] numbers. Then, the harmonic mean of the given numbers is
(3) 2 (4) 4 [Karnataka 2015]
184. If 2m – 2m –1 – 4 = 0. Then value of mm is 2 1 3
[Delhi 2016] (1) (2) 2 (3) (4)
3 2 2
(1) 4 (2) 27 (3) 6 (4) 29
HINTS & SOLUTIONS
SECTION 1. PHYSICS 13. (3) The quality or timbre of sound enables us to distinguish
between two sounds having same pitch and loudness.
1. (3) On hills or higher altitudes - lower atmospheric pressure
and thus lower boiling point, so food needs to be energy mgh
14. (1) Power, P = =
cooked for a longer time. Hence cooking food is difficult time t
on hills. m 30
2. (3) When a plane mirror moves towards the stationary Given, = 30 Kg/min = Kg/s, h = 10 m and g is
object with speed u, the image will move with speed 2u t 60
9.8 m/s2
in same direction as that of mirror.
3. (2) From momentum, 30
\ P = ´ 9.8 ´ 10 = 49 J/s.
P = 2mk Þ P 2 = 2 mk 60
P 2 (3000) 2 5
R´ R
\ Kinetic energy, k = = = 90,000 J æR Rö
2m 2 ´ 50 15. (1) R AB = R || ç + ÷ = 6 =5 R
4. (2) Power of the lens to be used, è 3 2 ø R + 5 R 11
1 1 6
P= = = 2 D. R 4
f (in metre) 50 ´ R
100 R æ Rö 2 3 4
R AC = || ç R + ÷ = = R
5. (2) Power of the lens in spectacles is –2D i.e., –ve hence 2 è 3ø R 4 11
+ R
person suffers from myopia or short-sightedness i.e., 2 3
can see near object clearly but cannot see far off objects R 3
clearly. ´ R
R æ Rö 3 2 3
6. (2) Acceleration of the system, RBC = || ç R + ÷ = = R
m - m1 12 - 8 g 3 è 2ø R 3 11
a= 2 ´g = ´g = + R
m1 + m2 12 + 8 5 3 2
Hence equivalent resistance between A and B is
7. (3) In circular motion, due to change in direction -
acceleration and velocity change but speed being 5
maximum i.e., R AB = R .
scalar does not depend on the direction hence it 11
remains constant. 16. (2) F = Kx Þ ma = Kx
total distance
8. (3) Average speed = Kx 15 ´ 20 ´10-2
time taken Þa= = = 10 m/s 2
x m 0.3
= 17. (4) In part AB acceleration is decreasing as line bending
x x
v + v towards time axis.
2 1 2 2 18. (2) To decelerate and bring at rest force F < 100 N.
x x
= = W
x x 3x + 2 x 19. (2) From formula, DV =
+ q
2 ´ 20 2 ´ 30 120
x ´ 120 W 18 18
= = 24 m/s. Þq= = = = 0.2 C
5x DV 100 - 10 90
9. (4) Given : u = 20 m/s, a = 4 m/s2, t = 2s 20. (4) Magnetic field lines form closed loop. Electric field lines
By using first equation of motion, never form closed loop.
V = u + a = 20 + 4 × 2 = 20 + 8 = 28 m/s total distance travelled
21. (1) Average speed =
W 9.8 total time taken
10. (4) Weight, W = mg Þ m = = = 1 kg. x 5v1v2
g 9.8 = =
11. (3) Kinetic energy due to its speed and potential energy x / 5 4 x v2 + 4v1
+
due to its height or configuration. v1 5v2
12. (2) Relation between momentum P, mass m and kinetic distance
energy K. thickness
22. (2) Time taken =
Given : K1 = K 2 , m1 = 2 g, m2 = 4 g Vmed
P1 = 2m1 K1 P2 = 2m2 K 2 t é cù
=
c êQVmed = n ú
P1 2m1K1 2m1K1 2g 1 ë û
Þ = = = = n
P2 2m2 K 2 2m2 K 2 4g 2
nt
\ P2 : P1 = 2 :1 \ Time taken =
c
S-58 Target NTSE
v 34. (4) Alcohol is organic compound which does not ionise in
23. (3) Current, I = water. Therefore alcohol does not conduct electricity.
R
V 2.2 ´ 3 6.6 35. (4) When light ray travels from rarer to denser medium, it
I max = = = = 0.33 A bends towards the normal and vice-versa.
( Req ) 20 20 36. (3) Swayam is a 1-U Picosatellite placed in low earth orbit
V 2.2 2.2 (LEO) around the earth at a height of 515 Km.
I min = = = = 0.06 A 37. (4) Earth revolves round the Sun due to gravitational force.
Req + R0 6.66 + 30 36.66
Tension provides, required centripetal force due to
24. (1) Using v 2 - u 2 = 2 gh we get VA = VB = VC which stoned tied to a string whirled in a circle. Electron
around the nucleus revolves due to nuclear force and
v h1 due to electro-magnative force motion of blades of fan
25. (3) From magnification, m = = occurs.
u h0
2GM E
v -5 -12 ´ -5 38. (1) Escape velocity VE = = 11.2 Km / s
Þ = Þv= = +30 cm RE
-12 2 2
Given :
1 1 1 1 1 1 Mass of planet, M P = 25M E (ME = Mass of earth)
Now from, - = Þ - =
v u f 30 (-12) f Radius of planet, R P = 125R E (RE = Radius of earth)
1 1 1 2+5
Þ = + = 2G ´ 25M E 1
f 30 12 60 \ VP = = VE
60 125R E 5
\f = = +8.6 cm 39. (2) Velocity of light in sapphire,
7
thrust C 3 ´108
26. (2) Pressure = , placing a round piece of cloth Vsap = = @ 1.67 ´108
area n 1.8
increases the surface area, hence, decreasing the
pressure. QC = 3 ´ 108 velocity of light in vacuum of air
total distance travelled \ % decrease in velocity of light
27. (1) Average speed = 3 ´ 108 - 1.67 ´108
total time = ´ 100
260 + 300 560 3 ´ 108
= = = 14 m/s
20 + 20 40 1.33
= ´100 @ 45%.
28. (4) Because of gravitational force which is the force 3
between any two masses, the moon is binded to earth, 40. (4) Vector quantities have both magnitude as well as
we are binded to earth and tides are formed also direction and obey law of vectors addition.
because of gravitational pull of moon. 41. (4) An object is said to have uniform velocity, if magnitude
29. (2) From the figure, u = –20 cm, v = 20 cm, f = ? as well as direction of its velocity remains the same.
1 1 1 æ AC ö Force×displacement
Using lens formula, - = 42. (3) çè ÷=
v u f B ø momentum
1 1 1 1 1 1 Change in momentum displacement
Þ - = Þ + = = ´
20 (-20) f 20 20 f time momentum
displacement
1 1 +1 = = Velocity.
or, = time
f 20 43. (1) Mass is a quantity of matter contained in a body. It is
20 independent of acceleration due to gravity (g).
\f = = 10 cm Gm1m2
2 44. (3) From, F =
30. (4) As the image is formed behind the retina, so person is d2
suffering from hypermetropia or long sightedness. The Gm1m2
lens used to correct hypermetropia is convex lens. \F' = = 4F . [Q d = halved]
2
31. (3) Power of combination of lenses, ædö
çè ÷ø
Peq = P1 + P2 + P3 = +3.5 + 2.5 + 1 = +7 D 2
32. (2) Petrol is volatile liquid. So particles of petrol evaporate 45. (2) From v = f l
by using energy from palm and causes cooling effect. v 250m/s
Þ f = =
l 50M
100
33. (2) \ Frequency f = 500 Hz.
46. (2) From, F = ma
Atomic number of oxygen = 8 10
Q m = 10g = kg, a = 10 m/s2
Electronic configuration of oxygen = 2, 6. 1000
Hints & Solutions S-59

10 59. (1) Far point is infinity and near point 25 cm for normal
\ Force, F = ´ 10 = 0.1N human eyes. The far point of the eye is the maximum
1000 distance to which the eye can see the objects clearly.
1 1 1 60. (2) Refractive index of water hw = speed of light in air or
47. (3) - = vacuum/speed of light in water
v u f
or, hw = 3 × 108/2.25 × 108 = 4/3 = 1.33
v = image distance 61. (3) Power of lens P = 1/focal length (f) of lens in metres
u = object distance and Given, f = 20 cm = (20/100) metres = (1/5) metres
f = focal length of lens \ Power of lens = 1/(1/5) = +5D (+ve as convex lens)
1 1 62. (2) A voltmeter is a high resistance and is always connected
48. (2) Power of lens, P = = =2D in parallel in the circuit. It is used to measure potential
f (in metre) 50
difference between any two points of the circuit.
100 Ammeter is always connected in series in the circuit.
R 63. (3) 1 H.P. = 746 watt
49. (3) F= Þ R = 2F 64. (4) A magnet attracts magnetic materials like iron, cobalt,
2
50. (4) Angle of refraction for critical angle qC = 90°. nickel, etc.
65. (1) Mass m, linear momentum p and kinetic energy related
Rarer medium
as p = 2mE
66. (4) From equation,
ft
90° u = 0, t = 4s, a = g = 32
s2
1 2
qC using, s = ut + at
2
1 32 ´16
=0×4+ × 32 × (4)2 \ s = = 256 ft
2 2
67. (1) Volume, V = L 3
Denser medium
V2 V2 DV 3DL
51. (2) Power, P = ÞR= Þ =
R P V L
y
220 ´ 220 Þ y = 3x Þ x =
= = 4840W 3
10 68. (1) Using, refractive index,
energy mgh 100 ´ 10 ´ 5 1 1 1
52. (1) Power = = = = 500 Watt m= = =
time t 10
53. (3) a is temperature coefficient of resistance and its S.I. sin i, sin 45° 1/ 2
unit is /°C. \ Refractive index of the medium with respect to air.
69. (4) Unit of heat and work is same i.e., joule
F 70. (3) Gamma ray (g ray) is an electromagnetic wave.
54. (2) Pressure, P = Þ F = mg
A 71. (3) The minimum distance between a real object and its
Given, real image formed by convex lens is 4f i.e. four times of
F = 40 ´ 10 = 400 N and focal length.
V 2 (200)2 40000
A = 80 cm 2 = 80 ´10 -4 m 2 72. (1) Power consumed, P = = =
R 50 50
F 400 = 800w = 0.8 kW
\P = = = 5 ´ 104 Pa 73. (3) Distance travelled in first 10 seconds
A 80 ´10-4
55. (2) Given, emf v = 1.5 V, Charge, C = 0.5 C, energy provided 1
x1 = ut + at 2
by the cell, W = ? 2
Using, W = qv Þ W = 0.5 ´1.5 = 0.75 J 1
x1 = 0 + a(10)2
56. (1) Force in west direction, 2
FW = 4N + 3N = 7N is cancelled by force equal but x1 = 50a ...(1)
in opposite direction east, Fe = 7 N. Distance travelled in 20 seconds,
1
Force in south direction, FS = 4 N and in opposite x1 + x 2 = ut + a (20)2
direction. 2
Force FN = 5 N. 1
x1 + x 2 = 0 + a ´ 400
Hence, net force of 5 N – 4 N = 1 N in north direction. 2
57. (3) Magnetic lines of force start from North pole and end x1 + x2 = 200 a
at South pole.
x 2 = 200a - 50a
distance travelled
58. (3) Velocity or speed = x 2 = 150a ...(2)
time
S-60 Target NTSE
Distance travelled in 30 seconds gravitational force is quadrupled.
1 2 G m1 m2
x1+ x2 + x3 = 0 + a (30) F=
2
x1+ x2 + x3 = 450a r2
Now,
x3 = 450a – 50a – 150a r¢ = r/2
x3 = 250a ...(3)
From (1), (2) and (3) G m1 m2
F¢ =
x1 : x2 : x3 = 1 : 3 : 5 (r¢) 2
74. (1) Applying law of conservation of momentum,
Gm1 m2
0 = m × 16 + 2m × n F¢ =
( r 2)
2
2mn = – 16m
n = – 8 m/s
1 2 1 2
Total kinetic energy released = m (16) + m (8) F ' = 4F
2 2
=128 m + 64 m 80. (1) Wave velocity (n) = 350 m/s
=192 mJ Wave length (l) = 70 cm = .7m
n1 2 Wave frequency (f) = ?
75. (2) Given n12 = = n
n2 3 f =
Applying Snell’s law l
n1 sin i = n2 sin r wave velocity
n1 = wave length
sin x = sin 45º
n2
350
2 1 =
sin x = 0.7
3 2 = 500 Hz
3 1 1 1 1
sin x = 81. (3) = + +
2 Req 20 10 20
x = 60º 1 1+ 2 + 1
=
76. (4) The power and focal length of concave lens is Req 20
negative.
1 1 4
P= =
f Req 20
1 Req = 5 W
-4 = 82. (3) option (3) is correct because in doing so focal length of
f
f = – 0.25 m eye lens effectively decreases and we know this person
77. (4) New value of current I¢ = 1.5I requires a high power of the eye lens.
Power dissipated, P = I2 R ...(1) 83. (3) Metal dishes used for receiving TV signals from distant
New power dissipated P¢ = I¢2 R communication satellites are concave reflectors.
P¢ = (1.5 I)2 R 84. (3) When a convex lens is cut into two parts along principal
= 2.25 I2 R axis, focal length of each part does not change.
P¢ = 2.25P 1 1 1
Percentage increase in power dissipation 85. (4) = +
Req 6 6
æ P¢ - P ö 1.25P
=ç ÷ ´100 = ´100 6
è P ø P Req =
Percentage increase in Power dissipation = 125% 2
78. (2) The distance between two consecutive compressions =3W
or two consecutive rarefactions is called wavelength (l) 86. (4) 48×103J in 1 minute
As compression and rarefaction occurs alternatively, 48 ´103
therefore distance between a consecutive compression \ In 1s ®
60
and rarefaction is half of the wavelength (l/2). = 8×102 = 800J of energy will be released.
79. (3) m1 m2
r 800
Now, \ Number of fussion taking place =
3.2 ´10-11
m1 m2 = 250×1011
r/2 = 2.5×1013
As the distance between two masses is halved,
Hints & Solutions S-61

87. (3) Speed of sound depends upon density of medium; so


speed of sound in air is independent of pressure. 100. (1) T = 2h
g
88. (1) At 4ºC density of water is maximum due to anomalous
expansion of water. Below 4ºC its density will start Ta : Tb = a : b
decreasing and hence volume of water at 4ºC will be less 101. (1) u = V
than the volume of water at 0ºC. v = 3V
displacement 1 a=g
89. (4) Velocity = =
time tan 60º v2 – u2 9V 2 – V 2 4V 2
S= ; S = ÞS=
disp 1 2a 2g g
= = cot 60º = 102. (4) The air bubble has less density than water so it will be
time 3
F1 collected at the neck.
90. (1) Minimum number of forces
103. (3) V3 = V1 + V2 (as V3 is connected in parallel with V1 and V2)
required is 3. A 104. (4)
VA + 3 – 7 × 3 = VB
91. (1) Resistance of the heater be R. F2
VA – VB = 18 V
New resistance of heater is R/2.
V2 V2
V 2
V 2
V 2 105. (4) P1 = = 10 watt; P2 =
Initial power = Þ Final power = =2 3R R/3
R R/2 R
\ Heat generated is doubled. V2
P2 = 3 ´ = 90 watt
92. (3) Force acting at different parts of R
the magnet is different. So both
m0 i m0 (ni ) 2m i
torque and force will be acting. N S 106. (3) B = Þ B¢ = Þ B¢ = n 0 ÞB¢ = n2B
2R 2R / n 2R
93. (2) Power, P = 2 × 103 watt 1 1 1
Energy in 1 min = 2 × 103 × 60 J. 107. (3) + =
V u f
Þ 2 × 103 × 60 = m × 10 × 10
Þ m = 1200 kg ; Volume = 1200 litre. 1 1 1
94. (3) Initial velocity = V Þ + =
f + p f +q f
Final velocity = V¢
Þ f 2 = qp
1 1
mv 2 ´ 4 = m(v') 2 f f
2 2 108. (3) –m = – ;m=
u1 + f –u2 + f
V¢ = 2 V.
Initial momentum = mv –f f
Final momentum = 2mv. \ =
f – u1 f – u2
\ Momentum is doubled.
95. (3) Total work done by gravity is zero. u1 + u2
Þ f=
2 2
Mm æ Re ö
96. (2) gm ´ç ÷ ´ g e = 0.49 m/s
2
109. (3) P µ (V)3
Me è R m ø
\ If velocity is doubled power will become 8 times.
97. (2) P = P1 + P2 110. (4) (A) Slope of distance time graph is constant for uniform
speed
1 1 f +f
ÞP= + ÞP= 1 2 (D) Slope of velocity time graph is zero for uniform
f1 f 2 f1f 2 speed
sin i sin 30o V 111. (3) mb = 50 gm
98. (1) = = Þ V ' = 3V vb = 100 m/s
sin r sin 60o V '
mg = 10 kg
1 1
99. (2) g(t)2 - g(t - 2) 2 = 40 Þ 5t 2 - 5(t - 2)2 = 40 vg = ?
2 2
Þ t 2 - (t - 2) 2 = 8 -
50
´ 100
-m b v b 1000 1
1 2 1 Vg = = Þ Vg = m / s
\ Height = gt = ´10 ´ 9 = 45m. mg 10 2
2 2
S-62 Target NTSE

112. (4) v=0 v2 – u2 = 2g (100) 121. (4)


o– u2 = –2g (100) u
u2 = 2g (100) ... (i)
100 m and when initial velocity is doubled h
(2u)2 = 2gH ... (ii)
(ii) 2gH 4u2 3u
= = 2 Þ H = 400 m
u (i) 2g(100) u
(–3u)2 = (u)2 + 2(–g)(–h)
GM g M
113. (2) g = 2 Þ = Þ 9u2 = u2 + 2gh
R G R2
Þ 8u2 = 2gh
1
114. (2) f = é
T 4u 2 ù
Þ ê h = ú
so, unit of frequency is (second)–1. ëê g ûú
115. (1) v = lf 122. (2) Let v2 = 2v1
v = 10 × 1000 × 3 × 10–3 k1 1/ 2m1v12 v1
v = 30 m/s \ = =
p1 mv1 2
distance 3 1
t= = Þ t = = 0.1sec. k2 1/ 2mv22 2v1 æv ö æk ö
speed 30 10 Now, = = = 2ç 1 ÷ = 2 ç 1 ÷
p2 mv2 2 è2ø è p1 ø
3 ´108
123. (3) Q V = vl Þ l = = 0.3 m = 30 cm
116. (3) 109
124. (2)
C F P F = 16 N

8 cm
117. (4) f = +30 cm, u = –15 cm
1 1 1 1 1 1 m
= - Þ = -
8c

f v u v 30 15 8 cm
V = –30 cm F 16
v -30
P= = = 0.25 ´104 Pa
m= = =2 A 8 ´ 8 ´10–4
u -15 \ [P = 2500 Pa]
m= 2 125. (4) W = Fs cos q
118. (1) Resistivity do not depend on shape and size of material. when q = 0°, Þ cos 0° = 1(maximum)
119. (2) \ W = Fs is maximum
1W 2W F
1W 3W 2W \ s is correct
1W 2W 10W 10W 10W
126. (1)
1/3W 3W 2/3W 20W
A B A B
R eq=4W
A
20W 20W
1 2 RAB = ?
Req = + 3 + = 4W 1 1 1 1
3 3 = + +
R AB 30 20 40
60 km/h ® 40 km/h®
120. (3) 1 4 + 6 + 3 13 120
A B C = = Þ RAB = = 9.23 W
30 km 30 km R AB 120 120 13
30 30 1 3 5 rL
T = tAB + tAC = + = + = h 127. (2) R1 = ...(1)
60 40 2 4 4 4A
= 75 min r´ 2L
R2 = ...(2)
A
Hints & Solutions S-63

R1 rL A (i). 2n – 1 = 2 × 2 – 1 = 3
= ´ (ii). 2n – 1 = 2 × 3 – 1 = 5
R2 4 A r´ 2 L
ü
R1 1 ï
= = 1: 8 (iii). 2n – 1 = 2 × 5 – 1 = 9 ý Ratio = 3: 5 :9
R2 8
ï
128. (1) On increasing the length of the conductor by strecthing, þ
the new resistance is given by 141. (1) For an incident ray directed towards centre of
R¢ = (n)2R Where n is no. of times of stretching curvature of a spherical mirror the reflected ray retraces
R¢ = (3)2R [n = 3] its path.
R¢ = 9R 142. (4) By the rule of conservation of momentum
129. (1) Our solar system lies in“Milky way” Galaxy M1V1 = M2V2
130. (3) Resistance is obstruction to the flow of electric current. 3 × V1 = 6 × V2 ...(1)
131. (3) Radiation
1
Conduction and convention needs medium to travel, K= ´ 3 ´ V12 = 216
2
while radiation does not need medium to travel.
132. (4) 1000 cubic centimeter 216
V1 = ´ 2 = 12 m / s
1 litre = 10–3m3 3
\ 1 meter = 100 cm from eq. (1)
\ 1 litre = 10–3 (100 cm)3 = 10–3 × 106 cm3 3 × V1 = 6 × V2
1 litre = 103 cm3 3 × 12 = 6 × V2
133. (4) Irregular & non - periodic vibration V2 = 6 m/s
Music is produced by regular and periodic vibrations 143. (4) 144. (2) 145. (1)
Noise is produced by irregular & non - periodic vibrations. 146. (1) A real and enlarged image can be formed by using a
134. (3) convex mirror.
147. (4) There are 8 planets in the solar system. (Pluto is not
considered as a planet).
O rL
148. (4) Resistance (R) =
A
Length is stretched to double
Ray which passes through optical centre remains
undeviated. L¢ = 2L
A
5 C F – 32 Area A¢ =
135. (2) C = (F – 32) Q = 2
9 100 180
r ´ 2L
9 5 \ R¢ = A
Þ C = F – 32 Þ C = (F – 32)
5 9 2
136. (2) Due to refraction when a light ray passes from dense rL
medium is rarer medium it bends away from normal. R¢ = 4
A
137. (1) 4 Meter. Image formed by the plane mirror is at equal R¢ = 4R
distance as that of the object from plane mirror. R = 1W \ New Resistance, R¢ = 4W
138. (1) According to the law of floation weight of a floating
1
body is equal to the weight of displaced liquid. 149. (3) K.E1 = M1V 2 ...(1)
\ Ans (1) wt. of displaced liquid. 2
139. (1) Req = R1 + R2 1
= (2 + 2) W = 4 W K.E2 = M 2V 2 ...(2)
2
1 1 1 The velocity of both the body is same as they have
= +
R eq R1 R 2 equal momentun.
1 1 2 Dividing eq (2) by (1)
+ = =1W
2 2 2 K.E 2 M 2 2
So, Req= 4 W, 1 W = =
140. (2) no matching option. K.E1 M1 3
(i). n = 2, (ii) n = 3, (iii) n = 5 Thus, the ratio is M1 : M2 = 3 : 2
1 150. (2)
S = u + a (2n -1)
2
S-64 Target NTSE

SECTION 2. CHEMISTRY
10. (3) H 2SO 4 + Na 2 CO3 ¾¾
® Na 2SO 4 + CO 2 + H 2 O
100 ml Excess
1. (3) C + O2 ¾¾
® CO2 0.1 M
12 g 22.4 L at STP
Moles of H2SO4 = M × V
12 g of C is completely burnt in air to give 22.4 L CO2 at = 0.1 × 100 mL
STP. = 10 millimole = 0.01 moles
22.4
\ 4g of C will give ´ 4 = 7.46 L Produces
1 Mole of H2SO4 ¾¾¾¾ ® 1 mole of CO2
12
2. (1) Molar mass of CuSO4 = 159.609 g mol–1 0.01 moles of H2SO4 ¾¾¾¾ Produces
Mass of Cu present in 159.609 g (1 mol) CuSO4 = 63.5 g ® 0.01 moles of CO2
Mass of Cu present in 50 g CuSO4 1 mole of CO2 molecules occupies 22.4 L.
0.1 moles of CO2 occupy = 0.01 × 22.4 L = 0.224 L
63.5 11. (2) Molar mass of NH3 = 17 g
= ´ 50 = 19.89 » 19.90 g
159.609 Molar mass of HCl = 36.5 g
3. (3) 6.022 × 1023 number of nitrogen atoms = 14 g 1
3.022 × 1023 number of nitrogen atoms Rate of diffusion µ
Molar mass
14 ´ 3.022 ´ 1023 NH3 gas will diffuse at faster rate than HCl. So, white
= = 7 g.
6.022 ´ 1023 ammonium chloride ring will get formed near HCl bottle.
4. (2) Electronic configuration of potassium (K) NH 3 + HCl ¾¾
® NH 4 Cl
K[19] : 1s2 2s2 2p6 3s2 3p6 4s1
If we observe above electronic configuration, 19th Heat
12. (2) 2Pb(NO3 ) ¾¾¾
® 2PbO + 4NO 2 + O 2
electron is present in 4s orbital. Lead nitrate Nitrogen
\n = 4 dioxide
l = 0 (for s orbital)
m1 = 0 13. (3) Pb(NO3 ) 2 + 2KI ¾¾
® PbI 2 + KNO3
(Yellow
1 ppt)
ms = +
2 Conc. H SO
14. (1) CH3CH 2OH ¾¾¾¾¾¾
2 4 ® CH = CH + H O
2 2 2
5. (2) At anode O2 and at cathode H2 will be obtained. Ethyl alcohol
443 K
Ethene Water
Na 2SO 4 ® 2Na + + SO 42 - 15. (2) CuO is insoluble in water.
16. (1) Number of atoms = moles × NA
ˆˆ† H + + OH -
H 2O ‡ˆˆ Given mass
Moles of an atom =
At cathode : 2H + + 2e - ¾¾® H2 Molar mass
- 1
At anode : 2OH ¾¾ ® O 2 + H 2O + 2e - Atoms of He =
100g
´ N A = 25N A
2 4g
H2SO4 Acidified
6. (3) X Y (gas) Q (green solution) 100g
dichromate Atoms of Na = ´ N A = 4.347N A
solution 23g
Baryta
Ba(OH)2 100g
Atoms of Li = ´ N A = 14.49N A
(Turbidity)
6.9g
SO 2 gas changes dichromate (orange) to green 100 g
substance. Atoms of Al = ´ N A = 3.7N A
27 g
So, SO2 can be produced if substance containing
Hence, Helium will have largest number of atoms.
(SO32–) reacts with dil. H2SO4.
Alcoholic Electrolysis
7. (3) C2H5I C2H4 + KI + H2O 17. (3) NaCl(aq) ¾¾¾¾¾® NaOH(aq) + Cl 2 + H2
KOH Brine (Anode) (Cathode)
Ethyl Iodide solution
Alkaline
KMnO4 (purple) 18. (1) 4Al + 3O 2 ¾¾
® 2Al2 O3
Reactant Product
(Colourless)
substance Al 4×1 2×2
H S HCl O 3×2 2×3
8. (1) X ¾¾¾
2
® FeS ¾¾¾
® FeCl2 + H 2S
(Green (Black ppt.) x = 4, y = 3 and z = 2
solution
contains So, coefficient of Al2O3 = 2.
Fe2 + ) 19. (3) Sunlight
I2 CH 3 - Cl + Cl 2 ¾¾¾¾® CH 2 Cl 2 + HCl
Acidified
9. (4) CH3 – CH – CH3 CH3 – C – CH3 CHI3 It is a substitution reaction.
K2 Cr2O 7 Na2CO3
OH O
20. (1) Collenchyma is the living mechanical tissue as it
(X) Y (gas) provides flexibility and elasticity to the plant. The plant
Hints & Solutions S-65

bends without breakage due to collenchyma. The cells 30. (3) HCl gas is used for precipitation of pure common salt
are elongated and have differential thickening at the (NaCl) from saturated solution of common salt. When
corners of the cell wall due to the deposition pectin, HCl gas is passed, the concentration of Cl– increases
cellulose and hemicellulose. and value of ionic product of [Na+] and [Cl] becomes
Collenchymatous tissue provide mechanical support greater than Ksp of NaCl.
to the growing parts of the plant such as young stem Hence, pure NaCl is precipitated.
and petiole of a leaf. 31. (2) The correct increasing order of reactivity of elements
H is
| Au < Cu < H < K
21. (1) H - C - C º C - H 32. (2) X has 8 electrons in M shell when gains one electron so
| K L M
H =
Covalent bonds = Total no. of 6 bonds 2 8 8
+ Total no. of p bonds K L M
= 6 + 2 = 8. 2, 8, 7
[O] 3rd period, 17th group
22. (3) CH3OH ¾¾¾¾¾
® HCOOH So, valency = 1
Oxidation Formic acid
Methyl alcohol
Hence, (ii) and (iii) are incorrect.
23. (3) Germanium (Ge) is the element of 14th group and IVth 33. (1) Kossel and Lewis proposed the octet rule.
period. Antimony (Sb) is the element of 15th group and 34. (2) Compound (b) is ether and name of this compound is
Vth period. methoxy methane.
24. (4) Total no. of atoms in 1 molecule of Ca(HCO3)2 = 11 35. (2) DEN > 1.7 ® Bond will be Ionic
\ Total no. of atoms in 1 mole of Ca(HCO3)2
= 11 × 6.02 × 1023. DEN < 1.7 ® Bond will be covalent
25. (1) Carbon dating is a technique used to determine the DEN = 1.7 ® 50% Ionic and 50% covalent.
approximate age of once living materials. It is based on 36. (3) Conc. HNO3 being a strong oxidising agent deposits a
the decay rate of radioactive carbon isotope 14C. thin layer of iron oxide making it passive.
26. (3) 1 mol. of H2O (water) contains = 6.022 × 1023 molecules 37. (3) Rain water is slight acidic having pH range 5 – 5.5.
\ 0.25 moles of H2O will contain Q pH + pOH = 14
= 6.022 × 1023 × 0.25 = 1.5055 × 1023 molecules. pOH = 14 – 5.5 = 8.5.
27. (1) 18 g (1 mol) of water contains 6.022 × 1023 molecules. 38. (3) LPG is composed of hydrocarbons containing three or
9 g of water contain 3.011 × 1023 molecules. four carbon atoms. The normal components of LPG are
(1) Mass of 1 mol of Mg = 24 g propane (C3H8) and butane (C4H10)
39. (3) When chlorine is passed over dry slaked lime at room
24 g of Mg contains = 6.022 × 1023 molecules temperature, the main reaction product is CaOCl2
12 g of Mg contains = 3.011 × 1023 molecules (bleaching powder). The reaction is as follows :
(2) Mass of 1 mol of C = 12 g Ca(OH)2 + Cl2 ® CaOCl2 + H2O
12 g of Carbon contains = 6.022 × 1023 atoms 40. (1) Magnalium : Al + Mg
(3) Mass of 1 mol of NH3 = 17 g Bronze : Cu + Sn
17 g of NH3 contains = 6.022 × 1023 molecules Brass : Cu + Zn
(4) Mass of 1 mol. of CO2 = 44 g German silver : Cu + Ni + Zn
44 g of CO2 contains = 6.022 × 1023 molecules 41. (3) 1. Li, Na, K-Alkali metals (Group 1)
11 g of CO2 contains = 1.505 × 1023 molecules 2. Be, Mg, Ca-Alkaline earth metals (Group 2)
28. (1) Molecular formula of chloride = MCl2 3. N, O, F- (Period 2)
\ Valency of metal M = 2 4. He, Ne, Ar- Noble gases (Group 18)
42. (3) Soaps are sodium or potassium salt of fatty acids, made
Molecular formula of its oxide will be
by hydrolysis of fats and oils with bases. This process
M O yields soap as a product and glycerol as by-product.
O

Valency 2 2 CH2 – O – C

M2O2 = MO R1
Formula +
O CH2 – OH R1 – COONa
29. (1) Conjugate Acid ¾® H+ + Conjugate base
+
CH – O – C + 3 NaOH CH – OH + R2 – COONa
1. HCO3- ¾¾
® H + + CO 32 -
R2 CH2 – OH R3 – COONa
+
2. NH +4 ¾¾
® H + + NH 3 O
3. ® H + + OH -
H 2 O ¾¾ CH2 – O – C
R3
4. ® H + + NO3-
HNO3 ¾¾
Triglyceride + Sodium ¾¾® Glycerol + 3soap molecules
® H + + NO 2-
HNO 2 ¾¾ (fat or oil) hydroxide
S-66 Target NTSE
43. (2) The structure of proypyne is 56. (2) Electrons present in Cl– = 18
H Electronic configuration of Cl– = 2, 8, 8
H–CºC–C–H As it contains 8 electrons in outermost shell so, valency
electron in Cl– ion is 8
H 57. (1) Isoelectronic species are elements or ions that have the
There are 6 s bonds and 2p bonds. Hence ratio of s and
p bonds in propyne is 6 : 2 or 3 : 1 same or equal number of electrons.
44. (2) At a given temperature Na+, Mg2+, H3+ = 10e–
P1V1 = P2V2 Ca2+ = 18e–
P1 = P, P2 = ? 58. (4) K > Mg > Al > B (Reactivity series)
V1 = V, Q Volume is decreased by 5% Reactivity of metals decreases as we move from left to
\ V2 = 0.95 V right in a period.
Hence, P × V = P2 × 0.95 V
59. (1) Oxalie acid COOH is solid at room temperature.
P2 = 1.0526 P |
increase in pressure = 1.0526 P – P
COOH
= 0.0526
% increase = 5.26% Oxidation
60. (3)
Hence there will be 5.26% increase in pressure of the
gas.
O +1 +8/3 O
45. (4) Pb(NO3 ) 2 + H 2S ® PbS ¯ +2HNO 3
(Black) 3Fe(s) + 4H2O(g) ® Fe3O4(s) + 4H2(g)
ZnCl 2 + H 2S ® ZnS + 2HCl Reduction
(white)
46. (3) In both the structure, functional groups are same, only (Oxidising agent)
there positions are different. Hence, they are positional The increase in oxidation state of an atom, through a
isomers. chemical reaction, is known as an oxidation, a decrease
47. (2) Egg albumin in water forms a colloid. in oxidation state is known as a reduction.
48. (2) 4NH 3 + 5O 2 ¾¾ 61. (3) d - block elements are known as transition elements.
® 4NO + 6H 2 O
62. (1) Silver nitrate is used in voting ink.
2NO + O 2 ¾¾
® 2NO 2 63. (4) Balanced chemical equation:
4NO2 + O2 + 2H 2O ¾¾
® 4HNO3 NaOH (aq) + HCl (aq) ® NaCl (aq) + H2O(l)
15 ml of NaOH neutralizes 10 ml of HCl from reaction, it
49. (3) NH4Cl is a salt made by NH4OH and HCl. HCl being is clear that 1mol. of NaOH reacts with 1 mol of HCl
strong acid will give acidic salt and thus acidic solution. This means
50. (1) pH = – log [H3O+] nNaOH = nHCl
51. (4) Methyl orange changes its colour from orange to yellow Let the concentration and volume of NaOH and HCl be
in basic solution, as 8.3 to 10.0 pH is for basic solution, C1 V1 and C2 H2 respectively.
thus the colour changes to yellow. C1 V1 = C2 H2
52. (1) Aqueous solution of SO2 is acidic. C1×15 = C2×10
SO2 (g) + H2O(l) ® H2SO4 (aq)
C1 ´15
sulphur water sulfuric acid C2 =
­ 10
dioxide 3
C2 = C1
53. (3) Tritium = 1H3 2
Atomic mass = no. of neutrons + no. of protons 3
So C1×30 = C1 ´ V2
3=n+p 2
3 = n +1 V2 = 20 ml
n=2 Hence we require 20 ml of HCl of the same concentration
to neutralize the 30 ml of NaOH.
O
54. (2) H– N 64. (1) ZnO - Cr2 O3
.. CO + CH3OH
O 300 atm Methanol
HNO3 consists of co–ordiante bond between N & O 65. (3) C2H5OH + NaHCO3 ® No reaction
55. (1) Order of atomic radius Li > Be > B > C
CH3COOH + NaHCO3 ® CH3COONa + CO2 ( ­ ) + H2O
1 In case of ethanoic acid, brisk effervescence of carbon
Effective nuclear charge µ
atomic radius dioxide is evolved.
Hints & Solutions S-67

66. (1) s, p, d, f subshells are present in atom 95. (1) S ¾® K L M


67. (1) Boron and cadmium used in atomic reactors to 16 2 8 6
control speed of neutron. 96. (3) Fe ¾® Fe2+, Fe3+
68. (2) 69. (3) 97. (1)
70. (1) Cr = [Ar] 4s1 3d 5 98. (1) Freon – 112 is C2F2Cl4
Half filled configuration of d – subshell is more 99. (2) 14 g N2 contains 14/28 = 0.5 moles (if molecular
symmetrical weight of N2 is = 28)
71. (3) Amphoteric. Al2O3 can react with acid and base both Number of molecules in 0.5 moles of
72. (3) As solution is acidic so pH < 7 N2 = 0.502×6.022×1023 = 3.011×1023
73. (3) In basic solution phenolphthalein shows pink colour. 100. (1)
74. (1) 4s, 4p, 4d, 4f Mg + Cl2 ¾¾
® MgCl 2 (high m.p. / b.p .) ¾¾
® Mg(OH) 2
4s = one orbital, 4d = five orbital MgCl2 is ionic compound Water soluble
Total = 7 + 5 + 3 + 1 = 16 (ionic compound have high
melting and boiling point)
4p = three orbital 4f = seven orbital
So, total no. of electron = 32 O O
75. (1) CO2, CH4, N2O and O3 are act as green green house 101. (4) nHO – C – (CH2)4 – C – OH + nH2N – (CH2)6 – NH2
gases. Adipic acid Hexamethylene diamine

76. (2) Due to Ca(NO3)2 and CaSO4. As oxides of nitrogen & O O


sulphur reacts with limestones on Taj Mahal to form –– C – (CH2)4 – C – NH(CH2)6 – NH ––n + nH2O
nitrates and sulphates of calcium.
102. (2)
77. (4) Cheese is gel.
103. (3) Ethylene has the formula C2H4 or H2C = CH2.
78. (4) CaO + H2O ¾¾® Ca(OH)2
104. (1) All non-metal oxide on reacting with water gives acidic
Ca(OH)2 + 2NH4Cl ¾¾® CaCl2 + 2NH3­ + 2H2O solution.
79. (1) Cu + SO42– ¾¾® Cu2+ + SO2­ 105. (3) Anthracite has maximum percentage of carbon.
Choking smell 106. (1)
80. (1) CH3 COOH ¾¾¾¾ LiAlH 4
® CH3 CH2 OH 107. (1) Zn + 2HCl ® ZnCl2 + H2 ­
108. (2) Bromine liquid at room temperature.
81. (2) Sodium is present in first group of s block.
109. (1) Na reacts with cold water to give H2 gas
82. (4) Baking powder is a mixture of sodium hydrogen
carbonate and tartaric acid. 110. (3)
83. (1) CO2 is a gas and SiO2 is solid due to polymeric 111. (4) Flint glass
structure because it does not form Si = O. 112. (3) Controlled nuclear fission required in nuclear reactor.
84. (1) 46 g Na atom contains = 46/23 = 2 mole = 2NA atom 113. (4) CaC2 + H2O ® C2H2 + Ca(OH)2
8 g O2 = 8/32 = 0.25 mole = 0.5 NA atom 114. (2) Stainless steel contains Iron, Chromium and Carbon.
115. (4)
0.1 mole C= 0.1 NA atom
116. (1) Na2CO3 is formed from NaOH and H2CO3 i.e, strong
28 g N2 = 28/28 = 1.0 mole = 1.0 NA atom
base and weak acid.
0.1 mole carbon atom = 0.1 Na atom
117. (3) Ionisation energy decreases down the group and
28 g N2 molecules = NA molecules
85. (3) Valency of X = 2, So correct formula is MgX. increases across the period. Hence K
118. (4) K+ = 19 – 1 = 18
MnO 4- +Cl- ¾¾¾
H+
86. (3) ® Mn 2+ + Cl 2 ­ Cl– = 17 + 1 = 18
Ca2+ = 20 – 2 = 18
87. (4) Carbon has small size and maximum catenation property.
88. (3) Fe2O3 + 2Al ¾¾® Al2O3 + 2Fe (Molten) Hence only K
119. (3) Na2SO4 + BaCl2 ® BaSO4 + 2NaCl
89. (2) NH4Cl (sublime); NaCl (not sublime) 120. (2) CuSO4 . 5H2O ¾® CuSO4
- H+
90. (3) HCl ¾¾¾® Cl - Blue white anhydrous CuSO 4
Acid Conjugate base 121. (4) Ant sting contains methanoic acid also called formic
373K acid.
91. (3) CaSO4 . 2H2O ¾¾¾® CaSO4 . 1/2 H2O + 3/2H2O
122. (2) HCl. Rest all are ionic
Gypsum Plaster of paris
123. (1) Copper lies below H in reactivity series.
92. (2) 93. (4) 124. (2) CH3COOH and C2H5COOH a difference of –CH2
3 2 1 125. (2) CH3 – CH = CH2
94. (2) H3 C - C == CH2 126. (1) Stainless steel is a mixture of iron, nickel, chromium
|
CH3 and carbon.
2-methylprop-1-ene 127. (1) Non-metals form acidic oxides. Hence atomic no. 7
128. (2) CnH2n+1 is alkyl group formula.
S-68 Target NTSE

129. (4) CO2 and water (CO2 is reduced to C6H12O6 and H2O is Atom X and Y show same number of protons and
oxidized to oxygen). different number of neutrons. On the basis of this
130. (2) 114 (N2O remains in the atmosphere for 114 years) property, they will show different physical properties.
143. (1) Atomic number = 20
131. (2) The German silver, an alloy, shows composition,
Cu = 60%, Ni = 20% Zn = 20%. K L M N
132. (4) Gold number is defined as the minimum amount of 20 = 2, 8, 8, 2
protective colloid in milligram which prevents the 144. (1) When the rice straw is burnt then carbon dioxide,
change of colour. methane and sulphur dioxide gases are produced which
are green-house gases.
133. (3) Hydrogen : Oxygen Þ 1 : 4 = WH2 :WO2 145. (2) A – (iv); B – (i); C – (ii); D – (iii)
nH 2 WH2 ´ M O2 146. (1) In the solution of hydrogen fluoride and acetic acid,
1 ´ 32 4
nO2 = M H2 ´ WO2 = 4 ´ 2 = 1 Þ 4 : 1 an immersed glass exhibits apparent disappearance.
147. (2) Increasing order of pH value
134. (4) Coal is called fossil fuel because they have been formed Tooth paste ® blood ® saliva (after meal) ® coffee
from fossilized remains of prehistoric organisms as long ® tomato Juice
as millions and millions years ago. 148. (3)
135. (1) Functional group of butanal is aldehyde showing Oxidation
structure – CHO. e.g.
H H H (–2) (–1) (+6) (–2)
O
H–C C C C PbS + 4 H2O2 ¾¾¾® PbSO4 + 4 H2O
H
H H H
Butanal Reduction
149. (3) As the temperature of water increase, the water
136. (2) NH 4 Cl dissociation constant (Kw) is increases. Hence pH of
the water decreases but water remains neutral.
H 150. (2) Oxides of metals are basic and that of non-metals are

NH 4+ Cl N+ Cl

acidic.
H H \ Na2O < CaO < ZnO < CO2
H
Strongly Basic Amphoteric Acidic
NH4Cl shows ionic bond between NH4+ and Cl– ions
Basic
and three covalent bonds are formed between nitrogen
151. (2) 152. (1) 153.(1)
and three hydrogen atoms and one coordinate bond is
154. (1) In homologous series, the members are same in
formed between nitrogen and one hydrogen atom.
chemical properties.
hot, conc. 155. (4) Fe2O3 + 2Al ® Al2O3 + 2Fe
137. (3) CH 3 - CH 2 - OH ¾¾¾¾¾
H SO
® CH 2 = CH 2 + H 2 O
2 4 (Ethene) This reaction is an example of displacement reaction.
156. (3) Antacids like Mg(OH)2 Milk of magnesia are used for
138. (1) 11 Na – 2, 8, 1 treating indigestion.
Na – 2, 8, 8, 1
19
157. (1) CFC react with ozone (O3) to form dioxygen molecule
Na, and K have same electronic configuration of and thus deplete the ozone layer.
valance shell. hv
O3 ¾ ¾® O2 + O
139. (4) Ultra violet radiation is absorbed by CO2 molecules in nascent
oxygen
the atmosphere.
140. (2) Chemical formula – 158. (3) D
CH 4 + O 2 ¾¾® CO 2 + H 2 O
CuSO4.5H2O (Blue vitriol)
After combustion, methane forms carbon dioxide and
NaHCO3 (Baking soda)
water.
Na2CO3.10H2O (Washing soda)
159. (3) Rusting takes place in ordinary water as well as distilled
CaSO4.2H2O (Gypsum)
water. It occurs in the presence of water and oxygen.
Thus baking soda's chemical formula does not show
160. (2) Mg + 2H2O ® Mg (OH)2 + H2 + Heat
the presence of water (H2O)
When Mg get dissolve in hot water, it forms Mg (OH) 2
141. (2) C2H4O2 (CH3COOH) is ethanoic acid.
magnesium hydroxide.
CH3COOH + NaHCO3 ® CH3COONa + H2O + CO2­
161. (2) The cations which have more electronegativity, have
X Y more covalent character.
RbCl < LiCl < MgCl2 < BeCl2
Proton 8 8
142. (3) 162. (2) 10 g of H2 contains 5 moles of Hydrogen gas.
Neutron 8 10 64 g of O2 contains 2 moles of Oxygen gas.
Hints & Solutions S-69

2H2 + O2 ® 2H2O 5. (3) When the blood flows through a wound, the platelets
If 1 mole of oxygen gives 2 moles of water then, 2 mole release an enzyme called Thrombokinase to stop the
of O2 give 4 moles of water. blood flow. This enzyme facilitates the conversion of
prothrombin to thrombin results is coagulation of
163. (1) Mass % of Nitrogen in N2O3 is blood.
28 6. (4) Tuberculosis is caused by bacterium. Mycobacterium
= ´ 100 = 36.84% tuberculosis that affects the lungs.
76
7. (4) While auxin, florigen and cytokinin are plant hormones,
164. (4) Butane and Isobutane have same chemical formula but Oxytocin is an animal hormone and act as neuro-
different arrangement of atoms and have different transmitter that is involved in child birth and breast
structure. feeding.
H H H H H H H 8. (2) Carbohydrates and fats are high energy rich food. The
glucose from carbohydrates is converted into energy
H–C–C–C–C–H , H–C–C–C–H which is required by brain and muscles.
H H H H H H Fat is required by the body for development of brain,
H–C–H controlling inflammation and blood clotting.
(Butane)
9. (3) Electricity is produced by humans, so it is not a natural
H (Isobutane) resource.
165. (1) H – C º C – H 10. (3) Pure breed plant A × Pure breed plant B
Triple bond is shorter in length than double bond and F1 Generation : Only B type plant
single bond. Means A is recessive trait, which re-emerges in F2
166. (4) Aldehydes and ketones are functional isomers of each generation.
other belonging to family of carbonyl compounds with TT × tt
Parents (Pure Tall) (Pure Dwarf)
general molecular fomula CnH2nO.
167. (2) It is due to greater screening effect in aluminium, its Gametes T T t t
radius is greater than gallium.
168. (3) Sample contains 3.01 × 1023 MgCO3 molecules. Cross breeding
Therefore, 6.022 × 1023 molecules weigh 84 gms F1 Generation Tt
Hybrid Tall
84
Þ 3.01 × 1023 molecules weigh g Gametes T t
Self polination
2
= 42 g
T t
= 0.042 kg TT Tt Pure Tall 1
169. (3) 170. (1)
F2 Generation T Pure Tall Hybrid Tall
171. (2) Graphite is used as a lubricant due to layer structure. Tt tt Hybrid Tall 2
t Hybrid Tall Pure Dwarf Pure Dwarf 1
172. (3) H3PO2 is a monobasic acid due to it has only one
replaceable hydrogen. F2 generation ratios : –1 : 2 : 1
So, A is dwarf pea plant while B is tall pea plant.
O
11. (1) Good ozone is found in the upper part of the atmosphere
called stratosphere and it acts as a shield absorbing
P ultraviolet radiation from the Sun.
12. (4)
H H OH 13. (4) Opening of guard cells is facilitated by the entry of
water inside guard cells. This makes the guard cell
SECTION 3. BIOLOGY turgid. Whereas closing of guard cells is facilitated by
1. (1) Thyroid Stimulating Hormone (TSH), is produced in water coming out of guard cells. This will make the
the anterior pituitary gland that promotes bone growth. guard cells flaccid.
Ovaries produce and release egg in the female 14. (2)
reproductive tract at the mid (14th day) of every 15. (1) Diaphragms, cerevical caps and vaults are barrier
menstrual cycle. Adrenal gland causes secure high method of contraception that are made of rubber and
blood pressure results in increase heart rate. Testis is are inserted into the female reproductive tract to cover
responsible for development of male sex organs. the cervix during coitus.
2. (1) Abscisic acid helps in ripening of fruits and its 16. (1) Sperms are produced in the seminiferous tubules. As,
development. seminiferous tubules is lined on inside by two types of
3. (1) Pyrethroid is an organic compound which are produced cells called male germ cells and sertoli cells. The male
by the flowers of pyrethrums (chrysanthemum germ cell undergo meiotic divisions finally leading to
cinerariaefolium and chrysanthemum coccineum). sperm formation, while sertoli cells provide nutrition to
4. (2) Plasma membrane of a cell is made up of protein and the germ cell.
lipids. It acts as a semipermeable membrane that allows 17. (4) The villi and microvilli present in the small intestine
the exchange of materials between cells and its increase the surface area for the absorption of digested
surrounding. food.
S-70 Target NTSE
18. (1) During excessive and rigorous exercise, the muscle 35. (3) Statements (a) and (b) are correct.
cells shifts from aerobic respiration to lactic acid Statements (c) and (d) are false.
fermentation due to limited supply of oxygen. In lactic As dominant factor is expressed only in F1 generation
acid fermentation, the pyruvate produced during of a monohybrid cross.
glycolysis is converted into lactic acid with the help of Gregor Mendel conducted his experiment on pea plant.
the enzyme lactate dehydrogenase. 36. (1) The RBC are devoid of nucleus, mitochondria ribosome
19. (3) The adrenal gland produces the hormone adrenalin and and endoplasmic reticulum. The absence of these
similarly pancreas produces the hormone insulin. organelles accommodates more haemoglobin thereby
20. (2) Amoeba undergoes binary fission, yeast undergoes maximising the oxygen carrying capacity of the cell.
budding, Leishmania undergoes longitudinal binary Biconcave shape increase the surface area for oxygen
fusion and Bryophyllum undergoes vegetative
binding, loss of mitochondria allow the RBC to
propagation by leaves.
21. (1) Octopus is most clever animal among all non-chardates transport all the oxygen to tissues and loss of
and which can change its colour. Due to the presence endoplasmic reticulum allows more flexibility for RBC
of chromatophare cell, they are able to change its skin to move through the narrow capillaries.
colour. 37. (1) Mitochondria are membrane-bond cell organelles found
22. (4) Rifamycin is used for the treatment of tuberculosis as in the cytoplasm of all eukaryotic cell that generate the
it is effective antibiotics against tuberculosis. cells primary energy molecule ATP Mitochondria are
23. (4) Glucose and fructose syrup can be obtained from known as the power house of the cell because it is
carnflour by action of enzymes obtained from Bacillus responsible for extracting energy from food through
and Streptomyces. cellular respiration.
24. (3) Consumption of tobacco product lead to cancer of 38. (3) Cnidocytes, also known as stinging cells, are specialized
lungs by damaging the cells that lines the lungs. neural cells that typify the phylum cnidaria (sea anemones,
25. (2) Pneumonia is a communicable disease which is caused eorals, hydroids, and jellyfish) which often contains
by bacteria called streptococcus pneumoniae and poison, is to ward off enemies or to capture prey.
Mycoplasma pneumoniae. 39. (1) Liver is the largest gland of body which produces bile
26. (2) Chitineous cell wall is found in fungi. Fungi are juice which is stored into the gallbladder for
eukaryotes that lack chlorophyll and are heterotropic. concentration, storage, or transport into the first region
The cell wall of fungi is rigid as it is composed of of the small intestine, the duodenum. Bile juice is
nitrogen - containing polysaccharide called chitin. It is responsible for emulsification of fat.
also composed of branched and filamentous hyphae, 40. (3) Glycolysis is the first stage of aerobic and anaerobic
which form a net like structure known as mycelium. respiration where one molecule of glucose is converted
27. (1) Lack of oxygen occurs when algal bloom is formed in a
into two molecules of pyruvate in the cytoplasm of cell.
water body. Algal bloom is formed because of the
presence of large amounts of nutrients in water results 41. (1) Auxin is a plant hormone produced in the stem tip that
in excessive growth of planktonic or free floating algae. promotes cell elongation. Auxine promote stem
It causes deterioration of the water quality and fish elongation inhibit growth of lateral buds & maintains
mortality. apical dominance.
28. (2) Lichens are symbiotic association between algae and 42. (2) Cholera is an acute diarrheal infection caused by
fungi. The algal component is known as phycobiont ingestion of food or water contaminated with the
and fungal component as mycobiont, that are bacterium Vibrio cholerae.
autotrophic and heterotrophic. Lichens are very good 43. (4) Blind spot of eye lacks receptor cells (rod & cones)
pollution indicators as they do not grow in polluted because the optic nerves leave the eye ball from here,
areas. so no image is formed in this part of the eye.
29. (2) Earthworm is not a member of phylum Arthropod as it 44. (4) Tetanus is acquired through infection of a cut or wound
belongs phylum Annelida. with the spores of bacterium Clostridiumtetani. This
30. (4) DNA synthesise takes place in S-phase or synthesise disease affects the nervous system results in painful
phase. In this phase of cell cycle, DNA replication muscle contraction.
begins in the nucleus and the centriole duplicates in 45. (2) Lysosome stores large number of hydrolytic enzymes,
the cytoplasm. which are also known as acid hydrolases they are also
31. (3) Fats are broken down by lipases with the help of bile known as suicide bags of the cell because they have
into di- and monoglycerides. capability of digesting cells and unwanted materials.
32. (1) Julius Van Sachs was a German Botanist, for the first 46. (3) Cardiac muscles are striated as they have light & dark
time plants could be grown to maturity in a defined bands. These muscles are involuntary in nature and
nutrient solution in the complete absence of soil. This
are located in the wall of the heart.
technique of growing plants in a nutrient solution is
known as Hydroponics. 47. (2) Oesophagus is the part of human alimentary canal where
33. (3) Stele is the central part of the stem inner to endodermis. digestive glands are absent, only mucus glands are
It consist pericycle, vascular bundle, pith. Cambium is present.
a tissue layer that provides partially undifferentiated 48. (1) lgG is the only antibody that significantly crosses the
cells for plant growth and is found in the area between human placenta.
xylem and phloem. 49. (4) Leishmania is a protozoan and it has flagella.
34. (2) Skeletal muscle tissue is attached to bones and its 50. (1) Since the wings of housefly and wings of sparrow have
contraction results in the locomotion, facial expressions, same function but dissimilar structure, they would be
posture, and other voluntary movements of the body. analogus organs.
Hints & Solutions S-71

51. (4) Cytokinin is a growth promoter hormone which promotes 81. (2) As Lichens are SO2 sensititive and can’t grow in the
division of cell in plants. area where sulphur dioxide pollution is present.
52. (1) Adrenaline is an emergency hormone which is secreted 82. (3) Cycas & Pinus are gymnosperms which are perennial,
from adrenal gland during a fight or flight. evergreen & woody.
53. (1) ATP is formed by addition of phosphate to ADP in the 83. (1) Vacuole maintains the osmotic pressure in plants &
presence of light during photosynthesis. This ATP maintain turgidity.
formation by a photosynthesizing plant cell would be 84. (3) Sunlight, chlorophyll, CO2 are the main elements es-
known as photophosphorylation. sential for photosynthesis.
54. (2) Pollen tube carries two male gamete inside ovule. 85. (3) The nature of nerve impulse is both electrical & chemi-
55. (3) Tuberculosis is a bacterial disease caused by cal, so eletrochemical.
Mycobacterium tuberculosis 86. (4)
56. (3) Ovary is a primary reproductive organ. 87. (4) Monohybrid Cross
57. (2) Cardiac (heart) muscles are cylindrical and inter Parents - Tall ´ dwarf
connected, smooth muscles are spindle in shape, whereas
skeletal muscles are cylindrical with no branching. TT ´ tt
58. (2) Potassium hydroxide absorbes CO 2. Therefore,
Gametes - T T t t
photosynthesis will not occur due to absence of CO2.
59. (4) The recessive character in pea plant is green seed. Rest
of them are dominant character. °
60. (4) Haemophilia is more common in males because it is a ° T T
recessive trait carried by X chromosome. t Tt Tt
61. (1) Transcription is the first step of gene expression, in which t Tt Tt
a particular segment of DNA is copied into RNA
(especially mRNA) by the enzyme RNA polymerase. F1 generation - Tt (tall)
62. (1) Outermost whorls is calyx, followed by corolla,
androecium and gynoecium is at centre. selfing
63. (1) Carbohydrates are broken by glycolysis into pyruvic
acid which converts into Acetyl COA which then enters Tt ´ Tt
krebs cycle to produce CO2 and H2O and energy. Gametes - T t T t
64. (4) When haemoglobin combines with carbon monoxide it
forms carboxy haemoglobin and when haemoglobin
° T t
combines with oxygen it forms oxyhaemoglobin. °
65. (2) Chromosomes carry genes, which are the hereditary T TT Tt
units, which carry hereditary characters to the off- t Tt tt
spring.
66. (3) MItochondria is called the power house of the cell. F2 generation-Phenotypic ratio - 3 : 1
67. (4) Plasma membrane is made up of both protein and lipid. tall : dwarf
68. (2) Oviduct is the site of fertilization in humans. Genotypic ratio - 1 : 2 : 1
69. (1) Heart never takes rest as it has cardiac muscles which TT : Tt : tt
never gets fatigue. 88. (1) Cartilage is skeletal connective tissue which forms en-
70. (1) Lacteal present in the villi of the small intestine help to doskeleton of human body.
absorb fatty acids and glycerol. 89. (4) Echidna is a mammal & lay eggs & act as a connecting
71. (1) The experiment ‘origin of primitive life on Earth’ was link between reptiles & mammals.
performed by Urey and Miller. 90. (1) 91. (3)
72. (2) Bicuspid valve is present in the human heart in be- 92. (4) CO2 and water (CO2 is reduced to C6H12O6 and H2O is
tween left atrium and left ventricle. oxidized to oxygen)
73. (3) During the light reaction NADPH and ATP are synthe- 93. (4) Testis is male reproductive organ while others are fe-
sized which are utillzed in dark reaction. male.
74. (4) Grafting in monocot plants is not possible because 94. (1) 50 decibel (in day time, 40 decibel at night in silent
they have scattered Vascular Bundles. zone)
75. (1) Haemophilia disease is linked with sex chromosome. 95. (2) 114 (N2O remains in the atmosphere for 114 years)
76. (3) The primary building blocks of DNA Nitrogenous base, 96. (3) Two (Deoxygenated blood enters the heart and then
phosphorus and deoxyribose. lungs, oxygenated blood enters the heart and pumped
77. (1) Islets of Langerhans helps in formation of insulin to other parts of the body.
78. (3) Sclerenchyma is simple permanent tissue in which 97. (4) Seismonastic movement (Nastic movement,
lignin is present which makes it dead & it provides nondirectional movement towards direction of touch)
mechanical strength as in the fibrous covering of 98. (2) Regeneration is not truly a reproductive process, rather
coconut. it’s a process of renewal, restoration and growth in
79. (3) Nucleus is the headquarter of the cell which control organisms.
cell and discovered by Robert Brown. 99. (2) 23 pairs of chromosomes (22 pairs of autosome and
80. (3) Cytokinin is a plant hormone which mainly control cell one pair of allosome)
division & promote growth in plants.
S-72 Target NTSE
100. (1) Darwin explained natural selection, Lamarck explained 116. (3) One of the two male gametes fuses with two polar nu-
inheritance of acquired characters and Mendel is clei located in the central cell to produce a triploid pri-
known for pioneering work in inheritance. mary endosperm nucleus (PEN). PEN becomes primary
101. (4) 1, 2 and 3 belongs to Thallophyta and 4 belongs to endosperm cell (PEC) & develops into endosperm.
Bryophyta 117. (4)
102. (2) Raphide crystals are sharp needle like crystals of cal- 118. (4) The noosphere is the sphere of human thought. The
cium oxalate that dart and cause discomfort to throat, word derives from the Greek words (nous "mind") and
activates inflammatory reaction by (sphaira "sphere"), in lexical analogy to "atmosphere"
production of histamines. and "biosphere"
103. (3) Area A is for perception of touch, pain etc. 119. (3) The excretory organs of earthworm are minute, coiled
Area B is for perception of sound tube like structures called as nephridia.
Area C is occipital lobe of brain for visual perception 120. (2) Plasmid” is made up only of one type of macro
Area D is for thinking, intelligence etc. molecule.
104. (2) Estrogen is secreted by ovary 121. (2) The relative energy yield in Kcal/gm is best represented
105. (4) 1, 2 and 3 are pteridophytes and 4 is bryophyte in first ATP, followed Lipid and protein
106. (4) Psammophytes is a plant that grows in shifting sands, 122. (4) The sub units of ribosomes in the cells of nephron of
primarily in deserts. They are marked by a number of mouse is 60s & 40s
adaptations which enable them to exist on wind-blown 123. (3) Involuntary muscles are not found in “tongue”
sands for e.g. Haloxylon persicum, Calligonum, 124. (1) Only “denitrifying bacteria” work strictly under anaero-
Ammondendron, Eremosparton, Smirnowi. bic conditions.
107. (2) In the human skin, melanogenesis is initiated by 125. (1) The correct sequence of the given figure is (iii), (ii),
exposure to UV radiation, causing the skin to darken. (iv), (i)
Melanin is an effective absorbent of light due to which 126. (2) Mylein sheath is not present in grey neurons. (non
it is thought to protect skin cells from UVB radiation myleinated neuron)
damage i.e. reducing the risk of cancer. 127. (3) The incorrect statement is they convert water and CO2
108. (2) Binomial nomenclature is a system of providing a name into carbohydrate only in the absence of light)”
with two components. This naming system given by 128. (1) The correct pathway of blood in circulatory system is
Carolus Linnaeus is being practised by biologists all Atria ® Ventricles ® Artery ® Vein
over the world. The first name represents the genus 129. (4) “Iodine” is essential for formation of thryoxine n thy-
while second name represents the species name. For roid gland.
e.g. the scientific name of mango is written as Mangifera 130. (2) According to Lindemauns’ 10% low of energy trans-
indica. fer, only 10% of the energy is transferred from one
109. (3) Ozone is located in upper part of the atmosphere called tropic level to heat.
stratosphere and it acts as shield absorbing UV radia- 131. (3) Both similarities and variation are transmitted from par-
tion from sun. Ozone-depleting substances: CFCs, ents to offspring during reproduction.
HCFCs, halons, etc result in depletion of ozone & for- 132. (3) During excess of physical exercise, lactic acid gets ac-
mation of a large area of thinned ozone layer, com- cumulated in the muscles which causes pain.
monly called as ozone hole.
133. (2) A good food chain includes Grass, Goat, Lion
110. (2) The process of copying genetic information from one 134. (1) The Oxygen rich blood from lungs comes to the heart
strand of the DNA into RNA is termed as transcription. in left atrium, the left chamber of the heart.
It takes place in nucleus with the help of RNA poly-
135. (4) Growth of pollen tube in the style towards the ovule in
merase.
plants is an example of chemotropism which is a
111. (2) movement of plant part in response to a chemical stimulus.
112. (2) Insectivorous plants are plants that derive some or 136. (4) Urethra in human male, is a common passage of urine
most of their nutrients from trapping and consuming and sperm. The urethra is held closed by the urethral
animals or protozoans, typically insects and other sphincter, admuscular structure that helps keep urine
arthropods. They have adapted to grow in places where
in the bladder.
the soil is thin or poor in nutrients, especially nitrogen.
137. (4) DNA is found in both mitochondria and nucleus.
113. (2) Spleen is a secondary lymphoid organ where prolifera-
138. (3) The plants of Bryophyta are called amphibious plants
tion of lymphocytes takes place.
because their habitats are concerned with aquatic and
114. (4) Tobacco Mosaic Virus (TMV) is single stranded RNA terrestrial region.
virus.
139. (2) Protein is digested with the help of enzyme pepsin.
115. (2) RNA was the first genetic material. There is now enough This process occurs in stomach. It happens in acidic
evidence to suggest that essential life processes (such medium.
as metabolism, translation, splicing, etc.) evolved
140. (4) Calciferol (Vitamin D) is helpful in the development of
around RNA. RNA used to act as a genetic material as
bone.
well as a catalyst. But, RNA being a catalyst was reac-
tive and hence unstable. Therefore, DNA has evolved 141. (1) Bile does not contain any digestive enzyme. It has
from RNA with chemical modifications that make it more bile pigments - bilirubin and biliverdin. It makes the
stable. medium alkaline through NaHCO3.
Hints & Solutions S-73

142. (1) Spongy parenchyma is responsible for storage of SECTION 4. HISTORY


starch in plant leaves. The spongy parenchyma has
many spaces between cells to facilitate the exchange 1. (4) The period that started in 1789 and ended in November
1799 with the formation of the French Consulate is referred
of gases. to as the French Revolution. During this period, Louis XVI,
143. (3) Sclerenchyma contains stone cells having hardness. It who belonged to the ‘Bourbon’ dynasty, was the king of
protects the plant from outside. France.He the last ruler of France before the monarchy came
144. (4) Homo sapiens is the scientific name of man. This is a to an end during the French Revolution.
modern man. 2. (2) Hitler was born in Austria. He was born on 20 April
145. (4) Ammonia (NH3), urea, uric acid are harmful products 1889 in Braunau am Inn. He was an Austrian-born German
of bio-chemical reactions but lymph is not produced politician who dictated Germany from 1933 to 1945. He
biochemically. became the leader of the Nazi Party. During his tenure, he
invaded Poland on September 1, 1939 ad, therefore, initiated
146. (1) Lungfish is an organism which is a connecting link World War II in Europe.
between pisces and amphibian. 3. (2) The book, The Folklore of Southern India was
147. (2) The ratio of F2 generation = 9 : 3 : 3 : 1 published by Natesa Sastri. The book is a collection of
9 Tamil folktales and was published in four volumes. He
\ ´ 320 = 180 (yellow & round seeds) considered folklore as national literature. According to him,
16
real thought and characteristics could be shown and
148. (4) Earthworm (Lumbricus terrestris) is a beneficial projected by folktales only. Natesa was a noted supporter
organism for farmers. It belongs to phylum-Annelida. of Harikatha.
149. (2) 4. (3) The secret society or a political society ‘young Italy’
150. (3) Enzymes are proteins having linear chain of amino was formed by Giuseppe Mazzini in the year 1831. The
acids. movement aimed to unite people as a republican Italian
151. (2) Pepsin converts proteins into proteoses & peptones nation. It was the movement that was founded for the
(peptides). Rennin helps in the digestion of milk independence of Italy. He had a belief this movement could
proteins in infants. Later, pancreatic juices (trypsin lead to the unification of people and would contribute to a
chymotrypsin & carboxypeptidase) digest proteins, European-wide revolutionary movement.
peptones & proteoses into dipeptides. 5. (4) Gandhi Irwin Pact was an agreement that was signed
152. (1) The cerebellum located at the back of the head by Mahatma Gandhi and Lord Irwin in 1931. According to
coordinates voluntary muscle movements and helps the pact, Gandhi agreed to participate in the Second Round
in maintaining posture, balance, and equilibrium. Table Conference. He also promised to withdraw the civil
153. (1) Mushroom exhibits saprophytic mode of nutrition disobedience movement. The agreement also included the
while others (ticks, tape worms, Cuscuta exhibit withdrawal of all prisoners with the expectation of who
parasitic mode of nutrition). were involved in severe violent crimes.
154. (3) 6. (4) Gandhiji had designed the Swaraj flag by the year of
155. (1) Ginger is a root, which is used as spice. 1921. The flag was a tricolour,i.e., red, green and white and
156. (2) Food is stored in the form of glycogen in liver. had a spinning wheel in the centre. The wheel represented
157. (2) Olfactory lobes have speciallized sensory cells of the the idea of Gandhian of self-help. At the time of struggle,
nasal cavity in vertebrates. freedom fighters used to hold the flag, representing a
158. (1) Phototropic and geotropic movements of a plant symbol of opposition.
depend upon different concentrations of auxin. 7. (3) Michelangelo made the statue of David. It was a
masterpiece made in the period between 1501 and 1504. An
159. (4) Pulmonary Vein carries oxygenated blood from the
Italian artist, Leonardo da Vinci made the best known, the
lungs to the left auricle of the heart. most visited and the most painting, and the most parodied
160. (3) In animal cells like muscles during exercise, when work of art in the world the Mona Lisa. It is a half-length
oxygen is inadequate for cellular respiration pyruvic portrait of the Italian Renaissance. Leonardo da Vinci made
acid does not enter the Kreb?s cycle and gets reduced the famous painting, the last supper. It depicts the last meal
to lactic acid by lactate dehydrogenase in cytoplasm. Jesus having with his apostles. Albrecht Durer made the
161. (1) Cutin, a waxy layer prevents transpiration from aerial painting, Praying Hands. It is a famous ink and pencil sketch
parts of xerophytes. drawing that was created in the early 16th century.
162. (4) The increase in concentration of non-biodegradable 8. (3) Machiavelli wrote the book, The Prince in 1513.
substance with subsequent trophic level of a food Montesquieu wrote the book, The spirit of Laws in 1748.
chain is called bio-magnification. Thomas More wrote the book, Utopia in 1516. Erasmus
163. (1) Phototropic and geotropic movements of a plant wrote the book, Praise of Folly in 1509 but was printed in
depend upon different concentrations of auxin. 1511. This book was a satirical attack on several traditions
164. (1) and superstitions.
165. (1) Changes in blood volume, body fluid volume and ionic 9. (3) There was an attack on the Bastille prison on July 14,
concentration activate osmoreceptors which stimulate 1789, in Paris. On the day, the violent attack took place on
the hypothalamus to release antidiuretic hormone the government by French residents as the onset of the
(ADH) or vasopressin from the pituitary gland. They French Revolution. People attacked the prison in search of
regulate water re-absorption to maintain blood volume arms and ammunitions as they had a belief that they had
etc. stored them at the fort.
S-74 Target NTSE
10. (2) Gandhi Ji, the father of the nation, said the statement, 23. (1) Although nominally allied with the German Empire and
If I am to die by the bullet of a mad man, I must do so the Empire of Austria-Hungary in the Triple Alliance, the
smiling on 28 January 1948.He said so due to the disturbance Kingdom of Italy did not join the Central Powers, Germany
created by a section of people in his secular prayer meetings and Austria–Hungary had taken the offensive while the
a couple of times. Triple Alliance was supposed to be a defensive alliance.
11. (1) In 1439, the first printing press was made by Johannes 24. (2) Anandamath is a Bengali novel, written by Bankim
Gutenberg and his associates. The Bible was the first book Chandra Chatterjee and published in 1882. Set in the
that was printed in the press in Europe. During the Eastern background of the Sannyasi Rebellion in the late 18th
Han period (25–220 CE), China developed paper and century, it is considered one of the most important novels
printing with blocks. Paper and ink were two earlier Chinese in the history of Bengali and Indian literature.
inventions thatledto blocking printing. 25. (1) Dhapi Dadi from village Kudan (Sikar, Rajasthan) was
12. (1) Oceans cover 71 per cent of the earth’s surface. There a brave widow, a leading Freedom Fighter who took part in
are five oceans in the world. These five oceans are in the Shekhawati farmers movement in Rajasthan. She was
descending order by area, the Pacific, Atlantic, Indian, of Fandan Gotra from village Raseedpura.
Southern (Antarctic), and Arctic Oceans. Therefore, the 26. (2) Riyasat was the name of the newspaper. The main
Pacific Ocean is the largest ocean and the smallest ocean is objective of General Dyer behind the Jallianwala Bagh
the Arctic Ocean in the world. massacre was to ‘produce a moral effect’, to create in the
13. (2) Government (1690) by Locke and The Social Contract minds of satyagrahis a feeling of terror and awe.
(176 (2) by Jean-Jacques Rousseau (1712–78) proposed 27. (1) Flying shuttle, Machine that represented an important
justifications of political association grounded in the newer step toward automatic weaving. It was invented by John
political requirements of the age. Kay in 1733. In previous looms, the shuttle was thrown, or
14. (1) The symbols of New Britain – the British flag (Union passed, through the threads by hand, and wide fabrics
Jack), the national anthem (God save our Noble King) and required two weavers seated side by side passing the
the English language were actively promoted and the older shuttle between them.
nations survived only as subordinate partners in this union. 28. (2) In 1589, the French throne was taken by the Bourbon
15. (1) Andhra Pradesh emerged as the first linguistic state on dynasty. It took place after the killing of the last Valois
Nov. 1, 1956. In Oct. 1953, the government of India was forced king, the childless Henry III. After the fall of Napoleon, the
to create the first linguist state known as Andhra Pradesh by dynasty returned to the throne In 1814. However, the last
separating the Telugu speaking areas from Madras state. Bourbon king was absconded after the Revolution of 1830
16. (2) He was one of the leaders of the Young Bengal group in and was replaced by a cousin from the Orleans dynasty.
19th-century India. As an orator, editor of several periodicals, 29. (4) The King of Sardinia named Charles Albert was on the
and a social reformer, he donated land for the Bethune School throne for the period from April 27 1831 to March 23, 1849.
17. (4) Alexander II became known as Tsar the Liberator able The first Italian constitution, the Albertine Statute, and the
to implement the most challenging reforms undertaken in First Italian War of Independence are associated with him.
Russia since the reign of Peter the Great. He stayed in France during the Napoleonic period. There
18. (3) Black Thursday is the name given to Thursday, October he received a liberal education.
24, 1929, when panicked investors sent the Dow Jones 30. (1) A Russian author, Leo Tolstoy wrote a book, War and
Industrial Average plunging 11% at the open in very heavy Peace in 1869.This book focuses on the invasion of Russia
volume. Black Thursday began the Wall Street crash of 1929, in 1812. It projects the Russian society against the invasion.
which lasted until October 29, 1929. It has spirituality and happiness as the ultimate reward for
19. (2) France was named “a museum of economic errors” by spiritual sufferings.
Adam Smith. He is considered as the “Father of Economics” 31. (2) King Suryavarman II was the successor of Cholas.
and known for his magnum opus, The Wealth of Nations. Hebuilt the Angkorwat temple in the 12th century. The
20. (3) On 4th February 1922, a group of protestors temple has a huge complex. The complex consists of a
participating in the Non-Cooperation Movement clashed thousand buildings. At the onset of the 12th century, it
with the police who opened fire. The angry protestors in was converted into a Buddhist temple.
turn set the police station on fire, killing all its occupants. 32. (2) The “Vernacular Press Act was passed by Lord Lytton
This took place in Chauri Chaura, in Gorakhpur district of in 1878. He was the then viceroy of India (governed 1876–
erstwhile United Provinces. On seeing the movement turn 80). In British India, the act aimed to restrict the freedom of
violent, Mahatma Gandhi put a halt to the Non-Cooperation the Indian press and prevent the expression of criticism
Movement in the subcontinent. toward British policies. This criticism was the outcome of
21. (1) It was started on 17 December 2010. It was a series of the beginning of the Second Anglo-Afghan War (1878–80).
anti-government protests, uprisings, and armed rebellions 33. (3) The Jallianwala Bagh Massacre took place on April
that spread across much of the Arab world in the early 13, 1919. The moment Rabindranath Tagore got to know
2010s. It began in response to oppressive regimes and a about the Jallianwala Bagh Massacre. He returned the title,
low standard of living, starting with protests in Tunisia. ‘the sir’ (Knight hood) to the British government as a sign
22. (2) The Korean War began on June 25, 1950. Suez Crisis, of protest. In this event, there were 379 people died at the
(1956), an international crisis in the Middle East, precipitated hands of the British army.
on July 26, 1956, when the Egyptian president, Gamal Abdel 34. (1) Dwarakanath Vidyabhusan started the newspaper, Som
Nasser, nationalized the Suez Canal. The Congo Crisis Prakash. It was a weekly newspaper which was started on
(French: Crise congolaise) was a period of political upheaval 15 November 1858. However, the original idea or plan for
and conflict in the Republic of the Congo (today the the newspaper was given by Ishwar Chandra Vidyasagar.
Democratic Republic of the Congo) between 1960 and 1965. The aim was to provide the opportunity of employment for
From 1988 to 1990, the Somali Armed Forces began engaging a deaf scholar.
in combat against various armed rebel groups.
Hints & Solutions S-75

35. (4) Sanchi is a Buddhist complex, famous for its Great 48. (3) Vietnam National Party was formed in the year of 1939.
Stupa, on a hilltop at Sanchi Town in Raisen District of the The Fascist Party was formed in 1921 by Benito Mussolini.
State of Madhya Pradesh, India. It is located 46 kilometres The union of South Africa came into existence in 1910. It
northeast of Bhopal, the capital of Madhya Pradesh. included the territories that were originally a part of the
36. (2) After Ashoka’s successful but devastating conquest South African Republic and the Orange Free State. French
of Kalinga early in his rule, he converted to Buddhism and communist party formed in the year 1920.
was inspired by its doctrine of dharma. Thereafter, he ruled 49. (3) Hundred Days of Reform took place in 1898. It was an
his empire through peace and tolerance and focused on attempt to renovate the social and other conditions of China.
public works and building up the empire rather than Russian Revolution began in 1917 and ended in 1923. In
expanding it. this revolt, peasants And the workingclass started the
37. (3) Tamil was the oldest language of South India. Of the protest against the government of Tsar Nicholas II. First
four literary languages in the Dravidian family, Tamil is the Opium War was started in 1839. It was fought between
oldest, with examples dating to the early Common Era. In China and Britain. The Chinese Revolution took place in
the early 21st century, Tamil was spoken by more than 66 1911. It leads to the end of the Chinese dynasty.
million people, mostly residing in India, northern Sri Lanka, 50. (2) The meaning of this phrase is that if one does not
Malaysia, Singapore, Mauritius, Fiji, and Myanmar (Burma). grant liberty, equality, or fraternity to others—one does not
38. (2) In the early 14th century, the Delhi Sultanate treat others like they would treat their brother—one will
ruler Alauddin Khalji (r. 1296-1316) instituted price controls meet death. The three ideals, namely freedom, equality and
and related reforms in his empire. brotherhood were the foundation of the ‘new France’ that
39. (2) It was established in 1336 by the brothers Harihara I the revolutionaries sought.
and Bukka Raya I of the Sangama dynasty, members of a 51. (4) Kachch is not recognised by UNESCO. There are 11
pastoralist cowherd community that claimed Yadava biosphere reserves of India that are now considered as
lineage. The empire rose to prominence as a culmination of UNESCO Protected Biosphere Reserve.
attempts by the southern powers to ward off Islamic 52. (1) Michel Foucault was the French philosopher who
invasions by the end of the 13th century. wrote the book Archaeology of Knowledge in which he
40. (2) The royal city at Fatehpur Sikri, situated 26 miles west argued that the prevailing practice of arranging historical
of Agra, Uttar Pradesh, was built under the orders of the events in chronological order is not right.
great Mughal Emperor Akbar. In honour of Saint Shaikh 53. (2) Ishwardas Nagar, Bhimsen Saxena, Khafi Khan and
Salim Chisti, Akbar founded a magnificent city on Sikri Niccolo Manucci were among the historians of Aurangzeb’s
ridge. times whose accounts are important sources of Mughal
41. (3) Vasco da Gama was best known for being the first to history. Many British officials in India wrote about Indian
sail from Europe to India by rounding Africa’s Cape of Good history. Their writings display the strong influence of the
Hope. Throughout two voyages, beginning in 1497 and colonial policies of the British.
1502, da Gama landed and traded in locales along the coast 54. (3) This style consists of coloured paintings. - It occurs
of southern Africa before reaching India on May 20, 1498. as murals and also miniatures used in manuscripts. - The
42. (4) Retired British Indian Civil Service (ICS) officer Allan murals are seen in old wadas, temples, ceilings etc. - Maratha
Octavian Hume founded the Indian National Congress (A style was influenced by the Rajput and European painting.
political party of India (British India to Free India)) to form 55. (2) Kesari began to publish articles about the nationwide
a platform for civil and political dialogue among educated situations, books in the native languages and the politics
Indians. in England
43. (4) The Story of My Experiments with Truth is 56. (3) Shejwalkar primarily wrote in the Marathi language
the autobiography of Mohandas K. Gandhi, covering and was the founder-editor of the now-defunct Marathi
his life from early childhood through to 1921. periodical Pragati (1929–1932).
44. (2) The Aravalli Range, an eroded stub of ancient mountains, 57. (4) Baji Prabhu Deshpande (c. 1615-1660) was a
is the oldest range of fold mountains in India. The natural commander of Maratha king Chatrapati Shivaji Maharaj,
history of the Aravalli Range dates back to times when the the founder of the Maratha empire. The legend of Baji
Indian Plate was separated from the Eurasian Plate by an ocean. Prabhu is linked with an important rear guard battle enabling
45. (1) The first Peshwa of Shahu was Balaji Vishwanath. He Chatrapati Shivaji Maharaj’s escape from Panhala fort; he
was appointed as Peshwa by Sahu on November 16, 1713. was the hero who sacrificed his life for his king.
He helped a young Maratha emperorto get his kingdom 58. (3) Games such as Chess, Snakes and Ladders, Playing
back in a better state as it was damaged due to regular Cards, Polo, the martial arts of Judo and Karate had originated
attacks by the Mughals under the rule of Aurangzeb. in India and it was from here that these games were transmitted
46. (4) Before French Revolution, the political body of France to foreign countries, where they were further developed.
was called Estates General. In this period, France was 59. (2) King Lear is a tragedy written by William Shakespeare.
divided into three parts or states, namely First Estate that There are two versions, but modern editors usually conflate
was related to clergy, the Second Estate was related to these to produce a single play. Both versions are based on
nobility and the Third Estate was related to commoners. the mythological Leir of Britain. King Lear relinquishes his
The king was above or beyond all these three estates. power and land to two of his daughters.
47. (2) From 1756 to 1763, the Seven Years War was a global 60. (1) Menelik II was crowned King of Kings and Emperor
conflict that took place. It pitted a coalition of Great Britain of Ethiopia on November 3, 1889, with the additional royal
and its allies against a coalition of France and its allies. The sobriquet of “the Conquering Lion of the Tribe of Judah.”
war escalated from a regional conflict between Great Britain The coronation, which took place in the great Entotto
and France in North America, known today as the French Mariam Church in Addis Ababa, was captured for posterity
and Indian War. by the Italian artist Pio Joris (1843-1921).
S-76 Target NTSE
61. (4) Genghis Khan was one of the most feared and admired 73. (3) Nationalist Vietnames fought against Japan and France.
leaders of his time. He created a Mongol military might that 74. (3) Belgium and Germany became new colonial powers who
was superior to any other he faced. His campaigns were joined European powers in the process of carving Africa.
brilliant, and showcased his military genius and established
US also became colonial power in the late 1890s.
standards for manoeuvre warfare, brutality, and human
endurance never before seen. 75. (1) In England and Europe, there was large scale industrial
62. (3) production for an international market. This was not based
63. (2) The Ryotwari System was first implemented in the on factories. Many historians refer to this phase of
Baramahal region by Alexander Reed in 1792. This system industrialisation as proto industrialisation.
was later implemented in Madras and Mysore region by 76. (3) Mahe was the French colony until 1954 when it was
Thomas Monroe in 1801. In the Ryotwari system, there were integrated into the Union of India.
no middlemen to collect the revenue. The East India
Company and the farmers were directly linked. 77. (1) The declaration of Poorna Swaraj in December, 1929 under
64. (2) Mahajanapadas were sixteen kingdoms that existed in the Presidency of Jawaharlal Nehru, the Lahore congress
northern ancient India from the sixth to fourth centuries formalised the demand for full independence for India.
BCE. The capital of Anga was located on the bank of this Dandi Satyagrahs was an act of non-violent Civil
river and was also named Champa. Varanasi was the capital
of Kashi. Saketha was the capital of Kosala. Kushinagara Disobedience in colonial India led by Gandhji to produce
was the capital of Malla. salt From sea water. The Quit–India movement started
65. (4) The third Carnatic war took place in 1756. The second in August 1942. Mahatma Gandhi decided to initiate a
Anglo-Mysore war took place in 1780. The second Anglo- new phase of movement against the British in the middle
Marata war took place in 1803. The first Anglos Sikh war
of the second World War. The moplah uprising against
took place in 1845.
66. (2) Allauddin Hasan Gangu was the founder of the Bahamani British spread like Wildfire and took the Character of a
Kingdom. The Iranian painters were brought by Humayun were mass rebellion.
Mir Sayyid Ali and Abdus Samad. Both the painters were the 78. (4) Some sepoys of the regiment at Meerut refused to do
founders of an independent branch of Persian art popularly
known as the Mughal school of miniature painting. the army drill using new catridges. The sepoys of Meerut
67. (3) An Indian social reformer, Shishunal Shariffa believed rode all night of 10 May to reach Delhi. Soldiers persisted
in the philosophies given in different religions. He took and proclaimed Bahadur Shah Zafar as their leader.
Deeksha from Govindabhatta. He wrote several Tatvapadas Thereafter, the queen of Britain passed a declaraction
which means moral poems. The objective of the poems was assuring stable govt. for Indians. Tantia Tope was killed
to bring social transformation. in 1859.
68. (4) Sripurusha was the king of the Western Ganga dynasty.
He is the author of the book “Gajasastra” which was written 79. (1) Liberal nationalism developed in Europe in 19th C. Liberal
in Sanskrit. nationalism means individual freedom, equality before
This book is considered western Ganga literature and also law, government by consent, freedom of markets and
a very sacred text. abolition of state imposed restrictions on the movement
Gadya Chintamani was written byVadiba Simha.. Shabdavathara of goods and services.
was written by Durvinita.Raghava Pandaviya was written by 80. (3) In December, 1929 under the presidency of Jawaharlal
Hemasena.This book tells the stories of Pandavas and Rama. Nehru, the Lahore congress formalised the demand for
69. (4) Akshay Ramanlal Desai was a renowned Indian Purna Swaraj. Dr. B. R. Ambedkar organised the Dalits
sociologist. He was influenced by Leon Trotsky and therefore into the depressed classed Association in 1930 clashed
was a Marxist and a social activist. He worked as the UGC at Mahatma Gandhi at the second Round Table
national fellow and as the Head of the Department of Sociology
conference. Ambedkar accepted Gandhiji’s position and
at Bombay University. One of his best works was “The
Sociological Background of Indian Nationalism’’. His other major the result was Poona Pact, 1932.
works include Indian Rural Sociology, Slums and Urbanisation. 81. (c) It was organised to support the peasants of kheda
70. (1) In Africa, a fast-spreading disease of cattle plague or district. Due to crop failure and plague epidemic, the
rinderpest had a terrifying impact on people’s livelihood presents could not pay the revenue.
and the local economy. It arrived in Africa in 1880s. 82. (2)
71. (3) The Gudem rebellion spread in response to the non- 83. (2) Anguriya Binimoy was the first historical novel written
cooperation movement in 1921. In the Gudem hills of in 1857.
84. (2) On 5th May, 1789, Louis XVI called together an assembly
Andhra Pradesh, the militant guerrilla movement spread of Estate General to pass proposal for new taxes.
under the leadership of Alluri Sitaram Raju. It was against Bastille, was destroyed on 14th July 1789. A group of
forest laws. several hundred people marched towards the eastern
72. (3) Architect and planner Ebenezer Howard developed the part of the city and stormed the fortres prison, the
Bastille. On the night of 4 August, 1789, the Assembly
principle of the Garden City. The Garden city was a
passed a decree abolishing the feudal system. On 20
pleasant space full of plants and trees where people lived June 1789, the members of the Third Estate, who began
and worked. to call themselves, the National Assembly took the
Tennis court oath.
Hints & Solutions S-77

85. (1) On 6 April he reached Dandi and Ceremonially Violated 102. (3) Ebenezer Howard, architect and planner developed
the salt law manufactured salt by boiling water. the principle of the Garden City, a pleasant space full
Mahatma Gandhi was accompanied by his 78 trusted of plants and trees where people would both live and
volunteers. The march was other 240 miles. work.
86. (3) The battle of Plassey (1757) laid foundation for British 103. (1) It was in 1517, the religious reformer Martin Luther
rule in India. wrote Ninety Five Thesis.
87. (2) The Great Economic Depression began around 1929
104. (1) Kashibaba, a Kanpur Mill Worker in 1938 to show the
and lasted till the mid 1930s. During this period most
links between caste and class exploitation wrote the
parts of the world experience is great decline in pro-
book chhote Aur Bade Ka Sawal.
duction, employment, incomes and trade.
105. (1) The Ryotwan Settlement was introduced in the Madras
88. (4) Anandmath, a Bengali fiction written by Bankim
Presidency. It was introduced by Thomas Munno.
Chandra Chattopadhyay and publised in 1882.
106. (4) On August 8, 1942, the Quit India Resolution was
89. (1) Lenin’s Imperalism the Highest stage of capitalism
passed at the Bombay (Mumbai) Session of All India
describes the function of financial capital in generating
Congress Committee.
profits from imperalists colonialism as the final stage of
capitalist development to ensure profit. 107. (4) Duma is a Russian Assembly with advisory or
90. (3) The Wall Street Crash began on October 24th 1929, it legislative functions. It was introduced into the Russian
marked the beginning of widespread and long-lasting Empire by Tsar Nicholas II in 1906.
consequences for the united states. 108. (1) Arthastra is written by kautilya who was chief advisor
91. (4) Tsar Nicholas II ruled Russia and its empire. of the Indian Emperor Chandra Gupta Maurya (350-275
92. (4) John Kay was the inventor of flying shuttle. Flying BC). He was a stateman and philosopher.
shuttle was the important development in the 109. (3) Chittaranjan Das was a founder member of the Swaraj
industrialisation of weaving during early industrial Party in Bengal in British India. He was a famous
revolution. politician called as Deshbandhu.
93. (3) Sardar Vallabhbhai Patel is known as the Iron man of 110. (3) Canning was the Governor – General of India during
India. He was the first Deputy Prime Minister of India. the Indian Rebellion of 1857.
94. (2) The communal Award 1932 was declared by the Ramsay
111. (3) Sarojini Naidu, popularly known as 'Nightingale of
MacDonald granting separate electorate in India.
India' was the first Indian woman to become congress
95. (3) Pralilata Waddedar, was Bengali revolutionary
President. She presided the Kanpur session in 1925.
nationalist attacked the Pahartati European club which
112. (4) Khan Abdul Ghaffar Khan was called as "Frontier
had a sign board that real ‘Dogs and Indians not
Gandhi".
allowed’.
96. (2) AITUC, was founded in 31st October 1920 in Bombay 113. (1) 114. (4) 115. (4) 116. (3)
by Lala Lajpat Rai, Joseph Baptista, N.M. Joshi, Chaman 117. (1) Kheda Movement was a no-tax peasant struggle,
Lal and others. launched in 1917 under the leadership of Gandhiji,
97. (2) Hindustan Socialist Republican Association was a Sardar Vallabhbhai Patel, Indulal Yagnik and several
revolutionary organisation also known as Hindustan others.
Socialist Republican Army was established in 1924 at
Feroz Shah Kotla, Delhi. 118. (4) Bharat Mata is an epic painting by Abanindranath
98. (2) In the 19th C the Kannada-speaking regions of Belari Tagore.
came under Madras Province. 119. (4) German inventor Johannes Gutenberg invented Print-
99. (4) Shaikh Abdullah, known as Sher-e-Kashmir played a ing press in 1430s.
central role in the politics of Jammu and Kashmir. He 120. (3) Dr. B.R. Ambedkar founded Depressed class
founded National Conference. Fazal Ali was an Indian Association in 1930.
judge the governor of two Indian states. He was the 121. (2) Kulaks were the rich peasant.
head of the states Reorganisation Commission which 122. (3) Reichstag is a German word which means parliament.
determined the boundaries of several Indian states in 123. (4) Civil Disobedience Movement was one of the most
the 1950. Sardar Vallabhbhai Patel the first deputy prime important events launched during the reign of Lord
minister and also India’s first Home minister. He was Irwin, the then viceroy of India.
instremental in uniting the coutry through merger of
small princely states. Qasim Razvi organised a private SECTION 5. POLITICAL SCIENCE
militia the Razakars Raja Harsingh was the last ruling 1. (1) The United Nation Secretary General Kofi Annan declared
Maharaja of the princely state of Jammu and Kashmir
explicitly for the first time that the US led war on Iraq was
in India.
100. (3) French East India Company was commercial enterprise not legal. The war against Iraq was not authorised by
founded in 1664. It officially state owned company. the UN Security Council.
101. (4) The Elementary Education Act 1870, commonly known 2. (3) The President can declare emergency on the basis of the
as Forster’s Education Act set the framework for written request by the cabinet headed by the Prime
schooling of all children between the ages of 5 and 12 Minister.
in England.
S-78 Target NTSE

3. (2) Amnesty International is an international human rights 25. (1) Both the statements are true power sharing is the very
organisation. It is a London-based non-governmental spirit of democracy. It also helps reduce the possibility
organisation. The organisation claims to have more than of conflict between social groups. It ensures stability
of political order.
seven million members and supporters around the world. 26. (2) 27. (2)
4. (3) Option three is odd pair of persondities the others are 28. (3) Concurrent list includes subject of common interest to
similiar, i.e., 1, 2 and 4. both the union government and the state government
5. (3) The speaker of the Lok Sabha presides the joint session such as marriage adoption etc.
of Parliament. 29. (1) Cochabamba
6. (2) The autobiography written by Nelson Mandela is the 30. (4) Religious Justice does not exist in preamble of Indian
constitution. It provides to its citizens social,
Long Walk to Freedom. It was published in 1994. economical and political justice.
7. (1) The Union government legislates on residuary subjects. 31. (2) P.V. Sindhu won the silver medal for India in badminton
8. (2) Directive Principles enshrined in the Constitution of India in Rio Olympics 2016.
for the Government administration. People cannot 32. (3)
question in the court for not implementing Directive 33. (2) United Nations Organisation is the institution which
principles. People can move to court if their fundamental functions for international law, security, social equality
and world peace.
rights are violated.
34. (2) 35. (1) 36. (4) 37. (2)
9. (3) 10. (2) 38. (4) The constitution of India was adopted on 26 January,
11. (2) The finance bill is first presentated in the Lok Shabha. 1950.
Lok Sabha exercises more powers in money matters. 39. (1) The chairman of the drafting committee of the
12. (3) Constituent Assembly of India was B.R. Ambedkar.
13. (4) Parliament is the final authority for making laws in the 40. (4) In contemporary world legitimacy governance makes
country Parliament all over the World can make new democracy superior to any other form of government
laws, charge existing laws or abolish existing laws and that ensures appropriate use of power by a legally con-
make new one is their place. stituted government.
14. (3) Right to protect one’s culture is considered as the fun- 41. (4) Gender division refers to the difference between the
damental Right. It means any section of the citizen tasks assigned to men and women.
with a distinct language or culture have a right to 42. (3) The “Kittiko-Hachchiko” movement was started in
conserve. Karnataka in 1987.
15. (1) 16. (4) 43. (2) In case of conflict between the centre and the State
17. (2) Garner, An American prosessor of political science state Government over a subject in the concurrent list, the
the statement centre has the upper hand. The constitution provides
18. (1) In the Lok Sabha two members of the Anglo-Indian are primacy to parliament on concurrent list items; if there
community nominated by the president of India. is a conflict, a central law will override a state law.
19. (2) Joint Session of the Indian Parliamen tis presided by 44. (4) There are total 22 languages which got recognition in
speaker. In his absence by the Deputy speaker of Lok the eighth schedule of Indian constitution.
Sabha. 45. (3) Corruption is considered as anti–national and economic
20. (3) The councial of Ministers of responsible to the crime.
Parliament, in Indian Parliamentary system of 46. (4) The untouchability crime act was implemented in 1955.
governemnt. 47. (3) Right to Information Act is the law which empowers
21. (4) Section 25 says that all persons are equally entitled to the people to carry on democratic reforms.
freely profens practice and propagate religion. It
48. (2) 49. (3) 50. (3)
guarantees free entry to all people to social and religious
places. SECTION 6. CIVICS
22. (1)
1. (4) Dr Babasaheb Ambedkar was the chairman of the
23. (3) The Governors of the state are appointed by the
drafting Committee. The final draft of the Indian Constitution
president of India for a period of five years Vice was presented by him to Dr Rajendra Prasad on 25 November
President acts as the ex-officio chairman of Rajya Sabha. 1949. He was known as the father of the Indian Constitution.
In this capacity, his powers and functions are similar to Based on the seniority level, Sachchidananda Sinha was the
those of the speaker of Lok Sabha. On 1 January 2015 temporary President of the Constituent Assembly.
a cabinet resolution was passed to replace the Planning 2. (1) Narivadi Andolan was the movement that aimed to
Commission with the newly formed NITI Ayog (National fight for equal rights for women. It aimed to establish equal
Institution for Transforming India) Its chairperson is political, economic, and social rights and opportunities for
Shri Narendra Modi, Hon’ble Prime Minister Finance women in India. Many Indian feminists also have struggled
Minister presents Union Budget in the Parliament. against issues related to our culture, and patriarchal society
24. (1) The anti-defection law was passed by parliament in in our country, such as inheritance laws.
1985, it sets the provision for disqualification of elected 3. (3) A concurrent list is a list in which both Centre and
members on the grounds of defection to another state can make laws on subjects mentioned in it. On the
political parts. other hand, if there is a situation of a conflict between the
Hints & Solutions S-79

Centre and state over the law exist in the concurrent list, Money bills passed by the Lok Sabha are sent to the Rajya
the law of the central government is obeyed. Sabha (the upper house of parliament, elected by the state
4. (4) In India, seats are reserved for women in Panchayat and territorial legislatures or appointed by the president).
Raj bodies which are local self-governing bodies. 16. (3) The International Court of Justice is composed of 15
According to the 73rd amendment, one-third of the total judges elected to nine-year terms of office by the United
number of seats to be filled by direct elections present in Nations General Assembly and the Security Council.
local bodies, such as municipalities and panchayats are 17. (1) Originally the age of the retirement of the judges of
reserved for women. According to the data, more than ten the High Courts was fixed at 60 but it was raised to 62 in
lakh women have been elected in rural and urban areas. 1963 according to the 15th amendment of the Constitution.
5. (1) On 1 January 1979, the Janata Party government under Supreme court judges retire at the age of 65.
Prime Minister Morarji Desai established the Mandal 18. (3) The World Health Organization (WHO/OMS)
Commission, or the Socially and Educationally Backward Classes headquarters is situated on the outskirts of Geneva at the
Commission (SEBC). It was led by an Indian parliamentarian, end of Avenue Appia, approximately three kilometres from
the late B.P. Mandal. The commission aimed to identify the the centre of town, and about one and a half kilometres from
socially or educationally backward classes of India. the Palais des Nations (UN).
6. (3) Apart from general elections and by-elections, midterm 19. (3) The sixty-first amendment of the Constitution of India,
election refers to a type of election where the people can elect officially known as The Constitution (Sixty-first
their representatives and other subnational officeholders in Amendment) Act, 1988, lowered the age for voting of
the middle of the term of the executive. elections to the Lok Sabha and the legislative assemblies of
7. (3) All registered voters in the electoral roll of the Panchayat States from 21 years to 18 years. This was done by amending
in the village are members of Gram Sabha. The council leader Article 326 of the Constitution, which concerns elections
is named Sarpanch in Hindi, and each of the five members is to the Lok Sabha and the Assemblies.
a Gram Panchayat Sadasya or Panch. In such a system, each 20. (1) The Panchsheel, or Five Principles of Peaceful
villager can voice his opinion in the governance of his village. Coexistence, was first formally signed on April 29, 1954,
8. (3) If a person who is not a member of parliament is between India and the Tibet region of China. The agreement
appointed as a minister he has to get elected to either house was signed between then Prime Minister Jawaharlal Nehru and
of parliament within six months. After six months he ceases China’s first Premier (Prime Minister) Chou En-Lai.
to be a minister. In other words, a non-member may be 21. (1) Rajasthan was formed on 30 March 1949. When India
appointed as a minister, but he/she will have to get elected became independent, 23 princely states were consolidated
to either house within six months. to form the State of Rajasthan, “home of rajas”.
9. (2) Part IVA of the constitution stipulates fundamental duties 22. (2) There are significant difference
of Indian citizens which includes to abide by the constitution between the traditional Panchayati Raj system, which was
and respect its ideals and institutions, the National Flag and envisioned by Gandhi, and the system formalized in India
the National Anthem. This Act stipulates the punishment to in 1992. The Panchayat Raj system was first adopted
be awarded for violating Article 51Aa of the constitution. by the state of Rajasthan in Nagaur district on 2 Oct 1959
10. (3) The 42nd Constitutional amendment took place in 1976. but first started by Andhra Pradesh.
It was called Mini Constitution as it brought many 23. (3) The Republic is governed in terms of the Constitution
amendments to the Constitution. The main aim of the of India which was adopted by the Constituent Assembly on 26th
amendment is to reduce the power of the Supreme Court November 1949 and came into force on 26th January 1950.
and High courts and strengthening the central government. 24. (1) The eighteenth summit of ‘South Asian Association
It had no amendment related to bonded labour. of Regional Cooperation’ (SAARC) was held in Kathmandu,
11. (1) The Kesavananda judgment also defined the extent to the capital of the Federal Democratic Republic of
which Parliament could restrict property rights, in pursuit Nepal during 26–27 November 2014.
of land reform and the redistribution of large landholdings 25. (3) The Rajya Sabha should consist of not more than 250
to cultivators, overruling previous decisions that suggested members - 238 members representing the States and Union
that the property right could not be restricted. Territories, and 12 members nominated by the President.
12. (3) Constitution Day or Samvidhan Divas is also known as Rajya Sabha is a permanent body and is not subject to
National Law Day. It is celebrated in India every year on the dissolution
day the Constitution of India was adopted, i.e., 26 November 26. (1) “Clause (1) of Article 217 of the Constitution of India
1949 by the Constituent Assembly of India. However, the allows every Judge of a High Court to hold office until he
Constitution came into effect on 26 January 1950. attains the age of sixty-two years.
13. (1) The preamble of the Indian Constitution is:WE, THE 27. (3) The preamble is the summary of the Constitution. It
PEOPLE OF INDIA, having solemnly resolved to constitute highlights the main points and basic structure of the
India into a SOVEREIGN SOCIALIST SECULAR Constitution. It was adopted by the constituent assembly
DEMOCRATIC REPUBLIC and to secure to all its citizens. on November 26, the preamble aims to secure justice and
Therefore, we can say that the correct sequence of the liberty, provide equality to all citizens and maintain unity
terms in the Constitution is Sovereign, Socialist, Secular, and integrity of the nation.
Democratic, Republic. 28. (1) The Parliament has the power to amend the
14. (4) To be chosen a member of Parliament, a person must Constitution, though it has limited power. It cannot bring
be a citizen of India and not less than 30 years of age in the any change into the basic structure of the Constitution.
case of Rajya Sabha and not less than 25 years of age in the Under Article 368 the parliament has no power to change
case of Lok Sabha. regarding the Amendment in Part III of the Constitution.
15. (1) Money Bills can be introduced only in Lok Sabha (the Also, the Parliament does not have the power to change or
directly ele cted ‘people’s house’ of the Indian Parliament). increase its power to amend by amending Article 368.
S-80 Target NTSE
29. (4) A system of government, such as in the United States, have a State Legislative Council. These are Andhra Pradesh,
that divides power and responsibilities between the two Karnataka, Telangana, Maharashtra, Bihar, and Uttar
levels of government, i.e., national and state is known as the Pradesh The latest state to have a council is Telangana.
federal system. In the legal context, Independence of the 40. (2) An indirect election is an election in which voters do
Judiciary can be defined as the power to uphold without fear not choose between candidates for an office, but elect
or favour, the Rule of Law, personal freedom and liberty, equality people who then choose. In an indirect election, the voters
before the law and impartial and effective judicial control over elect a body which in turn elects the officeholder in
administrative and executive actions of the Government. They question. It is also used for some supranational legislatures.
must be free to judicial functions. 41. (1) When the General Assembly decides on important
30. (1) Ganesh Vasudev Mavalankar (27 November 1888 – 27 questions such as those on peace and security, admission
February 1956) popularly known as Dadasaheb was an of new members and budgetary matters, a two-thirds
independent activist, the President (from 1946 to 1947) of majority of those present and voting is required.
the Central Legislative Assembly, then-Speaker of the 42. (1) The INC has increased women’s participation by
Constituent Assembly of India, and later the first Speaker instituting a 33% quota for women at all levels of the party.
of the Lok Sabha. 43. (2) There were ten fundamental duties. Then, due to the
31. (2) The Right of Children to Free and Compulsory Education 86th Amendment in the Constitution in the year 2002, 11th
Act or Right to Education Act (RTE) is an Act of the duty was added.11 Fundamental duties have been
Parliament of India enacted on 4 August 2009, which delineated in the 4th chapter of the Indian constitution.
describes the modalities of the importance of free and 44. (4)
compulsory education for children between the age of 6 to 45. (3) Unnikrishnan v/s Andhra Pradesh Government,1993
14 years in India under Article 21A of the constitution. was a watershed judgement in the history of education in
32. (3) The All India Forward Bloc (AIFB) is a left-wing India. In this case, the Supreme Court declared the right to
nationalist political party in India. It emerged as a faction education as a fundamental right. It also mentioned that it
within the Indian National Congress in 1939, led by Subhas was the duty of the state to provide free and compulsory
Chandra Bose. The party re-established as an independent education to all children between the age of 6 to 14 years
political party after the independence of India. It has its under Article 21A of the Indian Constitution.
main stronghold in West Bengal.
33. (1) The National Democratic Alliance is an Indian political SECTION 7. GEOGRAPHY
alliance made up of predominantly centre-right political
parties and led by the Bhartiya Janata Party. It was founded 1. (4) Coffee was brought to India before the existence of
in 1998 and currently controls the Indian union government the East India Company. In India, the coffee plant was
as well as the government of 18 Indian states. introduced in 1670. An IndianSufi saint, Baba Budan
34. (1) A communication allotted by a proper authority stating broughtcoffee beans to India from Yemen during the later
the policy or decision of the government is referred to as 17th century. In India,coffee cultivation wasintroduced to
an office memorandum. The chief purpose that a the hills of Chikmagalur of Karnataka.
memorandum justifies is that it targets to record and relay 2. (2) A type of commercial farming in which a single crop is
information and to make brief requests. grown for the entire year is known as plantation agriculture.
35. (3) The Institutional Revolutionary Party (Spanish: Partido It is both intensive and capital intensive agriculture. Some
Revolucionario Institucional, PRI) is a Mexican political of the common examples for this crop are rubber, tea, coffee
party founded in 1929 that held uninterrupted power in the and sugarcane. This method is best known as the meeting
country for 71 years from 1929 to 2000, first as the National point for agriculture and industry.
Revolutionary Party (Spanish: Partido Nacional 3. (2) Neyveli lignite mines are located in the Cuddalore
Revolucionario, PNR). district state of Tamil Nadu. These mines come under the
36. (3) The Government of India Act, 1935 was passed by public sector. These mines own reserves of about 34,764
British Parliament in August 1935. With 321 sections and million tonnes. Lignite is low-grade brown coal. It has a
10 schedules, this was the longest act passed by British high content of moisture and therefore has a soft texture.
Parliament so far and was later split into two parts. The act 4. (4) The longest earthen dam in the world is the Hirakud
helped in the reorganization of the states. dam. This dam is situated don the great river, Mahanadi in
37. (4) The elections to the 243-seat Bihar Assembly took the Sambalpur region of Odisha about 15km. Post-
place in three phases the first for 71 seats on 28 October Independence, it was the first major multipurpose river
2020, the second for 94 seats on 3 November 2020, and the valley project in India. This dam aimed to control flood,
third for the remaining 78 seats on 7 November 2020. These supply water for irrigation purpose and generate electricity.
were the first big elections to take place amid the Covid-19 5. (2) In 1904, the first cement plant was set up in India in
pandemic outbreak. Chennai. This plant was set up in Porbandar, Gujarat but
38. (4) A cabinet is a body of high-ranking state officials, the production in the plant was started in Chennai. It was
typically consisting of the executive branch’s top leaders. set up by India Cement Company Limited. The capacity of
Members of a cabinet are usually called cabinet ministers the plant was 10,000 tonnes.
or secretaries. Cabinet committees are extra-constitutional 6. (3) Ernst Friedrich Schumacher was born on August 16,
bodies. However, they derive their legality from 1911 in Bonn, Germany. He died on September 4, 1977, in
the constitution. In constitutional parlance, a government Romont, Switzerland.He was a German-born British
is called an executive, which also includes the President of economist who developed the concepts of intermediate
India. technology. He wrote the book, Small is Beautiful in 1974.
39. (1) Its establishment is defined in Article 169 of the The book was based upon Gandhian philosophy.
Constitution of India. As of Jan 2020, 6 out of 28 states
Hints & Solutions S-81

7. (2) Industries can be classified into two categories on the years or above. It includes the population that can both
basis of their main role, namely key and consumer read and write. They should also be able to comprehend a
industries. The industries that supply their product or raw short simple statement he/she uses regularly.
materials to manufacture other goods are known as basic 17. (1) Gujarat and Maharashtra have the largest number of
industries, for example, iron and steel, copper smelting and cotton textile centres. Although cotton textile mills are
aluminium smelting. The industries that sell their products located in over 80 towns and cities of India, yet its larger
to the consumer directly are known as consumer industries, concentration is found in Maharashtra, Gujarat, West
for example, automobile. Bengal and Uttar Pradesh.
8. (4) Non-renewable energy comes from sources that will 18. (3) The northern plains are suitable for constructing
run out or will not be replenished for thousands or even railway lines. The plains give the platform for the tracks.
millions of years. Most sources of non-renewable energy 19. (2) Greenwich Mean Time (GMT) is the mean solar time at the
are fossil fuels. ... Most non-renewable energy sources are Royal Observatory in Greenwich, London which is measured at
fossil fuels, such as coal, petroleum, and natural gas. midnight. In English speaking nations, GMT is often used
9. (3) McAdam devised the method of creating pakka roads. synonymously with Universal Time Coordinate (UTC).
This method was invented by John Loudon McAdam in 20. (3) The city has a Mediterranean climate, with rainy
1820 for the construction of roads. In this method, layers of winters and dry summers.
single-sized crushed stone are placed in shallow lifts and 21. (4) Tropic of Cancer is an imaginary line, at an angle of
compacted thoroughly. In 1829, Robert Stephenson 23.50 degrees North from the Equator, that passes through
developed a steam locomotive named rocket. It was the the middle of India
only locomotive that completed the trials with a maximum 22. (1) Kolleru is located between Krishna and Godavari
speed of 30 mph. deltas. Kolleru spans into two districts - Krishna and West
10. (4) Mediterranean vegetation includes trees with thick God kavari. The lake is fed directly by water from the
barks and wax-coated leaves. These trees help them reduce seasonal Budameru and Tammileru streams and is
transpiration. This adaptation helps them in their survival, connected to the Krishna and Godavari irrigation systems
especially in dry summer. These trees are known as the by over 67 major and minor irrigation canals.
orchards of the world for their fruit cultivation. It consists 23. (2) The Diamond Quadrilateral is a project of the Indian
of trees with a height of 2.5 m or 8 feet. railways to establish a high-speed rail network in India.
11. (2) In the 2018/2019 crop year, around 148.5 million metric The Diamond Quadrilateral will connect the four mega cities
tons of milled rice was produced by China. This amount is a in India, Delhi, Mumbai, Kolkata and Chennai, similar to
higher volume than any other country. India produced 116.42 the Golden Quadrilateral expressway system.
million metric tons of milled rice in that crop year and became 24. (2) The colour brown is used to denote most contour lines
the second-largest producer in the world. India accounts for on a map, which are relief features and elevations.
25% of global production is the largest producer in the world. Topographic maps use green to denote vegetation such as
Its consumption accounts for 27% of world consumption woods, while blue is used to denote water features like
and 14 % of the total import of pulses in the world. lakes, swamps, rivers, and drainage
12. (4) The seawater is diluted due to the addition of 25. (2) Rajsamand Lake is a lake near the town of Rajsamand in
freshwater from many rivers to it. It reduces the content of the Indian state of Rajasthan. Builtin 1660 by Rana Raj Singh,
salt and therefore reduces the density of the seawater. The it is approximately 1.75 miles wide, 4 miles long and 60 feet
Baltic Sea is almost enclosed by northern Europe and deep. It was built across the Gomati, Kelwa, and Tali rivers.
Scandinavia. The sea has a very low salinity of about 10 ppt. 26. (3) Barley is the fourth most important cereal crop in the
13. (3) Coromandel Coast is situated on the coastal plain in world after wheat, maize, and rice. Although generally a
eastern Tamil Nadu state. It spreads over an area of about temperate crop, barley is also grown in many tropical countries,
8,800 square miles The Northern Circars is situated on the typically by poor farmers in hostile, dry, cool environments.
western coast of the Bay of Bengal in the state of Andhra 27. (1) Chittorgarh is the cement city of Rajasthan.
Pradesh. Malabar is situated on the coast of the Arabian 28. (3) Literacy in India is a key for socio-economic progress.
Sea in Kerala. Canara is situated on the coast of the Arabian Despite government programmes, India’s literacy rate
Sea, western Karnataka state, India increased only “sluggishly”. The 2011 census, indicated a
14. (1) The south-west monsoon winds are not very strong and 2001–2011 decadal literacy growth of 9.2%, which is slower
therefore start withdrawing gradually. By the onset of October, than the growth seen during the previous decade
the monsoon withdraws from the northern plains. This 29. (2) Cobalt is a ferrous metallic mineral. The metallic
withdrawal of the monsoon is shown by clear skies and an minerals that contain iron are known as ferrous minerals. In
increase in temperature. On the other hand, Tamil Nadu receives the total production of metallic minerals, 75% is constituted
scanty rainfall as it is in the rain shadow area of the Arabian by ferrous metallic minerals. Its common examples include
sea branch and is parallel to the Bay of Bengal branch. iron ore, manganese, chromite and nickel
15. (1) The leading world’s jute producing countries are India, 30. (4) The people in the region depend on rainwater
Bangladesh, China and Thailand. India is the world’s largest harvesting. The harvested rainwater from the monsoon
producer of raw jute and jute goods, contributing to over season (during July and August) is stored in pucca tanks
50 percent and 40 percent respectively of global production. and used throughout the year for drinking purposes.
The first cement plant was set up in the year 1904, in 31. (1) The given quote was given by Smt. Amrita Devi. This
Chennai (Tamil Nadu). quote was given to save trees. She fought against tree cutting.
16. (4) Infant mortality rate (IMR) is the number of children 32. (1) The Green Revolution in India started in the late 1960s
who die within one year, out of 1000 live children born and with its success, India attained food self-sufficiency
under one year of age. The literacy rate is defined as the within a decade. In India, it was first introduced in Punjab in
percentage of the population belongs to the age group of 7 the late 1960s as part of a development program issued by
S-82 Target NTSE
international donor agencies and the Government of India. 45. (3) The Tea Board of India is a state agency of
During the British Raj, India’s grain economy hinged on a the Government of India established to promote the cultivation,
unilateral relation of exploitation. processing, and domestic trade as well as export of tea from
33. (2) Bharatpur Bird sanctuary is also known as Keoladeo India. It was established by the enactment of the Tea Act in
National Park. It is situated in Bharatpur, Rajasthan. It is a 1953 with its headquarters in Kolkata (formerly Calcutta).
man-made sanctuary. In the sanctuary, more than350 species 46. (3) Westerlies wind is known with different names such
of birds can be found including herons, cormorants and eagles. as“roaring forties” at 400 ‘furious fifties’ at 500 and ‘screaming
Here, the birds can be seen mostly in the winter season. sixties’. These winds blow in the Southern Hemisphere. Their
34. (3) In India, the literacy rate is 74.04 per cent, 82.14 for names are given based on the latitudes on which they occur
males and 65.46 for females. The state of Kerala has the highest in the Southern Hemisphere and their intensity.
literacy rate, 93.91%. After this, the literacy rate in Laksha 47. (3) The city, Pulicat is located in the district of Tiruvallur
dweep is 92.28 per cent and in Mizoram, it is 91.58 per cent. in the state of Tamil Nadu. It is situated almost 60kms north
35. (2) Rajgir ropeway is the oldest ropeway situated in the of Chennai city particularly on the seaward side of Sri
state of Bihar, India. It is a ropeway that takes towards the Harikota island blockade that segregates Pulicat Lake from
hills of Ratnagiri where the famous Vishwa Shanti Stupa is the Bay of Bengal. It is a saltwater lagoon.
situated, also known as the Peace Pagoda. It goes to a 48. (1) Tehri Dam is situated on the Bhagirathi river. Hirakud
height of over 1000 feet from the ground level. Dam is situated on the great river, Mahanadi. Nagarjuna
36. (4) The Kosi River is known as the “Sorrow of Bihar”.The Sagar Dam is situated on the Krishna river. Indira Gandhi
river got its name since it is affected a huge area of Canal is situated on the river Sutlej.
agricultural land, 21,000 km2 since it ia flood-prone area. 49. (4) Kachchi is not a part of the world network biosphere
This leads to the disturbance in the rural economy. The reserves of UNESCO. The World Network of Biosphere
headquarters of Maruti Industries situated in Delhi, India. Reserves of the MAB Programme consists of a dynamic
Varanasi is located on the left bank of the GangaRiver and and interactive network of sites of excellence. 157 sites in
is one of the seven sacred cities of Hinduism. 24 countries in Asia and the Pacific. 302 sites in 38 countries
37. (1) Patna airport is also known as Jay Prakash Narayan in Europe and North America. 130 sites in 21 countries in
Airport. It is situated in Patna, the state capital of Bihar in Latin America and the Caribbean.
India. It is a custom airport. It is named after independence 50. (3) Print Media is the most common but powerful means
activist, Jayaprakash Narayan. It has a short runway and of communication. It creates credibility. Investing in printed
therefore it is known as a restricted international airport. material signals to customers that you are serious about
38. (2) Medha Patkar with other is allies of NAPM were the your business and that you offer a worthwhile product or
founder of Narmada Bachao Andolan (NBA) and Ghar service. Printed materials can also create engagement
Bachao Ghar Banao Andolan. She started NBA in three states, between the customer and the brand.
Madhya Pradesh, Maharashtra and Gujarat. Mangrove forest 51. (2) Hurricane Committee determines a pre-designated list
is found in the coastal area in India. A type of farming in of Hurricane names southern India, on 16. Cyclone
which only one crop is grown is known as plantation Gaja is named Sri Lanka and Cyclone title was named
agriculture. It is also a type of commercial farming. Gaja which means elephant in language!, 2020 each country
39. (3) A national highway network that connects most of the deep depression strengthened into a cyclonic storm
the major industrial, agricultural and cultural centres of 52. (1) The full form of HCR is Head Count Ratio. The head
India is known as the Golden Quadrilateral Super highway.
count ratio (HCR) is the population proportion that exists, or
It forms a quadrilateral connecting the four major metro cities
lives, below the poverty threshold. One of the undesirable
of India, i.e., Delhi (north), Kolkata (East), Mumbai (west)
and Chennai (south). features of the head count ratio is that it ignores the depth
40. (3) Black soil is also called Regur soil. It is black in colour of poverty; if the poor become poorer, the head count index
and ideal for growing cotton. This type of soil is typical of does not change.
the Deccan trap (Basalt) region spread over the North-West 53. (2) India’s and Asia’s first nuclear reactor was the Apsara
Deccan plateau and is made up of lava flows. research reactor at Mumbai. The domestic uranium reserve in
41. (4) The Nile River, located in Africa, is listed as being 6,853 India is small and the country is dependent on uranium imports
kilometres (4,258 miles) long and is hence commonly considered from other countries to provide fuel to its nuclear power industry.
to be the longest river in the world. On the other hand, The 54. (1) Jaisalmer is called the driest place in India because,
Amazon is considered the world’s largest river by volume, considering the past weather records, this district receives
42. (2) Toda people are a Dravidian ethnic group who live in the lowest annual rainfall in India. Ruyli located in the
the Nilgiri Mountains of the Indian state of Tamil Nadu. Their Jaisalmer district get the least amount of rainfall – 8.3 cms
economy was pastoral, based on the buffalo, whose dairy products 55. (3) As per the National Highways Authority of India, about
they traded with neighbouring peoples of the Nilgiri Hills. 65% of freight and 80% of passenger traffic is carried by the roads.
43. (1) In 1992, more than 100 heads of state met in Rio de Janeiro, 56. (2) The Brazilian coast with its 8000 km reaches from north
Brazil for the first international Earth Summit convened to of the equator to the far south, the border to Argentina. A
address urgent problems of environmental protection and big part is in the tropic regions, the southeast is sub-tropical
socio-economic development. (with Rio and Sao Paulo) and further in the south the climate
44. (2) MGNREGA was implemented in 200 districts in the first is about as in the Mediterranean
phase with effect from February 2006 and extended 57. (1) Pakistan is not a major trading partner of Brazil. The
subsequently, to additional 113 and 17 districts with major countries are the US, China, India, Canada, Argentina
effect from April 1st 2007 and May 15th 2007, respectively. and Nether land. Various products that Brazil import are
The remaining districts were included under the Act with iron ore, crude petroleum, soybeans and raw sugar.
effect from April 1st, 2008.
Hints & Solutions S-83

58. (3) Brazil ishome toone ofthe most popular ecotourism destinations 73. (3) The East-West corridor connects Silchar (Assam) and
on Earth: the Amazon. The river itself and the surrounding Porbandar (Gujarat). The highway project are
jungle are a major draw for thousands of tourists every year. implemented by the National Highway Authority of India
Everything from structured day hikes from jungle lodges
to extended treks is very popular. (NHAI).
59. (2) India is a vast country. Lying entirely in the 74. (1) The Himalayan yew is an endangered native high value
Northern hemisphere (Figure 1.1) the mainland extends medicinal plant of the Himalayan region. The several
between latitudes 8°4’N and 37°6’N and longitudes 68°7’E medicinal properties of the bark and leaves of this
and 97°25’E. species have increased its risk of extinction.
60. (3) Goa is the most urbanized state in India. Among the 75. (2) Black soil is ideal for growing cotton. This type of soil is
States, Goa is now the most urbanised State with a 62.2
percent urban population, a significant increase since 2001 typical of the Deccan trap plateau and is made up of lava
when the urban population of Goa was 49.8%. Another flows.
significant instance of rapid urbanisation is that of Kerala, 76. (4) In Madhya Pradesh it is known as Bewar or Dahiya. In
its urban population is now 47.7 per cent, while a decade Odisha it is known as Pama Dabe or Koman. In western
ago it was just 25.9 percent. Ghats it is called kumari . In North-east region it is known
61. (4) Bhima River, a major tributary of the Krishna River,
by the name Jhumming.
flowing through Maharashtra and Karnataka states,
western India. It rises in the Bhimashankar heights of 77. (2) 78. (2) 79. (3)
80. (1) Uttar Pradesh, is the major sugarcane producing state.
the Western Ghats and flows southeastward for 450 miles
The other states include are Maharashtra, Karnataka,
(725 km) in Maharashtra to join the Krishna in Karnataka. Tamil Nadu, Andhra Pradesh, Bihar, Punjab and
62. (2) Sri Lanka gave the name Gaja to the cyclone. It came Haryana.
on November 11 and was the deep depression that 81. (4) 82. (1)
strengthened into a cyclonic storm.
83. (4) The basket of egg topography is a glacial landform
63. (2)
due to depositions of glacial sediments or till.
A. Nhava Sheva is situated in Maharashtra.
84. (3)
B. Tuticorin is situated in Tamil Nadu.
85. (1) Kaziranga National Park is a national park in the
C. Paradip is situated in Odisha.
Golaghat and Nagaon districts of the state of Assam.
D. Haldia is situated in West Bengal. Sunderbans National Park is a national Park Tiger
64. (1) When granite is subjected to intense heat and pressure, Reserve and a Bioshere reserve in west Bengal. It is
it changes into a metamorphic rock called gneiss. It has a located in the sunder bans delta of West Bengal. Gir
more sandy and less clayey composition. It does not retain National Park is a wildlife sanctuary in Gujarat western
moisture. Total area covered in India about 5.2 lakh sq. km. India. It is famous Asialic lions. Tandova National park
65. (3) A Blue Sheep is a vulnerable species. Wild Asiatic is a tiger reserve in Chandrapur district of Maharashtra.
buffalo is a rare species. The Nicobar region is an endemic 86. (4) Both the statements are true
species. Asiatic cheetah is an extinct species. 87. (4)
66. (2) Bhakra Nangal project is a joint venture of the states 88. (1) Names of cylones of different countries such as in India
of Punjab, Haryana and Rajasthan. It is situated on the it is called Jal. In Bangaladesh it is known as Giri. In
river Sutlej. It was a multipurpose project. It produces myanmar it is known as Thane. In Oman it is called
hydroelectric power. Mujan.
67. (4) A national highway network that connects most of the 89. (2)
major industrial, agricultural and cultural centres of India is 90. (2) These states such as Sikkim, West Bengal, Assam and
known as the Golden Quadrilateral Superhighway. It forms Arunachal Pradesh hare Common frontiers with Bhutan.
a quadrilateral connecting the four major metro cities of 91. (3) 92. (3)
India, i.e., Delhi (north), Kolkata (East), Mumbai (west) and 93. (3) Tea an important beverage crop introduced in India
Chennai (south). inetially by the British. Tea cultivation in an example of
68. (4) Mica is a mineral made up of series of plates or leaves. plantation agriculture.
Mica can be clear, black, green, red, yellow or brown. 94. (3) Limestone is not used in oil refinery industry.
Due to its excellent di-electric strength, low power loss 95. (2) Koyana Multipurpose Project is set up on the Koyana
factor, imulating properties and resistance to high river in Satara district (Maharashtra). On the basis of
electricity generating potential, this river is called as
voltage, mica is one of the most important minerals used the life line of Maharashtra.
in electric and electronic industries. 96. (3) 'Slash and Burn' agriculture is known as 'Roca' in Brazil.
69. (1) Lake Victoria lies on the equator. It is a shifting cultivation in which field is cultivated for
70. (4) The state of Gujarat receives South-West monsoon short period then left for recovering.
lately. 97. (2) Mumbai–Pune Expressway is India's first six–lane high
speed expressway and was made by Maharashtra State
71. (2) The per capita consumption of electricity is considered Road Development Corporation.
as an index of development. 98. (1) Kavaratti is the capital of the Union Territory
72. (4) Hills from west to east are Garo, Khasi, Jantia and Naga. Lakshdweep and the region comes under the
jurisdiction of Kerala High Court.
S-84 Target NTSE
99. (1) Copper Mines are found in Khetri, a town in Rajasthan November 2020. This celebration took place for a week for
State. various purposes. Here, cooperatives have the opportunity
100. (3) Rusting of rock is caused by the process of oxidation. to share their success stories, and achievements with
101. (2) Roches Montanes are the hillocks of rocks that are others. They also develop future action plans for their
formed by Glacial Abrassion and Plucking. organisations.
102. (1) 103. (1) 104. (1) 105. (2) 7. (2) The WTO began life on 1 January 1995, but its trading
106. (1) Gondwana land is the name given to an ancient system is half a century older. Since 1948, the General
supercontinent that included India, Australia, South Agreement on Tariffs and Trade (GATT) had provided the
Africa and South America. rules for the system. (The second WTO ministerial meeting,
107. (1) Tapi river run through rift valleys. held in Geneva in May 1998, included a celebration of the
108. (1) Western Disturbances is an extratropical storm 50th anniversary of the system.)
originating in the Mediterranean region that rainfall in 8. (1) Laissez-faire economics is a key part of free-
market capitalism. The policy of laissez-faire received
the Northern and North-Western parts of India in the
strong support in classical economics as it developed in
month of October-November. Great Britain under the influence of the philosopher and
109. (2) The slash and burn agriculture is the primitive form of economist Adam Smith.
cultivation and known as Milpa in Mexico. 9. (3) In economics, the exclusion principle states “the
110. (2) B, C and D owner of a private good may exclude others from use unless
All the three methods such as construction of check they pay.”; it excludes those who are unwilling or unable to
dam, contour plonghing, afforestation can be adopted pay for the private good, but does not apply to public
to minimize soil erosion. goods that are known to be indivisible: s
111. (3) (i) Nagarhole (B) 10. (3) Indian railways in an example of monopoly as there is
(ii) Gir (A) so close substitute for Indian railways which provide
(iii) Kaziranga (D) transportation at such nominal rates.
(iv) Jim Corbett (C) 11. (1) The ability to pay is another criterion of equity or
112. (2) The slash and bum agriculture is known as Milpa in fairness in taxation. This theory requires that individuals
Mexico. should be asked to pay taxes according to their ability to
113. (2) Coal is non-renewable as well as non-recyclable re- pay. The rich have a greater ability to pay, therefore they
source. should pay more tax to the Government than the poor.
114. (3) 115. (4) 12. (3) In underdeveloped countries, most of the labour force
is generally engaged in the agricultural sector. People are
SECTION 8. ECONOMICS involved in agriculture as there is a fewer number of
1. (3) In India, the Golden revolution referred to the period employment opportunities.
between 1991 to 2003. This revolution is associated with 13. (2) India too is a mixed economy and it has adopted this
the production of honey and horticulture. It is one of the approach post-independence. The industrial policies
important agricultural revolutions of India. It led to an implemented in the year 1948 and 1956 have helped the
increase in the production of fruits, vegetables, flowers, private and the public sector to co-exist.
aromatic plants, spices, etc. Nirpakh Tutaj is known as the 14. (4) The Reserve Bank of India (RBI) is the central bank of
father of the golden revolution. India, which was established on Apr. 1, 1935, under the Reserve
2. (2) Voluntary unemployment can be defined as the Bank of India Act. The Reserve Bank of India uses monetary
situation when the workers intentionally and consciously policy to create financial stability in India, and it is charged
choose to leave due to certain reasons such as low salary with regulating the country’s currency and credit systems.
package, lack of opportunity to work. Under this type of 15. (2) Privatization can be defined as the process by which
employment, the worker remains unemployed even if the something can be moved from the public sector to the
jobs are available in the market. private sector. This process increases the role of the private
3. (4) The Human Development Report (HDR) is an sector in the economy. Nowadays privation of the different
annual report published by the Human Development Report sectors is taking place.
Office of the United Nations Development Programme 16. (2) The money which is issued by the central bank of a
(UNDP). The first HDR was launched in 1990 by the country in form of metals that acts as an unlimited legal
Pakistani economist Mahbub ul Haq and Indian Nobel tender in the economy is known as metallic money. This
laureate Amartya Sen. kind of money is made of precious metals such as gold and
4. (2) Gurgaon is the most developed commercial and business silver. The face value of the money can be determined by
centre of Haryana, located on the outskirts of Delhi. It is spread the value inscribed on the coins.
over 2,766 sq km and has a population of over 600,000. The 17. (3) A mixed economy is can be defined as when both
industrial areas around Gurgaon house most of the automobile public and private enterprises exist together. Along with
and auto component manufacturers in the state. there is a level of freedom to utilize capital to earn profit.
5. (4) A tax that is paid by a person or organisation directly However, the interference of the government aims to benefit
to the entity that imposed it. For example, a person pays society. Therefore we can say that a mixed economy is where
direct taxes to the government for various purposes, both capitalism and socialism exist at the same time.
including income tax, property tax, corporate tax, or taxes 18. (1) In Maharashtra, the Employment Guarantee Scheme
on assets. Direct taxes are dependent upon the income of (EGS) evolved from the relief employment programs that
an individual. were undertaken during a severe drought in the state from
6. (1) The National Cooperative Union of India celebrated 1970 to 1973. The droughtaffected 15 to 30 million rural
the 67th All India Cooperative Week from 14th to 20th people, constituting from 43 percent to 86 percent of the
state’s rural population.
Hints & Solutions S-85

19. (1) It was certainly a shock to Marco Polo, who wrote 32. (3) To prefer sustainability of development
about it in his travels, saying: With these pieces of paper they 33. (1) 34. (1) 35. (4)
can buy anything and pay for anything. And I can tell you 36. (1) In India, the Reserve Bank of India issues currency
that the papers that reckon as ten bezants do not weigh one. notes on behalf of the central government. As per Indian
20. (1) The entrepreneur is commonly seen as a business leader law no other individual or organisation is allowed to issue
and innovator of new ideas and business processes. currency.
21. (3) 37. (3) 38. (4) 39. (1)
(1) Minimum wages Act came into existence in 1948. 40. (1) To control situation of deflation it is essential to
(2) Prevention of unethical activities Act came into increase the demand for bank loan.
enforcement in 1956. 41. (1) Economic rent is the price paid for the use of land or
(3) Prohibition of bonded labour Act came into natural resources inhose supply is perfectly inalastic.
enforcement in 1975. 42. (1) Money is not the factor of production because it is a
(4) Human rights protection Act came into enforcement
resource used to acquire resources that go into producing
in 1993.
22. (1) In the journal, business transactions are initially goods.
recorded and therefore it is called the book of original entry. 43. (4) Proximate cause is a key principle of insurance and is
It is known so because in this book. All business concerned with how the loss or damage actually occured
transactions, their details and descriptions are first recorded and whether it is indeed as a result of an insured peril.
in the book of original entry. The full particulars of the 44. (1) Naresh Goyal is an Indian businessman and founder
transactions are recorded first in the Journal. chairman of Jet Airways. He started operating Jet Airways
23. (4) Organised labour in India is covered under the in 1993. Kiran Mazumdar Shaw an Indian billionaire
Gratuity Act of 1971. In India, agriculture is one of the fields entreprenuer. She is the chairperson and managing director
that give employment to unorganised labourers. Child of Biocon Ltd, a biotechnology company based in Banglore.
labour mainly comes under the unorganised sector. Children Narayan Murthy, is an Indian IT industrialist and the co-
are hired as cheap and fast workers. There is no security founder of Indosys Dhirubhai Ambani, a business tycoon
for the employers, due to the involvement of highly who founded Reliance industries in Bombay.
unskilled children. This sector also faces the challenges of 45. (1) 46. (3) 47. (2) 48. (2)
migration. 49. (3) Under Antyoday Anna Yojana the wheat is supplied at
24. (4) A direct tax is a tax that is paid directly to an imposing the rate of `2 per kg and `3 per kg for rice.
entity, usually the government, by an individual or an 50. (3) 51. (3)
organisation. This type of tax cannot be transferred to 52. (4) All the statements given about Annapurna Scheme are
another individual or entity. Income Tax, Corporate Tax, correct.
Wealth Tax, Stamp Duty are direct taxes. The collection 53. (3) A. D is parities in income in a developing economy
and levying of direct taxes are regulated by the Central — Less redistribution of income in favour of
Board of Direct Taxes. poor.
25. (2) Antyodaya Anna Yojana was launched in 2000 to B. Economic development benefits the poor-Trickle
provide foodgrains, namely, rice and wheat to extremely down theory
poor families in rural India. Under this scheme, one crore
C. Shifting of labour from agricultural to non
BPL (Below Poverty Line) families covered under the
agricultural sector.
targeted public distribution system were identified. Poor
families were identified by the respective state rural Change in occupational structure
development departments through a BPL survey. D. Increase in the capability of people–Human
It was first implemented in Rajasthan. Each of the identified development
families was issued 25 kg foodgrains (rice and wheat) 1 1 1
initially, but this was later increased to 35 kg per family with 54. (3) HDI = (LEI) + (EI) + (GDP Index)
3 3 3
effect from 1st April 2002.
1 1
26. (3) National Bank for Agriculture and Rural Development Þ [ 0.53 + 0.67 + 0.42 ] Þ ´ 1.62 Þ 0.54
(NABARD) is an apex development financial institution in 3 3
55. (2) Cash reserve ratio = 20%
India. It looks after the credit needs of agriculture and rural
Banking system amount = 50 million
development in India.
Maximum amount of demand deposit =
27. (2) There are three different phases in circular flow of
1
income. These are production phase, income phase and Banking system amount ´
expenditure phase. Re serve ratio
28. (3) Bombay stock exchange is the tenth largest stock ex- 1 5
change in the world and oldest stock exchange in South = 50 million ´ 20 ´ 100 = 250 million
Asia. 56. (3) The Govt of India supplies food grain eg. rice, wheat
29. (3) Indicators involved in HDI are life expectancy, literacy sugar, etc. and other essential commodities to below
attainment and purchasing power of people poverty line house holds through Public distribution
30. (1) The group which classifies the above aspects are system. PDS is an Indian food security system.
narrow and broad money. 57. (3) 'Renaissance' word is used for the phase in
31. (1) Removing barriers or restrictions set by the govern- European history for many changes between 14th
ment is called liberalisation. and16th centuries in the field of classical art and
intellect.
S-86 Target NTSE
58. (2) 59. (1) 60. (3) 61. (1) 4. (2) Let number of boys be x, girls be y.
62. (1) GDP is the logical extension of measuring economic \ ( x + y )15.8 = 16.4 x + 15.4 y
growth in terms of monetary expenditures
OR Þ 15.8 x + 15.8 y = 16.4 x + 15.4 y
Economic growth is the increase in the market value Þ 0.6 x = 0.4 y
of goods and services produced by an economy over
x 0.4 2
time. It is measured as the percent rate of increase in Þ = = .
real GDP. y 0.6 3
63. (3) The Indian Constitution provides for a Finance x y x
Commission to advice the President on distribution of 5. (1) Q = = =a
financial resources between the union and the states. 2x + y + 2 x + 2 y + 2 x + y + 2z
64. (2) G.B.P.A.U. Pant Nagar is the first Agricultural University x+ y+ z
of India established in 1960. Þa=
2x + y + z + x + 2 y + z + x + y + 2 z
65. (3) Creation of Utility for exchange is called production.
66. (2) Rise in population size will increase the demand for a x+ y+ z 1
Þa= = .
necessary commodity. 3( x + y + z ) 3
67. (3) Inflation indicates a situation of continuous rise in 6. (4) Number of cards = 48
prices. Number of cards divisible by 5 = 10
68. (3) M. S. Swaminathan is known as the father of Green Number of prime cards = 12
Revolution in India for his contributions to the Number of cards divisible by 19 = 3
development of high-yielding varieties of wheat. Number of composite number cards = 36
69. (4) Entertainment Tax is an example of Indirect taxes. Which \ (i) - (r), (ii) - (p), (iii) - (s), (iv) - (q).
are not directly levied on the income of an individual 7. (4) Let three prime number are a, b, c
but is indirectly levied on the expense incurred by the A.T.Q.
individual. 437 = a ´ b = 19 ´ 23 ...(i)
70. (1) The Academy of development Science supports food and 551 = b ´ c = 19 ´ 29 ...(ii)
security programme in Maharashtra by setting up From (i) and (ii), we get
Grains Banks.
71. (2) The 6th summit of BRICS hosted by Brazil was held in \ b = 19, a = 23, c = 29
July 2014. \ a + b + c = 19 + 23 + 29 = 71.
SECTION 9. MATHEMATICS 8. (4) Let p( x ) = x 3 + 8 x 2 - 3kx + 7 divided by x – 1
1. (2) Let `x, `y and `z money invest by P, R and S Remainder p(1) = 1 + 8 - 3k + 7 = 16 - 3k
respectively.
Time duration of P = 60 – 26 = 34 years and q( x) = 2 x3 + kx 2 - 5x + 6 also divide by x – 1
Time duration of R = 60 – 27 = 33 years Remainder q(1) = 2 + k - 5 + 6 = 3 + k
Time duration of S = 60 – 28 = 32 years A.T.Q.
34 33 32
æ 1ö æ 1ö æ 1ö p(1) + q(1) = 9
\ x ç1 + ÷ = y ç1 + ÷ = z ç1 + ÷
è 10 ø è 10 ø è 10 ø Þ (16 - 3k ) + (3 + k ) = 9 Þ 10 = 2k Þ k = 5.
121 11 9. (4) Since x 2 + bx + a = 0 and x 2 + ax + b = 0 have equal
Þx =y =z
100 10 roots.
x y z For equal roots, we have
Þ = = D = 0 and D' = 0
100 110 121
\ x : y : z = 100 :110 :121. Þ b2 - 4a (1) = 0 and a 2 - 4(b)(1) = 0
28 + 34 + 41 + 23 + 45 + 18 + 21 210 Now, b 2 - 4a = a 2 - 4b = 0
2. (1) Mean = =
7 7
Þ b 2 - a 2 = 4a - 4b
\ x = 30
Þ (b - a)(b + a ) = -4(b - a)
\ 2 - log x 2 - log x 3 - log x 5 = 2 - log x (2 × 3 × 5)
\ a + b = -4 (Q a ¹ b)
= 2 - log x 30 = 2 - log30 30 = 2 - 1 = 1.
10. (2) A
3 3
3. (3) Given that x + = 5 Þ 5 - x = and x 2 + 3 = 5 x.
x x
2 3 H
x × P
x 2 (5 - x ) x 2 + x + 3 x + 5x + x
\ + =
2 2
x + x + 3 x (5 - x) 5 x + x 3
x2 ×
x
3x 6 x 1 5
= + = +2= . B B C
6 x 3x 2 2
Hints & Solutions S-87

A.T.Q.
H–B=2 Þ B=H–2 ...(i) x 3 x
cos 30° = Þ =
H –1 OA 2 4
and H = 2P + 1 Þ P = ...(ii) (Q ÐAOB = 120° and ÐOAB = ÐOBA)
2
In DABC, Þ x =2 3
\ Side = 2 x = 4 3
Þ H 2 = P2 + B 2 (By pythagoras theorem)
2 14. (4) A
æ H - 1ö
Þ H 2 = ( H - 2)2 + ç
è 2 ÷ø
(By using equation (i) & (ii)) 12 cm
2
Þ H - 18 H + 17 = 0
Þ ( H - 17)( H - 1) = 0
Þ H = 17, H = 1 (not possible) B 3 cm X C
\ Base = B = H – 2 = 17 – 2 = 15. Since X is mid point of BC.
11. (3) A.T.Q. \ BX = CX = 3 cm
1 Þ BC = 3 + 3 = 6 cm
T p = = a + ( p - 1)d ...(i)
q AX is also angle bisector of ÐA , we have
1 AB BX 12 3
Tq = = a + (q - 1)d ...(ii) = Þ = Þ AC = 12 cm
p AC XC AC 3
Subtract eqn. (ii) from (i), we get 12 + 6 + 12 30
Now, S = AB + BC + AC = = = 15
1 1 p-q 1 2 2
Þ ( p - q)d = - = Þ d=
q p pq pq Area of DABC = 15(15 - 12)(15 - 12)(15 - 6)
Put in eq. (i)
= 15 ´ 3 ´ 3 ´ 9 = 9 15 cm 2
1 1 1 15. (3) Let radius of circle is r cm
Þ a + ( p - 1) = Þ a=
pq q pq \ Circumferance = 2pr
Since square enclose the circle.
Now, \ Length of square = 2r
1 1 Now, perimeter of square = 4(2r ) = 8r
T pq = a + ( pq - 1)d = + ( pq - 1)
pq pq
P
1 1 A.T.Q., 2pr + 8r = P Þ r =
= +1- =1 2p + 8
pq pq 16. (4) Since x + k is a common factor of x 2 + px + q and
12. (4) Given that,
2 2 x 2 + lx + m
x = 2sin q , y = 2cos q
\ k 2 - pk + q = k 2 - lk + m = 0
We have A.M. ³ G.M.
Þ lk - pk = m - q
x+ y x+ y 2 2
So, ³ x× y Þ ³ 2sin q × 2cos q Þk=
m-q
2 2
2 2
l- p
q q
2sin + 2cos 17. (3) Let original price of rice be Rs. x / Kg.
Þ ³ 21 (Q cos 2 q + sin 2 q = 1) 1400
2 \ Total quantity =
2 2 x
Þ 2sin q + 2 cos q ³ 2 2.
13. (3) In equilateral triangle centroid and circumcentre are æ 40 ö 7 x
New price = x ç 1 + ÷=
coincide è 100 ø 5
C 1400 ´ 5 1000
Total quantity = =
7x x
(–2, 2) A.T.Q.
O 1400 1000
- = 10
x x
30° Þ 400 = 10 x Þ x = 40.
A x x B 18. (1) Total possible outcomes = 4 × 4 = 16
(2, 2) The product of the numbers is more than 14
= {(3, 6), (3, 8), (5, 4), (5, 6), (5, 8), (7, 4), (7, 6), (7, 8)}
OA = (2 + 2)2 + (2 - 2)2 = 4 \ Number of favourable outcomes = 8
S-88 Target NTSE

8 1 C
1
\P = = .
16 2
19. (2) Given that, sin A + sin 2 A = 1
sin A = 1 - sin 2 A = cos 2 A A
cos 2 A + cos 4 A = cos 2 A + (cos 2 A)2 25. (2)
2 2
= cos A + sin A = 1 45°
20. (3) Since both less than and more than ogive intersect at C B
13 6 m
æ Nö
In DABC,
(40, 30) = çè Median, ÷ø
2 AB AB
Þ Median = 40. tan 45° = Þ1=
BC 13 6
21. (2)
\ AB = 13 6m
a/2
BC 1 13 6
And cos 45° = Þ =
AC 2 AC
a/2
Þ AC = 13 6 ´ 2 = 26 3
Total height of tree
Since diameter of circle = a units
= AB + AC = 13 6 + 26 3 = 13 3(2 + 2) m
a
Þ radius = 3
2 26. (3) Q sin q + cos q =
So, side of inner square = a. 2
\ Area of shaded region Squaring both side
9
1 a2 a2 (sin q + cos q)2 =
= a2 - 4 ´ ´p´ = (4 - p ) 4
4 4 4 9
22. (4) Since equations has infinitely many solutions. Þ sin 2 q + cos2 q + 2 sin q × cos q =
4
a b c 9 5
\ 1 = 1 = 1 Þ 1 + 2 sin q × cos q = Þ sin q × cos q =
a2 b2 c2 4 8
P 3 -( P - 3)
27. (1) Let first term of an A.P. be a and common difference be d.
Þ = = Þ P = 6. A.T.Q.
12 P -P
a3 + a8 = 7
23. (2) Q a10 = a + 9d = 16 ...(i)
Þ a + 2d + a + 7 d = 7
and a16 = a + 15d = 10 ...(ii) Þ 2a + 9d = 7 ...(i)
On solving (i) and (ii), we get And, a7 + a14 = -3
a = 25 and d = –1
a26 = a + 25d = 25 – 25 = 0. Þ a + 6d + a + 13d = -3
Þ 2 a + 19 d = -3 ...(ii)
24. (2) A
45°
30°
_ 2 a + _ 9 d = _ 7
10d = -10
80 m Þ d = -1 Þ a = 8
\ a10 = a + 9d = 8 - 9 = -1.
30° 45°
B 28. (1) Let number of blue balls be x.
D C
Q Number of red balls be 5.
In DABC, \ Total number of balls = 5 + x
AB 80 A.T.Q.
tan 45° = Þ1= P (Blue) = 2P (Red)
BC BC
x 5
Þ BC = 80 m Þ = 2´
5+ x 5+ x
In DABD,
\ x = 10
AB 1 80 Total balls = 10 + 5 = 15.
tan 30° = Þ =
BD 3 BD 29. (3) a 2 + b2 = (a + b)2 - 2ab = (2 3)2 - 2 ´ 3
Þ BD = 80 3 = 12 – 6 = 6.

\ CD = 80 3 - 80 = 80( 3 - 1)m. \ a 4 + b4 = (a 2 + b 2 )2 - 2a 2 b2 = (6) 2 - 2 ´ (3) 2


= 36 – 18 = 18.
Hints & Solutions S-89

30. (4) Here PQ = QR = RS = PS = 7 3 cm Here, n(V) = 60 and n(C) = 40


\ n(V È C ) = 100 - 10 = 90
\ Area of square = (side)2 = (7 3)2 = 147 cm 2 Now, n(V Ç C ) = 60 + 40 - 90 = 10
1 2 \ n (Only C) = 40 – 10 = 30.
Area of sector RQAS = pr A B
4 28°
38. (3)
1 22
= ´ ´7 3´7 3 E
4 7
= 115.5 cm2 y
\ Area of shaded portion = 147 – 115.5 = 31.5 cm2.
31. (3) Let denominator of fraction is x 40°
C D
then numerator of fraction is x – 3. Given that,
x-3 AB || CD Þ ÐEDC = ÐA = 28° (Alternate angles)
\ Fraction =
x In DCED,
New numerator = x – 3 + 4 = x + 1 y + 40° + 28° = 180° (Angle sum properties)
New denominator = x – 2
A.T.Q. \ y = 112°.
x + 1 = 2( x - 2) Þ x = 5 39. (1) Since, total surface area of cube = 6a2 = 96
Þ a=4
2 \ Volume of cube = a3 = 64 cm3.
\ Fraction = . 40. (3) Given word : EDUCATION
5
32. (4) Let ratio be x Vowels = E = {E, U, A, I, O}
\ Angles of triangle are 3x, 4x and 5x. n(E) = 5
\ 3x + 4 x + 5x = 180° (Angle sum property) 5
\ P( E ) =
Þ x = 15° 9
\ Greatest angle = 5 x = 5 ´ 15 = 75°. A
33. (1) Since area of each face of the cube = l2 41. (4)
2
Þ 144 = l Þ l = 12 cm
Total surface of two cuboid 5m E
= Area of cube + 2 × area of one face
100 cm
= 6l 2 + 2l 2 = 8l 2 = 8 ´ 144 = 1152 cm 2 .
34. (2) Q cosecA = 2 Þ A = 30° B y C x ? D
cos A cos 30° BC = Distance walked by girl = speed × time y = 1.9 × 4
\ tan A + = tan 30° + y = 7.6 m
1 + sin A 1 + sin 30°
DABD ~ DECD (AA similarity)
1 3/2 1 3 2
= + = + ´ AB BD
3 1+ 1 3 2 3 =
2 EC CD
1 1 2 5 7.6 + x
= + = Þ = Þ 5 x = 7.6 + x
3 3 3 1 x
35. (1) Ascending order of given data : Þ 4 x = 7.6
2, 5, 6, 7, 8, 9, 10, 14, 15, 17, 18, 19, 24, 27, 28 Þ x = 1.9 m = 190 cm
Number of observations (N) = 15 (odd)
th th 42. (3)
æ N + 1ö æ 15+1ö
\ Median = ç ÷ Obs.= ç Obs.
è 2 ø è 2 ÷ø
= 8th Obs. = 14. h = 21 cm
36. (2) Here a = 3 3, b = 7, c = 3
\ Discriminant (D) = b2 - 4ac = (7)2 - 4 × 3 3 × 3
= 49 – 36 = 13.
Total = 100 Circumference of base of cone = 2pr = 220
37. (4)
60 40 22
? Þ 2 ´ ´ r = 220 Þ r = 35
7
\ Volume of cone
V C
1 22
= ´ ´ 35 ´ 35 ´ 21 = 26,950 cm 3
10 3 7
S-90 Target NTSE
43. (1) Q ax =
b ...(i) 121 æ x ö 100 x
by = c ...(ii) Þ1= ç1 - ÷ Þ = 1-
cz = a ...(iii) 100 è 100 ø 121 100
putting the value of b from eqn.(i) to eqn. (ii), we get x 100 x 21
(ax)y = c Þ axy = c ...(iv) Þ = 1- Þ =
Putting the value of c from eqn (iv) to eqn (iii) 100 121 100 121
(axy)z = a Þ axyz = a1 21
Þ xyz = 1 Þx= ´100 = 17.355 = 17.36% approx.
121
3 -1 48. (1) x +1 - x -1 =1
44. (1) Given that, = a +b 3
3 +1
Rationalise the demonimator, we get Þ x + 1 =1+ x -1
On squaring both side, we get
3 -1 3 -1
Þ ´ =a+b 3
( )
2
x + 1 = 1 + x -1
3 +1 3 -1
( 3 - 1) 2 Þ x + 1 = 1 + x -1 + 2 x -1
Þ =a +b 3
( 3)2 - 12 Þ 1 = 2 x -1
3 +1- 2 3 1
Þ = a +b 3 Þ x -1 =
3 -1 2
On squaring both side, we get
4 2 3
Þ - = a +b 3 1
2 2 Þ x -1 =
4
Þ 2- 3 =a+b 3
On comparing both side, we get 1 5
a = 2, b = –1 Þ x = +1 =
4 4
45. (1) In a leap year, total 366 days in which 52 weeks and and
2 day remaining 49. (1) Given that 3 - 2.3x2 + x + 2 + 32( x + 2) = 0
2 x 2

\ Possible outcomes = (Sun, Mon), (Mon, Tues),


Þ (32 x ) - 2.3 x .3 x + 2 + ( 3( x + 2) ) = 0
2 2 2
(Tues, Wed), (Wed, Thurs), (Thurs, fri), (Fri, Sat), (Sat, Sun)
Favourable outcomes = (Sun, Mon), (Sat, Sun)
Þ [3 x - 3 x + 2 ]2 = 0 éëQ ( a - b ) = a - 2 ab + b ùû
2 2 2 2
Probability of getting 53 Sundays
2
No. of favourable outcomes 2
= Þ 3x = 3x + 2
=
Total no. of outcomes 7 Þ x2 = x + 2
Þ x2 = x + 2 (By comparing the powers)
1
46. (3) sin q cos q = ´ 2 sin q cos q Þ x2 – x – 2 = 0
2 Þ x2 – 2x + x – 2 = 0
1 Þ x (x – 2) + 1 (x – 2) = 0 Þ x(x – 2) (x + 1) = 0
= sin 2q [Q sin 2q = 2sin q.cos q] Þ x = 2, –1
2 Þ x Î {–1, 2}
We have, –1 £ sin 2q £ 1
\ Min. value of sin 2q is –1 50. (4) Q 3x + y = 81 = 34
1 1 Þx+y=4 ...(i)
so, min. value of sin q cos q = ´ -1 = - and 81x–y = 3
2 2 Þ 34(x–y) = 34
47. (2) Let radius and height of right circular cylinder be r and
h respectively 1
Þ x- y = ...(ii)
\ volume of cylinder = pr2h 4
When radius is increased by 10% from (i) & (ii)
æ 10 ö 11r 17 15
New radius = r ç1 + ÷= so, x = and y =
è 100 ø 10 8 8
Let the decreased in height is x%, 51. (1) Volume of larger cube = Volume of 125 cubes
a3 = 125 b3
æ x ö Þ a = 5b
\ New height = h ç1 - ÷
è 100 ø \ S.A. of larger cube = 6a2 = 6 (5b)2 = 150 b2
A.T.Q
Original volume = New volume
11r ö 2 x ö 125 cubes
pr 2 h = p æç æ
÷ ´ h ç1 - ÷
è 10 ø è 100 ø Length of cuboid = 125b
121 öæ x ö Breadth = b
Þ pr 2 h = pr 2 h æç ÷ç 1 - ÷ Height = b
è 100 øè 100 ø
Hints & Solutions S-91

\ S.A. of 125 smaller cubes forming a cuboid 2a


= 2 [125 × b + 125b × b + b × b] = 502b2 \ side of sq1 = = 2a
Þ % of increase in S.A. 2
502b2 - 150b2 side of sq1 = (radius of c2) × 2
= ´ 100 a
150b2 \ radius of c 2 =
352b 2 2
= ´100
150b2 a 2 a
\ side of sq2 = ´ = ´2 = a
704 2 2 2 2
= = 234 %
3 3 56. (2) a cos q - b sin q = c & a sin q + b cos q = p
3 5 7 19 On squaring and adding both equations
52. (1) + + + .... +
1.4 4.9 9.16 81.100
4 - 1 9 - 4 16 - 9 100 - 81
2 2
( 2 2 2 2
) 2
Þ a cos q + sin q + b sin q + cos q = c + p (
2
)
= + + + .... +
1.4 4.9 9.16 81.100 Þ a2
+ = + b2 c2 p2
1 1 1 1 1 1 1 ö Þ a2 + b2 – c2 = p2
= æç1 - ö÷ + æç - ö÷ + æç - ö÷ .... + æç - ÷
è 4 ø è 4 9 ø è 9 16 ø è 81 100 ø Þ p = ± a 2 + b2 - c 2
1 99
= 1- = 57. (3) Q x2 – 3x + 2 is factor
100 100 \ (x –1) (x –2) are also factors.
53. (4) Given that sinq + cosq = x
\ By remainder theorem,
1 1
2 çæ sin q + cos q ÷ö = x (1)4 – p(1)2 + q = 0
è 2 2 ø Þ 1–p=q ... (1)
p p
= 2 æç sin q.cos + cos q.sin ö÷ = x and 24 – p(2)2 + q = 0
è 4 4ø Þ 16 – 4p + q = 0 ... (2)
æ pö From (1) and (2)
Þ 2 sin ç q + ÷ = x p = 5, q = 4.
è 4ø
58. (1) Let common difference of AP = d
[Q sin A.cos B + cos A.sin B = sin( A + B)]
As, 0 < q < 90° 1 é1 1 ù 1
\ = ê - ú´
Þ 1< x £ 2 x1 x2 ë x1 x2 û d
54. (1) Let the ratio is k : 1, using section formula, we get é1
1 1ù 1
3k + 2 7k - 4 ö = ê - ú´
x= ; y = æç ÷ x2 x3 ë x2 x3 û d
k +1 è k +1 ø
1 é 1 1ù 1
=ê - ú´
K xn -1 xn ë xn -1 xn û d
1
A B 1 1 1
(2, –4) P (3, 7) Þ + +×××××× +
(x, y) x1x2 x2 x3 xn-1xn
Since P (x, y) lies on 9x – 3y – 14 = 0 1é1 1 1 1ù
3k + 2 ö æ 7 k - 4 ö = ê - + ×××××× ú
\ 9 æç ÷ - 3ç ÷ - 14 = 0 d ë x1 x2 x2 xn û
è k +1 ø è k +1 ø
Þ 27k + 18 – 21k + 12 – 14k – 14 = 0 1é1 1 ù
Þ 16 = 8k Þ Sum = d ê x - x ú
ë 1 nû
Þk=2
Ratio = 2 : 1 1 é xn - x1 ù
Þ Sum = d ê x .x ú
ë 1 n û
55. (4) C1 1 é x1 + ( n - 1) d - x1 ù
Þ Sum = d ê x1.xn
ú
ë û
( n - 1)
Þ Sum =
x1.xn
Sq2 1
59. (2) Consider : b-a
C2 1+ x + x c -a
Sq1 1
=
radius of c1 = a x a - a + xb - a + x c - a
S-92 Target NTSE

1 For sum to become 12 times at same time


=
a b c
x x x P ´ R ´ 100
+ + Þ 11P =
a a a 100
x x x
xa Þ R = 11%
= 64. (2) Q p, q, r are roots
x a + xb + xc
Similarly on solving other two parts we get \p+q+r=–5
pq + qr + rq = 16
xa xb xc
+ + pqr = – 48
xa + xb + xc xa + xb + xc xa + xb + xc \ p(qr + q + r) + qr
a b c
x +x +x = pqr + (pq + pr + qr)
= =1
x a + xb + xc = – 48 + 16
= – 32
sin 2 r q
(2) Q p cos 2 q =
r
60. 65. (2) P(Hindi) = 0.6
2 r sin q P(English) = 0.4
Converting whole series in terms of cos
13p p 11p 3p 9p 5p P(English and Hindi) = P ( E Ç H) = 0.2
sin = sin ; sin = sin ; sin = sin
14 4 14 4 14 14 \ P (English or Hindi) = P ( H ) + P ( E ) - P ( E Ç H )
2
æ p 3p 5p ö = 0.6 + 0.4 – 0.2
\ S = ç sin × sin × sin ×1 ÷
è 14 14 14 ø
P ( E È H ) = .8
p 6p 3p 4p 5p 2p
also, sin = cos ,sin = cos ;sin = cos \ Probabily of people who does not read neither
14 14 14 14 14 14
2 Hindi = 1 – 0.8
æ 2p 4p 6p ö
\ S = ç cos × cos × cos ÷ 1
è 14 14 14 ø = 0.2 =
5
6p 8p
Q cos = - cos 66. (2) Q Centroid divides the line joining
14 14
2 circumcentre and orthocentre in 2 : 1 ratio
æ 2p 4p 8p ö 2:1
\ S = ç - cos × cos × cos ÷
è 14 14 14 ø H O
G
Þ S = ( - cos q × cos 2q × cos 4q ) 2 (x,y) (2,4) (1,7)

2 2+ x 14 + y
æ 16p ö \ = 2 and =4
ç sin 14 ÷ 3 3
1
Þ S=ç ÷ = Þ x = 4 and y = – 2
2p ÷ 64
çç 2 sin
3
÷ \ H (x, y) = (4, –2)
è 14 ø 67. (2) p = 39 + 312 + 315 + 3n
61. (4) Sum of 25 observations = 25 × 36 = 900
Þ p = 39 (1 + 33 + 36 + 3n–4)
Sum of first 13 observations = 13 × 32 = 416 ... (1)
( ) ( )
æ 3 6 2 ö
Sum of last 13 observations = 13 × 39 = 507 ... (2) 2
If we have ç 1 + 3 + 3 3 (1) + 3 3 (1) ÷
2 2
S26 Þ sum of 25 observations + 13th observation è ø
Þ 900 + 13th observation = 416 + 507 then it is of the perfect cube form
Þ 13th observation = 23
(1 + 32 ) = (1 + 36 + 33 + 35 )
3
62. (3) Container which can measure exactly the two
containers \ For p, if we keep n = 14,
= HCF of (volume 1, volume 2) We get p as perfect cube.

( )
3
= HCF (240, 112)
i.e. p = éê33 1 + 32 ùú
= 16 litres ë û
= 16000 cm3 1
63. (4) Sum becomes 6 times, at 5 % per annum rate. 68. (2) x + = 5
x
\ S I = 5 × Principle 2x 2
For 2 =
P ´ 5´ T 3x - 5 x + 3 3 x + 3 - 5
Þ 5P =
100 x
Þ T = 100 2 2 2 1
= = = =
æ 1 ö 3 ´ 5 – 5 10 5
3ç x + - 5 ÷
è x ø
Hints & Solutions S-93

69. (4) Q log 4 7 = x 74. (2) A

1 1 2
\ log 7 16 = = = Q
E
F
log16 7 1 × log 7 log 4 7
4
2 D
2
= B C
x P
70. (4)
A
Q AB||EP and BC||QF
\ ÐABC = ÐQEP (opposite angles of parallelogram)
F E \ ÐABC + ÐDEF = ÐQEP + ÐDEF
60º
= 180º
B C 75. (3) In DABC, 40º + 2a + 2b = 180º
P D A
BFED is a parallelogram
Þ a + b = 70º 40º
1
Q FE = BD = BC In DBOC, ÐBOC + a + b = 180º
2
Þ ÐBOC = 110º = x
\ ÐFBP = ÐFED = 50° D x E

Q F is mid point of hypoteomuse AB. α O


β

\ FP = BF = AF α β
B C
\ ÐFBP = ÐFPB = 50° 76. (1) A

\ ÐFPC = 180° – 50° = 130°


71. (1) Q x < 1 and y < –1 h

\ let x = 0.5 and y = – 2 60º 30º


\ (x –1, y –3) = (0.5–1, –2 –3) B C
D
10
= (– 0.5, –5)
= Third quadrant h
BC = ( Q tan 60º = 3 )
3
ax 1
72. (3) = bc Þ a x = abc ... (1) also BD = h 3 ( Q tan 30º = )
a 3
by h
= ac Þ b y = abc ... (2) h 3 – 3 = 10
b
cz Þ h = 5 3 m.
= ab Þ c3 = abc ... (3) 77. (3) Sn = 3n + 2n2
c
(1) × (2) × (3) S1= T1 = 3 + 2 = 5 ...(1)
S2 = T2+T1 = 3(2) + 2(2)2
Þ a x × b y × c z = ( abc )3
= 14 ...(2)
Þ x = 3, y = 3, z = 3
From eqn. (1) and (2)
1 1 1 1 T1 = 5, T2 = 9
\ + + = ´3 = 1
x y z 3 For common difference = T2 – T1= 9 – 5 = 4
78. (4) y
73. (3) a x = b y+ z
x–y =2
y+ z
(0, 3)
Þ a =b x
Taking log on both sides, with base b (2, 0)
x
y+ z æ3 ö
ç k ,0 ÷ kx + y = 3
Þ è ø
log b a = log x b x

y+z
Þ logb a =
x For intersection in 1st quadrant
log a y + z 3 3
Þ = > 2 (Q has to be ahead of (2,0))
log b x k k
S-94 Target NTSE

3 84. (4) C
k > O (Q will be positive) and R ¹ 0
3 k
Q O< k<
2
79. (1) In chess board total 32 Black and 32 White squares are
there. y
Total possible solutions = 64 C3
and favourible solutions = (2 Black + 1 White)
or
(2 White + 1 Black) B E D A
32 32
= C2 ´ C1 x x x
32 AC2 - EC 2
C2 ´ 32C1 ´ 2 16 For
\ P(E) = = DC2 - BC2
64 21
C3
éë y 2 + (3 x )2 ùû - éë y 2 + x 2 ùû
5 7 = é 2
3 2 æ5 3ö æ7 3ö ë y + (2 x ) ùû - éë y ùû
2 2

80. (2) = ç ´ ÷-ç ´ ÷


3 3 è3 2ø è2 4ø
4 2 y 2 + 9 x2 - y 2 - x 2 8 x2
= = 2 =2
5 21 -1 y 2 + 4 x2 - y 2 4x
- = =
2 8 8 æ y2 - y1 ö 3+ 2 3 3+ 3
81. (4) A: slope of a line is, obtained by ç ÷ 85. (3) ´
è x2 - x1 ø 3- 3 3+ 3
formula
B: slope of a line parallel to y–axis is infinite (3 + 2 3)(3 + 3)
=
C: The tangent ratio of an angle made by line and 9-3
positive X axis gives the slope of a line. 9 + 6 3 + 3 3 + 6 15 + 9 3
D: Slope of line which makes acute angle lies = =
6 6
between (0, ¥ ) and slope of line which makes 15 9
= + 3 [Q a, b are rational number]
obtuse angle lies between (– ¥ , 0) 6 6
Hence all statements given are wrong. 5 3
= + 3
P Q R 2 2
82. (4) 5 3
a+ 3b= + 3
2 2
5 3
A 5 B 13 C 17 D a= , b = Þ a +b =2
2 2
Q Angles in semi circle is 90º 86. (2) 2x2 + px + 8 = 0
Q ÐAPB = ÐBQC = ÐCRD = 90° D=0 [Q roots are equal]
Q By using Pythagorean tripletes p2 – 4 × 2 × 8 = 0
In DAPB, (3, 4, 5) triplet p2 = 8 × 8 Þ p = 8
In DBQC, (5, 12, 13) triplet p(x + x) + k = 0
2

In DCRD, (8, 15, 17) triplet Þ 8x2 + 8x + k = 0 [ Q p = 8]


\ (AP + PB) + (BQ + QC) + (LR + RD) Again D= 0
= (3 + 4) + (5 + 12) + (8 + 15) Þ 64 – 4 × 8 × k = 0 Þ 64 – 34k = 0
= 47 Units 64
Þ k= 32 = 2.
83. (3) Q
87. (4) x – p(x + 1) – k = 0
2

M x2 – px – p – k = 0
O
x2 – px – (p + k) = 0 ...(1)
60º
R Since, a and b are zeroes of polynomial.
T Then, a + b = p [Q equation (1) is quadratic equation]
P
ab = – (p + k)
By Alternate Segment theorem,
Now (a + 1) (b + 1) = 6
We know that ÐQPT = ÐPMQ = 60º Þ ab+ (b + a) + 1 = 6
Q PMQR is cyclic quadrilateral Þ – (p + k) + p + 1 = 6
\ ÐPMQ + ÐPRQ = 180º Þ –k+1 = 6
Þ ÐPRQ = 120º \ k = – 5.
Hints & Solutions S-95

88. (3) 6 = 3 , 6 = 216, 6 = 1296, , ...


2 6 3 4
é 1 ù
2
52 = 25, 53 = 125, 54 = 625, ... = cos A ê 2 2 ú
cos 2 B
We notice for all powers of 6 and 5 the units digit is 6 and 5 ë cos B sin B û
respectively. cos 2 A
\ Unit digit of 611 – 510 = 6 – 5 = 1. ==n 2
sin 2 B
89. (3) DPQS 93. (4) Let ABC and PQR be two triangles with height BD and
P
PS respectively.
a 20°
2
Now, ar (ABC) = æç BD ö÷
40+x
T
O ar (PQR) è QS ø
2
x æ 4 ö 16
50° =ç ÷ =
Q 40° R è9ø 81
S
ÐPQS = ÐQPS B Q
[Q Angles opposite to equal sides]
\ ÐQPS = 40 + x
Now in DPQT,
40 + x + a + x = 90
60 + 2x = 90
Þ x = 15°.
A C P R
1 1 D S
90. (3) 20, 19 , 18 , ... 94. (3) (10 – x) + y = 122
2 2
...(1)
4 2
-3 x2 + y2 = 102 ...(2)
Þ a = 20, d = From equation (1) and (2) we get
4
æ -3 ö x = 2.8 cm. ; y = 9.6 cm.
Tn = 20 + (n – 1) ç ÷ <0 BC = 2y = 19.2 cm.
è 4 ø
3 A
= 20 + (1 – n) < 0
4
or 80 + 3 – 3n < 0 12 12
83 10 – x
\ n > Þ n = 28 y
3
B C
91. (2) C y

10 x
10

12 m O

q 90 – q 95. (4) p + q = – r [Q p + q + r = 0]
A B
squaring both sides
16 x r × (p2 + q2 + 2pq) = (r2) × r
[multiply by r on both sides]
12
tan q =
16 ( )
r p 2 + q 2 + 2pqr = r 3 ...(i)
12
cot q = [Q tan (90 – q) = cot q] Similarly
x
x = 12 tan q ( )
p q 2 + r 2 + 2pqr = p3 ...(ii)
12
= 12 × = 9 Þ x = 9 m. &
16

92. (2) m =
cos A
;n =
cos A ( )
q p 2 + r 2 + 2pqr = q 3 ...(iii)
cos B sin B
From equation (i) & (ii) & (iii)
æ cos 2 A cos2 A ö 2
(m + n )cos B = çç
2 2 2
2
+ 2 ÷÷ cos B p2 ( q + r ) + q 2 ( p + r ) + r 2 ( p + q ) + 6pqr = p3 + q3 + r3
è cos B sin B ø
...(iv)
S-96 Target NTSE

Also if p + q + r = 0 100. (2) Sum of numbers possible are 2, 3, 4, ...., 12


Perfect square numbers are 4, 9
then p3 + q 3 + r 3 = 3pqr ...(v) Possible cases for 4 = (1, 3), (3, 1), (2, 2)
From (iv) and (v) Possible cases for 9 = (3, 6), (6, 3), (4, 5), (5, 4)
Favourable outcomes = 7
p2 (q + r) + q2 (p + r) + r2 (p +q) = – 3pqr
Total number of outcomes = 6 × 6 = 36
2p 2 ( q + r ) + 2q 2 ( p + r ) + 2r 2 ( p + q ) 7
Þ =–6 Hence probabilitiy =
pqr 36
101. (3) A
96. (3)

h
2.5 b a
B C D
Since, BC = h cot b
Also BD = h cot a
2.5
Radius of inner circle (r 1) = \ Distance between objects = hcota – hcotb
2 = h (cota – cotb)
Diameter of outercircle = diagonal of square
102. (3) By distance formula
2.5 ´ 2 (4 – 1)2 + (q – 0)2 = 52
\ Radius of outer circle (r 2) =
2 32 + q2 = 52 Þ q2 = 16
pr22 Þq=± 4
Hence, ratio of areas = 103. (3)
pr12 A
2 a a
æ 2.5 ´ 2 ö 2 60° 2
pç ÷
è 2 ø = 2 = 2 :1 a
= 2 2
æ 2.5 ö 1 60° 60°
pç ÷ B a a C
è 2 ø 2 2
3 2
1 3 +1 3 +1 Since, area of equilateral triangle = a
97. (1) x = ´ = [Rationalizing] 4
3 –1 3 +1 2
3 2
2x – 1 = 3 \ a = 49 3
4
Squaring on both sides Þ a = 14 cm
4x2 – 4x + 1 = 3 Þ 4x2 – 4x – 2 = 0 Radius of circles = 7 cm
2x2 – 2x – 1 = 0 60° 22 1
Since, the given expression is, Area of each sector = pr2 × = × 72 ×
360° 7 6
4x3 + 2x2 – 8x – 3
77
The above expression can also be written as: =
4x3 – 4x2 – 2x + 6x2 – 6x – 3 3
= 2x (2x2 – 2x – 1) + 3 (2x2 – 2x – 1) 77
Area of all 3 circular sectors = 3 × = 77
= 2x × 0 + 3 × 0 = 0 [ Q 2x2 – 2x – 1 = 0] 3
98. (1) Distance covered in 1 revolution = 2pr
Shaded area = 49 3 – 77
Let number of revolutions be k
= 49 × 1.73 – 77 = 7.77 cm2
Distance covered in k revolutions = 2prk
104. (2) Q A + B = 90°
According to questions
2prk = 100 × 2r tan A × tan B + tan A × cot B sin 2 B
100 \ -
k= sin A.sec B cos2 A
p
99. (2) a + (p – 1) d = q ...(1) cot B × tan B + cot Bcot B cos2 A éêQ B = 90°- A ùú
= -
a + (q – 1) d = p ...(2) sin A × cosecA cos2 A êë\ sin B = cos Aúû
From eqn (1) & (2)
(p – q) d = q – p Þ d = – 1 1 + cot 2 B
= - 1 = cot2 B
Now eqn (1) becomes 1
a + (p – 1) (–1) = q
y 4 - x4 y3
Þ a = p + q –1 -
\ mth them is a + (m – 1) d = (p + q – 1) + (–1) (m – 1) 105. (3) x(x + y) x
=p+q–m y 2 - xy + x 2
Hints & Solutions S-97

(y2 + x 2 )(y- x) - y3 When 9999 is divided by 315, it leaves remainder 234


= hence 9999 – 234 = 9765 which is exactly divisible by
x(y2 - xy+ x 2 )
315 which means it is also divisible by 3, 5, 7, 9
- x ( y 2 - xy+ x 2 ) exactly but it should leave remainder 1, 3, 5, 7 when
= =–1
x( y 2 - xy+ x 2 ) divided by 3, 5, 7, 9 difference between remainder = 2
x 2 - bx m - 1 \ required number = 9765 – 2 = 9763
106. (1) = 111. (2) According to question
ax - c m + 1
In standard form, given equation is (m + 1)x2 – x(bm + b + ef gh ef + gh
=
ma – a) + cm – c = 0 100 2
Since roots are equal in magnitude but opposite From the given options 4950 satisfies the condition
in signs
4950 49 + 50
Þ Sum of zeros = 0 =
or bm + b + ma – a = 0 100 2
99
a -b 49.5 =
Þ m= 2
a+b 49.5 = 49.5
107. (2) 10a + b = (a + b) 4 + 3
112. (1) x2 + xy + x = 12 ...(1)
10a + b = 3ab + 5
6a = 3b + 3 y + xy + y = 18.
2
..(2)
æ b + 1ö Adding equations (1) and (2) we get
5 ´ (b+ 1) + b = 3 ´ ç ´b+5
è 2 ÷ø x2 + y2 + 2xy + x + y = 30
3 3 Þ (x + y) (x + y + 1) = 30
5b + 5 + b = b2 + b + 5
2 2 Let x+ y= t
3 2 9 Þ t (t + 1) = 30
b - b=0
2 2 Þ t2 + t – 30 = 0
3b é 1ù Þ (t – 5) (t + 6) = 0
(b - 3) = 0 ê b ¹ 0 as a ¹ ú
2 ë 2û Þ t = 5 or t = – 6
b= 3
Þ x + y = 5 or x + y = – 6
a= 2
Number is 23. Odd prime. 113. (2) 217x + 131y = 913
131x + 217y = 827
d
108. (1) a + b + c + d = 125 a + 4 = b - 4 = 4c = =t From eqn (1) and (2),
4
x = 3 and y = 2
t - 4 + t + 4 + (t / 4) + 4t = 125 Þ x+y= 5
25t 1
= 125 t = 20 114. (1) x =
4 1
2-
a = 24, b = 16 1
2-
c = 5, d = 80 2-x
109. (2) Let the three digit number be ‘x’ 1
x= (2 - x)
So when 34369 and 31513 is divided by x it leaves 2-
remainder ‘r’ and quotient p, q respectively 2 (2 - x) - 1
\ 34369 = x . p + r ...(1) 1
x=
31513 = x . q + r ...(2) (2 - x)
2-
3 - 2x
Subtracting equation (2) from equation (1) we get
3 - 2x
x (p – q) = 2856 x=
2 (3 - 2x) - (2 - x)
Þ x (p – q) = 119 × 24
[Q 2856 = 2 × 2 × 2 × 3 × 119] 3 - 2x
x= Þ x2 – 2x + 1 = 0
Since x is three digit number, let x = 119 4 - 3x
So on dividing 34369 and 31513 by 119 remainder = 97 Þ (x – 1)2 = 0
110. (1) Let the greatest four digit number be 9999 Þ x= 1
LCM of 3, 5, 7, 9 = 315
S-98 Target NTSE

115. (1) Since, x1 + x7 + x10 = – 6 1 æ 36 2 ö æ 6h ö æ 216 ö 1


= p r =ç ÷ p r2 h
Þ a + a + 6d + a + 9d = – 6 3 çè 25 ÷ø çè 5 ÷ø è 125 ø 3
[Q x1, x2, x3, ... are in A.P.] æ1 ö
= (1.728) ç pr 2 h ÷ = 1.728V
Þ 3a + 15d = – 6 è3 ø
\ % Increase in Volume
Þ a + 5d = – 2 ...(1)
New volume – old volume
And x3 + x8 + x12 = – 11 =
old volume
Þ a + 2d + a + 7d + a + 11d = – 11
1.728V – V
Þ 3a + 20d = – 11 (2) = × 100 = 72.8%
V
From equation (1) and (2), we get a = 3, d = – 1 120. (None) Given options are wrong
‘– 4’ is a root of equation x2 + px – 4 = 0
Now Þ 16 – 4p – 4 = 0
x3 + x8 + x22 = a + 2d + a + 7d + a + 21d = 3a + 30d Þ 4p = 12 Þ p = 3
Þ x3 + x8 + x22 = – 21 Since x2 + 3x + m = 0 has equal roots
Þ b2 – 4ac = 0 Þ 9 – 4m = 0
[Q a = 3, d = – 1] Þ m = 9/4
116. (2) Since, 2, 5, 8 ... is in A.P. where a1 = 2, d1 = 3 121. (2) (5 + )
24 = x+ y
and 7, 11, 15 ... is also in A.P. where b1 = 7, d2 = 4
Þ 5+2 6 = x+ y
2 + 5 + 8 + ...n.terms 23
\ = Þ
7 + 11 + 15 + ...n.terms 35 5+2 3 2 = x+ y
n/2 [2(2) + (n - 1) 3] 23
( 3) + 2 3 2 + ( 2)
2 2
Þ = Þ = x+ y
n/2 [2(7) + (n - 1) 4] 35
( 3 + 2) = x + y
2
3n + 1 23 Þ
Þ = Þ n = 15
4n + 10 35
Þ 3+ 2= x + y
117. (4) In DABC, D, E and F are the midpoints of side BC, CA Þ x = 3 and y = 2
\ x + y = 5 & xy = 6
and AB respectively.
Statement-1 is false; Statement-II is true.
\ Centroid of DABC = Centroid of DDEF A 122. (3) (cos2 5 + cos2 85) + (cos2 10 + cos2 80)
æ 1 + 2 + 3 1 - 3 + 4ö + .... (cos2 40 + cos250) + cos2 45 = (cos2 5 + sin2 5) + ....
=ç , ÷
è 3 3 ø F E
+ (cos2 40 + sin2 40) + cos2 45
1 1 1
= æç 2, ö÷
2 = (1 + 1+ ... + 1) + = 8 + = 8
è 3ø 2 2 2
B D C 123. (1) x = – a is a root of x2 + px + q & x2 + mx + n
118. (3) pR2 = 1386 Þ – a 2 – pa + q = a 2 – ma + n
R
Þ R = 21 (m – p) a = n – q
r n–q
a=
pr2 = 962.5 m–p
124. (3) LCM = 14 GCD
Þ r = 17.5 [Given]
LCM + GCD = 600
\ Width = R – r = 21 – 17.5 = 3.5 cm [Given]
1 2 Þ 15GCD = 600
119. (4) Volume of cone, V = pr h
3 GCD = 40
æ 20 ö 6 LCM = 14 × 40 = 560
New radius = r + ç r÷ = r LCM × GCD = a.b
è 100 ø 5
[Since product of LCM & GCD of two integers is equal
6
Similary new height = h to the product of the integers]
5 Þ 560 × 40 = 80 × b
2
1 æ 6 ö æ 6h ö
\ New volume of cone = p ç r÷ ç ÷ b = 280
3 è5 ø è 5 ø 125. (3) (P¢ Ç Q¢)¢ = P Ç Q
Hints & Solutions S-99

126. (1) A x2 y2 z2
Now; = = =2 [Given]
by + cz cz + ax ax + by
Þ x2 = 2by + 2cz ; y2 = 2cz + 2ax and z2 = 2ax + 2by
N M \ From equation (1)
cz by ax
k= + +
B C 2ax + 2by + 2cz 2ax + 2by + 2cz 2ax + 2by + 2cz
L
D ALB ~ D CNB cz + by + ax (ax + by + cz) 1
Þ k= = = .
AL BL AB AB . BN 2ax + 2by + 2cz 2(ax + by + cz) 2
Þ = = Þ BL = ...(1)
CN BN BC BC 134. (3) Since,
D BMC ~ D ALC log4[log4{log4(log4 x)}] = 0
BM CM BC BC . CL Þ log4{log4(log4 x)} = 40 = 1
Þ = = Þ CM = ...(2) Þ log4 (log4 x) = 41 = 4
AL CL AC AC
D BMA ~ D CNA Þ log4 x = 44 = 256
Þ x = 4256 = 2512.
BM AM AB AM . AC
Þ = = Þ AN = ...(3) 1 1
CN AN AC AB 135. (4) + = logx (z – y) + logx (z + y)
log z - y x log z - y x
On mulitiplying equation (1), (2) & (3) we get
AL × BL × CM = BN × LC × AM
é 1 ù
127. (2) F A T E êQ log b a = ú
ë log a bû
A – 3! = 6 Q First place is finced]
= logx (z2 – y2) ...(1)
E – 3! = 6 [again first place is finced] Since x2 + y2 = z2
Þ z2 – y2 = x2
F A E T – = 1 [first word for letter F]
\ Equation (1) becomes
\ Rank = 6 + 6 + 1 = 13 logx (z2 – y2) = logx x2 = 2 logx x = 2.
128. (4) Since expression ax2 + bx + c is factorised into two 136. (1) Let f (x) = 2x3 + ax2 + bx + 10
equal real linear factors a (x – p) (x – p) i.e. a (x – p)2 Now since x + 2 is the factor of f (x)
Hence b2 – 4ac = 0 Þ b2 = 4ac. \ f (– 2) = 0
129. (2) The above equation is only valid when (x + 4) ³ 0 Þ 2 (– 2)3 + a (– 2)2 + b (– 2) + 10 = 0
Þ x ³ – 4. Þ 2 a – b =3 ...(1)
130. (1) 3x2 + 4 = 0 Also (2x – 1) is the factor of f (x)
2 -4 æ -4 ö \ f (1/2) = 0
Þx = Þx= ç ÷
3 è 3 ø Þ 2 (1/2)3 + a (1/2)2 + b (1/2) + 10 = 0
which is not possible because square of any quantity Þ a + 2b = – 41 ...(2)
cannot be negative (i.e. always equal or greater than from equation (1) and (2)
zero). a = – 7, b = – 17
Hence the given equation has no real roots. \ a2 + b2 = (– 7)2 + (– 17)2 = 338.
131. (1) 9x + 6x = 2.4x ...(1) 137. (3) a + b= 2c = c + c
Let 3x = p and 2x = q Þ a – c = c – b Þ a – c = – (b – c)
\ equation (1) becomes
p2 + pq = 2q2 Þ p2 + 2 pq – pq – 2q2 = 0 a b a b
\ + =- +
Þ (p – q) (p + 2q) = 0 Þ p – q = 0 a-c b-c b-c b-c
[Q p ¹ – 2q as 3x ¹ – 2 (2x)
Þ x x
3 – 2 =0 Þ 3 =2 Þ x=0x x b - c b - (2c - b) 2(b - c)
= = = = 2.
132. (4) f (x) = 2x3 – 3x + 4 b-c b-c (b - c)
\ f (– x) = – 2x3 + 3x + 4 t
f (x) + f (– x) = 4 + 4 = 8. æ r ö
138. (1) CI = p ç 1 + ÷ -p
è 100 ø
c b a
133. (2) Let k = + + 2
2c + z 2b + y 2a + x æ 8 ö
Þ CI = 7500 ç 1 + ÷ - 7500
è 100 ø
cz by ax Þ CI = 8748 – 7500
Þ k = + + ...(1)
2cz + z 2
2by + y 2
2ax + x 2 Þ CI = Rs. 1248.
S-100 Target NTSE
139. (3) Let cost price = x 143. (2)
\ After increase of 40% P Q
140 ´ x 7x O
selling price = =
100 5
After the discount of 20% S
7x 7x R
selling price = - 20% of We know that OQ × OP = OS × OR
5 5 Þ 8 × 14 = 7 × OR
28x Þ OR = 16 cm.
=
25 \ RS = OR – OS = 16 – 7 = 9.
Since, Gain = 48 144. (3) Since, D ABC ~ D DBA
28x
Þ - x = 48 AB AC AD
25 Þ = Þ BD = ...(1)
BO AD 2
48 Again D ABC ~ D DAC
Þ x= ´ 25 Þ x = Rs. 400.
3
140. (1) Curved surface area of right circular cylinder AB AC
Þ = Þ DC = 2AA ...(2)
= 2 prh AD DC
Also curved surface area of sphere = 4 pr2 From (1) and (2)
\ According to the question, 5 AD
2 prh = 4 pr2 BC = = 5 BD.
2
Þ h = 2r 145. (2) We know that;

mean =
å fi × x i
Now,
Volume of cylinder
=
2
pr h
=
2
pr (2r) 3
= å fi
Volume of sphere 4 pr 3 4 2.
3 3 12 (x + 2) + 15x + 20 (x - 1)
Þ mean = 3x + 1
141. (2) Let l, b, h be the length, breadth and height of a
rectangular parallelopiped respectively. 29 12 (x + 2) + 15x + 20 (x - 1)
Þ =
\ l + b + h = 25 (Given) 2 3x + 1
Also2 (lb + bh + lh) = 264(Given) (whole surface area) Þ x = 3.
Since l + b + h = 25 146. (3) Let the third angle be x
\ (l + b + h)2 = 252 Þ x = 180° – (87° 24¢ 54° + 32° 321¢ 6²)
Þ l2 + b2 + h2 + 2(lb + bh + lh) = 625 Þ x = 180° – (119° 56¢ 0²)
Þ l2 + b2 + h2 + 264 = 625 Þ x = 60° 4¢
Þ l2 + b2 + h2 = 361 = p/3 (approx.)
147. (2) Since, x sin a – y cos a = 0
Now, length of diagonal = l2 + b2 + h 2 Þ x sin a = y cos a ...(1)
Also x sin3 a + y cos3 a = sin a cos a ...(2)
( )
2
l2 + b 2 + h 2 From (1) and (2) we get
\ Area =
Þ y cos a sin2 a + y cos3 a = sin a × cos a
= l2 + b2 + h2 = 361. Þ y cos a (sin2 a + cos2 a) = sin a cos a
142. (2) Þ y = sin a
P S \ x = cos a (from (1))
Q Þ x2 + y2 = sin2 a + cos2 a = 1.
148. (1) Four positive integers be a, b, c, and d respectively.
d
A B then, a + b + c + d = 125. Let a + 4 = b – 4 = 4c = = t
4
t
Þ t – 4 + t + 4 + + 4t = 125
In D PSQ: 4
PS2 = PQ2 + SQ2 25t
= 125 ; t = 20
(Q SQ = PA – BQ = r 1 – r2 and PS = AB) 4
\ AB = PQ + (r1 – r2)2
2 2 a = 20 + 4 = 24
b = 20 – 4 = 16
Þ PQ = AB 2 - (r1 - r1 ) 2 Þ PQ = 169 - 25 20
c= =5
Þ PQ = 12 cm. 4
d = 20 × 4 = 80
Hints & Solutions S-101

149. (3) Required number of squares on a chessboard = 156. (3) If 2a2 – 2ab + b2 + 6a + 9 = 0
8 (8 + 1)( 2 ´ 8 + 1) a2 + 6a + 9 + a2 –2ab + b2 = 0
12+ 22 + 32 + 42 + 52 + 62 + 72 + 82 = (a + 3)2 + (a – b)2 = 0, a = b = –3
6 \ a + b + ab = – 6 + 9 = 3
8 ´9 ´17 157. (2) The sum of all the terms common to the arithmatic
= = 204.
6 progression 1, 3, 5.......1991 and 1, 6, 11,..... 1991
150. (2) Here, area of square ABCD = 4cm2 LCM ( 2, 5) = 10
According to question, 1+ (n –1) 10 = 1991 Þ n = 200
Area of D ADE = Area of DBEC = 1 200
4 \ sum = (1+1991) =100 (1992) =199200.
2
Area of D DEC = = 2
2 1 1æ 1 1ö
A E B 158. (2) x + y + 3x 3 y 3 ç x 3 + y 3 ÷
è ø
F
3
G æ 1 1ö
H x 3 + y3
K Þ ç ÷
D C è ø
2
Area of D DFC = =1 æ 1 1ö
2 x 3 + y3
Area of D DGC = 1/2 So, cube root is ç ÷
è ø
Area of D DHC = 1/4
1 1 159. (2) 0.23 + 0.23
\ Area of D KDC = = cm2 Þ 0.2323 2323...... + 0.2333333
4´2 8
151. (3) According to question, Þ 0.465656565 Þ 0.465
xy = z, yz = x. x z = y
160. (1) Kx 2 - 2x + 1
\ x2y2z2 = xyz
xyz = 1 é x + 2 = 0ù
z2 = 1 \ z = ± 1, x = ± 1, y = ± 1 Þ K(- 2 ) 2 - ( 2 )( - 2 ) + 1 = 0 ê ú
êë x = - 2 úû
\ xy + zy + zx = 3.
152. (1) \ Volume of cylinder = pr2h Þ 2K + 2 + 1 = 0
Total surface area of cylinder = 2prh + 2pr2 Þ 2K + 3 = 0
Here, height of cylinder =1 unit Þ 2K = – 3
Then,
-3
V p r2 1æ r ö 1æ 1 ö Þ K=
= = ç ÷ = ç1 - ÷ 2
S 2p r + 2p r 2 2 è1+ r ø 2 è r +1ø
153. (3) 101 x 2 + 123 x + 123 = 0 161. (3) 3x + 2y =13xy ...(i) ´ 5
4x - 5y = 2xy ...(ii) ´ 2
1 1 a + b -123/101 ____________________
+ = = =1
a b ab 123/101 15x+10y = 65xy
154. (4) If x3 + y = 2249, where x and y are natural numbers and 8x-10y = 4xy
________________
HCF of x and y is not 1.
Here, we put x = 13, 133 = 2197 23x = 69xy
y = 2249 – 2197 = 52 Þ 23x = 69 xy
HCF ( 13, 52) = 13 ¹ 1 Þ x = 3xy
\ x + y = 13 + 52 = 65 1
Þ 1 = 3y Þ y =
155. (1) a = 6 - 11 and b = 6 + 11 3
æ 1ö
ab = ( 6 - 11 )( 6 + )
11 = 25 = 5 In equation (i) ® çè y = ÷ø
3
3x + 2y = 13xy
a 2 + b 2 = 6 - 11 + 6 + 11 = 12
1 1
2
3x + 2 ´ = 13x ´
(a + b) = a 2 + b 2 + 2ab = 12 + 10 = 22 3 3
2 13x
\ a + b = 22 3x + =
3 3
S-102 Target NTSE

2 13x 13x - 9x 4x 1 2
= - 3x = = 167. (3) c - 2c - 9
3 3 3 3 3
2 4x
Þ
3
=
3 1
=
3
(
1 2
)1
c - 6c - 27 = (c 2 - 9 c + 3c - 27)
3
Þ 4x = 2 x=
2 1
1
Þ x=
2 1
= y=
1 = [ c(c - 9) + 3(c- 9)] = 3 (c + 3)(c - 9)
4 2 3
3
h h æ1 ö
162. (1) tan q = , tan(90 - q) = = ç c - 3 ÷ (c + 3)
9 16 è 3 ø
h 168. (2) In DCFE ~ DCDB
tan q = ...(i)
9 EC 4
= ...(1)
h BC 4 + n
cot q = ...(ii)
16
A
9 cm
D
F
h

4c
m
B E C
90°– q q A C = 9 cm
7m 9m CF = 4 cm
Eq. (i) × (ii) In DCDE ~ DCAB

h2 EC 4 + n
Þ =1 = ...(2)
16 ´ 9 BC 9
h2 = 16 × 9 From (1) and (2)
h = 16 ´ 9 = 4 × 3 = 12 mt. 4 4+ n
= Þ (4 + n)2 = 4 × 9
4+ n 9
163. (3) log5 [log 2 (log3 x)] = 0
Þ n2 + 8n + 16 = 36
1 = log2 (log3x) Þ n2 + 8n = 36 – 16 = 20
21 = log3x Þ n2 + 8n – 20
32 = x Þ x=9 Þ n2 + 10n – 2n – 20
164. (2) x = – 6, y = 3 Þ n (n + 10) – 2(n + 10)
\ Angle between them is 90° Þ n–2 Þn=2
y 4 + n = 4 + 2 = 6 cm
3 15
y=3 169. (1) a :b= =
5 25
5 15
a:c= =
7 21
x \ (b – c) : (b + c)
x=–6 O
Þ (25 – 21) : (25 + 21)
Þ 4 : 46
165. (2) 6, 8, 2x – 5, 2x – 1, 15, 17, 20, 22 1
170. (1) mth term of an AP =
2x - 1 + 15 n
= 14
2 1
2x + 14 = 14 × 2 = 28 nth term of an AP =
m
2x = 28 – 14 = 14 mnth term = ?
14 1
x= =7
2 a + (m – 1)d =
n
166. (3) A' = {1, 2, 7, 8}
B' = {2, 4, 6, 8} 1
a + (n – 1) d =
A' – B' = {1, 7} m
Hints & Solutions S-103

1 D (300 + 100)
\ a=d= 176. (1) T= =
mn S 25
1 400
= = 16 seconds
mnth term = 25
mn
177. (3) Let the given observations be x1, x2, ....... x20 and xM
171. (2) According to question
be their mean.
6 3 Given that n = 20 and variance in 5.
m 4n4 ´ m2 n2 ´ m 2n 2 = (mn)k
Þ (mn)2 × (mn)1/3 × (mn)2/3 = (mn)k 1 20
å ( x - xM ) = 5
2
So,
20 i =1
1 2
Þ (mn)2 + 3 + 3 = (mn)k 20
å ( x - xM )
2
= 100.(1)
3 k
Þ (mn) = (mn) i =1
If each observation is multiplied by 2, then the new
\k=3
observations are yi = 2xi, i = 1, 2, .....20.
172. (3) Let total students in a group = x So, xi = yi/2
< 20 = 10% of total students
1 20
20 – 40 = 20% of total students Hence, ym = 20 å yi
40 – 60 = 35% of total students i =1

60 – 80 = 20% of total students 1 20 20


80 – 100 = 30% of total students = (100 – 85) % =15% = å 2 xi Þ ym = 2 ´ 1
20 i =1 å xi
20 i =1
x ´15 This gives us xM = 1/2 yM
Q = 30
100 Substituting the values of xi and xM in (1) we have
\ x = 200 students. 20
æ1 1 ö
2

So, (40 – 60) will have higher no. of students. å çè 2 yi - 2 yM ÷ø = 100


i =1
173. (4) According to question. 20 2
1
One root of quadratic equation = 3 - 2 å 4 ( yi - yM ) = 100
i =1
another will be = 3 + 2 20 2

\ sum of roots = 6 å ( yi - yM ) = 400


i =1
products of roots = 7 Hecne, the variance of the resulting observations is =
Þ x2 – (sum of the roots) x + product of the roots = 0 1/20. 400
Þ x2 – 6x + 7 = 0 = 20. = 22.5
174. (2) According to question 178. (1) According to question,
P ( –1) = (–1 –1) = –2
A
P ( –1) = (1)2 +1 = 2
4 \ R1 + R2 = 0
D
–2 + 2 = 0.
A
4 5 179. (4) Here,

90° C ar.AXY 1
B =
ar . ABC 2
4 5 4 X Y
= 2
AC 4 5 é AX ù 1
Þê
AB ú =2
AC = 20 ë û
AX 1
BC = 8 5 = B C
AB 2
\ Area of DABC = 80 sq.m
2 AX + BX 2
æ3 ö =
4 ç a ÷ - 4a 2 AX 1
è2 ø ´100
175. (1) Required percentage = BX 2
4a 2 1+ =
= 125%. AX 1
S-104 Target NTSE

BX 2 -1 185. (4) Let d1 be the common difference of 1st A.P. and d2 be


= the common difference of 2nd A.P.
AX 1
AX 1 \ y = x + 3d1 = x + 4d 2
\ =
BX 2 –1 Þ 3d1 = 4d 2
= 2 +1 4
Þ d1 = d 2
180. (2) According to question, 3
1 4
Þ + 5+ 2 6 Þ (a 2 - a1) = (b3 - b2 )
5+ 2 6 3
Þ 5 – 2 6 + 5+ 2 6 4
Þ (a 2 - a1) = - (b2 - b3 )
Þ 10 3
a 2 - a1 4
1 + a4 Þ =-
\ 2
= 10 b 2 - b3 3
a
2 B 186. (2) 4x – 3(2x + 3) + 128 = 0
181. (4) According to question, D
Þ (2x)2 – 24.2x + 128 = 0
b a
\AD2 = AB2 –22 Þ y2 – 24y + 128 = 0
AD2 = 42 – CD2 C Þ y2 – 8y – 16y + 128 = 0
AB2 – 22 = 42 – CD2 4 A
Þ y(y – 8) – 16(y – 8) = 0
\ a2 + b2 = 20. Þ (y – 8)(y – 16) = 0
5 é x2 5 k ù Þ y = 8 or y = 16
182. (4) a + b = – êQ + x + = x 2 + b2 + abú
2 êë 2 2 2 úû Þ Now y = 8 Þ 2x = 23 Þ x = 3
k Þ y = 16 Þ 2x = 24 Þ x = 4
ab =
2 \ sum of roots = 3 + 4 = 7
Now, (a + b)2 = a2 + b2 + 2ab 7 3 126 + 39
187. (4) CP = 18 ´ + 13 ´ =
25 10 10 10
Þ = (a2 + b2 + ab) + ab
4 165
= = ` 16.5
25 21 k 10
Þ = + SP = ` 18.15
4 4 2
Þk=2 Gain = ` 1.65
Gain 1.65
183. (1) x2 = x - 2 Þ x = x – 2 % gain = ´ 100 = ´ 100 = 10%
CP 16.5
Þ 0 = – 2, Which is not true
188. (4) Let a and b are two given number and b > a
Hence no solution
184. (2) 2m – 2m – 1 = 4 a+b
then = 2 Þ a + b = 4 ...(i)
æ 2m ö 2
Þ 2m - ç ÷=4 G.M. = a(b + 1) = 2
ç 21 ÷
è ø Þ a (b + 1) = 4
Þ 2 ´ 2m - 2 m = 4 ´ 2 Þ (4 – b) (b + 1) = 4
Þ 2m [2 - 1] = 4 ´ 2 Þ 4b + 4 – b2 – b = 4
m Þ 3b – b2 = 0 Þ b (3 – b) = 0
Þ 2 =8
Qb ¹ 0 , \ b = 3
Þ 2m = 23 \a=1
Þ m=3 2ab 2 ´ 1 ´ 3 3
m 3 Now, H.M. = = =
Þ m = 3 = 27 a+b 1+ 3 2
1
(Stage 1)

Time : 120 Minutes Max. Marks : 100

INSTRUCTIONS FOR CANDIDATES


Read the following instructions carefully before you answer the questions.
1. There are 100 questions in this test all are compulsory. Each question has 4 choices (1), (2), (3), and (4), out of which
ONLY ONE is correct.
2. Answers are to be given on separate answer sheet provided at the end of this paper. You can tear this answer sheet
along the cut marks provided.
3. If you do not know the answer to any question, do not waste time on it and pass on to the next one. Time permitting,
you can come back to the questions, which you have left in the first instance and attempt them.
4. Since the time allotted for this question paper is very limited you should make the best use of it by not spending too
much time on any one question.
5. For all questions 1-100 put a cross mark (×) on the number of the correct alternative on the answer sheet against the
corresponding question number by using a blue ball point pen. Example

Correct Way : Q.No. Alternatives


1. 1 2 3 4

Wrong Way : Q.No. Alternatives


1. 1 2 3 4
1. 1 2 3 4

6. Rough work can be done anywhere in the booklet but not on the answer sheet/loose paper.
7. Every correct answer will be awarded one mark. There is no negative marking.
8. After completion of this test you can evaluate your performance by comparing your answers with answers provided
in solution booklet.
MT-2 Target NTSE
DIRECTIONS (Qs. 1 - 3) : In each of these questions, four terms 11. How many meaningful words can be formed with the letters
are given. While three of them are identical in some EAML ?
way, one is different from the rest. Select the odd one as your (1) 1 (2) 3
answer. (3) 4 (4) 2
1. (1) T (2) Z 12. How many such pairs of letters are there in the word
(3) Q (4) H INSTRUCTION which have as many letters between them
2. (1) Hat (2) Bag in the word as in the English alphabet?
(3) Purse (4) Basket (1) Two (2) One
3. Find the odd one out. (3) Three (4) Four
(1) Grams (2) Litres 13. In a certain code, CONDEMN is written as CNODMEN. How
will TEACHER be written in that code ?
(3) Tonnes (4) Quintals
(1) TEACHER (2) TAEECHR
DIRECTIONS (Qs. 4 & 5) : Find the next term in the given
(3) TCAEER (4) TAECEHR
series in each of the questions below.
14. If Z = 52 and ACT is equal to 48, then BAT will be equal to
4. 5, 7, 9, 11, 13, .... (1) 39 (2) 41
(1) 15 (2) 10 (3) 44 (4) 46
(3) 8 (4) 6 15. If ‘light’ is called ‘morning’, ‘morning’ is called ‘dark’, ‘dark’
5. 125, 80, 45, 20, .... is called ‘night’, ‘night’ is called ‘sunshine’ and ‘sunshine’
(1) 8 (2) 12 is called ‘dusk’, when do we sleep ?
(3) 10 (4) 5 (1) Dusk (2) Dark
6. Find the wrong term in the following series : (3) Night (4) Sunshine
49, 49, 50, 54, 60, 79, 104 16. In a certain code language 'Ka Bi Pu Ya' means 'You are very
(1) 60 (2) 49 intelligent'; 'Ya Lo Ka Wo' means 'They seem very intelligent';
(3) 104 (4) 54 'La Pu Le' means 'You can see' and 'Sun Pun Yun Ya' means
7. Complete the following series: 'how intelligent she is', In that language, which of the
__ ab __ b __ bc __ ca __ following words means 'are'?
(1) cacab (2) abcca (1) Ka (2) Bi
(3) abacb (4) accbb (3) Ya (4) Pu
DIRECTIONS (Qs. 8 & 9) : In each of the following questions, 17. A watch reads 4:30 O’clock. If minute hand points towards
various terms of a series are given with one term missing as the East, in which direction does the hour hand point ?
shown bye (?). Choose the missing term. (1) North-East (2) South-East
(3) North-West (4) North
8. QPO, SRQ, UTS, WVU, (?)
18. At an enquiry office at a railway station, a passenger was
(1) XVZ (2) YXW told that a train for New Delhi has left 15 minutes ago, but
(3) ZYA (4) VWX after every 45 minutes a train leaves for New Delhi. The next
9. YEB, WFD, UHG, SKI, (?) train will leave at 8.30 p.m. At what time was this information
(1) QOL (2) TOL given to the passanger ?
(3) QGL (4) QNL (1) 7.45 pm (2) 8.00 pm
DIRECTION (Q. 10) : Identify the letter-series that follows the (3) 8.15 pm (4) 8.05 pm
given rule. DIRECTIONS (Qs. 19 & 20) : Study the information given below
10. In the following series, the number of letters skipped in carefully to answer these questions:
between the adjacent letters are in ascending order, i.e., 2, 3, On a playing ground, Dinesh, Kunal, Nitin, Atul and Prashant are
4, 5. Which one of the following letter groups does not standing, as described below, facing the North.
obey this rule? (I) Kunal is 40 metres to the right of Atul.
(1) GILPU (2) DFIMR (II) Dinesh is 60 metres to the south of Kunal
(3) CEHLQ (4) HJMPT (III) Nitin is 25 metres to the west of Atul.
(IV) Prashant is 90 metres to the north of Dinesh.

SPACE FOR ROUGH WORK


Mock Test-1 MT-3

19. Who is to the north-east of the person who is to the left of (1) Cousin (2) Daughter-in-law
Kunal? (3) Uncle (4) None of these
(1) Dinesh (2) Nitin 28. A and B are married couple. X and Y are brother. X is the
(3) Atul (4) None of these brother of B How Y is related to B ?
20. If a boy walks from Nitin, meets Atul followed by Kunal, (1) Brother (2) Brother-in-law
Dinesh and Prashant, then how many metres has he walked (3) Son (4) Son-in-law
if he has travelled the straight distance all through?
(1) 155 metres (2) 185 metres DIRECTIONS (Qs. 29 & 30) : In the questions below choose one
(3) 215 metres (4) 245 metres of the figures labelled (1), (2), (3) and (4) which best represents
21. Ashish leaves his house at 20 minutes to seven in the the relationship among the items given in the questions that
morning reaches Kunal’s house in 25 minutes. They finish follow.
their breakfast in another 15 minutes and leave for their
office which takes another 35 minutes. At what time do they
leave Kunal’s house to reach their office?
(1) 7.40 a.m. (2) 7.20 a.m (1) (2)
(3) 7.45 a.m. (4) 8.15 a.m.
22. In a month of 31 days, the third Wednesday falls on the
15th. What will be the last day of that month?
(3) (4)
(1) Fifth Thursday (2) Fifth Wednesday
(3) Fourth Sunday (4) Fifth Friday
23. In a row at a bus stop, A is 7th from the left and B is 9th from
the right. Both of them interchange their positions and thus 29. Mangoes, Apples, Fruits
A becomes 11th from the left. How many people are there in 30. Coffee, Tea, Beverages
that row? DIRECTIONS (Qs. 31 - 34) : Answer these questions based on
(1) 18 (2) 19 the diagram given below.
(3) 20 (4) 21 Christians
24. A, B, C, D, E and F are sitting around a round table. A is
1 Professionals
between E and F, E is opposite D, and C is not in either of the
neighbouring seats of E. Who is opposite to B ? 12 Females
2
(1) F (2) C
(3) D (4) None of these 5 8 9
DIRECTIONS (Qs. 25 & 26) : Read the relationships given 7 10 11
below to answer the questions that follow. 4 6
Archana is elder than Suman. Arti is elder than Archana but 3
younger than Kusum. Kusum is elder than Suman. Suman is Asians
31. Asian Non-Christian females who are Professionals are
younger than Arti and Gita is the eldest.
represented by
25. Who is the youngest ?
(1) 3 (2) 10
(1) Archana (2) Suman
(3) 8 (4) 11
(3) Arti (4) Kusum
32. Asian females who are neither professional nor Christians
26. Age wise, who is in the middle ? are denoted by
(1) Suman (2) Archna (1) 6 (2) 9
(3) Arti (4) Kusum (3) 10 (4) 3
27. Pointing to a photograph, Arun said, she is the mother of 33. Non-Asian professional Christian male are represented by
my brother’s son’s wife’s daughter. How is Arun related to (1) 10 (2) 11
the lady ? (3) 12 (4) 9

SPACE FOR ROUGH WORK


MT-4 Target NTSE
34. The shaded portion depicts 39. Statements :
(1) Asian professional Christian males All chairs are houses.
(2) Non-Asian professional female Christians
Some shoes are houses.
(3) Non-Christian professional females Asians
(4) Non-Asian unprofessional female Christians Conclusions :
35. Which number is on the face opposite to 6 on the dice A : Some chairs are shoes.
whose four positions as shown below ? B : Some shoes are chairs.
6 6 5 1 40. Statements :
2 4 4 4 2 All men are dogs.
3 2 6
All dogs are cats.
(1) 1 (2) 2 Conclusions :
(3) 3 (4) 4
A : All men are cats.
36. How many triangles does the following figure contain?
B : All cats are men.
41. Which of the Answer Figures given below is exactly the
mirror image of the question figure ?
Question Figure

(1) 12 (2) 10
(3) 6 (4) 11
37. The following diagram shows 10 match-sticks forming three
squares. What is the minimum number of matches that must Answer Figures
be removed to make two squares?
(1) 3
(2) 2
(1) (2) (3) (4)
(3) 1 42. Pick up a figure from the Answer Figures to fit into the given
(4) 4 Incomplete Figure.
38. In the same diagram given above, what is the minimum Incomplete Figure
number of extra matchsticks needed so that by adding them
at suitable places, five squares can be formed?
(1) 5 (2) 5
(3) 3 (4) 2
DIRECTIONS (Qs. 39 & 40) : In the questions below, there are
two statements followed by two conclusions A and B. You have to
take the statements to be true even if they seem to be at variance
from commonly known facts and then decide which of the
Answer Figures
conclusions logically follows. Choose :
(1) if only A follows
(2) if only B follows
(3) if both A and B follow
(4) if neither A nor B follows (1) (2) (3) (4)

SPACE FOR ROUGH WORK


Mock Test-1 MT-5

DIRECTIONS (Qs. 43 & 44) : Based on the relationship between 46. Problem Figures
the first two figures in the Problem Figures, choose the suitable
figure from the Answer Figures to fit in the blank space in the
Problem figures. ?
43. Problem Figures
Answer Figures

?
Answer Figures
(1) (2) (3) (4)

47. Problem Figures

(1) (2) (3) (4)


?
44. Problem Figures
Answer Figures
?
Answer Figures

(1) (2) (3) (4)

48. If a square paper is folded as shown in the question figures


and then folded paper is punched. What will be the pattern
(1) (2) (3) (4)
on opening the paper ?
DIRECTIONS (Qs. 45 - 47) : In these questions, the series of Question figures :
diagrams called Problem Figures consists of figures in a
particular sequence. Below the Problem Figures is given a series Paper ® First fold ® Second fold ® Punched
of figures called Answer Figures. You have to pick out the figures
from the Answer Figures which should come at the end of the
Problem Figures in the place with a questions mark. You have to
choose the letter of the figure as the answer figures which should
complete the given series.
Answer figures :
45. Problem Figures

? (1) (2)

Answer Figures

(3) (4)

(1) (2) (3) (4)

SPACE FOR ROUGH WORK


MT-6 Target NTSE
49. A square sheet of paper has been folded and punched as DIRECTIONS (Qs. 54-56): Read the given information carefully
shown in the question figure. You have to figure out from and answer the following questions:
amongst the four answer figures, how it will appear when There are five cars i.e. A, B, C, D, E, in a parking area at some
opened? distance with each other, they all are of different weights. Car B is
Question figure : heavier than only two cars. Second heaviest car is 3m south of car
A. Car D is lighter than atleast two cars. Heaviest car is 4m east of
car C. lightest car is north east of car D. Car D is heavier than car
A. Car C is somewhere in between car D and car E in a straight line.
Car B is south of car D.
54. What is the shortest distance between car A and E?
Answer figures :
(1) 8m (2) 10m
(3) 6m (4) 5m
(1) (2) 55. Which of the following car is 2nd heaviest car?
(1) C (2) A
(3) E (4) Either (1) or (3)
56. What is the direction of car B with respect to car A?
(1) South (2) South west
(3) (4)
(3) North west (4) Cannot be determined
DIRECTIONS (Qs. 57-61): Each of the following questions below
50. Find the correct option for the water images below : consists of a question and two statements numbered I and II given
below it. You have to decide whether the data provided in the
ZEBRA statements are sufficient to answer the question. Read both the
¬ Water surface
? statements and give answer.
(1) (2) (1) if the data in statement I alone are sufficient to answer the
question, while the data in statement II alone are not sufficient
(3) (4)
in answer the question.
DIRECTIONS (Qs 51-53): Study the following information
carefully and answer the questions given below. (2) if the data in statement II alone are sufficient to answer the
There are some persons sitting in a row facing in north direction. question, while the data in statement I alone are not sufficient
Two persons sit between M and C. Four persons sit between C to answer the question.
and Q. R sits second to the left of Q. Only six persons sit to the left (3) if the data in either in statement I alone or in statement II
of Q. As many persons sit between C and M as sit between S and alone are sufficient to answer the question.
C. W sits third from one of extreme ends of the row but not an (4) if the data in both the statements I and II together are not
immediate neighbour of C. sufficient to answer the question.
51. If E sits between Q and S, then what is the position of M 57. What does 'ra' mean in the code language?
with respect to E?
I. 'st qm ra' means 'Look at me' and 'qm ms st rt' means
(1) Fifth to the right (2) Seventh to the right
'don't look at him' in that code language.
(3) Fifth to the left (4) Seventh to the left
52. How many persons can sit in a row? II. 'ka tv ne ra' means 'take me and go' and 'vw wx si ra'
(1) Twelfth (2) Ten means 'either me or you' in that code language.
(3) Fifteen (4) Eleven 58. How is Sarita related to Rajesh?
53. What is the position of W with respect to R? I. Soni, the cousin of Rajesh, is the niece of Sarita.
(1) Fifth to right (2) Second to the left II. Swarna is sister of Sarita who is the wife of Om. Om is
(3) Fifth to the left (4) Seventh to the left the father of Rajesh.

SPACE FOR ROUGH WORK


Mock Test-1 MT-7

59. What is M's rank in the class of 40? 65. Which country is the third biggest in terms of Gross
I. S, who is 9th from the top in the class is above R by 12 Domestic Product (GDP)?
ranks who is below M by 5 ranks. (1) A (2) C
II. N, who is between M and Q, is 15th from the bottom. (3) E (4) F
60. Which train did Amita catch to go to school? 66. By what percent is the Gross Domestic Product (GDP) of
Statements: Country A lesser than that of Country F?
I. Amita missed his usual train of 11.25 a.m. A train comes (1) 20% (2) 50%
in every 5 minutes. (3) 25% (4) 40%
II. Amita did not catch the 11.40 a.m. train or any train 67. What is the ratio of Gross Domestic Product (GDP) of
after that time. Country E to Total of GDPs of countries A, C and F taken
together?
61. P, Q, R, S, T, U are earning different salaries, who among
(1) 3 : 5 (2) 5 : 3
them gets lowest salary?
(3) 9 : 5 (4) 5 : 9
I. P earns more than S and T both but less than Q.
68. If the combined GDP of the six countries has grown by 50%
II. R earns more than U but less than S. in the last decade, then what was their combined annual
DIRECTIONS (Qs. 62-64): Study the following information Gross Domestic Product (in billion USD) before 10 years?
carefully and answer the given questions. (1) 540 (2) 180
In a certain code language, (3) 240 (4) 200
'water earth fixing food' is coded as 'zq ie mn as' 69. A word is represented by only one set of numbers as given
'award no earth fixing' is coded as 'mn bn st ie' in any one of the alternatives. The sets of numbers given in
'award word fixing food' is coded as 'cd as mn bn' the alternatives are represented by two classes of letters as
'water food coaching glass' is coded as 'zq as zx yx' in two matrices given below. The columns and rows of
62. Which of the following is the code for 'no'? Matrix-I are numbered from 0 to 4 and that of Matrix-II are
(1) st (2) bn numbered from 5 to 9. A letter from these matrices can be
(3) ie (4) mn represented first by its row and next by its column. E.g., 'F'
63. Which of the following word is coded as 'bn'? can be represented by 03, 24 etc., and 'S' can be represented
(1) word (2) food by 56, 89 etc. Similarly, you have to identify the set for the
(3) no (4) Award word 'ROAD'.
64. What may be the code of 'word orange'? Matrix-I Matrix-II
(1) st ie (2) cd mn
(3) ie bn (4) cd qw 0 1 2 3 4 5 6 7 8 9
DIRECTIONS (Qs. 65-68) The bar graph GDP for the year 2016 0 L K A F H 5 Q S R U Z
in billion USD of six countries (A, B, C, D, E, F) of a regional 1 K I I C D 6 S R O V T
trade block. Study the diagram and answer the following questions. 2 J E E E F 7 O U V S Z
3 M I A C G 8 V Y T Y S
120 4 M B K C G 9 Y Z O U V
100 (1) 24, 14, 67, 76 (2) 30, 22, 86, 56
100
90 (3) 66, 67, 02, 14 (4) 13, 43, 56, 86
GDP Billion USD

80 70 70. Fact 1: Most stuffed toys are stuffed with beans.


Fact 2: There are stuffed bears and stuffed tigers.
Fact 3: Some chairs are stuffed with beans.
60
50 If the first three statements are facts, which of the following
statements must also be a fact?
40 30 I. Only children's chairs are stuffed with beans.
20 II. All stuffed tigers are stuffed with beans.
20 III. Stuffed monkeys are not stuffed with beans.
(1) I only
0 (2) II only
A B C D E F
(3) II and III only
Country (4) None of the statements is a known fact.

SPACE FOR ROUGH WORK


MT-8 Target NTSE
71. Fact 1: Pictures can tell a story. 75. Statement: "Buy pure and natural milk of company Z"-An
Fact 2: All storybooks have pictures. advertisement in a Newspaper.
Fact 3: Some storybooks have words. Assumptions:
If the first three statements are facts, which of the following I. Artificial milk can be prepared.
statements must also be a fact? II. People do not mind paying more for pure and natural milk
(1) If only assumption I is implicit
I. Pictures can tell a story better than words can.
(2) If only assumption II is implicit
II. The stories in storybooks are very simple. (3) If either I or II is implicit
III. Some storybooks have both words and pictures. (4) If neither I nor II is implicit
(1) I only DIRECTIONS (Qs. 76-78): In the following diagram, three classes
(2) II only of population are represented by three figures. The triangle
(3) III only represents the school teachers, the square represents the married
(4) None of the statements is a known fact. persons and the circle represents the persons living in joint
72. Select a suitable figure from the four alternatives that would families.
complete the figure matrix.

E B C
? D
A

76. Married persons living in joint families but not working as


school teachers are represented by
(1) (2) (3) (4) (1) C (2) F
(1) 1 (2) 2 (3) D (4) A
(3) 3 (4) 4 77. Person who live in joint families, are unmarried and who do
73. Select a suitable figure from the four alternatives that would not work as school teachers are represented by
complete the figure matrix.
(1) C (2) B
(1) 1 (2) 2
(3) 3 (4) 4 (3) E (4) D
78. Married teachers living in joint families are represented by
(1) C (2) B
(3) D (4) A
? DIRECTION (Q. 79): The question given below is based on the
letter series, In series, some letter are missing. Select the correct
alternative. If more that five letter are missing, select the last
five letters of the series.
79 xyzu_yz_v__uv_______
(1) (2) (3) (4) (1) uvxyz (2) vuzyx
(1) 1 (2) 2 (3) uvzyx (4) vuxyz
(3) 3 (4) 4 DIRECTIONS (Qs. 80 to 81): Find the missing number(s) :
74. Statements: "please drop this letter in the letter box". An
officer tells his assistant.
Assumptions:
I. The assistant would follow the instructions. 5 4 6 3 8 3
II. The assistant knows the address where the letter is to 80. 26 ? 29
be sent.
(1) If only assumption I is implicit 6 4 5
(2) If only assumption II is implicit (1) 32 (2) 22
(3) If either I or II is implicit (3) 18 (4) 27
(4) If neither I nor II is implicit

SPACE FOR ROUGH WORK


Mock Test-1 MT-9

(1) America and India (2) Japan and Australia


3 6 (3) America and Australia (4) India and Australia
88. If P $ Q means P is the father of Q, P # Q means P is mother
81. 6 10 2 9 30 3 5 of Q, & P * Q means P is the sister of Q. Then how is Q
related to N if N # L $ P * Q
4 5 6 ? 5 (1) grandson (2) grand daughter
(3) nephew (4) data inadequate
2 89. Soni, who is Dubey's daughter, says to Preeti, "Your mohter
(1) 15 (2) 20 Shyama is the youngest sister of my father, Dubey's Father'
(3) 25 (4) 40 third child is Prabhat"/ How is Prabhat related to Preeti ?
82. Manisha ranked sixteenth from the top and twenty ninth (1) Uncle (2) Father
form the bottom among those who passed an examination. (3) grandmother (4) Father in law
90. I am facing South. I turn right and walk 20 m. Then I turn
Six students did not participate in the competition and five
right again and walk 10 m. Then I turn left and walk 10 m and
failed in it. How many students were there in the class ? then turning right walk 20 m. Then I turn right again and
(1) 40 (2) 44 walk 60 m. In which direction am I from the starting point ?
(3) 50 (4) 55 (1) North (2) Northwest
83. If × means ÷, – means ×, ÷ means + and + means –, then (3) East (4) Northeast
(3 – 15 ÷ 19) × 8 + 6 = ? 91. Raj walked 20 metres towards South. Then he turned to his
(1) 8 (2) 4 left and alked 25 metres. He then turned to his left and walked
(3) 2 (4) –1 20 metres. He again turned to his right and walked 10 metres.
84. If × stand for 'addition'; < for 'subtraction', + stands for At what distance is the form the starting point and in which
direction?
'division', > for 'multiplication', – stands for 'equal to', ÷ for
(1) 35 metres, East (2) 35 metres, North
'greater than and = stands for ' less than', state which of the (3) 40 metres, East (4) 60 metres, East
following is true ? 92. Which of the following diagrams correctly represents the
(1) 3 × 2 < 4 ÷ 16 < 2 + 4 relationship among Tennis fans, Cricket players and
(2) 2 > 2+ 2 = 10 < 4 ÷ 2 Students.
(3) 3 × 4 > 2 – 9 + 3 < 3
(4) 5 × 3 < 7 ÷ 8 + 4 × 1
DIRECTIONS (Qs. 85 to. 86): Read the following information
and answer the questions that follow.
There are 70 clerks working with M/s. Jha Lal Khanna & Co.
chartered accountant, of which 30 are female. (1) (2)
I. 30 clerks are married.
II. 24 clerks are above 25 years of age
III. 19 Married clerks are above 25 years of age ; among
them 7 are males.
IV. 12 males are above 25 years of age
V. 15 males are married. (3) (4)
85. How many unmarried girls are there ? DIRECTIONS (Qs. 93 to 97): Study the figure below and answer
(1) 12 (2) 15 the following questions.
(3) 18 (4) 10
No. of
86. How many of these unmarried girls are above 25 ?
No. of families families having
(1) 12 (2) 15 50
having V.C.R. 110 T.V.
(3) 4 (4) 0 18 80 30
87. In the Olympic games, flags of 6 nations were hoisted in the 20 20 No. of
following way. The flag of America was to the left of Indian 10 15 15 families having
Tricolour and to the right of the flag of France. The flag of 15 scootors
Australia was on the right of the Indian flag but to the left of 40 25
the flag of Japan, which was to the left of the flag of China.
Find the two flags which are in the centre. No. of families having maruti

SPACE FOR ROUGH WORK


MT-10 Target NTSE
93. Find out the number of families which have all the four things 99. What is the number of straight lines in the following
mentioned in the diagram. figure ?
(1) 40 (2) 30
(3) 35 (4) 20
94. Find out the number of families which have scooters.
(1) 145 (2) 100
(3) 188 (4) 240
(1) 11 (2) 14
95. Find out the number of families which have V.C.R. and T.V.
(3) 16 (4) 17
both
DIRECTION (Q. 100): In each of the following questions, there
(1) 84 (2) 24 is a diagram marked (X), with one or more dots placed in it. The
(3) 104 (4) 100 diagram is followed by four other figures, marked (1), (2), (3) and
(4) only one of which is such as to make possible the placement of
96. Find out the number of families which have only one thing, the alternative in each case.
that is, either V.C.R. or T.C. or Scooter of Maruti. 100.
(1) 160 (2) 184
(3) 225 (4) 254
97. Find out the number of families which have T.V. and scooter
both but have neither V.C.R. nor Maruti.
(1) 15 (2) 30
(3) 4 (4) 50
98. A dice has been thrown four times and produces following
result. (X)

6 5
1 4
3 2

(i) (ii)
(1) (2)
3 2
1 6
5 3

(iii) (iv)
Which number will appear opposite to the number 3 ?
(1) 4 (2) 5
(3) 6 (4) 1
(3) (4)

SPACE FOR ROUGH WORK


Mock Test-1 MT-11

ANSWER SHEET

Name : ..................................School .............................. Test Code............Roll No. : ...................

City : ................... Date : ................... Class : ..................

(1) (2) (3) (4) (1) (2) (3) (4) (1) (2) (3) (4)
1. 35. 68.
2. 36. 69.
3. 37. 70.
4. 38. 71.
5. 39. 72.
6. 40. 73.
7. 41. 74.
8. 42. 75.
9. 43. 76.
10.
44. 77.
11.
45. 78.
12.
46. 79.
13.
47. 80.
14.
48. 81.
15.
49. 82.
16.
50. 83.
17.
51. 84.
18.
52. 85.
19.
53. 86.
20.
54. 87.
21.
22. 55. 88.

23. 56. 89.

24. 57. 90.


25. 58. 91.
26. 59. 92.
27. 60. 93.
28. 61. 94.
29. 62. 95.
30. 63. 96.
31. 64. 97.
32. 65. 98.
SPACE FOR ROUGH WORK
33. 66. 99.
34. 67. 100.

Sig. of Invigilator.................................
SPACE FOR ROUGH WORK
1
(Stage 1)

Time
Time :: 120 Minutes
90 Minutes Max. Marks : 100
INSTRUCTIONS FOR CANDIDATES
Read the following instructions carefully before you answer the questions.
1. There are 100 questions in this test all are compulsory. Each question has 4 choices (1), (2), (3), and (4), out of which
ONLY ONE is correct.
2. Answers are to be given on separate answer sheet provided at the end of this paper. You can tear this answer sheet
along the cut marks provided.
3. If you do not know the answer to any question, do not waste time on it and pass on to the next one. Time permitting,
you can come back to the questions, which you have left in the first instance and attempt them.
4. Since the time allotted for this question paper is very limited you should make the best use of it by not spending too
much time on any one question.
5. For all questions 1-100 put a cross mark (×) on the number of the correct alternative on the answer sheet against the
corresponding question number by using a blue ball point pen. Example

Correct Way : Q.No. Alternatives


1. 1 2 3 4

Wrong Way : Q.No. Alternatives


1. 1 2 3 4
1. 1 2 3 4

6. Rough work can be done anywhere in the booklet but not on the answer sheet/loose paper.
7. Every correct answer will be awarded one mark. There is no negative marking.
8. After completion of this test you can evaluate your performance by comparing your answers with answers provided
in solution booklet.
MT-14 Target NTSE
1. Transportation in the living organisms is necessary because I
of the following reasons:
(1) Food, oxygen, water and other essential substances
II
are distributed through this system.
(2) It displaces the excretory wastes from the cells of the
III
body to the excretory organs from where they are
expelled out.
(3) Temperature and pressure throughout the body are
kept uniform.
IV
(4) All of the above
2. Match column I with column II and select the correct answer (1) I - Dendron, II-Cytoplasm, III-Axon, IV - Synapse
using the codes given below :
(2) I -Dendrites, II-Nucleus, III-Axon, IV-Axon endings
Column I Column II
(3) I-Axon, II-Nucleus, III-dendrites, IV-Axon endings
(A) Consists of 3 atoms (p) Unidirectional
(4) I-Dendron, II-Cell body, III-Axon, IV-Dendrites
of oxygen
5. Excessive exposure of humans to UV-rays results in
(B) Flow of energy in an (q) Ozone (O3)
ecosystem (i) damage to immune system
(C) Main cause of (r) Carnivores (ii) damage to lungs
depletion of ozone (iii) skin cancer
layer (iv) peptic ulcers
(D) Third trophic level (s) Producers (1) (i) and (ii) (2) (ii) and (iv)
(E) Green plants (t) CFCs (3) (i) and (iii) (4) (iii) and (iv)
(1) (A) ® p, (B) ® q, (C) ® (t); (D) ® r, (E) ® (s) 6. Consider the following statements:
(2) (A) ® q, (B) ® p, (C) ® (t); (D) ® s, (E) ® (r) (a) Plants are divided into five groups: Thallophytes,
(3) (A) ® q, (B) ® (t); (C) ® p, (D) ® r, (E) ® (s) Bryophytes, Pteridophytes, Gymnosperms and
Angiosperms.
(4) (A) ® q, (B) ® p, (C) ® (t); (D) ® r, (E) ® (s)
(b) Monera is divided into Archaebacteria and Eubacteria.
3. Consider the following statements :
(c) Penicillium and Yeast both belong to group fungi.
(a) Connective tissue functions in binding and supporting
Which of the above statements are correct?
animal tissues.
(b) Histology is the study of cells and tissues. (1) (a) and (b) (2) (b) and (c)
(c) Adipose tissue is a type of connective tissue. (3) (a), (b) and (c) (4) None of these
(d) Blood is a type of connective tissue. 7. It is important to make small check dams across the flooded
Which of these statement(s) is/are correct ? gullies because they
(1) (a), (b) and (c) (2) (a), (c) and (d) (i) hold water for irrigation.
(3) (b), (c) and (d) (4) All are correct (ii) hold water and prevent soil erosion.
4. The figure depicting neuron is given below. The correct (iii) recharge ground water.
identification of labels I, II, III and IV respectively is (iv) hold water permanently.
__________, __________, __________ and (1) (i) and (iv) (2) (ii) and (iii)
__________. (3) (iii) and (iv) (4) (ii) and (iv)

SPACE FOR ROUGH WORK


Mock Test-1 MT-15

8. Match column I with column II and select the correct answer 12. Consider the following statements:
using the codes given below : (a) Fore brain is centre of intelligence, control of
Column I Column II movements, hearing, smell and sight.
(A) Fish farming (p) Broiler (b) Control and coordination are functions of nervous and
(B) Cattle used for tilling (q) Culture fishery endocrine systems.
and carting (c) Rise in sugar level in blood stops secretion of insulin
(C) Chicken better fed for (r) Draught animals by pancreas.

obtaining meat Which of the above statements are correct?

(D) Bee-keeping (s) Apiculture (1) (a) and (b) (2) (b) and (c)

(1) (A) ® q, (B) ® r, (C) ® p, (D) ® s (3) (a) and (c) (4) None of these
(2) (A) ® r, (B) ® q, (C) ® p, (D) ® s 13. Following figures represent the type of waste:
(3) (A) ® r, (B) ® q, (C) ® s, (D) ® p
(4) (A) ® r, (B) ® s, (C) ® q, (D) ® p
9. Identify the plant which indicates the following features?
(I) Auto-trophic in nature
(II) Grow in water logged swampy soils Leaves Banana peel
(III) They capture and digest insect to fulfill the requirement
of nitrogen compound.
(1) Saprophytic plants (2) Parasitic plants
(3) Semi-parasitic plants (4) Insectivorous plants
10. On the basis of following features identify correct
option
I. It is the process of reduction of nitrates into gaseous
nitrogen which escape into atmosphere.
Flower Paper
II. It is a reduction process.
III. It is caused by bacteria present in water logged soils, (1) Biodegradable (2) Non-biodegradable
e.g., Pseudomonas. (3) Both (1) and (2) (4) Neither (1) nor (2)
(1) Nitrification (2) Denitrification
14. A student soaked 10 g of raisins in 100 ml of distilled water
(3) Nitrogen fixation (4) Nitrogen assimilation
in two beakers A and B each. She maintained beaker A at
11. A group of laboratory mice having tails are bred together 30°C and beaker B at 60°C. After two hour, the percentage of
and their progeny studied. They again bred them for four
water absorbed will be
successive generations. What do you think would be the
nature of the new progeny? (1) the same in both A and B.
(1) All mice born will have tails. (2) more in A than in B.
(2) All mice born will have no tails.
(3) more in B than in A.
(3) The ratio of tail less to tailed mice will be 1 : 3
(4) exactly twice as much in B as in A.
(4) The ratio of tail less to tailed mice will be 1 : 4

SPACE FOR ROUGH WORK


MT-16 Target NTSE
15. Match column I with column II and select the correct answer (1) (a), (b) and (c) (2) (b) and (c)
using the codes given below : (3) (a), (c) and (d) (4) (a) and (d)
Column I Column II 19. When sodium hydrogen carbonate powder is added to acetic
(A) AgNO3(aq) + NaCl(aq) ® (p) Redox reaction acid, a gas evolves. Which one of the following statements
AgCl(s) + NaNO3(aq) (q) Oxidation is not true for this gas? It
(B) H2(g) + Cl2(g) ® 2HCl(g) (r) Precipitation (1) turns lime water milky

(C) C2H4(g) + 3O2(g) ® (s) Endothermic (2) extinguishes a burning splinter


(3) dissolves in a solution of sodium hydroxide
2CO2(g) + 2H2O(l)
(4) reacts with water to form sulphuric acid
(D) CaCO3(s) ®
20. Which of the following pairs of substances are chemically
CaO(s) + CO2(g)
same?
(1) (A) ® p, (B) ® r, (C) ® q, (D) ® s (1) Lime water and milk of lime
(2) (A) ® r, (B) ® p, (C) ® q, (D) ® s (2) Dead burnt plaster and gypsum
(3) (A) ® r, (B) ® q, (C) ® p, (D) ® s (3) Both the above
(4) (A) ® r, (B) ® p, (C) ® s, (D) ® q (4) None of the above is correct

16. Consider the following statements : 21. Four test-tubes were taken and marked A, B, C and D
respectively. 2 mL of solution of Al2(SO4)3 in water was
In 90 2 + :
38 Sr filled in each of the four test-tubes. Clean piece of metal Zn
(i) atomic number is 36 was placed in test-tube A, clean iron nail was put in
(ii) number of electrons is 38 test-tube B, clean copper wire was placed in test-tube C and
(iii) number of neutrons is 52 a clean aluminium wire was placed in test-tube D. It was
(iv) number of protons is 38 observed that no change occurred in any of the test-tubes.
Which of these are correct ? The correct inference drawn is:
(1) (i) and (iii) (2) (ii) and (iii) (1) Zinc is more active than aluminium
(3) (iii) and (iv) (4) (i) and (iv) (2) Zinc is more active than copper
17. Element A belongs to Group VII in p-block and element B (3) Copper is more active than aluminium
belongs to Group I in s-block of the periodic table. Out of (4) Zinc, iron and copper are less active than aluminium
the following assumptions, the correct one is : 22. Match column I with column II and select the correct answer
(1) A and B are metals using the codes given below :
(2) A and B are non-metals Column I Column II
(3) A is a metal and B is a non-metal
(A) Metallic character (p) Increases down a group
(4) A is a non-metal and B is a metal
(B) Atomic size (q) Increases across a period
18. Consider the following statements:
(C) Non-metallic character (r) Decreases down a group
(a) Mass is not affected by temperature and pressure.
(D) Electronegativity (s) Decreases across a period
(b) Plasma contains ionised gases.
(1) (A) ® p, s, (B) ® p, s, (C) ® q, r, (D) ® q, r
(c) Alloys are homogeneous mixtures. (2) (A) ® p, s, (B) ® p, s, (C) ® q, r, (D) ® p, r
(d) Solution is a heterogeneous mixture. (3) (A) ® q, s, (B) ® p, s, (C) ® q, r, (D) ® q, r
Which of the above statements are correct? (4) (A) ® p, s, (B) ® p, q, (C) ® q, r, (D) ® q, r

SPACE FOR ROUGH WORK


Mock Test-1 MT-17

23. A mixture of sulphur and carbon disulphide is: Which one of the following pairs is the correct description
(1) heterogeneous and shows Tyndall effect of both the samples?
(2) homogeneous and shows Tyndall effect (1) A = Gel and B = Gel
(2) A = Solution and B = Solution
(3) heterogeneous and does not show Tyndall effect
(3) A = Gel and B = Suspension
(4) homogeneous and does not show Tyndall effect (4) A = Solution and B = Gel.
24. Match column I with column II and select the correct answer 27. Generally metals react with acids to give salt and hydrogen
using the codes given below : gas. Which of the following acids does not give hydrogen
Column I Column II gas on reacting with metals (except Mn and Mg)?
(1) H2SO4 (2) HCl
(A) Detergent (p) Ca2+ and Mg2+ ions
(3) HNO3 (4) All of these
(B) Carboxymethyl (q) Used in pickles
28.
cellulose
(C) Marsh gas (r) Non-biodegradeble
(D) Hard water (s) Methane
(E) Solution of acetic acid (t) Helps in suspending dirt 0 2 4 6 8 10 12 Time(s)

(1) (A) ® r, (B) ® s, (C) ® t, (D) ® p, (E) ® q In the above velocity-time graph of a moving object
(2) (A) ® t, (B) ® r, (C) ® s, (D) ® p, (E) ® q (1) Acceleration in the first 2 seconds is 2.3 ms–1
(3) (A) ® r, (B) ® t, (C) ® s, (D) ® p, (E) ® q (2) Acceleration in the last 2 seconds is –2.3 ms–1
(4) (A) ® r, (B) ® t, (C) ® s, (D) ® q, (E) ® p (3) Motion is uniform between second and tenth second
25. Consider the following statements: (4) All are correct

(a) pH of NaCl solution is 7. 29. Consider the following statements :

(b) Copper sulphate gives red colour with universal (a) No net force acts on a rain drop falling vertically with a
indicator. constant speed.
(b) If net force acting on the body is zero, momentum of
(c) De-greesing of metals can be done with H2SO4.
the body remains constant.
Which of the above statement(s) is/are correct?
(c) Particle of different masses falls with different
(1) Only (b) (2) (a) and (b)
acceleration on earth.
(3) (b) and (c) (4) Only (c)
(d) It is easier to start motion in a heavier body than a
26. When light is passed through samples A and B placed in a lighter body.
line in a dark room, the effects observed are as shown in the
Which of the above statements are correct?
figure.
(1) (b) and (c) (2) (a) and (b)
(3) (a), (b) and (d) (4) All of the above
Visible 30. A block of ice is floating on water contained in a beaker.
path of light
Torch When all the ice melts, the level of water
(1) rises (2) falls
Sample A Sample B
(3) remains unchanged (4) None of these

SPACE FOR ROUGH WORK


MT-18 Target NTSE
31. Acceleration due to gravity ‘g’ on the surface of earth and Column I Column II
universal gravitational constant ‘G’ are correlated as :
(Here M = mass of earth and R = radius of earth)
(A) Reflection of light (p) R
GR 2 GM V
(1) g= (2) g=
M R2

g=
GR GM 2 (B) Refraction of light (q)
(3) (4) g=
M2 R
32. Match column I with column II and select the correct answer
using the codes given below :
(C) Total internal reflection (r)
Column I Column II

1
(A) Mechanical to Electrical (p) mv2
2
(D) Dispersion (s)
i>c
(B) Electrical to mechanical (q) mgh
(C) Kinetic energy (r) A. C. generator (1) (A) ® r, (B) ® q, (C) ® p, (D) ® s
(D) Potential energy (s) Electric motor (2) (A) ® p, (B) ® r, (C) ® s, (D) ® q
(1) (A) ® r, (B) ® s, (C) ® p, (D) ® q (3) (A) ® s, (B) ® p, (C) ® r, (D) ® q
(2) (A) ® p, (B) ® r, (C) ® s, (D) ® q (4) (A) ® q, (B) ® r, (C) ® s, (D) ® p
(3) (A) ® s, (B) ® p, (C) ® r, (D) ® q 37. For a normal eye, the near point is 25 cm from the eye lens.
(4) (A) ® q, (B) ® r, (C) ® p, (D) ® s The distance between eye lens and retina of the eye is
2.5 cm. What is the power of the eye lens?
33. A girl of mass 40 kg climbs 20 steps of a staircase, each
measuring 20 cm high in 30s. Calculate the power developed (1) 4 D (2) 40 D
by the boy. [Take g = 10 m/s2] (3) 44 D (4) – 44 D
(1) 160 W (2) 80 W 38. Two conductors each of resistance R ohm are connected in
(3) 53.33 W (4) 26.66 W series and then in parallel. Find the ratio of the resistance of
34. The minimum distance required to here an echo from a the combination in series and in parallel.
reflecting wall if speed of sound is 344 m/s is (1) 1 (2) 2
(1) 17.2 m (2) 34.8 m (3) 50.1 m (4) 60.2 m (3) 4 (4) 6
35. Which of the following waves can never be transverse ? 39. Consider the following statements :
(1) Sound waves through air
(a) The magnitude and direction of alternating current
(2) Light waves through air
reverses periodically.
(3) Waves set up on a stretched string
(b) Frequency of alternating current in India is 50 Hz.
(4) Waves set up on the surface of water when a stone is
hit on the surface of water. (c) The frequency of direct current is zero.
36. Match column I with column II and select the correct answer Which of the above statements are correct?
using the codes given below : (1) (b) and (c) (2) (a) and (b)
(3) All of them (4) None of them

SPACE FOR ROUGH WORK


Mock Test-1 MT-19

40. The bulk of energy released in nuclear fission appears as 47. If the mid-point of the line joining (3, 4) and (p, 7) is (x, y) and
(1) kinetic energy of nuclear fragments 2x + 2y + 1 = 0, then p =
(2) heat energy 17 17
(1) 15 (2) – (3) – 15 (4)
2 2
(3) radiant energy 48. An aeroplane flying horizontally 1 km above the ground is
(4) chemical energy observed at an elevation of 60°. After 10 seconds, its
elevation is observed to be 30°. The speed of the aeroplane
is
1 2
(1) 240 km/hr (2) 420 km/hr
41. If x = 1 + 53 + 5 3 , then find the value of x3 – 3x2 – 12x + 6.
(3) 240 3 km/hr (4) 420 3 km/hr
(1) 22 (2) 20 (3) 16 (4) 14 49. Which constant should be added and subtracted to solve
42. How many terms of the A.P. 9, 17, 25, ... must be taken to
give the sum 636? the quadratic equation 4x2 – 3 x - 5 = 0 by the method of
completing the square?
(1) 15 (2) 14 (3) 13 (4) 12
9 3 3 3
(1) (2) (3) (4)
æ1 ö 10 16 4 4
43. The ratio in which the point ç ,6 ÷ divides the line segment 50. If the polynomials 2x3 + mx2 +3x – 5 and x3 + x2 – 4x + m
è2 ø
leaves the same remainder when divided by x – 2, then the
joining the points ( 3,5) and ( –7,9) is value of m is
(1) 3 : 1 (2) 1 : 3 (3) 1 : 2 (4) 2 : 1 3 13 3 13
(1) – (2) – (3) (4)
44. A cone of semi-vertical angle a is inscribed in a sphere of 13 3 13 3
51. AB is the chord of a circle with centre O. AB is produced to
radius 2 cm. The height of the cone is
C, such that BC = OB. CO is joined and produced to meet
(1) 4 sin2 a (2) 4 cos2 a the circle in D. If ÐACD = y° and ÐAOD = x°, then relation
(3) 4 sin a (4) 4 cos a between x° and y° is

45. If x = psec q and y = q tan q then B


A C
(1) x2 – y2 = p2q2 (2) x2q2 – y2p2 = pq y°

1 x°
(3) x2q2 – y2p2 = 2 2 (4) x2q2 – y2p2 = p2q2 O
p q D
46. Find the value of ÐADC from the given figure. (1) x° = 3y° (2) 3x° = y°
A (3) 2x° = y° (4) x° = 2y°
52. In figure, a circle touches all the four sides of a quadrilateral
ABCD, whose sides AB = 6 cm, BC = 7 cm, and CD = 4 cm
30°
then, the length of AD is
a D R C
D

S Q
50° 55°
B C
(1) 225° (2) 135° (3) 360° (4) 50° A P B

(1) 10 cm (2) 7 cm (3) 3 cm (4) 4 cm

SPACE FOR ROUGH WORK


MT-20 Target NTSE
53. A day is selected from April, whose first day is Monday. 64. Where was the Elgin Mill set up?
Find the probability that the day selected is a Monday. (1) Kanpur (2) Ahmedabad
1 1 1 2 (3) Banaras (Varanasi) (4) Mirzapur
(1) (2) (3) (4)
7 5 6 5 65. What was done during captain swing movement?
54. If the mean of the numbers (1) Threshing machines were broken.
27 + x, 31 + x, 89 + x, 107 + x, 156 + x is 82, then the mean of
(2) Rich farmers were compelled to help poor farmers.
130 + x, 126 + x, 68 + x, 50 + x, 1 + x is
(1) 75 (2) 157 (3) 82 (4) 80 (3) Landowners were looted.
55. The A.M. of the first ten odd numbers is (4) Agriculture was expanded.

(1) 10 (2) 100 (3) 1000 (4) 1 66. What was the name of book written by Phan Boi Chau?
56. One side of an equilateral triangle is 6 cm. Its area by using (1) History of Vietnam
Heron’s formula is (2) History of Laos
(1) 9 cm2 (2) 3 cm2 (3) History of Cambodia
(4) The History of the Loss of Vietnam
(3) 9 3 cm2 (4) 3 8 cm2
67. In the meeting of the Estate General, the members of the
57. If y = logx – 3 (x – 6x + 9), then 2 logy5 = ______.
2
third Estate demanded
(1) 4 (2) 8 (3) 2 (4) 5 (1) All the three Estates should have one vote altogether
58. A triangle and a parallelogram have the same base and the (2) Each member should have one vote
same area. If the sides of the triangle are 26 cm, 28 cm and (3) Each estate should have one vote
30 cm, and the parallelogram stands on the base 28 cm, then (4) Each estate should have more than one vote
the height of the parallelogram is 68. In the context of Russia, what was KULAK?
(1) 42 cm (2) 12 cm (3) 24 cm (4) 21 cm (1) A collective farm (2) A Russian Church
59. If L.C.M. {1, 2, 3, ...., 200} = b × L.C.M. {102, 103, ..., 200}, (3) Well-to-do Peasants (4) Landless labourers
then b = 69. The French tried to stem the invasion of rats in Hanoi by :
(1) 201 (2) 401 (3) 101 (4) 301 (1) offering the price for every rat caught by the
Vietnamese
60. The value of tan2 60° cosec2 45° + sec2 45° sin 30° is
(2) By plugging the Sewers
(1) –7 (2) 6 (3) – 6 (4) 7
(3) By starting a rat hunt in 1902, hiring Vietnamese
61. Who was Olympe de Gouges?
workers and paying them for each rat caught
(1) A politically active woman in revolutionary France.
(4) By poisoining the rats
(2) A social reformer
70. Name the film which reflected the moral confusion that had
(3) A supporter of Jacobin club been caused in the US due to US Vietnam War?
(4) All of the above (1) John Ford Coppala’s ‘The War of Vietnam’
62. Who said these words, “I always believed that my silence (2) James Cameroon’s ‘Vietnam War’
on several topics will be an advantage in the long run”. (3) Stephen Spelberg’s Vietnam War
(1) Lala Lajpat Rai (2) Pt. Jawahar Lal Nehru (4) John Ford Coppala’s ‘Apocalypse Now’.
(3) Lokmanya Tilak (4) Madan Mohan Malviya 71. Which two leaders advocated the adoption of socialist
63. What was the Napoleonic code known as? ideas?
(1) The Napoleonic code of 1808 (1) Subhash Chandra Bose
(2) The civil code of 1809 (2) Jawaharlal Nehru
(3) The civil code of 1804 (3) Mahatma Gandhi and Tagore
(4) The Napoleonic code of 1802 (4) Jawaharlal Nehru and Subhash Chandra Bose

SPACE FOR ROUGH WORK


Mock Test-1 MT-21

72. The principle of ‘Garden City’ was developed by 83. The Indus in the West and the Ganga-Brahmaputra in the
(1) Ronald Russo (2) Martin King east are the river streams which constitute the
(3) Ebenezer Howard (4) Perez Patrick (1) Eastern Plain (2) Southern Plain
73. The two volcanic islands in the Indian territory are – (3) Western Plain (4) Northern Plain
84. The diverse flora and fauna of the planet are under great
(1) Kavaratti and New Moor
threat mainly due to :
(2) Bitra and Kavaratti (1) insensitivity to our environment
(3) Pamban and Barren (2) Jhum cultivation
(4) Norcondam and Barren (3) Modern method of agriculture
74. Which of the following state is not connected with HVJ (4) Commercial agriculture
pipeline? 85. What do you mean by Residuary subjects?
(1) Madhya Pradesh (2) Uttar Pradesh (1) Matters of state level importance.
(3) Haryana (4) Gujarat (2) Subjects and matters of national issues.
75. In which state, terrace farming is practised? (3) Subjects which are not included in union, state and
(1) Punjab (2) Haryana concurrent list.
(3) Plains of Uttar Pradesh (4) Jammu & Kashmir (4) Subjets which are included in union list and state list
both.
76. To which one of the following types of vegetation does
86. There is a third type of government apart from central and
rubber belong to? state government in Belgium –
(1) Tundra (2) Himalayan (1) Local government
(3) Tidal (4) Tropical evergreen (2) Provincial government
77. Mountain ranges in the eastern part of India forming the (3) Community government
boundary with Myanmar are collectively called as- (4) Regional government
(1) Himachal (2) Purvanchal
87. Which of the following countries face challenge of expansion
(3) Uttarakhand (4) Uttar Pradesh
of democracy?
78. What is the other name of the Cardamom Hills ?
(1) China and Nepal
(1) Palni Hills (2) Nilgiri Hills
(2) Mynamar and Pakistan
(3) Yelagiri Hills (4) Annamalai Hills
(3) India and US
79. Which of the following are matched correctly?
(a) Area of highest growth rate of population - Nagaland (4) France and Nepal
(b) Area with lowest sex ratio - Delhi 88. What does Kittiki - Hachchhiko mean?
(c) Area with lowest density of population - Rajasthan (1) Catch and save (2) Sow and harvest
(d) Area with highest density of population - Delhi (3) Pluck and Plant (4) Pluck and throw
(1) b, d (2) c, d (3) a, c, d (4) a, d 89. Which of the following states have bicameral legislature?
80. Who said these lines ‘There is enough for everybody’s A. Andhra Pradesh B. Bihar
need but not for anybody’s greed’? C. Karnataka D. Maharashtra
(1) S. C. Bose (2) Mahatma Gandhi
E. Tamil Nadu F. Uttar Pradesh
(3) Jawaharlal Nehru (4) Vallabh bhai Patel
Select the correct answer from the code given below-
81. When running water cuts through clayey soils and makes
deep channels they lead to (1) B, C, D and F (2) A, B, C and F
(1) Gully erosion (2) Sheet erosion (3) C, D, E and F (4) A, C, D and F
(3) Deforestation (4) Afforestation 90. A direct vote in which an entire electorate is asked to either
82. Which islands are our Southern neighbours accept or reject a particular proposal is termed as
(1) Thailand, Indonesia (2) Oman, Kuwait (1) Coalition (2) Referendum
(3) Madagascar, Oman (4) Maldives, Sri Lanka (3) Veto (4) Election

SPACE FOR ROUGH WORK


MT-22 Target NTSE
91. Which political party came to power in Bolivia in 2006? 96. The organisations that monitor and issue ISI, Agmark or
(1) Democratic Party (2) Socialist Party Hallmark certificates allow producers to use their logos
(3) Liberal Party (4) Conservative Party provided they follow certain
92. Captain of the first national Hockey team who was also a (1) Work permit (2) Quality standards
member of the constituent assembly (3) Debit card (4) Topic of press
(1) Baldev Singh (2) Somnath Lahiri 97. Revamped Public Distribution system provides benefit to:
(3) Jaipal Singh (4) K. M. Munshi (1) cities
93. What are national parties? (2) remote and backward areas
(1) Parties which have units in various states (3) self-help groups
(2) Parties which have no units (4) cooperative societies
(3) Parties which have units in two states 98. The State level consumer court deals in cases involving
(4) Parties which have units in all states claims between __________ to __________.
94. Which among the following cannot be challenged in a court (1) 10 lakhs; 90 lakhs (2) 20 lakhs; 1 crore
of law? (3) 30 lakhs; 5 crores (4) 40 lakhs; 10 crores
(1) Directive Principles of state policy 99. According to the World Development Report 2006, brought
out by the World Bank, countries with per capita income of
(2) Fundamental Rights
`4,53,000 p.a. and above are called ___________.
(3) Free to choose any profession or business
(1) Weak countries
(4) Right to move freely to any part of the country (2) Rich countries
95. A very good example of sharing power among different social (3) Low income countries
group is (4) All of them
(1) Panchayati Raj system in India 100. How do you describe the judicial machinery under COPRA?
(2) Demand for separate Eelam by Tamils (1) 3 tier quasi-judicial machinery
(3) Community government in Belgium (2) 2 tier quasi-judicial machinery
(4) Federal arrangement in India (3) 4 tier quasi-judicial machinery
(4) 5 tier quasi-judicial machinery

SPACE FOR ROUGH WORK


Mock Test-1 MT-23

ANSWER SHEET

Name : ..................................School .............................. Test Code............Roll No. : ...................

City : ................... Date : ................... Class : ..................

(1) (2) (3) (4) (1) (2) (3) (4) (1) (2) (3) (4)
1. 35. 68.
2. 36. 69.
3. 37. 70.
4. 38. 71.
5. 39. 72.
6. 40. 73.
7. 41. 74.
8. 42. 75.
9. 43. 76.
10.
44. 77.
11.
45. 78.
12.
46. 79.
13.
47. 80.
14.
48. 81.
15.
49. 82.
16.
50. 83.
17.
51. 84.
18.
52. 85.
19.
53. 86.
20.
54. 87.
21.
55. 88.
22.
23. 56. 89.

24. 57. 90.


25. 58. 91.
26. 59. 92.
27. 60. 93.
28. 61. 94.
29. 62. 95.
30. 63. 96.
31. 64. SPACE FOR ROUGH WORK 97.
32. 65. 98.
33. 66. 99.
34. 67. 100.

Sig. of Invigilator.................................
SPACE FOR ROUGH WORK
2
(Stage 1)

Time : 120 Minutes Max. Marks : 100

INSTRUCTIONS FOR CANDIDATES


Read the following instructions carefully before you answer the questions.
1. There are 100 questions in this test all are compulsory. Each question has 4 choices (1), (2), (3), and (4), out of which
ONLY ONE is correct.
2. Answers are to be given on separate answer sheet provided at the end of this paper. You can tear this answer sheet
along the cut marks provided.
3. If you do not know the answer to any question, do not waste time on it and pass on to the next one. Time permitting,
you can come back to the questions, which you have left in the first instance and attempt them.
4. Since the time allotted for this question paper is very limited you should make the best use of it by not spending too
much time on any one question.
5. For all questions 1-100 put a cross mark (×) on the number of the correct alternative on the answer sheet against the
corresponding question number by using a blue ball point pen. Example

Correct Way : Q.No. Alternatives


1. 1 2 3 4

Wrong Way : Q.No. Alternatives


1. 1 2 3 4
1. 1 2 3 4

6. Rough work can be done anywhere in the booklet but not on the answer sheet/loose paper.
7. Every correct answer will be awarded one mark. There is no negative marking.
8. After completion of this test you can evaluate your performance by comparing your answers with answers provided
in solution booklet.
MT-26 Target NTSE
DIRECTIONS (Qs. 1 - 2) : In each of these questions, four terms 11. In a certain code, PROSE is written as PPOQE. How would
are given. While three of them are identical in some way, one is LIGHT be written in that code ?
different from the rest. Select the odd one as your answer. (1) LIGFT (2) LGGHT
1. (1) UNICEF (2) IMF (3) LGGFT (4) JIEHR
(3) WHO (4) SAARC 12. In a certain code, 15789 is written as EGKPT and 2346 is
2. (1) 5 8 78 (2) 6 4 8 2 written as ALUR. How is 23549 written in that code ?
(3) 5 7 8 8 (4) 9 74 8 (1) ALEUT (2) ALGTU
DIRECTIONS (Qs. 3 - 5) : Find the next term in the given series (3) ALGUT (4) ALGRT
in each of the questions below. 13. In a certain code language, ‘po ki top ma’ means ‘Usha is
3. 5, 6, 8, 9, 11, .... playing cards’. ‘kop ja ki ma’ means’ Asha is playing tennis’,
(1) 15 (2) 12 ‘ki top sop ho’ means ‘they are playing football’ and ‘po sur
(3) 17 (4) 20 kop’ means ‘cards and tennis’. Which word in that language
4. 35, 30, 25, 20, 15, 10, .... means ‘Asha’ ?
(1) 15 (2) 10 (1) ja (2) ma
(3) kop (4) top
(3) 5 (4) 2
14. In a certain code, TERMINAL is written as SDQLJOBM.
5. 0, 2, 6, 12, 20, ....
How is CREDIBLE written in that code?
(1) 38 (2) 30
(1) BQDCJCMF (2) BQDCHAKD
(3) 45 (4) 60 (3) DSFEJCMF (4) DSFEHAKD
6. Complete the following series: 15. Laxman went 15 km to the west from my house, then turned
a__bb a__b__a__b__ left and walked 20 km. He then turned East and walked 25 km
(1) aabab (2) ababb and finally turning left covered 20 km. How far was he from
(3) bbaba (4) baaba my house?
DIRECTIONS (Qs. 7 & 8) : In each of the following questions, (1) 5 km (2) 10 km
various terms of a series are given with one term missing as (3) 40 km (4) 80 km
shown bye (?). Choose the missing term. 16. Rama remembers that she met her brother on Saturday, which
7. AZ, CX, FU, (?) was after the 20th day of a particular month. If the 1st day of
(1) IR (2) JQ that month was Tuesday, then on which date did Rama meet
her brother ?
(3) IV (4) KP
(1) 24th (2) 23rd
8. 2Z5, 7Y7, 14X9, 23W11, 34V13, (?)
(3) 25th (4) None of these
(1) 27U24 (2) 45U15
17. I am facing west. I turn 45° in the clockwise direction and
(3) 47Ul5 (4) 47V14 then 180° in the same direction and then 270° anticlockwise.
9. Find the wrong number in the series: 7, 28, 63, 124, 215, 342, Which direction am I facing now?
511 (1) South-West (2) South
(1) 7 (2) 28 (3) West (4) North-west
(3) 124 (4) 215 18. Persons X, Y, Z and Q live in red, green, yellow or blue
10. If the letters in each of the following four words are first coloured houses placed in a sequence on a street. Z lives in
rearranged in the alphabetical order and then the groups of a yellow house. The green house is adjacent to the blue
letters so formed are rearranged as in a dictionary, which house. X does not live adjacent to Z. The yellow house is in
word would have its group of letters as third among the between the green and red houses. The colour of the house
four? X lives in is
(1) Deaf (2) Road (1) green (2) blue
(3) Code (4) Lack (3) red (4) cannot be determined

SPACE FOR ROUGH WORK


Mock Test-2 MT-27

DIRECTIONS (Q. 19 - 23) : Study the following information to DIRECTIONS (Qs. 27 - 31) : The questions below are based on
answer these questions. the following statements.
There are six person A, B, C, D, E and F. C is the sister of F. B Asha and Charu are good in Mathematics and Athletics.
is the brother of E’s husband. D is the father of A and grandfather Deepa and Asha are good in Athletics and Studies, Charu and
of F. There are two fathers, three brothers and a mother in the Beena are good in General Knowledge and Mathematics. Deepa,
group. Beena and Ela are good in Studies and General Knowledge. Ela
19. Who is the mother? and Deepa are good in Studies and Arts.
(1) A (2) D 27. Who is good in Studies, General Knowledge, Athletics and
(3) B (4) E Arts ?
20. Who is E’s husband? (1) Asha (2) Beena
(1) B (2) A (3) Charu (4) Deepa
28. Who is good in Studies, General Knowledge and
(3) C (4) F
Mathematics ?
21. How many male members are there in the group?
(1) Asha (2) Beena
(1) One (2) Three
(3) Charu (4) Deepa
(3) Two (4) Four
29. Who is good in Studies, Mathematics and Athletics ?
22. How is F related to E? (1) Asha (2) Beena
(1) Uncle (2) Son (3) Charu (4) Deepa
(3) Husband (4) Daughter 30. Who is good in Athletics, General Knowledge and
23. Which of the following is a group of brothers? Mathematics ?
(1) ABF (2) BFC (1) Asha (2) Beena
(3) ABD (4) BDF (3) Charu (4) Deepa
DIRECTIONS (Qs. 24 & 25) : In the questions below choose one 31. Who is good in Studies, General Knowledge and Arts but
of the figures labelled (1), (2), (3) and (4) which best represents not in Athletics ?
the relationship among the items given in the questions that (1) Asha (2) Beena
follow. (3) Charu (4) Ela
DIRECTIONS (Qs. 32 - 34) : In the questions below, there are two
statements followed by two conclusions A and B. You have to take
(1) (2) the statements to be true even if they seem to be at variance from
commonly known facts and then decide which of the conclusions
logically follows. Choose:
(1) if only A follows
(2) if only B follows
(3) (4) (3) if both A and B follow
(4) if neither A nor B follows
24. Musicians, Men, Women 32. Statements :
25. Parrots, Birds, Mice All men are married.
26. How many cubes are there in the following figure? Some men are educated.
Conclusions :
A : Some married persons are educated.
B : Some educated are married.
33. Statements :
All boys are girls.
No girl is a father.
Conclusions :
A : All girls are boys.
(1) 6 (2) 10 B : No boy is a father.
(3) 12 (4) 8

SPACE FOR ROUGH WORK


MT-28 Target NTSE
34. Statements :
All boys are rivers. 39.
Some rivers are girls
Conclusions :
A : Some girls are boys. A B C D
B : Some boys are girls. (1) C (2) B
35. Pick up a figure from the Answer Figures to fit into the given (3) D (4) A
Incomplete Figure.
Incomplete Figure 40.

A B C D
(1) C (2) D
(3) B (4) A
41. A car travels 20 miles in the same time as another car,
travelling 20 MPH faster, covers 30 miles. How long does
Answer Figures the journey take?
(1) 31 minutes (2) 29 minutes
(3) 30 minutes (4) 28 minutes
42. One statement is given followed by two conclusions. You
have to consider the statement to be true even if it seems to
(1) (2) (3) (4) be at variance from commonly known facts. You have to
decide which of the given conclusions, if any, follow from
DIRECTIONS (Qs. 36 - 40) : In the questions that follow, four
the given statement.
figures are given, out of which one does not belong to the group.
Statement:
Identify the odd one out.
Go by aeroplane to reach Delhi from Chennai quickly.
36. Conclusions:
I. Chennai and Delhi are connected by aeroplane service.
II. There is no other means of going from Chennai to Delhi.
A B C D (1) Only conclusion I is implicit
(1) C (2) A (2) Only conclusion II is implicit
(3) B (4) D (3) Both conclusions I and II are implicit
(4) Neither conclusion I nor II is implicit
37. 43. Which number space indicated Indian teachers who are also
advocates?

A B C D Indians Advocates
1 2 7
(1) B (2) A
(3) D (4) C 3
4 6
5
38.
Teachers

A B C D (1) 2 (2) 3
(1) B (2) A (3) 4 (4) 6
(3) C (4) D

SPACE FOR ROUGH WORK


Mock Test-2 MT-29

44. I have a few sweets to be distributed. If I keep 2, 3 or 4 in a 49. Find the correct option for the mirror image for the following
pack, I am left with one sweet. If I keep 5 in a pack, I am left word:
with none. What is the minimum number of sweets I have to
pack and distribute ?
(1) 25 (2) 37

45.
(3) 54 (4) 65
Akhilesh is taller than Sheebu, Aman is not as tall as Akhilesh
WAT E R ?
but is taller than Tejinder. Sheebu is also not as tall as Aman
but is taller than Tejinder. Who is the tallest?
mirror
(1) Akhilesh (2) Sheebu surface
(3) Aman (4) Tejinder
46. Select from the alternative, the box that can be formed by (1)
folding the sheet shown in figure (X) :
(2)

(3)

(4)

50. Find the correct option for the water images below:
STORE
water surface
(X) ?
(1) (2)
(3) (4)
DIRECTIONS (Qs. 51-53): Read the following information carefully
(A) (B) (C) (D) and answer the given questions.
(1) A only (2) A and C only M, A, B, C, Y, and Z are six friends. Each of them has different
(3) A , C and D only (4) A, B, C and D height. Height of the tallest person is 189 cm. A is taller than
47. Which number is on the face 4, if the four different positions
only B. Y is shorter than only two people. C is not the tallest. M
of a dice are as shown in the figures given below.
is neither taller than Z nor shorter than C. The difference between
the height of third shortest and the tallest person is 9. M is
4 2 1 2 taller than Y.
5 5 1 5 4
51. How many persons are taller than C?
6 6 3
(1) Four (2) Three
(3) Two (4) One
(i) (ii) (iii) (iv) 52. Which of the following statement is true if height of B is
(1) 5 (2) 3 176 cm?
(3) 2 (4) 1 (1) The difference between the height of C and B is 4 cm.
48. Two position of a dice are shown below. Identify the number
(2) Height of Y is 155 cm.
at the bottom when the top is 3.
(3) Height of A is 154 cm.
4 1 (4) The difference between the height of Z and B is 10 cm.
6 5 6 3 53. What could be the possible height of M?
(1) 182 cm (2) 156 cm
(1) 2 (2) 4 (3) 170 cm (4) 198 cm
(3) 5 (4) 6

SPACE FOR ROUGH WORK


MT-30 Target NTSE
DIRECTIONS (Qs. 54-58): Each of the questions below consists of In a certain code language:
a question and two statements numbered I and II given below ‘India and Australia relation’ is written as ‘xz mo nk mn’,
it. You have to decide whether the data provided in the ‘India increase in Power’ is written as ‘ij fa rs mn’,
statement are sufficient to answer the question. Read both ‘Relation Power and Inspection’ is written as ‘mo rs xz da’ and
the statements and ‘India relation in Inspection’ is written as ‘xz fa mn da’.
Give answer: 59. What is the code for ‘relation Power’ in the given code
(1) If the data in statement I alone are sufficient to answer language?
the question, while the data in statement II alone are not (1) xz fa (2) xz rs
sufficient to answer the question. (3) da fa (4) rs da
(2) If the data in statement II alone are sufficient to answer (5) None of these
the question, while the data in statement I alone are not 60. What is the code for ‘increase’ in the given code language?
sufficient to answer the question. (1) i j (2) fa
(3) If the data either in statement I alone or in statement II (3) rs (4) da
alone are sufficient to answer the question. (5) None of these
(4) If the data even in both statements I and II together are 61. What is the code for ‘Power’ in the given code language?
not sufficient to answer the question. (1) i j (2) fa
54. What is the distance between point P and Q? (3) rs (4) da
I. Point S is 4m away in east direction from point P. (5) None of these
Point T is in 2m north of point S. DIRECTIONS (Qs. 62 – 65) The following graph shows the
II. Point Q is in north-west of point T. profits and Losses, (in thousands) in a business for the years
55. M, N, O, P, R, and Q are sitting in a circular table. How 2005- 2010. Study the graph and answer question.
many persons are facing opposite to the centre?
I. There is only one person sit between P and Q. R is not
an immediate neighbour of Q and sits immediate left P Profit
of P. L Loss
II. O sits second to the left of R. N is not an immediate 5 P
neighbour of R, but faces outside the centre. O faces 4
Amount (in thousand)

inside the centre. P


56. How is Rita related to Nita? 3 P
Statements: 2
P
L
I. Prem, who has only two kids, Rita and Nita, is the
mother-in-law of Queen, who is sister-in-law of Nita. 1
II. Roma, the sister-in-law of Rita, is the daughter-in-
5
law of Soni, who has only two kids, Rita and Nita.
57. What is the number? 2005 2006 2007 2008 2009 2010
I. The sum of the two digits is 8. The ratio of the two Years
digits is 1 : 3.
II. The product of the two digit of a number is 12. The
quotient of two digits is 3. 62. The amount of maximum profit in the given diagram.
58. A man mixes two types of rice (X and Y) and sells the (1) `2000 (2) `1500
mixture at the rate of Rs. 17 per kg. Find his profit (3) `1000 (4) `4500
percentage. 63. The total amount of loss incurred during 2005-2010 is-
I. The rate of X is Rs. 20 per kg. (1) `3000 (2) `2500
II. The rate of Y is Rs. 13 per kg. (3) `1500 (4) `2000
DIRECTIONS (Qs. 59-61): Study the following information carefully 64. The ratio of the maximum profit to the minimum loss is
and answer the questions given below: (1) 4 : 3 (2) 3 : 4
(3) 9 : 2 (4) 2 : 9

SPACE FOR ROUGH WORK


Mock Test-2 MT-31

65. If the loss is x% of the profit then the value of x is- (1) I only
(1) 15 (2) 25 (2) II only
(3) 20 (4) 19 (3) I and II only
66. A word is represented by only one set of numbers as given (4) All the statements are facts.
in any one of the alternatives. The sets of numbers given 69. Select a suitable figure from the four alternatives that
would complete the figure matrix.
in the alternatives are represented by two classes of letters
as in two matrices given below. The columns and rows of
Matrix-I are numbered from 0 to 4 and that of Matrix-II
are numbered from 5 to 9. A letter from these matrices
can be represented first by its row and next by its column.
E.g., ‘P’ can be represented by 34, 43 etc., and ‘L’ can be
represented by 56, 68 etc. Similarly, you have to identify
the set for the word ‘CREEP’.
Matrix-I Matrix-II

?
0 1 2 3 4 5 6 7 8 9
0 E C P X T 5 R L N O M
1 C P X T E 6 O M R L N
2 P X T E C 7 L N O M R
3 X T E C P 8 M R L N O
4 T E C P X 9 N O M R L

(1) 10, 79, 23, 32, 42 (2) 24, 55, 14, 41, 12 (1) (2) (3) (4)
(3) 33, 86, 32, 13, 43 (4) 42, 98, 41, 00, 34 (1) 1 (2) 2
67. Fact 1: Eyeglass frames cost between $35 and $350. (3) 3 (4) 4
Fact 2: Some eyeglass frames are made of titanium. 70. Select a suitable figure from the four alternatives that
Fact 3: Some eyeglass frames are made of plastic. would complete the figure matrix.
If the first three statements are facts, which of the
following statements must also be a fact?
I. Titanium eyeglass frames cost more than plastic
frames.
II. Expensive eyeglass frames last longer than cheap
frames.
III. Only a few eyeglass frames cost less than $35.
(1) I only (2) II only
(3) II and III only
(4) None of the statements is a known fact.
68. Fact 1: Mary said, “Ann and I both have cats.”
?
Fact 2: Ann said, “I don’t have a cat.”
Fact 3: Mary always tells the truth, but Ann sometimes
lies.
If the first three statements are facts, which of the
following statements must also be a fact?
I. Ann has a cat. (1) (2) (3) (4)
II. Mary has a cat. (1) 1 (2) 2
III. Ann is lying. (3) 3 (4) 4

SPACE FOR ROUGH WORK


MT-32 Target NTSE
71. Statement: If you are an engineer, we have a challenging 77. Find the missing letters from left to right.
job for you. – V
Z
Assumptions:
R K –
I. We need an engineer.
II. You are an engineer. – C F
(1) Only assumption I is implicit (1) JSN (2) JNS
(2) Only assumption II is implicit (3) JRS (4) KRS
(3) Either I or II is implicit 78. In certain language, if 1 is coded as A, 2 as B, 3 as C, and
(4) Neither I nor II is implicit so on, how is FLOWER coded in that code ?
72. Statement: In spite of poor services, the commutators (1) 6121523518 (2) 6121823515
have not complained against it. (3) 6211523518 (4) 6218123515
Assumptions: 79. If the animals which can walk are called swimmers,
I. Generally people does not tolerate poor services. animals which can crawl are called flying, those which
live in water are called snakes, and those which fly in the
II. Complaints sometimes improve services.
sky are called hunters, then what will a lizard be called ?
(1) Only assumption I is implicit
(1) Swimmer (2) Snake
(2) Only assumption II is implicit (3) Hunter (4) Flying
(3) Either I or II is implicit DIRECTIONS (Qs. 80 to 84): Read the following information
(4) Neither I nor II is implicit carefully and answer the questions given below it.
DIRECTIONS (Qs. 73-75): In the following questions, answers I. Five professors (Dr. Joshi, Dr. Davar, Dr. Natrajan, Dr.
are to be based on the diagrams given below, where the triangle Choudhary and Dr. Zia) teach five different subjects
represents doctors, the circle represents players and the (zoology, physics, botany, geology and history) in four
rectangle represents artists. universities (Delhi, Gujarat, Mumbai, and Osmania). Do
not assume and specific order.
8 II. Dr. Choudhary teaches zoology in Mumbai University.
4 3 III. Dr. Natranjan is neither in Osmania University nor in Delhi
7
University and he teaches neither geology nor history.
6 5
2 IV. Dr. Zia teaches physics but neither in Mumbai University
1 nor in Osmania University.
V. Dr. Joshi teaches history in Delhi University.
VI. Two professors are from Gujarat University.
73. Which numbered space in the diagram represents doctors VII. One professor teaches only one subject and in one
who are also players and artists? University only.
(1) 2 (2) 3 80. Who teaches geology ?
(3) 4 (4) 5 (1) Dr Natrajan (2) Dr. Zia
74. Which number represents artists who are also players (3) Dr. Davar (4) Dr. Joshi
only? 81. Which university is Dr. Zia from ?
(1) 4 (2) 6 (1) Gujarat (2) Mumbai
(3) Delhi (4) Osmania
(3) 7 (4) 8
82. Who teaches botany ?
75. Which number represents artists who are neither players
(1) Dr. Zia (2) Dr. Davar
nor doctors? (3) Dr. Joshi (4) Dr. Natrajan
(1) 1 (2) 2 83. Who is from Osmania University ?
(3) 3 (4) 4 (1) Dr. Natrajan (2) Dr. Davar
DIRECTIONS (Qs. 76 to 77): Find the missing number(s) : (3) Dr. Joshi (4) Dr. Zia
76. 11 9 15 7 25 21 84. Which of the following combinations is correct ?
(1) Delhi University - Dr. Zia
(2) Dr. Choudhary - geology
40 176 ? (3) Dr. Davar - Mumbai University
(1) 184 (2) 210 (4) Dr. Natrajan - Gujarat University.
(3) 241 (4) 425
SPACE FOR ROUGH WORK
Mock Test-2 MT-33

85. Pointing towards a man is the photograph, lady said “the 89. Which number represents the group of educated poor boys
father of his brother is the only son of my mother”. who are employed somewhere but do not help in family
How is the man related to day ? business ?
(1) Bother (2) Son (1) 3 (2) 11
(3) Cousin (4) Nephew (3) 2 (4) None of these
86. The town of Paranda is located of Green lake, The town of 90. Which section does number 12 represent ?
Akram is West of Paranda, Tokhada is East of Akram but (1) Uneducated poor boys who do not help in family
West Paranda. Kokran is East of Bopri but West of Tokhada business
and Akram. If they are all in the same district, which town (2) Educated poor boys employed in service
is the farthest West ? (3) Uneducated boys who help in family business
(1) Paranda (2) Kokran (4) Educated poor boys who help in family business.
(3) Akram (4) Bopri 91. Which number represents that section of poor boys who
87. Which of the following diagrams correctly represents the are neither educated nor are in any employment or have
relationship among smokers, bidi smoker ’s cancer any family business ?
patients. (1) 5 (2) 1
(3) 11 (4) 12
92. Which of the following dices is identical to the unfolded
figure as shown here ?
(1) (2) 3 1
(1) 6 4 (2) 3 5

2 3
(3) (4) (3) 6 4 (4) 2 4

DIRECTIONS (Qs. 88 to 91): Read the following information


2
carefully and answer the questions based on them:
The circle represents poor boys, the triangle represents the 5 1
boys who are employed somewhere and the rectangle represents 4 3
those help in the family business. Each section of the diagram
6
is numbered.
93. How many squares does the figure have ?

12
9
3
5 2 1 11
8 6
4

7
(1) 6 (2) 7
(3) 9 (4) 10
88. Which number represents those poor boys who help in DIRECTIONS (Q. 94): In each of the following questions, there is
family business but are not educated or employed a diagram marked (X), with one or more dots placed in it. The
elsewhere ? diagram is followed by four other figures, marked (A), (B), (C)
(1) 2 (2) 3 and (D) only one of which is such as to make possible the
(3) 4 (4) 5 placement of the alternative in each case.

SPACE FOR ROUGH WORK


MT-34 Target NTSE

94.
(1) (2)

(X)
(3) (4)

97. Consider the following three figures, marked X, Y, Z showing


(1) (2) one fold in X, another in Y and cut in Z. From amongst the
answer figures 1, 2, 3, and 4, select the one, showing the
unfolded position of Z.

(3) (4)

X Y Z
95. Study the problem figures marked 1, 2 and 3 carefully and
to establish the relationship between them. From the
answer figures marked 1, 2, 3 and 4, pick out the figure (1) (2)
which most appropriately completes the series.
Problem Figures
(3) (4)

98. Raman remembers that the examination is after 15th May


but before 18th May, while Deep remembers that the
examination is before 21st May but after 16th May. On
(1) (2) (3)
which date of May is the examination?
Answer Figures
(1) 17 (2) 18
(3) 19 (4) 20
(1) (2) 99. In the following question, select the word which cannot
be formed using the letters of the given word.
CARBONATE
(1) CARBON (2) BORN
(3) EARN (4) BOSE
(3) (4) 100. In the given figure, how many white apples are there?
White
18
96. In the following questions problems, a square transparent 37
sheet with a pattern is given. Figure out from amongst 43 Apples
32 36
four alternatives as to how the pattern would appear when
the transparent sheet is folded at the dotted line. 61 52

Sweet
(1) 37 (2) 73
(3) 36 (4) 88
(X)
SPACE FOR ROUGH WORK
Mock Test-2 MT-35

ANSWER SHEET

Name : ..................................School .............................. Test Code............Roll No. : ...................

City : ................... Date : ................... Class : ..................

(1) (2) (3) (4) (1) (2) (3) (4) (1) (2) (3) (4)
1. 35. 68.
2. 36. 69.
3. 37. 70.
4. 38. 71.
5. 39. 72.
6. 40. 73.
7. 41. 74.
8. 42. 75.
9. 43. 76.
10.
44. 77.
11.
45. 78.
12.
46. 79.
13.
47. 80.
14.
48. 81.
15.
49. 82.
16.
50. 83.
17.
51. 84.
18.
52. 85.
19.
53. 86.
20.
54. 87.
21.
55. 88.
22.
23. 56. 89.

24. 57. 90.

25. 58. 91.


26. 59. 92.
27. 60. 93.
28. 61. 94.
29. 62. 95.
30. 63. 96.
31. 64. 97.
32. 65. SPACE FOR ROUGH WORK
98.
33. 66. 99.
34. 67. 100.

Sig. of Invigilator.................................
SPACE FOR ROUGH WORK
2
(Stage 1)

Time : 120 Minutes Max. Marks : 100

INSTRUCTIONS FOR CANDIDATES


Read the following instructions carefully before you answer the questions.
1. There are 100 questions in this test all are compulsory. Each question has 4 choices (1), (2), (3), and (4), out of which
ONLY ONE is correct.
2. Answers are to be given on separate answer sheet provided at the end of this paper. You can tear this answer sheet
along the cut marks provided.
3. If you do not know the answer to any question, do not waste time on it and pass on to the next one. Time permitting,
you can come back to the questions, which you have left in the first instance and attempt them.
4. Since the time allotted for this question paper is very limited you should make the best use of it by not spending too
much time on any one question.
5. For all questions 1-100 put a cross mark (×) on the number of the correct alternative on the answer sheet against the
corresponding question number by using a blue ball point pen. Example

Correct Way : Q.No. Alternatives


1. 1 2 3 4

Wrong Way : Q.No. Alternatives


1. 1 2 3 4
1. 1 2 3 4

6. Rough work can be done anywhere in the booklet but not on the answer sheet/loose paper.
7. Every correct answer will be awarded one mark. There is no negative marking.
8. After completion of this test you can evaluate your performance by comparing your answers with answers provided
in solution booklet.
MT-38 Target NTSE
1. Consider the following statements: 5. Which of the following characteristics does not belong to
(a) Epithelial cells have a lot of intercellular spaces. flowering plants?
(b) Epithelial tissue is the simplest tissue.
(1) Seeds are naked.
(c) Ciliated epithelium helps in removing unwanted
(2) Conducting tissue is well developed.
particles from trachea.
(d) Epithelial layer does not allow regulation of materials (3) Shows nodes and internodes.
between body and external environment. (4) Shows the presence of root hairs.
Which of these statement(s) is/are correct? 6. Consider the following statements :
(1) (b) and (c) (2) (c) and (d) (a) Rise in sugar level in blood stops secretion of insulin
(3) (b) and (d) (4) All are correct by pancreas.
2. Match column I with column II and select the correct answer (b) Cytokinin are present in greater concentration in young
using the codes given below: fruits and seeds.
Column I Column II Which of these statement(s) is/are correct?
(A) Jaundice (p) Loss of motion functions (1) (a) only (2) (b) only
(B) Paralysis (q) Immune system (3) Both (a) and (b) (4) Neither (a) nor (b)
(C) Trypanosoma (r) Vaccination 7. Some dinosaurs had feathers although they could not fly
(D) Body defence against (s) Increase in the bile but birds have feathers that help them to fly. In the context
pathogens pigments in the blood of evolution this means that
(E) Edward Jenner (t) Sleeping sickness (1) reptiles have evolved from birds.
(1) A - (p); B - (s); C - (t); D - (q); E - (r) (2) there is no evolutionary connection between reptiles
(2) A - (s); B - (p); C - (t); D - (q); E - (r) and birds.
(3) A - (s); B - (p); C - (t); D - (r); E - (q) (3) feathers are homologous structures in both the
(4) A - (s); B - (t); C - (p); D - (r); E - (q) organisms.
3. Consider the following statements: (4) birds have evolved from reptiles.
(a) Vaccines are small doses of live, pathogenic viruses or 8. Match the Column-I and Column-II and select the correct
bacteria. option:
(b) Hypersecretion of insulin results in diabetes mellitus. Column I Column II
(c) Malarial infection is characterized by paroxysms of (A) BB × bb (p) 100% Bb
chills, fever, sweating, enlargement of liver and spleen (B) Bb × Bb (q) 25% BB, 50%Bb, 25%bb
etc. (C) BB × BB (r) 100% BB
Which of the above statements is correct? (D) b × bb (s) 100% bb
(1) only (a) (2) only (b) (1) (A) ® (q); (B) ® (p); (C) ® (r); (D) ® (s)
(3) only (c) (4) None of these (2) (A) ® (p); (B) ® (q); (C) ® (r); (D) ® (s)
4. Match the Coloumn I and Coloumn II and select the correct (3) (A) ® (p); (B) ® (q); (C) ® (s); (D) ® (r)
option. (4) (A) ® (p); (B) ® (s); (C) ® (q); (D) ® (r)
Column I Column II 9. An experiment was set up to determine the percentage of
(A) Protozoan (p) Jointed legs water absorbed by raisins. If the mass of dry raisins was
(B) Chordata (q) Spiny skeleton 40 g and mass of wet raisins was 45 g, the percent water
(C) Echinoderms (r) Primitive organism absorbed would be
(D) Arthropoda (s) Notochord present
(1) A – (s); B – (q); C – (p); D – (r) 45 40
(1) ´ 100 (2) ´ 100
(2) A – (s), B – (r), C – (q); D – (p) 40 45
(3) A – (q); B – (r); C – (p); D – (s)
45 - 40 45 - 40
(4) A – (r); B – (s); C – (q), D – (p) (3) ´ 100 (4) ´ 100
40 45

SPACE FOR ROUGH WORK


Mock Test-2 MT-39

10. Match column I with column II and select the correct answer (2) Receptors ® Motor neuron ® Spinal cord ® Sensory
using the codes given below: neuron ® Muscle
Column I Column II (3) Receptors ® Spinal cord ® Sensory neuron ® Motor
(A) Cell wall (p) Site of protein synthesis neuron ® Muscle
(B) Ribosomes (q) External support (4) Receptors ® Sensory neuron ® Spinal cord ® Motor
protection and made of neuron ® Muscle
cellulose 15. On the basis of following features identify the correct option
(C) Endoplasmic (r) Production and segrega- I. They can be identified by their melting or boiling points
reticulum tion of proteins to be II. Their properties are different from their constituents.
secreted. (1) Elements (2) Compounds
(D) Golgi body (s) Storage of water cell sap
(3) Both (1) and (2) (4) Neither (1) nor (2)
(E) Vacuoles (t) Sorting, packaging,
labelling of cell products 16. Four students (A), (B), (C) and (D) independently observed
(1) A - (p); B - (q); C - (r); D - (t); E - (s) the evaporation of water under different conditions, and
(2) A - (q); B - (p); C - (r); D - (s); E - (t) recorded the temperature of water at regular intervals as
(3) A - (q); B - (p); C - (r); D - (t); E - (s)
shown below.
(4) A - (q); B - (p); C - (t); D - (r); E - (s)
11. A student accidentally spilled a few drops of a solution over Placing of experimental Temperature recording
S tudents
the shirt. The area became blue-black because the solution s etup in/under in 1 5 minutes
was (A ) sun increas ed gradu ally
(1) an acid which burnt the patch of cloth. (B) o pen air decreas ed grad u ally
(2) iodine which reacted with starched part of shirt. (C) a fan initially increas ed , then
(3) potassium dichromate which oxidised the piece of shirt. became con s tant
(4) HCl which reacted with metanil pigment of the shirt. (D) a co rner o f th e roo m initially increas ed , then
12. Consider the following statements : gradu ally decreas ed
(a) The amount of usable energy remains constant as it is The correct recording of observations is that of the student:
passed from one trophic level to another. (1) (A) (2) (B)
(b) The energy within an ecosystem is fixed and never (3) (C) (4) (D)
changes. 17. Match column I with column II and select the correct answer
Which of these statement(s) is/are correct? using the codes given below:
(1) (a) only (2) (b) only Column-I Column-II
(Common name) (Chemical name/
(3) Both (a) and (b) (4) Neither (a) nor (b)
Chemical present)
13. Geeta was asked to prepare a temporary mount of onion peel (A) Slaked lime (p) Zinc carbonate
and list the steps. While writing, she wrote the following (B) Baking soda (q) Calcium hydroxide
steps which may not be in proper sequence. The correct (C) Calamine solution (r) Sodium bicarbonate
sequence would be (D) Milk of Magnesia (s) Magnesium
(i) add a few drops of safranine stain and transfer to a hydroxide
slide. (1) A ® (p); B ® (q); C ® (r); D ® (s)
(ii) cover it with a cover slip. (2) A ® (q); B ® (r); C ® (p); D ® (s)
(iii) add a drop of glycerine.
(3) A ® (r); B ® (s); C ® (q); D ® (p)
(iv) take out onion peel.
(v) keep the peel in water in a petridish. (4) A ® (s); B ® (p); C ® (q); D ® (r)
(1) i, ii, iii, iv, v (2) iv, v, iii, ii, i 18. Which of the following is incorrect?
(3) iv, v, i, iii, ii (4) v, iv, i, iii, ii (1) Periods are the horizontal rows of elements
14. Which is the correct sequence of the components of a reflex (2) Elements in the same group have equal valency
arc? (3) Groups have elements with consecutive numbers
(1) Receptors ® Muscles ® Sensory neuron ® Motor (4) Isotopes are elements of the same group
neuron ® Spinal cord

SPACE FOR ROUGH WORK


MT-40 Target NTSE
19. Four students observed the colour and odour of acetic acid (1) (a) and (c) (2) (a) and (d)
in its reaction with sodium hydrogen carbonate. They (3) (a) and (c) (4) (b) and (d)
tabulated their observations as below: 23. Electricity generated from fossil fuels such as coal and crude
oil to run the industries lead to high concentrations of harmful
Colour of Odour of Action with sodium gases in the atmosphere causing ozone depletion and global
Students warming. What does coal and oil have in it that releases as
acetic acid acetic acid hydrogen carbonate
(A) Blue Fruity Gas evolves without oxides into the atmosphere?
bubbles (1) Sulphur (2) Magnesium
(3) Phosphorus (4) Lead
(B) Colourless Smell of Effervescence
vinegar 24. Which of the following is a physical change?
(C) Light green Odourless Gas evolves without A. Hammering of a red hot iron rod to make a flat sheet.
bubbles B. Formation of ice by cooling of water.
(D) Light brown Rotten egg Effervescence C. Rising up of water vapours from sea to make clouds.
D. Heating of charcoal in air to high temperature till it
The correct set of observations is that of student
(1) (A) (2) (B) begins to glow.
(3) (C) (4) (D) (1) A, B and C (2) B, C and D
20. Consider the following statements: (3) C, D and A (4) D, A and B
(a) An oxidation reaction can proceed only in the company 25. Consider the following statements :
of a reduction reaction. Hard water does not give lather with soap because hard
(b) Ionic reactions taking place in aqueous solution are water contains:
generally fast. (a) Calcium bicarbonate
(c) The precipitate of copper sulphide is red in colour. (b) Magnesium bicarbonate
(d) Silver can displace copper from an aqueous solution of (c) Chlorides of calcium and magnesium
copper sulphate. (d) Sulphates of calcium and magnesium
Which of the above statements are correct? Which of these statements are correct ?
(1) (a) and (b) (2) (a) and (c) (1) (a) and (b) (2) (c) and (d)
(3) (b) and (d) (4) (c) and (d) (3) (a), (b) and (c) (4) (a), (b), (c) and (d)
21. Local conditions, such as heavy rainfall or the removal of 26. Which statement is incorrect regarding the following reaction?
plants, may limit the amount of nitrogen, phosphorus, or Cu (s) + 2AgNO3 (aq) ® Cu(NO3)2 (aq) + 2 Ag(s)
calcium available to a particular ecosystem, but the amount (1) This is the example of redox reaction as well as
of carbon available to the system is seldom a problem – displacement reaction.
(1) Organisms do not need very much carbon. (2) Cu is reducing agent, because it reduces the Ag + to
(2) Plants can make their own carbon using water and Ag.
sunlight. (3) Cu is reducing agent because it itself gets reduced and
(3) Plants are much better at absorbing carbon from the soil. oxidises AgNO3
(4) Many nutrients come from the soil, but carbon comes (4) Both (1) and (2) are incorrect
from the air. 27. Consider the following statements:
22. In which of the following conditions, the distance between (a) Elements in a group have same number of valence
the molecules of hydrogen gas would increase? electrons.
(a) Increasing pressure on hydrogen contained in a closed (b) Variation of atomic size along a period show a erratic
behaviour towards the end.
container (c) Electro-positive character decreases on moving down
(b) Some hydrogen gas leaking out of the container a group.
(c) Increasing the volume of the container of hydrogen (d) Electro-negativity in a period increases right from the
gas alkali metal to the inert gas element.
Which of the above statement(s) is/are correct?
(d) Adding more hydrogen gas to the container without (1) Only (a) (2) Only (d)
increasing the volume of the container (3) (b) and (c) (4) (a) and (b)

SPACE FOR ROUGH WORK


Mock Test-2 MT-41

28. A bus travelling the first one third distance at a speed of 10 Column I Column II
km/h, the next one third at 20 km/h and the last one-third at (A) Magnitude of work (p) 20 J
60 km/h. The average speed of the bus is done by F1
(1) 9 km/h (2) 16 km/h (3) 18 km/h (4) 48 km/h
(B) Magnitude of work (q) 30 J
29. Consider the following statements :
(a) Change in momentum is impulse. done by F2
(b) A body is momentarily at rest when it reverses the (C) Magnitude of work (r) 40 J
direction.
(c) For stable equilibrium force has to be zero and potential done by friction
energy should be minimum. (D) Magnitude of increase (s) 50 J
(d) Force required to accelerte a mass in two perpendicular
directions is never same. in K.E of the block
Which of the above statements are correct? (1) (A) ® r, (B) ® q, (C) ® p, (D) ® s
(1) (b) and (c) (2) (a) and (b) (2) (A) ® p, (B) ® r, (C) ® s, (D) ® q
(3) (a), (b) and (d) (4) All of the above (3) (A) ® s, (B) ® p, (C) ® r, (D) ® q
30. In order that a floating object be in a stable equilibrium, its
(4) (A) ® q, (B) ® r, (C) ® p, (D) ® s
centre of buoyancy should be
34. On the basis of following features identify correct option:
(1) vertically above its centre of gravity
I. It is the characteristic of a sound.
(2) vertically below its centre of gravity
II. It distinguishes an acute or a shrill note from a dull or
(3) horizontally in line with its centre of gravity
flat note.
(4) may be anywhere (1) Loudness (2) Quality
31. Two objects of masses m1 and m2 are placed at a distance d.
(3) Pitch (4) Both (1) and (2)
The force of gravitational attraction between them is 35. The minimum size of a room required to hear an echo of
proportional to
sound with a speed of 300 m/s is
m1m2 (1) 16 m (2) 15 m
(1) (2) m1m2 d 2
d2 (3) 14 m (4) 17 m
36. The pupil of the eye changes in size to adjust for
d2 m1m2
(3) (4) (1) objects at different distances
m1m2 d
(2) different amount of light
32. A body of mass 10 kg at rest is acted upon simultaneously
(3) objects of different sizes
by two forces 4N and 3N at right angles to each other. The
(4) different colours
kinetic energy of the body at the end of 10s is
37. On passing through a prism, a parallel beam of sunlight splits
(1) 50 J (2) 100 J
into lights of several colours. Take a combination of two
(3) 125 J (4) 144 J
identical prisms as shown below. A parallel beam of sunlight
33. In the situation shown in the figure, coefficient of friction is incident on the face AD. The emergent light from the face
between the block and ground is 0.2, m = 2 kg, F1 = 5N and BC, consists of A B
F2 = 4N. If the block is displaced by 10 m, match the following. (1) a parallel beam of light of
(g = 10 ms–2) several colours
F2
(2) a divergent beam of lights
of several colours. D C
60º
(3) a parallel beam of white light
(4) a divergent beam of white light.
F1 m

SPACE FOR ROUGH WORK


MT-42 Target NTSE
38. Match column I with column II and select the correct answer tan q
45. From the figure, find the value of and it is given that
using the codes given below: tan f
Column I Column II ‘D’ be the mid point of BC.
A
(A) Electric potential (p) The obstruction offer to
the flow of charge
(B) Electric current (q) The electric work done per q
unit time f
(C) Electric power (r) The rate of flow of
charges
(D) Resistance (s) The electric work done per
unit charge
(1) (A) ® s, (B) ® r, (C) ® q, (D) ® p B D C
(2) (A) ® p, (B) ® r, (C) ® s, (D) ® q
(3) (A) ® s, (B) ® p, (C) ® r, (D) ® q 3 1
(1) (2)
(4) (A) ® q, (B) ® r, (C) ® p, (D) ® s 4 2
39. Power dissipated across the 8W resistor in the circuit shown 1 4
here is 2 watt. The power dissipated in watt units across the (3) (4)
2 7
3W resistor is 1W 3W i1
(1) 1.0 46. In the given figure PQ is a diameter of a circle with cente O
(2) 0.5 i and PT is a tangent at P. QT meets the circle at R. If ÐPOR =
72°, find ÐPTR.
(3) 3.0
Q
(4) 2.0 8W i2
40. Ocean thermal energy is due to
(1) temperature difference at different levels in the ocean
(2) tides in the ocean
O
(3) energy stored by the water waves
72° R
(4) pressure difference at different levels in the ocean
41. The solution of 5x - 1 + x - 1 = 2 is
(1) x = 2, x = 1 (2) x = 2 P T
(3) x = 1 (4) None of these (1) 50° (2) 54°
42. If three points (x1,y1), (x2, y2), (x3, y3) lie on the same line, (3) 27° (4) 108°
y2 - y3 y3 - y1 y1 - y2 47. If n is any positive integer, then the last two digits in the
then + + is equal to
x2 x3 x 3 x1 x1x 2 expression (81)m(121)m – 1 are
(1) 1 (2) – 1 (1) 02 (2) 12
(3) 0 (4) 2 (3) 21 (4) 00
43. For what least value of n, (26n – 62n) is divisible by ? n being 48. If two zeros of the polynomial f (x) = x3 – 4x2 – 3x + 12 are
even + ve.
3 and – 3 , then its third zero is
(1) 100 (2) 27 (1) 4 (2) – 4
(3) 3 (4) – 3
(3) 8 (4) 13
44. If q is an acute angle and tan q + cot q = 2, find the value of a 4 + b4 + c4
tan25q + cot25q + sec2q + cos2q. 49. If a + b + c = 0, then is equal to
9 a 2 b 2 + b 2 c 2 + c2 a 2
(1) (2) 1
2 (1) 4 (2) 2
5
(3) 0 (4) (3) 1 (4) 16
3

SPACE FOR ROUGH WORK


Mock Test-2 MT-43

50. The figure given is made up of a rectangle, 2 identical 56. Sum of n terms of the series
semicircles and a quadrant. 2 + 8 + 18 + 32 + .... is
n (n + 1)
(1) (2) 2n( (n + 1)
2
42 cm n (n + 1)
(3) (4) 1
2
57. If x + y = 1, then x3 + y3 + 3xy = ...............
(1) 0 (2) 1
22 (3) 2 (4) None of these
Find the unshaded area of the figure. (Use p = )
7
58. AB and CD are two perpendicular diameters of a circle with
(1) 1350 cm2 (2) 1154 cm2 centre O, OD is the diameter of smaller circle. If OA = 7 cm,
(3) 1400 cm2 (4) 1260 cm2 then the area of the shaded region in figure, is
51. Find the sum of
B
æ 1ö æ 2ö æ 3ö
çè1 - ÷ø + çè1 - ÷ø + çè1 - ÷ø ..... upto ‘n’ terms
n n n
n n -1
(1) (2)
2 2
D O C
n +1 n (n + 1)
(3) (4)
2 n2
52. One hundred cards are numbered from 1 to 100. The
probability that a card chosen at random has the digit 5 is
19 100 A
(1) (2) (1) 66.5 cm2 (2) 66 cm2
100 19
10 5 (3) 65 cm2 (4) 67.5 cm2
(3) (4)
100 100
59. In figure, O is the centre of the circle, ÐPOQ = 70° and
53. If x = r cos a cos b, y = r cos a sin b and z = r sin a then x2 + PS ^ OQ. Find ÐMQS.
y2 + z2 is equal to
S
(1) r2 (2) r4
O Q
(3) 1 (4) None of these 70° M
54. In a D ABC, D and E are points on the sides
AB an d AC respectively such th at DE || BC. P
If AD = 4x – 3, AE = 8x – 7, BD = 3x – 1 and
(1) 45° (2) 55°
CE = 5x – 3, find the value of x.
(3) 65° (4) 90°
(1) 6 (2) 5 60. Tickets numbered 1 to 20 are mixed up and then a ticket is
(3) 4 (4) 1 drawn at random. What is the probability that the ticket
55. A vertical stick 12 m long casts a shadow 8 m long on the drawn has a number which is a multiple of 3 or 5?
ground. At the same time a tower casts the shadow 40 m 1 2
long on the ground. The height of the tower is (1) (2)
2 5
(1) 12 m (2) 8 m
(3) 60 m (4) 40 m 8 9
(3) (4)
15 20

SPACE FOR ROUGH WORK


MT-44 Target NTSE
61. Cricket was said to represent- (3) Post-War Political System
(1) English racial superiority (4) Pre-war economic system
(2) English sportsman spirit 70. The first movie in India was shot in 1896 by
(3) English sense of enterprise (1) Dada Saheb Phalke
(4) English values of fair play and discipline (2) Harishchandra Bhatwadekar
62. What was Auschwitz famous for? (3) Raj Kapoor
(1) Centre for mass killing during Nazi Germany (4) Prithvi Raj Kapoor
(2) Centre for educating children during Nazi Germany 71. The person who got people from villages ensured them jobs,
helped them settle in cities and provided them money in
(3) Centre for giving military training to the youth during
times of need was known as :
Nazi Germany.
(1) Stapler (2) Fuller
(4) None of the above
(3) Gomastha (4) Jobber
63. Which was the earliest novel in Marathi?
72. Whio wrote a novel titled ‘Tom Brown’s School Days’ which
(1) Kadambari (2) Panchatantra
became popular in 1857?
(3) Yamuna paryatan (4) Ganga (1) Thomas Arnold (2) Kim Hughes
64. Kissa Goi means : (3) Thomas Hughes (4) John Middleton
(1) The art of writing 73. Why is a time tag of two hours from Gujarat to Arunachal
(2) The art of printing Pradesh?
(3) The art of story telling (1) Because of India’s east-west extent
(4) Novels written in the series of books (2) Because of India’s north-south extent
65. Champaran Satyagrah was launched by Gandhiji against- (3) Because of India’s proximity to oceans
(1) High revenue demands (4) Because of India’s landmass
(2) Indigo planters 74. What is mango shower?
(3) Mill owners (1) Pre monsoon rainfall in Karnataka and Kerala
(4) Salt tax (2) Rainfall from June to September
66. Which of the following constitute pastoral communities of (3) Rainfall in winter days
Africa? (4) Rainfall caused by western disturbances
(1) Bedouins, Berbers, Boran, Maasai, Somali, Turkana, 75. ITCZ is the abbreviated form of -
(2) Kurumas, Kurubas, Dhangar, Gujjars, Gaddis. (1) Indian Tropical Convergence Zone
(2) Inter Tropical convergence Zone
(4) Penda, Chena, Milpa,
(3) Inter Tropical Climate Zone
(3) Yerukula, Korava, Karacha,
(4) Inter Temperate Convergence Zone
67. The landlords and farmers decided to buy threshing
76. ________climate is also known as the Tundra climate.
machines to :
(1) Humid (2) Dry Mesothermal
(1) increase profit from land
(3) Monsoon (4) Polar
(2) reduce their dependence on labourers
77. The monsoon reason in India is affected by –
(3) bring more land under cultivation (1) EI – Nino effect
(4) prevent the labourers from toiling day and night (2) CFC releases
68. To discuss the Wavell plan, a conference was convened at (3) Cyclones in the Bay of Bengal
(1) Shillong (2) Shimla (4) Rains in the Himalayan region
(3) Chennai (4) Delhi 78. Which one of the following ports is the deepest landlocked
69. The Bretton Woods system was : and well-protected port?
(1) Post War Military system (1) Chennai (2) Paradip
(2) Post-War international economic system (3) Tuticorin (4) Visakhapatnam

SPACE FOR ROUGH WORK


Mock Test-2 MT-45

79. The drainage basin of which of the following rivers covers (3) The President after an address by the Parliament
parts of Karnataka, Kerala and Tamil Nadu. (4) The speaker
(1) Godavari (2) Krishna 89. Which is the most diverse country in the world according
(3) Kaveri (4) Musi to language?
80. Which state of India, passed the first resolution for Joint (1) Australia (2) U. S. A
Forest Management?
(3) Brazil (4) India
(1) West Bengal (2) Odisha
90. The Horizontal sharing of power is
(3) Bihar (4) Chhattisgarh
(1) Power sharing between different states
81. The Magnitude of poulation growth is :
(2) Power sharing between different organs of the
(1) The number of persons added each year
government
(2) The number of females per thousand males
(3) Power sharing between different levels of government
(3) The total population of an area
(4) Power sharing between different political parties
(4) The rate at which the population increase
91. Strengthening of the institutions and practices of
82. Resources which are found in a region but have not been
democracies implies the following challenge to democracy:
utilised are called
(1) Challenge of expansion
(1) Developed Resources (2) Potential Resources
(2) Foundational challenge
(3) Full Resources (4) Renewable Resources
(3) Challenge of deepening
83. Agriculture practiced only for the family’s own needs is
(4) Structural challenge
known as-
92. Which of the following countries disintegrated due to the
(1) Commercial farming (2) Subsistence farming
ethnic tension between Serbs and Albanians?
(3) Shifting agriculture (4) Horticulture
(1) Sri Lanka (2) Belgium
84. What made countries like Japan rich/developed?
(3) Yugoslavia (4) Bolivia
(1) Scientific inventions and discoveries
93. Which party gave the slogan ‘Save Democracy’ in the Lok
(2) Efficiency and technology evolved by people Sabha elections held in 1977’?
(3) Raw materials and Natural resources (1) Congress Party
(4) None of them (2) Bahujan Samaj Party
85. Tamil Eelam means - (3) Janata Party
(1) Tamil state (2) Tamil court (4) Communist Party of India Marxist.
(3) Tamil government (4) Tamil language 94. The basic idea behind the power sharing of Indian federalism
86. The political party which believes in Marxism-Leninism is – is:
(1) Nationalist Congress Party (1) Centralisation of power
(2) Communist Party of India (2) Distribution of power
(3) Dravida Munnetra Kazhagam (3) Division of subjects
(4) Bahujan Samaj Party (4) Decentralisation of power
87. Who imposed martial law in Poland in December 1981? 95. The activities such as small manufacturing, transport and
(1) Walesa (2) Jaruzelski shopkeeping can be referred as
(3) Pinochet (4) Bachelet (1) non-market activities
88. A judge of the Supreme Court can be removed by- (2) non-farming activities
(1) President of India (3) non-traditional activities
(4) non-economic activities
(2) The Prime minister

SPACE FOR ROUGH WORK


MT-46 Target NTSE
96. In which one of the following state have the land reform (1) Tertiary Sector (2) Primary Sector
measures helped to reduce poverty? (3) Secondary Sector (4) None of the above
(1) Kerala (2) West Bengal 99. How much percentage of a bank's deposits does the bank
(3) Tamil Nadu (4) Gujarat hold as cash?
97. Why is it necessary that banks and cooperatives increase (1) 15% (2) 20%
their lending particularly in rural areas? (3) 10% (4) 12%
(1) To reduce the dependence on informal sources of credit 100. The sectors are classified into public and private sector on
(2) To promote informal sources of credit in urban areas the basis of ________.
(3) To provide high interest (1) employment conditions
(4) None of them (2) the nature of economic activity
98. Using leathers we make shoes, belts etc. It is an example of (3) ownership of enterprises
which sectors? (4) members of workers employed in the firm

SPACE FOR ROUGH WORK


Mock Test-2 MT-47

ANSWER SHEET

Name : ..................................School .............................. Test Code............Roll No. : ...................

City : ................... Date : ................... Class : ..................

(1) (2) (3) (4) (1) (2) (3) (4) (1) (2) (3) (4)
1. 36. 71.
2. 37. 72.
3. 38.
73.
4. 39.
74.
5. 40.
6. 41. 75.
7. 42. 76.
8. 43. 77.
9. 44. 78.
10. 45.
79.
11. 46.
80.
12. 47.
13. 48. 81.
14. 49. 82.
15. 50. 83.
16. 51. 84.
17. 52.
85.
18. 53.
86.
19. 54.
20. 55. 87.
21. 56. 88.
22. 57. 89.
23. 58. 90.
24. 59.
91.
25. 60.
92.
26. 61.
27. 62. 93.
28. 63. 94.
29. 64. 95.
30. 65. 96.
31. 66.
97.
32. 67.
SPACE FOR ROUGH WORK 98.
33. 68.
34. 69. 99.
35. 70. 100.

Sig. of Invigilator.................................
SPACE FOR ROUGH WORK
3
(Stage 2)

Time : 120 Minutes Max. Marks : 100

INSTRUCTIONS FOR CANDIDATES


Read the following instructions carefully before you answer the questions.
1. There are 100 questions in this test all are compulsory. Each question has 4 choices (1), (2), (3), and (4), out of which
ONLY ONE is correct.
2. Answers are to be given on separate answer sheet provided at the end of this paper. You can tear this answer sheet
along the cut marks provided.
3. If you do not know the answer to any question, do not waste time on it and pass on to the next one. Time permitting,
you can come back to the questions, which you have left in the first instance and attempt them.
4. Since the time allotted for this question paper is very limited you should make the best use of it by not spending too
much time on any one question.
5. For all questions 1-100 put a cross mark (×) on the number of the correct alternative on the answer sheet against the
corresponding question number by using a blue ball point pen. Example

Correct Way : Q.No. Alternatives


1. 1 2 3 4

Wrong Way : Q.No. Alternatives


1. 1 2 3 4
1. 1 2 3 4

6. Rough work can be done anywhere in the booklet but not on the answer sheet/loose paper.
7. Every correct answer will be awarded one mark. There is no negative marking.
8. After completion of this test you can evaluate your performance by comparing your answers with answers provided
in solution booklet.
MT-50 Target NTSE
1. Which figure completes the statement?
(3) (4)
is to as is to ?
4. Manoj walked 30 metres towards East, took a right turn and
walked 40 metres. Then he took left turn and walked 30
metres. In which direction is he now from the starting point?
(1) (2)
(1) East (2) South-East
(3) South (4) North-East
5. In the question given below, there are four statements which
(3) (4) are to be taken as truth even if they do not seem to be so.
There are conclusions numbered, I, II, III and IV. Decide
which of these logically follow from the given statements.
2. Fact 1: All hats have brims.
All boys who like Egg also like Milk. Some boys like Coconut.
Fact 2: There are black hats and blue hats. All boys who like Coconut do not like Milk boys who like
Fact 3: Baseball caps are hats. Milk also like Egg.
If the first three statements are facts, which of the following I. Boys who like Coconut also like Egg
statements must also be a fact? II. Boys who like Milk also like Coconut
I. All caps have brims. III. Boys who like Milk do not like Coconut
II. Some baseball caps are blue. IV. Boys who like Egg do not like Coconut
III. Baseball caps have no brims. (1) I and II (2) I and III
(1) I only (3) I and IV (4) III and IV
(2) II only 6. Which number should replace the question mark in third
(3) II and III only figure?
(4) None of the statements is a known fact.
3. Look at the patterns in the squares and understand their
11 14 15 17 19 26
relationship to one another so as to fill in the square with
92 73 76
missing symbols.
15 21 ?
(1) 21 (2) 31
(3) 41 (4) 51
DIRECTIONS (Qs. 7-9): Answer these questions by using the
following diagram.
C
D 1
12 2
? 3

4 5 8 9
7 10 11
B
(1) (2)
A

SPACE FOR ROUGH WORK


Mock Test-3 MT-51

Each square stands for different class. 12. How many persons can speak only English?
(A) Represents Indians (1) 9 (2) 12
(B) Represents Teacher (3) 7 (4) 19
(C) Represents talented students 13. In the question given below, some argument / arguments is/
(D) Represents Students are logical and others are not. Identify the logical argument/
7. How many Indian who are talented students? arguments.
(1) 5 (2) 7 I. Eating lots of green leafy vegetables increases
(3) 10 (4) 12 hemoglobin levels I eat lots of green leafy vegetables,
so my hemoglobin levels is high.
8. How many talented students are teachers ?
II. Eating lots of green leafy vegetables increases
(1) 13 (2) 17
hemoglobin levels, I do not eat green leafy vegetables,
(3) 11 (4) 22
so hemoglobin levels is low.
9. How many students are there, who are not Indians? III. Eating lots of green leafy vegetables increases
(1) 13 (2) 15 hemoglobin levels, My hemoglobin levels is low which
(3) 17 (4) 19 means I don’t eat green leafy vegetables
DIRECTIONS (Qs. 10-12): In the following figure small squares (1) Only I (2) I and II
represents the persons who know English, triangle to those who (3) I and III (4) II and III
know Marathi, big square to those who know Telugu and circle to 14. Consider the following figure:
those who know Hindi. In the different regions of the figures from
1 to 12 are given.

12 English
Marathi 4
3 5
6 7 2
?
9
8 11 Hindi
Which of the following alternatives should replace the
question mark?
10 Telugu

Artist (1) (2)


Doctors
Players

(3) (4)
10. How many persons can speak English and Hindi both the
languages only?
(1) 5 (2) 8 15. Choose the alternative which is closely resembles the water-
(3) 7 (4) 18 image of the given combination.
11. How many persons can speak Marathi and Telugu both? NUCLEAR
(1) 10 (2) 11 (1) (2)
(3) 13 (4) None of these
(3) U (4)

SPACE FOR ROUGH WORK


MT-52 Target NTSE
16. Statements: Prime age school-going children in urban India 24. Select a suitable figure from the four alternatives that would
have now become avid as well as more regular viewers of complete the figure matrix.
television, even in households without a TV. As a result
there has been an alarming decline in the extent of readership
of newspapers.
Based on the above situation the following possibilities are
to be followed.
I. Method of increasing the readership of newspapers
should be devised.
II. A team of experts should be sent to other countries to
study the impact of TV. on the readership of
newspapers. ?
III. Television is an essential for children
(1) Only conclusion I and II follow
(2) Either I or II follows
(1) (2)
(3) None follows
(4) Both II and III follow
17. How many times in a day, are the hands of a clock in straight
line but opposite in direction?
(3) (4)
(1) 20 (2) 22
(3) 24 (4) 48
18. If FUTURE is coded as UFTUER then how will NATURE be 25. Arrange the given words in a meaningful sequence and find
coded ? the correct sequence from the given options :
(1) ANUTER (2) ANTURE (1) Key (2) Door
(3) ANTUER (4) ANUTRE (3) Lock (4) Room
19. In a certain code, RELIEF is written as QFKKDI. How is (5) Switch on
WELCOME written in that code? (1) 5, 1, 2, 4, 3 (2) 4, 2, 1, 5, 3
(1) VDKCNLM (2) VFKENMF
(3) 1, 3, 2, 4, 5 (4) 1, 2, 3, 5, 4
(3) VFKENPD (4) VFKENPF
20. What will be the missing term in the given sequence? 26. Identify the figure that completes the pattern.
SCD, TEF, UGH, ____, WKL
(1) CMN (2) UJI
(3) VIJ (4) IJT
21. Which number comes in place of ?’
8, 6, 9, 23, 87 , ?
(1) 128 (2) 226 ?
(3) 324 (4) 429
22. Select the missing number/letters from the given options.
AB, FG, KL, PQ, ?
(1) TU (2) UV (1) (2)
(3) VW (4) ST
23. Look at this series:
U32, V29, ___, X23, Y20
What number should fill the blank? (3) (4)
(1) W26 (2) W17
(3) Z17 (4) Z26

SPACE FOR ROUGH WORK


Mock Test-3 MT-53

27. Select a suitable figure from the Answer Figures that would Which of the assumptions is implicit in the statement ?
replace the question mark (?). (1) Only I is implicit. (2) Only II is implicit.
Problem Figures: (3) Either I or II is implicit. (4) Neither I nor II is implicit.
31. Identify the conclusion(s) which logically follow(s) from
the given statements
?
(A) In a one day cricket match, the total runs made by a
team were 200.
(A) (B) (C)A (D) (B) Out of these 160 runs were made by spinners.
Answer Figures: Conclusions:
I. 80% of the team consists of spinners.
II. The opening batsmen were spinners.
(1) Only conclusion I follows
(2) Only conclusion II follows
(1) (2) (3)A (4) (3) Both I and II follow
28. Find out from amongst the four alternatives as to how the (4) Neither I nor II follows
pattern would appear when the transparent sheet is folded 32. A watch which gains 5 seconds in 3 minutes was set right at
at the dotted line. 7 a.m. In the afternoon of the same day, when the watch
indicated quarter past 4 o’clock, the true time is:
(1) 6:00 pm (2) 4:30 pm
(3) 4:00 pm (4) 5:00 pm
33. In a school, students are offered subjects in such a manner
that they have to choose at least one subject from
Mathematics, and Biology. Accordingly :
(1) (2) All students who study Mathematics also study Biology
logically implies:
I. There are no students who study Biology and do not
study Mathematics.
(3) (4) II. There are no students who study Mathematics and do
not study Biology.
29. Fact 1: All dogs like to run. III There are no students who do not study Mathematics
Fact 2: Some dogs like to swim. and do not study Biology.
Fact 3: Some dogs look like their masters. IV All students who do not study Biology are students
If the first three statements are facts, which of the following who study Mathematics.
statements must also be a fact? (1) I and II (2) I and III
(3) II and III (4) II and IV
I. All dogs who like to swim look like their masters.
34. How many minimum right turns will you take to reach school
II. Dogs who like to swim also like to sun.
from home ?
III. Dogs who like to run do not look like their masters.
(1) I only (2) II only Home
(3) II and III (4) Both I and II
30. Given below is a statement followed by two assumptions.
“Please drop this letter in the letter box”. An officer tells his
assistant.
Assumptions:
I. The assistant would follow the instructions. School
II. The assistant knows the address where the letter is to (1) 5 (2) 2
be sent. (3) 8 (4) 10

SPACE FOR ROUGH WORK


MT-54 Target NTSE
DIRECTIONS (Qs. 35-39): Study the diagram given below and Circle indicates total population, pentagon is males,
answer each of the following questions. rectangle is singer, and triangle is cricketer.
What do the shaded regions represent in the diagram ?
(1) males who are cricketers
10 25 (2) males who are singers
20 17 15 30 40 (3) females who are cricketers and singers
7 20 (4) males who are neither singers nor cricketers
20
41. Each of these questions given below contains three
elements. These elements may or may not have some inter
Persons who takes tea
linkage. Each group of elements may fit into one of these
Persons who takes coffee diagrams at (1), (2), (3) and (4). You have to indicate the
group of elements which correctly fits into the diagrams.
Persons who takes wine
Which of the following diagrams indicates the best relation
between Judge, Thieves and Criminals ?
35. How many persons who take tea and wine but not coffee?
(1) 20 (2) 17
(3) 25 (4) 15
36. How many persons are there who take both tea and coffee (1) (2)
but not wine ?
(1) 22 (2) 17
(3) 7 (4) 20
37. How many persons take wine ? (3) (4)
(1) 100 (2) 82
(3) 92 (4) 122
42. A man walks 5 km toward south and then turns to the right.
38. How many persons are there who takes only coffee ?
After walking 3 km he turns to the left and walks 5 km. Now
(1) 90 (2) 45 in which direction is he from the starting place?
(3) 25 (4) 20 (1) West (2) South
39. How many persons takes all the three ? (3) North-East (4) South-West
(1) 20 (2) 17 DIRECTIONS (Qs. 43-47): Each of the questions below consists
(3) 25 (4) 15 of a question and two statements numbered I and II given below
40. The figure shows the Gender and Profession (GP) of a it. You have to decide whether the data provided in the statements
number of people. Each shape shows a different GP. are sufficient to answer the question.
Read the question and both the statements and give answer.
(1) If the data in statement I alone are sufficient to answer
the question, while the data in statement II alone are
not sufficient to answer the question.
(2) If the data in statement II alone are sufficient to answer
the question, while the data in statement I alone are
not sufficient to answer the question.
(3) If the data either in statement I alone or in statement II
alone are sufficient to answer the question.
(4) If the data even in both the statements I and II together
are not sufficient to answer the question.

SPACE FOR ROUGH WORK


Mock Test-3 MT-55

43. How many students are there between Kapil and Raghav in 51. The square boxes in the figures below are to be painted with
a row of fifty students? different colours such that no two adjacent boxes (even
I. Kapil is twelfth from the left end and Raghav is diagonally) have same colour. How many minimum colours
seventeenth from the right end. do you need in each case?
II. Kapil is six places away from Deepu, who is twentieth
from the left end.
44. P is in which direction with respect to S?
I. R is 3 km towards north of S. T is 7 Km towards west of
R. P is 3 km towards north of R.
II. P is 4 km towards west of Q, who is 5 m towards south
of R. S is 2 km towards west of R.
45. On which date is A’s Father’s birthday?
I. A’s mother correctly remembers that their A’s father
birthday is after 7th but before 11th December.
II. A correctly remembers that his father’s birthday is after
5th but before 10th December.
46. What was the per cent profit/loss made/incurred by selling (1) (3, 4) (2) (4, 3)
an article for Rs. 24000? (3) (4, 5) (4) (3, 5)
I. The ratio between the selling price and the cost price 52. What is the number in place of ?
of the articles is 5 : 3 respectively. 21F15, 23J13, 25N11, 27R9, 29V7, ?
II. The difference between the cost price and the selling (1) 31T5 (2) 32V9
price is Rs. 9600. (3) 31W7 (4) 31Z5
47. How many marks did Nigam get in Biology? 53. Identify which number does not fit in the sequence?
I. Nigam got 42 marks in English which were half the 445, 221, 109, 46, 25, 11, 4
marks he got in Biology.
(1) 221 (2) 109
II. Nigam’s marks in Biology were 14% of the total marks
(3) 46 (4) 25
he got in all the subjects together.
54. What is the number in place of ‘?’ ?
48. If TEA is coded as 100, then how will you code FAT?
(1) 120 (2) 150 9, 8, 10, 16, 11, ?, 12, 64
(1) 28 (2) 36
(3) 140 (4) 130
(3) 25 (4) 32
49. Rasik walked 20 m towards north. Then he turned right and
walks 30 m. Then he turns right and walks 35 m. Then he 55. A figure is given below, to form a cube, choose from amongst
turns left and walks 15 m. Finally he turns left and walks 15 the alternatives (1), (2), (3), (4), the cubes that are similar to
m. In which direction and how many metres is he from the the cube formed.
starting position?
(1) 15 m West (2) 30 m East 1
(3) 30 m West (4) 45 m East
6 5 2
50. A boy rode his bicycle North-ward, then turned left and
rode 1 km and again turned left and rode 2 km. He found 4
himself 1 km west of his starting point. How far did he ride
northward initially? 3
(1) 1 km (2) 2 km
(3) 3 km (4) 5 km

SPACE FOR ROUGH WORK


MT-56 Target NTSE
62. Find out the numbers that would fit in the second row and
1 4 third row middle and last blank spaces (?) respectively.
(A) (B) 18 23 16
6 5 2 3
17 19 ?
22 ? ?
4 1 (1) 26, 24, 25 (2) 15, 21, 20
(C) (D) (3) 21, 15, 20 (4) 25, 24, 26
3 2 6 2
63. In a code language the following alphabets are coded in a
particular way :
(1) Only (A) (2) Only (A), (B) and (C) A B C D E F G H I O P R S T U
(3) Only (D) (4) Only (A) and (B)
I II „ × # ƒ ã / Ä Ú Ù < > \
56. Which group of letter given in the alternatives will complete
Which word can be decoded from the following?
the sequence ?
Gfe __ ig __ eii __ fei __ gf __ ii I # < >
(1) elfgi (2) figie (1) BOAST (2) TOAST
(3) ifgie (4) ifige (3) GHOST (4) TASTE
DIRECTIONS (Qs. 57-60): Seven friends – A, B, C, D, E, F and G 64. Ram walks 2 km to the East, and then he turns to South and
are sitting in a row facing North. walks 6 km. He again turns to East and walks 2 km. Then he
Some additional information is also given: turns to North and walks 12 km. How far is he from the
starting point?
If and only if D sits adjacent to B, only then C is at one of the
extreme ends of the row. F is sitting adjacent to G. The one sitting (1) 7 km (2) 7.1 km
to the immediate right of A is two places away from D, who is not (3) 7.2 km (4) 7.3 km
sitting at either ends of the row. B is sitting to the right of E. Two 65. Joel and Tom were travelling from town X to town Y which
people are sitting between C and A. E is not sitting adjacent to D. was 210 km apart. Joel set off 1 hour 15 minutes later than
57. Who is sitting at the extreme right end of the row? Tom but arrived 15 minutes earlier. If the average speed of
(1) A (2) B Tom was 42 km/h, find the average speed of Joel.
(3) C (4) D (1) 50 km/h (2) 60 km/h
58. Who is sitting two places to the left of F? (3) 65 km/h (4) 70 km/h
(1) B (2) C 66. A bus driver wants to travel a distance of 200 km. He travels
(3) E (4) Cannot be determined with the speed of 40km/hr. He takes rest at 3 different hotels
59. Who is sitting to the immediate left of B? for 15 minutes, 30 minutes and 15 minutes, respectively. He
(1) E (2) B starts his journey at 9:00 a.m. At what time does he reach his
(3) D (4) A final destination?
60. How many persons are sitting between D and C? (1) 3 : 00 a.m. (2) 3 : 00 p.m.
(1) 1 (2) 2 (3) 3 : 30 a.m. (4) 2 : 30 p.m.
(3) 3 (4) 4 67. Looking at the portrait of a man, Ram said, “His mother is
61. After interchanging ÷ and ×, 10 and 5, which one of the the wife of my father’s son. Brothers and sisters I have
following becomes a correct equation? none”. Whose portrait was Ram looking at?
(1) (30 ÷ 5) × 10 = 24 (2) (30 × 10) ÷ 5 = 60 (1) His grandfather (2) His son
(3) (30 ÷ 10) × 5 = 18 (4) (10 ÷ 30) × 5 = 70 (3) His brother (4) His cousin

SPACE FOR ROUGH WORK


Mock Test-3 MT-57

68. Few cubes are arranged as shown in the figure. How many 71. Which answer figure will complete the question figure?
cubes are unseen? Question Figure:

Answer Figure:

(1) 8 (2) 10
(3) 12 (4) 14
69. In the figure, circle represents business people, triangle the (1) (2) (3) (4)
educated persons and the rectangle stand for persons with 72. Find out from amongst the alternatives as to how the pattern
income more than Rs. 10, 000 per month. The number would appear when the transparent sheet is folded at the
dotted line.
standing for educated business people with income more
Question Figure:
than Rs. 10, 000 per month is

10
Answer Figure:

4
7 5
3
2 4 (1) (2) (3) (4)
73. Which of the answer figures is exactly mirror of given figure?
Question Figure:
(1) 7 (2) 10
(3) 3 (4) 4 654
Answer Figure:
70. Which one of the following diagrams represents the correct
relationship among Table, Chair and Furniture?
(1) (2) (3) (4)
74. IF FAITH is coded as 82731, HABIT is coded as 12573 and
(1) (2) HEALTH is coded as 192431, how can BELIEF be coded as ?
(1) 594598 (2) 594789
(3) 594978 (4) 594798
75. If NATIONAL can be written as LNAANTOI, how can
DOMESTIC be written?
(1) CDIOMTES (2) CDIOTMSE
(3) CTSIMODE (4) CDITEMOD
(3) (4)
76. Vikash said, “That boy is the grandson of my mother’s
husband. I have no brother and sister.” How is the boy
related to Vikash?
(1) Uncle (2) Son
(3) Nephew (4) Cousin

SPACE FOR ROUGH WORK


MT-58 Target NTSE
77. Out of 1865 people, 660 can speak English and 1305 can Answer Figure:
speak Marathi. But, 120 persons can’t speak either language.
Then how many can speak both languages?
(1) 220 (2) 440
(3) 120 (4) 1085
78. Monoranjan walks 10 km towards South of point ‘P’, turns (1) (2) (3) (4)
to his right and walks 4 km. Turn to his right and walks 10 km
and then he turns to his left and covers a distance of 5 km. 84. Select the missing number from the given responses.
How far is he from the point ‘P’?
(1) 1 km (2) 14 km 3 7 10
(3) 14 km (4) 20 km
79. Select the correct combination of mathematical signs to 1 60 4 6 84 1 3 ? 3
replace * signs and to balance the given equation.
5 2 1
7*2*3*5*6
(1) = + = × (2) + × = +
(1) 90 (2) 12
(3) = × + – (4) × = + +
80. If ‘×’ means ‘+’, ‘+’ means ‘÷’, ‘– ’ means ‘×’ and ‘÷’ means (3) 48 (4) 16
‘–’, then 85. Select the missing number from the given responses.
6× 4–5+2÷1=?
6 2 1
(1) 10 (2) 11
0 4 8 5 2 5
(3) 12 (4) 15
81. Raju is facing North. He goes 35 metres ahead, turns left 133 196 ?
and walk 20 metres. He turns right and covers 25 metres,
then turns right to cover 30 metres. In which direction is he 5 3 6 3 4 7
heading? 1 4 3
(1) North (2) South
(1) 535 (2) 451
(3) East (4) West
82. Select the missing number from the given responses. (3) 154 (4) 702
86. Find the missing number at the place of M?
? 1
2 2 2 3 5 1 4 3
8 2
12 3 30 5 –5 –1 M –1

5 3
(1) 12 (2) 9
(3) 7 (4) –12
(1) 10 (2) 12
(3) 13 (4) 15 87. Two positions of a dice are shown below. When three is at
83. A piece of paper is folded and cut punched as shown below the top what number will be at the bottom?
in the question figures. From the given answer figures,
indicate how it will appear when opened?
Question Figure:

(1) 2 (2) 1
(3) 4 (4) 5

SPACE FOR ROUGH WORK


Mock Test-3 MT-59

DIRECTIONS (Qs. 88-90): The pie-chart given below shows Answer Figures:
expenditure incurred by a family on various items and their
savings. Study the chart and answer the questions based on the
pie-chart

(1) (2) (3) (4)


Education 94. The following table shows the distribution of Boys and
70° Girls students of seven different colleges:
Food Others
120° College Boys Girls
36°
(Total – 35300) (Total – 25500)
Savings
60° P 16% 15%
oth
Cl 0°Ho Q 12% 16%
2 using
54° R 10% 12%
S 22% 22%
88. The ratio of expenditure on food to savings is : T 14% 10%
(1) 3 : 2 (2) 10 : 9
U 18% 17%
(3) 3 : 1 (4) 2 :1
89. If the expenditure on educations is ` 1600 more than that on V 8% 8%
housing, then the expenditure on food is: What is the difference between the number of Girls students
(1) ` 12000 (2) ` 6000 from colleges P and T and Boys students from colleges S
(3) ` 3333 (4) ` 7000 and U?
90. If the monthly income is ` 36000, then the yearly savings is: (1) 7855 (2) 7745
(1) ` 70000 (2) ` 72000 (3) 8235 (4) 7825
(3) ` 60000 (4) ` 74000 95. What is the code used for ‘devil’ derived from the given
DIRECTIONS (Qs. 91- 92): Read the following information to coded statements as per a code language?
answer these questions:
I. ‘fire the deal burn’ is written as ‘pan cha ga mo’,
Seven poles A, B, C, D, E, F and G are put in such a way that the
distance between the next two decreases by 1 meter. The distance II. ‘burn are pick stick’ is written as ‘ta ha cum pan’,
between the first two poles, A and B, is 10 meters. III. fire the devil burn’ is written as ‘ga cha she pan’ and
91. What is the distance between the first pole A and the last IV. ‘pick stick you’ is written as ‘ha cum va’.
pole G? (1) she (2) cha
(1) 40 m (2) 45 m (3) ha (4) pan
(3) 49 m (4) None of these
92. If a monkey hops from pole G to pole C, then how much 96. In a certain code language.
distance did it cover? ‘mars in group virtual’ is written as ‘yo al na jo’,
(1) 19 m (2) 22 m ‘mars in indians’ is written as ‘na jo io’,
(3) 26 m (4) None of these ‘indians enter money’ is written as ‘io pe mo’ and
93. In which one of the answer figures will complete the question
‘hammer to the nail money’ is written as ‘pe ca ra da la’.
figure?
Question Figures: Then ‘hammer to the enter’ will be coded as _____
(1) ca ra da mo (2) mo da ra la
(3) ca la mo da (4) Can’t be determined
97. In a certain code, BRAIN is written as *%÷#× and TIER is
? written as $#+%. How is TRAIN written in that code?
(1) $ %×#* (2) $%÷#×
(3) $ %+×+ (4) ÷+×%$

SPACE FOR ROUGH WORK


MT-60 Target NTSE
98. In the given matrix, there are two matrices of 25 cells each 99. How many rectangles are there in the given figure?
containing two classes of alphabets. The columns and rows
of Matrix I are numbered from 0 to 4 and those of Matrix II
from 5 to 9. A letter from these matrices can be represented
first by its row number and next by its column number. For
example ‘B’ can be represented as 00, 14 etc. Similarly ‘M’
can be represented by 55, 67 etc. what from the given (1) 10 (2) 9
alternatives will be the correct code for ‘TRUE’? (3) 8 (4) 7
100. How many dots lie opposite to the face having three dots,
Matrix - I Matrix - II when the given figure is folded to form a cube?
0 1 2 3 4 5 6 7 8 9
0 B D E T O 5 M U I L R
1 D E T O B 6 U L M R I
2 E B O D T 7 I M R U L
3 T O B E D 8 L R U I M
4 O T D B E 9 R I L M U

(1) 24, 77, 56, 03 (2) 41, 86, 99, 23


(3) 30, 95, 87, 20 (4) 03, 58, 78, 11 (1) 2 (2) 4
(3) 5 (4) 6

SPACE FOR ROUGH WORK


Mock Test-3 MT-61

ANSWER SHEET
Name : ..................................School .............................. Test Code............Roll No. : ...................

City : ................... Date : ................... Class : ..................

(1) (2) (3) (4) (1) (2) (3) (4) (1) (2) (3) (4)
1. 35. 68.
2. 36. 69.
3. 37. 70.
4. 38. 71.
5. 39. 72.
6. 40. 73.
7. 41. 74.
8. 42. 75.
9. 43. 76.
10.
44. 77.
11.
45. 78.
12.
46. 79.
13.
47. 80.
14.
48. 81.
15.
49. 82.
16.
50. 83.
17.
51. 84.
18.
52. 85.
19.
53. 86.
20.
54. 87.
21.
55. 88.
22.
23. 56. 89.
24. 57. 90.
25. 58. 91.
26. 59. 92.
27. 60. 93.
28. 61. 94.
29. 62. 95.
30. 63. 96.
31. 64. SPACE FOR ROUGH WORK
97.
32. 65. 98.
33. 66. 99.
34. 67. 100.

Sig. of Invigilator.................................
SPACE FOR ROUGH WORK
3
(Stage 2)

Time :: 90
Time 120Minutes
Minutes Max. Marks : 100

INSTRUCTIONS FOR CANDIDATES


Read the following instructions carefully before you answer the questions.
1. There are 100 questions in this test all are compulsory. Each question has 4 choices (1), (2), (3), and (4), out of which
ONLY ONE is correct.
2. Answers are to be given on separate answer sheet provided at the end of this paper. You can tear this answer sheet
along the cut marks provided.
3. If you do not know the answer to any question, do not waste time on it and pass on to the next one. Time permitting,
you can come back to the questions, which you have left in the first instance and attempt them.
4. Since the time allotted for this question paper is very limited you should make the best use of it by not spending too
much time on any one question.
5. For all questions 1-100 put a cross mark (×) on the number of the correct alternative on the answer sheet against the
corresponding question number by using a blue ball point pen. Example

Correct Way : Q.No. Alternatives


1. 1 2 3 4

Wrong Way : Q.No. Alternatives


1. 1 2 3 4
1. 1 2 3 4

6. Rough work can be done anywhere in the booklet but not on the answer sheet/loose paper.
7. Every correct answer will be awarded one mark. There is no negative marking.
8. After completion of this test you can evaluate your performance by comparing your answers with answers provided
in solution booklet.
MT-64 Target NTSE
1. Consider the following statements regarding the living cell: 5. A biology practical was conducted to visualize the opening
(a) The Golgi apparatus links carbohydrates with proteins of stomata under a microscope. Ravi was a curious science
to form glycoproteins. enthusiast who made an alterartion to the experiment by
(b) In plants, the Golgi complex synthesizes pectin. placing a leaf mount in clearly labelled watch glass
(c) The lysosomes store the hydrolysing enzymes. containing pure water and salt solution respectively.he
Which of these statement(s) is/are correct ? recorded observations under a microscope after regular time
(1) Only (a) and (b) (2) Only (b) intervals. On seeing this, Seema got excited and tried the
(3) Only (c) (4) (a), (b) and (c) same experiment ,however, she forgot to label the watch
glass and took salt water in both and made observations
2. Which among the folllowing statements are true for unisexual after time intervals. The below observation were recorded:
flowers? A. The stomata was opened in one watch glass and
(a) They possess both stamen and pistil. shrunken in another
(b) They possess either stamen or pistil. B. The stomata was opened in both
(c) They exhibit cross-pollination. C. The stomata was shrunken in both
(d) Unisexual flowers possessing only stamens cannot D. The stomata was shrunken initially ,however, with time
produce fruits. it opened in both the samples
(1) (a) and (d) (2) (b), (c) and (d) (1) Observation A was made by Ravi
(2) Observation B was made by Ravi and observation D
(3) (c) and (d) (4) (a), (c) and (d)
was made by Seema
3. Match the column-I and column-II and select the correct (3) Observation A was made by Ravi and Seema
option. (4) Observation D was made by Ravi and Seema
Column I (Cell) Column II (Function) 6. It is said, the Tajmahal may be destroyed due to
(A) Parenchyma (p) Water transport (1) flood in Yamuna river.
(B) Sclerenchyma (q) Provide buoyancy to (2) decomposition of marble as a result of high
aquatic plants temperature.
(C) Tracheids (r) Storage, division (3) air pollutants released from oil refinery of Mathura.
(D) Aerenchyma (s) Mechanical strength (4) All of the above
(1) A – (r) ; B – (s) ; C – (p) ; D – (q) 7. A cross was made between the Tall Red plant and Dwarf
(2) A – (s) ; B – (r) ; C – (p) ; D – (q) white plant as follows:
(3) A – (s) ; B – (r) ; C – (q) ; D – (p) TTRr × ttrr
(4) A – (s) ; B – (q) ; C – (r) ; D – (p) The progenies obtained would be in the ratio:
4. Match column-I with column-II and select the correct answer (1) All Tall Red
using the codes given below : (2) 1 Tall Red: 1 Tall White
Column I Column II (3) 1 Tall Red : 1 Dwarf White
(A) Duodenum (p) Opening of wind pipe (4) 1 Tall red : 1 Dwarf red
(B) Epiglottis (q) Small wind sac 8. Why should we not test for glucose instead of starch in the
(C) Caecum (r) A cartilaginous flap green leaves?
(D) Glottis (s) ‘V’ shaped structure (1) The glucose formed during photosynthesis gets
emerging from the stomach polymerised into starch. So stable product one can
test is starch.
(1) (A) ® s, (B) ® r, (C) ® q, (D) ® p
(2) The glucose formed during photosynthesis gets
(2) (A) ® p, (B) ® q, (C) ® r, (D) ® s polymerised into fructose. So starch gives positive test.
(3) (A) ® s, (B) ® q, (C) ® r, (D) ® p (3) Both (1) and (2)
(4) (A) ® s, (B) ® r, (C) ® p, (D) ® q (4) None of these

SPACE FOR ROUGH WORK


Mock Test-3 MT-65

9. Ram used to be a lazy kid who seldom played on the ground. Which of the above statements are correct?
One fine day he was dragged to the ground by his friend (1) (a) and (c) (2) (a) and (b)
Teja who forced him to play football. He thoroughly enjoyed (3) (b) and (c) (4) (c) and (d)
playing, however, after getting back home he experienced 13. A teacher focussed the slide below under a compound
severe cramps in his muscles. Which of the following is microscope. Which student identified it correctly?
responsible for the cramps?
(1) Build up of CO2 and Water
(2) Build of Lactic Acid due to anaerobic respiration
(3) The muscles wereb painful due to the build up of
ethanol,CO2 and Water
(4) Build up of pyruvate
10. Match column-I with column-II and select the correct answer
using the codes given below :
Column I Column II (1) Sheela identified it as cheek cells.
(A) Jaundice (p) Infectious diseases (2) Madhu identified it as squamous epithelium.
(B) Encephalitis (q) Malaria (3) Balaji identified it as parenchyma.
(C) Immune system (r) Water borne disease (4) Shanti identified it as onion peel.
(D) Liver (s) Penicillin 14. Which of the following statements about transmission of
(E) Immunisation (t) Mosquito bite nerve impulse is correct?
(u) HIV (a) Nerve impulse travels from dendritic end towards
(1) (A) ® r, (B) ® t, (C) ® u, (D) ® q, (E) ® p axonal end.
(2) (A) ® t, (B) ® u, (C) ® r, (D) ® q, (E) ® p (b) At the dendritic end electrical impulses bring about
(3) (A) ® p, (B) ® q, (C) ® t, (D) ® u, (E) ® r the release of some chemicals which generate an
electrical impulse at the axonal end of another neuron.
(4) (A) ® r, (B) ® t, (C) ® p, (D) ® u, (E) ® q
(c) The chemicals released from the axonal end of one
11. Identify a, b, c and d in the table given below:
neuron cross the synapse and generate a similar
TR Tr tR tr electrical impulse in a dendrite of another neuron.
TR Tall Red a Tall Red Tall Red (d) A neuron transmits electrical impulses not only to
Tr Tall Red b Tall Red Tall white another neuron but also to muscle and gland cells.
tR c Tall Red d Dwarf Red Which of the above statements are correct?
tr Tall Red Tall White Dwarf Red Dwarf White (1) (a) and (c) (2) (b) and (d)
(1) a–Tall Red, b–Tall Red, c–Dwarf Red, d–Dwarf White (3) (a), (c) and (d) (4) (b), (c) and (d)
(2) a–Tall Red, b–Tall White, c–Tall Red, d–Dwarf Red 15. A hydrocarbon has a molecular formula as C 6H12 . It
(3) a–Dwarf Red, b–Dwarf White, c–Tall Red, d–Tall White does not react with hydrogen to give C6H14 nor does it
(4) None of these react with chlorine to give C6H12Cl2 . The hydrocarbon
12. Consider the following statements regarding manure: C6H12is
(a) Manure contains large quantities of organic matter and A. A saturated hydrocarbon
small quantities of nutrients. B. An unsaturated hydrocarbon
C. An open chain hydrocarbon
(b) It increases the water holding capacity of sandy soil.
D. A cycloalkane
(c) It helps in draining out excess of water from clayey soil. (1) A and B (2) C and D
(d) Its excessive use pollutes environment because it is (3) D and B (4) A and D
made of animal excretory waste.

SPACE FOR ROUGH WORK


MT-66 Target NTSE
16. A teacher instructed four students A, B, C and D to prepare 20. Calamine is used to reduce the irritating effect of ant bite /
a suspension of chalk in water. The students prepared sting because it reacts with (X) released due to the bite /
suspension in the following manner: sting of ants with (Y) present in calamine. Then (X) and (Y)
l student ‘A’ places a piece of chalk in water. respectively are:
l student ‘B’ places powdered chalk in water. (1) Sodium hydrogen carbonate and formic acid
l student ‘C’ places powdered chalk in water and then (2) Formic acid and zinc carbonate
shakes the mixture vigorously. (3) Acetic acid and common salt
(4) Hydrochloric acid and zinc oxide
l student ‘D’ places powdered chalk in dil. HCl.
21. Three beakers labelled as A, B and C each containing 25 mL
Which of the following students has correct way of
of water were taken. A small amount of NaOH, anhydrous
preparation of suspension?
CuSO4 and NaCl were added to the beakers A, B and C
(1) Student A (2) Student B respectively. It was observed that there was an increase in
(3) Student C (4) Student D the temperature of the solutions contained in beakers A and
17. Match column-I with column-II and select the correct answer B, whereas in case of beaker C, the temperature of the solution
using the codes given below : falls. Which one of the following statement(s) is (are)
Column I Column II correct?
(A) CuFeS2 (p) Distillation (i) In beakers A and B, exothermic process has occurred.
(ii) In beakers A and B, endothermic process has occurred.
(B) ZnCO3 (q) Liquation
(iii) In beaker C exothermic process has occurred.
(C) Tin (r) Calcination (iv) In beaker C endothermic process has occurred.
(D) Mercury (s) Roasting (1) (i) only (2) (ii) only
(1) A – (p), B – (q), C – (r), D – (s) (3) (i) and (iv) (4) (ii) and (iii)
(2) A – (q), B – (r), C – (s), D – (p) 22. Match column-I with column-II and select the correct answer
(3) A – (s), B – (r), C – (q), D – (p) using the codes given below :
(4) A – (r), B – (s), C – (q), D – (p) Column I Column II
(A) Carbon (p) Unsaturated hydrocarbons
18. Consider the following isotopes of chlorine :
35, 37, (B) CnH2n (q) Conductor of electricity
17Cl 17Cl
(C) Graphite (r) Allotropy
Atomic masses of these isotopes are different, while their
chemical properties are same. (D) Ethyl alcohol (s) Fermentation of molasses
In the Mendeleev’s periodic table, isotopes of chlorine was (1) (A) – p, (B) – r, (C) – q, (D) – s
placed: (2) (A) – r, (B) – p, (C) – q, (D) – s
(1) In the different groups (3) (A) – r, (B) – q, (C) – p, (D) – s
(2) In the same group (4) (A) – r, (B) – p, (C) – s, (D) – q
(3) In different periods 23. What is the correct designation of A, B and C respectively
(4) Both (1) and (3) are correct for the size of the particles in the figures given below?
19. During the preparation of hydrogen chloride gas on a humid
day, the gas is usually passed through the guard tube
containing anhydrous calcium chloride. The role of
anhydrous calcium chloride taken in the guard tube is to
A B C
(1) absorb the evolved gas
(1) A = Solution B = Gel C = Suspension
(2) moisten the gas (2) A = Gel B = Solution C = Suspension
(3) absorb moisture from the gas (3) A = Suspension B = Solution C = Gel
(4) absorb Cl– ions from the evolved gas (4) A = Solution B = Suspension C = Gel

SPACE FOR ROUGH WORK


Mock Test-3 MT-67

24. A student takes Cu, Al, Fe and Zn strips separately in four 27. Which of the following statements are usually correct for
test-tubes labelled as I, II, III and IV respectively. He adds carbon compounds? These
10 mL of freshly prepared ferrous sulphate solution to each (a) are good conductors of heat and electricity
test-tube and observes the colour of the metal in each case.
(b) are poor conductors of electricity
(I) (II) (III) (IV)
(c) have strong forces of attraction between their molecules
(d) do not have strong forces of attraction between their
molecules
(1) (a) and (c) (2) (b) and (c)
(3) (a) and (d) (4) (b) and (d)
28. Motion of bodies A and B is depicted by the x-t graph. Now
consider the following statements (I), (II), (III) and (IV) and
select the incorrect option.

Cu Al Fe Zn x
B
He would observe a black residue
X A
(1) I and II (2) I and III
(3) II and III (4) II and IV
25. Consider the following statements: t
(a) The discovery of inert gases later on did not disturb
Mendeleev’s arrangement. (I) A has uniform motion
(b) In his classification of element, Mendeleev was guided (II) B has less velocity than A initially
by two factors. (III) B crossses A at X
(c) In the present periodic table, periodicity in the (IV) A comes to rest at X
properties of elements is related to the periodicity in
(1) Only (I) (2) (II) and (III)
their electronic configurations.
(3) (II), (III) and (IV) (4) All of them
Which of the above statement(s) is/are correct?
(1) Only (a) (2) Only (c) 29. When a net force acts on an object, the object will be
accelerated in the direction of the force with an acceleration
(3) (a) and (b) (4) (a), (b) and (c)
proportional to
26. Somebody wanted to calculate the number of moles of
(1) the force on the object
oxygen atoms comprising of 9.033 × 1023 number of its atoms.
The person further thought to calculate its mass and to find (2) the velocity of the object
the number of moles of hydrogen atoms required to combine (3) the mass of the object
completely with this amount of oxygen to form water. (4) the inertia of the object
The number of moles of oxygen atoms, their mass (in grams) 30. Two spheres of masses m and M are situated in air and the
and the number of moles of hydrogen atoms are
gravitational force between them is F. The space around the
(1) 1.5, 3 and 24 respectively
masses is now filled with a liquid of specific gravity 3. The
(2) 15, 18 and 3 respectively
gravitational force will now be
(3) 0.15, 27,3 respectively
F F
(4) 1.5, 24 and 3 respectively (1) (2) 3F (3) F (4)
9 3

SPACE FOR ROUGH WORK


MT-68 Target NTSE
31. When a solid block is fully immersed in water, the volume of 36. Which of the following ray diagram is correct for image
the water formation in a hypermetropic eye ?
(1) greater than the volume of the block
(2) less than the volume of the block
(1) (2)
(3) equal to the volume of the block
(4) depends upon the manner in which the block is
immersed in t.
32. A ball is dropped from a height of 10m. If the energy of the
(3) (4)
ball reduces by 20% after striking the ground, the ball will
rebound to
(1) 8 m (2) 6 m 37. The resistance of a wire is 'R' ohm. If it is melted and stretched
(3) 10 m (4) 5 m to 'n' times its original length, its new resistance will be :-

33. Bats detect the obstacles in their path by receiving the R R


reflected (1) (2) n2R (3) (4) nR
n n2
(1) infrasonic waves (2) ultrasonic waves 38. On the basis of following features identify the correct option
(3) micro waves (4) radio waves I. They always start from north pole and end at the south
pole of the magnet outside the magnet.
34. The distance between two consecutive compressions or
rarefactions in a wave of wavelength, l is II. They never intersect each other.
(1) Electric lines of force
(1) 2 l (2) l/2
(2) Magnetic lines of force
(3) l / 4 (4) l
(3) Both (1) and (2)
35. The path of a ray of light coming from air passing through a (4) Neither (1) nor (2)
rectangular slab traced by four students are shown by
39. Which part of the solar cooker is responsible for green
figures A, B, C and D. Which one of them is correct?
house effect?
(1) Coating with black colour inside the box
(2) Mirror
(3) Glass sheet
(1) (2)
(4) Outer cover of the solar cooker
40. Consider the following statements :
(a) Our energy requirements increase with our standard of
living.
(b) We need to look for more and more sources of energy
to meet our energy requirements.
Which of the above statement(s) is/are correct?
(3) (4)
(1) Only (a) (2) Only (b)
(3) Both (a) and (b) (4) Neither (a) nor (b)

SPACE FOR ROUGH WORK


Mock Test-3 MT-69

143 47. A park, in the shape of a quadrilateral ABCD, has ÐC = 90º,


1
41. If å k + k +1
= a - b , then a and b are respectively AB = 9 m, BC = 12 m , CD = 5 m, and AD = 8 m.
k =4 How much area does it occupy?
(1) 10 and 0. (2) – 10 and 4 (1) 515.40 m2 (2) 555.40 m2
(3) 10 and 4 (4) – 10 and 0 (3) 151.40 m2 (4) 520.40 m2
42. If 112 = q × 6 + r, then the possible values of r are:
48. Two A.P’s have the same common difference.The difference
(1) 1, 2, 3, 4 (2) 0, 1, 2, 3, 4, 5
between their 100th terms is 111 222 333. What is the
(3) 0, 1, 2, 3 (4) 2, 3, 5
difference between their Millionth terms?
43. If x = 2 + 2 + 2 then value of x3 – 6x2 + 6x – 2 is
1/3 2/3

(1) 0 (2) 1 (1) 111 222 333 (2) 222 111 333
(3) – 1 (4) 3 (3) 333 111 222 (4) 111 333 222
æ 132 ö 49. x1, x2, x3, ..... are in A.P. If x1 + x7 + x10 = – 6 and
44. If log a ç ÷ = 2 log a 13 - log a 5 - x , then x3 + x8 + x12 = – 11, then the value of x3 + x8 + x22 is
ç 3 ÷
è 2 ´5 ø (1) 21 (2) –21
(3) 20 (4) –20
(1) a x = 23/2 (2) xa = 23/2 50. If 4x – 3y = 7xy and 3x + 2y = 18xy, then (x, y) =
(3) ax = 22/3 (4) xa = 22/3
45. In the given figure, O is the centre of a circle and AB is a æ 1 1ö
(1) ç 4,3÷ (2) (3, 4)
diameter. If ÐEOF = 40°, then ÐEDF is è ø

A æ1 1ö
(3) (4, 3) (4) ç 3, 4 ÷
è ø
O 51. Solve : x + 2y + z = 7
E
40° x + 3z = 11
2x – 3y = 1
B D
F Values of x, y and z are respectively
(1) 70° (2) 45° (1) x = 1, y = 2, z = 3 (2) x = 2, y = 3, z = 1
(3) 90° (4) 180° (3) x = 2, y = 1, z = 3 (4) x = 3, y = 2, z = 1
46. In the given figure, the value of p + q is equal to 52. A vertical tower stands on a horizontal plane and is
surmounted by a vertical flag staff of height 5 meters. At
P point on the plane, the angle of elevation of the bottom and
the top of the flag staff are respectively 30° and 60°. The
height of tower is
p 60° T
q (1) 2 m (2) 5m
Q S
(3) 2.5 m (4) 3m
53. Value of q, if sin (q + 36°) = cosq, where q + 36° is an acute
R
angle, is
(1) 120° (2) 125° (1) 90° (2) 36°
(3) 126° (4) 130° (3) 54° (4) 27°

SPACE FOR ROUGH WORK


MT-70 Target NTSE
54. The monthly salaries (in `) of 10 employees of a factory are: 61. Consider the following statements:
12000, 8500, 9200, 7400, 11300, 12700, 7800, 11500, 10320, (a) In French society about 60% of the total land was
8100. The median salary is owned by nobles and other members of the third estate.
(1) ` 9670 (2) ` 9760 (b) The population of France declined from 28 million to 20
(3) ` 6970 (4) ` 7960 million in 1789.
55. Water flows out through a circular pipe whose internal (c) First and second estates participated in revolts against
diameter is 2 cm, at the rate of 6 metres per second into a increasing taxes and food security.
cylindrical tank. The radius of whose base is 60 cm. The rise (d) Montesquieu advocated the principle of distribution
in the level of water in 30 minutes is of power within the government between the legislative,
(1) 3 m (2) – 3 m the executive and the judiciary.
(3) 6 m (4) – 6 m Which of the above statements are correct?
56. In DABC, if ÐA = 50° and ÐB = 60°, then the shortest and (1) (a), (c) and (d) (2) (a) and (d)
largest sides of the triangle are (3) (b) and (c) (4) (b), (c) and (d)
(1) AC and AB (2) BC and AB 62. Match column I with column II and select the correct answer
(3) AC and BC (4) BC and AC using the codes given below :
57. A hollow sphere of internal and external diametres 4 cm and
Column I Column II
8 cm respectively, is melted into a cone of base diametre 8
cm. The height of cone is (A) Ghettos (p) Russian parliament
(1) 4 cm (2) 14 cm (B) Reichstag (q) Jacobins
(3) 16 cm (4) 56 cm (C) Duma (r) German parliament
58. An aeroplane at a height of 600 m passes vertically above (D) Sans Culottes (s) Areas of jews
another aeroplane at an instant when their angles of (1) (A) ® q, (B) ® r, (C) ® s, (D) ® p
elevation at the same observing point are 60º and 45º (2) (A) ® s, (B) ® r, (C) ® p, (D) ® q
respectively. How many metres higher is the one from the (3) (A) ® r, (B) ® s, (C) ® q, (D) ® p
other ? (4) (A) ® s, (B) ® p, (C) ® r, (D) ® q
(1) 286.53 m (2) 274.53 m
63. What was the style of clothing of women during Ist world
(3) 253.58 m (4) 263.83 m war in Europe?
59. In the figure, OACB represents a quadrant of a circle.
The radius OA = 3.5 cm, OD = 2 cm. (1) Blouse and trousers became popular as working uniform
(2) Skirts became shorter.
B
(3) Women stopped wearing jewellery.
C D (4) Bright colours came into vogue.
64. Consider the following statements –
2 cm

(a) Unlike an ordinary labourer who works on the fixed


A wages and free to work, an indentured labourer is
3.5 cm O bound to the employer by contract.
The area of the shaded portion is (b) In 19th century, the concept of indentured labour was
(1) 613 cm2 (2) 6.13 cm2 introduced as a system of slavery.
(3) 316 cm 2 (4) 3.16 cm2 (c) Most Indian indentured labourers came from regions
60. If a two digit number is chosen at random, what is the where unemployment was high
probability that the number chosen is a multiple of 3? (d) A indentured or bonded labourer always worked
3 29 willingly and recruited by the agents.
(1) (2)
10 100 Which of the above statements are correct ?
1 7
(3) (4) (1) (a), (b) and (c) (2) (b), (c) and (d)
3 25
(3) (a) and (d) (4) (b) and (c)

SPACE FOR ROUGH WORK


Mock Test-3 MT-71

65. Consider the following statements – 70. Which of the following statement is correct with regard to
(a) Women writer wrote about their emotions, identities, Cavour’s contribution to Italian Unification?
the experiences and problems. I. Diplomatic allinace with enemies of Austria
(b) The novels showed how people dressed their forms of II. War with Austrian Bourbons
worship, beliefs and practices. III. Diplomatic allinace with France in 1859 and
(c) Use of print technology, circulated libraries, strengthening Sardinia Piedmont
innovations in marketing and serialisation increased
IV. Defeated the bourbon Kings.
affordability.
(d) The novels created a feeling of connection with the (1) I and III (2) Only III
fate of rural communities. (3) I, IV and II (4) I and IV
Which of the above statements are correct ? 71. What was the Ordinance 10?
(1) (a), (b) and (c) (2) (b), (c) and (d) (1) Communist led government
(3) None of them (4) All of them (2) The coup that overthrew Bao’s regime
66. With reference of Act of Union 1707, consider the following (3) French law that permitted Christianity but outlawed
statements- Buddhism
(a) It resulted in the formation of the ‘United Kingdom of (4) The reason for North and South Vietnam
Great Britain. 72. Tsar dismissed the first Duma within 75 days of its election
(b) It gave England control over Scotland. because:
(c) It was an agreement between England and Ireland (1) he was incapable of taking good decisions
(d) It was an agreement between England and Scotland. (2) the Tsar did not want anyone to question his authority
Which of the above statements are correct? (3) the term of the first Duma was only 74 days
(1) (a) and (b) (2) (a), (b) and (d) (4) After 1905 most committees and unions worked
(3) (b), (c) and (d) (4) (b) and (c) officially.
67. Consider the following statements : 73. Consider the following statements-
(a) The Second World War was fought between Axis (a) The cold weather season starts from November and
powers and Allies. stays till February
(b) It was fought between 1914 to 1918.
(c) The World War saw death and destruction on a large (b) Monsoon climate influences agricultural crops.
scale. (c) Sudden rise in temperature in summer affects partially
(d) The Soviet union also emerged as a dominant power. grown foodgrains.
Which of these statement(s) is/are correct? (d) Jet stream is moving air from high pressure to low
(1) (a), (c) and (d) (2) (a) and (b) pressure areas.
(3) (b) and (d) (4) (a), (b) and (c) Which of the above statements are correct?
68. Arrange the following social reform organization
chronologically on the basis of their foundation. (1) (a) and (b) (2) (a), (b) and (c)
A. Arya Samaj (3) (c) and (d) (4) (b), (c) and (d)
B. Prarthana Samaj 74. On the basis of following features identify the correct option-
C. Veda Samaj
I. It links Delhi-Kolkata-Chennai- Mumbai.
D. Sree Narayan Dharma Paripalana (SNDP) Yogam
(1) A, D, C, B (2) A, B, C, D II. It connects all major cities by six lane super highways.
(3) D, C, B, A (4) C, B. A. D III. It reduces time and distance.
69. Which of the following statements is correct about ‘Bastille
IV. It is maintained by National Highway Authority of
storming’.
I. It was a fortress prison in France India.
II. It symbolised despotic powers of the French King (1) National Highways.
III. French commonmen used to hate the Bastille. (2) State Highways
IV. All the above
(1) I and II (2) III and IV (3) Golden Quadrilateral Super Highways
(3) I and III (4) Only IV (4) Grameen Sadak Yojna

SPACE FOR ROUGH WORK


MT-72 Target NTSE
75. On the basis of following features, identify the correct option– 80. Which of the following statements regarding natural
I. The FCL was set up in 1961. vegetation are correct?
II. It is related to petro-chemicals. (I) Natural vegetation is classified into forests, grasslands
III. Naptha is used as basic raw material. and shrubs.
IV. Sindri, Nangal, Trombay, Durgapur, Kota are famous (II) The changes in the type of natural vegetation occur
for this industry mainly because of the changes of climatic condition.
(1) Pharmaceuticals (2) Cement (III) The type and thickness of natural vegetation varies
(3) Aluminium smelting (4) Fertilizers from place to place.
76. Match column I with column II and select the correct answer (IV) Human beings grow a variety of natural vegetation.
using the codes given below: Select the correct answer using the codes given below:
Column I Column II Codes:
(A) Thorn forests (p) Sundari (1) I, II and III (2) I and II
(B) Mangrove forests (q) Babool (3) I, II, III and IV (4) IV only
81. Consider the following statements and choose the correct
(C) Dry Dedciduous (r) Pine option from the choice given below
forests Statement I : India has a strategic location along the trans-
(D) Montane forests (s) Peepal Indian ocean routes.
(1) (A) ® s, (B) ® r, (C) ® s, (D) ® p Statement II : India is centrally located at the head of the
Indian ocean.
(2) (A) ® r, (B) ® q, (C) ® p, (D) ® s (1) Statement I is true but statement II is false
(3) (A) ® q, (B) ® p, (C) ® s, (D) ® r (2) Both Statement I and Statement II are true but
(4) (A) ® p, (B) ® s, (C) ® r, (D) ® q Statement II is not the correct explanation of Statement
77. Consider the following statements - I.
(a) Individual resources are owned privately by individuals. (3) Both the Statements are false
(b) At present, petroleum is the only multipurpose fuel (4) Both Statement I and Statement II are true and
which is the most important source of thermal energy. Statement II is the correct explanation of Statement I.
(c) We have abundance of hydro-electricity resources. 82. Which of the following statement is/are correct with regard
(d) Thermal power is not a permanent source of electricity to Western Coastal Plains.
as the coal, petroleum and natural gas deposits are I. Western Coastal Plains is a belt of plain region lying
exhausting. towards west between the Western Ghats and Arabian
Which of the above statements are correct? Sea.
II. Western plain receives less rainfall
(1) (a), (b) and (c) (2) (b), (c) and (d)
III. These are comparatively wider and very fertile
(3) (a), (b) and (d) (4) All of the above
because the rivers here deposit their sediments during
78. The most important objective behind the establishment of delta formation.
biosphere reserves is- IV. These plains are divided into Konkan Coast, Kannad
(1) Protection of endangered species of animals Coast and Malabar Coast
(2) Preservation of tigers and Crocodiles (1) I and II (2) I, II and III
(3) Preservation of the overall biotic diversity (3) I and IV (4) II and III
(4) Optimum utilization of the forest products 83. Lakes are of great value to human beings. Which of the
79. Which of the following is correct definition of bhangar ? following statements about lakes given below is incorrect.
(1) Wet, swampy and marshy region of northern plain. I. Helps to regulate the flow of rivers.
II. Can be used for developing hydel power.
(2) The new alluvial soil forms a plain after flood.
III. It results in flooding.
(3) The older alluvial soil lie above the flood plains of the IV. They do not moderate the climate of the surroundings.
rivers and present a terrace like feature. (1) I and II (2) I, III and IV
(4) The land between two water bodies. (3) III and IV (4) I and III

SPACE FOR ROUGH WORK


Mock Test-3 MT-73

84. Match Column I with Column II and select the correct option 88. Which one of the following pairs is not correctly matched ?
using the code given below : (1) Equality before law – Guaranteed to citizens & non
Column I Column II citizens
A. Tropical moist (p) Arunachal Pradesh
(2) Altering the name of the state – Power of a state
Deciduous
B. Tropical dry Deciduous (q) Sahyadris legislatures
C. Alpine (r) Middle Ganga Plain (3) Creating a new state – Power of parliament
D. Tropical wet evergreen (s) Tarai (4) Equality of opportunity in Public employment –
(1) A-s, B-p, C-q, D-r (2) A-s, B-r, C-p, D-q Guaranteed only to Indian citizens
(3) A-r, B-s, C-q, D-p (4) A-s, B-r, C-p, D-q 89. Which of the following statements is not correct so far as
85. Consider the following statements- the judicial system of India is concerned?
(a) Democracy promotes equality and freedom of
expression. (1) The Supreme Court is at the apex of the judicial
hierarchy of the country.
(b) In non-democratic countries people cannot freely
choose their leaders. (2) The courts also play the role of the custodian of
(c) The relationship between different countries has fundamental rights and do not allow the governments
become more democratic than ever before to become autocratic.
(d) Now a days there is not any military government in the (3) Although according to the Constitution, there can be
world. a direct appointment to the Supreme Court from the
Which of the above statements are correct? Bar, usually the judges are promoted from the High
(1) (a), (b) and (c) (2) (a) and (b) Courts only
(3) (c) and (d) (4) (b), (c) and (d) (4) There is a quasi-federal judicial set up in the country
86. Consider the following statements – with each state having a High Court and the Supreme
(a) Some election laws have been made to ensure that Court at the centre.
every political party and candidates get a fair and equal 90. Which of these statements is/are correct about the
chance to compete. International Monetary Fund (IMF).
(b) According to codes of conduct of election campaign, I. It is one of the biggest moneylenders for any country
any parties or candidate can use any place of worship in the World.
for election propaganda. II. Less than 50 percent of the voting power is in the
(c) People who are eligible to vote can go to electoral office hands of 5 countries.
to vote. III. Its 188 (on September, 2012) member states don’t have
(d) Elections are conducted by Election Commission. an equal voting rights.
IV. The vote of each country is weighed by how much
Which of the above statements are correct ?
money it has contributed to the IMF.
(1) (a), (b) and (c) (2) (a) and (d) (1) I, II and III (2) I, II and IV
(3) (b) and (d) (4) (b), (c) and (d) (3) I and IV (4) II, III and IV
87. Consider the following statements – 91. Consider the following statement about the pressure groups
(a) Communalism Means to distinguish people on the basis and political parties. Which of the statements given below
of religion. are correct.
(b) Poverty in India is a great challenge to the successful I. Pressure groups are the organised expression of
working of democracy. interests and views of special social sections.
(c) Casteism plays important role in the development of II. Pressure groups take positions on political issues.
Indian democracy. III. All pressure groups are political parties.
(d) Social and economic inequality is an important factor IV. Pressure groups and political parties do not seek to
for the successful working of democracy. mobilise people.
Which of the above statements are correct ?
(1) I, II and III (2) III and II
(1) (a), (b) and (c) (2) (a) and (b)
(3) I, II, III and IV (4) I and II
(3) (c) and (d) (4) None of the above

SPACE FOR ROUGH WORK


MT-74 Target NTSE
92. Consider the following statement. Which is not the feature (B) Accident due to faulty (q) Right to information
of Indian democracy. engine in scooter
I. India has largest number of voters in the World. (C) Filing a case in Consumer (r) Right to redressal
II. In India, the Election Commission is not an
Court
independent body.
III. In India, the losing parties refuse to accept the electoral (D) Choosing the best product (s) Right to choose
verdict. (1) (A) ® q, (B) ® p, (C) ® r, (D) ® s
IV. In India, everyone has the voting right above the age (2) (A) ® p, (B) ® q, (C) ® r, (D) ® s
of 21. (3) (A) ® s, (B) ® r, (C) ® q, (D) ® p
(1) I, II and III (2) II, III and IV (4) (A) ® r, (B) ® q, (C) ® p, (D) ® s
(3) I, II and IV (4) I, II, III and IV
98. A train ticket booked by Ravi in one of the super fast trains.
93. How is an MNC defined?
(1) A MNC is a company that owns or controls production The train got delayed for long hours without any reason in
in more than one district in a country this situation.
(2) A MNC is a company that owns or controls production (1) he cannot approach the consumer court as train delays
in more than one state in a country can happen sometime.
(3) A MNC is a company that owns or controls production (2) he can file a complaint in Railway office and claim refurd
in more than one nation of the ticket amount
(4) All of the above (3) he can approach the consumer court for deficiency in
94. Which of the following is an example of Formal sector loan? service and claim refund of super fast charges as
(i) Bank Loan
(ii) Loan from friends damage
(iii) Loan from cooperatives (4) he can cancel the ticket without paying cancellation
(1) Only (i) (2) Only (ii) charges to Railways.
(3) Only (iii) (4) Only (i) and (iii) 99. On the basis of following features, identify the correct
95. What are the functions of RBI? option.
(1) Supervises the formal sources of loan (I) It was launched in December 2000.
(2) Monitors whether banks actually maintain the cash
(II) Under the scheme one crore of the poorest among the
balance or not
BPL families covered under the targeted PDS were identified.
(3) Ensures that the banks give loan not just to profit
making businesses and traders but also to small scale (III) Poor families were identified by the respective state
industries and borrowers too. rural developments through a BPL survey.
(4) All of them (IV) In 2004, 2 crore families have been covered under this
96. Consider the following statements- scheme.
(a) Migration is the movement of people across the regions (1) Antyodaya Anna Yojna
and territories.
(b) The rural-urban migration has resulted in a decline in (2) National Food for Work programme
the percentage of population in cities. (3) Poverty Alleviation programme
(c) Migration can be internal as well as external.
(4) Targeted Public Distribution Systems.
(d) Migration is the important determinant of population
change. 100. What do you understand by Social Exclusion?
Which of the above statements are correct? (1) It is a process through which individuals or groups are
excluded from facilities, benefits and opportunities that
(1) (a) and (d) (2) (a), (c) and (d)
others enjoy
(3) (b) and (c) (4) (b), (c) and (d) (2) It is a process through which every individual is given
97. Match column-I with column-II and select the correct answer a sense of security
using the codes given below :
(3) It is a process through which a group can be determined
Column I Column II
'untouchables'
(A) Availing details of (p) Right to safety
ingredients of a product (4) None of them

SPACE FOR ROUGH WORK


Mock Test-3 MT-75

ANSWER SHEET

Name : ..................................School .............................. Test Code............Roll No. : ...................

City : ................... Date : ................... Class : ..................

(1) (2) (3) (4) (1) (2) (3) (4) (1) (2) (3) (4)
1. 35. 68.
2. 36. 69.
3. 37. 70.
4. 38. 71.
5. 39. 72.
6. 40. 73.
7. 41. 74.
8. 42. 75.
9. 43. 76.
10.
44. 77.
11.
45. 78.
12.
46. 79.
13.
47. 80.
14.
48. 81.
15.
49. 82.
16.
50. 83.
17.
51. 84.
18.
52. 85.
19.
53. 86.
20.
54. 87.
21.
55. 88.
22.
23. 56. 89.

24. 57. 90.

25. 58. 91.


26. 59. 92.
27. 60. 93.
28. 61. 94.
29. 62. 95.
30. 63. 96.
31. 64. SPACE FOR ROUGH WORK 97.
32. 65. 98.
33. 66. 99.
34. 67. 100.

Sig. of Invigilator.................................
SPACE FOR ROUGH WORK
4
(Stage 2)

Time : 120 Minutes Max. Marks : 100


Time : 90 Minutes

INSTRUCTIONS FOR CANDIDATES


Read the following instructions carefully before you answer the questions.
1. There are 100 questions in this test all are compulsory. Each question has 4 choices (1), (2), (3), and (4), out of which
ONLY ONE is correct.
2. Answers are to be given on separate answer sheet provided at the end of this paper. You can tear this answer sheet
along the cut marks provided.
3. If you do not know the answer to any question, do not waste time on it and pass on to the next one. Time permitting,
you can come back to the questions, which you have left in the first instance and attempt them.
4. Since the time allotted for this question paper is very limited you should make the best use of it by not spending too
much time on any one question.
5. For all questions 1-100 put a cross mark (×) on the number of the correct alternative on the answer sheet against the
corresponding question number by using a blue ball point pen. Example

Correct Way : Q.No. Alternatives


1. 1 2 3 4

Wrong Way : Q.No. Alternatives


1. 1 2 3 4
1. 1 2 3 4

6. Rough work can be done anywhere in the booklet but not on the answer sheet/loose paper.
7. Every correct answer will be awarded one mark. There is no negative marking.
8. After completion of this test you can evaluate your performance by comparing your answers with answers provided
in solution booklet.
MT-78 Target NTSE
1. Which figure completes the statement?

A B C D
A

(1) (2) (3) (4) (5) 5. In an organization of pollution control board, engineers are
represented by a circle, legal experts by a square and
2. Fact 1: Robert has four vehicles. environmentalist by a triangle. Who is most represented in
Fact 2: Two of the vehicles are red. the board as shown in the figure ?
Fact 3: One of the vehicles is a minivan.
If the first three statements are facts, which of the following
statements must also be a fact?
I. Robert has a red minivan.
II. Robert has three cars.
III. Robert’s favourite colour is red. (1) Environment
(1) I only (2) Legal Expert
(2) II only (3) Engineer with legal background
(3) II and III only (4) Environmentalist with Engineering background
(4) None of the statements is a known fact. Directions (Qs. Nos. 6 to 10): Answers these questions based
3. Fact 1: Mary said, “Ann and I both have cats.” on the diagram given below:

Fact 2: Ann said, “I don’t have a cat.”


1 Christians
Fact 3: Mary always tells the truth, but Ann sometimes lies.
12 2 Professionals
If the first three statements are facts, which of the following
statements must also be a fact? Females
I. Ann has a cat. 5 8 9
II. Mary has a cat. 7 10 11
III. Ann is lying. 4 3
(1) I only Asians
(2) II only
(3) I and II only 6. Asian Non-Christian Females who are Professional are
(4) All the statements are facts. represented by
4. How many blocks touch block A? (1) 3 (2) 10
(1) 4 (2) 3 (3) 8 (4) 11
(3) 5 (4) 6

SPACE FOR ROUGH WORK


Mock Test-4 MT-79

7. Asian Females who are neither Professional nor Christians 15. If ‘P + Q’ means ‘P is the father of Q’, ‘P × Q’ means P is the
are denoted by brother of Q’; ‘P – Q’ means ‘P is the mother of Q’, then
(1) 6 (2) 9 which of the following is definitely true about C – A + B?
(3) 10 (4) 3 (1) B is the son of A (2) A is the son of C
8. Non-Asian Professional Christian males are represented by (3) B is the father of A (4) C is the mother of B
(1) 10 (2) 11 16. Fact 1: Jessica has four children
(3) 12 (4) 9 Fact 2: Two of the children have blue eyes and two of the
children have brown eyes.
9. Christian Females who are Non-Professional and Asian are
represented by Fact 3: Half of the children are girls.
(1) 5 (2) 10 If the first three statements are facts, which of the following
(3) 9 (4) None statements must also be a fact?
I. At least one girl has blue eyes.
10. The shaded portion depicts
II. Two of the children are boys.
(1) Asian Professional Christian males
III. The boys have brown eyes.
(2) Non-Asian Professional female Christians (1) I only
(3) Non-Christians Professional female Asians (2) II only
(4) Non-Asian Unprofessional female Christians. (3) II and III only
11. M and N have children A and B. F is spouse of B. D is child (4) None of the statements is a known fact.
of F. P is son in law of N. K is kid of P. Who is male child of Directions (Qs. 17-18): The figure given below is the finding of a
M and N ? survey conducted in an organisation for measuring the behaviour
(1) B (2) A of its employees in terms of four characteristics - Adventurous,
(3) None (4) Can’t be determined Humble, Active and Original. Circle A represents Adventurous,
circle B represents Humble, circle C represents Active, and circle
12. In all family photos, if Ramu is present, his two brothers are
D represents Original.
also present. A certain photo features five members of his
family. Among the people in that photo are Ramu’s parents
with their daughter. Which of these statements is necessarily A B
10 1
true? 11
(1) Ramu is present in the photo
(2) Ramu is absent from the photo 9 7 2
(3) Ramu may or may not be present in the photo 8 6
(4) The two brothers of Ramu are present in the photo 5
C 4 3 D
13. A man is facing north-west. He turns 90o in
the clockwise
direction, then 180o in the anticlockwise direction and then
another 90o in the same direction. Which direction is he 17. Which region represents the people who are Humble,
facing now? Original and Active but not Adventurous?
(1) South (2) South-west (1) 6 (2) 8
(3) South-east (4) East (3) 7 (4) 11
14. Which of the following alternatives will fit in the place of‘?’? 18. Which region represents the people who are Adventurous,
111 13 112 24 113 35 117 ? Original and Active but not Humble?
(1) 46 (2) 57 (1) 7 (2) 8
(3) 68 (4) 79 (3) 6 (4) 11

SPACE FOR ROUGH WORK


MT-80 Target NTSE
19. Given mix of letters in order and/or numbers took after by (B) Apple tree is just next to lemon tree.
four alternatives a, b, c and d. Select the choice which is (C) Raspberry tree is either left to Pomegranate or after.
most nearly takes after the water picture of the given mix.
(D) Papaya or Pomegranate tree is just opposite to
raspberry tree.
C9Y8S6 (1) A & B (2) A & C
OPTIONS: (3) B & D (4) None
22. If ‘ski rps tri’ stands for ‘nice Sunday morning’, ‘teh sti rps’
(1) (2) stands for ‘every Tuesday morning’ and ‘ski ptr qlm’ stands
for ‘nice market place’, which word stands for ‘Sunday’?
(1) ski (2) rps
(3) (4)
(3) tri (4) qlm
20. Find from amongst four choice figures, the one which looks 23. In a certain code language, ‘123’ means ‘bright little boy’,
like the pattern framed when the crystalline piece of paper, ‘145’ means ‘tall big boy’ and ‘637’ means ‘beautiful little
conveying an outline, is folded along the dotted line. flower’. Which digit in that language means ‘bright’?
Crystalline paper: (1) 1 (2) 2
(3) 3 (4) 4
24. If ‘SPEAK’ is coded as ‘URGCM’, then what is the code of
‘LAUGH’?
Choice Figures: (1) DCVIK (2) ODXHJ
(3) DCVIK (4) NCWIJ
25. If ‘FLOWER’ is coded as ‘ZGKTCQ’, then what is the code
(1) (2)
of ‘NATURE’?
(1) HPVRPD (2) HVPRPD
(3) PRDVHP (4) QZNPRS
(3) (4) 26. From the four positions of a dice given below, find the color
which is opposite yellow?
21. Each of the following questions is based on the following
information: Yellow Orange Blue Red

Yellow

Yellow
1. 8-trees ® mango, guava, papaya, pomegranate, lemon,
Blue

Blue

banana, raspberry and apple are in two rows 4 in each Orange Violet Red Rose
facing North and South.
2. Lemon is between mango and apple but just opposite
to guava. (1) Violet (2) Red
3. Banana is at one end of a line and is just next in the (3) Rose (4) Blue
right of guava or either banana tree is just after guava 27. Arrange the words given below in a meaningful sequence.
tree. 1. Word 2. Paragraph
4. Raspberry tree which at one end of a line, is just 3. Sentence 4. Letters
diagonally opposite to mango tree.
5. Phrase
Which of the following statements is definitely true? (1) 4, 1, 5, 2, 3 (2) 4, 1, 3, 5, 2
(A) Papaya tree is just near to apple tree. (3) 4, 2, 5, 1, 3 (4) 4, 1, 5, 3, 2

SPACE FOR ROUGH WORK


Mock Test-4 MT-81

Directions (Qs. 28-31): A, B, C, D, E, F and G are sitting in a row How is the woman in the picture related to Praveen?
facing North : F is to the immediate right of E. E is 4th to the right (1) Cousin (2) Wife
of G. C is the neighbour of B and D. Person who is third to the left (3) None of these (4) Data inadequate
of D is at one of ends. 36. Which of the following alternatives will fit in the place of.?.?
28. Who are to the left of C ? 13, 32, 24, 43, 35, ? , 46, 65, 57, 76
(1) Only B (2) G, B and D (1) 45 (2) 52
(3) G and B (4) D, E, F and A (3) 54 (4) 55
29. Which of the following statement is not true ? 37. Which of the following alternatives will fit in the place of.?.?
(1) E is to the immediate left of D BG, GC, HN, NI, ? P, PM
(2) A is at one of the ends (1) R (2) Q
(3) G is to the immediate left of B (3) L (4) M
(4) F is second to the right of D 38. A monkey climb 30 feet at the beginning of each hour and
30. Who are the neighbours of B ? rests for a while when he slips back 20 feet before he again
starts climbing in the beginning of the next hour. If he begins
(1) C and D (2) C and G his ascent at 8.00 am, at what time will he first touch a flag at
(3) G and F (4) C and E 120 feet from the ground?
31. What is the position of A ? (1) 4 pm (2) 5 pm
(1) Between E and D (3) 6 pm (4) None of these
(2) Extreme left 39. Identify which number does not fit in the sequence ?
(3) Centre 11, 5, 20, 12, 40, 26, 74, 54
(4) Extreme right (1) 5 (2) 20
32. When the digit 5 is on the bottom then which number will be (3) 40 (4) 26
on its upper surface? 40. The number in the place of ‘?’ should be

1 5 3 4 3
5 7 5
4 6 39 51 ?
2 2
6 3 5 4 5 4

(1) 1 B. 3 (1) 35 (2) 37


(3) 4 D. 6 (3) 45 (4) 47
33. There are six persons P, Q, R, S , T and U. S is the father of P 41. The number in the place of ‘?’ should be
and grandfather of U.Q is the brother of T’s husband. R is (1) 40 (2) 48
the sister of U. There are one mother, two fathers & three (3) 30 (4) 24
brothers in the group.How is U related to T ?
(1) Son (2) Friend
(3) Grandson (4) Cousin 9
34. Ankita is B`s sister. C is B`s mother. D is C`s father. E is D`s 102 12
mother. Then, how is Ankita related to D ?
(1) Grandmother (2) Granddaughter
54 18
(3) Son (4) Aunty
35. Pointing to a lady in the photograph, Praveen said, “Her ?
mother has only one grandchild whose mother is my wife.”

SPACE FOR ROUGH WORK


MT-82 Target NTSE
42. A statement with two conclusions is given below followed Answer Figures:
by four alternatives. Choose any one alternative which is
the most suitable.
Statement: He stressed the need to stop the present
examination system and its replacement by other methods
which would measure the real merit of the students.
(1) (2) (3) (4)
Conclusions :
I. Examinations should be abolished. 48. Which one of the Answer Figures shall complete the given
II. The Present examination system does not measure the question figure?
real merit of the students.
Question Figures:
(1) Only I (2) Only II
(3) Either I or II (4) Neither I nor II
43. Ramakant walks northwards. After a while, he turns to his
right and a little after further to his left. Finally, after walking
a distance of one kilometer, he turns to his left again. In
which direction he is moving now?
(1) North (2) South
(3) East (4) West
44. P, Q, R and T answered in a examination. In the result Q was Answer Figures:
immediately followed by “P’ but no one was there after ‘P’.
R was ahead of Q, but could not score as much as T. Who
scored the second highest?
(1) P (2) Q
(3) R (4) T (1) (2) (3) (4)
45. While the group photo of a family was taken, the father was
found to be sitting to the left of his son and right to the 49. A piece of paper is folded and punched as shown below.
grandfather. Mother was sitting to the right of her daughter From the given responses, indicate how it will appear when
but left to grandfather. Who is occupying the central place? opened.
(1) Son (2) Grandfather Question Figures:
(3) Father (4) Mother
46. Which of the following diagrams represents Colour, Cloth
and Merchant?

Answer Figures:

47. Find the missing figure from the given responses.


(1) (2) (3) (4)
Question Figures:
50. Look at the patterns in the squares and understand their
relationship to one another so as to fill in the square with
? missing symbols.

SPACE FOR ROUGH WORK


Mock Test-4 MT-83

(1) (2) (3) (4)

53. If ‘B × C’ means ‘B is the daughter of C’, ‘B + C’ means ‘B is


? the husband of C’ and ‘B – C’ means ‘B is the sister of C’,
then what does ‘M + N – P × Q’ mean?
(1) M is the brother-in-law of Q.
(2) M is the uncle of Q.
(3) M is the son-in-law of Q.
(4) Q is the mother-in-law of M.
(1) (2) (3) (4) 54. In a certain code, BRAIN is written as * % ÷ # × and TIER is
51. Look at the patterns in the squares and understand their written as $ # + %. How is ENTER written in that code?
relationship to one another so as to fill in the square with (1) + × $ + % (2) + × $ % +
missing symbols.
(3) + × + $ % (4) + × $ % +
55. Which one will replace the question mark ?

18 24 18
12 30 16 40 45 27

6 8 ?
? (1) 18 (2) 12
(3) 9 (4) 6
56. Which one will replace the question mark?

10 15 15

(1) (2) (3) (4) 12 4 15 12 21 28 11 ? 23


52. Look at the patterns in the squares and understand their 16
9 20
relationship to one another so as to fill in the square with
missing symbols. (1) 18 (2) 12
(3) 13 (4) 16
Directions (Qs. 57-58): These questions are based on the
information given below.
Eight persons A, B, C, D, E , F, G and H are standing in such way
that C is 20 m apart from B towards West. B is 30 m South with
respect to A. A is 40 m towards West with respect to E. D is 50 m
towards South with respect to E. F is 15 m apart from G towards
? North. H is 20 m towards East with respect to G. F is 40 m towards
West with respect to D.

SPACE FOR ROUGH WORK


MT-84 Target NTSE
57. In which direction B standing with respect to F? 64. Given below is a statement followed by two assumptions”In
(1) North-West (2) North order to bring punctuality in our office, we must provide
(3) North-East (4) Cannot be determined conveyance allowance to our employees.” – In charge of a
company tells Personnel Manager.
58. If one more person I is standing towards South-West with
Assumptions:
respect to D, then in which direction is H, standing with
respect to I? I. Conveyance allowance will not help in bringing
punctuality.
(1) South-West (2) North-East
II. Discipline and reward should always go hand in hand.
(3) North-West (4) Cannot be determined
Which of the assumptions is implicit in the statement?
Directions (59-61): Read the information given below and answer
(1) Only assumption I is implicit
the questions.
(2) Only assumption II is implicit
The age and height of six children in a class are as follows (3) Either I or II is implicit
(i) Amit is taller and older than Tanu but shorter and (4) Neither I nor II is implicit
younger than Shruti.
65. How many minimum right turns will you take to reach office
(ii) Bharat is taller than Chander who is not as tall as Tanu. from home?
(iii) The oldest is the shortest.
(iv) The youngest would be fourth if the children stood in Home
a line according to their height one started counting
from the tallest.
(v) Bharat is younger than Ritu but older than Chander Office
who is older than Shruti.
59. Who among them is the tallest?
(1) Tanu (2) Chander
(3) Shruti (4) Data inadequate
60. Who is older than Tanu but younger than Shruti? (1) 4 (2) 5
(1) Ritu (2) Bharat (3) 6 (4) 7
(3) Amit (4) Data inadequate Direction (Qs. 66-70): In each of the questions below consists of
61. Which of the following statements is definitely true? a question and two statements numbered I and II given below it.
(1) Bharat is the most old person You have to decide whether the data provided in the statements
(2) Tanu has the maximum height are sufficient to answer the question. Read both the statements
and
(3) Amit is older than Bharat
Give answer:
(4) Ritu is the shortest
(A) If the data in statement I alone are sufficient to answer
62. If in a code language PARENT is written as TNERAP, how the question, while the data in statement II alone are
is CHILDREN written in that code? not sufficient to answer the question
(1) NERLIDCH (2) NERDCLIH (B) If the data in statement II alone are sufficient to answer
(3) NERDLIHC (4) NERCDLIH the question, while the data in statement I alone are
63. In the following question, which one set of letters when not sufficient to answer the question
sequentially placed at the gaps in the given letter series (C) If the data either in statement I alone or in statement II
shall complete it? alone are sufficient to answer the question
b _ abbc _ bbca _ bcabb _ ab (D) If the data given in both statements I and II together
are not sufficient to answer the question
(1) acaa (2) acba
(3) cabc (4) cacc 66. What will be the total weight of 10 poles, each of the same
weight ?

SPACE FOR ROUGH WORK


Mock Test-4 MT-85

Statements:
I. One-fourth of the weight of each pole is 5 kg.
II. The total weight of three poles is 20 kilograms more Binding and
than the total weight of two poles. Cutting
Charges, Printing Cost,
(1) (B) (2) (A) 18% 35%
(3) (C) (4) (D)
67. How many children does M have ? Advertisement
Charges,
Statements: 18%
I. H is the only daughter of X who is wife of M. Paper Cost,
II. K and J are brothers of M. Royalty, 10%
15%
(1) (B) (2) (A)
Miscellaneous
(3) (C) (4) (D) Expenses, 4%
68. The last Sunday of March, 2006 fell on which date ? 71. The central angle for the sector on “Paper-Cost” is.
Statements: (1) 221/2° (2) 16°
I. The first Sunday of that month fell on 5th. (3) 54.8° (4) 36°
II. The last day of that month was Friday. 72. If the ‘Printing-Cost is ` 17500, the royalty paid is
(1) (B) 2. (A) (1) ` 8750 (2) ` 7500
(3) (C) 4. (D) (3) ` 3150 (4) ` 6300
69. In how many days will Mohit alone complete the work? 73. If the “miscellaneous expenses” are ` 6000. How much more
are “binding and cutting charges than “Royalty”?
Statement: (1) ` 6000 (2) ` 5500
I. Soham alone can do the work in 20 days .Mohit is 10% (3) ` 4500 (4) ` 10500
more efficient than Soham and Rita . 30% of work is 74. The central angle corresponding to the sector on “Printing
done by Mohit ? Cost” is more than that of “Advertisement Charges” by:
II. Soham and Mohit together can finish the work in 8(2/ (1) 72° (2) 61.2°
11) days , Mohit and Rita in 7(1/5) days and, Rita and (3) 60° (4) 54.8°
Soham can do the same work in 6(2/3) days. 75. The “Paper Cost” is approximately what percent of “Printing
(1) (B) (2) (A) Cost”?
(3) (C) (4) (D) (1) 20.3% (2) 28.6%
70. If the selling price of an article is ` 12000, then what is the (3) 30% (4) 32.5%
profit/loss% on it? 76. The following table shows the distribution of man players
and woman players of seven different countries:
Statement:
I. The ratio of the selling and price and cost price of the Co untry Man Players Woma n Pla yers
article is 4 : 3. (Total – 23500) (Total –32500 )
II. The difference between the cost price and selling price A 8% 1 2%
of the article is ` 2000 B 12% 1 6%
(1) (B) (2) (A) C 10% 1 5%
(3) (C) (4) (D)
D 22% 2 0%
Directions (Qs. 71-75): The following questions are based on E 14% 1 0%
the pie-chart given below. Study the pie-chart and answer the
F 16% 1 9%
questions. The percentage expenses on various items during book
production and sale. G 18% 8%

SPACE FOR ROUGH WORK


MT-86 Target NTSE
What is the ratio between the number of man players from (1) 0# 4# 0@ 7* 2# 3$
countries D and G and number of woman players from (2) 0# 6@ 2# 5# 3* 3$
countries A and G? (3) 1# 7@ 4* 1$ 2@ 1@
(1) 78 : 55 (2) 94 : 45 (4) 0# 2$ 0@ 6* 2# 1@
(3) 94 : 65 (4) 78 : 25 81. In the adjoining figure, if the centres of all the circles are
77. It takes eight hours for a 600 km journey, if 120 km is done by joined by horizontal and vertical lines, then find the number
train and the rest by car. It takes 20 minutes more, if 200 km of squares that can be formed.
is done by train and the rest by car. The ratio of the speed of
the train to that of the speed of the car is?
(1) 4 : 3 (2) 3 : 4
(3) 3 : 2 (4) 2 : 3
78. What is the code used for ‘Milk’ derived from the given
coded statements as per a code language?
I. ‘milk is very tasty’ is written as ‘ta la ja sa’,
II. ‘tea is black’ is written as ‘ha ja ka’ and
III. ‘sweet milk and tea’ is written as ‘ha pa sa ra’.
(1) ja (2) la (1) 6 (2) 7
(3) sa (4) pa (3) 8 (4) 1
79. In a certain code language 82. Find the number of triangles in the following figure.
‘open the math book’ are coded as ‘ cv mt zb fa’,
‘book are good friend’ are coded as ‘ at fa la ka’,
‘tough are math question’ are coded as ‘ ka qm nt cv ’,
‘open friend ask question’ are coded as ‘ at yb zb qm’,
Then Tough question find will be coded as ____?
(1) nt qm at (2) qm at zb
(3) qm at ka (4) qm nt yx
(1) 44 triangles (2) 14 triangles
80. In the given matrix, first now and the first column consist of (3) 27 triangles (4) 36 triangles
symbols and numbers respectively, the combination of those
would be the code for specific alphabets given in other 83. Choose a figure which would most closely resemble the
cells. For example, the code for ‘G’ could be 1$ or 2@. In the unfolded form of Figure (Z).
same manner, what from the given alternatives will be the
correct code for ‘PARROT’?

@ # * $ X Y Z
0 A P Q P
1 T P S G
2 G R N E
3 F M O T
4 C E A C (1) (2) (3) (4)
5 J R P V 84. Choose the box that is similar to the box formed from the
6 A B L J given sheet of paper (X)

SPACE FOR ROUGH WORK


Mock Test-4 MT-87

88. A boy and a girl are playing in a park. The only daughter of
the maternal grandfather of the girl, is the sister of the boy’s
father. How is the boy related to the girl?
(1) Brother (2) Cousin
(3) Son (4) Father
89. After 6 years, the ratio of ages of A and B will be 7 : 8 and the
difference between their ages is 2 years. Find the present
age (in years) of A and B respectively?
(1) 7, 9 (2) 10, 8
(3) 8, 10 (4) 9, 7
90. If 3 × 9 = 3 and 6 × 12 = 18, then 8# × 32 =?
# #

(1) 128 (2) 32


(1) (2) (3) (4)
(3) 16 (4) 64
(1) 1 and 2 only 91. Which letter replaces the question mark (?) ?
(2) 2 and 3 only
(3) 2 and 4 only 9 3
7 6
(4) 1, 2, 3 and 4
85. Choose the alternative which is closely resembles the water- 5
6 ? N
image of the given combination.
D6Z7F4 R X
3 8
(1)
(2) 6 9 6 10
(3)
(1) M (2) T
(4)
(3) U (4) V
86. Identify the figure that completes the pattern.
92. In the following question correct the equation by
interchanging two signs.10 – 2 + 5 ÷ 5 = 5
(1) + and = (2) + and –
(3) – and ÷ (4) + and ÷
93. Which one of the following diagram represents the correct
relationship among Brain, Cerebrum, Liver and Human body.

(1) (2)

(1) (2) (3) (4)


(3) (4)
87. In a class, Amit has more marks than Anu and Reena does
not have the least marks. Sagar has more marks than Tarun
and Tarun has more marks than Amit, who among them will 94. Which of the following year is a leap year?
have the least marks? (1) 398 (2) 194
(1) Amit (2) Anu (3) 784 (4) 226
(3) Sagar (4) Tarun

SPACE FOR ROUGH WORK


MT-88 Target NTSE
95. Find the statement that must be true according to the given 98. Replace ‘?’ by the appropriate figure from the given options.
information.Erin is twelve years old. For three years, she
has been asking her parents for a dog. Her parents have
told her that they believe a dog would not be happy in an ?
apartment, but they have given her permission to have a
bird. Erin has not yet decided what kind of bird she would
like to have.
(1) Erin’s parents like birds better than they like dogs.
(2) Erin does not like birds.
(3) Erin and her parents live in an apartment. (1) (2) (3) (4)
(4) Erin and her parents would like to move.
96. Ankita travels 35 km towards the West, takes a right turn 99. What is the number in place of ‘?’ ?
and travels 50 km. Next, she takes another right turn and 3A21, 7E23, 9I25, 11O27, ?
travels 35 km in that direction. How far is she now from her
(1) 13U27 (2) 13V29
original position?
(1) 75 km (2) 50 km (3) 13W29 (4) 13U29
(3) 85 km (4) 70 km 100. In the given figure which number should replace ‘M’ ?
Direction (Q. 97): Two statements are given here which are
followed by two conclusions. You have to consider the two 18
statements to be true even if they seem to be at variance from
14 8
commonly known facts. You have to decide which of the given
conclusions, if any, follow from the given statements. 16 14
97. Statements:
12 20
1. Life is a journey with few known people, few partially
known and mostly strangers. 6
7
2. In a journey, the importance of destination is as equal
as the pathway of reaching that destination.
M 10
Conclusions:
I. Importance of people in life is considerable, as they are
our journey mates. 16 13 7 24
II. Life teaches us many lessons to face various known
and unknown difficulties. (1) 15 (2) 18
(1) Both I and II follow. (3) 12 (4) 25
(2) Neither I nor II follows.
(3) Only conclusion II follows.
(4) Only conclusion I follows.

SPACE FOR ROUGH WORK


Mock Test-4 MT-89

ANSWER SHEET

Name : ..................................School .............................. Test Code............Roll No. : ...................

City : ................... Date : ................... Class : ..................

(1) (2) (3) (4) (1) (2) (3) (4) (1) (2) (3) (4)
1. 35. 68.
2. 36. 69.
3. 37. 70.
4. 38. 71.
5. 39. 72.
6. 40. 73.
7. 41. 74.
8. 42. 75.
9. 43. 76.
10. 44. 77.
11.
45. 78.
12.
46. 79.
13.
47. 80.
14.
48. 81.
15.
49. 82.
16.
50. 83.
17.
51. 84.
18.
52. 85.
19.
53. 86.
20.
54. 87.
21.
55. 88.
22.
23. 56. 89.

24. 57. 90.


25. 58. 91.
26. 59. 92.
27. 60. 93.
28. 61. 94.
29. 62. 95.
30. 63. 96.
31. 64. 97.
32. 65. SPACE FOR ROUGH WORK 98.
33. 66. 99.
34. 67. 100.

Sig. of Invigilator.................................
MT-90 Target NTSE
SPACE FOR ROUGH WORK

SPACE FOR ROUGH WORK


4
(Stage 2)

Time :: 120
Time 90 Minutes
Minutes Max. Marks : 100

INSTRUCTIONS FOR CANDIDATES


Read the following instructions carefully before you answer the questions.
1. There are 100 questions in this test all are compulsory. Each question has 4 choices (1), (2), (3), and (4), out of which
ONLY ONE is correct.
2. Answers are to be given on separate answer sheet provided at the end of this paper. You can tear this answer sheet
along the cut marks provided.
3. If you do not know the answer to any question, do not waste time on it and pass on to the next one. Time permitting,
you can come back to the questions, which you have left in the first instance and attempt them.
4. Since the time allotted for this question paper is very limited you should make the best use of it by not spending too
much time on any one question.
5. For all questions 1-100 put a cross mark (×) on the number of the correct alternative on the answer sheet against the
corresponding question number by using a blue ball point pen. Example

Correct Way : Q.No. Alternatives


1. 1 2 3 4

Wrong Way : Q.No. Alternatives


1. 1 2 3 4
1. 1 2 3 4

6. Rough work can be done anywhere in the booklet but not on the answer sheet/loose paper.
7. Every correct answer will be awarded one mark. There is no negative marking.
8. After completion of this test you can evaluate your performance by comparing your answers with answers provided
in solution booklet.
MT-92 Target NTSE
1. Match column-I with column-II and select the correct answer 5. On the basis of following features identify correct option
using the codes given below : I. These animals are cold-blooded, have scales and
Column I Column II breathe through lungs.
II. Most of them have a three-chambered heart.
(A) Amoeba, Paramoecium (p) Lysosomes
III. They lay eggs with tough coverings and do not need
Chlamydomonas and to lay their eggs in water.
bacteria (1) Reptilia (2) Aves
(B) Protoplasm (q) Unicellular organisms (3) Amphibia (4) Pisces
6. A beaker consisting of a solution and a small thin walled
(C) ‘Suicide bags’ of a cell (r) Purkinje in 1839
bag was kept inside the beaker solution that consisted of an
(D) Chromatin material (s) Schleiden and Schwann ionic solution. After some time the same ions found inside
and nucleolus the thin membrane bag came out into the beaker solution.
Which of the following process explains the process:
(E) Cell theory (t) Nucleus
(1) Plasmolysis (2) Dialysis
(1) (A) ® r, (B) ® q, (C) ® p, (D) ® t, (E) ® s (3) Osmosis (4) Diffusion
(2) (A) ® q, (A) ® r, (A) ® p, (A) ® s, (A) ® t 7. On the basis of following features identify correct option.
(3) (A) ® q, (B) ® r, (C) ® p, (D) ® t, (E) ® s I. It is the covering or protective tissues in the animal
body.
(4) (A) ® q, (B) ® p, (C) ® r, (D) ® t, (E) ® s
II. It forms a barrier to keep different body systems
2. Consider the following statements : separate.
(a) A nerve is a bundle of axons of the neurons. III. The skin, the lining of the mouth, the lining of blood
(b) Collenchyma cells are dead and have irregular
vessels, lung alveoli and kidney tubules are made up
thickenings at the corners.
of this type of tissue.
(c) Skin epithelium is called stratified squamous epithelium
(1) Epithelial tissue (2) Connective tissue
because it has one layer of cells.
(3) Muscular tissue (4) Nervous tissue
(d) Tendons connect muscles to bones.
Which of these statement(s) is/are correct? 8. Match column-I with column-II and select the correct answer
(1) (a) and (b) (2) (c) only using the codes given below :
(3) (a) and (d) (4) All are correct Column I Column II
3. A cell organelle expressing cytochrome oxidase was A. Global warming (p) Eutrophication
ruptured. Which of the following outcomes are most likely B. Algal bloom (q) CO, CO2
to occur in the cell: C. Nitrogen fixation (r) Rhizobium
A. Blockage of cellular respiration D. Pseudomonas (s) Fossil fuel
B. Blockage of photsynthesis E. Non-renewable resource (t) Denitrification
C. Blockage of Kreb's cycle (1) (A) ® p, (B) ® q, (C) ® r, (D) ® t, (E) ® (s)
D. Disruption of lipid sysnthesis (2) (A) ® q, (B) ® p, (C) ® r, (D) ® t, (E) ® (s)
(1) A and C (3) (A) ® q, (B) ® p, (C) ® r, (D) ® s, (E) ® (t)
(2) A and B (4) (A) ® q, (B) ® p, (C) ® s, (D) ® r, (E) ® (t)
(3) A and D 9. On the basis of following features identify correct option.
(4) B and D (I) It is able to express its effect even in the presence of
4. Which of the following influence the unidirectional recessive factor.
movement of water through xylem (II) It produces an effective or complete protein for
(1) Phloem sap (2) Transpriration expressing its effect.
(1) Recessive factor (2) Heterozygous factor
(3) Imbibition (4) Sunlight
(3) Homozygous factor (4) Dominant factor

SPACE FOR ROUGH WORK


Mock Test-4 MT-93
10. The rat in set-up (B) with a potted plant remains healthy, 15. Acids, acidic oxide and acidic salts
whereas the rat in the set-up (A) without potted plant dies (a) will turn red litmus blue
(b) will turn blue litmus red
(c) will turn pH paper orangish red
(d) will turn pH paper green
O2 (1) (a) and (b) (2) (b) and (c)
CO2 (3) (c) and (d) (4) (a) and (d)
16. Which of the following represents saponification reaction?
CaO
(1) CH3COONa + NaOH ¾¾¾
®
(A) (B) CH4 + Na2CO3
conc. H 2SO 4
(2) CH3COOH + C2H5OH ¾¾¾¾¾¾
(1) due to unavailability of O2 ®
(2) due to unavailability of CO2 CH3COOC2H5 + H2O
(3) 2C2H5OH + 2Na ¾¾® 2CH3COONa + H2
(3) due to absence of food
(4) CH3COOC2H5 + NaOH ¾¾ ®
(4) because rat loves plants CH3COONa + C2H5OH
11. Column I Column II 17. Match Column-I with Column-II and select the correct
(A) Cell growth (p) Abscisic acid answer using the codes given below the columns.
(B) Wilting (q) Nerve conduction Column I Column II
(C) Emergency (r) Adrenaline (A) Electronegativity (p) Isotopes
(D) Electrical impulse (s) Auxin (B) Lanthanides (q) increases along a period
(1) A – (r); B – (s); C – (q); D – (p) (C) Transition elements (r) f-group of elements
(2) A – (s); B – (r); C – (q); D – (p) (D) Ionisation energy (s) d-group of elements
(3) A – (s); B – (r); C – (p); D – (q) (E) Elements of same (t) decreases along a group
(4) A – (s); B – (p); C – (r); D – (q) atomic number
but different
12. Consider the following statements: mass number
(a) Plasmodium multiplies by binary fission. (1) A – (q); B – (r); C – (s); D – (p) ; E – (t)
(b) Bryophyllum propagates through spore formation. (2) A – (r); B – (q); C – (s); D – (t); E – (p)
(3) A – (q); B – (r); C – (s); D – (t) ; E – (p)
(c) Hibiscus has unisexual flowers. (4) A – (q); B – (s); C – (r); D – (t); E – (q)
Which of the above statements are correct? 18. On the basis of following features identify the correct option.
(1) (a) and (b) (2) (b) and (c) I. These rays travel in straight line.
(3) (a), (b) and (c) (4) None of these II. These rays consist of positively charged particles
13. The substance that triggers the fall of mature leaves and
III. These rays are deflected by electric and magnetic fields
fruits from plants is due to
(1) auxin (2) gibberellin IV. e/m values for the particles are not same for all gases.
(3) abscisic acid (4) cytokinin (1) Cathode rays (2) Anode rays
14. Which of the following statements are true about the brain? (3) Both (1) and (2) (4) Neither (1) nor (2)
(a) The main thinking part of brain is hind brain. 19. Consider the following statements:
(b) Centres of hearing, smell, memory, sight, etc, are located (a) Rusting is exothermic process
in fore-brain. (b) Dissolution of ammonium chloride in water is
(c) Involuntary actions like salivation, vomiting, blood exothermic process
pressure are controlled by the medulla in the hind brain. (c) Addition of chlorine is also oxidation process.
(d) Cerebellum does not control posture and balance of (d) The formula of calcium phosphate is Ca3(PO4)2.
the body.
Which of the above statements are correct?
(1) (a) and (b) (2) (a), (b) and (c)
(1) (a), (b) and (c) (2) (a), (b) and (d)
(3) (b) and (c) (4) (c) and (d)
(3) (b), (c) and (d) (4) (a), (c) and (d)

SPACE FOR ROUGH WORK


MT-94 Target NTSE
20. Four students A, B, C and D noted the initial colour of the Which student noted the colour change in all the four beakers
solution in beakers I, II, III and IV. After inserting zinc rods correctly?
in each solution and leaving it undisturbed for two hours, (1) (A) (2) (B)
noted the colour of each solution again.
(3) (C) (4) (D)
21. Which of the following are the characteristics of isobars of
Zn Zn different elements?
(i) Isobars of different elements have same number of
neutrons
(ii) Isobars of different elements have same number of
protons and neutrons
(iii) Isobars of different elements have similar physical
properties
Al2(SO4)3 ZnSO4
(iv) Isobars of an elements have same mass number
(I) (II)
(1) (i) and (iii) (2) (ii) and (iv)
Zn Zn (3) (i), (iii) and (iv) (4) (ii), (iii) and (iv)
22. Arrange the following in increasing order of number of
molecules
I. 0.5 mol of H2 II. 4.0 g of H2
III. 18 g of H2O IV. 2.2 g of CO2
(1) II > III > I > IV (2) IV < I < III < II
(3) I < II < III < IV (4) IV < III < I < II
FeSO4 CuSO4
23. During smelting, an additional substance is added which
(III) (IV)
combines with impurities to form a fusible product. It is
The recorded observations in the form of table given below: known as:
Student Colour [I] [II] [III] [IV] (1) Slag (2) Mud
of the (3) Gangue (4) Flux
solution 24. Match the Columns and mark the correct option :
Column - I Column - II
(A) Initial Colour- Colour- Light Blue (A) PVC (p) Artificial silk
less less green (B) PHBV (q) Artificial wool
Final Colour- Colour- Colour- Colour- (C) Orlon (r) Coating of electrical wires
less less less less (D) Rayon (s) Biodegradable polymer
(1) A – (p); B – (q); C – (r); D – (s)
(B) Initial Colour- Light Light Blue (2) A – (q); B – (s); C – (s); D – (p)
less yellow green (3) A – (r); B – (s); C – (p); D – (q)
Final Colour- Colour- Light Colour- (4) A – (r); B – (s); C – (q); D – (p)
less less green less 25. Match column-I with column-II and select the correct answer
using the codes given below :
(C) Initial Colour Colour Light Blue Column I (Acid) Column II (Use)
less less green
(A) Oxalic acid (p) As an eye-wash,
Final Light Colour- Colour- Light antiseptic and grain
blue less less green preservation
(D) Initial Light Colour Light Blue (B) Nitric acid (q) For making explosives
green less green (C) Boric acid (r) In food preservation
Final Colour- Colour- Dark Colour- (D) Benzoic acid (s) As a constituent of ink
less less green less stain remover

SPACE FOR ROUGH WORK


Mock Test-4 MT-95

(1) (A) ® s, (B) ® q, (C) ® r, (D) ® p 30. On the basis of following features identify correct option :
(2) (A) ® q, (B) ® s, (C) ® p, (D) ® r I. It is independent of nature of intervening medium.
(3) (A) ® r, (B) ® q, (C) ® p, (D) ® s II. It obeys Newton’s third law of motion.
(4) (A) ® s, (B) ® q, (C) ® p, (D) ® r (1) Gravitational force (2) Nuclear force
26. Burning magnesium ribbon is brought in the gas jar of carbon (3) Both (1) and (2) (4) Neither (1) nor (2)
dioxide. Which of the following is correct?
31. Two point masses each equal to 1 kg attract one another
(i) It keeps on burning with a force of 10–9 kg wt. The distance between the two
(ii) It gets extinguished point masses is (G = 6.6 × 10–11 MKS units).
(iii) Although CO2 is non-supporter of combustion but (1) 8 cm (2) 0.8 cm
burning magnesium breaks CO2 into carbon and oxygen
(3) 80 cm (4) 0.08 cm
helps in burning.
32. A force F acting on an object varies with distance x as
(iv) Carbon dioxide is supporter of combustion
shown here. The force is in N and x in m. The work done by
(1) (i) and (iv) (2) (i) and (iii) the force in moving the object from x = 0 to x = 6 m is
(3) (i) and (ii) (4) (iii) and (iv)
27. Consider the following statements: (1) 18.0 J F(N)

(a) All hydrocarbons show phenomena of isomerism.


(2) 13.5 J 3
(b) The members of a homologous series have similar 2
chemical and physical properties. 1
(3) 9.0 J x(m)
(c) Cyclic hydrocarbons are saturated or unsaturated 0
1 2 3 4 5 6 7
organic compounds that contain ring of carbon atoms. (4) 4.5 J
(d) Synthetic detergents are sodium or potassium salts of 33. The kinetic energy of a body will become eight times if
long chain carboxylic acids.
(1) its mass is made four times
Which of the above statement(s) is/are correct?
(2) its velocity is made four times
(1) Only (c) (2) Only (d)
(3) both the mass and velocity are doubled
(3) (a), (b) and (c) (4) (a), (c) and (d) (4) both the mass and velocity are made four times
28. The distance travelled by a particle starting from rest and 34. Consider the following statements :
4 -2 (a) Sound waves are longitudinal in nature.
moving with an acceleration ms , in the third second is:
3
(b) Sound waves cannot travel through vacuum.
10 19 (c) Sound waves are produced by oscillating charged
(1) 6 m (2) 4 m (3) m (4) m
3 3 particles only.
29. Which of the following changes when a particle is moving (d) Sound waves are electromagnetic waves.
with uniform velocity? Which of the above statements are correct?
(1) Speed (2) Velocity (1) (a) and (b) (2) (b) and (c)
(3) Acceleration (4) Position vector (3) (a), (b) and (d) (4) (a), (b), (c) and (d)

SPACE FOR ROUGH WORK


MT-96 Target NTSE
35. Which of the following properties of sound is affected by 43. If the ratio of the roots of equation x2 + px + q = 0 be equal to
change in air temperature? the ratio of the roots of x2 + lx + m = 0, then
(1) Amplitude (2) Frequency (1) p2m = q2l (2) pm2 = q2l
(3) Wavelength (4) Intensity (3) p2l = q2m (4) p2m = l2q
36. Two mirrors M1 and M2 are placed
M1 44. A clock strikes the number of times of the hour. How many
at right angle to each others as
strikes does it make in one day?
shown. The total number of images
of an object ‘O’ that can be seen are (1) 165 times (2) 156 times
(3) 78 times (4) 87 times
(1) Two (2) Three O M2
(3) Four (4) Infinite 45. If x = a(b - c); y = b(c - a ); z = c (a - b ) , then
37. Blue colour of sky is due to
3 3 3
(1) dispersion of light (2) scattering of light æ xö æ yö æzö
ç a ÷ + ç b ÷ + ç c ÷ is equal to
(3) reflection of light (4) refraction of light è ø è ø è ø
38. There are three copper wires of length and cross sectional xyz 1 xyz
(1) (2)
1 1 abc 3 abc
area (L, A), (2 L, A) , ( L, 2 A) . In which case is the
2 2 xyz 3(x + y + z)
(3) 3 (4)
resistance minimum? abc (abc)
(1) It is the same in all three cases
(2) Wire of cross-sectional area 2A 5+2 + 5 -2
(3) Wire of cross-sectional area A 46. Value of x2, if x = is
5 +1
1
(4) Wire of cross-sectional area A (1) – 2 (2) 2 (3) (4)
2 2 - 2
39. The direction of induced current is obtained by
47. If x + y = 18 + 6 5 , find the value of x.
(1) Fleming’s left hand rule.
(2) Maxwell’s left hand rule. (1) 12 (2) 15 (3) 6 (4) 8
48. 2 2
What is minimum value of sec q + cos q
(3) Ampere’s rule.
(4) Fleming’s right hand rule. (1) –1 (2) 1 (3) 2 (4) 2
40. Which of the following is a renewable source of energy? 49. In the figure OAB is an equilateral triangle. The co-ordinate
of vertex B is
(1) Coal (2) Natural gas
(3) Sun light (4) Uranium B
Y
41. If (x +1)(x + 2)(x + 3)(x + k) + 1 is a perfect square, then the
value of k is
(1) 4 (2) 5 (3) 6 (4) 7
42. If a and b are the zeroes of the quadratic polynomial
f (x) = 2x2 – 5x + 7, then a polynomial whose zeroes are
2a + 3b and 3a + 2b. is
X' O (0, 0) M A(2a, 0) X
æ 25 ö æ 25 ö
(1) k ç x2 - x - 41÷ (2) k ç x2 + x + 41÷ Y'
è 2 ø è 2 ø
(1) (a, a) (2) (– a, – a)
æ 25 ö æ 25 ö
(3) k ç x2 - x + 41÷ (4) - k ç x2 - x + 41÷ (3) (a, 3a) (4) (a, – 3a)
è 2 ø è 2 ø

SPACE FOR ROUGH WORK


Mock Test-4 MT-97
R
50. A coin is tossed 40 times and it showed tail 24 times. The
probability of getting a head was
o
2 3 1 17 35 y
o

(1) (2) (3) (4) Q


5 5 2 40
O 2 cm
51. In the given figure, the measure of ÐPQB is
P 2 cm
P

A 42° (1) y° (2) 35° (3) 105° (4) 70°


O B
55. The value of 2(sin q + cos q) – 3(sin q + cos4q) is...
6 6 4

(1) 1 (2) –1 (3) 0 (4) ±1


Q
56. Instead of walking along two adjacent sides of a rectangular
(1) 40° (2) 48° (3) 42° (4) 90° field, a boy took a short cut along the diagonal and saved a
52. The area of the shaded region in the diagram below where distance equal to half the longer side. Then the ratio of the
the given triangle is isosceles with vertices of base lying on shorter side to the longer side is
axis of the radius perpendicular to the diameters of the two 1 2 1 3
small semicircles, is (1) (2) (3) (4)
2 3 4 4
57. If HCF (105, 120) = 15, then LCM (105, 120) is
(1) 210 (2) 420 (3) 840 (4) 1680
4 58. Out of the following four relations :
sin A sin A 2
(I) + =
1 + cos A 1 - cos A sin A
4 æ 1 + cos A ö
2
1 + cos A
(II) ç ÷ =
è sin A ø 1 - cos A
8 O
8 sin10°
(III) =1
cos80°
(1) 16 (p + 1) (2) 16 (p – 1) (IV) sin4A – cos4 A = 1 + sin2A
(3) 32 (p – 1) (4) 32 (p + 1) the wrong one is
(1) I (2) II (3) III (4) IV
1 1+ 2 p 1 + 4 p 59. The median for the following distribution class
53. The nth term of the sequence , , , ... is
p p p
Class 0 – 10 10 – 20 20 – 30 30 – 40 40 – 50 50 – 60
Frequency 5 10 20 7 8 5
1 + 2np + 2 p 1 – 2np – 2 p
(1) (2) is
p p
(1) 26 (2) 25 (3) 26.25 (4) 27
60. From the top of a Church spire 96 m high, the angle of
1 + 2np – 2 p 1 + 2np depression of two vehicles on the road, at the same level as
(3) (4)
p p the base of the spire and on the same side of it are x° and y°,
1
54. O is the centre of the circle as shown in the diagram. The where tan x° = 4 and tan y° = 17 . Then, distance between
distance between P and Q is 4 cm. Then the measure of the vehicles is
ÐROQ is (1) 384 m (2) 672 m
(3) 288 m (4) None of these

SPACE FOR ROUGH WORK


MT-98 Target NTSE
61. With reference to the invention of print, consider the (b) Gandhiji began the salt march from his Sabarmati
following statements- Ashram to Dandi and manufactured a handful of salt
to break colonial laws
(a) Richard M. Hoe of New York had perfected the power
driven cylindrical press which had capacity to print Which of the above statement(s) is/are correct?
8000 sheets per hour.
(1) (a) only (2) (b) only
(b) Buddhist missionaries from china introduced hand
printing technology into Japan and first Japanese book (3) Both (a) and (b) (4) Neither (a) nor (b)
Diamond Sutra Printed. 65. On the basis of following features identify the correct option-
Which of the above statement(s) is/are correct? I. He was the follower of Mahatma Gandhi.
(1) (a) only (2) (b) only II. He was one of the votaries of Gandhi’s concept of
(3) Both (a) and (b) (4) Neither (a) nor (b) Gram Swarajya.

62. Consider the following statements, with reference to the III. He started Bhoodan-Gramdan movement.
origin of Test Cricket- IV. His movement is also known as Blood less revolution.
(a) Test cricket was initially organised as a competition (1) Vinoba Bhave (2) Jawaharlal Nehru
between different parts of the British empire.
(3) Subhash Chandra Bose (4) Sardar Patel
(b) The first test was played between England and India
when India was a British colony at that time. 66. On the basis of the following features identify the correct
option -
Which of the above statement(s) is/are correct?
I. It started in South Africa and soon spread to the other
(1) (a) only (2) (b) only parts of the continent.
(3) Both (a) and (b) (4) Neither (a) nor (b) II. It had a terrifying impact on people’s livelihoods and
63. With reference to the female allegory of France, consider the local economy.
the following statements
III. It spread through infected cattle imparted from British
(a) She was named Marianne. Asia.
(b) She took part in the French revolution. IV. It spread in the year 1880.
(c) She was a symbol of national unity. (1) Great Depression (2) Rinderpest
(d) Her characteristics were drawn from liberty and the (3) Indenture (4) Influenza
republic.
67. Statement-I : In the 19th century, London was a colossal city.
Which of the above statements are correct?
Statement-II : London had grown as an industrial city.
(1) (a), (b) and (c) (2) (b), (c) and (d)
(1) Statement I is true, Statement II is false.
(3) None of the above (4) All of the above
(2) Statement I is false, Statement II is true.
64. Consider the following statements with reference to
Nationalist movement in South-east Asia- (3) Both statements are true, and Statment II provides
explanation to Statement I.
(a) Ho-Chi-Minh reorganized the nationalist groups as
Vietnamese communist party in 1930. (4) Both statements are true, but Statement II does not
provide explanation of Statement I.

SPACE FOR ROUGH WORK


Mock Test-4 MT-99

68. Statement-I : Louis Blanc built a cooperative community. IV. The traditional Chinese accordion book was not folded
or stitched at the side.
Statement-II : He believed the community could produce
goods together and divide the profits among the members. (1) I and II (2) I and III

(1) Statement I is true, Statement II is false. (3) I and IV (4) I, II, III and IV

(2) Statement I is false, Statement II is true. 72. What was Dawes Plan?

(3) Both statements are true, and Statment II provides (1) A plan to secure loans from US bankers
explanation to Statement I. (2) A plan to ease reparation terms on Germany
(4) Both statements are true, but Statement II does not (3) A plan to overtake Rhineland mines
provide explanation of Statement I.
(4) An extension of Versailles treaty
69. Which of the following statement is not correct about the
Rowlatt Act? 73. Consider the following statements –
(a) Alluvial soil are formed by sediments deposited by
I. It allowed the detention of prisoners for five years
rivers and waves
wihtout trial
(b) Black soil is found in the dry and Semi- dry areas of
II. Gave the government the enormous powers to repress Rajasthan and Gujarat.
political activity
(c) Red soil cannot retain much moisture.
III. The act was not passed because of the opposition
from Indian members (d) Laterite soil is found over lava deposits in the Deccan.

IV. Led to launch of a movement under Gandhiji’s Which of the above statements are correct?
leadership (1) (a), (b) and (c) (2) (a) and (c)
(1) II and III (2) I and III (3) (b) and (d) (4) None of them
(3) I and IV (4) I, II and IV 74. On the basis of following features identify the correct
option–
70. The Simon Commission was boycotted in India because
I. These are areas set aside as protected areas for the
(1) There was no Indian member in the commission
preservation of flora, fauna and wildlife
(2) It supported the Muslim League II. Rare and endangered species are often kept here.
(3) Congress felt that the people deserved Swaraj III. The main purpose of these reserves is to preserve
(4) There were differences among the members nature.

71. Which of the following statement is true with regard to the IV. It provides opportunity to study and research the
first printed books is correct sheltered animals.

I. Print technology was developed in China, Japan and (1) National parks (2) Nature reserves
Korea from 694 onwards (3) Wildlife Sanctuary (4) Forests
II. The books in China were printed by rubbing paper 75. Consider the following statements-
from AD 594 onwards.
(a) With the rapid advancement of Industries, the use of
III. This was a System of hard printing. metal has increased.

SPACE FOR ROUGH WORK


MT-100 Target NTSE
(b) Mineral resources are renewable. (2) Igneous and metamorphic rocks
(c) High quality coal deposits are depleting. (3) Sedimentary rocks
(d) To conserve our resources, we should take measures (4) None of the above
to reduce wastage during mining and processing of
an ore. 79. In which of the following groups of cities were most of the
Which of the above statements are correct? manufacturing units located in the pre-independence period?

(1) (a), (b) and (c) (2) (a), (c) and (d) (1) Delhi, Kanpur, Moradabad

(3) (a), (c) and (d) (4) (a), (b) (c) and (d) (2) Bangalore - Hyderabad

76. On the basis of following features identify the correct option- (3) Mumbai, Kolkata, Chennai

I. Total length of the river is about 760 km. (4) Chandigarh, Ludhiana, Amritsar

II. It rises in the Brahmagiri range of the western ghats. 80. Which one of the statements is not an argument in favour
of multipupose rivor projects.
III. Its main tributaries are Amravati, Bhavani, Hemavati
and Kabini. (1) Multi purpose projects beings water to those areas
which suffer from water scarcity
IV. Its basin drains parts of Karnataka, Kerala and
Tamilnadu. (2) Multi purpose projects by regulating water flow, help
to control floods
(1) The Krishna Basin (2) The Godavari Basin
(3) Multi purpose projects leads to large scale
(3) The Kaveri Basin (4) The Narmada Basin displacement and loss of livelihood
77. Consider the following statements- (4) Multi purpose projects generate electricity for our
(a) Peninsular rocks contain most of the reserves of coal, industries and our homes
metallic minerals, mica and other non-metallic minerals. 81. On the map of India mark the following :
Match the fishing ports indicated on the map of India (I, II,
(b) Limestone is the basic raw material for the Iron and III and IV) with their respective names.
Steel Industry.

(c) Biogas is produced from shrubs, farm waste, animal


and human wastes.

(d) Hydro electricity is non-renewable resource.

Which of the above statements are correct?

(1) (a), (b) and (c) (2) (a) and (c)

(3) (a) and (d) (4) (b), (c) and (d)

78. Minerals formed from solidification of molten matter in the


A. Kakinada B. Alappuzha
crack, crevies, faults or joints are found in which type of C. Porbandar D. Tuticorin
rocks? (1) II-A, III-B, I-C, IV-D (2) II-A, III-B, I-D, IV-C
(3) II-C, III-B, I-A, IV-D (4) II-D, III-B, I-A, IV-C
(1) Stratified rocks

SPACE FOR ROUGH WORK


Mock Test-4 MT-101

82. What are placer deposits? 88. Consider the following statements-
(1) Metals that occur as Sandy deposits in sands of sea (a) Right is a claim of the individual over other fellow
floor. beings over the society and over the government.
(2) Metals that occur as sandy deposits in sand of land (b) A claim becomes Right only when it is recognised by
floor society.
(3) Minerals that occur as allurial deposit in sand of valley (c) It should be sanctioned by Law.
floor (d) Rights are not unlimited.
(4) None of these
Which of the above statements are correct?
83. Which of the following statements is/are correct about
(1) (a) and (c) (2) (b) and (c)
bituminous coal?
I. It contains 60 to 80 percent carbon. (3) (a), (b) and (c) (4) None of the above
II. It is produced by decaying of plants in swamps. 89. On the basis of following features identify the correct option-
III. It is used for smelting iron in blast furnaces. I. It is formed under the leadership of Kanshi Ram.
IV. It is low grade brown coal, whch is soft with high II. It represent dalits, adivasis and OBCs.
moisture content III. It has its main base in the state of U.P.
(1) I and IV (2) I and II IV. It is formed in 1984 and secured the interests and welfare
(3) II and III (4) I, II, III and IV of the dalits
84. Which of the following is not true regarding the National (1) Bhartiya Janta Party
Jute Policy 2005. (2) Communist party of India
I. It was formulated with the objective of increasing (3) Bahujan Samaj Party
productivity. (4) Nationalist Congress party
II. Creating awareness about the use of bio-degradable 90. Which of the following sentiments about the reason for
materials. conducting elections are incorrect.
III. The jute farmers were ensured good prices. I. Elections enable the people to judge the performance
IV. The yield per hectare enchanced.
of the government.
(1) I and II (2) Only II
II. People cannot indicate the policies they prefer.
(3) I, II and III (4) Only I
85. On the basis of following features identify the correct III. People select the representatives of their in an election.
options- IV. Elections enable the people to evaluate the
I. It reduces the possibility of conflict between social performance of the judiciary.
groups. (1) I and IV (2) I and III
II. It ensures political stability in the long run (3) I, II and III (4) II and IV
III. It upholds the spirit of democracy 91. In what ways does the Lok Sabha exercise Supreme Power
IV. It promotes an equal division of power among all social over Rajya Sabha select the correct option.
groups. I. Lok Sabha controls the President of India.
(1) Democracy II. During the joint session the final decision is taken by
(2) Power sharing Lok Sabha because of its larger number of members.
(3) Provincial government III. Lok Sabha exercise more powers on money matters.
IV. It guides the functioning of Rajya Sabha.
(4) Majoritarian measures
(1) I and IV (2) I and III
86. Tamil Eelam means -
(3) II and III (4) I, II, III and IV
(1) Tamil state (2) Tamil court
92. Judiciary (courts) can take up any dispute -
(3) Tamil government (4) Tamil language I. Between citizens and the judiciary.
87. What do you mean by Residuary subjects? II. Between citizens and the government.
(1) Matters of state level importance. III. Between two or more state governments.
(2) Subjects and matters of national issues. IV. Between government at the Union and government of
(3) Subjects which do not include in union, state and the othe rcountries.
concurrent list. (1) Only I (2) Only II
(4) Subjects which include in union list and state list both. (3) I, II and III (4) Only IV

SPACE FOR ROUGH WORK


MT-102 Target NTSE

93. Match column I with column II and select the correct answer 96. Which states of India have a common border with Pakistan?
using the codes given below : A. Jammu and Kashmir B. Himachal Pradesh
Column I Column II C. Punjab D. Gujarat
(A) PMRY-1993 (p) Aim is to create self employment E. Rajasthan
opportunities for educated unem- (1) A, B, C, D (2) A, B, C, E
ployed youth in rural areas and (3) A, C, D (4) A, C, D, E
small towns 97. Which of the following statements is/are correct with regard
(B) REGP-1995 (q) Additional central assistance is to the higher calorie requirement in rural areas :
given to states for basic services I. In rural areas people do not enjoy as much as people
such as primary health, primary in the urban areas.
education, rural shelter and drink-
II. Food items are expensive.
ing and rural electrification
III. In rural areas people are engaged in mental work.
(C) SGSY-1999 (r) A target for creating 25 lakh new
jobs has been set for the IV. In rural areas people are engaged in physical work.
programme under the tenth five (1) Only I (2) II and III
year plan (3) I and IV (4) Only IV
(D) PMGY-2000 (s) Aims at bringing the assisted poor 98. Assume there are four families in a country. The average per
families above the poverty line by capita income of these families is `5000. If the income of
organizing them into self help three families is `4000, `7000 and `3000 respectively. What
groups through a mix of bank is the income of the fourth family?
credit and government subsidy (1) `7500 (2) `3000
(1) (A) ® s, (B) ® r, (C) ® q, (D) ® p (3) `2000 (4) `6000
(2) (A) ® p, (B) ® r, (C) ® s, (D) ® q 99. The district level court deals with the cases involving claim
(3) (A) ® p, (B) ® q, (C) ® r, (D) ® s upto `20 lakhs, the state level courts between `20 lakh and
(4) None of them `1 crore and the national court deals with cases involving
94. Globalisation does not involve which one of the following? claims exceeding
(1) Rapid integration between countries. (1) `10 crore (2) `1 crore
(2) Increased taxes on imports.
(3) `100 crore (4) `50 crore
(3) More goods and services moving between countries.
100. Which one of the following statements is not true regarding
(4) Movement of people between countries for job,
education etc. impact of globalisation of India?
95. Why is it necessary that banks and cooperatives increase I. Labour laws not implemnted properly and workers are
denied their rights.
their lending particularly in rural areas?
II. People with education skill and wealth have not been
(1) To reduce the dependence on informal sources of credit
benefitted.
(2) To promote informal sources of credit in urban areas
III. It has created jobs in the service sector.
(3) To provide high interest
IV. Benefits of globalisation are not shared equally.
(4) None of them (1) Only I (2) II and III
(3) Only II (4) I, III and IV

SPACE FOR ROUGH WORK


Mock Test-4 MT-103

ANSWER SHEET

Name : ..................................School .............................. Test Code............Roll No. : ...................

City : ................... Date : ................... Class : ..................

(1) (2) (3) (4) (1) (2) (3) (4) (1) (2) (3) (4)
1. 35. 68.
2. 36. 69.
3. 37. 70.
4. 38. 71.
5. 39. 72.
6. 40. 73.
7. 41. 74.
8. 42. 75.
9. 43. 76.
10. 44. 77.
11.
45. 78.
12.
46. 79.
13.
47. 80.
14.
48. 81.
15.
49. 82.
16.
50. 83.
17.
51. 84.
18.
52. 85.
19.
53. 86.
20.
54. 87.
21.
55. 88.
22.
23. 56. 89.

24. 57. 90.

25. 58. 91.


26. 59. 92.
27. 60. 93.
28. 61. 94.
29. 62. 95.
30. 63. 96.
31. 64. SPACE FOR ROUGH WORK
97.
32. 65. 98.
33. 66. 99.
34. 67. 100.

Sig. of Invigilator.................................
SPACE FOR ROUGH WORK
5
(Stage 2)

Time : 120 Minutes Max. Marks : 100

INSTRUCTIONS FOR CANDIDATES


Read the following instructions carefully before you answer the questions.
1. There are 100 questions in this test all are compulsory. Each question has 4 choices (1), (2), (3), and (4), out of which
ONLY ONE is correct.
2. Answers are to be given on separate answer sheet provided at the end of this paper. You can tear this answer sheet
along the cut marks provided.
3. If you do not know the answer to any question, do not waste time on it and pass on to the next one. Time permitting,
you can come back to the questions, which you have left in the first instance and attempt them.
4. Since the time allotted for this question paper is very limited you should make the best use of it by not spending
too much time on any one question.
5. For all questions 1-100 put a cross mark (×) on the number of the correct alternative on the answer sheet against
the corresponding question number by using a blue ball point pen. Example

Correct Way : Q.No. Alternatives


1. 1 2 3 4

Wrong Way : Q.No. Alternatives


1. 1 2 3 4
1. 1 2 3 4

6. Rough work can be done anywhere in the booklet but not on the answer sheet/loose paper.
7. Every correct answer will be awarded one mark. There is no negative marking.
8. After completion of this test you can evaluate your performance by comparing your answers with answers provided
in solution booklet.
MT-106 Target NTSE
1. The number in the place of ‘?’should be 5. A statement with four conclusions is given below followed
by four alternatives. Choose any one alternative which is
5 4 7 6 11 2 the most suitable.
23 54 40 Statement: During the Puja days’, people visit those houses
where ‘puja’ is performed. They make it a point to go even
3 1 3 4 ? 9
if they are not invited. Manmohan visited the house of
Keshav, his office colleague, during ‘puja days’.
(1) 04 (2) 01
Conclusion:
(3) 02 (4) 03
(1) Keshav had invited Manmohan for some other
2. What term will fill the blank space?
function.
ELFA, GLHA, ILJA, _____, MLNA (2) Manmohan, being a religious man, went to Keshav’s
(1) OLPA (2) KLMA house uninvited.
(3) LLMA (4) KLLA (3) In Keshav’s house, ‘puja’ was performed.
3. Which one of the answer figures shall complete the given (4) Manmohan was invited by Keshav.
question figure? 6. In the following questions, one number is missing in the
Question Figures: series. You have to understand the pattern of the series and
insert the number.
0, 2, 6, 12, ? , 30, 42
(1) 24 (2) 20
(3) 21 (4) 22
7. A statement with two conclusions is given below followed
Answer Figures: by four alternatives. Choose any one alternative which is
the most suitable.
Statement: The standard of education in private schools is
much better than Municipal and Zila Parishad-run schools.
Conclusions:
(1) (2) (3) (4)
I. The Municipal and Zila Parishad should make serious
4. A piece of paper is folded and punched as shown below. efforts to improve standard of their schools.
From the given responses indicate how it will appear when II. All Municipal and Zila Parishad schools should be
opened. closed immediately.
Question Figures: (1) if only conclusion I follows
(2) if only conclusion II follows
(3) If either I or II follows
(4) if both I and II follow
Answer Figures: 8. DIRECTIONS: In the following question a piece of paper
is folded, cut and unfolded. One of the four figures given
below is exactly like this unfolded paper. Find this out.

(1) (2) (3) (4)

SPACE FOR ROUGH WORK


Mock Test-5 MT-107

13. In a code language, if BANGED is coded as JJKQCC, then


the word STRAY will be coded as
(1) DEUTV (2) DEUVT
(3) EFVWT (4) EFVVS
(1) (2) (3) (4)
14. Identify which number does not fit in the sequence?
9. Identify which number does not fit in the sequence?
2, 3, 7, 22, 155, ?
13, 16, 38, 124, 504, 2535
(1) 1706 (2) 1550
(1) 16 (2) 38
(3) 3411 (4) 3100
(3) 124 (4) 504
15. In a code language, if TARGET is coded as 201187520,
10. Given below is a statement followed by three assumptions then the word WILLIUM will be coded as
In order to reduce the gap between income and expenditure, (1) 239121292113 (2) 239121291213
the company has decided to increase the price of its product (3) 239122191213 (4) 239121292213
from next month.
16. Sanjay is taller than Suresh but shorter than Rakesh. Rakesh
Assumptions:
is taller than Harish but shorter than Binit. Who among is
I. The rates will be same after the increase. the tallest?
II. The expenditure will remain same in near future. (1) Suresh (2) Sanjay
III. The rival companies will also increase the price of the (3) Binit (4) Rakesh
similar product.
17. In a row of 62 persons. Rahul is 36th from left side of the
Which of the assumptions is implicit in the statement? row and Nitesh is 29th form the right side of the row. Find
(1) Only I and II are implicit out the number of persons sitting between them?
(2) Only II and III are implicit (1) 1 (2) 2
(3) Only III is implicit (3) 3 (4) 4
(4) None of these 18. Find the missing number at the place of ‘K’?
11. If A denotes ‘+’ , B denotes ‘ ’ and C denotes ‘×’
Then what is the value of (10 C 4) A (4 C 4) B 6 ? 2 3 9
(1) 60 (2) 50
1 41 5 4 159 6 4 K 8
(3) 56 (4) 46
12. Which one of the following diagram represents the correct 3 2 3
relationship among
Colour, Black and White. (1) 888 (2) 788
(3) 848 (4) 842
DIRECTIONS (Qs. 19-23): Read the following information
carefully and answer the following questions.
(1) (2)
Seven persons A, B, C, D, E, F and G were born on different
months viz. January, February, March, April, June, August and
October of the same year, but not necessarily in the same order.
Only three persons were born before E and D is not one of
(3) (4) them. F was not born immediately after E. B was born after F. A
was born immediately before the month in which G was born.
Only two persons were born between G and F.

SPACE FOR ROUGH WORK


MT-108 Target NTSE
19. How many persons were born between C and E? 27. Find the number of triangles ?
(1) Three (2) Two
(3) Four (4) Five
20. Who amongst the following is the oldest?
(1) A (2) C
(3) E (4) B
21. Who amongst the following was born between the months
in which A and D were born?
(1) E (2) G (1) 22 (2) 20
(3) C (4) Both E and G (3) 24 (4) None of these
22. How many persons were born after D? 28. Select a figure from amongst the Answer Figures which
(1) One (2) Three will continue the same series as established by the five
(3) Four (4) Two Problem Figures.
Problem Figures:
23. Who amongst the following is the person who was born in
the month which has less than 30 days? S N L
(1) F (2) B
(3) G (4) A
24. What is the code used for ‘Time’ derived from the given (A) (B) (C) (D) (E)
coded statements as per a code language?
Answer Figures:
I. ‘card win team time’ is written as ‘la ta ja sa’
II. ‘fight game play card’ is written as ‘ja pa ra da’ X O C S
III. ‘in win team fight’ is written as ‘da ta fa la’.
(1) sa (2) da
(3) ja (4) la (1) (2) (3) (4)
25. If 9q 11 a 3 = 60 and 13 q 17 a 4 = 120, then 40 q 41 a 5 = ? (A) 1 (B) 2
(1) 405 (2) 400 (C) 3 (D) 4
(3) 360 (4) 375 29. Amit is taller than Aman. Uma is taller than Atulika, but
26. In the given figure, how many pages are either new or white shorter than Jagrati.
but not both. Aman is shorter than Atulika and Atulika is taller than Amit.
(1) 111 (2) 100 Who is the tallest?
(3) 125 (4) 168 (1) Uma (2) Amit
(3) Atulika (4) Jagrati
New 19 43 21 Pages 30. Sumit remembers that the examination is after 15th May
68 but before 18th May, while Deepak remembers that the
51 57 examination is before 21st May but after 16th May. On
which date of May is the examination?
28 (1) 17 (2) 16
(3) 18 (4) 20

White

SPACE FOR ROUGH WORK


Mock Test-5 MT-109

31. If A + B means ‘A is father of B, A – B means ‘A’ is mother


of B, A * B means ‘A’ is brother of B and A % B means ‘A’
is sister of B, how is Q related to S in P + Q * R – S.
(1) Husband (2) Uncle
(3) Brother (4) Father
32. Two positions of a cube are shown below. Identify the
Symbol at the bottom when the Symbol & is at top.
?
& *

* $
% ^

(1) ^ (2) %
(3) $ (4) None of these (1) (2) (3) (4)
33. Raman walked 8m towards the North. He turned to his right 36. Look at the patterns in the squares and understand their
and walked 16m. He turned to his left and walked 5m. Again relationship to one another so as to fill in the square with
he turned to his left and walked 16m. In which direction is missing symbols.
he from his starting point?
(1) North-East (2) North
(3) South-East (4) South
34. Look at the patterns in the squares and understand their
relationship to one another so as to fill in the square with
missing symbols.

(1) (2) (3) (4)


37. Replace ‘?’ by the appropriate figure from the given options.
?

(A) (B) (C)A (D) (E)

(1) (2) (3) (4)

35. Look at the patterns in the squares and understand their


relationship to one another so as to fill in the square with (1) (2) (3) (4)
missing symbols.

SPACE FOR ROUGH WORK


MT-110 Target NTSE
38. Choose the box that is similar to the box formed from the
given sheet of paper (X).
(1) (2)

(3) (4)

42. Arrange the words given below in a meaningful sequence.


1. Poverty 2. Population
(X) (1) (2) (3) (4) 3. Death 4. Unemployment
5. Disease
(1) 1 and 2 only (2) 2 and 3 only (1) 2, 3, 4, 5, 1 (2) 3, 4, 2, 5, 1
(3) 2 and 4 only (4) 1, 2, 3 and 4 (3) 2, 4, 1, 5, 3 (4) 1, 2, 3, 4, 5
39. Choose the alternative which is closely resembles the water- 43. In the given figure which number should replace ‘N’ ?
image of the given combination.
96FSH52 4 6 3 8
(1) (2)
(3) (4)
6 10 5 12
40. Identify the figure that completes the pattern.

4 8 2 N
?
(1) 8 (2) 12
(3) 16 (4) 20
44. Deepak said to Nitin, “That boy playing with the football is
the younger of the two brothers of the daughter of my
father’s wife.” How is the boy playing football related to
(1) (2) Deepak?
(1) Son (2) Brother
(3) Cousin (4) Brother-in-law
45. If A + B means A is the sister of B; A × B means A is the
(3) (4) wife of B, A % B means A is the father of B and A – B
means A is the brother of B. Which of the following means
41. Find out from amongst the four alternatives as to how the T is the daughter of P?
pattern would appear when the transparent sheet is folded (1) P × Q % R + S – T (2) P × Q % R – T + S
at the dotted line. (3) P × Q % R + T – S (4) P × Q % R + S + T
DIRECTIONS (Qs. 46-49): In a class there are seven students
(including boys and girls) A, B, C, D, E, F and G. They sit on
three benches I, II and III. Such that at least two students on each
bench and at least one girl on each bench. C who is a girl student,

SPACE FOR ROUGH WORK


Mock Test-5 MT-111

does not sit with A, E and D. F the boy student sits with only B. I. Pictures can tell a story better than words can.
A sits on the bench I with his best friends. G sits on the bench III. II. The stories in storybooks are very simple.
E is the brother of C. III. Some storybooks have both words and pictures.
46. How many girls are there out of these 7 students ? (1) I only
(1) 3 (2) 3 or 4 (2) II only
(3) 4 (4) Data inadequate (3) III only
47. Which of the following is the group of girls ? (4) None of the statements is a known fact.
(1) BAC (2) BFC 53. Fact 1: All drink mixes are beverages.
(3) Data inadequate (4) CDF Fact 2: All beverages are drinkable.
48. Who sits with C ? Fact 3: Some beverages are red.
(1) B (2) D If the first three statements are facts, which of the following
(3) G (4) E statements must also be a fact?
49. On which bench there are three students ? I. Some drink mixes are red.
(1) Bench I (2) Bench II II. All beverages are drink mixes.
(3) Bench III (4) Bench I or II III. All red drink mixes are drinkable.
50. Hemant in order to go to university started from his house (1) I only
in the east and came to a crossing. The road to the left ends (2) II only
in a theatre, straight ahead is the hospital. In which direction (3) I and III
is the university? (4) None of the statements is a known fact.
(1) North (2) South
54. Which letter replaces the question mark (?) ?
(3) East (4) West
51. Fact 1: Most stuffed toys are stuffed with beans.
C K D M
Fact 2: There are stuffed bears and stuffed tigers.
Fact 3: Some chairs are stuffed with beans. R 32 37 T
If the first three statements are facts, which of the following
statements must also be a fact? H ? 22 F
I. Only children’s chairs are stuffed with beans. E P B N
II. All stuffed tigers are stuffed with beans.
III. Stuffed monkeys are not stuffed with beans. (1) 26 (2) 31
(1) I only (3) 27 (4) 29
(2) II only 55. What is the number in place of ‘?’ ?
(3) II and III only P5QR, P4QS, P3QT, _____, PQV
(4) None of the statements is a known fact. (1) PQW (2) PQV2
52. Fact 1: Pictures can tell a story. (3) P2QU (4) PQ3 U
Fact 2: All storybooks have pictures. DIRECTIONS (Qs. 56-60): In each of the questions below
Fact 3: Some storybooks have words. consists of a question and two statements numbered I and II given
If the first three statements are facts, which of the following below it. You have to decide whether the data provided in the
statements must also be a fact? statements are sufficient to answer the question. Read both the
statements and

SPACE FOR ROUGH WORK


MT-112 Target NTSE
Give answer (1) (B) (2) (A)
(A) If the data in statement I alone are sufficient to answer (3) (D) (4) (C)
the question, while the data in statement II alone are
DIRECTIONS (Qs. 61 - 64): The pie-graph given below shows
not sufficient to answer the question
the breakup of the cost of construction of a house. If the total
(B) If the data in statement II alone are sufficient to answer cost of construction is `15,00,000, answers the questions given
the question, while the data in statement I alone are below:
not sufficient to answer the question
(C) If the data either in statement I alone or in statement II
alone are sufficient to answer the question
(D) If the data given in both statements I and II together Timber
are not sufficient to answer the question 36°
56. What day is the fourteenth of a given month ? Labour
I. The last day of the month is a Wednesday. Supervisor 90°
II. The third Saturday of the month was seventeenth. 54°
(1) (B) (2) (A)
(3) (D) (4) (C) Steel
Cement
54°
57. In a certain code ‘13’ means ‘stop smoking’ and ‘59’ means 72°
‘injurious habit’. What is the meaning of ‘9’ and ‘5’
Bricks
respectively in that code ? 54°
I. ‘157’ means ‘stop bad habit’.
II. ‘839’means ‘smoking is injurious’.
(1) (B) (2) (A)
61. The amount spent on labour is:
(3) (D) (4) (C)
(1) ` 90,000 (2) ` 2,50,000
58. Vipin’s and Javed’s salaries are in the proportion of 4 : 3
(3) ` 3,60,000 (4) ` 3,75,000
respectively. What is Vipin’s salary?
I. Javed’s salary is 75% that of Vipin’s salary. 62. The amount spent on bricks, steel and cement is what
percentage of the total cost of construction?
II. Javed’s salary is Rs.4500.
(1) 50% (2) 54%
(1) (B) (2) (A)
(3) 72% (4) 75%
(3) (D) (4) (C)
59. At what time did Sonali leave her home for office? 63. The amount spent on timber is what percent of the amount
spent on cement?
I. Sonali received a phone call at 9.15 a.m. at her home.
(1) 36% (2) 50%
II. Sonali’s car reached office at 10.15 a.m., 45 minutes
after she left her residence. (3) 72% (4) 18%
(1) (B) 2. (A) 64. Out of total cost of construction what amount has been spent
(3) (D) 4. (C) on labour and supervision?
60. In what proportion would Raj, Karan and Altaf distribute (1) ` 1,44,000 (2) ` 3,00,000
profit among them? (3) ` 6,00,000 (4) ` 7,50,000
I. Raj gets two - fifth of the profit. 65. The following table shows the number of boys and girls of
II. Karan and Altaf have made 75% of the total different schools that have participated in a scholarship test
investment. over five years.

SPACE FOR ROUGH WORK


Mock Test-5 MT-113

School A B C D 68. How many persons who like Hindi and Tamil language but
not English language?
Year Boys Girls Boys Girls Boys Girls Boys Girls
(1) 40 (2) 34
2001 300 80 280 60 320 84 450 70 (3) 50 (4) 30
2002 320 70 300 80 424 100 320 60 69. How many persons are there who like both Hindi and
2003 340 90 420 120 230 70 360 90 English language but not Tamil language?
2004 370 100 480 140 360 120 500 120 (1) 44 (2) 34
How many girls more participated in year 2004 as compared (3) 14 (4) 40
to 2003 for all schools taken together? 70. How many persons like Tamil language ?
(1) 110 (2) 370 (1) 200 (2) 164
(3) 480 (4) 210 (3) 184 (4) 244
66. Two women and two men are playing cards and are seated 71. How many persons are there who likes only English
at North, East, South and West of a table. No woman is language?
facing East. Persons sitting opposite to each other are not (1) 180 (2) 90
of the same sex. One man is facing South. Which directions (3) 50 (4) 40
are the women facing? 72. How many persons likes all the three language ?
(1) East and West (2) South and East (1) 40 (2) 34
(3) North and East (4) North and West (3) 50 (4) 30
67. A postman was returning to the post office which was in DIRECTIONS (Qs. 73-76): In the following diagram rectangle
front of him to the north. When the post office was 100 represents men, Triangle represents educated, Circle represents
meters away from him, he turned to the left and moved 50 urban and square represents government employees.
meters to deliver the last letter at Shanti villa. He then moved
in the same direction for 40 meters, turned to his right and 8
moved 100 meters. How many meters was he away from 10 7
the post office? 6 13
12
(1) 0 (2) 90
9 4
(3) 150 (4) 100 14
DIRECTIONS (Qs. 68-72): Study the diagram given below and 11
answer each of the following questions. 5
73. Which one of the following represents the educated men
20 but not urban ?
50 (1) 9 (2) 5
(3) 4 (4) 11
34 30 60 80 74. Which one of the following represents a woman who is
40
urban as well as government employee?
14 40 (1) 7 (3) 13
40
(3) 10 (4) 6
75. Which one of the following represents the government
Persons who likes Hindi
employee who is men but not urban ?
(1) 19 (2) 15
Persons who likes English
(3) 13 (4) 11
Persons who likes Tamil

SPACE FOR ROUGH WORK


MT-114 Target NTSE
76. Which one of the following represents the educated men 82. In the following questions, select the missing number from
and live at urban ? the given series
(1) 9 (3) 5 84, 42, 44, 22, 24, 12, ?
(2) 4 (4) 11 (1) 20 (2) 14
77. In a certain code language, ‘JOCKEY’ is written as (3) 24 (4) 28
‘@!^*?>’ and ‘STREAM’ is written as ‘=$/?<+’. How is 83. Find the minimum number of straight lines required to make
‘JAMMER’ written in that code language? the given figure.
(1) @<++?= (2) @<++?/
(3) @<*+?/ (4) @<++$/
78. In the following questions, select the missing number from
the given series.

93 199 961

57 157 944
(1) 13 (2) 15
(3) 17 (4) 19
36 42 ? 84. What is the minimum number of colour pencils required to
fill the spaces in the below figure with no two adjacent
(1) 81 (2) 17 spaces have the same colour ?
(3) 20 (4) 22
79. If “*” denotes “added to”, “&” denotes “divided by”, “@”
denotes “multiplied by” and “%” denotes “subtracted from”,
then 144 & 16 @ 4 % 2 * 8 = ?
(1) 43 (2) 48
(3) 42 (4) 40
80. In the following question, which one set of letters when (1) 5 (2) 2
sequentially placed at the gaps in the given letter series shall (3) 3 (4) 6
complete it? 85. Choose the alternative which is closely resembles the water
ab_ccd_b_c_d image of the given combination/figure.
(1) cccd (2) babc ADVAN C E
(3) bdbc (4) dccc (1) (2)
81. A series is given with one term missing. Choose the correct (3) (4) None
alternative from the given ones that will complete the series.
86. Mahendra, introducing a girl in a party, said,’ she is the
AZ, BY, CX, ? wife of the grandson of my mother and my mother has only
(1) WD (2) DW one son’. How is Mahendra related to the girl?
(3) DE (4) DX (1) Husband (2) Father-in-law
(3) Grandfather (4) Father

SPACE FOR ROUGH WORK


Mock Test-5 MT-115

87. P’s father is Q’s son. M is the paternal Uncle of P and N is (1) Conclusion I follows
the brother of Q. How is N related to P? (2) Conclusion II follows
(1) Nephew (2) Cousin (3) Neither I nor II follows
(3) Grand Father (4) None of these (4) Both I and II follows
91. Sumit wants to get his clothes washed. The laundry service
88. In the given matrix, first now and the first column consist
charges in his city are such that for the first kg of clothes,
of symbols and numbers respectively, the combination of
he is charged ` 150 and after that, he is charged ` 50 for
those would be the code for specific alphabets given in other
every kg that follows. If he gets 10kg of clothes washed,
cells. For example, the code for ‘G’ could be 1$ or 2@. In
how much money does he pay for the laundry service?
the same manner, what from the given alternatives will be
(1) ` 250 (2) ` 450
the correct code for ‘MOBILE’ ?
(3) ` 650 (4) ` 600
$ @ # %
92. A girl is standing facing towards the east. She turns 45
0 P H K R degree in the anticlockwise direction and then turns 180
1 G N L D degree in the clockwise direction. Which direction is she
2 N I W B facing now?
3 M O Q A (1) West (2) South
4 S M R S (3) North (4) South-West
5 D P S G 93. Find the part which represents those actors who are also
6 E S E R singers?
(1) 3@ 3$, 2#, 2@, 1#, 6$
(2) 3$, 3@, 2%, 2@, 1#, 6$ a
(3) 3$, 3@, 2#, 2@, 1%, 6% College
Students b Actors
(4) 3$, 3%, 2#, 2@, 1#, 6$ d c
89. Bumrah, Jaggu, Mahi, Yuvi and Sehwag play cricket. f
e
Sehwag scored more runs than Yuvi but lesser runs than
Mahi. Bumrah scored more runs than Mahi but lesser runs g Singers
than Jaggu. Who scored the maximum runs?
(1) Yuvi (2) Bumrah (1) a (2) b
(3) Mahi (4) Jaggu (3) c (4) f
94. Vishu, Pooja, Vishakha, Rani and Ram are sitting in a line.
90. In the following question, two statements are given each
Pooja is third to the extreme right end. Vishu is second to
followed by two conclusions I and II. You have to consider
the left of Pooja. Vishakha is to the right of Pooja. Rani is
the statements to be true even if they seem to be at variance
third to the right of Ram, who is the immediate neighbour
from commonly known facts. You have to decide which of
of Vishu. Who is sitting in the middle?
the given conclusion, if any, follows from the given
(1) Pooja (2) Ram
statements.
(3) Visakha (4) Rani
Statements:
95. There are five rows in a class in which five children are
I. Ramaa is brilliant sitting behind each other in five different rows in the
II. Ramaa is a woman following manner:
Conclusion: A is sitting behind C but in front of B.
I. Women are brilliant
C is sitting behind E.
II. Ramaa is brilliant as she is a woman
D is sitting in front of E.

SPACE FOR ROUGH WORK


MT-116 Target NTSE
Then in which order are they sitting from first to the last G is 2 km to the east of C.
row? D is exactly in the middle of B and E.
(1) DECAB (2) BACED 97. A is in the middle of
(3) ACBDE (4) ABEDC (1) E and C (2) E and G
96. If the below alphabet is written in the reverse order, which (3) F and G (4) G and C
will be the fifth letter to the left of the ninth letter from the
98. Which two villages are the farthest from one another?
right?
(1) D and C (2) F and E
A B C D E F G H I J K L M N O P Q R S T UV W X Y Z
(3) F and B (4) G and E
(1) P (2) N
99. How far is E from F (in km) as the crow flies?
(3) D (4) M
(1) 4 (2) 20
DIRECTIONS (Qs. 97-99): These questions are based on the
following information: (3) 5 (4) 26
Seven villages A, B, C, D, E, F and G are situated as follows: 100. If MADRAS can be written as ARSMDA, how can
E is 2 km to the west of B. ARKONAM be written in that code?
F is 2 km to the north of A. (1) ROAAKNM (2) ROAKANM
C is 1 km to the west of A. (3) ROAKNNM (4) ROAKNAM
D is 2 km to the south of G.

SPACE FOR ROUGH WORK


Mock Test-5 MT-117

ANSWER SHEET

Name : ..................................School .............................. Test Code............Roll No. : ...................

(1) (2) (3) (4) (1) (2) (3) (4) (1) (2) (3) (4)
1. 35. 68.
2. 36. 69.
3. 37. 70.
4. 38. 71.
5. 39. 72.
6. 40. 73.
7. 41. 74.
8. 42. 75.
9. 43. 76.
10. 44. 77.
11.
45. 78.
12.
46. 79.
13.
47. 80.
14.
48. 81.
15.
49. 82.
16.
50. 83.
17.
51. 84.
18.
52. 85.
19.
53. 86.
20.
54. 87.
21.
55. 88.
22.
23. 56. 89.

24. 57. 90.


25. 58. 91.
26. 59. 92.
27. 60. 93.
28. 61. 94.
29. 62. 95.
30. 63. 96.
31. 64. SPACE FOR ROUGH WORK 97.
32. 65. 98.
33. 66. 99.
34. 67. 100.

Sig. of Invigilator.................................
SPACE FOR ROUGH WORK
5
(Stage 2)

Time : 120 Minutes Max. Marks : 100

INSTRUCTIONS FOR CANDIDATES


Read the following instructions carefully before you answer the questions.
1. There are 100 questions in this test all are compulsory. Each question has 4 choices (1), (2), (3), and (4), out of which
ONLY ONE is correct.
2. Answers are to be given on separate answer sheet provided at the end of this paper. You can tear this answer sheet
along the cut marks provided.
3. If you do not know the answer to any question, do not waste time on it and pass on to the next one. Time permitting,
you can come back to the questions, which you have left in the first instance and attempt them.
4. Since the time allotted for this question paper is very limited you should make the best use of it by not spending too
much time on any one question.
5. For all questions 1-100 put a cross mark (×) on the number of the correct alternative on the answer sheet against the
corresponding question number by using a blue ball point pen. Example

Correct Way : Q.No. Alternatives


1. 1 2 3 4

Wrong Way : Q.No. Alternatives


1. 1 2 3 4
1. 1 2 3 4

6. Rough work can be done anywhere in the booklet but not on the answer sheet/loose paper.
7. Every correct answer will be awarded one mark. There is no negative marking.
8. After completion of this test you can evaluate your performance by comparing your answers with answers provided
in solution booklet.
MT-120 Target NTSE
1. In your botany class an experiment was performed that (1) A - (t); B - (s); C - (q); D - (r); E - (p)
messed up with the levels of gibberllins in a plant. As a
(2) A - (s); B - (t); C - (q); D - (p); E - (r)
result of an unplanned experiment that went wrong the
height of plant was found to be drastically low. Which of (3) A - (s); B - (t); C - (q); D - (r); E - (p)
these hormones is likely to be affected: (4) A - (s); B - (q); C - (t); D - (r); E - (p)
(1) Auxin (2) Cytokinin 5. A cell will swell up if
(3) Gibberellins (4) Abscisic acid (1) the concentration of water molecules in the cell is higher
2. Consider the following statements : than the concentration of water molecules in
surrounding medium.
(a) The amount of usable energy remains constant as it is
passed from one trophic level to another. (2) the concentration of water molecules in surrounding
medium is higher than water molecules concentration
(b) The energy within an ecosystem is fixed and never
changes. in the cell.

Which of these statement(s) is/are correct? (3) the concentration of water molecules is same in the cell
and in the surrounding medium.
(1) (a) only (2) (b) only
(4) concentration of water molecules does not matter.
(3) Both (a) and (b) (4) Neither (a) nor (b)
6. The growing tip of the plant was covered using an opaque
3. Consider the following statements:
object as given below: The possible outcomes would be:
(a) Light soil has good water retention capacity.
(b) A common method of introduction of desired trait in a
crop plant is hybridisation.
(c) Mullets are marine fish farmed in sea water.
Which of the above statements are correct?
(1) (a) and (b) (2) (b) and (c)
(3) (a), (b) and (c) (4) None of these
(1) The stem tip curves and grows towards light
4. Match column I with column II and select the correct answer
using the codes given below: (2) The stem grows straight as rest of the part receive light

Column I Column II (3) The entire plant drfits towards light


(4) none of above
(A) Aspergillus, Penicillium (p) Gymnosperms
7. Consider the following statements:
and Agaricus
(a) High blood pressure is an infectious disease.
(B) Ulothrix, Cladophora (q) Bryophytes
(b) Penicillin is effective against bacteria because it
and Chara
inhibits their wall formation.
(C) Riccia, Marchantia (r) Pteridophytes
(c) AIDS spreads, through sex, blood to blood contact
and Funaria and from mother to child.
(D) Marsilea, ferns and (s) Fungi Which of the above statements(s) is/are correct?
horse-tails (1) (a) and (b) (2) (b) and (c)

(E) Pinus, Cycas and Gingko (t) Algae (3) (a), (b) and (c) (4) None of these

SPACE FOR ROUGH WORK


Mock Test-5 MT-121

8. On your way back from school to tuition centre you saw a 12. Find out the height of hybrid tall plant (Tt).
dog barging at you. The very minute you saw the dog you
ran away with all your might. Which of the below hormones Pure Pure
are responsible for this immediate response: Tall plant Dwarf plant
(1) Adrenalin (2) Thyroxine Height Height
Parent TT tt
(3) Dopamine (4) Serotonin = 15 cm = 5 cm
9. As a part of an adverstisement campaign ,a reputed
company that sells salt conducted a study in which the Gamete T t
diet intake by two families were observed. It was found
that one family uses iodised salt sold by the company and
the other one uses normal salt. Which of the families are
more likely to be :
(1) Affected by goitre and thyroid issues F1 generation Tt Height = ?
(2) Affected by issues with digestion
Hybrid tall
(3) Affected by issue with circulation
(4) None of the above (1) 15 cm (2) 10 cm
10. When the tip of stem was covered with an opaque subtance, (3) 5 cm (4) Can’t say
it was found that the stem tip curved towards the sun. On 13. Assertion : Every biological system resist a change and
observing this, Sanjay and his brother Ranjeev got into an wants to remain in state of equilibrium.
argument regading the cause of th curvature. Sanjay argued
Reason : Climax communities of an ecosystem are produced
that this is due to auxin whereas Ranjeev said it was due to
after several changes it has gone through succession.
cytokinin that responsible for shoot formation.Which of
(1) Both A and R are individually true and R is the correct
the following is true?
explanation of A:
(1) Sanjay is right (2) Both A and R are individually true but R is not the
(2) Ranjeev is right correct explanation of A.
(3) Both of them are wrong, the curvature was due to (3) A is true but R is false
abscissic acid (4) A is false but R is true.
(4) Both of them are right, the curvature due to covering 14. Consider the following statements:
was due to both cytokinin and auxin (a) Sudden action in response to something in the
11. Which of the following would stop evolution by natural environment is called reflex action.
selection from occurring? (b) Sensory neurons carry signals from spinal cord to
(1) If humans became extinct because of a disease epidemic. muscles.
(2) If a thermonuclear war killed most living organisms and (c) Motor neurons carry signals from receptors to spinal
changed the environment drastically. cord.
(3) If ozone depletion led to increased ultraviolet radiation, (d) The path through which signals are transmitted from a
which caused many new mutations. receptor to a muscle or a gland is called reflex arc.
(4) If all individuals in a population were genetically Which of the above statements are correct?
identical, and there was no genetic recombination, sexual (1) (a) and (b) (2) (a) and (c)
reproduction, or mutation. (3) (a) and (d) (4) (a), (b) and (c)

SPACE FOR ROUGH WORK


MT-122 Target NTSE
15. Consider the following statements. (3) Natural gas > Charcoal > Bituminous coal > Wood
A. Baking soda is an acidic salt whereas washing soda is (4) Natural gas > Bituminous coal > Wood > Charcoal
a basic salt. 19. A test-tube contains a colourless solution of potassium
B. The aqueous solution of common salt does not change iodide. The presence of iodine can be tested by:
the colour of either red litmus solution or blue litmus (1) adding a few drops of blue litmus solution.
solution.
(2) adding a few drops of copper sulphate solution.
C. Amla tastes sour whereas soap solution is bitter in taste.
D. The water extract of spinach does not change the colour (3) passing carbon dioxide gas through the solution.
of blue litmus solution. (4) passing chlorine gas through the solution.
Which alternative has the correct statements? 20. A student tested the pH of distilled water and found that
(1) A, B and C (2) A and C the colour of pH paper changed to green. He checked the
(3) B and C (4) B and D pH again after dissolving a pinch of common salt in water.
16. Ram took 2-3 crystals of KMnO4 and dissolved them in 50 The colour of the pH paper this time was:
mL of water. He took 10 mL of this solution (half test-tube) (1) green (2) yellow
and added 40 mL of water (2 test-tubes). He took 10 mL of (3) red (4) blue
new solution and added 40 mL of more water. He kept on
21. Match column I with column II and select the correct answer
diluting (adding water) like this 5 to 6 times and labelled the
using the codes given below:
solutions as I, II, III, IV, V and VI. Which of the following is
correct on the basis of experiment? Column I Column II

(1) The colour of solution is green. 14 13


(A) 6 C and 6 C (p) Different number
(2) The order of darkness of purple coloured solutions is of neutrons
I > II > III > IV > V > VI
40 40
(3) The darkness of purple coloured solutions is (B) 20 C and 18 Ar (q) Different number of

I < II < III < IV < V < VI protons and


(4) One crystal of KMnO4 has one tiny particle. neutrons

17. Consider the following statements: (C) K (19) (r) Non-metal


(D) Chlorine (17) (s) Metal
(a) In extraction of Fe, SiO2 is flux
(1) A – (p); B - (q); C - (r); D - (s)
(b) Tin is purified by liquation.
(2) A – (q); B - (p); C - (s); D - (r)
(c) Bauxite is purified by chemical separation.
(3) A – (p); B - (q); C - (s); D - (r)
(d) Silver turns black due to formation of Ag2O.
(4) A – (p); B - (s); C - (q); D - (r)
Which of the above statement(s) is/are correct? 22. Two chemical species X and Y combine together to form a
(1) Only (d) (2) Only (a) product P which contains both X and Y
(3) (b) and (c) (4) (a) and (d) X+ Y®P
18. The calorific values of wood, bituminous coal, natural gas X and Y cannot be broken down into simpler substances by
and charcoal vary in the order – simple chemical reactions. Which of the following
(1) Wood > Bituminous coal > Natural gas > Charcoal concerning the species X, Y and P are correct?
(2) Wood > Bituminous coal > Charcoal > Natural gas

SPACE FOR ROUGH WORK


Mock Test-5 MT-123

(a) P is a compound 27. Consider the following statements:


(b) X and Y are compounds (a) Structural formula of 2-butyne contains one triple bond
(c) X and Y are elements and 4 single bonds between carbon atoms.
(d) P has a fixed composition (b) Both alcohol and carboxylic acids react with sodium
(1) (a), (b) and (c) (2) (a), (b) and (d) metal and give H2 with effervescence.
(3) (b), (c) and (d) (4) (a), (c) and (d) (c) Esters are used in making perfumes and flavouring
23. Which of the following statements is not correct? agents.
(1) All metal carbonates react with acid to give a salt, water (d) Ethyl alcohol is obtained by the fermentation of
and carbon dioxide. molasses with enzymes in the absence of air.
(2) All metal oxides react with water to give salt and acid Which of the above statement(s) is/are correct?
(3) Some metals react with acids to give salt and hydrogen. (1) Only (c) (2) (c) and (d)
(4) Some non-metal oxides react with water to form an acid. (3) (a), (b) and (c) (4) (b), (c) and (d)
24. Consider the following statements: 28. A man walks on a straight road from his home to market
(a) Green crystals of ferrous sulphate become dirty white 2.5km. away with a speed of 5 km/h. Finding the market
upon strong heating. closed, he instantly turns and walks back home with a speed
of 7.5 km/h. The average speed of the man over the interval
(b) The chemical formula of aluminium phosphate is
of time 0 to 40 min. is equal to –
Al(PO4)3.
(1) 5 km/h (2) 25/4 km/h (3) 30/4 km/h (4) 45/8 km/h
(c) Silver salts are mostly sensitive to light.
29. On a smooth plane surface (figure) two block A and B are
(d) The gas released in respiration is oxygen.
accelerated up by applying a force 15 N on A. If mass of B is
Which of the above statements is correct?
twice that of A, the force on B is
(1) Only (a) (2) Only (b)
(3) Only (c) (4) Only (d)
15 N A B
25. When a little sulphur in a spoon is heated, it burns with a
blue flame which slowly disappears after some time and we
can feel a pungent odour. This pungent odour is due to
(1) 30 N (2) 15 N (3) 10 N (4) 5 N
(1) carbon dioxide (2) sulphur dioxide
(3) sulphur gas (4) sulphuric acid 30. The weight of an object in the coal mine, sea level and at the
26. Select the correct statements top of the mountain, are respectively W1, W2 and W3 then
(i) Plants use CO2 during the process of photosynthesis. (1) W1< W2 > W3 (2) W1= W2 = W3
(ii) CO2 dissolved in water reacts with lime stone present (3) W1< W2 < W3 (4) W1> W2 > W3
in rocks to form A which is absorbed by marine animals.
31. A block of ice floats in water such that the temperature of
The compound A helps in the formation of shells by
water as well as ice is 0°C. The weight of ice is equal to
marine animals.
(iii) The % of oxygen in air dissolved in water is about (1) weight of water displaced by the immersed part of ice
32-35%. (2) volume of water displaced by the immersed part of ice
(iv) Aquatic animals use dissolved oxygen to respire.
(1) (i) only (2) (i), (ii) (3) weight of water displaced by the complete block of ice
(3) (i), (ii), (iii) (4) All of these (4) None of these

SPACE FOR ROUGH WORK


MT-124 Target NTSE
32. In which of the following situations, the work done by a 37. A wire X is half the diameter and half the length of a wire Y
force on a body will be positive. of similar material. The ratio of resistance of X to that of Y is
(1) body does not move (1) 8 : 1 (2) 4 : 1
(3) 2 : 1 (4) 1 : 1
(2) body moves perpendicular to the direction of applied
force 38. A student connects a coil of wire with a sensitive
galvanometer as shown in figure. He will observe the
(3) body moves along the direction of applied force
deflection in the galvanometer if bar magnet is
(4) body moves opposite to the direction of applied force
33. Which of the following statements is wrong? Axis of coil

(1) Sound travels in a straight line.


(2) Sound travels as a wave.
(3) Sound is a form of energy. G
(4) Sound travels faster in vacuum than in air.
(1) one of the faces of the coil placed near and parallel to
34. Out the following which statement is correct? the axis of the coil.
(1) A pulse is of long duration (2) placed near one of the faces of the coil and
(2) A pulse is a short and sudden disturbance perpendicular to the axis of the coil.
(3) Clapping of hands once produces a wave (3) placed inside the coil.
(4) A pulse transports mass from one place to another (4) moved towards or away from the coil parallel to the
35. A man driving a car can read a distant road sign clearly but axis of the coil.
finds difficulty in reading the odometer on the dashboard of 39. The energy stored as heat in the earth is
the car. Which of the following statement is correct about
(1) fossil fuels (2) bio-mass
this man?
(3) geo-thermal energy (4) hydro-electricity
(1) The near point of his eyes has receded away.
(2) The near point of his eyes has come closer to him. 40. Four situations are given below-

(3) The far point of his eyes has receded away. I. An infinitely long wire carrying current
(4) The far point of his eyes has come closer to him. II. A rectangular loop carrying current
36. A ray of light falls on a plane mirror A kept at an angle 90° to III. A solenoid of finite length carrying current
mirror B as shown in the figure. The angle N is
IV. A circular loop carrying current.
(1) 30°
30°
In which of the above cases will the magnetic field produced
x
be like that of a bar magnet?
(2) 45°
A (1) I (2) I and III
(3) 60° N
90° (3) Only III (4) Only IV
B Y
(4) 90°

SPACE FOR ROUGH WORK


Mock Test-5 MT-125

41. If a1, a2, a3, a4 are the roots of equation 47. The area of triangle formed by the points (p, 2 – 2p),
(1 – p, 2p) and (– 4 – p, 6 – 2p) is 70 sq. units. How many
( )
x 4 + 2 - 3 x 2 + 2 + 3 = 0, then the value of integral value of p are possible?
(1) 2 (2) 3
(1 – a1) (1 – a2) (1 – a3) (1 – a4) is
(3) 4 (4) None of these
(1) 2 3 (2) 5 48. Solve the equations : 4(2x – 1 ) + 9(3y – 1 ) = 17 and
3(2x) – 2(3y) = 6.
(3) 1 (4) 4
(1) (x, y) = (2, 1) (2) (x, y) = (–2, –1)
42. If 4x 4 + 12x 3 + 25x 2 + 24x + 16 = ax2 + bx + c, then which (3) (x, y) = (1, 2) (4) (x, y) = (2, – 1)

The roots of the equation, x - 1 + x - 3 = 2 1 , (where


of the following is true ?
49.
(1) 2b = a – c (2) 2a = b + c x-2 x-4 3
(3) 2b = a + c (4) 2b = c – a x ¹ 2, 4 ) are
43. The roots of 2x - 3 + 3x - 5 - 5x - 6 = 0 are (1) 6 + 10, 6 - 10 (2) 6 + 2 10, 6 - 10
7 (3)
(1) 2 only (2) only 6 + 6 10, 6 - 6 10 (4) 6 + 2 10, 6 - 2 10
6
50. If x + 3 is the common factor of the expressions ax2 + bx +1
7
(3) 2 or only (4) None of these - (9 a + 3 p)
6 and px2 + qx – 3, then =
3b + q
2x
44. If x = log3 log 2 log 2 256 , then 2log 4 2 = ______. (1) –2 (2) 2
(3) 3 (4) –1
(1) 4 (2) 8
51. A circle with radius 2 is placed against a right angle. Another
(3) 2 (4) 1 smaller circle is also placed as shown in the adjoining figure.
1 1 1 What is the radius of the smaller circle?
45. If the roots of + = are equal in magnitude but
x+a x+b c
opposite in sign, then the product of the roots is
æ a 2 + b2 ö æ a 2 + b2 ö
(1) -ç
ç 2 ÷
÷
(2) - çç ÷
÷
è ø è 4 ø

a+b a 2 + b2 (1) 3-2 2 (2) 4-2 2


(3) (4)
2 2 (3) 7-4 2 (4) 6 - 4 2
46. In the following figure, DPQR is right angled at R and S is
52. The sum of first n terms of the series :
the mid-point of hypotenuse PQ. If RS = 25 cm and PR=48
cm, then find QR. (5a – b) + (3a – 2b) + (a – 3b) + ..... is (– 280a – 210b). Then
P what is the value of n?
(1) 21 (2) 18
S
(3) 16 (4) 20
53. If tan q + cot q = 2, then the value of tann q + cotn q
Q R is _______.
(1) 7 cm (2) 25 cm (1) 2 (2) 2n
(3) 14 cm (4) cannot be determined (3) 2n – 1 (4) 2n +1

SPACE FOR ROUGH WORK


MT-126 Target NTSE
61. Consider the following statements-
54. 63 + 56 =
(a) Ho Chi Minh was the chairman of the Democratic
(1) 4
7( 2 + 3) (2) 4
7( 3 + 1) Republic of Vietnam.
(b) He founded the Vietnamese communist party.
4 4
(3) 7( 3 + 5) (4) 7( 2 + 1)
(c) He took control of South Vietnam after Vietnam split.
55. A thin hollow hemispherical sailing vessel is made of metal (d) He gave help to the National Liberation Front.
surmounted by a conical canvas tent. The radius of the
Which of the above statements are correct?
hemisphere is 14 m and the total height of the vessel
(including the height of tent) is 28 m. Find area of metal (1) (a), (b) and (c) (2) (b), (c) and (d)
sheet and the canvas required. (3) (a) and (d) (4) (b) and (c)
(1) 1232 (2) 1322 62. Consider the following events and arrange them in their
(3) 2132 (4) 2312 correct chronological order -
56. In the figure (not to scale), O is the centre of the circle. PB I. Swadeshi Movement
= PC, ÐPBO = 25° and ÐBOC = 130°, then find
ÐABP + ÐDCP. II. Home Rule Movement

A D III. Anti-Rowlatt Act Movement


P
IV. Khilafat Movement
B O C (1) I, II, III, IV (2) I, II, IV, III
(3) II, I, IV, III (4) III, I, II, IV
(1) 10° (2) 30° 63. Consider the following statements-
(3) 40° (4) 50°
(a) Industrialisation changed the force of urbanisation in
57. If the ratio of mode and median of a certain data is 6 : 5, then
find the ratio of its mean and median. the modern period.
(1) 8 : 9 (2) 9 : 10 (b) Manchester and Leeds attracted large number of
(3) 9 : 7 (4) 8 : 11 migrants.
58. If a three digit number is chosen at random, what is the (c) By 1750, many people migrated from London.
probability that chosen number is a multiple of 2?
(d) With the technological developments, women got
499 5 industrial jobs.
(1) (2)
900 9
Which of the above statements are correct?
1 500
(3) (4) (1) (a) and (d) (2) (b) and (c)
2 899
(3) (a) and (b) (4) (b) and (d)
59. The hypotenuse of a grassy land in the shape of a right
triangle is 1 metre more than twice the shortest side. If the 64. On the basis of following features identify the correct option-
third side is 7 metres more than the shortest side, The sides I. It is related to the three demands made by Lenin.
of the grassy land are
II. It laid emphasis on the transfer of land to the peasants.
(1) 8 m, 17 m, 15 m (2) 2 m, 16 m, 13 m
III. It advocated on the nationalisation of banks.
(3) 10 m, 4 m, 5 m (4) 7 m, 10 m, 14 m
IV. Lenin argued that the Bolshevik party rename itself as
60. If ap = bq = cr = abc, then pqr =
the communist party through his demands.
(1) p2q + q2r (2) pq + qr + pr
(1) Reign of terror (2) Bloody Sunday
(3) (pq + qr + rp)2 (4) pq (qr + rp)
(3) April theses (4) October Revolution

SPACE FOR ROUGH WORK


Mock Test-5 MT-127

65. Match column I with column II and select the correct answer (2) There was no rainfall in the region for a very long time
using the codes given below: (3) The whole plain was an arid zone without much vegeta-
tion
Column I (Attribute) Column II (Significance)
(4) The area faced heavy flooding
(A) Broken chain (p) Strength Consider the following statements and choose the correct option
(B) Breastplate with Eagle (q) Heroism from the choice given below :
69. Statement I : Hitled said, ‘In my state the mother is the most
(C) Sword (r) Being freed important citizen’.
(D) Crown of oak (s) Readiness to fight Statement II : In Nazi Germany, while boys were taught to
be aggressive, muscular and sted hearted, girls were told
(1) (A) ® q, (B) ® s, (C) ® p, (D) ® r that they had to become good mothers.
(2) (A) ® p, (B) ® r, (C) ® q, (D) ® s (1) Statement I is true but statement II is false
(2) Both Statement I and Statement II are true but Statement
(3) (A) ® r, (B) ® p, (C) ® s, (D) ® q
II is not the correct explanation of Statement I.
(4) (A) ® s, (B) ® q, (C) ® r, (D) ® p (3) Both the Statements are false
66. Why did dalits prefer western clothing? (4) Both Statement I and Statement II are true and
Statement II is the correct explanation of Statement I.
(1) They were under the pressure of Britishers.
70. ‘The poor peasants resort to riots to protest against the use
(2) It gave them more liberating performance is society. of threshing machines’. Why?
(3) They were against the traditional dress. (1) Thresh ing machines had become a sign of
unemployment and bad times
(4) They did not like Hindu way of life and dressing.
(2) They were opposed to machines
67. With reference to the cause of novels
(3) They found machines too difficult to operate
Consider the following statements : (4) The peasants were too poor to buy threshing
(a) Potheri Kunhambu wrote Saraswativijayam in 1892. It machines
was a strong attack on caste opperssion. In this novel, 71. Match Column I with Column II and select the correct option
a young man converted his religion from schedule caste from the codes given below :
to Christianity. Column I Column II
(b) Bhudeb Mukhopadhyay wrote Titash Ekti Nadir Naam. A. Epis tolary (p) Novels is a format in
It is based on the story of the community of fisherfolk. which the story is
published in
Which of these statement(s) is/are correct? instalments each part in
a new issue of a journal
(1) (a) only (2) (b) only
B. Serialised (q) is the normal spoken
(3) Both (a) and (b) (4) Neither (a) nor (b) form of a language
68. Ordinary dust storm took the form of the Black Blizzard in rather than formal
U.S. plains in 1930 because literary form
C. Vernacular (r) is a form of
(1) The entire ploughed landscape was stripped of all grass
which earlier held the land together representation through
writing, drawing,
painting etc.
SPACE FOR ROUGH WORK
MT-128 Target NTSE
D. Satire (s) Novels are written in (4) Manganese is a raw material in cement industry.
the form of series of
letters 76. Consider the locational coordinates of stations X and Y
and mark the correct answer-
(1) A-s, B-p, C-q, D-r (2) A-r, B-q, C-s, D-p
(3) A-p, B-r, C-s, D-q (4) A-q, B-s, C-q, D-r I. X. 50°E longitude and 40°S latitude

72. Who painted “Germania’ and for what occasion? II. Y. 50°E longitude and 60°s latitude

(a) Artist Phillip Veit painted it to celebrate the unification Time of station X will be-
of Germany.
(1) Ahead of Y (2) behind of Y
(b) Phillip Veit painted it to hang from the ceiling of St.
Paul’s where the Frankfurt parliament was held in 1848. (3) Same as station Y (4) Independent of that of Y
(c) Phillip Veit painted it to celebrate Bismarck’s Victory. 77. Assertion (A) : In recent years sugar mill are shifting towards
(d) Artist Phillip Veit prepared the painting of Germania on southern and western states.
silk banner.
Reason (R) : The cooler climate, favourable cooperatives
Which of the above statement/s is/are correct: and availability of sugarcane with higher sucrose content
(1) (a) and (b) (2) (a) and (d) are found in Maharashtra.
(3) only (c) (4) only (b) (1) A and R true and R explains A.
73. Copper-gold-iron-coal are connected with -
(2) A and R are true but R does not explain A.
1. Kolar - Kudremukh - Khetri - Jharia
(3) A is true but R is false.
2. Khetri - Kolar - Kudremukh - Jharia
(4) A is false but R is true.
3. Kudremukh - Kolar - Khetri - Jharia
4. Kolar - Khetri - Jharia - Kudremukh 78. On the basis of ownership industries are categories as-

74. With reference to the characteristics of Himalayan river (1) Large scale and small scale industries
consider the following statements –
(2) Public sector, private sector, joint & cooperative sectors
(a) They have great capacity for erosion.
(3) Basic / Key and consumer Industries
(b) Many of them have their sources is the Inner Himalayas.
(4) Agro based and mineral based Industries
Which of the above statement(s) is/are correct?
79. On the basis of following features identify correct option
(1) (a) only (2) (b) only
(3) Both (a) and (b) (4) Neither (a) nor (b) (I) The ocean origins in mesozoic age

75. What is the importance of Manganese? (II) Divides Gondawana land from Laurasia.

(1) Manganese is a good conductory of electricity. (1) Caspean Sea (2) Bay of Bengal

(2) Manganese produces aluminium. (3) Tethys Sea (4) None of these

(3) Manganese is used in manufacturing of steel and paint


industry.

SPACE FOR ROUGH WORK


Mock Test-5 MT-129

80. Consider the following statements : 83. What is the correct meaning of agglomeration economies?
(1) Many industries set up in rural centres
(1) Gondwana land is the southern part of ancient super
(2) Industries are basically agro-based
continent Pangea.
(3) Many industries tend to come together to make use of
(2) The sedimentary rock which were accumulated in the the advantages offered by the urban centres
geosyncline known as Tethys. (4) Industries set up to produce raw material for secondary
sector
Which of these statement(s) is/are correct? 84. Match Column I with Column II.
(1) (1) only (2) (2) only Column I Column II
(A) Normal species (p) Cattle, sal, pine rodents
(3) Both (1) and (2) (4) Neither (1) nor (2) (B) Endangered species (q) Himalayan brown bear,
81. Which of the following industrial regions is shown on the Wild Asiatic buffalo
given outline map of India? (C) Rare species (r) Asiatic Cheetah, Pink
head duck
(D) Extinct species (s) Black buck, Crocodiles,
Indian Wild ass
(1) A-p, B-r, C-s, D-q (2) A-s, B-p, C-r, D-q
(3) A-q, B-s, C-q, D-r (4) A-p, B-s, C-q, D-r
85. On the basis of following features identify the correct option-
I. He is the real ruler of the country.
II. He presides over the meetings of the Cabinet.
III. It has an important role in the management of foreign
affairs
IV. He is the ex-offices chairman of the planning
(1) Mumbai- Pune industrial region commission.
(2) Gurgaon - Delhi - Meerut industrial region (1) President
(3) Hugli industrial region
(2) Prime Minister
(4) Bangalore- Tamil Nadu industrial region
(3) Chief Justice of Supreme Court
82. Match column I with column II and select the correct answer
using the codes given below: (4) Governor
Column I Column II 86. Consider the following statements-
(A) Public Sector (p) Owned by a group of (a) Democracy aims to provide a fair share to every citizen
Industries people in the natural resources of the country.
(B) Private Sector (q) Owned by individuals (b) A democracy gives every social group equal importance
Industries or firms to avoid any domination in the society.
(C) Joint Sector (r) Owned by state and private
(c) Democracy means rule by the government.
Industries firm
(d) Social democracy means there is different law for
(D) Co-operative (s) Owned by the State
different sections of the society.
Sector Industries
Which of the above statements are correct?
(1) (A) ® s, (B) ® r, (C) ® p, (D) ® q
(1) (a), (c) and (d) (2) (a) and (b)
(2) (A) ® s, (B) ® q, (C) ® r, (D) ® p
(3) (b) and (d) (4) (a), (b), (c) and (d)
(3) (A) ® p, (B) ® s, (C) ® q, (D) ® r
(4) (A) ® q, (B) ® p, (C) ® s, (D) ® r

SPACE FOR ROUGH WORK


MT-130 Target NTSE
87. Consider the following statements- (1) Only I is correct (2) I and II are correct
(a) The UN Security Council is responsible for maintaining (3) Only II is incorrect (4) I and II are incorrect
peace and security among countries. 92. Can the Houses of Parliament be dissolved?
(b) The World Bank gives loans to the individuals for the Select the correct option.
monetary help. (1) Lok Sabha cannot be dissolved because it is a perma-
(c) The UN secretary General is the chief administrative nent house whereas Rajya Sabha can be d i s -
officer of UN. solved.
(d) The UN is only responsible for maintaining (2) Both the Houses of Parliament cannot be dissolved
international law. (3) Rajya Sabha is a permanent house so it cannot be
Which of the above statements are correct? dissolved whereas Lok Sabha can be dissolve
(1) (a), (b) and (c) (2) (a) and (c)
(4) Both Rajya Sabha and Lok Sabha can be dissolved.
(3) (b) and (d) (4) (b), (c) and (d)
93. The Human Development Report ranks countries on the
88. With reference to creation of new states by parliament under
following basis:
Article 3. Consider the following statements-
(i) Income (ii) Health
(a) Parliament can not introduce a Bill to this effect without
(iii) Education (iv) Technology
the prior recommendations of the President.
(1) Only (ii) and (iii) (2) Only (i), (ii) and (iv)
(b) The president is bound to accept the opinion of the
(3) All (i), (ii) and (iii) (4) None of them
affected state in making recommendations to the
parliament. 94. What is the function of Academy of Development Sciences
Which of the above statement(s) is/are correct? (ADS) in Maharashtra?
(1) (a) only (2) (b) only (1) Facilitated a network of NGO's for setting up grain banks
(3) Both (a) and (b) (4) Neither (a) nor (b) in different regions
89. ‘The political executive have more powers than permanent (2) It organizes training and capacity building programmes
executive’. Why? on food security for NGO's
(1) Because hardly any expertse is required in taking (3) It efforts to set up Grain Banks, to facilitate replication
policy decisions through other NGO's and to influence the governments
(2) Because political executive consist of the direct policy on food security are thus paying rich dividends
representatives of the people. (4) All of them
(3) Political leaders are more educated
95. Match Column I with Column II and select the correct
(4) None of these
option from the codes given below :
90. We need rights in a Democracy because
I. Rights protect minorities from the appression of Column I Column II
majority. (A) MNC Buy at cheapt (p) Call centres
II. Right ensure individual liberty. rates from small producers
III. Right guarantee limitation on an individual’s right to (B) IT has helped in (q) Trade barriers
equality.
spreading of production
IV. Rights are placed higher than the government to check
the government’s misuse of power. (C) Indian companies who (r) Garments, footwear,
(1) Only III (2) Only IV have invested abroad sports items.
(3) I, II, III and IV (4) I, II and IV (D) Quotas and taxes on (s) Tata Motors, Ranbaxy,
91. From the following given statements select the correct option imports are used to Infosys
given below :
regulate trade
Statement I : ZANU PF is a political party of Zimbabwe.
Statement II : Elections have been held regularly and always (1) A-p, B-s, C-q, D-r (2) A-s, B-q, C-s, D-r
won by ZANU-PF. (3) A-r, B-q, C-p, D-s (4) A-s, B-p, C-s, D-r

SPACE FOR ROUGH WORK


Mock Test-5 MT-131

96. Annapurna Scheme launched in 2000 fufils the food 99. On the basis of following features identify the correct op-
requirement of tion
(1) indigent senior citizens (2) children (I) Its aims is to liberalise international trade.
(II) Its main function is to establish rules regarding inter-
(3) pregnant ladies (4) young persons national trade and insure that these rules are obeyed.
97. What do you mean by Disguised Unemployment? (III) It has 150 countries as its members.
(1) When a person is seasonally employed (IV) It forced the developing countries to remove trade bar-
(2) When a person appear to be employed riers -
(1) SEZ (2) WTO
(3) When educated people are not able to find job
(3) MNC (4) GDP
(4) None of them
100. A payment made by account payee cheque means
98. When is a person considered poor?
(1) If his/her income or consumption level falls below a (1) anyone can present the cheque and get payment from
given "minimum level" necessary to fulfil basic needs. the bank
(2) If his/her all needs are fulfilled in a fraction of seconds (2) only drawee can deposit the cheque and get credit in his
without any delay. bank account
(3) If his/her desires are taken into consideration by their (3) the cheque can be given by the drawee to third party
boss after considering all other desires.
who can get credit in his account.
(4) All of them

SPACE FOR ROUGH WORK


MT-132 Target NTSE

SPACE FOR ROUGH WORK


Mock Test-5 MT-133

ANSWER SHEET

Name : ..................................School .............................. Test Code............Roll No. : ...................

City : ................... Date : ................... Class : ..................

(1) (2) (3) (4) (1) (2) (3) (4) (1) (2) (3) (4)
1. 35. 68.
2. 36. 69.
3. 37. 70.
4. 38. 71.
5. 39. 72.
6. 40. 73.
7. 41. 74.
8. 42. 75.
9. 43. 76.
10.
44. 77.
11.
45. 78.
12.
46. 79.
13.
47. 80.
14.
48. 81.
15.
49. 82.
16.
50. 83.
17.
51. 84.
18.
52. 85.
19.
53. 86.
20.
54. 87.
21.
55. 88.
22.
23. 56. 89.

24. 57. 90.

25. 58. 91.


26. 59. 92.
27. 60. 93.
28. 61. 94.
29. 62. 95.
30. 63. 96.
31. 64. 97.
32. 65. SPACE FOR ROUGH WORK 98.
33. 66. 99.
34. 67. 100.

Sig. of Invigilator.................................
SPACE FOR ROUGH WORK
SOLUTIONS MT-1

Mental Ability Test MOCK TEST 1


ANS W ER KEY
1 (3) 11 (2) 21 (2) 31 (2) 41 (4) 51 (2) 61 (4) 71 (3) 81 (2) 91 (1)
2 (1) 12 (4) 22 (4) 32 (1) 42 (2) 52 (3) 62 (1) 72 (1) 82 (4) 92 (1)
3 (2) 13 (4) 23 (2) 33 (3) 43 (3) 53 (2) 63 (4) 73 (3) 83 (3) 93 (4)
4 (1) 14 (4) 24 (1) 34 (2) 44 (1) 54 (4) 64 (4) 74 (1) 84 (2) 94 (3)
5 (4) 15 (4) 25 (2) 35 (1) 45 (4) 55 (1) 65 (2) 75 (1) 85 (2) 95 (4)
6 (1) 16 (2) 26 (3) 36 (4) 46 (4) 56 (2) 66 (4) 76 (3) 86 (2) 96 (3)
7 (1) 17 (1) 27 (3) 37 (2) 47 (3) 57 (3) 67 (1) 77 (3) 87 (4) 97 (2)
8 (2) 18 (2) 28 (2) 38 (1) 48 (2) 58 (2) 68 (4) 78 (2) 88 (4) 98 (1)
9 (1) 19 (4) 29 (2) 39 (4) 49 (4) 59 (1) 69 (3) 79 (1) 89 (1) 99 (2)
10 (4) 20 (3) 30 (2) 40 (1) 50 (3) 60 (4) 70 (4) 80 (2) 90 (4) 100 (1)

Hints and Explanations

1. (3) T, Z and H has even position in the alphabet where Q H J M P T


has odd position. Option (4) :
+2 +3 +3 +4
2. (1) Except that, all are used for carrying.
So option (4) does not obey the given rule.
3. (2) All except litre are units or measurement of the mass of
a body. 11. (2) MEAL, MALE & LAME

5 7 9 11 13 15 I N S T R U C T I O N
4. (1) 12. (4)
+2 +2 +2 +2 +2
125 80 45 20 05 13. (4) The second and third letters and fifth and sixth letters
5. (4) are interchanged leaving other letters fixed.
–45 –35 –25 –15 Thus TEACHER is coded as TAECEHR
6. (1) 14. (4) Z = 52 = 26 × 2
7. (1) In three consecutive letters, a, b, c are each repeated ACT = 1 × 2 + 3 × 2 + 20 × 2 = 48 [Alphabetical position
once. Hence the series would be. numbers has been
c ab / a b c / b c a / c a b doubled]
+2 +2 +2 +2 Þ BAT = 2 × 2 + 1 × 2 + 20 × 2 = 46
8. (2) Q P O S R Q U T S W V U Y X W
15. (4) In the code, night is called sunshine. As we sleep in
night, the correct answer is sunshine.
+2 +2 +2 +2 16. (2) From first 2 sentences ‘Ka Ya’ means ‘very intelligent’.
+2 +2 From 1st and 3rd sentences ‘Pu’means ‘you’
\ In first sentence ‘are’ means ‘Bi’
9. (1) –2 –2 –2 –2 17. (1) Clearly the direction of the hour’s hand is North-east.
Y E B W F D U H G S K I Q O L
H N
+1 +2 +3 +4 2 3 4
+2 +3 +2 +3 1 5
12 6 M W E
G I L P U
10. (4) Option (1) :
+2 +3 +4 +5
S
D F I M R
Option (2) :
+2 +3 +4 +5 18. (2) Next train for N. Delhi leaves at 8:30 p.m. Since time
interval between two trains for N. Delhi is 45 minutes.
C E H I Q
Option (3) : A train for New Delhi has left 15 minutes ago.
+2 +3 +4 +5 \ Time of information = 8:30 – 45 + 15 = 8 P.M.
MT-2 Target NTSE
For (Qs. 19 & 20) 25. (2)
26. (3)
Prashant 27. (3) Clearly, Arun is uncle of the lady in the photograph.
Arun — Brother
|
30m Son + Wife mother = lady
|
daugher
Nitin Atul
25m 40m Kunal Mother — B
|
60m A-C
28. (2) Clearly, from the relationship diagram. Y is the brother-
in-law of B.
Dinesh
Married couple
A B
19. (4) Left of Kunal is Atul.
Brother Brother-in-law
North East of Atul is Prashant (obvious from the above
diagrams).
X Y
20. (3) Total distance travelled = 25 + 40 + 60 + 90 = 215 mts. Brothers

21. (2) Ashish leaves his house at 6:40 AM. 29. (2) Both mangoes and apples are fruits.
30. (2) Both Coffee and tea are beverages.
Ashish reaches Kunal’s house at 7:05 AM.
31. (2) Asian Non-Christian females who are professionals are
They finish Breakfast at 7:05 + 0:15 = 7:20 AM. represented by the common figure of all square except
That’s the time when they leave Kunal’s house for thier Christians’s square i.e., 10
office. 32. (1) Asian females who are neither professional nor
22. (4) The day repeats itself after seven days. Christian are denoted by the common figure of Asians
square and Females square i.e., 6
Third Wednesday falls on 15th of the month.
33. (3) Non-Asian professional christian male are represented
Fourth Wendesday will fall on 22nd of the month. by the common figure of christian’s square and
Fifth Wendesday will fall on 29th of the month. professional square i.e., 12
Hence, Friday will fall on 31st of the month. 34. (2) The shaded portion is the common portion of all square
except Asians.
23. (2) After interchanging their positions, position of A from 35. (1) The numbers adjacent to 6 are 2, 3, 4 and 5 (from the
left = 11 first three figure). Hence, number 1 will be opposite
then positions of A form right = 9. to 6.
\ The total no. of people in the row 36. (4) Lebelling all the points of intersection in the given
= (9 + 11) – 1 = 19. figure, we get
24. (1) The sitting arrangement can be shown as follows: A

C
B D
E

E F F

A
Clearly, F is opposite to B.
B D C
For (Qs. 25 & 26) :
The age wise order is as follows : the total number of triangles viz ABC, ABD, ADC,
ABF, ACF, BFD, CFD, CFE, AFE, BFC, and BEC.
Gita > Kusum > Arti > Archana > Suman
SOLUTIONS MT-3

37. (2) A minimum of 2 matchsticks have to be removed from 50. (3) The water image of ‘Z’ is ‘ ’, ‘E’ is ‘ ’, ‘B’ is , ‘R’ is
the arrangement to get two squares. ‘ ’ and ‘A’ is ‘ ’
Solution (51-53):

w R Q S/M C S/M

51. (2)
52. (3)
53. (2)
38. (1) Bottom face of figure 1 means opposite face of 5. It
Solution (54-56):
will be 3 because 4, 1, 6 and 2 are adjacent faces
of 5. (i) Car B is heavier than only two cars. Second heaviest
car is 3m south of car A, it means A is not 2nd heaviest
car. Car D is lighter than atleast two cars, so D cannot
eit

Shoes
39. (4)
he

be 2nd heaviest car. hence either car E or C is second


r

heaviest. Car D is heavier than car A, so A is lightest


Chairs car. In case 2, C is heaviest and in case 1, E is heaviest.
Shoes Case 1 – E > C > B > D > A
ses
Hou Case 2 – C > E > B > D > A

No mediate inference is possible since the middle term (ii) Heaviest car is 4m east of car C, from this condition
is not distributed. Hence none of the conclusion case 2 will be eliminated because car C itself is heaviest
follow. car. lightest car is north east of car D. Car C is somewhere
40. (1) The conclusion (A) is valid as it is clear from the in between car D and car E in a straight line. Car B is
following Venn Diagram: south of car D. We will get final answer.

Cats
A
Dogs

Man 3m

D D
E
C 4m
41. (4) Only (4) is the latterly opposite figure. It is mirror image.
42. (2) Superimposing (2) on the given figure, we are able to
complete it.
B B
43. (3) The second figure is the first figure in another form
(each having two rectangles). Similary, the fourth
figure will be a form of the 3rd figure. E> C> D>A
44. (1) Studying the figures, we find the in the next figure, 54. (4) AE2 = AC2 + CE2
the rectangle gets halved and arrow enters the circle
and retates 90° clockwise direction. AE2 = 9 +16
45. (4) There are four line segments in the odd numbered AE = 5m
figures.
55. (1)
46. (4) There are four line segments in the odd numbered
figures. 56. (2)
47. (3) Alternating figures are identical. Solutions (57-61):
48. (2)
57. (3) if the data in either in statement I alone or in
49. (4)
statement II alone are sufficient to answer the
question. ra=me
MT-4 Target NTSE
58. (2) From statement II- Sarita is mother of Rajesh 67. (1) ATQ,

Swarna(–) Sarita(–) Om(+) GDP of E : GDP of A,C and F


90 : 30 + 70 + 50
3 :5

68. (4) ATQ,


Rajesh
360
Required GDP = 100 + 50 ´ 100 = 240
59. (1) if the data in statement I alone are sufficient to answer ( )
the question, while the data in statement II alone are
69. (3)
not sufficient in answer the question.
70. (4) None of the three statements is supported by the known
S's rank from top = 9th
facts.
R's rank from top = 21st
71. (3) Statements I and II are not supported by the facts.
M's rank from top in the class = 16th Statement III is true because if all storybooks have
60. (4) From I and II, we conclude that Amita went to school pictures and only some have words, then some
by either 11.30 a.m. or 11.35 a.m. train. So we cannot storybooks have both words and pictures.
determine that which train will catch by Amita. 72. (1) In each row, the second figure is obtained from the first
61. (4) From both the statements we cannot find that who figure by reversing the direction of the RHS arrow and
gets lowest salary, either T or U gets lowest salary. the third figure is obtained from the second figure by
reversing the direction of both the arrows.
Solution (62-64):
73. (3) The second figure is obtained from the first figure by
moving the line segment to the opposite side of the
square boundary and replacing it with two similar line
Word Code
segments. Also, the element in the lower-left corner
Fixing mm gets replaced by two similar elements - one placed in
Earth ie the upper-left and the other placed in the lower-right
food as corner.
Award bn 74. (1) It is assumed that assistant will follow the instructions
word cd but it is not assumed that the assistants knows the
Water zq address because his job is to drop the letter in the
letter box.
no st
75. (1) Advertisement suggests the milk because of its quality
Coaching/glass zx/yx
of being pure, then it can be assumed that milk can be
prepared.
62. (1) 76. (3)
63. (4) 77. (3)
64. (4) 78. (2)
Solutions (65-68): 79. (1) The series is: x y z u v / y z u v x/ z u v x y/u v x y z
65. (2) ATQ, 80. (2) In first figure, 5 × 4 + 6 = 26
GDP of B = 100 In second figure, 8 × 3 + 5 = 29
GDP of E = 90 So, missing number in third figure, 6 × 3 + 4 = 22
GDP of C = 70 81. (2) Clearly, In first figure, 6 × 3 – 4 × 2 = 18 – 8 = 10
66. (4) ATQ, In second figure, 9 × 5 – 5 × 3 = 45 – 15 = 30
So, In third figure, 6 × 5 – 2 × 5 = 30 –10 = 20
( 50 - 30)
Required amount = ´ 100 = 40% 82. (4) Number of students who passes = (15 + 1 + 28) = 44.
50
So, Total number of students in the class = 44 + 6 + 5 = 55
SOLUTIONS MT-5

83. (3) Using the proper signs The movements of Raj are as shown in fig.
Expression (3 × 15 + 19) ÷ 8 – 6 = 64 ÷ 8 – 6 = 2 Raj's distance from starting point A
84. (2) Using the proper notations in (2), we get the statement = AE = (AD + DE)
as
= (BC + DE) = (25 + 10) m = 35 m.
5 × 2 ÷ < 10 – 4 + 2 or 5 < 8, which is true.
So. E is to the East of A.
Solutions (85 to 86):
92. (1) From the relationship given in the question, we observe
From the given data, we can make the following table with the help that each of the objects carries something in common
of which rest of the questions can be solved very easily. to one another. A Tennis fan can be a cricket player as
well as student. hence Diagram (A) represents this
Male (40) Female (30) relationship.
Above25 93. (4)
Married 7 12 94. (3)
Unmarried 5 0
95. (4)
Below 25
Married 8 3 96. (3)
Unmarried 20 15 97. (2)
Total 40 30 98. (1) From the figures (i), (ii) and (iv) we find that numbers 6,
1, 5 and 2 appear on the adjacent surfaces to the number
85. (2) There are 15 unmarried girls. 3. Therefore, number 4 will be opposite to number 3.
86. (2) In these 15 unmarried girls no one is above 25. Hence option (1) is the answer.
87. (4) Clearly, the correct sequence is : France, America, India, 99. (2)
Australia, Japan, China.
The two flags in the centre are of India and Australia. A B C D E
88. (4) The sex of Q is not given hence the exact relation ship
b/w N & Q cannot be established. M N
89. (1) Preeti's mother shyama is youngest sister of Dubey & L F
P Q
sister of Prabhat. Therefore Prabhat Preeti's uncle. R O S
90. (4) The movements of the person are from A to F, as shown
in fig. Clearly, the final position is F which is to the K J I H G
Northeast of the starting point A

E 60m F The figure is labelled as shown.


Clearly, there are 3 horizontal lines namely AE, LF and KG.
20 m 10 m C
D 10 m There are 5 vertical lines : AK, BJ, CI, DH and EG.
B 20 m A There are 6 slating lines : LC, KE, IF, LI AG and CF.
91. (1) According to question, Thus, there are 3 + 5 + 6 = 14 straight lines in the figure.
100. (1) In figure (X), one of the dots is placed in the region
10 m common to the circle and the triangle and the other dot
A
D E is placed in the region common to the triangle and the
20 m 20 m square. From amongst the figures (A), (B), (C) and (D),
only figure (A) has both the regions, one common to
circle and triangle and the other common to triangle
B C and square. Hence, figure (A) is the answer.
25 m
MT-6 Target NTSE

SAT (Scholastic Aptitude Test) MOCK TEST 1


ANS WER KEY
1 (4) 11 (1) 21 (4) 31 (2) 41. (1) 51. (1) 61. (1) 71. (4) 81. (1) 91. (2)
2 (4) 12 (1) 22 (1) 32 (1) 42. (4) 52. (3) 62. (1) 72. (3) 82. (4) 92. (3)
3 (4) 13 (1) 23 (4) 33 (3) 43. (2) 53. (3) 63. (3) 73. (4) 83. (4) 93. (1)
4 (1) 14 (3) 24 (3) 34 (1) 44. (2) 54. (1) 64. (1) 74. (3) 84. (1) 94. (1)
5 (3) 15 (2) 25 (2) 35 (1) 45. (4) 55. (1) 65. (1) 75. (4) 85. (3) 95. (3)
6 (3) 16 (3) 26 (4) 36 (4) 46. (2) 56. (3) 66. (4) 76. (4) 86. (3) 96. (2)
7 (4) 17 (4) 27 (3) 37 (3) 47. (3) 57. (4) 67. (2) 77. (2) 87. (3) 97. (2)
8 (1) 18 (1) 28 (4) 38 (3) 48. (3) 58. (2) 68. (3) 78. (3) 88. (3) 98. (2)
9 (4) 19 (4) 29 (2) 39 (3) 49. (2) 59. (3) 69. (3) 79. (4) 89. (1) 99. (2)
10 (2) 20 (1) 30 (3) 40 (1) 50. (2) 60. (4) 70. (4) 80. (2) 90. (2) 100. (1)

Hints and Explanations

1. (4) Transportation is the movement of materials from one 9. (4) 10. (2)
part to another, usually from the region of their 11. (1) The new progeny will have tails because the length of
availability to the region of their use, storage or
tails or presence of tail depends on genetic composition.
elimination. Transportation occurs in all organisms, from
Continuous breeding will show no change in the
microscopic ones to large sized trees and animals.
physical appearance of tail.
2. (4) Green plants are called producers because they not
only manufacture food for themselves but also for the 12. (1) Insulin is produced by b-cells of islet of Langerhans of
rest of members of biotic community. Primary carnivores pancreas. Cells, tissues and organs recognise and
are called secondary consumers because they feed on absorb glucose only in the presence of insulin. In
first order or primary consumers (herbivores). Flow of deficiency of insulin, the cells fail to recognise and
energy in ecosystem is unidirectional from sun to absorb glucose. Extra glucose is excreted through
plants, plants to animals, animals to animals, organic urine. The disorder is called diabetes mellitus.
remains to decomposers and dissipation as heat. Ozone 13. (1) Biodegradable wastes are wastes which are broken
(O3) is a triatomic molecule made up of three atoms of
down and disposed off naturally by saprophytes or
oxygen. Chlorofluorocarbons (CFCs) are ozone
decomposers. They are generally organic wastes, e.g.,
depleting substances which are highly stable,
odourless, synthetic gaseous substances used as garbage, sewage, livestock waste, used tea leaves,
aerosol propellants, coolants, refrigerants, etc. waste paper, leather, wood and jute articles, food
3. (4) 4. (1) leftover, etc.
5. (3) Ozone layer present in stratosphere functions as a 14. (3) 15. (2)
shield against high energy and harmful ultraviolet 90 2+ , atomic number = no. of protons = 38
radiations. In the absence of ozone layer, the extremely 16. (3) In 38 Sr
harmful UV radiations will reach earth, cause large scale Since it has lost 2e– so no. of electrons = 38 – 2 = 36
mutations to organisms, reducing photosynthesis, Atomic mass = no. of protons + no. of neutrons
blinding of animals and humans, developing skin 90 = 38 + no. of neutrons
cancer and herpes and death of young ones. By filtering 90 – 38 = no. of neutrons
out harmful UV radiations, ozone protects the immune 52 = no. of neutrons
system of living beings against various diseases. Hence, no. of neutrons = 52, no. of protons = 38.
6. (3) 7. (4) 17. (4) Element A belong to halogens (Group VII) group and is
8. (1) Culture fishery is cultivating, rearing and harvesting a non-metal. While element B belongs to alkali metal
of fish. Bee keeping or apiculture is the rearing, care group (Group I) and is a metal.
and management of honey bees for obtaining honey, 18. (1) Solution is a homogeneous mixture of two or more pure
wax and other substances. Broilers are chicken, which substances, the relative ratio of which can be varied
are 7 - 8 weeks old and raised for meat. Draught animals within certain limits.
are strong and sturdy. They are the beast of burden
used for ploughing land and transporting materials from 19. (4) When sodium hydrogen carbonate is added to acetic
one place to another. acid CO2 gas is evolved.
SOLUTIONS MT-7

CH3COOH + NaHCO3 ¾¾
® therefore we have = 40 × 10 × 4 = 1600 J
CH3COONa + H2O + CO2 Hence power developed by the girl
W 1600
CO2 reacts with water to form carbonic acid. P= = = 53.33 W
t 30
CO2 + H2O ¾¾
® H2CO3 34. (1) To here echo the time difference between original
20. (1) sound and reflected sound should be more than 0.1
second.
21. (4) Zinc, iron and copper are less reactive than aluminium.
This is further supported by the reactivity series of 1
So, minimum distance = × (344 × 0.1) m = 17.2 m
metals. 2
23. (4) Sulphur dissolves in carbon disulphide to form a 35. (1) A mechanical wave in air can never be transverse.
solution which is homogeneous in nature. It does not 36. (4) Reflection of light : It is the turning back of light in the
show any Tyndall effect. same medium.
Refraction of light : It is the bending of light when
24. (3) Detergents are sodium salts of long chain sulphonic
acid. Thus they are non-biodegradable. Methane is passes from are medium to another medium.
also known as marsh gas. Hard water contains chlorides Dispersion of light : It is the splitting of white light
and sulphates of Ca 2+ and Mg2+ ions. Vinegar is into its seven constituent colours while passing
aqueous solution of acetic acid and is used in through a prism.
preserving pickles. Total internal reflection : For total internal reflection
25. (2) De-greasing of metals can be done by NaOH. light must goes from denser to rarer medium and in
26. (4) denser medium angle of incidence > critical angle.
27. (3) When a metal reacts with dilute nitric acid, then 37. (3) u = – 25 cm V = 2.5 cm
hydrogen gas is not evolved. Because nitric acid is a
strong oxidising agent. So, as soon as hydrogen gas Using 1 = 1 + 1 = – 1 + 1 = 11 = cm–1
is formed in the reaction between a metal and dilute f u v 25 2.5 25
nitric acid, the nitric acid, oxidises this hydrogen to
water. Very dilute nitric acid, however reacts with 100 11
\ P= = 100 ´ = 44D
magnesium and manganese to evolve hydrogen gas. f (in cm) 25
28. (4) Straight-line parallel to the X-axis indicates a uniform
motion. Calculate a by using equation v = u + at. Rs
38. (3) = n2 = (2)2 = 4
29. (2) Particle of different masses falls with same acceleration Rp
i.e. acceleration due to gravity on earth.
To change the state (state of rest or state of motion) of 39. (3) 40. (1)
a body we need external force. 1 2
Force = mass × acceleration 41. (1) x = 1 + 53 + 53
30. (3) Ice contracts on melting due to anomalous expansion 1 2
of water. Thus, the total volume of the melted ice is
x – 1 = 53 + 53
equal to the volume of floating ice inside the water.

= (5 )
31. (2) Gravitational force of attraction. 1 2 3
GMm (x – 1)3 3 + 53
F= 2 .... (i)
Also F = mg
R
From (i) and (ii)
... (ii)
1 2
(
x3 – 3x2 + 3x – 1 = 5 + 52 + 3 5 3. 5 3 5 3 + 5 3
1 2
)
x3 – 3x2 + 3x – 1 = 30 + 3 × 5 (x – 1)
GM
g= é 1 2ù
R2 êëQ x – 1 = 5 3 + 5 3 úû
32. (1) A.C. generator is a device which converts mechanical
into electrical energy. x3 – 3x2 + 3x – 1 = 30 + 15x – 15
Electric motor converts electrical energy into x3 – 3x2 – 12x – 16 = 0
mechanical energy. x3 – 3x2 – 12x + 6 = 16 + 6
x3 – 3x2 – 12x + 6 = 22
33. (3) Vertical height climbed = Height of step × No. of steps 42. (4) a = 9, d = 8, Sn = 636
= 20 cm × 20 cm = 400 cm = 4 cm
Mass of the girl = 40 kg n
\ 636 = [2 ´ 9 + ( n - 1) ´ 8]
Since, work done is equal to gain in potential energy, 2
MT-8 Target NTSE
Þ 4n2 + 5n – 636 = 0 In DBDC, Ðy + ÐBDC + 55° = 180° ...(2)
Þ (n – 12) (4n + 53) = 0 A
æ 53 ö
Þ n = 12 ç\ n ¹ - ÷
è 4ø
30°
æ1 ö
43. (2) Let ç ,6 ÷ divide the line segment joining the points D a
è2 ø
(3, 5) and (–7, 9) in the ratio of K: 1
Using section formula, the co–ordinate of the dividing x
point are given as 50° y 55°
B C
Kx + x Ky + y1 By adding (1) and (2)
x = 2 1 and y = 2
K +1 K +1
Ðx + ÐADB + 30° + Ðy + ÐBDC + 55° = 360°
1 K (-7) + 3 1 -7 K + 3
Þ = Þ = Þ K + 1 = -14 K + 6 Also, Ðx + Ðy = 50°
2 K +1 2 K +1
Then ÐADB + ÐBDC + 135° = 360°
1
Þ 15K = 5 Þ K = . Þ ÐADB + ÐBDC = 225°
3
\ Ratio is 1: 3 i.e., a = ÐADC = 135°
47. (3) Since (x, y) is the mid-point of the line joining (3, 4) and
44. (2) O (p, 7).
a 3+ p 4+7
\ x= , y=
2 2
Þ 2x = 3 + p, 2y = 11
L Since 2x + 2y + 1 = 0
2a
\ 3 + p + 11 + 1 = 0 Þ p = – 15
a
48. (3) Let ‘O’ be the observation point, and let A be the
A B position of aeroplane such that ÐAOC = 60°
AC = 1km = BD
Let B be the position of aeroplane after 10 seconds
AL = OL = 2 (radius of the sphere) such that ÐBOD = 30°, OC = x km. and CD = y km
ÐALM = 2a (exterior angle of the triangle is the same Now, in rt DOCA,
as sum of the interior opposite angles)
LM A B
In DALM, = cos 2 a
AL 1 km
1 km
LM
= cos 2a
OL
\ LM = OL cos 2a = 2 cos 2a 60°
Height of the cone = OM
30°
= OL + LM
= 2 + 2 cos 2a O x C y D
= 2 (1 + cos 2a)
= 2 × 2 cos2a x 1
cot 60° = Þx= ...(1)
= 4 cos2a 1 3
45. (4) We know that sec2 q – tan2 q = 1 and
In a DODB,
x y
sec q = , tan q = x+ y
p q cot 30° = Þ x+ y = 3 ...(2)
1
\ x2q2 – p2y2 = p2q2
From (1) – (2)
46. (2) Now in, DABD. join BD
Ðx + ÐADB + 30° = 180° ...(1)
SOLUTIONS MT-9

52. (3) A circle inscribed in a quadrilateral ABCD,


1 1- 3 2 AB = 6 cm, BC = 7 cm, CD = 4 cm,
-y = - 3 Þ -y = Þy= km
3 3 3
D R C
2
In 10 seconds, distance covered, y = km
3
\ In 3600 seconds, distance covered, S Q
3600 2
= ´ km / hr = 240 3
10 3
\ Speed of the plane = 240 3 km / hr A P B
Let the circle touch the sides AB, BC, CD, DA, at P,
49. (2) 4 x 2 – 3x – 5 = 0 Q, R and S, respectively.
3 5 AP = AS ìLengths of two tangents ü
Þ x2 – x– =0 ï ï
4 4 BP = BQ
DR = DS ídrawn from an external ý
2 ïpoint of circle, are equal ï
æ 3ö 5 3 CR = CQ î þ
Þ çx– ÷÷ – =
ç 8 ø 4 64 Adding all these, we get
è
(AP + BP) + (CR + RD) = (BQ + QC) + (DS + SA)
2
æ 3ö 3 AB + CD = BC + DA Þ 6 + 4 = 7 + AD
Þ 4ç x – ÷÷ – 5 – =0
ç 8 ø 16 Þ AD = 10 – 7 = 3 cm.
è
53. (3) Total number of days in month of April = 30 days
50. (2) Let p(x) = 2x3 + mx2 + 3x – 5 and
q(x) = x3 + x2 – 4x + m. Since, first day of month is Monday.
By remainder theorem, the remainders are p(2) and q(2) Then, number of Monday’s in month of April = 5
But p(2) = q(2) (given) P(selected day is Monday)
2(2)3 + m(2)2 + 3(2) – 5 = (2)3 + (2)2 – 4(2) + m Number of Mondays in April
=
Þ 16 + 4m + 6 – 5= 8 + 4 – 8 + m Total number of days in month of April
Þ 17 + 4m = 4 + m 5 1
= = .
Þ 3m = –13 30 6
13 54. (1) Since, the mean of given numbers is 82
Þ m= –
3 Then,
51. (1) Since, AB is a chord of a circle with centre O. AB is
produced to C, such that BO = BC (27 + x) + (31 + x ) + (89 + x ) + (107 + x) + (156 + x)
82 =
CO is joined and produced to meet the circle at D. 5
Since, BC = OB Þ 82 × 5 = 410 + 5x Þ 410 – 410 = 5x Þ x = 0
ÐOCB = ÐBOC = y° \ Required mean is,
[Angles opposite to equal sides are equal]
130 + x + 126 + x + 68 + x + 50 + x + 1 + x
ÐOBA = ÐBOC + ÐOCB x=
[Ext. angle of a D is equal to the sum of the opposite 5
interior angles] 375 + 5x 375 + 0 375
Þ ÐOBA = y° + y° = 2y° x= = = = 75
5 5 5
OA = OB [Radii of the same circle] 55. (1) First ten odd numbers are 1, 3, 5, 7, 9, 11, 13, 15, 17, 19
ÐOAB = ÐOBA respectively.
[Angles opp. to equal sides of a D]
1 + 3 + 5 + 7 + 9 + 11 + 13 + 15 + 17 + 19
= 2y° So A.M. (x) =
10
ÐAOD = ÐOAC + ÐOCA
= 2y° + y° 100
= = 10
= 3y° 10
[Exterior angle = Sum of opposite interior angles]. 56. (3) a = 6 cm, b = 6 cm, c = 6 cm
Þ x° = 3y° a+b+c 6+ 6+ 6
\ s= = = 9 cm
2 2
MT-10 Target NTSE
\ Area of the equilateral triangle 63. (3) The Napoleonic code of 1804 was a major step in
= s(s - a)(s - b)(s - c) establishing the rule of law. All privileges based on
birth were taken away.
= 9(9 - 6)(9 - 6)(9 - 6) = 9(3)(3)(3) = 9 3 cm 2 64. (1) The Elgin mill was started in Kanpur in the 1860s.
57. (4) y = log(x-3)(x2 – 6x + 9) 65. (1) Captain swing was a mythic name used in threatening
letters during the rural English. About 387 threshing
y = log(x-3)(x – 3)2 machines where broken.
y= 2 [logaam = m] 66. (4)
Now, 67. (2) In the meeting of the Estate General the members of the
log y 5 third estate wanted that each state should have one
2 = 2log 2 5 = 5 vote.
58. (2) For triangle, sides are a = 26 cm, b = 28 cm, c = 30 cm 68. (3) Well-to-do peasants. In 1928 the party members (Stalin)
a + b + c 26 + 28 + 30 84 toured grain producing areas, supervised, enforced
\ s= = = = 42 cm grain collection and raided Kulaks.
2 2 2
69. (3) Rat hunt started in 1902, thousands of rats were cuaght.
\ Area of the triangle = s ( s - a )( s - b )( s - c ) On 30 May e.g. 20,000 rats were caught. For the
(By Heron’s formula) Vietnamese rat hunt provided a lesson in the success
= 42 ( 42 - 26 )( 42 - 28 )( 42 - 30 ) of collective bargaining.
70. (4) John Ford Coppola’s Apocalypse Now (1979) reflected
= 42 (16 )(14 )(12 ) the moral confusion that the war had caused in the US.
= ( 6 ´ 7 )16 ( 7 ´ 2 )( 6 ´ 2 ) 71. (4) The leaders (Jawaharlal Nehru and S. C. Bose)
supported the socialist ideas. They wanted the
= 6 × 4 × 7 × 2 = 336 cm2 movement to be more intense through mass agitation.
Let the height of the parallelogram be x cm. 72. (3) Architect and planner Ebenezer Howard developed the
Then, area of the parallelogram = Base × Height principle of the Garden city, a pleasant space full of
= 28 × x cm2 plants and trees where people would both live and
Since, triangle and parallelogram have the same area work.
\ 28x = 336 73. (4)
336 74. (3) HVJ is a natural gas carrying pipeline popularly known
Þ x= Þ x = 12 cm
28 as HVJ (The Hazira-Vijaypur-Jagdishpur) pipline
Therefore, the height of parallelogram is 12 cm. 75. (4) Terrace farming is a type of farming that was developed
first by the Indian people. This method of farming uses
59. (3) LCM (1, 2, 3, ... ,200) = b × LCM (102, 103, ..., 200) steps that are built into the side of a mountain or hill.
LCM (1, 2, 3, ..., 200) 76. (4) Tropical evergreen forests require heavy rainfall with a
b =
LCM (102, 103, ..., 200) short dry season. The trees reach heights upto 60
Since, LCM (1, 2, 3, ..., 100) = LCM (101, 102, ..., 200) meters or even above.
Then,
77. (2) 78. (3)
LCM (101, 102, ..., 200)
b = 79. (4) Kerala has the highest sex ratio while Haryana has the
LCM (102, 103, ..., 200)
= 101 lowest sex ratio. Area with lowest population density
60. (4) We know that, is Arunachal Pradesh.
tan 60° = 3 ; cosec 45° = 2 80. (2) Gandhiji was very apt in voicing his concern about
sec 45° = 2 ; sin 30° = 1/2 resource conservation.
\ tan2 60° cosec2 45° + sec2 45° sin 30° 81. (1)
82. (4) Our southern neighbours across the sea consist of
2 2 2 1 1 two island countries e.g. Sri Lanka and Maldives.
= ( 3) ( 2) + ( 2) = 3 ´ 2 + 2 ´ = 6 +1 = 7
2 2 83. (4) The northern plain has been formed by the interplay of
61. (1) Olympe de Gouges was a French playwright and three major river systems viz., the Indus, the Ganga
political activist whose feminist writing influenced a
large audience. and the Brahmaputra alongwith their tributaries.
62. (1) Lala Lajpat Rai was popularly known as Punjab Kesari 84. (1) The flora and fauna of the planet are under great threat
or Lion of Punjab. He went to U.S.A. to promote India’s beacuse of insensitivity to our environment. It disturbs
freedom struggle. the ecological balance.
85. (3) 86. (3) 87. (3)
SOLUTIONS MT-11

88. (3) It was a non-violent protest in 1987 by the people in matter where they live. This government has the power
Karnataka. regarding cultural, educational and language related
89. (1) issues.
90. (2) The referendum is a direct vote in which, an entire 96. (2) These logos and certification help consumers get
electorate is asked either to reject or accept a particular assured of quality which purchase goods and services.
proposal. This may be adoption of new constitution, a The producer are allowed to use logos if they follow
law or a specific government policy. certain quality standards.
91. (2) The party came to power in 2006 was socialist party. It 97. (2) 1992, Revamped Public Distribution system was
supported the movement. introduced in 1700 blocks in the country. Provided the
92. (3) Jaipal Singh born (1903) in Jharkhand. A sportsman benefits of PDS to remote and backward areas.
and educationist, founder of Adivasi Maha Sabha. 98. (2) The State level court is higher than the district level
93. (1) There are some countrywide parties called national consumer court and deals in cases involving claims
parties. These have units in various states, all these between 20 lakhs and 1 crore.
units follow the same policies, programmes and 99. (2) According to the report, the countries with per capita
strategy that is decided at the national level. income of ` 37000 or less are called Low income
94. (1) The Directive principles of state policy are the directions countries.
given to the state these cannot be challenged in the 100. (1) The judicial machinery under COPRA is describes as
court. 3 tier quasi-judicial machinery. 3 tiers are at District
95. (3) The community government in Belgium is elected by level, State level and at National level.
people belonging to one language community, no
MT-12 Target NTSE

Mental Ability Test MOCK TEST 2


ANS W ER KEY
1 (4) 11 (3) 21 (4) 31 (4) 41 (3) 51 (2) 61 (3) 71 (1) 81 (1) 91 (4)
2 (2) 12 (3) 22 (2) 32 (3) 42 (1) 52 (1) 62 (4) 72 (1) 82 (4) 92 (1)
3 (2) 13 (1) 23 (1) 33 (2) 43 (2) 53 (1) 63 (2) 73 (4) 83 (2) 93 (3)
4 (3) 14 (1) 24 (4) 34 (4) 44 (1) 54 (4) 64 (3) 74 (2) 84 (4) 94 (2)
5 (2) 15 (2) 25 (3) 35 (1) 45 (1) 55 (4) 65 (3) 75 (1) 85 (4) 95 (4)
6 (1) 16 (4) 26 (2) 36 (1) 46 (1) 56 (1) 66 (4) 76 (1) 86 (4) 96 (2)
7 (2) 17 (1) 27 (4) 37 (4) 47 (4) 57 (3) 67 (4) 77 (1) 87 (2) 97 (3)
8 (3) 18 (2) 28 (2) 38 (1) 48 (2) 58 (4) 68 (4) 78 (1) 88 (4) 98 (1)
9 (2) 19 (4) 29 (1) 39 (2) 49 (4) 59 (2) 69 (4) 79 (4) 89 (4) 99 (4)
10 (2) 20 (2) 30 (3) 40 (4) 50 (4) 60 (1) 70 (3) 80 (3) 90 (1) 100 (2)

Hints and Explanations


1. (4) Except SAARC, all others are agencies of UNO. 13. (1) Comparing relation in each of the statement, we get
2. (2) 5878 º 5 + 8 + 7 + 8 = 28 Asha is coded as ‘ja’.
14. (1) Similarly As
6482 º 6 + 4 + 8 + 2 = 20
–1 –1
5788 º 5 + 7 + 8 + 8 = 28 T ¾¾® S C ¾¾®
–1
B
–1
9748 º 9 + 7 + 4 + 8 = 28 E ¾¾® D R ¾¾® Q
–1 –1
Thus, 6482 is odd. R ¾¾® Q E ¾¾® D
–1 –1
M ¾¾® L D ¾¾® C
5 6 8 9 11 12 +1 +1
3. (2) I ¾¾® J I ¾¾® J
+1 +2 +1 +2 +1 +1 +1
B
N ¾¾® O B ¾¾®
4. (3) The series progress with a difference of – 5. +1 +1
A ¾¾® B L ¾¾® M
0 2 6 12 20 30 +1 +1 F
L ¾¾® M E ¾¾®
5. (2)
+2 +4 +6 +8 +10
6. (1) The series is aabb/aabb/aabb 15. (2) B 20km A
The missing letters are thus aabab W
15 km
S N
–2 –3 –4 My House
E
7. (2) A Z C X F U J Q 25km

+2 +3 +4
8. (3) First number is increasing by 5, 7, 9, 11, 13....
Second letter is decreasing by 1 position. Third number
C 20km D
is increasing by 2.
From the above diagram required distance
9. (2) The given series is 23 – 1, 33 – 1, 43 – 1, 53 – 1 and so on
= 25 – 15 = 10 km.
\ The wrong number in the series is 28 which should 16. (4) 1st of month was Tuesday, hence the date on first
have been 26. Saturday was 5th.
10. (2) (1) adef ; (2) ador; (3) cdeo; (4) ackl Hence the other Saturdays of the month are 12, 19, 26.
11. (3) Second and fourth letters of the word PROSE are moved Rama met her brother on 26th.
two steps backwards in alphabatical order. Similarly, 17. (1) 45°+180° = 225° clockwise direction
LIGHT can be coded as LGGFT. 270° anticlockwise direction.
12. (3) Here, 2 ® A, 3 ® L, 5 ® G,, 4 ® U and 9 ® T.. 225 – 270° = – 45°
Hence, 23549 will be having the code ALGUT.
SOLUTIONS MT-13

1st ent Students Math Athletics Studies GK Arts


vem
Mo Asha × ×
Charu × ×
180° Deepa ×
45° Beena × ×
Initial
Position 45° Ela × ×

shows good
× shows not good
Mo Fina 2nd 27. (4) From the table above, it is clear that Deepa is good in
vem l Movement
en Studies, General Knowledge and Arts.
t
28. (2) Clearly, Beena is good in Studies, General Knowledge
i.e., 45° anticlockwise from initial position. and Mathematics.
29. (1) Obviously, Asha is good in Studies, Mathematics and
Hence, the required direction is south-west.
Athletics.
18. (2) The sequence is as below : 30. (3) Charu is good in Athletics, General Knowledge and
Mathematics.
Either side 31. (4) From the last row of the above table, it is clear that Ela
is good in Studies, General Knowledge and Arts but
Blue Green Yellow Red Blue not in Athletics.
or or 32. (3) Both the conclusions are valid.
Red Green
X Z X
Married

ed
For (Qs. 19-23) Men

t
uc a
The given information can be summarized as follows:

Ed
Educated
either
D (Father of A)

33. (2) Conclusion A does not follow.

B A + E (Wife of A)
(Brother of Boys Father
E’s husband) (Brother of B and
Father of C & F)
Girls
F C (Sister of F) The conclusion B is valid acording to the Venn
(Grandson of D
and Brother of C)
Diagram.

34. (4) either


19. (4) 20. (2) 21. (4) Girls
22. (2) 23. (1)
Girls

24. (4) Some men as well as some women may be musicians.


Boys
25. (3) All parrots are birds while mice are of a separate
rs
category. ve
Ri
26. (2) There are 10 cubes.
For (Qs. 27 to 31) : No conclusion of mediate inference is possible since
The given information can be summerised in a table the middle term is not distributed.
that follows: 35. (1) The given figure is completed by (1).
MT-14 Target NTSE
36. (1) Except (C) which is a hexagon, all others are Pentagons. 49. (4) We have to find the correct mirror image for the word
37. (4) Except (C) which has two lines and one arrow, all others ‘WATER’ for which we need to find the mirror image
have only one line and one arrow. for each letter separately and then arrange it, like the
38. (1) Except (A), which has two arrows; all others has only mirror image for the letters W is W, A is A, T is T, E is
one arrow. and R is .
39. (2) Except figure (B), all the arrangements are in horizontal Since, the word ends with R, i.e., where the mirror is
arrangement, while fig (B) has vertical arrangement. placed, therefore the mirror image will start from the
40. (4) Except (A), all the other arrangements have identical mirror images of R, i.e.; .Thus the mirror image for
figures. water is
41. (3) D = S × T
20 WAT E R
In first case, 20 = S × T Þ T = ... (1)
S Thus option (4) is the correct answer.
50. (4) In case of water image, the water reflection will usually
30
In second case, 30 = (S + 20) × T Þ T = S + 20 ... (2) be formed under the object / word.
In this case, the water image of the word will be an
From equation (1) & (2) outcome of the water images of each of the letters like,
20 30 the water images of S is , T is , O is , R is and
= Þ 20S + 400 = 30S Þ 10S = 400 E is . Thus the water image of the word ‘STORE’ is
S S + 20
‘ .’
Þ S = 40 mph STORE
20 20 1
\T= Þ = hr or 30 minutes
S 40 2 Solutions (51-53):
42. (1) By aeroplane service, one can reach Delhi from Chennai (189 cm) Z > M > Y > (180 cm) C > A > B
quickly. 51. (2)
43. (2) The number ‘3’ space represents Indian teachers who 52. (1) Then, the difference between the heights of C and B
are also advocates as this number is common to given is:
condition. 180 – 176 = 4 cm.
44. (1) Clearly, the required number would be such that it leaves 53. (1)
a remainder of 1 when divided by 2, 3 or 4 and no 54. (4) By combining both the statements together we cannot
remainder when divided by 5. Such a number is 25. find the distance between point P and Q.
55. (4) From both the statements we cannot find that how
Akhilesh Aman many persons are facing outside to the centre.
Akhilesh Aman (III) Sheebu 56. (1) From I, we conclude that Prem is the mother of Rita
45. (1) (II)
Sheebu Tejinder Tejinder and Nita, while Queen is the daughter-in-law of Prem
and sister-in-law of Nita. Thus, Queen is Rita’s wife
Akhilesh and hence, Rita is Nita’s brother.
Aman From II, we conclude that Rita and Nita are the children
(I + II + III) Sheebu of Soni. Also, Roma is the daughter-in-law of Soni and
Tejinder sister-in-law of Rita. So, Roma is Nita’s wife and thus,
46. (1) When the sheet shown in fig. (X) is folded to from a Nita is Rita’s brother. Hence, Rita is either brother or
cube, then the face bearing the dot lies opposite to the sister of Nita.
57. (3) Let the tens and units digit be x and y respectively.
shaded face, the face bearing a circle (With ‘+’ sign
Then,
inside it) lies opposite to a blank face and the remain-
ing two blank faces lie opposite to each other. Clearly, x 1
the cubes shown in figures (B) and (D) cannot be I. x + y = 8 and =
y 3
formed since they have the shaded face adjacent to
the face bearing a dot and the cube shown in fig. (C) \ I gives, 4 y = 24 Û y = 6
So, x + 6 = 8 Û x = 2
cannot be formed since it shown all the three blank
face adjacent to each other. Hence, only the cube shown x 3
II. xy =12 and =
in fig.(A) can be formed. y 1
47. (4) From figures (i) and (iv) we conclude that 6, 5, 2 and 3
\ II gives, x2 = 36 Û x = 6
lie adjacent to 4. It follows that lies opposite 4.
So, 3y = 6 Û y = 2
48. (2)
Therefore, Either I or II alone sufficient to answer.
SOLUTIONS MT-15

58. (4) The ratio, in which X and Y are mixed, is not given. 73. (4)
So, both I and II together cannot give the answer. 74. (2)
Correct answer is (d). 75. (1)
Solutions (59-61): 76. (1) The number at the bottom is the difference of squares
Relation ® xz of two numbers given at top
and ® mo In first, 112 = 92 = 121 – 81 = 40
India ® mn In second figure, 152 – 72 = 225 – 49 = 176
So, In third figure, 252 – 212 = 625 – 441 = 184
Australia ® nk
77. (1) In first column, Z = 26, R = 18
Inspection ® da In second column, K = 11, C = 3
Power ® rs We find the gap of 8 is there both columns.
in ® fa Adopting the same rule, we find that V = 22, N = 14
increase ® ij 78. (1) In the given code 1 = A, 2 = B. 3 = C,........ 24 = X, 25
59. (2) = Y, 26 = X
60. (1) So, in FLOWER, F is coded as 6, L as 12, O as 15, W as
61. (3) 23, E as 5 and R as 18.
Solutions (62-65): 79. (4) A lizard is an animal which crawls and hence is called
62. (4) ATQ, Maximum Profit = ` 4500 ‘flying’
63. (2) ATQ, Solutions (80-84):
Total loss = 1500 + 1000 = ` 2500
64. (3) ATQ, Names University Subject
Max. Profit : Min. loss Dr. Joshi Delhi History
4500 : 1000 Dr. Davar Osmania Geology
9 :2 Dr. Natrajan Gujarat Botany
Hence, Required ratio = 9: 2
Dr. Choudhary Mumbai Zoology
2500 Dr. Zia Gujarat Physics
65. (3) ATQ, x = ´ 100 = 20
12500
Þ x = 20 80. (3) Dr. Davar teaches geology.
Hence, Required value = 20 81. (1) Dr. Zia is form Gujarat University.
66. (4) 82. (4) Dr. Natarajan teaches botany.
67. (4) There is no information in the facts to support 83. (2) Dr. Davar is from Osmania University
statements I or II. Statement III is clearly wrong 84. (4) Dr. Natranjan-Gujarat University is the correct
because, according to Fact 1, no frames cost less than combination.
$35. 85. (4) The father of this brother means “his father” is the
68. (4) If Mary always tells the truth, then both Ann and Mary only son of my mother means “my brother”. If means
have cats (statements I and II), and Ann is lying lady is the father’s sister of the man’s father.
(statement III). So all the statements are facts. 86. (4) Bopri is the farthest West
69. (4) In each row, there are 3 types of shadings of circles –
a circle is unshaded, another circle has its right half Bopri Kokran Akram Tokhada Paranda
shaded with vertical lines and yet another circle has
its upper half shaded with horizontal lines. There are 87. (2) Bidi smokers is a subset of smokers and cancer patient
three specified positions of the two triangles each of may be a smokers, bidi smoker and nonsmoker.
which is used only once in a row. Also, two of the Hence third shares a common relationship with first
figures in each row have one triangle shaded. and second object as well.
70. (3) In each row, the third figure is a collection of the
88. (4)
common elements (line segments) of the first and the
89. (4)
second figures.
71. (1) Clearly, job is offered to an engineer. This means that 90. (1)
he is needed. So, I is implicit. The word ‘If’ in the 91. (4)
statement makes II not implicit. 92. (1) From the unfolded figure of dice, we find that number
72. (1) The statement expresses an expectation of complaints opposite to 2 is 4, for 5 it is 3 and for 1 it is 6. From this
from the people against poor services. So, I is implicit. result we can definitely say that figure (B), (C) and (D)
But the effect of complaints cannot be deduced. So, II cannot be the answer figure numbers lying on the
is not implicit. opposite pair of surfaces are present on the adjacent
surfaces. Hence fig. (A) is our answer.
MT-16 Target NTSE
95. (4) Note the direction of arrow which changes alternately.
93. (3) A B C D The dots are also changing alternately. Hence, we are
looking for a figure in which the arrow points down
and the dots and positioned as in figure (2)
J E 96. (2) Clearly, the lower half of the square sheet has been
L folded over the upper half. Hence, the bent line in the
K lower half will be inverted over the other half so that a
I H G F ‘V’ shaped figure is formed.
97. (3) In figure X, the upper triangular half of the paper has
The figure may be labelled as shown : been folded over the lower half. In figure Y, the paper
The squares composed to two components each, are is refolded to a quarter triangle. In figure Z, a square
ABKJ, BCLK, CDEL, LEFG, KLGH, and JKHI. Thus, has been punched in the folded paper. Clearly, the
there are 6 such squares. Only one square, KCEG is square will appear in each for the triangular quarters
composed of four components. Two squares namely, of the paper. Thus, when the paper is unfolded, four
ACGI and BDFH are composed of eight components squares will appear symmetrically over it and it will
each. Thus, there are 2 such squares. So, There are resemble figure (C).
6 + 1 + 2 = 9 squares is the given figure. 98. (1) Raman ® 15 May < Exam < 18 May
94. (2) In figure (X), one of the dot lies in the region common Deep ® 16 May < Exam < 21 May
to the circle and the triangle only and the other dot
\ Exam is on 17.
lies in the region common to the circle and the square
99. (4) ‘BOSE’; There is no ‘S’ in the given word.
only. In figures (A), (C) and (D) the region common to
100. (2) White apples = 37 + 36 = 73
the circle and the triangle lies within the square. Only
figure (B) contains a region common to the circle and
the triangle only and also a region common to the
circle and the square.
SOLUTIONS MT-17

SAT (Scholastic Aptitude Test) MOCK TEST 2


ANS WER KEY
1 (1) 11 (2) 21 (4) 31 (1) 41. (3) 51. (2) 61. (4) 71. (4) 81. (2) 91. (2)
2 (2) 12 (4) 22 (3) 32 (3) 42. (3) 52. (1) 62. (1) 72. (3) 82. (2) 92. (2)
3 (3) 13 (3) 23 (1) 33 (3) 43. (1) 53. (1) 63. (3) 73. (1) 83. (2) 93. (3)
4 (4) 14 (4) 24 (1) 34 (3) 44. (1) 54. (4) 64. (3) 74. (1) 84. (4) 94. (3)
5 (1) 15 (2) 25 (2) 35 (2) 45. (2) 55. (3) 65. (2) 75. (2) 85. (1) 95. (3)
6 (3) 16 (2) 26 (2) 36 (2) 46. (2) 56. (3) 66. (1) 76. (4) 86. (2) 96. (2)
7 (4) 17 (2) 27 (4) 37 (3) 47. (4) 57. (2) 67. (2) 77. (1) 87. (2) 97. (1)
8 (2) 18 (3) 28 (3) 38 (1) 48. (1) 58. (1) 68. (2) 78. (4) 88. (3) 98. (3)
9 (3) 19 (2) 29 (4) 39 (3) 49. (2) 59. (2) 69. (2) 79. (3) 89. (4) 99. (1)
10 (3) 20 (1) 30 (1) 40 (1) 50. (4) 60. (4) 70. (2) 80. (2) 90. (2) 100. (3)

Hints and Explanations


1. (1) Epithelial cells are tightly packed and form a continuous W2 - W1
sheet.They have only a small amount of cementing × 100
W1
material between them and almost no intercellular
spaces. The permeability of the cells of various epithelia 10. (1) 11. (4)
plays an important role in regulating the exchange of 12. (4) Energy flow is the key function in an ecosystem and it
materials. is unidirectional. Ten percent law states that during
2. (2) Sleeping sickness is an infectious disease caused by transfer of food energy, only 10% is stored at higher
protozoan Trypanosoma. Jaundice is caused by trophic level. The remaining 90% is lost during feeding,
hepatitis virus affects liver which is the most important digestion and respiration.
organ in the body and its inflammation affects digestion 13. (3)
adversely. Vaccination is the most common method of 14. (4) The pathway taken by a stimulus to travel from receptor
preventing infection of microorganisms, especially organ to effector organ is known as reflex arc. In reflex
bacteria and viruses, firstly developed by Dr. Edward action, the same stimulus always produces the same
Jenner in 1796 AD. response.
3. (3) Vaccines are preparations containing heat killed or 15. (2) Elements can not be further broken down into simpler
chemically weekend pathogens or their surface coatings constituents.
which function as antigens. Prolonged hyperglycemia 16. (2) Student (B) made the correct recording of observations.
(increased blood sugar) leads to a complex disorder The temperature of water is expected to decrease
called diabetes mellitus which is associated with loss gradually. Actually, molecules of water undergoing
of glucose through urine and formation of harmful evaporation take up energy from the rest of water
compounds known as ketone bodies. molecules in the container. As a result, the temperature
4. (4) is expected to decrease.
5. (1) Angiosperm is made from two Greek words: angio 17. (2)
means covered and sperma means seed. The seeds 18. (3) Periods have elements with consecutive numbers.
develop inside an organ which is modified to become a
fruit. These are also called flowering plants. 19. (2) This student (B) has made the correct observation.
Acetic acid is colourless with vinegar smell. It gives
6. (3) brisk effervescence of carbon dioxide on reacting with
7. (4) Birds arose from reptiles had been found out with the sodium hydrogen carbonate.
discovery of fossil Archaeopteryx (fossil bird). Prior 20. (1) Precipitate of copper sulphide is dark grey or black in
to Archaeopteryx, feathers had already evolved in colour. Silver cannot displace copper from CuSO4 as
dromeasaurs for protecting them from cold. In silver is less reactive than copper.
Archaeopteryx feathers attached to forelimbs were used 21. (4) Many nutrients comes from soil but CO2 comes from
for gliding and possibly flying. air.
8. (2) 22. (3) If hydrogen leaks out of the container or the volume of
9. (3) W1 = initial weight of raisins, 40 g the container increases, pressure of hydrogen gas
W2 = final weight of wet raisins, 45 g decreases in the container thus distance between
molecules of hydrogen gas increases.
MT-18 Target NTSE
23. (1) Sulphur 34. (1)
24. (1) In physical change no new substances are formed and
it is a reversible change. Burning charcoal is a chemical 35. (2) To hear a distinct echo the time interval between the
change as in this change new products are formed as a original sound and the reflected one must be at least
result of combustion and this change can not be 0.15
reversed.
25. (2) Since hard water contains soluble chloride and sulphate 1
As, V = f l = 2x
salts of calcium and magnesium, which forms insoluble T
curdy precipitate.
V ´T 300 ´ 0.1
\ x= = = 15 m
2C17 H35 COONa + CaCl2 ¾¾
® (C17 H35COO)2 Ca + 2NaCl 2 2
soluble sodium Calcium insoluble calcium
stearate (soap) chloride stearate (scum) 36. (4)
(in hard
water) 37. (3) On passing through a prism, white light breaks into
seven colours VIBGYOR. If the second identical prism
2C17 H35 COONa + MgSO4 ¾¾
® (C17 H35 COO)2 Mg + Na 2SO4 is placed in an inverted position w.r.t. the first prism the
soluble
sodium stearate (soap)
Magnesium
sulphate
Insoluble magnesium
stearate (scum)
colours of the spectrum recombine and emerges as a
(in hard
water)
parallel beam of white light.
w
26. (2) Cu + 2Ag+ ® Cu2+ + 2Ag 38. (1) Electric potential V =
q
27. (4) Electropositive character increases on moving down a
group. Electronegativity increases in a period but q
irregularly with exceptions. These exceptions are due Electric current I =
t
greater stability of exactly fully filled and half filled
configurations. w
Electric power P =
t
s s
28. (3) Av speed = = = 18km / h Resistance is the obstruction offers to the flows of
s/3 s/3 s/3 s / 18
+ + charge
10 20 60 39. (3) Power = V . I = I2R
29. (1)
Power 2 1 1
30. (1) A floating object be in a stable equilibrium, if its centre i2 = = = = A
R 8 4 2
of buoyancy is vertically above its centre of gravity.
1
31. (1) The gravitational force between two objects is Potential over 8W = Ri 2 = 8 ´ = 4V
proportional to the product of their masses and 2
inversely proportional to the square of the distance This is the potential over parallel branch. So,

mm 4
between them. Hence, F µ 1 2 i1 = =1A
4
d2
Power of 3W = i12R = 1 × 1 × 3 = 3W
40. (3)
32. (3) Net force on body = 42 + 32 = 5N
41. (3) 5x - 1 = 2 - x - 1
F 5 1 Squaring on both sides, we have
\ a= = = m/s 2
m 10 2
5x – 1 = 4 + x – 1 – 4 x -1
1 2 1 x – 1 = – x -1
Kinetic energy = mv = m (at )2 = 125 J
2 2 Again squaring on both sides, we have
33. (3) Work done by F1W1 = F1S = 5 × 10 = 50 J x2 – 2x + 1 = x – 1
1 x2 – 3x + 2 = 0
Work done by F2W2 = F2S cos 60° = 4 × 10 × = 20J
2 (x – 2) (x – 1) = 0
Work done by friction, W3 = – mmg s x = 2 or x = 1
= – 0.2 × 2 × 10 = – 40 J But x = 2 is not satisfying the given equation.
\ Change in K.E. = Net work done So x = 1 is the only solution.
42. (3) We have x1(y2 – y3) + x2 (y3 – y1) + x3 (y1 – y2) = 0
= W1 + W2 – W3 = 30 J Dividing each term by x1x2x3 on both sides, we get
SOLUTIONS MT-19

\ ÐPOR = 2 ÐOQR
y 2 - y 3 y 3 - y1 y1 - y 2
+ + = 0.
x 2x3 x1x 3 x1 x 2 72°
= ÐOQR
2
43. (1) 26n – 62n = (26)n – (62)n = 64n – 36n
Clearly it is divisible by 64 + 36 = 100 Þ ÐOQR = 36°
44. (1) tan q + cot q = 2 again, in DQPT,
ÐPQT + ÐQPT + ÐQTP = 180°
1 (angle sum property)
Þ tan q + =2
tanq ÐQTP = 180° – 90° – 36°
Þ tan2 q – 2tan q + 1 = 0 ÐQTP = 54°
Þ (tan q –1)2 = 0 or ÐPTR = 54°
Þ tan q = 1 Þ q = 45° Þ q = 45°
47. (4) (81)m × (121)m – 1
Þ cot q = 1
\ tan25q = 1, cot25q =1 = (81 × 12)m – 1
1
Also, sec 45° = 2 , cos 45° = This expression is always divisible by (81 × 121 – 1) i.e
2
1 9800, which is possible only if the last two digits of the
sec2q + cos2q = 2 + = 5/2 and given expression are 00.
2
tan25q + cot25q = 1 + 1 = 2 48. (1) Let a = 3, b = – 3 be the given zeros and g be the
Hence tan25q + cot25q + sec2q + cos2q third zero.Then,
5 9
= 2+ = æ – 4ö
2 2 a +b+ g = –ç ÷
è 1 ø
45. (2) BD = DC given as D is the mid point of BC.
é Coeff. of x 2 ù
A
ê Using a + b + g = – ú
êë Coeff. of x3 úû
q
f
Þ 3 – 3 +g =4
Þ g= 4
C
Hence, third zero of polynomial is 4.
B D
49. (2) a + b = – c or a2 + b2 + 2ab = c2
AC
In DADC, cot q = .....(1) \ a2 + b2 – c2 = – 2ab, squaring again both sides
CD
a4 + b4 + c4 + 2a2b2 – 2a2c2 – 2b2c2 = 4a2b2
AC
In DABC, cot f = .....(2) Þ a4 + b4 + c4 = 2(a2b2 + b2c2 + c2a2)
BC
a 4 + b4 + c 4
Þ =2
AC AC (a 2b2 + b 2c 2 + c2a 2 )
cot q CD
\ = = CD [Q 2CD = BC]
cot f AC AC
50. (4)
BC 2CD

cot q 2 tan q 1 42 cm
Þ = Þ =
cot f 1 tan f 2
46. (2) Since PT is a tangent to the circle.
\ OP ^ PT or ÐQPT = 90° ....(i)
In DOQR, ÐPOR = ÐOQR + ÐORQ
1 22
(exterior angle property) Area of quadrant = ´ × 42 × 42
4 7
also, ÐOQR = ÐORQ (angles opp. to equal sides are
equal) = 1386 cm2
MT-20 Target NTSE
Area of rectangle = 42 × (42 + 21) Þ x = 1 or x = – 1/2
= 2646 cm2 So, the required value of x is 1. [ x = –1/2 is neglected as
\ Unshaded area length cannot be negative]
= 2646 cm2 – 1386 cm2 = 1260 cm2 55. (3) In figure, AB represents the stick and BC is its shadow.
51. (2) (1 + 1 + .... ‘n’ terms) Therefore AB = 12 m and BC = 8 m.
æ1 2 ö P
- ç + + .....'n ' terms÷
èn n ø
A
æ1 + 2 + 3 + .... + ' n ' terms ö
= n-çç ÷
÷
è n ø
n (n + 1)
= n- 12 m
2n
n +1 n -1
= n- =
2 2
52. (1) Total number of cards = n = 100
All the numbers from 50 to 59 have the digit 5. They are C 8m B R 40 m Q
10 in number. Besides these numbers the numbers 5,
15, 25, 35, 45, 65, 75, 85, 95 have the digit 5. These are 9 Again PQ is tower and QR is its shadow. Therefore
in number. QR = 40 m
Number of favourable cases = m = Number of numbers Now, D ABC ~ D PQR
which have the digit 5 = 10 + 9 = 19
PQ AB PQ 12
m 19 \ = Þ = Þ PQ = 60 m
\ Probability = P = = QR BC 40 8
n 100
53. (1) x2 + y2 + z2 = r2 (cos2 a) (cos2 b + sin2 b) + r2 sin2 a 56. (3) Here a1 = 2, a 2 = 8 = 2 2
= r2 cos2 a + r2 sin2 a
\ d = 2 2 - 2 = 2, a = 2
= r2 (cos2 a + sin2 a) = r2
54. (4) In D ABC, we have DE || BC né
Sn = 2a + (n - 1) d ùû
AD AE 2ë
\ =
DB EC né n (n + 1)
(By basic proportionality theorem) = ê
ë2´ 2 + (n - 1) 2 ùúû =
2 2
A
57. (2) Q x + y = 1
& x3 + y3 + 3xy = (x + y)3 – 3xy(x + y) + 3xy = 1
4x–3 8x–7
58. (1) OA = OD = 7 cm.
The required area = Area of the circle with diameter
D E
OD + Area of semicircle with radius OA – Area of
3x–1 5x–3 D ACB

2
æ7ö 1 2 1
B C = p ç ÷ + p(7) - AB.OC
è 2ø 2 2
4 x - 3 8x - 7
Þ =
3x - 1 5 x - 3 22 7 7 1 22 1
= ´ ´ + ´ ´ 7 ´ 7 - ´ 14 ´ 7
Þ 20x2 – 15x – 12x + 9 = 24x2 – 21x – 8x + 7 7 2 2 2 7 2
Þ 20x2 – 27x + 9 = 24x2 – 29x + 7
11 ´ 7
Þ 4x2 – 2x – 2 = 0 = + 11 ´ 7 - 7 ´ 7 = 66.5 cm2
2
Þ 2x2 – x – 1 = 0 Þ (2x + 1) (x – 1) = 0
SOLUTIONS MT-21

59. (2) Since, angle subtended at the centre of a circle is twice 71. (4) Industrialists usually employed jobber to get new
the angle subtended by it on the remaining part of the recruits very often jobber was an old and trusted
circle. Then, worker.
72. (3) 73. (1)
2ÐPSQ = ÐPOQ = 70°
74. (1) Mango showers are the pre-monsoon showers in the
\ ÐPSQ = 35° Indian states of Karnataka, Kerala, Konkan and Goa
In D SMQ, that help in the ripening of mangoes. It is also known
as April rains or summer showers.
ÐMSQ + ÐSMQ + ÐMQS = 180°
75. (2) Inter-Tropical convergence zone (ITCZ) is a broad
(Sum of the angles of a triangle)
through of low pressure in equalorial latitudes.
\ 35° + 90° + ÐMQS = 180°
76. (4)
(Q PS^OQ Þ ÐSMQ = 90°)
77. (1) E1-Nino derived from Spanish means ‘the child’, i.e.
ÐMQS = 180° – 125° = 55°
baby christ. A warm ocean current that flows over Peru
60. (4) S = {1, 2, 3, 4, ............,19, 20} is an interval of two to five years.
Let E = event of getting a multiple of 3 or 5 78. (4) The Visakhapatnam port was originally conceived as
= {3, 6, 9, 12, 15, 18, 5, 10, 20}. an outlet for iron ore exports.
79. (3) The Kaveri rises in the Brahmagiri range of the
n(E) 9
\ P(E) = = Western Ghats and it reaches the Bay of Bengal in
n(S) 20 south Cuddalore. Its basins drains parts of Karnataka,
Kerala and Tamil Nadu.
61. (4) During British colonial rule cricket came to represent
80. (2) The state to pass first revolution for Joint Forest
all that the Victorian English valued fair play, discipline Management was Odisha.
and gentlemanliness 81. (2) 82. (2) 83. (2) 84. (4)
62. (1) Auschwitz concentration camp was a network of 85. (1)
concentration and extermination camps built and
operated by the third reach in polish areas annexed by 86. (2) The CPI came into existence in 1964 as a result of the
Nazis.
division of communist party of India. It is against
63. (3) It used a simple style of storytelling to speak about the capitalism and believes in socialism, democracy and
plight of widows.
secularism.
64. (3) With the writing of Prem Chand the Hindi novel 87. (2) Martial law in Poland refers to the period of time when
achieved excellence. He drew on the traditional art of the authoritarian government of the people’s Republic
Kissa goi (Story telling). of Poland restricted normal life.
65. (2) Gandhiji organised his first satyagraha movement in 88. (3) By the President after a resolution of Parliament passed
Champaran in 1916 to inspire peasants to struggle by a majority of its members and two third majority of
the members present and voting in each house.
against the oppressive plantation system.
89. (4) 90. (2) 91. (2) 92. (2)
66. (1)
93. (3) 94. (3) 95. (3) 96. (2)
67. (2) During the Napoleonic Wars, prices of foodgrains were 97. (1) Most loans from informal lenders carry a very heavy
high aid of farmers expanded production vigorously. interest rate and do little to increase the income of the
Fear shortage of labour they began buying new borrowers. Hence, it is necessary that banks and
threshing machines that came in the market. The cooperatives increase their lending particularly in rural
farmers thought machine would reduce their areas
dependence on labour.
98. (3) Secondary sector covers activities in which natural
68. (2) The conference regarding Wavell Plan discussion was products are changed into other forms through ways
held at Shimla. of manufacturing that we associate with industrial
activity.
69. (2) The post-war international economic system is also
often described as the Bretton Woods system. The 99. (1) 15% cash is kept as provision in the bank to pay the
IMF and the World Bank are referred to as institutions. depositors who might come to withdraw money from
the bank on any given day.
70. (2) Harish Chandra Sakharam Bhatwadekar, shot a scene
of wrestling match in Bombay’s Hanging Gardens and 100. (3) Public sector is government owned sector. Whereas,
it became India’s first movie in 1896. private sector is owned by private individuals.
MT-22 Target NTSE

MAT (Mental Ability Test) MOCK TEST 3


ANS WER KEY
1 (2) 11 (3) 21 (4) 31 (4) 41 (2) 51 (2) 61 (2) 71 (1) 81 (3) 91 (2)
2 (4) 12 (2) 22 (2) 32 (3) 42 (4) 52 (4) 62 (2) 72 (4) 82 (3) 92 (3)
3 (1) 13 (3) 23 (1) 33 (3) 43 (1) 53 (3) 63 (1) 73 (2) 83 (1) 93 (3)
4 (2) 14 (4) 24 (4) 34 (1) 44 (3) 54 (4) 64 (3) 74 (4) 84 (1) 94 (2)
5 (4) 15 (4) 25 (3) 35 (2) 45 (4) 55 (4) 65 (2) 75 (2) 85 (3) 95 (1)
6 (2) 16 (3) 26 (4) 36 (3) 46 (1) 56 (3) 66 (2) 76 (2) 86 (4) 96 (4)
7 (2) 17 (2) 27 (1) 37 (4) 47 (1) 57 (3) 67 (2) 77 (1) 87 (4) 97 (2)
8 (3) 18 (3) 28 (2) 38 (2) 48 (1) 58 (4) 68 (2) 78 (2) 88 (4) 98 (3)
9 (1) 19 (3) 29 (2) 39 (4) 49 (4) 59 (1) 69 (1) 79 (4) 89 (1) 99 (2)
10 (1) 20 (3) 30 (1) 40 (1) 50 (2) 60 (3) 70 (2) 80 (4) 90 (2) 100 (4)

Hints and Explanations


2. (4) All baseball caps have brims, since baseball caps are 19. (3) The first, third, fifth and seventh letters of the word are
hats (Fact 3) and all hats have brims (Fact 1). This rules each moved one step backward; the second, fourth,
out statement III, but it doesn’t follow that all caps, a and sixth letters are moved one, two and three steps
category that may include caps that are not baseball forward respectively to obtain the corresponding letters
caps, have brims (statement I). Statement II cannot be of the code.
confirmed, either, since it is possible, given the 20. (3) There are two alphabetical series here. The first series
information, that all baseball caps are black. is with the first letters only: STUVW. The second series
involves the remaining letters: CD, EF, GH, IJ, KL.
30 m 21. (4) 8 × 1 – 2 = 6
4. (2) (Starting)
6× 2–3=9
9 × 3 – 4 = 23
40 m 23 × 4 – 5 = 87
22. (2) Between any two consecutive groups, there is a gap of
3 letters. So, after PQ, leaving 3 letters gap, we will get
(Final)
30 m UV.
23. (1) In this series, the letters progress by 1; the numbers
6. (2) (11)2 + (14)2 – (15)2 = 92 decrease by 3.
(15)2 + (17)2 – (21)2 = 73 and 24. (4) The third figure in each row comprises of parts which
(19)2 + (26)2– (31)2 = 76 are not common to the first two figures.
7. (2) Number of Indian students who are talented = 7 25. (3) The correct order is:
8. (3) Number of talented students who are teachers = 11 Key Lock Door Room Switch on
9. (1) Number of students who are not Indian = (12 + 1) = 13
1 3 2 4 5
10. (1) Number of persons who can speak English and Hindi
27. (1) The upper element is converted to an element similar to
both only is 5.
11. (3) Number of persons can speak Marathi and Telugu both the lower elements and each one of the lower elements
= 6 + 7 = 13. is converted to an element similar to the upper element.
12. (2) Number of persons were can speak English is 12. 29. (2) Statement II is the only true statement. Since all dogs
16. (3) The statement concentrates on the increasing like to run, then the ones who like to swim also to run.
viewership of TV. and does not stress either on There is no support for statement I or statement III.
increasing the readership of newspapers or making 30. (1) It is assumed that assistant will follow the instructions
studies regarding the same. So, none follows. but it is not assumed that the assistants knows the address
17. (2) The hands of a clock point in opposite directions (in because his job is to drop the letter in the letter box.
the same straight line) 11 times in every 12 hours. 31. (4) According to the statement, 80% of the total runs were
(Because between 5 and 7 they point in opposite made by spinners. So, I does not follow. Nothing about
directions at 6 o’clock only).
the opening batsmen is mentioned in the statement.
So, in a day, the hands point in the opposite directions
So, II also does not follow.
22 times.
32. (3) Time from 7 a.m. to 4.15 p.m.
18. (3) In FUTURE, first two letters are reversed, last two letters
are reversed and middle two letters are unchanged. So, 37
= 9 hrs 15 min. = hrs.
NATURE will be coded as ANTUER. 4
SOLUTIONS MT-23

3 min. 5 sec. of this clock = 3 min. of the correct clock. From statement II.
37 2m
Þ hrs of this clock R
720 S
1
= hrs of the correct clock.
20 5m
37
Þ hrs of this clock
4
æ 1 720 37 ö P 4m Q
=ç ´ ´ ÷ hrs of the correct clock.
è 20 37 4 ø Hence P is to the southwest of S.
= 9 hrs of the correct clock. So, either alone I or II required.
\ The correct time is 9 hrs after 7 a.m. i.e., 4 p.m. 45. (4) A’s father’s Birthday may be on 8th or 9th December.
35. (2) 17 persons take tea and wine but not coffee. 46. (1) SP = 24000
From statement I
36. (3) Number of persons was taken both tea and coffee but
SP/CP = 5/3
not wine is 7.
CP = 144,00
37. (4) 122 persons take wine.
\ % profit = (24000 – 14400)/14000×100 = 68.5%
38. (2) Number of persons are there who takes only coffee From statement II
= 25 + 20 = 45 It is not clear that CP is greater than or less than SP.
39. (4) 15 persons take all the three. Hence, the data in statement I is sufficient.
41. (2) All the thieves are criminals while judge is different 47. (1) From statement I
from these. Marks in Biology = 42 × 2 = 84
42. (4) According to question, Hence, data in statement I alone is sufficient.
48. (1) In TEA, the alphabetical positions of T, E and A are 20,
A 5 and 1. The positions numbers are multiplied to get
the code as 100. Similarly, FAT can be coded as 120.
49. (4) Required distance = AF
C B = 30 + 15 = 45 m.
3 km.
30 m
5 km. B C

20 m
35 m
D A F
Hence required direction is South-West.
43. (1) From statement I. 15 m
Kapil = 12th from left
Raghav = 17th from right D E
= 50 – 17 + 1 = 34th from left 15 m
So number of studetns between them = 34 – 12 – 1 = 21 From the above diagram, F is in East direction from A.
From statement II. Hence the required answer is ‘45 m East’.
No data about Raghav
44. (3) From statement I. 1 km
50. (2) C B
P

3 km 2 km
7 km
T R

3 km D A
1 km
The boy rode 2 km. North-ward.
S 52. (4) 1st number of each term is forming a series of prime
Hence P is to the north of S. number in increasing order.
MT-24 Target NTSE
So, Next number = 31 16 + ? + ? = 57
Last number of each term is also forming a series of Þ ? + ? = 57 – 16 = 41
prime number is decreasing order. Þ 20 + 21 = 41
So, Next number = 5 63. (1) | # + < >
And middle letter (number) = Difference of 1st & 3rd ¯ ¯ ¯ ¯ ¯
number. B O A S T
So, Middle term (letter) = (31 – 5 = 26) = Z
So, 31Z5 is correct answer. 64. (3) E
53. (3) Go on subtracting 3 and dividing the result by 2 to
obtain the next number.
6 km
Clearly, 46 is wrong.
54. (4)
Starting B
×2 ×2 ×2 point A 2 km F
12 km
9 8 10 16 11 32 12 64 6 km

+1 +1 +1
55. (2) C 2 km D

56. (3) gfe i i/g f eii / g fei


North
i / gf e ii NW NE
West East
Solutions (57-60):
SW SE
Position 1 2 3 4 5 6 7 South
Required distance = AE
Person E B D A F/G G/F C
= ( AF )2 + ( FE )2 = (4)2 + (6)2
61. (2) Option (1)
(30 ÷ 5) × 10 = 24 = 16 + 36 = 52 = 7.2 km
Þ (30 ×10) ÷ 5 = 24 65. (2) Tom covered 210 km in = 5 hours
Þ 300 ÷ 5 = 24 Time taken by Joel to cover 210 km
Þ 60 ¹ 24 = 5. 00 – 1.30 = 3.30 hours
Option (2) = 3 ½ hours
(30 ×10) ÷5 = 60
Þ (30 ÷ 5) × 10 = 60 210
\ Average speed of Joel =
Þ 6 × 10 = 60 7
Option (3) 2
(30 ÷10) × 5 = 18
Þ (30 × 5) ÷ 10 = 18 210 ´ 2
= = 60 kmph
Þ 150 ÷ 10 = 18 7
Þ 15 ¹ 18 66. (2) Time taken to travel 200 km at the speed of 40 kmph
Option (4)
(10 ÷ 3) × 5 = 70 200
= = 5 hours
Þ (5 × 30) ÷10 = 70 40
Þ 150÷10 = 70 Total time of rest
Þ 15 ¹ 70 = (15 + 30 + 15) minutes
62. (2) The sum of all the three numbers in each column is = 60 minutes = 1 hour
equal to 57.
So, he will reach the destination at 9 : 00 + 6 : 00 = 3 : 00 PM
First Column
18 + 17 + 22 = 57 67. (2) Son of Ram’s father means Ram himself as he has no
Second Column brothers and sisters.
23 + 19 + ? = 57 Thus, man’s mother is wife of Ram.
Þ ? = 57 – 42 = 15 So, he was looking at the portrait of his son.
Third Column
SOLUTIONS MT-25

68. (2) Three cubes each in the column I and column II from 77. (1) Out of 1865 people, 120 people speak neither English
the left are not seen. Similarly, two cubes each in column nor Marathi.
III and IV are not seen. 1865 – 120 = 1745 people speak either English or Marathi
Total number of unseen cubes or both the languages.
= 3 + 3 + 2 + 2 = 10 The number of people who can speak both languages.
69. (1) The required region should be common in all the three = (660 + 1305) – 1745
geometrical figures. Such region is marked ‘7’. = 1996 – 1745 = 220
70. (2) Table is different from chairs but both come under the
class furniture.

English Marathi
Furniture 220
440 1085

Chair Table

78. (2) Required distance = (4 + 5) km = 9 km


5 km
P
North
71. (1)

10 km

10 km
West East

South

4 km
72. (4)
79. (4) 7 × 2 = 3 + 5 + 6
Þ 14 = 14
73. (2)
80. (4)
74. (4) F A I T H × Þ + +Þ ÷
¯ ¯ ¯ ¯ ¯
– Þ × ÷Þ –
8 2 7 3 1
H A B I T 6× 4–5+2÷1=?
Þ?=6+4×5÷2–1
¯ ¯ ¯ ¯ ¯
4´5
1 2 5 7 3 Þ?=6+ =1
2
H E A L T H Þ ? = 6 + 10 – 1 = 15
¯ ¯ ¯ ¯ ¯ ¯ 82. (3) 1 + 2 = 3
2+3=5
1 9 2 4 3 1 3+5=8
Therefore, 5 + 8 = 13
B E L I F E
¯ ¯ ¯ ¯ ¯ ¯ 83. (1)
5 9 4 7 9 8
75. (2) 1 2 3 4 5 6 7 8 8 7 6 5 4 3 2 1 84. (1) The product of outer four numbers is equal to the
central number.
N A T I O N A L ® L N AA N T O I First Figure
Therefore, 1 × 3 × 4 × 5 = 60
1 23 45 67 8 8 17 26 35 4 Second Figure
DOMEST I C ® CD I OT MSE 6 × 7 × 1 × 2 = 84
Third Figure
76. (2) Husband of Vikash’s mother means father of Vikash.
3 × 10 × 3 × 1 = 90
Therefore, that boy is the son of Vikash.
MT-26 Target NTSE
85. (3) First arrangement
Þ 6 + 4 + 3 + 1 + 5 + 0 = 19 95. (1) W ord Code
Þ 19 × 7 = 133 Pick/s tick ha/cu m
Second arrangement Yo u va
Þ 2+ 5 + 3 + 4 + 6 + 8 = 28
Bu rn pan
Þ 28 × 7 = 196
Third arrangement A re ta
Þ 1 + 5 + 7 + 3 + 4 + 2 = 22 Fire/ the ga/ cha’
Þ 22 × 7 = 154 Devil she
86. (4) First Figure Deal mo
2 × 2 × 3 = 12
Second Figure
2 × 3 × 5 = 30 96. (4) W ord Code
Third Figure M ars / in na/jo
5 × 1 × (– 1) = – 5 Ind ian s io
Fourth Figure M on ey pe
4 × 3 × (– 1) = – 12
Enter mo
87. (4) Six dots are on the face adjacent to one dot, three dots,
four dots and five dots. So, two dots lie opposite six Grou p/virtu al yo /al
dots. If we rotate the first dice downward two times, we Hammer/to/th e/nail ca/ra/ da/la
get the second dice. So, five dots like opposite three
dots.
97. (2) Letter B R A I N T E
88. (4) Expenditure on food : savings
= 120° : 60° = 2 : 1 Code * % ÷ # × $ +
89. (1) Q 70° – 54° = ` 1600 So, The code for TRAIN is $%÷#×.
Q 16° º ` 1600 99. (2) The figure may be labeled as shown.
1600 A B C
Q 1° º = ` 100
16
= ` 12000 J K
90. (2) Q 360° º ` 36000 H I D

36000
Q 1º º = ` 100 G F E
360
Q 60º º 60 × 100 = ` 6000 The simplest rectangles are ABJI, BCKJ, IJFG and JKEF
Q Annual savings i.e. 4 in number.
= ` (6000 × 12) The rectangles composed of two components each are
= ` 72000 ACKI, BCEF, IKEG and ABFG i.e. 4 in number.
91. (2) Distance between A and G The only rectangle composed of four components is
= (10 + 9 + 8 + 7 + 6 + 5) m = 45 m. ACEG.
92. (3) Required distance = Distance between G and C Thus, there are 4 + 4 + 1 = 9 rectangles in the given
= (5 + 6 + 7 + 8) m = 26 m. figure.
94. (2) Total number of Girls students from colleges P and T 100. (4) This is a different sort of flattened cube. But the
= 25500 × (15 + 10)/100 = 6375 fundamentals remain the same. If we want to find the
Total number of Boys students from colleges S and U side opposite the one with three dots, we consider it
= 35300 × (22 + 18)/100 = 14120 the base. The top side (opposite side) will then clearly
So required difference = (14120 – 6375) = 7745 be the one that has six dots.
SOLUTIONS MT-27

SAT (Scholastic Aptitude Test) MOCK TEST 3


ANS WER KEY
1. (4) 11. (2) 21. (3) 31. (3) 41. (1) 51. (3) 61. (1) 71. (3) 81. (2) 91. (4)
2. (2) 12. (2) 22. (2) 32. (1) 42. (2) 52. (3) 62. (2) 72. (2) 82. (3) 92. (2)
3. (1) 13. (4) 23. (1) 33. (2) 43. (1) 53. (4) 63. (3) 73. (2) 83. (2) 93. (3)
4. (1) 14. (3) 24. (4) 34. (4) 44. (1) 54. (2) 64. (1) 74. (3) 84. (4) 94. (4)
5. (4) 15. (4) 25. (4) 35. (2) 45. (1) 55. (1) 65. (4) 75. (4) 85. (2) 95. (4)
6. (3) 16. (3) 26. (3) 36. (3) 46. (1) 56. (2) 66. (2) 76. (3) 86. (2) 96. (1)
7. (2) 17. (3) 27. (4) 37. (2) 47. (1) 57. (2) 67. (1) 77. (3) 87. (2) 97. (1)
8. (1) 18. (2) 28. (1) 38. (2) 48. (1) 58. (3) 68. (4) 78. (1) 88. (2) 98. (3)
9. (2) 19. (3) 29. (1) 39. (2) 49. (2) 59. (2) 69. (4) 79. (3) 89. (4) 99. (1)
10. (1) 20. (2) 30. (3) 40. (3) 50. (4) 60. (3) 70. (2) 80. (1) 90. (2) 100. (1)

Hints and Explanations


1. (4) Golgi complex was discovered by Camillio Golgi. It is effects of the refinery. The panel found that the air
made up of various membrane system e.g. cisternae, has high levels of suspended particulate matter,
vesicles and vacuoles. It modifies macromolecules caused by factory emissions, dust, construction and
during their passage e.g., Glycosylation of protein (i.e., exhaust from automobiles. These are causing the Taj
addition of carbohydrates) to produce glycoproteins. Mahal to change colour.
Similarly, it synthesises hemicellulose, cellulose and 7. (2) 1 Tall Red: 1 Tall White
pectin compound during cell division and hence 8. (1) Photosynthesis is formation of organic food from
involved in cell wall formation in plant. Lysosomes are carbon dioxide and water with the help of sunlight inside
known as suicidal bags due to the presence of chlorophyll containing cells. Oxygen is produced as
byproduct. Simple carbohydrate or sugar (e.g. glucose)
hydrolytic enzymes.
is the main product from which other organic
2. (2) Flowers are said to be unisexual if they have either substances are formed. Excess of glucose is stored as
stamens (male or staminate) or carpels (female or starch to function as internal energy reserve to be used
pistillate), e.g. papaya, watermelon. They are known as as and when required. Starch develops a blue-black
bisexual if the flowers possess both stamens and colour with iodine solution.
carpels, e.g. mustard, Hibiscus. 9. (2) Build of Lactic Acid is due to anaerobic respiration.
3. (1) The rigourous activity results in anaerobic respiration
4. (1) Glottis is a narrow slit like opening through which air which results in the formation of lactic acid in the
enters into the trachea. Epiglottis guards the glottis muscle cells
during swallowing of food; and does not allow any 10. (1) 11. (2)
food particle to get into the trachea. Duodenum is a 12. (2) Manure provides a lot of organic matter (like humus)
proximal part of small intestine which receives partially to the soil which increases water retention capacity in
sandy soils and drainage as well as aeration in clayey
digested acidified food from stomach. Caecum is a small
soils. By using biological waste materials as a manure,
pouch that lies at the junction of small and large intestine
we recycle the wastes and protect our environment
which gives out a narrow blind tubular lymphoid from chemicals.
outgrowth called vermiform appendix. 13. (4)
5. (1) Observation A was made by Ravi 14. (3) Nervous tissue is made up of an organized network of
neurons specialised for conducting information via
As the stomata as placed in water, the water from the
electrical impulse from one part of the body to another.
solution enters into the guard cells and stomata open This impulse travels from the dendrite along the axon of
up. But in salt solution, water from the cells move into its end. At the end of axon, the electrical impulse sets off
the solution and hence they shrink the release of some chemicals, which cross the synapse
and start a similar impulse in a dendrite of the next neuron.
6. (3) The Mathura refinery was in the news for allegedly
15. (4) The hydrocarbon with the molecular formula C6H12 can
causing the white marble of the Taj Mahal to yellow. be an unsaturated, saturated, or cyclic hydrocarbon. It is
The India government hired a panel to examine the given that the compound does not react with hydrogen
MT-28 Target NTSE
and chlorine to give C6H14 and C6H12 Cl2, respectively. 24. (4) Black residue will appear due to formation of iron in
Therefore, it can not be an unsaturated hydrocarbon. tubes (II) and (IV) because of displacement reactions
It cannot be an open chain hydrocarbon also, because taking place.
molecular formula does not satisfy general molecular 2Al (s) + 3FeSO4 (aq) ® Al2(SO4)3(aq) + 3Fe(s)
formula of CnH2n + 2.
(Black residue)
The given formula satisfies with cyclohexane and Zn(s) + FeSO4 (aq) ® ZnSO4(aq) + Fe (s)
(Black residue)
cyclohexane is a saturated hydrocarbon.
25. (4)
(C6H12) 26. (3) The number of oxygen atoms = 9.033 × 1023
Therefore, both (a) and (d) are correct statements. (a) The no. of moles of oxygen

16. (3) 9.033 ´ 10 23


= = 1.499 moles = 1.5 moles
17. (3) Sulphide ores are roasted whereas carbonate ores are 6.022 ´ 10 23
calcinated. Liquation method is used for the easily (b) The mass of O2 = 1.5 × 16 g = 24 g
fusible metals like tin. Volatile metals like Zn and Hg
(c) 2H 2 + O 2 ¾¾
® 2H 2 O
etc., can be purified by distillation method.
18. (2) According to Mendeleev’s periodic low “properties of 2 mole of O2 requires = 4 g of H2
elements are periodic functions of their atomic masses”. 1.5 ´ 4
Thus isotopes should be place separately but as they 1.5 mole of O2 requires =
2
are chemically similar they are placed in same group.
= 3 moles H-atom
19. (3) Anhydrous CaCl2 absorbs moisture to keep the gas in
27. (4) Carbon compounds are of covalent nature. That is why
dry state. Otherwise, it will dissolve in moisture to form
these are poor conductors of electricity and do not
hydrochloric acid. have strong forces of attraction.
20. (2) Calamine solution contain ZnCO3 (base) and ant’s sting
28. (1) An object is said to be in uniform motion when it
contain formic acid. The reaction between acid and
base neutralises the irritating effect of ant’s sting. covers equal distances in equal intervals of time.
21. (2) Na2CO3 + CO2 + H2O ¾® 2NaHCO3 29. (1) According to Newton’s second laws of motion the
22. (2) The phenomenon of existence of a chemical element in external force acting on a body is directly proportional
two or more forms differing in physical properties but to the rate of change of linear momentum.
dp
having almost same chemical nature is known as i.e. F µ or F = Kma = ma [as K = 1]
dt
allotropy. Graphite and diamond are crystalline 30. (3) Gravitational force is independent of medium, Hence,
allotropes of carbon. Graphite is a good conductor of this will remain same.
electricity due to presence of mobile electrons in it. 31. (3) According to Archimedes’ principle and Law’s of
CnH2n is a general formula for alkenes. Ethanol is floatation, volume of solid immersed in the liquid =
prepared by fermentation of molasses. Molasses, a by volume of liquid displaced.
product of sugar industry, has traces of sugar, which 32. (1)
gets broken down into ethanol. The fermentation is 34. (4) The distance between two consecutive compressions
initiated by yeast, which secretes two enzymes called or two consecutive rarefactions is called the
invertase and zymase. The reaction is exothermic and wavelength l.
carbon dioxide is released in the process. 35. (2) When a ray of light passes from one medium to another
Invertase medium its direction changes.
C12 H 22 O11 + H 2 O ¾¾¾¾® C 6 H12 O6 + C6 H12 O6
Sucrose
36. (3)
Glucose Fructose
rl
Zymase
37. (2) We know that, R =
C6 H12 O6 ¾¾¾¾
® 2C 2 H5OH+ 2CO 2 A
Glucose or fructose Ethanol
rl 2
23. (1) or R= Þ R µ l2
Volume
SOLUTIONS MT-29

According to question l2 = nl1 Þ (x – 2)3 = [21/3(1 + 21/3)]3


Þ x3 – 8 – 3.x2.2 + 3.x.22 = 2 ( 1+21/3)3
R2 n 2l12 R 2
= 2 or,, 2 = n Þ R2 = n2R1 Þ x3 – 8 – 6x2 + 12x = 2 ( 1 + 2 + 3.12.21/3 + 3.1.22/3)
R1 l1 R1
Þ x3 – 6x2 +12x – 8 = 2 [ 3 + 3.2 1/3 +3.2 2/3]
38. (2) Properties of magnetic lines of force = 6 (1 + 21/3 + 22/3)
3 2
Þ x – 6x + 12x – 8 = 6(x– 1) ...(i)
(i) they always start from north pole and end at the
south pole of the magnet outside the magnet.
é 1 2 ù
(ii) they never intersect each other. êQ x = 2 + 2 3 + 2 3 ú
39. (2) ê ú
ê 1 2 ú
40. (3) êë\ x - 1 = 1 + 2 3 + 2 3 úû
143
1 1 Þ x3– 6x2 + 12x – 8 = 6x – 6
41. (1) a- b = å k + k +1
= å k + +1 + k Þ x3– 6x2 + 12x – 6x – 8 + 6 = 0
k =4 k =4
Þ x3– 6x2 + 6x – 2 = 0
1 1 1
= + + + ... æ 132 ö
5+ 4 6+ 5 7+ 6 44. (1) loga ç ÷ = 2 log 13 - log 5 - x
ç 3 ÷ a a
1 1 è 2 ´5 ø
+ +
143 + 142 144 + 143
log a 132 - log a ( 23 ´ 5) = 2 loga 13 - loga 5 - x

Þ 2log a 13 - log a 23 - log a 5


1 5- 4 1 ( 6 - 5)
= ´ + ´ = 2 loga 13 - loga 5 - x
5+ 4 5- 4 6 + 5 ( 6 - 5)
1 ( 7 - 6) 1 Þ x = log a 23
+ ´ + ... +
7 + 6 ( 7 - 6) 143 + 142 Þ a x = 23/ 2
( 143 - 142) 1 45. (1) Join BE and EF.
´ +
( 143 - 142) 144 + 143 Since AB is the diameter of the circle

´
( 144 - 143 ) ÐAEB = 90° [Angle in the semicircles is 90°]

( 144 - 143) 1 40°


ÐEBF = ÐEOF = = 20°
2 2
[Angle made by a chord at any point on the
= ( 5 - 4) + ( 6 - 5) + ( 7 - 6) + ... circumference is half the angle made by the chord at
the centre of the circle.]
+( 143 - 142) + ( 144 - 143)

= - 4 + 144 A
= – 2 +12 = 10
\ a - b = 10 + 0 O
E
40°
By comparision
a = 10 and b = 0
B D
Hence, required value is a = 10, b = 0. F
42. (2) For the relation x = qy + r, 0 £ r < y
So, here r lies between 0 £ r < 6. Also ÐAEB = 90°
Hence, r = 0, 1, 2, 3, 4, 5 \ ÐBED = 180° – 90° = 90°
43. (1) x = 2 + 21/3 + 22/3 In DBED,
x – 2 = 21/3 + 22/3 = 21/3 (1 + 21/3)
MT-30 Target NTSE
ÐBED + ÐEBD + ÐEDB = 180° Hence, required total area of quadrilateral
90° + ÐEBF + ÐEDB = 180° [Q ÐEBD = ÐEBF] = 30 + 485.40 = 515.40 m2
90° + 20° + ÐEDB = 180°
48. (1) Let the two A.P’s be a1, a2, a3, .... an, ... and
ÐEDB = 180° – 110° = 70°
i.e. ÐEDF = 70° b1, b2, b3, ...., bn, ....
Also, let d be the common difference of two A.P’s.
P
Then,
an = a1 + (n – 1) d and bn = b1 + (n – 1) d
p60° T
(1) Q S Þ an – bn = {a1 + (n – 1) d} – {b1 + (n – 1) d}
46. q
Þ an – bn = a1 – b1
R Clearly, an – bn is independent of n and is equal to
a1 – b1. In other words
ÐQPR = ÐQSR (angles subtended by same chord)
Þ ÐQPR = q an – bn = a1 – b1 for all n Î N.
Þ ÐQPT = q Þ a100 – b100 = a1 – b1
Now, in DPQT
and, ak – bk = a1 – b1,
ÐPQT + ÐQPT + ÐQTP = 180°
p + q + 60° = 180° (Q ÐQPT = q) where k = 10,00,000.
p + q + =180° – 60° But, a100 – b100 = 111 222 333
p + q = 120° \ a1 – b1 = 111 222 333
Hence, required value of p + q is 120°.
Þ ak – bk = a1 – b1 = 111 222 333,
47. (1) Given, a quadrilateral ABCD in which AB = 9 m,
BC = 12 m, CD = 5 m, and AD = 8 m. We divide the where k = 10,00,000.
quadrilateral in two triangular region ABD and BCD. 49. (2) Let the common difference = d.
Now, in D BCD, right angle at C. x1 + x7 + x10 = – 6
We have BD2 = BC2 + CD2 x1 + x1 + 6d + x1+ 9d = – 6 .........(i)
[By Pythagorous theorem] and x1 + 2d + x1+ 7d + x1+ 11d = – 11 .........(ii)
Equation (i) becomes 3x1 + 15d = – 6
Equation (ii) becomes 3x1 + 20d = – 11
From equation (i) – (ii),
– 5d = 5 Þ d = – 1
Put the value of d in equation (i),
3x1 + 15(– 1) = – 6 Þ x1 = 3
Now, x3 + x8 + x22 = x1 + 2d + x1+ 7d + x1 + 21d
= 3x1 + 30d = 3(3) + 30(– 1) = – 21.
= (5)2 + (12)2 = 25 + 144 = 169 50. (4) Consider the given equations
BD = 169 = 13 m 4x – 3y = 7xy ...(1)
3x + 2y = 18xy ...(2)
1 From equation (1) × 2 + (2) × 3
\ Area of DBCD = × base × height
2 8 x - 6 y = 14 xy
1 9 x + 6 y = 54 xy
= × 5 × 12 = 30 m2
2 17 x = 68 xy
In DABD, by Heron’s formula
17 x 1
Þ y= Þy=
Area (DABD) = s ( s - a )( s - b)( s - c ) 68 x 4
Then from equation (1)
= 30 ´ 22 ´ 21´ 17 = 485.40 m 2 1 1
4x - 3´ = 7x ´
4 4
SOLUTIONS MT-31

7 3 16 - 7 3 53. (4) We have,


Þ 4x - x = Þ x= sin (q + 36°) = cosq
4 4 4 4
Þ cos{90° – (q + 36°)} = cosq
9 3 1
Þ x= Þ x= Þ 90° – (q + 36°) = q Þ 2q = 54° Þ q = 27°
4 4 3 54. (2) Arranging the observation in ascending order :
æ1 1ö 7400, 7800, 8100, 8500, 9200, 10320, 11300, 11500, 12000,
Hence, ( x, y ) = ç , ÷ 12700
è3 4ø
Total number of observations (n) = 10 (even)
51. (3) We have,
1 éæ n ö ù
th th
x + 2y + z = 7 ... (i) æn ö
\ ê
median = ç ÷ observation + ç + 1÷ observation ú
x + 3z = 11 ... (ii) 2 êëè 2 ø è2 ø úû
2x – 3y = 1 ... (iii)
From equation (i), we get
1 éæ 10 ö ù
th th
z = 7 – x – 2y æ 10 ö
= ê ç ÷ observation + ç + 1 ÷ observation ú
Substituting z = 7 – x – 2y in equation (ii), we get 2 êè 2 ø è 2 ø úû
ë
x + 3 (7 – x – 2y) = 11
Þ x + 21 – 3x – 6y = 11 1 é th
= 5 observation + 6 th observation ù
Þ –2x – 6y = –10 ...(iv) 2ë û
Adding equations (iii) and (iv), we get
– 9y = – 9 Þ y = 1 1 19520
Median = [9200 + 10320] =
Putting y = 1 in equation (iii), we get x = 2 2 2
Putting x = 2, y = 1 in equation (i), we get Median = 9760
2+ 2+ z=7Þ z=3 Median Salary = ` 9760
Hence, x = 2, y = 1, z = 3
55. (1) Internal diameter of the pipe = 2 cm
52. (3) Let AB be the tower of height h meter and BC be the
height of flag staff surmounted on the tower. 1
So its radius = 1cm = m
Let the point of the plane be D at a distance m meter 100
from the foot of the tower. Water that flows out through the pipe in 6 ms–1
In D ABD, So volume of water that flows out through the pipe in
2
AB æ 1 ö
tan 30° = 1 sec = p´ ç ÷ × 6 m3
BD è 100 ø
\ In 30 minutes, volume of water flow
1 h 1
Þ = Þ x = 3h ...... (1) =p ´ 6 ´ 30 ´ 60 m3
3 x 100 ´100
This must be equal to the volume of water that rises in
AC the cylindrical tank after 30 min and height up to which
In DADC, tan 60° =
AD it rises say h.
C
60
5+ h 5+h Radius of tank = 60 cm = m
Þ 3= Þx= ...... (2) 5m 100
x 3 2
volume = p æç
B 60 ö
÷ h
5+h è 100 ø
From (1) and (2), 3h = æ 60 ö
2
1
3 pç ÷ h = p´ ´ 6 ´ 30 ´ 60
è 100 ø 100 ´100
Þ 3h = 5 + h Þ 2h = 5 h
60 ´ 60 6 ´ 30 ´ 60
Þ h=
5 100 ´100 100 ´ 100
Þ h= = 2.5 m 60°
3 ´ 36
2 Þ h= = 3m
30°
D A 36
x
So required rise in the level of water is 3 m.
So, the height of tower = 2.5 m
MT-32 Target NTSE
56. (2) We have
ÐA = 50°and ÐB = 60°
\ ÐA + ÐB + ÐC = 180°
[Angle sum property of a D]
Þ 50° + 60° + ÐC = 180°
Þ 110° + ÐC =180°
Þ ÐC = 180° – 110° So, AB = 600.
Þ ÐC = 70° ÐACB = 60º, ÐDCB = 45º
Since ÐA and ÐC are the smallest and largest angles, AB 600
therefore sides BC and AB are the smallest and largest From DABC, = tan 60º Þ BC = = 200 3
BC 3
sides of the triangle respectively.
DB
8 From DDCB, = tan 45° Þ DB = 200 3 .
57. (2) External radius of the hollow sphere = = 4 cm, BC
2
So, the distance AD = AB – BD = 600 – 200 3
4
Internal radius of the hollow sphere = = 2 cm = 200 (3 – 3 ) = 200 (3 – 1.7321) = 253.58 m.
2
1 2
59. (2) Area of quadrant = pr
\ Volume of the metal = Volume of external sphere 4
– Volume of internal sphere 1 22 7 7
= ´ ´ ´ = 9.63 cm2
4 7 2 2
4 4 4 4
= p(4)3 - p(2)3 = p é 43 – 23 ù = p [64 – 8] 1 1
3 3 3 ë û 3 Area of D AOD = ´ OA ´ OD = ´ 3.5 ´ 2 = 3.5 cm2
2 2
4
= p (56) cm3 \ Area of shaded portion = 9.63 – 3.5 = 6.13 cm2
3
60. (3) Total number of two digit numbers = 90
8 Now, find the total two digit numbers which are multiple
Radius of the cone = cm = 4 cm
2 of 3.
Since, the series 12 + 15 + .... + 99 contain all two digit
Let h be the height of the cone. numbers which are multiples of 3.
1 Then, an = a + (n – 1)d
Then volume of the cone = p(4) 2 h cm3
3 99 = 12 + (n – 1) × 3
Since the metal of the spherical shell is to be converted 99 - 12
into the conical solid, n = +1
3
n = 30
So, there are 30 two digit numbers which are multiple
of 3.
2ccmm
P (choosing a multiple of 3)
44c c
mm

number of two digit number which are multiple of 3


=
Total number of two digit numbers

30 1 1
4 1 4 ´ 56 = = Þ Hence, required probability is .
\ p(56) = p(16) h Þ h = = 14 cm. 90 3 3
3 3 16
61. (1) Population of France increased from 23 million to 28
Hence, the height of the cone is 14 cm.
58. (3) Let the aeroplanes are at point A and D respectively. million in 1789.
Aeroplane A is flying 600 m above the ground. 62. (2) 63. (3)
SOLUTIONS MT-33

64. (1) Indian indentured labour was described as coolie. 83. (2) A lake helps to regulate the flow of a river. During heavy
rainfall lakes prvents flooding and during dry season,
65. (4) Printed message persuaded people to think differently.
it helps to maintain even flow of wate. Lakes moderate
Their views was critical as well as rational. the climate of the surrounding and maintains acquatic
66. (2) 67. (1) 68. (4) ecosystem.
69. (4) On the morning of 14th July, 1789, people of Paris 84. (4)
stormed the Bastille, where they hoped to find the 85. (2) Democracy is a form of government in which the
hoarded ammunition. Bastille stood for the despotic administration of the country is run by the
power of the King.
representatives who are elected by the people.
70. (2) The correct statement with egad to Cavours contriution
to Italian unification is that he followed diplomatic 86. (2) If the elections for electing the representative are not
alliance with France in 1859 and strengthening Sadinia fair and impartial people will lose faith in the entire
and Piedmont. Cavour supported France and England democratic system.
against Russia in the Crimean War and gained their 87. (2) 88. (2) 89. (4)
support against Austria. 90. (2) The International Monetary Fund (IMF) gives loan to
71. (3) Diem retained Ordinance 10, it was a French Law which the government more than 52% of the voting power is
permitted Christianity but outlaed Buddhism. His in the hands of ten countries i.e., US, Japan, France,
dictational rule came to e opposed by a borad UK, Germany, Italy, Russia, China, Saudi Arabia and
opposition. Canada.
72. (2) During the 1905, Revolutio, the Tsar alloed the creation 91. (4) Pressure groups are organisations that attempt to
of an elected consultative Parliament or Duma. The influence government policies. But unlike political
Tsar dismissed the first Duma within 75 days. He did parties pressure control or share political power.
not ant any questioning of his authority or any Pressure groups much more on spontaneous mass
reduction in his power. participation.
73. (2) Meteorologists use the location of some of the jet 92. (2) In India, the Election Commission is independent and
streams as an aid in weather forecasting. very powerful. Everyone above the age of 18 has the
voting right. The losing political parties accept electoal
74. (3) Golden quadrilateral is managed by the National verdict.
Authority of India under the ministry of Road Transport 93. (3) MNC i.e. Multinational Companies set up offices and
and Highways. factories for production in regions where they can get
75. (4) India is the third largest producer of nitrogenous cheap labour and other sources.
fertilizers. There are about 10 public sector undertakings 94. (4) Banks and cooperatives come under formal lenders.
of fertiliser industry. 95. (4) RBI also monitors how much the banks are lending, to
76. (3) 77. (3) 78. (1) 79. (3) whom and at what interest rate.
80. (1) Natural vegetation grows on its own without any 96. (1) Migration resulted in a steady increase in the
interference of human beings. percentage of population in cities.
81. (2) 97. (1) 98. (3) 99. (1)
82. (3) Western Coastal Plain belt is narrow in width and less 100. (1) Social exclusion may lead to more damage than having
fertile because no major river is engaged in sediment a very low income.
deposition. These plains receive more rainfall.
MT-34 Target NTSE

MAT (Mental Ability Test) MOCK TEST 4


ANS WER KEY
1 (2) 11 (1) 21 (4) 31 (3) 41 (1) 51 (1) 61 (3) 71 (3) 81 (2) 91 (2)
2 (4) 12 (2) 22 (1) 32 (4) 42 (2) 52 (3) 62 (4) 72 (2) 82 (2) 92 (1)
3 (2) 13 (4) 23 (2) 33 (4) 43 (2) 53 (4) 63 (3) 73 (3) 83 (1) 93 (3)
4 (4) 14 (2) 24 (4) 34 (2) 44 (1) 54 (1) 64 (4) 74 (2) 84 (2) 94 (4)
5 (2) 15 (3) 25 (2) 35 (2) 45 (2) 55 (2) 65 (1) 75 (1) 85 (1) 95 (4)
6 (1) 16 (3) 26 (4) 36 (3) 46 (3) 56 (1) 66 (1) 76 (3) 86 (2) 96 (1)
7 (4) 17 (4) 27 (2) 37 (3) 47 (4) 57 (4) 67 (3) 77 (4) 87 (4) 97 (2)
8 (4) 18 (2) 28 (3) 38 (2) 48 (3) 58 (4) 68 (4) 78 (4) 88 (4) 98 (3)
9 (4) 19 (1) 29 (4) 39 (4) 49 (1) 59 (4) 69 (3) 79 (3) 89 (2) 99 (1)
10 (3) 20 (1) 30 (2) 40 (3) 50 (2) 60 (1) 70 (1) 80 (2) 90 (4) 100 (4)

Hints and Explanations


1. (1) In PF(1), top half darkened rectangle turned 90 ° OX. On further moving 180o anticlockwise, he faces in
clockwise, middle half darkened rectangle turned anti- the direction OY. Finally, on moving 90o anticlockwise,
clockwise 90° and bottom half darkened rectangle he faces in the direction OZ, which is South-east.
turned clockwise by 90°. So turn the rectangles in
PF(C), clockwise, anti-clockwise, and clockwise. P X
180°
So correct Option 1
2. (4) There is not enough information to support any of the 90°
N
statements. Robert is known to have a minvan, but it is
not known which of his vehicles is red. Robert may E
have a pickup or sport utility vehicle, so the second
W
statement cannot be supported. There is no way to S
know if Robert’s favorite color is red (statement III). 90°
3. (4) If Mary always tells the truth, then both Ann and Mary
have cats (statements I and II), and Ann is lying Y Z
(statement III). So all the statements are facts. 14. (4) The given puzzle follows a pattern that,
4. (1) There are five blocks including A. All others touch A Number ® < last digit > < sum of all digits >.
5. (4) Environmentalist with Engineering background 111 ® 13
6. (2) Asian, Non-Christian females who are professionals 112 ® 24
are represented by ‘10’. 113 ® 35
7. (1) Asian females who are neither professional nor 114 ® 46
Christians are denoted by ‘6’. 115 ® 57
8. (3) Non-Asian professional Christian males are 116 ® 68
represented by ‘12’. So, 117 ® 79
9. (4) All Christians females of Asia are Professionals.
10. (2) Shaded portion represents Non-Asian professional 15. (2) C (–)
female Christians.
11. (1) Spouse means a person’s partner in marriage, it can be
male or female. A (+)
But given F is spouse of B And P is Son in law of N
Relation between F and B Relation between P and A
only and p is male So, A must be female Þ B is male
child. B
12. (2) Call A the event that Ramu is present in the photo. Call 16. (2) Since one-half of the four children are girls, two must
B the event that his brothers are present. We are given be boys. It is not clear which children have blue or
that A®B. So, we know the identities of three of the brown eyes.
people in that photo i.e. Ramu and his three brothers. Solution (17-18):
But, we are told that the photo already has three people Not Adventurous = 4 + 5 + 3 + 2 + 1 + 6
(Ramu’s parents with their daughter). This would mean, Not Adventurous but Original = 5 + 3 + 2 + 6
there is a total of six people in the photograph. This is Not Adventurous but Original and Humble = 2 + 6
impossible as it is clearly stated that there are only five Not Adventurous but Original, Humble and Active = 6
people in the photo. Thus, Ramu cannot be in the photo. Similarly,
Hence, option (2) is the correct choice. Not Humble but Original, Adventurous and Active = 8
13. (3) As shown in fig. the man initially faces in the direction 17. (1)
OP. On moving 90o clockwise, he faces in the direction 18. (2)
SOLUTIONS MT-35

19. (3) 20. (1) 21. (4)


25. (2) F L O W E R Z G K T C Q
Banana Guava P/P Raspberry
–1
North –2
–3
Apple P/P –4
Mango Lemon
–5
or
–6
P/P Apple lemon Mango

or North N A T U R E H V P R P D
–1
Raspberry P/P Guava Banana –2
–3
Papaya or Pomegranate
–4
–5
Banana Guava Raspberry –6
26. (1) The colours adjacent to yellow are orange, blue, red
and rose. Hence violet will be opposite to yellow.
27. (4) The correct order is :
Mango Lemon Apple Letters Word Phrase Sentence Paragraph
4 1 5 3 2
Solutions (28-31):
Papaya or Pomegranate N
22. (3) ‘ski rps tri’ ® ‘nice Sunday morning’ ……(i)
‘teh sti rps’ ® ‘every Tuesday morning’ …..(ii)
‘ski ptr qlm’ ® ‘nice market place’ ……(iii) W E
From (i) & (ii) we get ‘rps’ stands for ‘morning’.
From (i) & (iii) we get ‘ski’ stands for ‘nice’. G B C D E F A
S
Thus, in (i) statement, ‘tri’ stands for ‘Sunday’. 28. (3)
23. (2) ‘123’ ® ‘bright little boy’ …..(i) 29. (1)
‘145’ ® ‘tall big boy’ …..(ii) 30. (2)
‘637’ ® ‘beautiful little flower’ …..(iii)
31. (4)
From (i) & (ii) we get ‘1’ means ‘boy’.
32. (1) According to the rule no. (3), common faces with
From (i) & (iii) we get ‘3’ means ‘little’.
Thus, in (i) statement ‘2’ means ‘bright’. number 2 are in same positions. Hence when the digit 5
is on the bottom then 1 will on the upper surface.
24. (4) S P E A K 33. (1)
S

+2 +2 +2 +2 +2
Q P I

U R G C M U R
husband wife, male, famale
L A U G H
34. (2)
35. (2)
+2 +2 36. (3) The given sequence is a combination of two series
+2 +2 +2
I. 13, 24, 35, 46 , 57 and II. 32. 43, ? , 65 , 76
The pattern in both I and II is + 11.
So, missing term = 43 + 11 = 54
N C W I J 37. (3) First letter of the first term and second letter of the
second term are in alphabetical order. Similarly, first
MT-36 Target NTSE
letter of the third term and second letter of the fourth 54. (1) B R A I N T E
term are in order. * % ÷ # × $ +
38. (3) Net ascent of the monkey in 1 h = (30 – 20) feet = 10 E N T E R
feet. ¯ ¯ ¯ ¯ ¯
So, the monkey ascends 90 feet in 9 h i.e, till 5 pm + × $ + %
Clearly , in the next 1 h i.e., till 6 pm the monkey ascends 12 + 18 + 30
remaining 30 feet to touch the flag. 55. (3) =6
10
39. (3) The given sequence is a combination of two series
I. 11, 20, 40, 74 and II. 5, 12, 26, 54 16 + 24 + 40
The correct pattern in I is + 9, +18, + 36, …. =8
10
So, 40 is wrong and must be replaced by (20 + 18) i.e.,
38.
45 + 18 + 27
Similarly, = 9.
40. (2) First figure 10
(6 × 5) + (3 × 3) = 30 + 9 = 39 56. (3) (15 – 12) + (10 – 9) = 4
Second Figure (28 – 12) + (15 – 20) = 12
(7 × 5) + (4 × 4) = 35 + 16 = 51 Similarly, (23 – 11) + (15 – 16) = 13.
Third Figure Solutions (57 – 58):
(5 × 5) + ( 3 × 4) = 25 + 12 = 37 According to the given information, the position of eight
41. (3) Proceed clockwise: persons is as follows
9 + 3 = 12 A E
12 + (3 × 2) = 18 40 m N
18 + (6 × 2) = 30 30 m
30 + (12 × 2) = 54 50 m
20 m W E
54 + (24 × 2) = 102 C
42. (2) The statement stresses the need to adopt a new method B
of examination. So, I does not follow. However, II directly S
follows from the given statement. F D
40 m
43. (4)
44. (3) According to question, 15 m
Q > P ....... (i) G H
T > R > Q ……..(ii) 20 m
Combining these two statements T > R > Q > P. Clearly, 57. (2) So, B is standing in North direction with respect to F.
R scored the second Highest. 58. (4) As total distance between I and D is not given, so this
45. (2) The sitting arrangement will be as follows. question cannot be determined.
Daughter ® Mother ® Grandfather ® Father ® Son Solutions (59 – 61):
So, The order of age is as following
Grandfather is occupying the central place. Shruti > Amit > Tanu
46. (1) All the three are different from one another. Ritu > Bharat > Chander > Shruti
47. (3) In each subsequent figure one dot is added and So, Ritu > Bharat > Chander > Shruti > Amit > Tanu
alternatively dots become white. Again, in each The order of height is as following
subsequent figure one arrow is deleted. Shruti > Amit > Tanu > Chander > Ritu
48. (2) 49. (2)
50. (4) The third figure in each row comprises of parts which Bharat
are not common to the first two figures. 59. (4) Either Shruti or Bharat is tallest. So, the correct answer
51. (2) In each row, the second figure is obtained from the first is (4).
60. (3) Amit is older than Tanu but younger than Shruti. So,
figure by adding two mutually perpendicular line
the correct answer is (3).
segments at the centre and the third figure is obtained 61. (4) Ritu is the shortest. So, the correct answer is (4).
from the first figure by adding four circles outside the 62. (3) Letters reversed from right to left.
main figure.
52. (1) In each row, the central part of the first figure rotates P A R E N T
either 90o CW or 90o ACW to form the central part of
the second figure and the central part of the first figure
rotates through 180o to form the central part of the third
figure. Also, in each row, there are 3 types of side T N E R A P
elements - rectangles, circles and triangles.
C H I L D R E N
53. (3) Q
|
M (+ ) Û N(-) — P (-)
N E R D L I H C
SOLUTIONS MT-37

63. (3) bcab/bcab/bcab/bcab/bcab 77. (2) Let the speed of the train & the car ® x km/hr & y km/
64. (2) Assumption I goes against the statement. So, it is not hr respectively
implicit. The allowance will serve as a reward to the
employees and shall provoke them to come on time. 120 480
+ =8 ...(i)
So, II is implicit. x y
65. (1)
66. (3) From I, we conclude that weight of each pole = (4 × 5) 200 400 25
+ = ...(ii)
kg = 20 kg. x y 3
So, total weight of 10 poles = (20 × 10) kg = 200 kg. From (i)
From II, we conclude that: 120y + 480x = 8xy ...(iii)
Weight of each pole From (ii)
= (weight of 3 poles) – (weight of 2 poles) = 20 kg.
So, total weight of 10 poles = (20 × 10) kg = 200 kg. 25
67. (4) From I, we conclude that H is the only daughter of M. 200y + 400x = xy ...(iv)
3
But this does not indicate that M has no son. The From (iii) & (iv)
information given in II is immaterial.
68. (3) From I, we conclude that 5th, 12th, 19th and 26th of 120 y + 480 x 3(200 y + 400 x)
=
March, 2006 were Sundays. 8 25
So, the last Sunday fell on 26th. x : y= 3 : 4
From II, we conclude that 31st March, 2006 was Friday.
Thus, 26th March, 2006 was the last Sunday of the 78. (3) Milk sa
month.
69. (1) From Statement II : is ja
(Soham + Mohit + Rita)’s 1 day’s work = 37 /180 tea ha
37 3 black ka
Mohit’s 1 day’s of work = – = 18 days Very/tasty ta/la
180 20
70. (2) From I, 4x = 12000. So CP = 9000 Sweet/and pa/ra
And profit% can be found.
From II we do not know that SP is greater than CP or
less 79. (4) W ord Code
10 open zb
71. (4) Required angle = × 360 = 36° boo k fa
100
72. (2) x × 35% = 17500 the mt
35 math cv
x× = 17500
100 are ka
x = 50000
goo d la
15
So, Royalty paid = 50000 × = 7500 frien d at
100
tou gh nt
4
73. (3) x × = 6000 ques tio n qm
100
as k yb
x = 150000
18 - 15 80. (2)
= 150000 × 81. (3) The figure may be labeled as shown.
100
= 4500
B
35 - 18 A C
74. (2) Required angle = ´ 360° = 61.2°
100 E
D F
10
75. (2) Required percentage = × 100 = 28.6%
35 H
76. (3) Total number of man players from countries D and G G I
22 + 18
= 23500 × = 9400 J L
100 K
Total number of woman players from countries A and G
We shall join the centres of all the circles by horizontal
12 + 8
= 32500 × = 6500 and vertical lines and then label the resulting figure as
100 shown.
Required ratio = 94 : 65
MT-38 Target NTSE
The simplest squares are ABED, BCFE, DEHG, EFIH, 89. (3) According to question,
GHKJ and HILK i.e. 6 in number.
The squares composed of four simple squares are ACIG x+6 7
=
and DFLJ i.e. 2 in number. x+ 2+6 8
Thus, 6 + 2 = 8 squares will be formed. Þ x=8
82. (1) The figure may be labeled as shown.
Hence, the ages of A and B = 8 years and 10 years
E respectively
A B
I J
90. (3) 33 ÷ 9 = 3

H F 63 ÷ 12 = 18
O
83 ÷ 32 = 16
L K
D C 91. (4) As,
G
3 + 6 + 5 = 14th letter of English alphabet = N
Triangles :
The simplest triangles are AEI, EOI, OHI, HAI, EBJ, 8 + 10 + 6 = 24th letter of English alphabet = X
BFJ, FOJ, OEJ, HOL, OGL, GDL, DHL, OFK, FCK, CGK 3 + 6 + 9 = 18th letter of English alphabet = R
and GOK i.e. 16 in number.
Similarly,
The triangles composed of two components each are
HAE, AEO, EOH, OHA, OEB, EBF, BFO, FOE, DHO, 6 + 9 + 7 = 22nd letter of English alphabet = V
HOG, OGD, GDH, GOF, OFC, FCG and CGO i.e. 16 in 92. (3) 10 – 2 ¹ 5 ÷ 5 + 5
number. 10 + 2 – 5 ÷ 5 ¹ 5
The triangles composed of four components each are
HEF, EFG, FGH, GHE, ABO, BGO, CDO and DAO i.e. 8 10 ÷ 2 + 5 – 5 = 5
in number. 10 – 2 ÷ 5 + 5 ¹ 5
The triangles composed of eight components each are 93. (4)
DAB, ABC, BCD and CDA i.e. 4 in number.
Total number of triangles in the figure 94. (3)
= 16 + 16 + 8 + 4 = 44. 95. (3) Since Erin’s parents think a dog would not be happy in
83. (2) an apartment, we can reasonably conclude that the
84. (4) The figure (X) is similar to Form II. So, when a cube is family lives in an apartment. We do not know if Erin’s
formed by folding the sheet shown in fig. (X), then the parents dislike dogs (choice a) or if Erin dislikes birds
two half-shaded faces lie opposite to each other and (choice b).There is no support for choice d.
one. of the three blank faces appears opposite to the
face bearing a dot. Clearly, each one of the four cubes 96. (2) 35 km
shown in figures (1), (2), (3) and (4) can be formed by
folding the sheet shown in fig. (X).
85. (3) 50 km 50 km
86. (4)

35 km
97. (1)
98. (3) Similar figure reappears in every fourth step and each
time a figure reappears, it rotates through 90oACW.
87. (2) Sagar > Tarun > Amit > Anu but Reena has not least 99. (4) Next number is 13U29
marks. 100. (1) As,
So, Anu has least marks.
12 + 16 + 14 + 18 = 60
88. (2)
18 + 8 + 14 + 20 = 60
Cousin
Girl Boy 20 + 6 + 10 + 24 = 60
16 + 13 + 7 + 24 = 60
Mother
Similarly,
12 + 7 + 15 + 16 = 60
Grand Father Sister
Daughter Father
SOLUTIONS MT-39

SAT (Scholastic Aptitude Test) MOCK TEST 4


ANS WER KEY
1 (3) 11 (4) 21 (4) 31 (1) 41. (1) 51. (2) 61. (3) 71. (3) 81. (1) 91. (3)
2 (3) 12 (4) 22 (2) 32 (2) 42. (3) 52. (2) 62. (1) 72. (2) 82. (3) 92. (3)
3 (1) 13 (3) 23 (4) 33 (3) 43. (4) 53. (3) 63. (4) 73. (2) 83. (2) 93. (2)
4 (2) 14 (3) 24 (4) 34 (1) 44. (2) 54. (4) 64. (2) 74. (2) 84. (2) 94. (2)
5 (1) 15 (2) 25 (4) 35 (3) 45. (3) 55. (2) 65. (1) 75. (3) 85. (2) 95. (1)
6 (2) 16 (4) 26 (2) 36 (2) 46. (2) 56. (4) 66. (2) 76. (3) 86. (1) 96. (4)
7 (1) 17 (3) 27 (1) 37 (2) 47. (2) 57. (3) 67. (3) 77. (2) 87. (3) 97. (3)
8 (2) 18 (2) 28 (3) 38 (2) 48. (4) 58. (4) 68. (2) 78. (2) 88. (3) 98. (4)
9 (4) 19 (4) 29 (4) 39 (4) 49. (3) 59. (3) 69. (2) 79. (3) 89. (3) 99. (2)
10 (1) 20 (1) 30 (1) 40 (3) 50. (1) 60. (3) 70. (1) 80. (2) 90. (2) 100. (2)

Hints and Explanations

1. (3) Purkinje in 1839 coined the term ‘protoplasm’ for the 13. (3) Abscisic acid is a plant hormone which promotes
fluid substance of the cell. The cell theory, that all the senescence of leaves and causes abscission of flowers
plants and animals are composed of cells and the cell is and fruits.
the basic unit of life, was presented by two biologists, 14. (3) Cerebrum is the main thinking part of brain. It has
Schleiden (1838) and Schwann (1839). A single cell may sensory, motor and association areas. Cerebellum is
constitute a whole organism as in Amoeba, second largest part of the brain. It coordinates muscular
Chlamydomonas, Paramoecium etc. The nucleus activity of the body. It also maintains equilibrium or
contains chromatin material visible as entangled mass posture of the body as during walking, jumping, lifting,
of thread like structures. Lysosomes are a kind of waste catching, bending etc.
disposal system of cell. 15. (2) Acids and substances of acidic nature have property
2. (3) Collenchyma cells are living, elongated and irregularly to change colour of blue litmus to red. They also turn
thickened at the corners. Skin epithelium is called strati- pH paper orangish red.
fied squamous epithelium because their cells are ar- 16. (4) Alkaline hydrolysis of esters is called saponification.
17. (3)
ranged in many layers to prevent wear and tear.
3. (1) A and C 18. (2) Cathode rays consist of negatively charged particle.
In case of cathode rays e/m values are same for the
Cytochrome oxidase is found on mitochondria, which
particles of all gases.
is the site for cellular respiration and Kreb's cycle.
19. (4) Dissolution of NH4Cl in water is endothermic process
Hence, the disruption of mitochondria results in the
blockage of Kreb's cycle and cellular respiration NH 4 Cl(s) + H 2 O(l) + Heat ¾¾
®
4. (2) The loss of water from the aerial parts of the plants
NH +4 (aq) + Cl- (aq)
triggers root pressure which results in pulling water
from the soil. 20. (1) The student designated as (A) made the correct
observation.
5. (1)
• In beaker (I), the solution remained colourless or
6. (2) Dialysis is the process by which ions are separated
unchanged in colour since no chemical reaction was
based on their ability to pass through a thin membrane
possible, as aluminium is placed above zinc in the
7. (1) 8. (2) 9. (4) 10. (1) 11. (4) activity series.
12. (4) Merogony is an asexual replication process used by • In beaker (II), no chemical reaction was possible and
some protozoan parasites that increases the number of the solution remained colourless.
infective cells by multiple fission. It is the process by • In beaker (III), the initial colour of ferrous sulphate
which the Plasmodium increases the infective cells. solution was light green. It became colourless after the
Buds produced in the notches along the margin of reaction.
Bryophyllum fall on the soil and develop into new
plants. Hibiscus has bisexual flowers.
MT-40 Target NTSE
Zn (s) + FeSO4(aq) ® ZnSO4(aq) + Fe(s) 4 4 10
\ d= 0 + (2 × 3 –1) = ´ 5 = m
Light green Colourless 3´ 2 6 3
• In beaker (IV), the initial colour of copper sulphate 29. (4) 30. (1)
solution was blue. It became colourless after the
reaction. G m1m 2
31. (1) F=
Zn(s) + CuSO4 (aq) ® ZnSO4(aq) + Cu(s) r2
Blue Colourless
G m1 m 2 G m1 m2 6.6 ´ 10-11 ´ 1 ´ 1
21. (4) Isobars are atoms of different elements having same \ r2 = \r = =
mass number (No. of protons + No. of neutrons) but F F 9.8 ´ 10 -9
different atomic number (No. of protons).
\ r = 8 cm
22. (2) 1 mol of H2 = 6.02 × 1023 molecules of H2 32. (2) Work done = area under F(N)
0.5 mol of H2 = 3.01 × 1023 molecules of H2
A B
F-x graph = 3
2 g of H2 = 6.02 × 1023 molecules of H2 area of trapezium 2
4 g of H2 = 12.04 × 1023 molecules of H2 1 1 C
OABC = ( 3 + 6)( 3) x(m)
18 g of H2O = 6.023 × 1023 molecules of H2O 2 0
1 2 3 4 5 6 7
= 13.5 J
44 g of CO2 = 6.023 × 1023 molecules of CO2 33. (3)
34. (1) Sound waves are longitudinal waves and need material
2.2 ´ 6.023 ´ 1023
2.2 g of CO2 = molecules of CO2 medium to travel.
44
35. (3)
= 3.01 × 1022 molecules of CO2 36. (2) When two plane mirrors are inclined to each other at
æ 360° ö
Thus required order will be an angle q then no. of images formed, n = ç -1÷ ;
è q ø
IV < I < III < II 360°
if is an even integer..
q
23. (4) Flux is a substance which may be either acidic oxide 360°
Number of image = -1 = 4 -1 = 3
(SiO2) or basic oxide (CaO, MgO). It combines with 90°
certain impurities (gangue particles) in ores to form a 37. (2)
substance called slag. This can be easily removed.
L 2L L rL
24. (4) 25. (4) 38. (2) R1 = r ; R2 = r ´ 2 ; R3 = r =
A A 2.2 A 4 A
26. (2) Magnesium burns in carbon dioxide because the Þ R3 < R1 < R2
oxygen in the carbon dioxide is able to bond with 39. (4) Fleming’s right hand rule
magnesium and produce an oxide

2Mg(s) + CO 2 (g) ¾¾
® 2MgO(s) + C(s)

27. (1) All hydrocarbons does not show isomerism. Members


of a homologous series posses similar chemical
properties only. Sodium salts of long chain aliphatic or
aromatic sulfonic acids are used as detergents.
28. (3) Distance travelled in the nth second is given by 40. (3) The sun is the ultimate source of all forms of energy
a available on the earth.
t n = u + (2n - 1) 41. (1) Since, the product of four consecutive numbers added
2
by 1 is always a perfect square.
4
put u = 0, a = ms -2 , n = 3 Then, among 4, 5, 6, 7 only 4 is perfect square number.
3 Hence, required value of k is 4.
SOLUTIONS MT-41

42. (3) Since, a and b are zeroes of polynomial 44. (2) For the first 12 hours of the day, the clock will strike
f (x) = 2x2 – 5x + 7.
12
1 + 2 + 3 + ××× + 12 = (1 + 12) = 78 times
æ 5ö 5 7 2
So, a + b = - çè - ÷ø = and ab =
2 2 2 For the next 12 hours, there will be another 78 times.
Let S and P denote respectively the sum and product So in one day, the clock will strike 156 times
of zeroes of the required polynomial. Then, polynomial
x y z
is p(x) = k(x2 – Sx + P) 45. (3) = b - c; = c - a; = a - b
a b c
5 25
S = (2a + 3b) + (3a + 2b) = 5(a + b) = 5 ´ = and,
2 2 x y z
Þ + + =0
a b c
P = (2a + 3b) (3a + 2b)
Þ P = 6(a2 + b2) + 13ab = 6a2 + 6b2 + 12ab + ab 3 3 3
\ æç ö÷ + æç ö÷ + æç ö÷ = 3æç öæ
x y z x y öæ z ö xyz
= 6 (a + b)2 + ab
a b c a ÷ç b ÷ç c ÷ = 3 abc
è ø è ø è ø è øè øè ø
2
æ 5ö 7 75 7
ÞP= 6´ç ÷ + = + = 41 5 + 2 + 5 - 2 + 2 ( 5) 2 - 2 2
è 2ø 2 2 2 x2 =
46. (2)
5 +1
Hence, the required polynomial is given by
p (x) = k (x2 – Sx + P) 2 5+2 2( 5 + 1)
= = =2
5 +1 5 +1
æ 25 ö
or, p( x) = k ç x 2 - x + 41÷ , where k is any non-zero
è 2 ø
47. (2) x + y = 18 + 6 5
real number.
43. (4) Let a and b be the roots of the equation x2 + px + q = 0 Þ x + y + 2 xy = 18 + 6 5
Then,
Þ x + y = 18
a + b = – p,
and xy = 45
a.b = q
which gives x = 15, y = 3
Let g and d be the roots of the equation x2 + ln + m = 0
48. (4) We know that
Then,
g + d = – l, (sec q - cos q)2 ³ 0
g.d = m
2 2
Since, ratio of roots of first equation is equal to the ratio of Þ sec q + cos q - 2sec q cos q ³ 0
roots of the equation second.
Þ sec 2 q + cos 2 q – 2 ³ 0
Then,
a g Þ sec 2 q + cos 2 q³ 2
=
b d
\ least value of sec2 q + cos 2 q = 2
a b a +b ab 49. (3) In figure, OAB is an equilateral triangle whose length
Þ = Þ =
g d g+d gd
of each side is 2a.
\ OA = AB = OB = 2a
-p q p2 q
Þ = Þ = Now, from the point B, draw BM perpendicular on
-l m l2 m
OA.
Þ p2m =l2q \ OM = MA = a
Hence, the required relation is p2m = l2q. Therefore from right triangle OMB,
OB2 = OM2 + MB2
MT-42 Target NTSE
or, (2a)2 = (a)2 + MB2 55. (2) (sin2 q + cos2 q)2 = sin4 q + cos4 q + sin2 q cos2 q
or, MB2 = 3a2 Þ 1 = sin4 q + cos4 q + 2 sin2 q cos2 q
Þ sin4 q + cos4 Þ = 1 – 2 sin2 q cos2 q
\ MB = 3a
Again
Since OM = a and MB = 3a (sin2q + cos2 q)3 = sin6 q + cos6 q + 3 sin2 q cos2 q
× (sin2 q + cos2 q)
Hence, co-ordinates of vertex B are (a, 3a) .
1 = sin6 q + cos6 q + 3 sin2 q . cos2 q
50. (1) P (getting a head) = 1 – P (getting a tail) sin6 q + cos6 q = 1 – 3 sin2 q . cos2 q
Then, 2 (sin6 q + cos6 q) – 3 (sin4 q + cos4 q) = 2 – 3 = – 1
40 - 24 16 2
= = = l
40 40 5 56. (4) Distance saved =
2
51. (2) In the figure, ÐABP = 42° l
Þ (l + b) - l 2 + b2 =
2
AB is a diameter of the circle. l
We have, ÐABP = 42° Þ + b = l 2 + b2
2
and ÐAPB = 90°
l2
[Q AOB is diameter and angle in semi-circle is 90°] Þ + b2 + lb = l2 + b2
In DAPB, ÐAPB + ÐABP + ÐPAB = 180° 4
ÐPAB = 180° – 90° – 42° Þ l2 + 4lb = 4l2 or 3l2 = 4lb
ÐPAB = 48° Þ 3l = 4b
Þ ÐPAB = ÐPQB [Chord PB subtends ÐPAB and b 3
ÐPQB in the same segment] Þ = .
l 4
But, ÐPAB = 48° 57. (3) HCF × LCM = Product of two numbers
\ ÐPQB = 48° 15 × LCM = 105 × 120
52. (2) Shaded area = Area of big semicircle of radius 8 unit
105 ´120
– Area of 2 semi-circles of radius 4 unit – Area of LCM =
triangle with base 8 unit & height 4 unit 15
LCM = 105 × 8 = 840.
p82 æ 2p42 1 ö 58. (4) sin4A – cos4 A = (sin2A – cos2A) (sin 2A + cos2A)
= -ç + ´ 8´ 4 ÷ = (sin2A – cos2A) which can never be expressed as
2 è 2ç 2 ÷
ø 1 + sin2A
= 32p – 16p – 16 = 16 (p – 1). th th
æ Nö æ 55 ö
1 1+ 2 p 1+ 4 p 59. (3) First we find ç ÷ value i.e. ç ÷ = 27.5th
è 2ø è 2ø
53. (3) Sequence, , , ,.............
p p p which lies in 20 – 30.
1 1+ 2 p 1 \ 20 – 30 class is median class.
Here a = ,d= -
p p p Here l = 20
1+ 2 p -1 Class Frequency c.f.
Þd= Þ d=2
p 0 – 10 5 5
1 1 2n - 2 10 – 20 10 15
tn = + (n - 1)2 = +
p p 1
20 – 30 20 35
1 + 2 np - 2 p
Þ tn =
p 30 – 40 7 42
54. (4) Since, PQ = 4 cm = 2 × OQ = 2 × radius, PQ is the 40 – 50 8 50
diametre of circle, join RQ.
50 – 60 5 55
ÐPRQ = 90° (Angle in semi-circle)
N
\ ÐORQ = 90° – 35° = 55° = 27.5, c.f. = 15, f = 20, h = 10
2
But OR = OQ. 27.5 – 15
Þ ÐORQ = ÐOQR = 55° \ Median = 20 + 20 × 10
Median = 26.25
Q DOQR is isoceles D
\ y = 180° – (55° + 55°) = 70°.
SOLUTIONS MT-43

60. (3) B 72. (2) The American intervened and bailed the Germany out
of crisis by introducing the Dawes Plan which
reworked the terms of reparation to ease financial
burden on Germans.
96 m 73. (2) The black soil is sticky and possess high moisture
retention capacity. Laterite soils are formed through
y° x° the process of laterisation and contain iron oxide.
D q C p A 74. (2)
In D ACB, 75. (3) Mineral resources are not renewable.
p 76. (3) 77. (2) 78. (2) 79. (3)
= cot x ° = 4 80. (2) Multi-purpose river valley projects are built not just
96
for irrigation but for electricity generation, water
Þ p = 384 m
supply for domestic area industrial uses, floods control,
In D ADC,
recreation, inland navigation and fsih breeding.
p+q 81. (1)
= cot y ° = 7
96 82. (3) Placer deposits are the minerals which occur as alluvial
Þ p + q = 96 × 7 m = 672 m deposit in sand of the valley floor.
\ CD = q = (672 – 384) m = 288 m 83. (2) It is most widely used variety of coal. It contains 60 to
61. (3) 80 % cabon. The Bituminous coal buried deep and
62. (1) The first Test Cricket was played between England and subjected to increased temperature. So it has less
Australia. moisture and high heating capacity.
63. (4) Marianne was a female allegory which symbolised the 84. (2) India is the largest producer of raw jute and jute goods
people's nation in France. stands at second place as an exporter after
64. (2) Ho chi Minch Trail formed the backbone of the Bangladesh. Jute industry faced stiff competition in
Vietnamese in their struggle for freedom, unification the international market. The government policy of
and during US attack. mandatory use of jute packaging. To stimulate the
65. (1) Mahatma Gandhi declared Vinoba Bhave as his spiritual demand the products needs to be diversified. In 2005
National Jute Policy was formulated.
heir and after Gandhiji’s martyrdom, he undertook
85. (2) 86. (1) 87. (3) 88. (3)
padyatra to spread Gandhiji’s message covered almost 89. (3) The Bahujan Samaj party is a centrist national political
the entire country and insisted on cooperative farming. party in India with the socialist leanings. It was formed
66. (2) Rinderpest was an infectious viral disease of cattle that to represent Bahujans or People of majority.
means cattle plague. 90. (2) Elections do not enable people to evaluate the
67. (3) 68. (2) performance of judiciary. People can indicate which
69. (2) Gandhiji in 1919, decided to launch a nationwide policies they prefer. Judiciary is independent.
Satyagraha against the propsed Rowlatt Act (1919). 91. (3) The President of India is a part of the Parliament,
This Act was huriedly passed through the Impeial although he/she is not a member of either house.
Legislative Council dspite the United Opposition of 92. (3) Supreme Court controls the judicial administration in
Indian members. Mahatma Gandhi wanted non-violent the country. Its decisions are binding on all other
civil disobedience against such unjust laws courts of the country. It is the higher court of appeal.
70. (1) The new Tory government in Britain constituted a 93. (2) All the porgrammes written in column A are poverty
statutory commission under Sir John Simon. When alleviation programmes.
the Simon Commission arrived in India in 1928, it was 94. (2)
greeted with the slogan Go back Simon’. The prblem 95. (1) Most loans from informal lenders carry a very heavy
was that the commission did not have a single Indian interest rate and do little to increase the income of the
member. They were all British. borrowers. Hence, it is necessary that banks and
71. (3) The earliest kind of print technology was developed cooperatives increase their lending particularly in rural
in China, Japan and Korea. From AD 594 onwards, areas
books in China were printed by rubbing paper. 96. (4) 97. (3) 98. (4) 99. (2) 100. (2)
MT-44 Target NTSE

MAT (Mental Ability Test) MOCK TEST 5


ANS WER KEY
1 (3) 11 (2) 21 (4) 31 (2) 41 (2) 51 (4) 61 (4) 71 (2) 81 (2) 91 (4)
2 (4) 12 (2) 22 (4) 32 (1) 42 (3) 52 (3) 62 (1) 72 (4) 82 (2) 92 (4)
3 (3) 13 (2) 23 (4) 33 (2) 43 (3) 53 (3) 63 (2) 73 (4) 83 (1) 93 (3)
4 (2) 14 (3) 24 (1) 34 (3) 44 (2) 54 (4) 64 (3) 74 (3) 84 (3) 94 (1)
5 (4) 15 (1) 25 (1) 35 (2) 45 (2) 55 (3) 65 (1) 75 (3) 85 (2) 95 (1)
6 (2) 16 (3) 26 (2) 36 (3) 46 (4) 56 (1) 66 (4) 76 (3) 86 (2) 96 (2)
7 (1) 17 (1) 27 (1) 37 (4) 47 (3) 57 (4) 67 (2) 77 (2) 87 (3) 97 (4)
8 (2) 18 (1) 28 (3) 38 (4) 48 (3) 58 (1) 68 (2) 78 (2) 88 (2) 98 (3)
9 (3) 19 (2) 29 (4) 39 (3) 49 (1) 59 (1) 69 (3) 79 (3) 89 (4) 99 (1)
10 (4) 20 (2) 30 (1) 40 (3) 50 (1) 60 (3) 70 (4) 80 (2) 90 (3) 100 (1)

Hints and Explanations


1. (3) (5 × 4) + (3 × 1) = 23 Similarly,
(7 × 6) + (3 × 4) = 54 S T R A Y D E U V T
(11 × 2) + (? × 9) = 40 +5
or , ? × 9 = 40 – 22 = 18 +4
18 +3
?= =2
9 +2
2. (4) The second and forth letters in the series, L and A, are +1
static. The first and third letters consist of an 14. (3)
alphabetical order beginning with the letter E. 2 3 7 22 155 3411
5. (3) No information is given about inviting.
6. (2) Here the respective difference between the terms is as
follows: 2×1+1 2×3+1 3×7+1 7×22+1 22×155+1
2 , 4 , 6 , 8 , ………. 15. (1) T A R G E T
Therefore, 12 + 8 = 20 ¯ ¯ ¯ ¯ ¯ ¯
Which is the required term. 20 1 18 7 5 20
7. (1) Conclusion I follows as it is directly implied by the Similary,
given statement. W I L L I U M
Conclusion II does not follow because it is too severe. ¯ ¯ ¯ ¯ ¯ ¯ ¯
9. (3) The number should be 123. (124 is wrong) 23 9 12 12 9 21 13
× 1 + 3, × 2 + 6, × 3 + 9…… 123 ´ 4 + 12 = 504 16. (3) From the question we get,
Rakesh > Sanjay > Suresh
10. (4) Clearly, the company intends to reduce the gap between Binit > Rakesh > Harish
income and expenditure by increasing the price of its So, Binit is the tallest among them.
product i.e. by keeping the expenditure unaltered and
17. (1) No. of Persons between Rahul and Nitesh
increasing the income only. So, II is implicit while I is
= (36 + 29) – 62 – 2 = 65 – 62 – 2 = 1
not. However, the rival companies may or may not follow
18. (1) (1 × 2 × 3 × 5) + (1 + 2 + 3 + 5) = 41
the same pursuit. So, III is not implicit.
(3 × 4 × 2 × 6) + (3 + 4 + 2 + 6) = 159
11. (2) Using correct symbols we have:
(9 × 8 × 3 × 4) + (9 + 8 + 3 + 4) = 888
(10 × 4) + (4 × 4) – 6 = 40 + 16 – 6 = 50 Solutions (19-23):
13. (2) As,
Month Person
B A N G E D J J K Q C C January C
+6 February A
+5 March G
+4 April E
+3 June D
+2 August F
+1 October B
SOLUTIONS MT-45

Solutions (24-25): 38. (4) The fig. (X) is similar to Form II. So, when a cube is
formed by folding the sheet shown in fig. (X), then the
W ord Code two half-shaded faces lie opposite to each other and
Card ja one of the three blank faces appears opposite to the
face bearing a dot. Clearly, each one of the four cubes
Time sa shown in figures (1), (2), (3) and (4) can be formed by
W in/team la/ta folding the sheet shown in fig. (X).
Figh t da 42. (3) The correct order is:
Population Unemployment Poverty Disease Death
Game/play p a/ra
2 4 1 5 3
In fa 43. (3) (4 × 6) ÷ 3 = 8
(6 × 10) ÷ 5 = 12
25. (1) (9 + 11 ) × 3 = 60
(4 × 8) ÷ 2 = 16
(13 + 17) × 4 = 120
44. (2) Father’s wife ® mother. Hence, the daughter of the
(40 + 41) × 5 = 405
mother means sister and sister’s younger brother means
26. (2) According to question,
brother. Therefore, the boy is the brother of Deepak.
Required number = 43 + 57 = 100
45. (2) P × Q ® P is the wife of Q
27. (1) 22 triangles are formed
Q % R ® Q is the father of R
28. (3) In each step, element at the upper-right position gets
R – T ® R is the brother of T
enlarged, inverts vertically and reaches the lower-left
T + S ® T is the sister of S.
corner; the existing element at the lower-left position,
Therefore, T is the daughter of P.
is lost and a new small element appears at the upper-
Solutions (46-49):
right position.
29. (4) Jagrati > Uma > Atulika > Amit > Aman 46. (4) Bench I I A E D
30. (1) According to question,
Bench II II
After 15th May ® 16th May, 17th May ¬ Before F B
18th May .......... (i)
Bench III III G C
But According to Deepak
After 16th May ® 17th ....... 20th May ¬ Before
There are three students in Bench I.
21st May .......... (ii)
From equation (i) and (ii), 50. (1) According to question,
Hence, Examination held on 17th May.
(N)
31. (2) P Father University
O

Crossing
(W) Hospital House (E)
le

Brother
nc
U

Theatre
S (S)
R
Mother Therefore university is in North.
33. (2) 16 m 51. (4) None of the three statements is supported by the known
facts.
52. (3) Statements I and II are not supported by the facts.
5m Statement III is true because if all storybooks have
North 16 m pictures and only some have words, then some
storybooks have both words and pictures.
8m 53. (3) If all drink mixes are beverages and some beverages are
red, then some drink mixes are red (statement I). Since
all beverages are drinkable and all drink mixes are
34. (3) The third figure in each row comprises of the parts beverages, then all red drink mixes must be drinkable
common to the first two figures. (statement III). Statement II can be ruled out.
35. (2) The third figure in each row comprises of parts which 54. (4) According to question,
are not common to the first two figures. As,
36. (3) The third figure in each row comprises of parts which C = 3, K = 11 and R = 18
are not common to the first two figures.
\ Sum = 3 + 11 + 18 = 32
37. (4) Three and two arcs are inverted alternately. The central D = 4, M = 13 and T = 20
element rotates 90oACW and 180o alternately.
MT-46 Target NTSE
\ Sum = 4 + 13 + 20 = 37 64. (3) Amount spent on labour and supervision
F = 6, N = 14 and B = 2
\ Sum = 6 + 14 + 2 = 22 ì ( 90 + 54) üï
=`í ´ 15, 00, 00 ý
Similarly, H = 8, E = 5 and P = 16 î 360 ïþ
\ Sum = 8 + 5 + 16 = 29
55. (3) The first two letters, PQ, are static. The third letter is in æ 144 ö
= ` çè ´ 15, 00, 000÷
alphabetical order, beginning with R. The number series 360 ø
is in descending order beginning with 5. = ` 6, 00, 000
56. (1) Statement II reveals that 17th was a Saturday and 65. (1) Number of girls participating in 2004
therefore, 14th was Wednesday. So, only statement II = 100 + 140 + 120 + 120 = 480
is needed. Number of girls participants in 2003
57. (4) ‘59’ means ‘injurious habit’ and ‘157’ means ‘stop bad = 90 + 120 + 70 + 90 = 370
habit’ (from I). Thus, the common code number ‘5’ So, Required number = 480 – 370 = 110
stands for common word ‘habit’. So, ‘9’ represents 66. (4) No lady facing east means a man faces east. Persons
‘injurious’. Hence, I is sufficient. Also, ‘59’ means opposite are not of same sex. So, a woman will be facing
‘injurious habit’ and ‘839’ means ‘smoking is injurious’. west. Again, a man faces south. So, opposite to him
Thus, the common code number ‘9’ stands for common will be a woman facing north.
word ‘injurious’. So, ‘5’ represents ‘habit’. Thus, II is
also sufficient. Man
58. (1) Statement I is merely an interpretation of the information
contained in the question.
However, Vipin’s salary can be determined from Man Woman
statement II as follows:
Let Vipin’s and Javed’s salaries be 4x and 3x
respectively.
Then, 3x = 4500 or x = 1500. Woman
Vipin’s salary = 4x = `6000. 67. (2) Clearly, the route of the postman is as shown. So, at the
Thus, II alone is sufficient. final point the distance of postman from the post office
59. (1) From statement II, we conclude that Sonali left her home = EA = BD
45 minutes before 10.15 a.m. i.e., 9.30 a.m. So, BC + CD = (50 + 40) = 90m.
Thus, only II is needed.
60. (3) Even both the statements together are not sufficient to E A
answer the question.

Timber
36° 100 m 100 m
Labour
Supervisor 90° C
54°
D B
40 m 50 m
Steel Shanti
54° Cement Villa
72°
Bricks 68. (2) 34 persons like Hindi and Tamil language but not English
54° language.
69. (3) Number of persons was liked both Hindi and English
61. (4) Amount spent on labour language but not Tamil language is 14.
æ 90 ö 70. (4) 244 persons like Tamil language.
= `ç ´ 15, 00, 000÷
è 360 ø 71. (2) Number of persons are there who likes only English
= ` 3,75, 000 language = 50 + 40 = 90
62. (1) Amount spent on bricks, steel and cement 72. (4) 30 persons like all the three language.
ì (54 + 54 + 72) ü æ 180 ö 77. (2) J O C K E Y S T R A M
=í ´ 100ý % = ç ´ 100÷ % = 50%
î 360 þ è 360 ø @ ! ^ * ? > = $ / < +
63. (2) Required % of amount spent on timber with respect to J A M M E R
the amount spent on cement @ < + + ? /
æ 36 ö 78. (2) Column 1: 36 + 57 = 93
= çè ´ 100÷ø % = 50% Column 2: 42 + 157 = 199
72 Column 3: 17 + 944 = 961
SOLUTIONS MT-47

79. (3) According to question, 92. (4) According to question,


144 ÷ 16 × 4 – 2 + 8 = 42
80. (2) abbccd/abbccd North-East
81. (2) According to question,
+1 +1 +1

45°
East
A B C D 180°

Z Y X W
82. (2) According to question,
84 42 44 22 24 12 14 South-East
93. (3) The common letters between singer and actor is c and
¸2 +2 ¸2 +2 ¸2 +2 e , but in options only c is given
83. (1) I J 94. (1) According to question,

A B
E F
Vishu Ram Pooja Vishakha Rani
M N 95. (1) Order of sitting: DECAB
O
H G 96. (2) The new alphabet-series is:
ZYXWVUTSRQPONMLKJIHGF
D C
EDC BA
L K Required letter = 9 + 5 =14th letter = N
The horizontal lines are IJ, AB, EF, MN, HG, DC and LK Solutions (97-100):
i.e. 7 in number. The villages are located as shown in the below figure:
The vertical lines are AD, EH, IL, FG, BC and JK i.e. 6 in
number.
F
Thus, there are 7 + 6 = 13 straight lines in the figure.

Mother 2 km

Mahendra 1 km 1 km
86. (2) G
C A

Grandson Girl
2 km
87. (3) According to question,
+
N Q B
E 1 km D 1 km

+ +
M 97. (4) Clearly, A is in the middle of G and C.
98. (3) Clearly, F and B are the farthest from one another.
99. (1) Required distance = FE = FA + AE = FA + GD
= (2 + 2) km = 4 km
P
100. (1) The word formed by first even number place letters
89. (4) According to question, then odd number place letters.
Jaggu > Bumrah > Mahi > Sehwag > Yuvi So, ARKONAM ® ROAAKNM
91. (4) Cost of laundry = 150 + (9 × 50)
= 150 + 450 = 600
MT-48 Target NTSE

SAT (Scholastic Aptitude Test) MOCK TEST 5


ANS WER KEY
1 (3) 11 (4) 21 (3) 31 (1) 41. (2) 51. (4) 61. (1) 71. (1) 81. (2) 91. (2)
2 (4) 12 (1) 22 (4) 32 (3) 42. (3) 52. (4) 62. (1) 72. (1) 82. (3) 92. (4)
3 (2) 13 (2) 23 (2) 33 (4) 43. (3) 53. (1) 63. (3) 73. (2) 83. (3) 93. (3)
4 (3) 14 (3) 24 (3) 34 (2) 44. (3) 54. (4) 64. (3) 74. (2) 84. (4) 94. (4)
5 (2) 15 (3) 25 (2) 35 (1) 45. (1) 55. (1) 65. (3) 75. (4) 85. (2) 95. (4)
6 (1) 16 (2) 26 (4) 36 (3) 46. (3) 56. (2) 66. (1) 76. (3) 86. (2) 96. (1)
7 (2) 17 (3) 27 (4) 37 (3) 47. (4) 57. (2) 67. (3) 77. (1) 87. (2) 97. (2)
8 (1) 18 (3) 28 (4) 38 (4) 48. (1) 58. (3) 68. (1) 78. (2) 88. (2) 98. (1)
9 (1) 19 (4) 29 (3) 39 (3) 49. (1) 59. (1) 69. (4) 79. (3) 89. (4) 99. (2)
10 (1) 20 (1) 30 (1) 40 (3) 50. (4) 60. (2) 70. (1) 80. (3) 90. (4) 100. (2)

Hints and Explanations

1. (3) Gibberellins 8. (1) Adrenalin


Gibberellins are responsible for stem elongation in plants Adrenalin is the hormone which triggers flight of fight
2. (4) Energy flow is the key function in an ecosystem and it response
is unidirectional. Ten percent law states that during
9. (1) The deficiency of iodine can result in the dysregulation
transfer of food energy, only 10% is stored at higher
of thyroid hormone and reult in goitre if left unchecked
trophic level. The remaining 90% is lost during feeding,
digestion and respiration. 10. (1) Sanjay is right
3. (2) The curvature of stem tip towards light is mediated by
4. (3) Fungi are achlorophyllous, heterotrophic spore auxin
producing, chitin in their cell wall and glycogen as 11. (4) Theory of natural selection states that organic
reserve food, e.g. Aspergillus, Agaricus. Algae are evolution occurs through natural selection and
thallophytes which are capable of manufacturing accumulation of inheritable variations which provide
their own food through photosynthesis, e. g. structural and functional superiority to some individuals
Ulothrix, Chara. Bryophyta is a division of non-
over others in their survival and reproduction.
vascular plants having an embryo stage in their life
12. (1) Law of dominance states that only one character
cycle, e.g. Riccia, Marchantia. Pteridophyta is a
division of seedless vascular plants, e.g. Marsilea, expresses itself in F1 generation. Here, ‘T’ is a
Equisetum. Gymnospermae is the subdivision of dominant trait and ‘t’ is a recessive trait. Hence ‘Tt’
seed plants in which the seeds are naked and the will show tha caracters of tall plants. Therefore, the
sporophylls are aggregated to form cones, e.g. height of ‘Tt’ will be 15 cm.
Cycas, Pinus. 13. (2)
5. (2) If the medium surrounding the cell has a high water 14. (3) Sensory neurons conduct impulses from receptor (e.g.,
concentration than the cell, meaning that the outside skin, eye, ear) to central nervous system. Motor
solution is very dilute, the cell will gain water by neurons conduct motor impulse from central nervous
osmosis. Such a solution is known as a hypotonic
system to the effector organ (e.g., muscle, gland or
solution. The cell is likely to swell up.
organ).
6. (1) The stem tip curves and grows towards light
15 (3) Baking Soda (NaHCO3) is an basic salt and washing
The stem tip curves and grows towards light as
soda (Na2CO3) is also basic salt. The aqueous solution
stimulated by auxin
of common salt is neutral in nature.
7. (2) High blood pressure is non-in fectious
NaCl(s) + H2O(l) ® Na+(aq) + Cl–(aq)
(non-communicable) disease. It is often caused by
excessive weight and lack of exercise.
SOLUTIONS MT-49
10
Amla contains ascorbic acid and soap contains base 28. (4) Distance in 10 min = 7.5 ´ = 1.25 km
60
(caustic soda or sodium hydroxide) spinach contains (2.5 + 1.25) km 45
So, Average speed = = km / hr.
oxalic acid and thus its aqueous solution change colour (40 / 60) hr 8
15 5
of blue litmus to red. 29. (3) The acceleration of both the blocks = =
3x x
16. (2) Darkness of purple colour of KMnO4 solution fades 5
with increase in dilution. \ Force on B = ´ 2x = 10 N
x
17. (3) In extraction of iron flux is CaO which reacts with silicate 30. (1) At the surface of earth, the value of g = 9.8m/sec2. If we
to form slag (CaSiO3). go towards the centre of earth or we go above the
surface of earth, then in both the cases the value of g
CaO + SiO2 ® CaSiO3
decreases.
(Flux) Slag
Hence W1 = mgmine, W2 = mgsea level, W3 = mgmoun
Silver turns black due to formation of Ag2S
So, W1< W2 > W3
2Ag(s) + H2S(g) ® Ag2S(s) + H2(g) (g at the sea level = g at the suface of earth)
18. (3) Natural gas > Charcoal > Bituminous coal > Wood. 31. (1) According to Law of floatation a body will float if the
19. (4) Chlorine will displace iodine from solution containing weight of the body is less than/equal to the weight of
potassium iodide since it is more reactive than iodine. the liquid displaced.
The liberated iodine will impart violet colour to the 32. (3) The positive work signifies that the external force or
solution. force applied favours the motion of the body.
2KI (aq) + Cl2 (g) ® 2 KCl (aq) + I2(g) 33. (4) Sound wave follows the laws of reflection. It needs
(violet) material medium for the propagation. So, it cannot travel
in vacuum.
20. (1) The colour of the pH paper must have remained green.
Sodium chloride (NaCl) is a salt of strong base (NaOH) 34. (2) A sudden disturbance which last for a short time interval
and strong acid (HCl). It dissolves in water to give a is called a pulse.
neutral solution which is just like distilled water. 35. (1)
Therefore, the colour of pH paper remained green. 36. (3) From figure, x = 30º (Q Ði = Ðr )
21. (3) Isotopes have different number of neutrons. Isobars Y = x = 30º (Alternate angles)
30º
differ in no. of nucleons i.e., sum of protons and neutrons. Z = 90º – y x
Potassium is a metal whereas chlorine is a non-metal. = 90º – 30º = 60º
22. (4) X and Y are elements as they can not be broken down N = Z (Q Ði = Ðr ) Z N
90º
into simpler substances. They combine in fixed or N = 60º Y
rl l
proportion to give compound P. 37. (3) R = or R µ .
2
( p D / 4) D2
23. (2) The statement is not correct as the metal oxide reacts
with water to form metal hydroxide i.e., base. Rx lx D y2 ly / 2 D y2 2
= ´ = ´ =
24. (3) Green crystals of ferrous sulphate are converted into a R y Dx 2 l y (D y / 2)2 l y 1
dirty – yellow anhydrous solid upon heating 39. (3) Geo-thermal energy is the energy stored in the hot spots
2FeSO4 ® Fe2O3 + SO2 + SO3 the earth’s crust.
Chemical formula of aluminium phosphate is AlPO4. 40. (3)
The gas released in respiration is CO2.
25. (2) This is due to the formation of sulphur dioxide, SO2.
41. (2) å a1= 0, å a1a 2 = 2 - 3,

26. (4) Zinc blende (ZnS) is the sulphide ore.


å a1a 2a 3 = 0, a1a 2 a3 a4 = 2 + 3,
Option (1) is not correct as haematite is chemically
Fe2O3. (1 - a1 )(1 - a 2 )(1 - a3 )(1 - a 4 )
Option (2) is not correct as bauxite is chemically = 1 - (Sa1 ) + (Sa1a2 ) - (Sa1a2 a3 ) + a1a 2 a3a 4
Al2O3.2H2O.
Option (3) is not correct as Rock salt is chemically NaCl. =1- 0 + 2 - 3 - 0 + 2 + 3 = 5
27. (4) Structural formula of 2-butyne is H3C – C º C – CH3 it 42. (3) Consider the given relation
contains one triple and two single bonds between
4x 4 + 12x 3 + 25x 2 + 24x + 16 = ax 2 + bx + c
carbon atoms.
MT-50 Target NTSE
Square on both sides
2x
log 2
4
4x 4 + 12x 3 + 25x 2 + 24x + 16 Hence, required value of 2 is 2.

2 4 2 2 2 3
= a x + b x + c + 2abx + 2bcx + 2acx
2 1 1 1
45. (1) + = can be rewritten as
x+a x+b c
Compare the coefficients of x 4 , x3 , x 2 , x and constant
x +a + x+b 1
terms. =
a2 = 4 Þ a = 2
(x + a)(x + b) c
c2 = 16 Þ c = 4 Þ [2x + (a + b)] c = (x + a) (x + b)
Þ x2 + x (a + b) + ab – 2cx – (a + b)c = 0
2ab = 12 Þ ab = 6
Þ x2 + x (a + b – 2c) + (ab – ac – bc) = 0
b2 + 2ac = 25 Þ b2 + 2 × 2 × 4 Þ b2 = 25 – 16 But since roots are equal in magnitude but opposite in
Þb=3 sign,
So, We have, a + b = 0
a = 2, b = 3, c = 4 - (a + b - 2c)
Þ =0
Then, 1
a + b – 2c = 0
2b = 2 × 3 = 6 and a + c = 2 + 4 = 6 Þ 2c = a + b
Therefore, the required relation is 2b = a + c. Also,
43. (3) a b = ab – ac – bc = ab – c(a + b)
2x - 3 + 3x - 5 = 5x - 6
Squaring on both sides, we have a+b
But, c =
2
2x – 3 + 3x – 5 + 2 2x - 3. 3x - 5 = 5x - 6
- a 2 + b2
(
2 2x - 3. 3x - 5 = 2
\ a b = ab -
(a + b)
2
=
)
2x - 3. 3x - 5 = 1 2 2
Again squaring on both sides, we have 46. (3) Since PQ2 = QR2 + PR2
(2x – 3) (3x – 5) = 1 Also PQ = PS + SQ
6x2 – 19x + 15 – 1 = 0 RS = 25 cm (Given)
6x2 – 19x + –14 = 0 Þ PS = SQ = RS = 25 cm
(x – 2) (6x – 7) = 0 [Q midpoint of hypotenuse of right
angled triangle is equidistant from 3 vertices]
7
x = 2 or \ PQ = 50 cm
6 Now,
Both satisfy the equation. 502 = QR2 + 482
44. (3) x = log3 log2 log2 256 Þ QR2 = 2500 – 2304
= log3 log2 (log2 28) Þ QR = 196 =14
= log3 log28
47. (4) A (p, 2 – 2p), B(1 – p, 2p), C(–4 –p, 6–2p)
= log3 (log2 23) Area (DABC) = 70
= log33 Then,
=1 1
Now, |p (2p – 6 + 2p) + (1 – p) (6 – 2p – 2 + 2p)
2
2x 21 + (– 4 – p) (2 – 2p – 2p)| = 70
log42 log42
2 =2 1
Þ |p (4p – 6) + (1 – p) (4) + (– 4 – p)
log 4 4 2
=2
(2 – 4p)| = 70
= 21
=2
SOLUTIONS MT-51

1 12 ± (12 ) – 4´1´ 26
2
Þ | 4p2 – 6p + 4 – 4p –8 + 16p – 2p + 4p2| = 70 x=
2 2 ´1
1
Þ | 8p2 + 4p – 4| = 70 12± 144 –104
2 =
2
Þ | 8p2 + 4p – 4| = 140
So we have, 12± 40 12 ± 2 10
= =
8p2 + 4p – 4 = 140 or 8p2 + 4p – 4 = –140 2 2
Þ 8p2 + 4p – 144 = 0 or 8p2 + 4p + 136 = 0 x = 6 ± 10
Þ 2p2 + p – 36 = 0 or 2p2 + p + 34 = 0
The first equation does not give integral values as Hence, the required roots are 6 + 10 and 6 – 10 .
square root of its discriminant is irrational number and 50. (4) Since, x + 3 is a factor of ax2 + bx + 1
the second equation has no solution as its discriminant Then, for x + 3 = 0 Þ x = – 3 is a root of the expression.
is negative. So we do not have any integral values. So, a (– 3)2 + b (– 3) + 1 = 0
48. (1) Consider the equations 9a – 3b + 1 = 0
4 (2x–1) + 9 (3y–1) = 17 9a = 3b – 1 ...... (1)
3 (2x) – 2 (3y) = 6 Also, x + 3 is a factor of expression px2 + qx – 3
The above system of equation can be written as Then, x = –3 is a root of px2 + qx – 3
2 (2x) + 3 (3y) = 17 So, 9p – 3q – 3 = 0
3 (2x) – 2 (3y) = 6 9p = 3q + 3
Let 2x = a and 3y = b 3p = q + 1 ......(2)
Then, equation becomes Now,
2a + 3b = 17 ...... (1) –[9a + 3p] – [3b –1 + q + 1]
=
3a – 2b = 6 ...... (2) 3b + q 3b + q
Now, from (1) × 2 + (2) × 3
– [3b + q]
4 a + 6 b = 34 = 3b + q = – 1
9 a – 6 b = 18
51. (4) Let the radii of the bigger and smaller circle be R and r
13 a = 52 respectively.
52
a = 13 = 4

But
2x = a Þ 2x = 4 Þ 2x = 22 Þ x = 2
A
Put the value of a in equation (2), D
R
2b = 3a – 6 = 3 × 4 – 6 = 6
P
Then, 2b = 6 Þ b = 3
B Q C
But 3y = b Þ 3y =31 Þ y = 1 \ In the figure AB = AD = R
Hence, required value is (2, 1). As ÐADC = 90°, ÐABC = 90° and ÐDCB = 90°

49. (1) x –1 + x – 3 = 21 \ BC = R and AC = 2 R and


x–2 x–4 3
AC = AP + PQ + QC = R + r + 2 r [QC = 2 r can be
x 2 – 5x +4 + x 2 – 5x + 6 = 7
x 2 – 6x +8 3 proved in the same way as we proved AC = 2 R]

2x 2 –10x +10 = 7
x 2 – 6x +8 3 \ r=
( 2 -1 R )
2 +1
6x2 – 30x + 30 = 7x2 – 42x + 56
x2 – 12x + 26 = 0 Rationalising the denominator, we get
MT-52 Target NTSE

r = (3 – 2 2 )R
63 + 56 = 4 7 ( 3+ 2 2 )
Given R = 2, we get r = 2 (3 – 2 2 )
=6–4 2 Let 3+ 2 2 = x + y ... (1)

52. (4) Sum of the n term of the series, Take square on both the sides

n Then, 3 + 2 2 = x + y + 2 xy
Sn = [2(5a - b) + (n - 1)(-2a - b)]
2
By comparing like terms
n x+y= 3 ... (2)
-280a - 210b = [–2(5a – b) + (n – 1) (–2a – b)
2
2 xy = 2 2
[first term = 5a – b, common difference = – 2a – b]
Þ 2n (5a – b) – (n2 – n) (2a + b) + 560a + 420b = 0 xy = 2
Þ 2n (5a – b) – n2 (2a + b) + n (2a + b) +
xy = 2
560a + 420b = 0
Now,
Þ (2a + b) n 2 – n [(10a – 2b) + (2a + b)] – 560a
– 420b = 0 x - y = (x + y)2 - 4xy = 9 - 4 ´ 2 = 9 - 8
2
Þ (2a + b) n – n (12a – b) – 560a – 420b = 0
x–y=1 (3)
Þ (2a + b) n2 – n [(40a + 20b) – (28a + 21b)]
From equation (2) + (3),
– 20 (28a + 21b) = 0
2
Þ (2a + b) n – n [20 (2a + b) – 7 (4a + 3b)] 2x = 4 Þ x = 2
– 20 × 7 (4a + 3b)] = 0 Put the value of x in equation (2), y = 3 - x = 3 - 2 = 1
Þ [(2a + b) n + 7 (4a +3 b)] (n – 20) = 0 Then, from equation (1)
Þ n = 20
53. (1) Since, 3 + 2 2 = x + y = 2 +1
tan q + cot q = 2
Since, 63 + 56 = 4 7( 3 + 2 2 ) = 4 7 × 2 + 1
tan q + tan1q = 2
55. (1) Total height, H = 28 m
Let x = tan q Radius of hemisphere = r = 14 m
x = + 1x = 2

1
Then, xn + x n = 2 h
H = 28 m
1
tann q+ = 2
tan nq r = 14 m

tann q + cotn q = 2
Hence, required value of tan n q + cotn q is 2. \ Height of conical tent = h = H – r = 28 – 14 = 14 m.
We can observe that radius of base of cone
54. (4) 63 + 56 = 7´9 + 7´8 = radius of the hemisphere = 14 m.
\ Area of canvas required = prl
= 7 9+ 7 8
22
= ´ 14 ´ 142 + 142 = 44 ´ 14 2 = 616 2 m 2
7
= 7( 9 + 8) Area of metal sheet required = surface area of
hemisphere
= 47 9+ 8
2 22
= 2 pr = 2 ´ ´ 14 ´ 14 = 1232 m 2
7
= 4
7 3+ 2 2
SOLUTIONS MT-53

56. (2) In D BOC All 3 digit even number


ÐOBC = ÐOCB [Q OB = OC] \ P (no. multiple of 2) = All 3 digit number

450 1
= 900 =
2
59. (1) Let the length of the shortest side be x metres.
hypotenuse = (2x + 1) metres, third side = (x + 7) metres.
(hypotenuse)2 = Sum of the square of the remaining
two sides [By Pythagorous theorem]
Þ (2x + 1) = x + (x + 7)2
2 2

Þ 4x2 + 4x + 1 = 2x2 + 14x + 49


Þ 2x2 – 10x – 48 = 0
Þ x2 – 5x – 24 = 0
Þ x2 – 8x + 3x – 24 = 0
\ ÐOBC = ÐOCB = 25° [\ ÐBOC =130]
Now in D PBC Þ x (x – 8) + 3 (x – 8) = 0
ÐPBC = 50° [ Q PBO = 25°] Þ (x – 8) (x + 3) = 0
\ ÐPBC = ÐPCB = 50° Þ x = 8, – 3
Þ ÐBPC = 80° Þ x=8 [Q x = – 3 is not possible]
Now Hence, the lengths of the sides of the grassy land are 8
ÐAPB = 100° [ Q ÐAPB, ÐBPC are adjacent’ss metres, 17 metres and 15 metres.
on same straight line] 60. (2) Since, a p = bq = c r = abc
\ ÐDPC = 100°
Then a p = abc Þ a = (abc)1/p
Since ÐBOC = 130°
ÐBOC bq = abc Þ b = (abc)1/q
Þ ÐBAP = 2 = 65° [Q angle at the centre is twice
the angle at the circumference] c r = abc Þ c = (abc)1/r
\ In D ABP,
ÐABP = 180° – (100° + 65°) Now, abc = (abc)1/p (abc)1/q (abc)1/r
= 15°
Similarly ÐDCP = 15° 1 1 1
+ +
\ ÐABP + ÐDCP = 30° p q r
abc = (abc)
57. (2) Since, mode : median = 6:5
æ pq + qr + rp ö
Then, mode =6x é x is some constant ù ç ÷
median =5x ëê ûú
abc = (abc)è pqr ø
We know that,
pq + qr + rp
mode = 3 × median – 2 × mean
pqr
Þ 6x =15x – 2 mean (abc)1 = (abc)
9x By comparing the powers on both sides,
Þ mean = 2
Now, the ratio of mean and median is, pq + qr + rp
=1
pqr
9x
mean = 2 = 9 Þ pqr = pq + qr + rp
median 5x 10 Hence, required relation is pqr = pq + qr + rp
58. (3) Total number of three digit numbers is 100 – 999 = 900 61. (1) 62. (1) 63. (3)
Out of 900 numbers 450 are even and all even number 64. (3) April theses was three demands made by Lenin on his
will be multiple of 2 return from exile to Russia in April 1917.
65. (3) 66. (1) 67. (3) 68. (1)
MT-54 Target NTSE
69. (4) In 1933, Hitler said In my state the mother is the most 85. (2) The prime minister of India as addressed in the
important citizen. But in Nazi Germany all mothers were constitution of India, is the chief advisor to the
not treated equally. The boys were taught to be President of India, head of the council of ministers and
aggressive, masculine and steal hearted girls were told
leader of the majority party in parliament.
to be good mothe and rear pue-blooded Aryan
children. 86. (2) 87. (2) 88. (2)
70. (1) The poor peasants resort to riots to protest against 89. (4) In a democracy the will of the people is supreme. The
the use of threshing machines. Threshing machines minister is elected by the people and thus empowered
had become a sign of unemployment and bad times. to exercise the will of the people on their behalf. He/
71. (1) The Epistolary novels are written in the form of series she is answerable to the people for all the
of letters, used private and personal form of letters to consequences of his/her decision. Political execulive
tell its story. Serialised, novels/ magazines were had more powers.
published in instalments. Serialisation allowed readers 90. (4) Rights are needed in a democracy as they protect
to relish the suspense discuss the characters of a novel minorities from the appression of majority. Right do
and live for weeks with stories. Vernacular are the not guarantee limitatio on an individual’s right to
spoken form of language rather than formal. Satire a equality.
form of representation through writing, drawing, 91. (2) Zimbabwe after attaining independence from White
painting etc. that provides a criticism of society in a minority rule in 1980. Since then the country has been
manner that is witty and clever. ruled by ZANU-PF. Statement I and II are correct.
72. (1) 73. (2) 74. (2) 75. (4) 76. (3)
92. (4) 93. (3)
77. (1)
94. (4) The ADS Grain Bank programme is acknowledged as a
78. (2) Public sector is owned and operated by government
successful and innovative food security intervention.
agencies. Joint sectors are jointly run by the state and
95. (4) 96. (1)
individuals Private sectors are owned and operated by
individuals or a group of individuals. Cooperative 97. (2) Disguised unemployment usually exists among family
members engaged in agricultural activity.
sector industries are owned by the produces or
suppliers of raw materials. 98. (1) “Minimum level” is described by the poverty line which
decides who is poor and who is not.
79. (3) 80. (3) 81. (2) 82. (3)
99. (2) WTO promotes free trade among the different countries of
83. (3) Agglomeration econmies are the benefits that come the world. It has provided an effective mechanism for the
when firms and people locate near another together in settlements of disputes among trading countries.
cities and industrial clusters. 100. (2)
84. (4)

You might also like